Você está na página 1de 650

OXFORD FAJAR ADVISORY BOARD

The board consists of a team of experienced


teachers who review our titles to ensure that the
contents are in line with the current syllabus
and examination requirements as set by the
Examination Syndicate, Ministry of Education
Malaysia.

Success Physics SPM e-book was reviewed by


• Adda Ong
• Rosaimi Abd Wahab
• Tam Siew Shong
• Yap Wei Yee
• Chen Kheng Bin

*****************************************

Oxford Fajar Sdn. Bhd. (008974-T)


(Formerly known as Penerbit Fajar Bakti Sdn. Bhd.)
4 Jalan Pemaju U1/15, Seksyen U1
Hicom-Glenmarie Industrial Park
40150 Shah Alam
Selangor Darul Ehsan

© Oxford Fajar Sdn. Bhd. (008974-T) 2013


First published 2013

ISBN 978 983 47076 0 6

All rights reserved.


No part of this publication may be reproduced,
stored in a retrieval system, or transmitted in any form or
by any means, electronic, mechanical, photocopying,
recording or otherwise, without the prior permission of
Oxford Fajar Sdn. Bhd. (008974-T)

Text set in 10 point ITC Giovanni by


Impression One Sdn. Bhd., Kuala Lumpur

ii
Contents

Word Power v CHAPTER 3


Forces and Pressure 166
3.1 Understanding Pressure 167
FORM 4
3.2 Understanding Pressure in Liquids 170
3.3 Understanding Gas Pressure and 176
CHAPTER 1
Atmospheric Pressure
 3.4 Applying Pascal's Principle 186
Introduction to Physics 1
3.5 Applying Archimedes' Principle 189
1.1 Understanding Physics 2 3.6 Understanding Bernoulli's Principle 196
1.2 Understanding Base Quantities and 4 SPM Exam Practice 3 201
Derived Quantities
1.3 Understanding Scalar and Vector 11
Quantities
1.4 Understanding Measurements 11
1.5 Analysing Scientific Investigations 22 CHAPTER 4
SPM Exam Practice 1 30 Heat 206
4.1 Understanding Thermal Equilibrium 207
4.2 Understanding Specific Heat Capacity 211
4.3 Understanding Specific Latent Heat 222
CHAPTER 2
4.4 Understanding the Gas Laws 233
SPM Exam Practice 4 245
Forces and Motion 37
2.1 Analysing Linear Motion 38
2.2 Analysing Motion Graphs 51
2.3 Understanding Inertia 58
2.4 Analysing Momentum 66 CHAPTER 5
2.5 Understanding the Effects of a Force 80 Light 251
2.6 Analysing Impulse and Impulsive Force 91
5.1 Understanding Reflection of Light 252
2.7 Being Aware of the Need for Safety 98
5.2 Understanding Refraction of Light 265
Features in Vehicles
5.3 Understanding Total Internal 276
2.8 Understanding Gravity 99
Reflection
2.9 Analysing Forces in Equilibrium 110
5.4 Understanding Lenses 285
2.10 Understanding Work, Energy, Power 124
SPM Exam Practice 5 307
and Efficiency
2.11 Appreciating the Importance of 141
Maximising the Efficiency of Devices
2.12 Understanding Elasticity 142
SPM Exam Practice 2 159

iii
FORM 5
3.3 Analysing Electromagnetic Induction 452

3.4 Analysing Transformers 469
CHAPTER 1
3.5 Understanding the Generation and 478
Waves 315 Transmission of Electricity
1.1 Understanding Waves 316 SPM Exam Practice 3 486
1.2 Analysing Reflection of Waves 324
1.3 Analysing Refraction of Waves 330
1.4 Analysing Diffraction of Waves 336
1.5 Analysing Interference of Waves 341
1.6 Analysing Sound Waves 352 CHAPTER 4

1.7 Analysing Electromagnetic Waves 358 Electronics 493


SPM Exam Practice 1 362 4.1 Understanding the Uses of the 494
Cathode-Ray Oscilloscope(CRO)
4.2 Understanding Semiconductor 508
Diodes
4.3 Understanding Transistors 515
CHAPTER 2
4.4 Analysing Logic Gates 523
Electricity 369 SPM Exam Practice 4 538
2.1 Analysing Electric Fields and Charge 370
Flow
2.2 Analysing the Relationship between 375
Electric Current and Potential
Difference CHAPTER 5

2.3 Analysing Series and Parallel Circuits 391 Radioactivity 548


2.4 Analysing Electromotive Force and 402 5.1 Understanding the Nucleus of an 549
Internal Resistance Atom
2.5 Analysing Electrical Energy and 409 5.2 Analysing Radioactive Decay 550
Power 5.3 Understanding the Uses of 564
SPM Exam Practice 2 417 Radioisotopes
5.4 Understanding Nuclear Energy 567
5.5 Realising the Importance of Proper 572
Management of Radioactive
Substances
CHAPTER 3
SPM Exam Practice 5 575
Electromagnetism 423
3.1 Analysing the Magnetic Effect of a 424 SPM Model Test 582
Current-carrying Conductor
Answers 600
3.2 Understanding the Force on a 437
Current-carrying Conductor in a
Magnetic Field

iv
Key Words
FORM 4 displacement – sesaran pressure – tekanan
Chapter 1 Introduction to Physics distance – jarak pressure in liquids – tekanan cecair
elastic collision – perlanggaran kenyal submerged – tenggelam
accuracy – kejituan explosion – letupan transmitted – tersebar
base quantity – kuantiti asas forces in equilibrium – daya-daya
base unit – unit asas dalam keseimbangan Chapter 4 Heat
consistency – kepersisan free fall – jatuh bebas
derived quantity – kuantiti terbitan gravitational force – daya graviti absolute temperature – suhu mutlak
derived unit – unit terbitan impulsive force – daya impuls absolute zero – sifar mutlak
error – ralat inelastic collision – perlanggaran tak boiling – pendidihan
fixed/constant variable – kenyal Boyle’s law – Hukum Boyle
pembolehubah dimalarkan inertia – inersia calibration – penentukuran
hypothesis – hipotesis kinetic energy – tenaga kinetik Charles’ law – Hukum Charles
inference – inferens mass – jisim condensation – kondensasi
investigation method – kaedah motion graph – graf gerakan freezing – pembekuan
penyiasatan potential energy – tenaga keupayaan fusion – pelakuran
manipulated variable – power – kuasa Gas law – Hukum gas
pembolehubah dimanipulasikan resolution of force – leraian daya heat – haba
measurement – pengukuran resultant force – daya paduan melting – peleburan
measuring instrument – alat speed – laju Pressure law – Hukum tekanan
pengukur specific heat capacity – muatan haba
ticker-timer – jangka masa detik
micrometer screw gauge – tolok skru tentu
velocity – halaju
mikrometer specific latent heat – haba pendam
weight – berat
natural phenomena – fenomena tentu
alam semula jadi work – kerja
specific latent heat of fusion – haba
oscillation – ayunan pendam tentu pelakuran
parallax error – ralat paralaks specific latent heat of vaporisation –
Chapter 3 Forces and Pressure
physical quantity– kuantiti fizik haba pendam tentu pengewapan
prefixes – imbuhan apparent weight – berat ketara sublimation – pemejalwapan
random error – ralat rawak Archimedes’ principle – Prinsip temperature – suhu
relative deviation – sisihan relatif Archimedes thermal equilibrium – keseimbangan
responding variable – pembolehubah atmospheric pressure – tekanan terma
bergerak balas atmosfera thermocouple – termogandingan
scientific skills – kemahiran saintifik Bernoulli’s principle – Prinsip Bernoulli vaporisation – pengewapan
sensitivity – kepekaan buoyancy – keapungan
standard form – bentuk piawai buoyant force/upthrust – daya tujah
stopwatch – jam randik ke atas Chapter 5 Light
systematic error – ralat sistematik density – ketumpatan angle of incidence – sudut tuju
vernier callipers – angkup vernier depth – kedalaman aperture – bukaan
zero error – ralat sifar enclosed – tertutup apparent depth – dalam ketara
fluid – bendalir concave mirror – cermin cekung
force multiplier – pembesar daya converge – menumpu
Chapter 2 Forces and Motion kinetic theory of matter – teori kinetik convex mirror – cermin cembung
acceleration – pecutan jirim critical angle – sudut genting
collision – perlanggaran lifting force – daya angkat diminished – diperkecilkan
conservation – keabadian matter – jirim diverge – mencapah
deceleration – nyahpecutan Pascal’s principle – Prinsip Pascal erect image – imej tegak

v
focal length – panjang fokus vibration – getaran step-up transformer – transformer
incident ray – sinar tuju injak naik
wave – gelombang
lateral inversion – songsang sisi transmission – penghantaran
wavefront – muka gelombang
magnified – diperbesarkan
wavelength – panjang gelombang Chapter 4 Electronics
optical centre – pusat optik
plane mirror – cermin satah base – tapak
principal axis – paksi utama cathode rays – sinar katod
Chapter 2 Electricity
ray diagram – rajah sinar collector – pengumpul
real depth – dalam nyata effective resistance – rintangan doping – pengedopan
real image – imej nyata berkesan emitter – pengeluar
reflected ray – sinar pantulan electric charge – cas elektrik fluorescent screen – skrin
reflection – pantulan electric current – arus elektrik berpendarfluor
refraction – pembiasan electric field – medan elektrik forward-biased – pincang ke depan
refractive index – indeks pembiasan electric power – kuasa elektrik full wave – gelombang penuh
total internal reflection – pantulan electromotive force (e.m.f.) – daya half wave – separuh gelombang
dalam penuh gerak elektrik (d.g.e.) hole – lohong
virtual image – imej maya electron flow – aliran elektron integrated circuit – litar bersepadu
energy efficiency – kecekapan tenaga light-dependant resistor (LDR) –
internal resistance – rintangan perintang peka cahaya (PPC)
dalaman light-emitting diode (LED) – diod
FORM 5
parallel circuit – litar selari pemancar cahaya (DPC)
Chapter 1 Waves potential difference – beza logic gate – get logik
keupayaan p-n junction – simpang p-n
amplitude – amplitud
resistance – rintangan rectification – rektifikasi
angle of incidence – sudut tuju
series circuit – litar bersiri reverse-biased – pincang songsang
angle of reflection – sudut pantulan voltage – voltan
angle of refraction – sudut pembiasan thermionic emission – pancaran
work – kerja termion
constructive interference – interferens
truth table – jadual kebenaran
membina
crest – puncak Chapter 3 Electromagnetism
destructive interference – interferens Chapter 5 Radioactivity
alternating current – arus ulang-alik
membinasa biomass – biojisim carbon dating – pentarikhan karbon
diffraction – belauan coil – gegelung chain reaction – tindak balas berantai
echo – gema current-carrying conductor –
critical mass – jisim genting
frequency – frekuensi konduktor membawa arus
direct current – arus terus cloud chamber – kebuk awan
interference – interferens
interference pattern – corak direct current motor – motor arus deflection – pemesongan
interferens terus electroscope – elektroskop
longitudinal wave – gelombang electromagnet – elektromagnet half-life – setengah hayat
membujur electromagnetic induction – aruhan ionising power – kuasa pengionan
elektromagnet nuclear fission – pembelahan nukleus
period – tempoh
magnetic field – medan magnet
plane wave – gelombang satah nuclear fusion – pelakuran nukleus
moving coil ammeter – ammeter
propagation – perambatan gegelung bergerak nucleon number – nombor nukleon
resonance – resonans national grid network – rangkaian grid penetrating power – kuasa
ripple tank – tangki riak nasional penembusan
sound wave – gelombang bunyi primary – primer proton number – nombor proton
superposition – superposisi renewable energy – tenaga yang radiation – sinaran
boleh diperbaharui radioactive decay – reputan radioaktif
transverse wave – gelombang
secondary – sekunder random – rawak
melintang
step-down transformer – transformer
trough – lembangan injak turun

vi
FORM 4

1
CHAPTER

Introduction to Physics

SPM Topical Analysis


Year 2007 2008 2009 2010 2011
Paper 1 2 3 1 2 3 1 2 3 1 2 3 1 2 3
Section A B C A B A B C A B A B C A B A B C A B A B C A B
Number of questions 3 – – – – – 3 – – – 1 – 2 – – – – – 2 1 – – 1 – 3 – – – – –

ONCEPT MAP

Scientific
INTRODUCTION TO PHYSICS
investigations

Observing a situation
Understanding Physical Measurements
Physics quantities Identifying a
problem or question

Scalar Measuring
quantities instruments Identifying
Natural Everyday variables involved
phenomena objects
Vector • Accuracy Formulating
quantities • Consistency a hypothesis
• Sensitivity
Physics
Designing and carrying
concepts
out an experiment
Errors in
Base Derived measurements
quantities quantities
Fields of study Collecting and
in Physics tabulating data

Systematic Random Presenting data


Prefixes errors errors
Interpreting data
and drawing conclusions
Standard
form/scientific
notation Techniques Writing a report
to reduce errors for the investigation
COMPANION WEBSITE
Learning Objectives 1
1.1 Understanding Physics

What is Physics?

1 The word ‘Physics’ comes from the Greek 4 The majority of natural phenomena can be
word ‘Physikos’ which means knowledge of explained using the principles of physics.
nature. For example,
2 The aim of physics is to explain the (a) we cannot see an object behind a wall
fundamental nature of the universe by using because light travels in a straight line.
the simplest explanations. (b) grass appears green because the leaves of
3 Mankind is often eager to understand natural the grass absorbs all the colours of the
F
phenomena. Since the beginning of time, man spectrum except green. Green light is
O
has had a keen interest in the wonders of his reflected, therefore grass appears green.
1

R
environment. This explains man’s interest in
CHAPTER

M
understanding how an appliance works.
4

What is physics? Why study physics?


The goal of physics is to gain a better understanding of the world in which we live. Observe the things around you. Have
you ever considered why and how things around you work?
The laws of physics help us to answer questions like those given below.

How do fireworks work? How does email from your What is Bluetooth technology?
What is launched into the sky to desktop get to a friend How does Bluetooth technology
make such beautiful displays? halfway around the world? create a connection between
electronic devices wirelessly?

How does a telescope function? How did the universe come about? How does a mobile phone
How can we see stars and galaxies, function without wires?
which are far beyond our vision? How do SMS get to our
mobile phone?

What do physicists do? Physicists explore the universe. They try to answer basic questions about the world.
Their investigations range from the study of the vast galaxies which are millions of kilometres away from us to that of the
tiny particles which are smaller than atoms. Many of the technological marvels we see today are the result of the
application of physics. The laws and concepts of physics can help us to understand the occurrence of natural phenomena
around us. We can apply the knowledge of physics to improve the environment and our standard of living.

Introduction to Physics 2
Fields of Study in Physics

1 In general, physics is
mechanic atomic,molecular and
concerned with the study electron physics
of energy and the heat

properties of matter.
2 The fields of study in
physics are divided into light
Modern Physics nuclear
classical physics and Classical Physics – scientific beliefs physics
modern physics. – regarding motion about the basic
and energy structure of the
3 Investigations in modern material world
physics involve the sound
F
observations of particles O

1
particle R
as tiny as protons and physics

CHAPTER
M
neutrons in atoms to
electricity and
objects as large as the magnetism astrophysics relativity 4
stars and planets in the
universe. Figure 1.1 Field of study in Physics

Importance of Physics

1 The knowledge obtained from the study of 4 Research in physics has also led to the use of
physics is important in other sciences, including radioactive materials in the study, diagnosis
astronomy, biology, chemistry and geology. and treatment of certain diseases.
2 There is a close connection between physics 5 The application of physics in the creation of
and practical developments in engineering, sophisticated medical instruments such as
medicine and technology. For example, magnetic resonance imaging (MRI), X-ray,
engineers design automobiles and machines computerised tomography (CT) scan are some
by taking into consideration concepts and examples of its contribution to medical
theories in physics. technology.
3 The study of physics is fast expanding, leading 6 Physics helps to improve the quality of life.
to advances in technology which have in Theories and principles of physics explain the
turn brought many benefits to mankind. The operation of many modern home appliances,
application of these fundamental laws and like the television or the microwave oven.
theories have enabled engineers and scientists to 7 Table 1.1 shows some of the objects which we
put satellites into orbit, receive information from use in our daily lives. These objects function
space probes, and improve telecommunications. and operate on the principles of physics.

Table 1.1
Object Principle of physics
Wristwatch Elasticity, electricity, electronics
Bread toaster Electrical energy, heat
Bicycle Mechanics, dynamics, kinematics
Fan Electromagnetism, mechanics
Computer Electronics, electricity, electromagnetism
Air conditioner Thermodynamics, electronics, mechanics
Mobile phone Waves, electronics
Video camera Optics, electronics

3 Introduction to Physics
Malaysia’s first satellites, MEASAT 1 and MEASAT 2, were built by Hughes Space and
Communications International, Inc. of California. MEASAT 1 was launched on
12 January 1996 from Kourou, French Guiana, and MEASAT 2 was launched on
13 November 1996.
These two satellites serve as a communication network for telephone, television,
and data transmission for trade, education, and other telecommunications services;
with a coverage stretching from India to Hawaii and from Japan to Eastern Australia.

TiungSAT-1 is a micro satellite designed and fully built in Malaysia. It was successfully launched
on 26 September 2000 from Baikonur Cosmodrome, Kazakhstan.
This satellite operates at a height of 1000 km from the Earth’s surface and serves as a
F
O
communication satellite with applications in telecommunications, education, fisheries, forestry,
and pollution control. It contains scientific apparatus, cameras, and other equipment for
1

R
relaying voice and data, signals from remote sensors and global positioning systems (GPS) for
CHAPTER

M
purposes of surveying, weather forecasting, detection of forest fires and oil spills.
4
Photograph of the Earth taken from the TiungSAT-1 at a height of
653 km, at a location with bearings 61° 12′ N, 128° 4′ W.

1.1

1 The following statements are true except


A the word ‘physics’ originated from a Greek word meaning ‘knowledge of nature’.
B physics is a scientific discipline based on experiments.
C theories in physics originated from sciences such as chemistry and biology.
D physicists often construct models to represent a phenomenon.
2 What is physics?
3 State some advantages of studying physics.
4 State several fields of study in physics.

1.2 Understanding Base Quantities and Derived Quantities

Base Quantities
1 A quantity that is measurable is called a 2 Examples of physical quantities are the room
physical quantity. temperature, the volume of a block of wood,
the speed of a car, the pressure of a gas, the
EN weight of a person, etc.
D 3 Quantities which cannot be measured are
non-physical quantities.
4 Physical quantities are categorised into base
quantities and derived quantities.
5 Base quantities are physical quantities that
cannot be defined in terms of other physical
(a) Mass (b) Time (c) Temperature quantities.
Figure 1.2 Daily activities that involve measurement

Introduction to Physics 4
6 Every physical quantity is expressed as a numerical 10 The 11th General Conference of Weights and
value in a particular unit of measurement. Measures held in Paris in 1960 adopted the
For example, International System of Units, universally
abbreviated SI (Système International d’Unités in
Length of a metre rule = 100 cm
French) for the recommended practical system
of units of measurement.
11 The SI defines seven base units: metre, kilogram,
physical numerical unit of second, kelvin, ampere, mole, and candela.
quantity value measurement 12 Table 1.2 shows five important base quantities
and their corresponding SI units.
7 A unit, in measurement, is a quantity adopted
as the standard by which any other quantity of Table 1.2 F
the same kind is measured.
Base quantity Base SI units O
8 There are many different units. A particular

1
R
quantity may be written in different units based Name Symbol Name Symbol

CHAPTER
M
on the cultural background of the community. Length  metre m
9 Below are some examples: 4
Mass m kilogram kg
Mass: grams, kilograms, tonnes, pounds, ounces
Length: inches, feet, miles, metres Time t second s
Time: seconds, minutes, day, hour, month Electric current I ampere A
But in scientific work, life is much easier if Temperature T kelvin K
everyone uses a common system of units.

Base unit Definition


1
metre One metre is the distance travelled by light in a vacuum in ––––––––––– of a second.
299 792 458
0.039 m

One kilogram is the mass of an international prototype in


the form of a platinum-iridium cylinder (0.039 m in
kilogram 0.039 m platinum diameter and height) kept by the International Bureau of
1 kg -iridium
Weights and Measures at Sevres, France.

second One second is the time taken for 9 192 631 770 periods of oscillation of the caesium-133 atom
to occur.
straight parallel conductors

One ampere is the constant current which, if maintained in


two straight parallel conductors of infinite length, of negligible
ampere I=1A I=1A
cross-sectional area, and placed one metre apart in vacuum,
would produce between these conductors a force of
2 × 10–7 newton per metre of length.

1 metre

1
kelvin One kelvin is of the temperature of the triple point of water.
273.16

5 Introduction to Physics
2 Table 1.3 shows some examples of derived
Derived Quantities
quantities and their corresponding derived units.
1 Derived quantities are physical quantities Several derived units are complex. Hence,
derived by combining base quantities. This can special names are used to replace these units.
be done by multiplication or division or both. For example, the special name for the derived
The unit for a derived quantity is known as a quantity, force, is newton (N).
derived unit.
Table 1.3 Derived quantities and units

Derived quantity Derived unit


Formula
F
Name Symbol Unit in SI base unit Special name
O
Length × Breadth m × m = m2
1

R Area A –
CHAPTER

4
Volume V Length × Breadth × Height m × m × m = m3 –

Displacement m = m s–1
Velocity v –
Time taken s

Change in velocity m s–1 = m s–2


Acceleration a –
Time taken s

Momentum p Mass × Velocity kg × m s–1 = kg m s–1 –

Mass kg
Density ρ = kg m–3 –
Volume m3

Force F Mass × Acceleration kg × m s–2 = kg m s–2 newton, N

Force kg m s–2 = kg m–1 s–2 or


Pressure P pascal, Pa
Area m2
N
= N m–2
m2

Frequency f 1 1 = s–1 hertz, Hz


Period s

Work W Force × Displacement kg m s–2 × m = kg m2 s–2 joule, J


or N × m = N m

Power P Work kg m2 s–2 = kg m2 s–3 or watt, W


Time taken s
J = J s–1
s

Electric charge Q Current × Time A × s =A s coulomb, C

Introduction to Physics 6
1

Figure 1.3 shows an electric kettle used to boil water power is used, the time taken to boil the same volume
of mass 2 kg. of water would be less than 15 minutes.
From the above description, identify the physical
quantities. Then, classify these quantities into base
quantities and derived quantities.
Solution
The physical quantities are mass, electric current,
Figure 1.3 energy, temperature, power, time and volume.
Classification: F
When the switch is turned on, the electric current in
Base quantities: mass, electric current, temperature O
the heating element produces heat energy. The

1
and time R
temperature of water increases steadily until it starts to

CHAPTER
Derived quantities: energy, power and volume M
boil after 15 minutes. If another kettle with a greater
4

Standard Form

1 Scientists have developed a shorter method of where


expressing very large or very small numbers. (a) 1  A  10 and A can be an integer or a
This method is called standard form or decimal number.
scientific notation. (b) n is a positive integer for a number greater
2 Standard form is based on powers of the base than one or a negative integer for a
number 10. In standard form, the magnitude number less than one.
or numerical value of a physical quantity can 3 All numbers can be written in the standard
be written as: form where only one digit appears before the
decimal point, the value being adjusted by
A × 10n
multiplying by the appropriate power of 10.

Green light of wavelength 0.00000051 m travels at a speed of 300 000 000 m s–1.
Express these values in standard form.
Solution
Wavelength of green light = 0.00000051 m
= 5.1  10–7 m
Speed of light = 300 000 000 m s–1
= 3.0  108 m s–1

Prefixes

1 A prefix is a letter placed at the beginning of a written in standard form as 2.0 × 103 metres,
word to modify its meaning. The SI allows can also be written as 2.0 kilometres where the
other units to be created from standard or base prefix kilo (k) means 1000. Table 1.4 gives the
units by using prefixes, which act as prefixes for other multiples or sub-multiples
multipliers. of 10.
2 The SI units, used with prefixes, indicate 3 From Table 1.4, it can be seen that the prefix
multiples or sub-multiples of a base unit. For symbols make it easier for us to express very
example, 2000 metres (metre as base unit) large or very small numbers.

7 Introduction to Physics
Table 1.4 For example,
(a) the unit of speed is metre per second or m s–1,
Power/
Prefix Symbol Value which is in terms of the base units metre and
Factor
second.
tera- T 1012 1 000 000 000 000 (b) the unit of area is square metre or m2, which is
giga- G 109 1 000 000 000 a multiple of the base unit metre.
mega- M 106 1 000 000
kilo- k 103 1000 3
hecto- h 102 100
deka- da 10 10 Identify the smallest and the largest measurements
deci- d 10–1 0.1 from the following.
F
O centi- c 10–2 0.01
2.15 × 10–2 cm
1

R milli- m 10–3 0.001 2.15 × 1010 µm


CHAPTER

M
micro- µ 10–6 0.000001 2.15 × 103 km
4 nano- n 10–9 0.000000001 2.15 × 108 nm
pico- p 10–12 0.000000000001
Solution
Convert each measurement to the same base unit, i.e.
metre, m.
2.15 × 10–2 cm = (2.15 × 10–2) × 10–2 m
Rules and conventions for using the SI: = 2.15 × 10–4 m
(a) A prefix is written in front of the symbol for the unit 2.15 × 10 µm = (2.15 × 1010) × 10–6 m
10

without a space. = 2.15 × 104 m


For example, kilowatt is written kW. 2.15 × 10 km = (2.15 × 103) × 103 m
3

(b) A space is used between symbols in derived units. = 2.15 × 106 m


For example, newton metre is written N m.
2.15 × 10 nm = (2.15 × 108) × 10–9 m
8
(c) Compound prefixes are not used, i.e. a unit may take
= 2.15 × 10–1 m
only one prefix.
For example, nm is 10–9 m but 10–9 m is not mµm. The smallest measurement is 2.15  10–2 cm.
(d) The symbol for a unit with its prefix is considered to
be a single symbol. The largest measurement is 2.15  103 km.
For example, 1 cm2 = (0.01 m)2 and not 0.01 m2.
(e) Gram is a special unit in the SI although gram is not a
SI unit. Prefixes are used with the symbol g and not
with kg. 4
For example, 103 kg is written Mg and not kkg.
(f) When writing numbers in SI units, digits are arranged Derive the units for the following physical quantities
in groups of three with a space, instead of a comma, (a) Density (d) Force
between the groups of digits. In the case of a number (b) Velocity (e) Work
consisting of 4 digits only, the number is written (c) Acceleration
without a space.
in terms of base units.
For example, 1000 W, 300 000 J and 0.0001 A.
Solution
Unit [Mass]
Expressing Derived Quantities and (a) Unit [Density] = _____________
Unit [Volume]
Their Units in terms of Base Quantities
kg
and Base Units = 3 = kg m–3
m
Derived quantities and their units can be separated Unit [Displacement]
into their respective base quantities and base units. (b) Unit [Velocity] =
Unit [Time]
Sometimes, the derived unit shows the relationship m = m s–1
between the derived quantity and the base =
s
quantity.

Introduction to Physics 8
(c) Unit [Acceleration] Solving Problems Involving Conversion
Unit [Change in velocity] of Units
=
Unit [Time]
A derived quantity in a specified unit can be
m s–1 = m s–2
= s expressed in terms of a different unit by the use of a
multiplication factor to change the value of the
(d) Unit [Force]
prefix in the SI unit.
= Unit [Mass] × Unit [Acceleration]
For example,
= kg × m s–2
(a) Convert the unit of area in m2 to cm2.
= kg m s–2
1 m = 100 cm
(e) Unit [Work]
1 m2 = (100 cm)2
= Unit [Force] × Unit [Displacement]
= 1002 cm2
= kg m s–2 × m F
= 10 000 cm2
= kg m2 s–2 O
= 104 cm2

1
R
(b) Convert the unit of volume in mm3 to m3.

CHAPTER
M
1 mm = 0.001 m
= 10–3 m 4
5 1 mm = (10–3 m)3
3

= (10–3)3 m3
Based on the formulae for derived quantities in Table = 10–9 m3
1.3, show the relationship between the following derived
quantities and their corresponding base quantities.
(a) Acceleration (c) Force 6
(b) Momentum (d) Density
Convert
Solution (a) 0.0008 Gm to µm.
Change in velocity
(a) Acceleration = (b) 78 µm to Mm.
Time Express your answers in scientific notation.
 
Displacement
Time Solution
= (a) 0.0008 Gm
Time
Length = 8 × 10–4 × 109 m 1 Gm = 109 m
= = 8 × 105 m
(Time)2
= 8 × 105 × 106 µm 1 µm = 1 × 10–6 m
(b) Momentum = Mass × Velocity = 8 × 1011 µm ∴ 1 m = 106 µm
Displacement (b) 78 µm
= Mass ×
Time = 7.8 × 101 × 10–6 m
Length = 7.8 × 10–5 m
= Mass × 1 Mm = 106 m
Time = 7.8 × 10–5 × 10–6 Mm ∴ 1 m = 10–6 Mm
= 7.8 × 10–11 Mm
(c) Force = Mass × Acceleration
Change in velocity
= Mass ×
Time
7
 
Length
Time The density of sea water is 1.05 g cm–3.
= Mass ×
Time Express this density in the unit kg m–3.
Length
= Mass × Solution
(Time)2
1 g = 10–3 kg
Mass
(d) Density = 1 cm = 10–2 m
Volume 1.05 g cm–3 = 1.05 × (10–3 kg) × (10–2 m)–3
Mass = 1.05 × (10–3 kg) × (106 m–3)
=
(Length)3 = 1.05  103 kg m–3

9 Introduction to Physics
8 1.2
1 The wavelength of blue light is 0.000000475 m.
Azman rides his bicycle at a velocity of 20 km h–1. Write its wavelength in nm.

2 Calculate the volume, in m3, of a block of wood


with dimensions 4.0 cm × 1.5 cm × 1.4 cm.

3 Convert
(a) 72 km h–1 to m s–1.
(b) 15 m s–1 to km h–1.
Figure 1.4
F4/1/5

F What is his velocity in m s–1?


O 4 Which of the following correctly matches a physical
1

R Solution quantity with its corresponding SI base unit?


CHAPTER

M
20 km
20 km h–1 = –––––– Physical quantity SI base unit
4 1h
A Temperature °C
20 × 103 m = 5.6 m s–1
= ––––––––––
3600 s B Area cm2
C Force kg m s–2
D Power J
1 SPM
Clone
’09
5 Which of the following quantities is correctly
Which of the following measurements is the smallest? matched to its SI base unit?
A 1.57  107 µA
F
B 1.57  102 mA Quantity SI base unit
O
C 1.57  10–2 A
1

RA Area cm2
CHAPTER

MB Momentum kg m s–1
Comment
First, convert all the measurements into the same 4C Frequency s
unit, that is ampere (A), then compare their values.
A 1.57  107 µA = 1.57  107  10–6 A
= 1.57  10 A 6 Write the following quantities in standard form.
B 1.57  102 mA = 1.57  102  10–3 (a) 0.000000000521 m
(b) 0.02413 J
= 1.57  10–1 A
–2 (c) 3 036 000 000 000 km
C 1.57  10 A
Answer C 7 Express the following measurements in the units
specified.
(a) 1 s = ____________ ns
2 SPM
Clone
(b) 1 ms = ____________ s
’06 (c) 3.86 ms = ____________ ns
Which of the following physical quantities is not a (d) 1 g = _____________ mg
base quantity? (e) 1 kg = ____________ g
A Time C Length (f) 643 mg = ____________ kg
B Weight D Electric current
Comment 8 Express each measurement in row I in the unit
specified in row II.
Weight is a derived quantity.
Weight = Mass  Gravitational strength
W = mg II 0.4 mm 240 mg 0.06 m 40 cm2 0.02 m2
II m kg mm m2 mm2
Answer B

Introduction to Physics 10
1.3 Understanding Scalar and Vector Quantities

Scalar and Vector Quantities

Most quantities measured in science are classified as either scalar or vector quantities.

Physical quantities

Scalar quantity Vector quantity


• A physical quantity which has only magnitude or size. • A physical quantity which has both magnitude
and direction.
Walk 500 m and you will
find the hospital. Walk 500 m due east and you F
will find the hospital. O
Which way?

1
R

CHAPTER
M

rth 4
no

eas
t

Common examples:
Length Speed Time Density Common examples:
Temperature Power Mass Energy Force Acceleration Velocity
Volume F4/1/4
Work Distance Momentum Displacement Weight
F4/1/5

1.3
1 How is a scalar quantity different from a vector 2 Give five examples of scalar quantities.
quantity? Give an example of each. 3 Give five examples of vector quantities.

(b) a metre rule,


1.4 Understanding (c) a pair of vernier callipers and
Measurements (d) a micrometer screw gauge.
1 Physics is a science which involves measurements
of various physical quantities.
2 Choosing an appropriate instrument to measure
a physical quantity depends on the size of the
measurement and the accuracy needed. Figure 1.5
For example, to measure a length, we can 3 Table 1.5 showsF4/1/6
various instruments suitable
choose between for use to measure the lengths of objects
(a) a measuring tape, depending on their sizes and accuracies.
Table 1.5
Length to be measured Suitable instrument Example Accuracy Example of reading
Several metres or more Measuring tape Length and breadth of a 1 cm 56 cm
(Figure 1.5) classroom
Several cm to 1 m Metre rule Length of a simple pendulum 0.1 cm 2.6 cm
Between 1 cm to 12 cm Vernier callipers External and internal 0.01 cm 2.07 cm
diameter of a test tube
Less than 2.5 cm Micrometer screw gauge Diameter of a wire 0.01 mm 2.00 mm or
(0.001 cm) 0.200 cm

11 Introduction to Physics
Measuring Tape
• Carpenters use a metal
1 A measuring tape is used for measuring a comparatively long distance
measuring tape for more
without the need for accuracy. accurate measurements. It
2 It is often used in sports events such as the long jump, shot-put and has an accuracy of 0.1 cm.
javelin. • Tailors also use a flexible
3 It is not accurate in its measurement and has an accuracy of plastic measuring tape
1 cm (i.e. the smallest division on the tape is 1 cm). However, it is which has an accuracy of
flexible and therefore is useful for measuring distances or lengths other 0.1 cm.
than a straight line.
4 It is also useful for measuring the circumference of a round object, for
F
example the sizes of the trees in the forest.
O
1

R
CHAPTER

M Parallax errors in measurement arise as a


Metre Rule
result of taking a reading, with the eye of
4
the observer in the wrong position with
1 A metre rule is used to measure lengths from a
respect to the scale of the ruler. Figure 1.6
few cm up to 1 m. A metre rule has an accuracy
shows the correct position of the eye when
of 0.1 cm (i.e. 1 mm).
reading the scale.
2 Precautions to be taken when using a ruler:
(c) Avoid zero errors and end errors.
(a) Ensure that the object is in contact with
The ends of a ruler, which may be worn
the ruler to avoid inaccurate readings.
out, are a source of errors in measurement.
(b) Avoid parallax errors by ensuring the line
Thus it is advisable to use the division
of sight of the observer is perpendicular to
mark ‘1’ of the scale as the starting point
the scale of the measuring instrument. F
when taking a measurement.
O
1

wooden
R
ruler block
CHAPTER

1 2 3 4 Figure 1.7 End error

Length of the block,  = 3.2 cm – 1.0 cm


Figure 1.6 Parallax error = 2.2 cm

accurate reading = 2.9 cm


9

Figure 1.8 shows the measurement of the length of a (c) State one category of error that must be avoided
wooden block with a ruler. when reading the scale.
(d) What is the length of the wooden block?
ruler

wooden block Solution


(a) Accuracy = 0.1 cm
(b) To avoid end errors.
(c) Parallax error
Figure 1.8 (d) Length of block
= (2.8 – 0.6 ) cm
(a) State the accuracy of the ruler.
= 2.2 cm
(b) Why is the zero mark on the ruler not used as the
origin of the measurement?

Introduction to Physics 12
Vernier Callipers
1 A pair of vernier callipers is used to measure an object with dimensions up to 12.0 cm with an accuracy
of 0.01 cm*.

There are two scales on the steel bars.


1 Main scale: Graduated in intervals of 0.1 cm from 0 to 12 cm.
2 Vernier scale: A scale which can slide on the main scale.

beaker F
How to take the reading O
6 Main scale reading

1
R

CHAPTER
3
•  Read the mark on the main scale M
3.24 cm inside jaws preceding the ‘0’ mark on the vernier
4
scale. The ‘0’ mark on the vernier scale
acts as pointer for the main scale
3.24 cm
reading.
•  The ‘0’ mark on the vernier scale in this
2 main scale 5 tail example lies between
vernier scale 1
3.2 cm and 3.3 cm. Therefore, the
3.24 cm
reading on the main scale is
3.2 cm.
4 outside
jaws
7 Vernier scale reading
•  Read the mark on the vernier scale that
is exactly in line or coincides with any
mark on the main scale.
•  In the example, the fourth mark on the
vernier scale is exactly in line with a
the ‘0’ mark on mark on the main scale.
the vernier scale Therefore, the vernier scale reading
= 4  0.01 cm
Figure 1.9
= 0.04 cm
F4/1/7 Vernier scale reading
= Main scale reading +
vernier scale reading
Parts and Functions = 3.2 cm + 0.04 cm
3 Inside jaws — To measure the internal = 3.24 cm
diameter of an object.
4 Outside jaws — To measure the external
diameter or length of an
object. *  It means any readings taken from the vernier
callipers has to be written to two decimal
5 Tail — To measure the depth of an object.
places even it is a whole number, e.g. 2.00 cm.

13 Introduction to Physics
2 An instrument which does not register a zero reading when the actual reading is zero has a zero
error. A pair of vernier callipers has a zero error if the ‘0’ mark on the main scale is not in line
with the ‘0’ mark on the vernier scale when the jaws of the callipers are fully closed.

To eliminate the zero error: Correct reading = Callipers reading – Zero error

Zero errors

Positive zero error No zero error Negative zero error

0 1 0 cm 1
0 1
F main scale
main scale main scale
O
vernier scale vernier scale
1

R vernier scale
0 10 0 10
CHAPTER

M 0 5
10
The fourth mark on the vernier scale is
4 in line with a mark on the main scale. The ‘0’ mark on the main scale is The second mark from the '10'
∴ Zero error = 0.04 cm exactly in line with the ‘0’ mark on mark on the vernier scale is in line
the vernier scale when the pair of with a mark on the main scale
The ‘0’ mark on the vernier scale is to the Zero error = –0.02 cm
vernier callipers is fully closed.
right of the ‘0’ mark on the main scale. The ‘0’ mark on the vernier
scale is on the left of the
‘0’ mark on the main scale.

10

Figure 1.10 shows the use of a pair of vernier callipers


to measure the diameter of a steel ball.
What is the diameter of the steel ball?
Solution
Main scale reading = 2.1 cm steel ball
Vernier scale reading = 0.05 cm
Diameter of the steel ball = 2.1 cm + 0.05 cm = 2.15 cm Figure 1.10

11

Figure 1.11(a) shows the use of a pair of vernier


callipers to measure the size of an object.
Figure 1.11(b) shows the reading before the pair of
object
vernier callipers is used.
Determine the correct size of the object.
Solution
First, determine the zero error.
Zero error = +0.02 cm (a)
Then, find the reading from the pair of vernier callipers.
Main scale reading = 3.2 cm 0 1
Vernier scale reading = 0.04 cm
∴ Vernier calliper reading = 3.2 + 0.04 = 3.24 cm
0 5 10
Correct size of object
= Vernier calliper reading – Zero error (b)
= 3.24 – (+0.02) = 3.24 – 0.02 = 3.22 cm Figure 1.11

Introduction to Physics 14
Micrometer Screw Gauge

1 A micrometer screw gauge is used to measure small lengths ranging between 0.10 mm and 25.00 mm. It
can be used to measure diameters of wires and thickness of steel plates to an accuracy of 0.01 mm
(therefore, any readings obtained from an micrometer screw gauge should be written to two decimal
places with the unit mm, e.g., 2.00 mm).

There are two scales.


1 Main scale:  A scale which is marked on the sleeve. F
2 Thimble scale:  A scale which is marked on the thimble. O

1
R

CHAPTER
M

4
3 4 2 thimble (with thimble scale)
3 and 4
anvil spindle lock
5 ratchet knob  hen taking a reading, the thimble is turned
W
until the object is gripped very gently
between the anvil and the spindle.

sleeve
(with
main
1
scale)
5 T
 he ratchet knob is then turned until a
‘click’ sound is heard.
mm 5 1 mm 30 The ratchet knob is used to prevent the
7 user from exerting too much pressure on
25 the object.
1 mm
horizontal 20 The grip on the object must not be
reference excessive, as this will affect the accuracy
line
of the reading.
6
Figure 1.12
F4/1/8

How to take the reading 7 Thimble scale reading


6 Main scale reading Read the thimble scale reading at the point
Read the main scale reading at the edge of the where the horizontal reference line of the main
thimble. Take note that an additional half scale scale is in line with the graduation mark on the
division (0.5 mm) must be included if the mark thimble scale.
below the horizontal reference line is visible. Therefore, the thimble reading = 0.26 mm
Therefore, the reading on the main scale = 7.5 Thus, the reading of the micrometer screw gauge
mm = Main scale reading + Thimble scale reading
= 7.5 mm + 0.26 mm = 7.76 mm

15 Introduction to Physics
2 The accuracy of the micrometer screw gauge is also affected by zero errors.
Before using it, determine its zero error, if any.
The zero error can be corrected by using the formula below:

Correct reading = Reading obtained – Zero error

No zero error
horizontal 5
reference line
0 ‘0’ mark is in line
with the horizontal
45 reference line

F
O
Positive zero error Negative zero error
1

R
CHAPTER

M horizontal horizontal
reference line 4th mark above ʻ0ʼ reference line 0 3rd mark below ʻ0ʼ
4 5 is in line with the is in line with the
horizontal Zero errors 45 horizontal
0 reference line reference line

Zero error = +0.04 mm Zero error = – 0.03 mm

12 Nature of Measurement

1 Measurements are trials to determine the


Figures 1.13(a) and (b) show the readings on a
actual value of a particular physical quantity.
micrometer screw gauge before and after being used
2 The difference between the actual value
to measure the size of an object.
of a quantity and the value obtained in
measurement is the error.
mm
0
3 To measure is to make an acceptable estimate.
45

Errors in Measurement
(a)
1 No measurement is exact. All measurements
mm 0 10 45 will have some degree of error or uncertainty.
40 2 There are two main types of errors.
35 (a) Systematic errors
(b) Random errors
(b)
Figure 1.13

What is the correct size of the object? Systematic Errors

Solution 1 Systematic errors are cumulative errors that


The reading of the gauge can be corrected, if the errors are known.
= 16.00 mm + 0.41 mm 2 Systematic errors in measurement result from
= 16.41 mm (a) an incorrect position of the zero point,
Zero error = – 0.02 mm known as zero error,
Correct reading (b) an incorrect calibration of the measuring
= 16.41 – (– 0.02) instrument.
= 16.41 + 0.02 3 Systematic errors always occur (with the same
= 16.43 mm value) if we continue to use the instrument in
the same way.

Introduction to Physics 16
4 Systematic errors explain why all readings Parallax errors are present in positions A, B,
taken are always larger or always smaller than and D, thereby giving inaccurate readings.
the actual value.
5 A zero error arises when the measuring
instrument does not start from exactly Reading = 15.2
zero.
Reading = 15.1
6 Zero errors can be corrected or eliminated.
(a) For measurements of lengths (using a pair Reading = 15.0

of vernier callipers and micrometer screw Reading = 14.9


gauge):
Keys:
Correct reading Wrong reading
= Reading obtained – Zero error Correct reading
(b) For electrical instruments, by adjusting the F
Figure 1.15 O
zero adjustment screw.

1
R
3 An observer P, who repeatedly puts his eye at

CHAPTER
3 4
M
2 2 3 4
1 1 point A will read 15.2 m every time, so
0 0
A V parallax has caused a systematic error of 4
5

5
or
+0.2 m.
4 Another observer Q, who positions his eye
carelessly, and randomly, anywhere between A
and D will read values which vary randomly,
ranging from 14.9 m to 15.2 m. For this
observer, parallax has become a source of
random error.
Zero adjustment screw is adjusted until the pointer 5 If the observer Q repeats his reading several
is at the ‘0’ mark before use.
times, and takes the average of the results, he’ll
Figure 1.14 end up with an answer that’s closer to the
actual value; repeating the measurements will
Random Errors do nothing at all for the observer P.
6 How to avoid parallax errors?
1 Random errors may occur for a variety of (a) To avoid parallax errors, the position of
reasons. They may be due to the eye must be in line with the reading to
(a) personal errors such as parallax error. be taken, as in position C (Figure 1.15).
(b) natural errors such as changes in wind, (b) To overcome parallax errors in instruments
temperature, humidity, refraction, magnetic with a scale and pointer, e.g. an ammeter,
field or gravity while the experiment is in it is often useful to have a mirror behind
progress. the pointer. The correct reading is obtained
(c) the use of a wrong technique of measurement by making sure that the eye is exactly in
such as applying excessive pressure when front of the pointer, so that the reflection of
turning a micrometer screw gauge. the pointer in the mirror is right behind it
2 Random errors can be minimised by repeating (refer Figure 1.16).
the measurements several times and taking the pointer's image
average or mean value of the readings.
anti-
parallax
mirror
Parallax Errors

1 A parallax error is an error in reading an


instrument because the observer’s eye and the (a) Wrong reading () (b) Correct reading ()
pointer are not in a line perpendicular to the – pointer’s image – pointer’s image
plane of the scale. can be seen in the cannot be seen, it is
2 Figure 1.15 shows the observer’s eye at four mirror right behind the
different positions A, B, C, and D, while pointer
reading the scale of a measuring cylinder. Figure 1.16

17 Introduction to Physics
Consistency, Accuracy and Sensitivity

Measurement

Consistency Accuracy Sensitivity


1 The consistency of a measuring 1 Accuracy is the degree 1 The sensitivity of a measuring
instrument is its ability to of how close a instrument is its ability to respond
register the same reading measurement is from quickly to a small change in the value
F when a measurement is made the actual value. of the quantity to be measured.
O repeatedly. 2 An accurate instrument 2 A measuring instrument that has a scale
1

R is able to give readings with smaller divisions is more sensitive.


CHAPTER

M close to or almost equal For example,


to the actual value of a Ruler A
4
quantity.
0 cm 1 2 3 4 5 6 7 8 9 10 11 12 13 14
actual
value
Ruler B
Consistent Inconsistent accurate 0 mm
10 20 30 40 50 60 70 80 90 100 110 120 130 140
Figure 1.17
2 A set of measurements is inaccurate Ruler B is more sensitive than ruler A as it has
consistent if all the values are an accuracy of 0.1 cm compared to ruler A
Figure 1.18
close to the mean value. which has an accuracy of 0.5 cm
3 An instrument with Figure 1.19
High consistency 100% accuracy does
⇒ Small deviation from 3 As an example, a micrometer screw
not exist.
the mean value gauge is more sensitive than a pair of
4 The error is the
vernier callipers. When the diameter of
difference between the
3 A deviation is the difference a heated wire increases by 0.01 mm,
measured value and
between the measured value and the pair of vernier callipers is unable to
the actual value.
its mean value or the average detect it.
5 A measured value with
value. 4 In addition to the size of the divisions
a very small error has a
4 The consistency of a measuring on the scale of the instrument, the
high accuracy.
instrument can be improved by design of the instrument has an effect
6 Ways to improve the
(a) eliminating parallax errors on the sensitivity of the instrument.
accuracy of a
during measurement. For example, a thermometer has a
measurement:
(b) exercising greater care and higher sensitivity if it can detect small
(a) Repeated readings
effort when taking readings. temperature variations. A thermometer
are taken and the
(c) using an instrument which is with a narrow capillary and a thin-
average value is
not defective. walled bulb is more sensitive.
calculated.
5 A sensitive measuring instrument
(b) Avoid parallax
responds faster and to a greater extent
errors.
with small variations in the physical
(c) Avoid zero errors.
quantity measured.
(d) Use measuring
6 The measurement of large quantities in
instruments with a
units of kilogram or kilometre does not
higher accuracy.
require sensitive instruments as small
For example, a
variations in such measurements can
pair of vernier
be ignored. On the other hand, the
callipers is more
measurement of the diameter of a wire,
accurate than a
which has a smaller value, requires a
metre rule.
more sensitive instrument.

Introduction to Physics 18
13

Why is a thermometer unable to measure the actual temperature of a warm liquid?


Solution
A portion of the heat in the liquid is absorbed by the thermometer after the thermometer is placed in the liquid.
This will lower the temperature of the liquid.
For example, the temperature of a liquid is 70 °C before measurement. When the thermometer is placed in the
liquid, it gives a reading of 69 °C.

Comparisons between Consistency, Accuracy, and Sensitivity


F
O

1
R

CHAPTER
M

Consistent but inaccurate Accurate and consistent Accurate but inconsistent Inaccurate and inconsistent

Figure 1.20

1 The drawings in Figure 1.20, which show the with a metre rule may be consistent but not
distribution of gunshots fired at a target board, accurate due to end errors. In this respect, this
serve to illustrate the meaning of consistency type of instrument gives readings which,
and accuracy. however, do not represent the actual value of
2 The bullseye in the centre of the target board the measured quantity.
represents the actual value of the quantity to 4 A sensitive measuring instrument too, may not
be measured. be accurate or consistent. This is due to
3 A consistent measuring instrument is not external variations which cause variations in
necessarily accurate. For example, a measurement the readings.

14 Other Instruments

Figure 1.21 shows the method of increasing the Stopwatch


accuracy of the metre rule to 0.01 cm compared to its
original accuracy of 0.1 cm. 1 A stopwatch is used to measure time intervals.
2 There are two types of stopwatches.
metre rule (a) The analogue stopwatch, which is
mechanically operated, with an accuracy
of 0.1 s or 0.2 s.
(b) The digital stopwatch, which is electronically
rod
operated, with an accuracy of 0.01 s.
10 turns of wire

Figure 1.21

What is the diameter of the wire?

Solution
0.5 cm Accuracy: Accuracy: Accuracy:
Diameter of wire, d = 10 turns
0.2 s 0.1 s 0.01 s
10 of wire.
(a) Analogue (b) Digital
= 0.05 cm
Figure 1.22 Types of stopwatches

19 Introduction to Physics
Thermometer Ammeter and Voltmeter

1 There are two types of mercury thermometers 1 An ammeter is an instrument used to measure
which are commonly used. the amount of electric current flowing
(a) Mercury thermometers with a range of through a particular point in an electrical
temperature measurements between –10°C circuit. It has a scale graduated in the SI unit,
and 110°C, with an accuracy of 1°C. ampere (A).
(b) Mercury thermometers with a range of 2 The common ammeter has an accuracy of
temperature measurements between 0°C 0.1 A or 0.2 A.
and 360°C, with an accuracy of 2°C. 3 For measuring very small currents, a
milliammeter with an accuracy of 0.1 mA or
0.2 mA is used.
F
O (a) Range: –10 °C to 110 °C Accuracy: 1 °C power source
1

R
CHAPTER

M
ammeter resistor
4

(b) Range: 0 °C to 360 °C Accuracy: 2 °C

Figure 1.23 Types of thermometers switch

Figure 1.24 Connection of an ammeter in an


2 A mercury thermometer is a sensitive electrical circuit
instrument because
4 A voltmeter is an instrument used to measure
(a) mercury is a liquid metal which is sensitive
the potential difference (voltage) between any
to temperature changes. It expands and
two points in an electrical circuit. Voltmeters
contracts uniformly with temperature.
have an accuracy of 0.1 V or 0.2 V.
(b) the thin-walled glass bulb allows a quick
heat transfer between the heat source and power source
the mercury.
(c) the capillary tube, which has a small
diameter, amplifies a small expansion in resistor voltmeter
the bulb into a large linear expansion
along the length of the capillary tube.
switch

Figure 1.25 Connection of a voltmeter in an electrical circuit

3 SPM
Clone
’06

The figure below shows two ammeters. Which ammeter is more sensitive?
Explain your answer.
1 2 0.5
0 3 0 1 Comments
A A
Sensitivity of a measuring instrument depends on the
value of the smallest division on the measuring scale.
The values of the smallest divisions for P and Q are
0.1 A and 0.05 A respectively.
Answer
Ammeter P Ammeter Q Ammeter Q is more sensitive as its smallest division is
smaller in value (0.05 A). The smaller the value of the
smallest division, the more sensitive the instrument is.

Introduction to Physics 20
1.4

1 The compression weighing scale used in a grocery


shop is exact but it lacks sensitivity. Why?

2 What is the difference between sensitivity, accuracy


and consistency?

3 The figure shows the measurement of the diameter


of a 20-sen coin using a metre rule with the help of
two set squares. F
O

1
R

CHAPTER
coin M
(a)
4
set
square

metre
rule

0 cm 1 2 3 4 5 6
(b)

What is the actual width of the spoon?


What is the diameter of the coin?

6 Figure (a) shows a micrometer screw gauge being


4 The figure below shows a pair of vernier callipers used to measure the diameter of a pencil. Figure (b)
being used to measure 100 sheets of paper. shows the reading when the gap of the micrometer is
closed.

main
40
scale
0 5
35
vernier
scale
30

(a)
What is the thickness of a sheet of paper?
What is the advantage of measuring 100 sheets of
paper instead of a sheet directly?

5 Figure (a) shows a pair of vernier callipers being used


to measure the width of a spoon. Figure (b) shows (b)
the reading when the jaws are closed. What is the actual diameter of the pencil?

21 Introduction to Physics
1.5 Analysing Scientific Investigations
3 The theories postulated have to be verified by
Scientific Investigations
experiments. Based on the experimental
results and his observations, the physicist will
1 Studies in physics involve precise and complete draw conclusions as to whether the theory can
observations, and accurate measurements. be accepted or rejected.
2 A physicist makes observations of natural 4 The steps in a scientific investigation are
phenomena and attempts to put forth theories shown in Table 1.6.
to explain the phenomena.

F Table 1.6
O
Step Explanation
1

R
CHAPTER

M 1 Making • Making observations and gathering all available information about


observations the object or phenomenon to be studied.
4
• Observations are made using the senses of sight, hearing, touch,
taste, and smell.

2 Identifying a • After all the information has been gathered, a suitable question is
suitable question suggested for a scientific investigation.

3 Drawing • Drawing inferences about the object or phenomenon studied.


inferences • An inference is an early conclusion that you draw from an
observation or event using information that you already have on it.

4 Identifying • Variables are factors or physical quantities which change in the


and course of a scientific investigation.
controlling • Identifying the manipulated variables, responding variables, and
variables fixed (or constant) variables.
• Manipulated variables are the physical quantities which you
control and change (i.e. manipulate) for the purpose of investigating
the results of an experiment.
• Responding variables are the physical quantities whose values
change as a result of the changes made to the manipulated variables.
• Fixed or constant variables are physical quantities which are kept
constant throughout the experiment.

5 Formulating • A hypothesis is a general statement that is assumed to be true


a hypothesis regarding the relationship between the manipulated variable and
the responding variable in the phenomenon under investigation.
• A hypothesis must be tested to verify its validity. The test is in the
form of experiments.

6 Designing • Experiments must be planned to include the selection of


and conducting apparatus/materials and working procedures.
experiments

7 Tabulating data • Experimental data are collected and tabulated in a suitable form.

8 Analysing • Analysis of data is carried out by plotting a graph, followed by the


data interpretation of the graph and the drawing of conclusions.

9 Writing a report • A complete report is written for the investigation.

Introduction to Physics 22
5 The following is an example of a systematic scientific investigation on the period of oscillation of a
pendulum, conducted by a student.

1 Making an observation 7 Drawing a conclusion

Hafiq took his brother to the playground. From the shape of the graph of T against , Hafiq was
His brother played on two different swings able to draw a conclusion on the validity of his
as shown in Figure 1.26. Swing A is hypothesis.
shorter than swing B.

Swing B

F
Swing A
O
6 Conducting the experiment

1
R

CHAPTER
M
Hafiq decided to verify his hypothesis by using
apparatus such as a simple pendulum. He conducted an 4
experiment and wrote a report as described in
Experiment 1.1 (page 24).

Figure 1.26

He noticed that the period of oscillation of


each swing was different.

5 Formulating a hypothesis

From his observations, Hafiq arrived at the hypothesis


that the period of oscillation of a swing increases as the
length of the swing increases.

2 Identifying a suitable question

From the observation, Hafiq started to


think, ‘What are the factors affecting the
time or the period of oscillation of the
swing? How does the period of the swing 4 Identifying and controlling
vary with its length?’ So he decided to variables
investigate how the period of oscillation of
Hafiq proceeded to identify the variables relevant to
the swing varies with its length.
the oscillation of the swing:
(a) Period of oscillation, T
(b) Length of swing, 
(c) Mass of the load, m
(d) Amplitude of oscillation, θ
Hafiq classified all the variables under specific
categories.
(a) Manipulated variable: length of swing, 
3 Drawing inferences
(b) Responding variable: period of oscillation, T
Hafiq inferred that the period of (c) Fixed variables: mass of load, m, and amplitude of
oscillation of a swing, i.e. the time taken oscillation, θ
for one complete oscillation, is larger He intends to conduct his experiment in a laboratory so
(i.e. slower) if the length of the swing as to control external variables such as wind
increases. disturbances.

23 Introduction to Physics
1.1

Analysing scientific investigation


Inference Arrangement of apparatus
The period of oscillation of a pendulum depends on plywood
its length. clamp
Hypothesis
The longer the length of the pendulum, the longer
its period of oscillation.
F Aim
O To investigate the relationship between the period of
1

R oscillation of a pendulum and the length of the


CHAPTER

M pendulum.
4 Variables stopwatch

(a) Manipulated: length of pendulum,  Figure 1.27


(b) Responding : period of oscillation, T
4 The bob is displaced to the side and allowed to
(c) Fixed : mass of pendulum bob, m and
oscillate at a small angle of oscillation.
amplitude of oscillation, θ
5 The time t1 for 20 oscillations is measured with
Apparatus/Materials a stopwatch.
Retort stand with clamp, metre rule, stopwatch, 6 The time t2 for another 20 oscillations is
protractor, 100 cm thread and two small pieces of measured. The mean value t20 is calculated.
plywood. t
––20– is calculated.
7 The period of oscillation, T = 20
Procedure
1 A simple pendulum is prepared. 8 The process is repeated for  = 30.0 cm,
2 The apparatus is arranged as shown in Figure 1.27. 40.0 cm, 50.0 cm and 60.0 cm.
3 The thread is adjusted so that the length, , from 9 The data is recorded in Table 1.7.
the centre of the bob to the point of suspension 10 A graph of period, T against length,  is plotted.
is 20.0 cm.
Tabulation of data
Table 1.7 ❺
❶ The third and
The row at the top Time for 20 oscillations, subsequent columns
of the table must Length of
t (s) Period, normally show
contain the name pendulum, values that are
T (s)
of the variable,  (cm) t1 t2 Mean, t20 derived from the
symbol, and unit 20.0 17.3 17.5 17.4 0.87 responding variable.
of measurement. These values are
30.0 21.9 22.4 22.2 1.11
used in plotting the
40.0 25.0 26.0 25.5 1.28 graph.

Manipulated variables 50.0 28.1 28.2 28.2 1.41
are filled in the first column. ❹
60.0 30.6 30.6 30.6 1.53 All numerical values
The values are fixed at
uniform intervals such as must be stated to the
10.0 cm, 20.0 cm, 30.0 cm, same number of

Experiment 1.1

and so on. The values must Responding variables are filled in the second column. decimal places. Values
also be in ascending or The values are obtained from measurements made during derived from measured
descending order. Avoid the experiment. Normally, this column is divided into values can be expressed
recording readings several columns when repeated measurements of the to the same number or
haphazardly. responding variable are taken. The average value is to an additional number
calculated and used in the calculations. of decimal places.

Introduction to Physics 24
Graph Conclusion
The graph of T against  shows a curve with a The period of oscillation increases when the length
positive gradient. of pendulum increases.
Graph of T against  The hypothesis is valid.

O
Figure 1.28
F
O

1
R

CHAPTER
M

4
Tabulating data
• The values of readings obtained in an experiment are known as data from the experiment.
• All data collected in an experiment must be orderly presented in tabular form to facilitate the investigation and
interpretation of the data.

Drawing Graphs

1 A correctly drawn graph can convey information in greater detail than a written explanation of
the experimental data.
2 A graph shows the relationship between two physical quantities. Figure 1.29 shows the
relationship between the period of oscillation, T of a pendulum and the length of the
pendulum, , as conducted in Experiment 1.1 by Hafiq.

T (s) Graph of T against


Title of the graph

1.6

Responding 1.4
variable on y-axis Correct plotting of
1
(name and unit 1.2 point (within 2
stated)
small square)
1.0

0.8
Draw a smooth curve
0.6 (with points balanced
for graph not passing
0.4 through all the points)

0.2
Experiment 1.1

O 10 20 30 40 50 60  (cm)
Manipulated
variable on x-axis
Suitable scales (name and unit
stated)

Figure 1.29

25 Introduction to Physics
Suitable Scales for Graphs

1 Suitable scales for the axes must be chosen to show the experimental data clearly and effectively.
A good graph has to be sufficiently large, filling about two thirds of the graph paper, and must
show special features, if any, of the graph.
2 Use a scale which can be read easily. Table 1.8 shows some examples of suitable scales.

Table 1.8
① Scale 2 cm : 1 unit ② Scale 2 cm : 2 units ③ Scale 2 cm : 5 units
or or or
multiples/submultiples. multiples/submultiples. multiples/submultiples.
F For example, For example, For example,
O
1

R
CHAPTER

0 0 1 2 0 2 2 4 4 0 5 5 5 10
0 0 1 10 20 0 200
200 400
400 0 50 5050 100
0 0 0 0.1 0.2 0 0.20.2 0.40.4 0 0.50.50.5 1.0

3 Avoid using odd scales such as:


2 cm : 3 units 5 The scales for the two axes need not be the
2 cm : 6 units same.
2 cm : 8 units For example,
For example, x-axis : 2 cm : 5 units
y-axis : 2 cm : 0.1 unit
y

0.1

x
0 3 6 5
0 6 12 x
0 8 16 5 10

Figure 1.30
Figure 1.31
4 Scale 2 cm : 4 units can be used but some
students do plot the points wrongly in using 6 The points plotted must cover the graph drawn
this scale. at least half of the graph paper, and not
concentrate on one section only, so that higher
accuracy can be obtained.

Introduction to Physics 26
Analysing Graphs

Common Graphs from Experiments

Table 1.9

1 p 2 p 3 p

F
q q q O

1
R

CHAPTER
M
p increases as q increases p decreases as q increases p increases linearly with q
4
4 p 5 p 6 p

q q 1
q

p decreases linearly with q p is directly proportional to q p is inversely proportional to q

15

Table 1.10 shows the results of an experiment to (c) What is the objective of taking two readings of the
investigate the relationship between the tensile force spring extension?
on a spring and its extension. (d) Are the data in each column written in the correct
way?
Table 1.10

Spring Solution
Mass of Tensile extension (a) Manipulated variable: Tensile force
weight force (cm) Responding variable: Spring extension
(g) (N) 1 2 Average (b) A ruler was used to measure the spring extension.
200.0 2.0 0.5 0.6 0.55 It has an accuracy of 0.1 cm.
400.0 4.0 1.1 0.9 1 (c) Two readings of the spring extension are taken,
the first after the weight is added and the second,
600.0 6.0 1.6 1.5 1.55 after the weight is removed. The repeated
800.0 8.0 2.0 1.9 1.95 measurements are to ensure the consistency and
1000.0 10.0 2.5 2.5 2.5 accuracy of the measurements.
(d) With the exception of the column for the average
spring extension, all other column data are written
(a) State the manipulated and responding variables in
in the correct way. The second and fifth data in the
this experiment.
column of average spring extension should rightly
(b) What instrument was used to measure the
be written as 1.00 and 2.50 respectively.
extension of the spring? What is its accuracy?

27 Introduction to Physics
16 (ii) when T = 1.3 s
Draw a horizontal
T 2 = 1.69 s2 line at T 2 = 1.69 s2,
A student carried out Experiment 1.1 and plotted a ∴  = 42.0 cm not at 1.3.
graph of T 2 against  as shown in Figure 1.32. (2.80 – 0.75) s2
(c) (i) m = –––––––––––––
T is the period of oscillation while  is the length of (70 – 19) cm
the pendulum. m = 0.04 s2 cm–1
T 2 (s2) 40 Normally write
Graph of T 2 against  (ii) g = –––
m the answer to
4 2 decimal
40 places and
=( –––––––––––––
0.04 s2 cm–1 don’t forget
about the unit.
F This point is selected because = 1000 cm s–2
O the line goes through the corner P
= 10.0 m s–2
1

of the small square.


R
(d) The oscillation should be in one plane (not a
CHAPTER

M 2.40 Need to circular oscillation).


Draw this draw
4 2 horizontal line. this Note: If the line goes through the origin, you can
1.69
triangle. take the original O as one of the points for the
triangle drawn to determine the gradient.
Same
comment Draw
Q
as P. this
R vertical
line.
 (cm)
0 10 20 30 40 50 60 70 80 17
Figure 1.32
The graph in Figure 1.33 shows the results of an
(a) State the relationship between T 2 and . experiment to investigate the relationship between
(b) From the graph, find the pressure and temperature of a gas. The gas
(i) the value of T, the period of oscillation when pressure is measured for temperature variations from
 is 60 cm, and 30 °C to 200 °C.
(ii) the length of the pendulum, , when T is 1.3 s. Pressure, P (N m–2)
Show on the graph how you determine the two
500
values.
(c) The acceleration due to gravity, g is given by the 400
formula below:
300
40
g = ––
m 200
2
where m is the gradient of the graph T against . 100
(i) Calculate the gradient, m.
Show how you determine m. 0 100 200 Temperature, T (°C)
40
(ii) Using the formula g = –– , calculate the
m
Figure 1.33
acceleration due to gravity, g.
(d) State one precaution that should be taken during (a) What is the gas pressure at temperatures of
this experiment. 0 °C and 250 °C?
(b) What is the temperature when the gas pressure is
Solution zero?
(a) T 2 is directly proportional to .
(b) From the graph, Draw the Solution
vertical By extrapolating (extending the graph beyond the
(i) when  = 60 cm dotted line at
T 2 = 2.40 s2  = 60 cm. experimental data) the graph to intercept the x and
∴ T = 1.55 s y-axes:

Introduction to Physics 28
Pressure, P (N m–2)
1.5
500
480 N m–2
400 1 Taylor arranges two similar springs with different
loads. He oscillates both loads and notices that the
300
heavy load makes one complete oscillation after the
200 250 N m–2 light one has completed two oscillations.
100
–270 °C
250 °C
–300 –200 –100 0 100 200 Temperature,
T (°C)

(a) Pressure at 0 °C = 250 N m–2


Pressure at 250 °C = 480 N m–2 F
O
(b) At zero pressure, temperature of gas = –270 °C

1
R

CHAPTER
M
Based on the above information, state
SPM
Clone
(a) one suitable inference, 4
4 ’06 (b) one appropriate hypothesis that could be
investigated,
The figure below shows two wooden tops P and Q, (c) the aim of the experiment to be carried out,
with P partly coated with tin. P and Q are of the same (d) the variables in the experiment,
size and are given the same initial rotational velocity. (e) the way you would tabulate your data,
tin (f) the expected graph, and
(g) your conclusion.
2 A student carried out an experiment to investigate
the relationship between the temperature of water
when heated, with its mass, m. He used the same
immersion heater for different masses with the
P Q same initial temperature θ0. However, the time of
heating was the same for all the masses. The final
(i) temperature θ of the water after being heated are
tin recorded.
The following figure shows the graph of θ against
1
–– obtained.
m
θ (°C) 1
Graph of θ against m
P Q 34

(ii) 33
Based on the above information:
(a) State one suitable inference. 32
(b) State one suitable hypothesis.
31
Comments
You will notice from the figures: 30
(i) top P is partly coated with tin, and thus possesses
more mass, and 29
(ii) top P still rotates while top Q has stopped.
28
Answer
(a) The time for the top to stop its rotation is affected 27
by its mass.
(b) The bigger the mass of the top, the longer it will 26 1
m (kg )
-1
O 0.1 0.2 0.3 0.4 0.5 0.6 0.7
rotate before stopping.

29 Introduction to Physics
(a) Extrapolate the graph and find its intercept on the (i) Draw a triangle on the graph and hence
θ-axis. The intercept gives the value of the initial determine the gradient, k, of the graph.
temperature θ0. What is the value of θ0? (ii) Calculate the value of the specific heat
(b) State the relationship between θ and m. capacity of water.
(c) The specific heat capacity, c, of water is given by (d) Determine the mass, m, of water used if the final
the equation: temperature is 31 °C. Show on the graph how
4.5 × 104 you determine the value of m.
c = –––––––
k (e) State one precaution for this experiment.
where k is the gradient of the graph.

F
O
1

R
CHAPTER

4 1. Physics is the knowledge of nature. It is a branch of 9. A parallax error is the error in reading a measurement
science concerning the study of natural phenomena, due to the position of the observer’s eye with respect
properties of matter and energy. to the scale of the measuring instrument is not
2. Physical quantities are quantities that can be perpendicular.
measured. 10. A zero error is the error when measurements do not
3. Base quantities are physical quantities that cannot be start from exactly zero.
defined in terms of other physical quantities. 11. A zero error can be corrected using the formula
4. The SI defines seven base units: metre (m), below:
kilogram (kg), second (s), kelvin (K), ampere (A),
mole (mol) and candela (Cd).
Actual measurement = Reading obtained – Zero error
5. Derived quantities are quantities derived from a
combination of two or more base quantities.
6. Prefixes and standard form are used to represent 12. Accuracy is the degree of how close a measurement
very large and very small numbers. A general form of a is to the actual value.
standard form is written as A × 10n, where 1  A  10 13. Consistency of a measuring instrument is its ability to
and n is an integer. get the same reading when a measurement is made
7. A scalar quantity is a physical quantity which has repeatedly.
magnitude only. 14. Sensitivity of a measuring instrument is its ability to
8. A vector quantity is a physical quantity which has detect small changes within a very short period of
both magnitude and direction. time.

1
Multiple-choice Questions

1.1 2 Modern physics consists of 1.2


Understanding Physics Base Quantities and
numerous disciplines (or branches)
Derived Quantities
1 Which field of Physics is of study. Which discipline deals with
concerned with the study of heat the spontaneous disintegration of 3 Which of the following is not an SI
and its effects on matter? radioisotopes? SPM
base quantity?
Clone

A Optics A Relativity ’06


A Length
B Mechanics B Astrophysics B Temperature
C Electromagnetism C Nuclear physics C Time
D Thermodynamics D Quantum mechanics D Weight

Introduction to Physics 30
4 The base SI unit of mass is the Which of the following lists of
SPM
A microgram
Clone B milligram C gram D kilogram capacities are arranged in
’03
ascending order?
5 Which of the following physical quantities is described correctly?
A 5  10–5 m3, 5 dm3, 500 cm3
Physical quantity Base or derived quantity SI unit B 500 cm3, 5 dm3, 5  10–5 m3
A C 5 dm3, 500 cm3, 5  10–5 m3
Temperature Base K
D 5  10–5 m3, 500 cm3, 5 dm3
B Volume Base m3
C Work Derived kg m s–2
D Momentum Derived kg m s–2
1.3 Scalar and Vector
6 A derived quantity can be B 1.034  10 6 Quantities
expressed as a combination of C 1.034  107 16 Which of the following lists of
base quantities. Which of the F
D 1.034  108 physical quantities consists only of
O
following is the combination of scalars?

1
base quantities for force? 12 Diagram 1 shows a box with R
A time, mass, velocity

CHAPTER
dimensions 6 cm  4 cm  M
mass B mass, velocity, acceleration
A 2 cm.
length  time C speed, volume, work 4
mass  length 2.0 cm D velocity, acceleration, force
B
time  time
17 Some equations are listed on a
4.0 cm
mass  length page of a physics book.
C 6.0 cm
time
mass  time Diagram 1
D mass
length  length What is the volume of the box, in density =
m3? volume
7 A useful method of expressing A 48 C 0.048 work = force  distance
very small or very large number is B 0.48 D 4.8  10–5 velocity
acceleration =
A Arabic numerals time
B Greek letters 13 The gravitational force, F, between
C significant figures
SPM
two objects of masses m1 and m2
Clone
’05 is given by the equation:
D scientific notation How many vector quantities are
Gm1m2
8 What does the prefix pico stand F= contained in these equations?
r2 A 1
SPM
for?
Clone
’03 A 10–12
where r is the distance between B 2
their centres and G is the universal C 3
SPM
Clone B 10–9
’07 C 109 gravitational constant. D 4
What is the unit of G?
D 1012
A N m kg –2
9 In which of the following sets are B N m2 kg –2
the prefixes arranged in ascending C m2 kg –2 1.4 Understanding
order? D N m–2 Measurements
A nano, mega, milli, kilo
14 The density of lead is 11.3 g cm–3 18 Diagram 2 shows a part of the
B mega, kilo, nano, milli SPM at room temperature. What is its scales on a pair of vernier callipers.
C nano, milli, kilo, mega
Clone
’08 value in SI unit?
D kilo, milli, mega, nano
A 1.13 × 10–5 kg m–3
10 Which of the following B 1.13 × 10–3 kg m–3 3 4
SPM
Clone measurements is the smallest? C 1.13 × 104 kg m–3
’09 A 4.7  104 m D 1.13 × 107 kg m–3
B 4.7  105 cm
15 The capacity of a container is the 0 5 10
C 4.7  107 mm
maximum volume of liquid it can Diagram 2
D 4.7  1011 µm
contain. The labels for three
11 In Tawau, Pop FM is being containers are given below. What is the correct reading?
SPM
Clone transmitted at the frequency of –5 3 3 A 2.90 cm
’06 113.5 MHz. What is the frequency 5  10 m 5 dm
B 3.03 cm
of the radio waves in Hz? C 3.15 cm
A 1.034  105 500 cm3
D 3.23 cm

31 Introduction to Physics
19 Diagram 3 shows the scale of a C
SPMmicrometer screw gauge.
Clone
’05
10
0 1 2
5
0
D
Diagram 3
P Q
What is the reading shown?
A 2.04 mm Diagram 4
B 2.05 mm
C 2.06 mm The period of an oscillation is the
D 2.40 mm 23 A, B, C and D shows parts of four time taken for the bob to move
SPM different galvanometers. from P to Q and back to P again.
F Clone
20 George has been asked to ’07 Which is the most sensitive? Using a stopwatch, how can a
O
student measure the period
1

R determine, as accurately as
possible, the volume of a piece of A accurately?
CHAPTER

M 0
wire. The wire is about 60 cm in A Time one oscillation carefully
0.2
4 length and about 0.3 cm in for a few times.
diameter. B Time ten oscillations and
0.4 divide by 10.
Which measuring instruments
should he use? C Time the motion from P to Q
B
0 and double it.
Length Diameter 0.4
A micrometer vernier 26 Diagram 5 shows a graph relating
callipers 0.8 SPM
Clone P and T.
’03
B metre rule vernier P
callipers C
0
0.5
C metre rule micrometer 1.0
b
D vernier micrometer 1.5
callipers 2.0
T
a O
21 A measuring instrument can D Diagram 5
0 1
detect a small change in the
2 The equation of the graph is
quantity to be measured. This
3 b
measuring instrument is A P= T+a
4 a
considered to be
A accurate b
B P= T+b
B consistent/precise a
C sensitive b
C P=– T+a
1.5 Scientific Investigation a
22 A, B, C and D shows the shooting
b
SPM
marks on a target. Which of the 24 An experiment was carried out to D P=– T+b
Clone
’05 following marks can be used to a
SPM
Clone investigate the relationship
SPM
Clone explain the concept of a ’08 between the period of a simple
’08
measurement which is consistent SPM
Clone pendulum and its length. A brass 27 Diagram 6 shows a graph of S
but not accurate? ’09 bob of 50 g was used in the
1
against .
A experiment. What is the T
manipulated variable?
S
A Amplitude
B Mass of brass bob
C Length of pendulum
B D Period of the simple
pendulum
1
T
25 Diagram 4 shows a swinging Diagram 6
pendulum.

Introduction to Physics 32
Given that the gradient of the 29 The graph in Diagram 7 shows the A M
graph is –g and the intercept on relationship between physical
y-axis is a, what is the equation of quantities X and Y.
the graph?
g X
A T = a – gs C T=a– N
S
g B M
B S = a – gT D S=a– SPM
Clone
T ’04

O Y
28 Which of the following graphs
shows that P increases with but is N
Diagram 7
not directly proportional to θ?
Which statement about the graph C M
A C
is correct?
P P F
A X increases if Y increases. O
B X is directly proportional to Y.

1
R
C The relationship between X 1

CHAPTER
N M
θ θ and Y is linear.
B D D The gradient of the graph D MN
4
P P decreases with increasing Y.
30 The following graphs show that
M is inversely proportional to N
θ θ N
except

Structured Questions
1 Diagrams 1(a) and (b) show the readings of the 2 A pair of vernier callipers is used to measure the
meters for an electrical circuit when the switch is external diameter of a water pipe.
closed. Diagram 2 shows the main and vernier scales of the
vernier callipers when the water pipe is lightly closed
2 3 by the jaws.
4
V
1

5
0

mirror 0 1 2 3

(a)
2 3
4 6
8
mA
2

Diagram 2
10
0

mirror
(a) Determine the vernier readings. [1 mark]
(b) Before the vernier callipers is used, the jaws are closed.
Diagram 1 Diagram 3 shows the main and vernier scales of the
(a) Write the readings of the voltmeter and of the vernier callipers with the jaws closed.
milliammeter.
(i) Voltmeter: _____________ V [1 mark] cm
0 1 2
(ii) Milliammeter: __________ mA [1 mark]
(b) Give an example of a systematic error that could
occur in the above measurements [1 mark]
(c) Some meters have a strip of mirror mounted 0 1
under the pointer and near the scale, as shown in
Diagrams 1(a) and (b). Suggest how this may
help to eliminate a possible source of error.
[1 mark] Diagram 3

33 Introduction to Physics
(b) Name this systematic error in the vernier callipers. 5 Diagram 6 shows a mercury thermometer.
[1 mark]
(c) This error can be taken into account by adding it -10 0 10 20 30 40 50 60 70 80 90 100 110
from the reading obtained. Determine the
bulb with thin bore
external diameter of the water pipe with the glass wall
correction. [2 marks]
Diagram 6
3 Diagram 4 shows a micrometer screw gauge used to
(a) What is the accuracy of the thermometer?
measure the diameter of a wire.
[1 mark]
lock (b) State the physical change in the mercury when
thimble X
anvil spindle the thermometer is placed in a cup of hot water.
0 5 30
25
[1 mark]
F wire
20
15 (c) Explain briefly why the bulb has a thin wall.
O [1 mark]
1

R (d) The sensitivity of the mercury thermometer


CHAPTER

M depends on the diameter of its bore. Underline


Diagram 4 the correct answer in the brackets to complete the
4 sentence below.
(a) (i) Name the part labelled X. [1 mark]
The sensitivity increases with (larger, smaller) bore
(ii) What is the function of X? [1 mark]
diameter. [1 mark]
(b) Four readings for the diameter were taken at
different places along the wire. The values are
tabulated in Table 1 below. 6 Four students each made a series of measurements
of the acceleration due to gravity, g. Table 2 shows the
Relative deviation results they obtained.
Diameter (mm)
(%)
3.47 3.52 3.49 3.50 0.43 Results: g (m s–2)
Student
Table 1 1 2 3 4
(i) Why is the diameter measured at four P 9.46 9.20 8.98 8.77
different places along the wire? [1 mark] Q 8.46 8.51 8.42 8.45
(ii) What is the purpose of calculating the R 9.82 10.09 9.87 8.93
relative deviation? [1 mark]
S 9.82 9.78 9.85 9.79
4 Diagram 5 shows the top part of a measuring cylinder Table 2
containing some mercury.
(a) If the actual value of g is 9.81, which student
R
obtained a set of results that could be described as
6
(i) precise and accurate? [1 mark]
P
S (ii) precise but not accurate? [1 mark]
Table 3 shows the smallest divisions of four types of
7 mercury measuring instruments.

Q
cm3 Instrument Smallest division
Diagram 5 Micrometer screw gauge 0.001 cm
(a) What is the physical quantity measured by the Vernier callipers 0.01 cm
measuring cylinder? [1 mark] Metre rule 0.1 cm
(b) Diagram 5 indicates four ways the observer’s eye Measuring tape 0.1 cm
could look when taking the reading from the
measuring cylinder. [1 mark] Table 3
Put a circle around the eye position which gives
the correct reading. (a) Name the instrument a student should use to
(c) What is the value of the smallest division on the measure
scale? [1 mark] (i) the diameter of a piece of wire, [1 mark]
(d) Why mercury is used, instead of other liquids, in (ii) the internal and external diameters of a
barometer? [1 mark] beaker used in the laboratory. [1 mark]

Introduction to Physics 34
Essay Questions
7 (a) Diagram 7 shows two ammeters, J and K, with (b) Using the different distributions of shooting marks
different sensitivities that can be used in electrical on a target, illustrate the differences between
circuit to measure the current. accuracy and consistency.
[4 marks]
2
(c) (i) Given a glass rod and a metre rule as shown
1 3
in Diagram 8, explain how a student can
0 4 determine the diameter of the wire.
A [6 marks]

metre rule

(a) Ammeter J F
wire
O

1
R
0.5

CHAPTER
M
0
glass rod
1

4
A
Diagram 8
(ii) Suggest a more suitable measuring
instrument which the student can find in the
(b) Ammeter K
laboratory to measure the diameter of the
Diagram 7 wire. [1 mark]
(i) What is meant by sensitivity? [1 mark] (iii) Which method will give you a more accurate
(ii) With reference to Diagram 7(a) and measurement? Explain your answer.
Diagram 7(b), determine the currents [1 mark]
measured by the ammeters. [2 marks] (iv) State the precautions that should be taken
(iii) Compare the sensitivities of ammeters J during the measurements using the
and K. Relate to the scales on the ammeters. instrument in (c)(ii).
[3 marks] [2 marks]

Experiment
1 A student carries out an experiment to determine the relationship between the length, , of a coil of wire wound round a
SPM rod and its number of turns, n. Vernier calipers are used to measure the length, , of the coil as shown in Diagram 1
Clone
’10

Vernier callipers

0 1 2 3
cm

0 5 10

rod

coil

Wire

Number of turns (n)

Diagram 1
The results of this experiment are shown in the graph of  against n in Diagram 2
The period of oscillation, T, of the simple pendulum is given by the following equation:

35 Introduction to Physics
(cm) Graph of againtst n
2.0

1.8

1.6

1.4

1.2

1.0
F
O
1

R 0.8
CHAPTER

M
0.6
4

0.4

0.2

0 n
0 5 10 15 20 25 30

Diagram 2 Graph of  against n


(a) Based on the graph in Diagram 2
(i) What will happen to  as n increases? [1 mark]
(ii) Determine the value of  when n = 12. Show on the graph, how you determine the value of  [2 marks]
(iii) Calculate the gradient, m, of the graph show on the graph how you determine m. [3 marks]
(b) The diameter, d, of the wire can be determined using the formula:
d = km
where, m = gradient of the graph
k = constant
what is the value of
(i) k?
(ii) d in SI unit [3 marks]
(c) This experiment is repeated by using a rod of bigger diameter.
(i) What happens to the gradient of the graph, m? [1 mark]
(ii) Give one reason for the answer in (c)(i) [1 mark]
(d) State one precaution that should be taken to improve the accuracy of the readings in the experiment. [1 mark]

COMPANION WEBSITE
Introduction to Physics 36 Online Tests
FORM 4

2
CHAPTER

Forces and Motion

2 Forces and Motion SPM Topical Analysis


Year 2007 2008 2009 2010 2011
Paper 1 2 3 1 2 3 1 2 3 1 2 3 1 2 3
Section A B C A B A B C A B A B C A B A B C A B A B C A B
Number of questions 8 1 – 1 – 1 7 1 1 – 1 – 8 1 – – – 1 9 1 – 1 – 1 9 1 1 – – –

ONCEPT MAP

FORCES AND MOTION

Kinematics Dynamics

Linear Motion graphs Inertia Momentum Elasticity


motion • s – t graph
• v – t graph
Mass Hooke’s
Impulse, Conservation law
Ft = mv – mu of F = kx
• Distance • Speed momentum
Newton’s first
• Displacement = Distance law of motion
Time Impulsive force,
• Velocity mv – mu Elastic potential
F= energy,
Displacement      t
= 1 2
Time Effects of a force E= kx
• Acceleration 2 
• Deceleration
v–u
a= t Newton’s second Forces in Newton’s third
law of motion equilibrium law of motion
F = ma
Acceleration due
to gravity, g Work, Energy
W = Fs
  Equations of
      motion
• v = u + at Gravity Potential Kinetic energy,
• s = 1– (u + v)t energy, 1
2 Ep = mgh Ek = 2 mv2
• s = ut + 1– at 2 Safety features
2 Weight, in the design of
• v2 =  u2 + 2as W = mg vehicles Power

COMPANION WEBSITE
Learning Objectives 37
2.1 Analysing Linear Motion

2.1 Analysing Linear Motion

Linear Motion

1 Linear motion is motion in a straight line. 3 Examples of non-linear motion:


2 Examples of linear motion: Earth

Sun


F
O (a)  A spinning top (b) The earth orbiting
2

R the sun
CHAPTER

M
(a) A passenger on a moving escalator Figure 2.2
4 4 The study of the motion of an object without
considering the forces acting on it is called
kinematics.
5 The study of the motion of an object and
(b)  An athlete running a 100 m race
F
the forces acting on the object is called
Figure 2.1 O dynamics.
2

R
CHAPTER

M
Distance, Displacement, Speed and Velocity
4

1 The physical quantities of motion are distance, displacement, speed, velocity, time and
acceleration.

Motion
An object in motion changes position.

Distance Displacement
• The total length of the path an object travels from • The distance an object travels in a specific
one location to another. direction.
•  Distance is a scalar quantity. • The magnitude of displacement is equal to the
shortest distance between two points.
•  Thus, displacement is a vector quantity.

Speed and velocity both describe how fast an object is moving but
there is an important difference between these two quantities,
i.e, velocity is related to direction.

Velocity
Speed • Velocity is the speed of an object in a specified
•  Speed is the distance covered per unit time, that is, direction, that is, the rate of change in
the rate of change in distance. displacement.
  Total distance travelled, s (m)
_________________________   Distance moved in a specific direction
______________________________________
Speed, v = Velocity, v =
Time taken, t (s) Time taken
•  Speed is a scalar quantity. ____________________
Displacement, s (m)
=
Time taken, t (s)
•  Velocity is a vector quantity.

Forces and Motion 38


2 Both distance and displacement have the same SI units.
They are measured in metres (m).
3 Both speed and velocity have the same SI units of metre per An object moves from position
second (m s–1). A to B. The figure below
4 However, since displacement and velocity are vector quantities, the shows five possible paths
direction of motion must be stated together with the magnitude. taken by the object. Which is
the shortest path?
Q

R B

1
P X
A
Figure 2.3 shows the location of two towns, P and Q.
Y F
N
lake O

2
The path with the shortest R

CHAPTER
Jamil's length is APB (the straight line M
Town Q
car that joins A and B) and APB is
Raven's 4
helicopter known as the displacement
Town P from A to B. Q
The magnitude of the
mountain displacement is the shortest R
Figure 2.3 distance that links the initial
and final positions of an
Jamil drives a car along the road from town P to town Q, which is 300 km away. object that has moved.
P
The journey takes 5 hours. Raven flies a helicopter due east from town P to Q A
The other paths, AQB, ARB,
for a distance of l00 km in half an hour. AXB and AYB are known as
Analyse, in terms of speed and velocity, Jamil's and Raven's journeys. the distances travelled from A
to B.
Solution
Jamil’s journey Raven’s journey
Distance travelled = 300 km Displacement = 100 km due east
Time taken = 5 hours Time taken = 0.5 hour
Distance travelled Distance in a
________________
Speed, v = Displacement
____________ specific direction.
Velocity, v =
Time taken Time taken
_______ 300 km _______ 100 km
= =
5h   0.5 h
= 60 km h–1 = 200 km h–1
The speed of the car is 60 km h–1. The velocity of the helicopter is 200 km h–1 due east.

Note:  SI unit is not used in this example.

Average Speed and Average Velocity


2
1 Refer to Example 2.
An athlete runs 100 m in 10 s.
2 The term average velocity is used because the
What is his average velocity?
athlete is not running at a constant velocity of
10 m s–1 in the whole race. Solution
3 Similarly, the speed of the car (in Example 1) is s

Average velocity, v = __
an average speed since the speed of the car is
t
not the same throughout the whole journey.
______ 100 m
4 Table 2.1 shows the difference between average =
speed/average velocity and constant speed/   10 s
constant velocity. = 10 m s–1

39 Forces and Motion


Table 2.1

Average speed / Average velocity Constant speed / Constant velocity


A car moves at an average speed/velocity of A car moves at a constant or uniform speed / velocity
20 m s–1 (equivalent to 72 km h–1). of 10 m s–1.
This means the car may move 15 m in the first This means the car moves 10 m in the first second,
second, 25 m in the next second and 20 m in the 10 m in the next second, 10 m in the third second,
third second. and so on.
On average, the car moves a distance/displacement The car always covers a distance or displacement
of 20 m in 1 second for the whole journey. of 10 m in 1 second for the whole journey.
The magnitude of speed/velocity remains the same.
15 m 25 m 20 m
F t=0 t=1s t=2s t=3s 10 m 10 m 10 m
O
2

R t=0 t=1s t=2s t=3s


CHAPTER

An object is said to be moving at a constant or uniform speed if it moves


equal distances in equal successive time intervals, no matter how small the
F
time interval is. O
2

For example, an object moving at Ra constant speed of 10 m s–1 covers a


distance of 10 m every second, or 1 M
m every 0.1 s.
CHAPTER

SPM
3 ’07/P1

In an activity for a Physics lesson, a student was (a) Total distance travelled = AB + BC
instructed by his teacher to run due north for = 12 + 16
a distance of 12 m before moving east for another = 28 m
16 m. The time taken was 20 s. _____________________ Total distance travelled
Average speed, v =
What was the student’s Time taken
(a) average speed, and 28 m
_____ =
(b)  average velocity?
 20 s
= 1.4 m s–1
Solution (b) Displacement
= The distance travelled in the direction of AC
16 m
B C = 122 + 162 Apply Pythagoras’ theorem
= 20 m
__
tan θ = 16 To determine the
direction of AC
12
= 1.333
12 m
tan θ = 53.1°
N Average velocity, v
Displacement
=
θ   Time taken
20 m
A =
 20 s
= 1 m s–1 in the direction N 53.1° E

Forces and Motion 40


4
Figure 2.4 shows a car moving round a roundabout. Solution
VR
VP (a) The speed of the car is 3 m s–1.
P (b) (i) At P:
R
The velocity, vP = 3 m s–1 due east
(ii) At Q:
The velocity, vQ = 3 m s–1 due west
N Q (iii) At R:
VQ The velocity, vR = 3 m s–1 due N 45° W

Figure 2.4 Note: F


If the car covers a distance of 3 m each second, find The speed remains constant throughout while the O

2
VR V
(a) the speed of the car, F4/2/3 velocity is changing (i.e. change in direction, but not
P
R
P

CHAPTER
(b) the velocity of the car at in magnitude, bearing in mind that velocity is a vector M
R
(i)  P        (ii) Q        (iii) R quantity which has both magnitude and direction.) 4

Acceleration and Deceleration N Q


VQ
1 When the velocity of an object changes with 3 Since velocity is a vector quantity (i.e., it has
time, the object is said to be accelerating. magnitude as well as direction), acceleration
2 Acceleration is defined as the rate of change is, thus, a vector quantity.
in velocity with time. F4/2/3
4 The SI unit for acceleration is metre per
Acceleration, a Initial velocity = u second per second or m s–2 (read as metre per
Change in velocity Final velocity = v second squared).
= Time taken = t
Time taken
Final velocity – Initial velocity
=
Time taken
     ∴
a = v – u
  t
v–u
v>u Acceleration, a = v<u
t

Acceleration Deceleration/Retardation

• When v > u, a is positive, the velocity is increasing. • When v < u, a is negative, the velocity is decreasing.
• Thus, the speed of the object increases and is said • Thus, the speed of the object slows down and is
to be accelerating. said to be decelerating.
For example, For example,
u=0 v = 30 m s–1 u = 30 m s–1 v=0
30 35 30 35 30 35 30 35
25 40 25 40 25 40 25 40
20 20 20 20
45 45 45 45
15 15 15 15
50 50 50 50
10 10 10 10
5 m s–1 55 5 m s–1 55 5 m s–1 55 5 m s–1 55
0 60 0 60 0 60 0 60

60 60 60 60
55 27
30
3
5 55 27
30
3
5 55 27
30
3
5 55 27
30
3
5
6 6 6 6

50
24

50
24

50
24

50
24

21 9 10 21 9 10 21 9 10 21 9 10
18 12 18 12 18 12 18 12
15 15 15 15

45 15 45 15 45 15 45 15

40 20 40 20 40 20 40 20
35 25 35 25 35 25 35 25
30 30 30 30

t=0 t=5s t=0 t=5s

The driver steps on the accelerator when the traffic The driver applies the brakes when he sees the
light turns green. The car increases its speed with traffic lights turn red to reduce its velocity with a
F4/2/4 F4/2/5
an acceleration of 6 m s–2. deceleration or retardation of 6 m s–2 until it stops.
Mathematically, we write as: a = 6 m s–2 Mathematically, we write as: a = –6 m s–2

41 Forces and Motion


5
From a constant velocity of 2000 m s–1, the velocity of Acceleration, a
a rocket increases to 3000 m s–1 in 5 s when the rate of v–u
=
combustion in the combustion chamber is increased. t
What is the acceleration of the rocket? 3000 m s–1 – 2000 m s –1
=
5s
Solution
1000 m s –1
= 200 metres per second per
5s second, i.e., a gain in velocity
200 m s –1 of 200 m s–1 in each second.
=
1s
u = 2000 m s–1 v = 3000 m s–1
F t=0 t=5s = 200 m s–2 Also read as 200 metres per
O second squared.
2

R
CHAPTER

The acceleration of the rocket in Example 5 is 200 m s–2. This means that its velocity increases by 200 m s–1
for every second, as illustrated in the following diagram.

F
t=0 t=1s Ot = 2 s t=3s t=4s t=5s
2

v = 2000 m s–1 v = 2200 m s–1 v = 2400 m s–1


R v = 2600 m s–1 v = 2800 m s–1 v = 3000 m s–1
CHAPTER

4 F4/2/9
6
Azmi cycles at a uniform speed of 20 m s–1. He then Solution
stops pedalling and his bicycle comes to a stop after u = 20 m s–1, v = 0 m s–1 and t = 8 s
8 s. What is his average deceleration? v – u
____
Acceleration, a =
u = 20 m s–1 v=0   t
_____ 0 – 20
=
  8 Negative means
= –2.5 m s–2 deceleration.
t=8s
stop pedalling bicycle stops Deceleration = 2.5 m s–2
Figure 2.5

change in displacement change in velocity


Displacement Velocity Acceleration
causes causes

Study of Linear Motion SPM


’03/P3/(B)

Ticker-timer

1 A ticker-timer is a device used in the laboratory to study the motion of a moving object,
usually a trolley.

Forces and Motion 42


2 The ticker-timer can be used to determine the (c) The velocity of the object
following variables. (d) The acceleration of the object
(a) The time interval of the motion (e) The type of motion of the object
(b) The displacement of the object

2 1
A vibrating metal strip with a pin is set to vibrate A ticker-timer consists of an electrical
up and down 50 times per second (i.e., at 50 hertz, vibrator, which is connected to an alternating
which is the frequency of the a.c. supply). current (a.c.) power supply (12 or 6 V).

1 vibrator
2 metal strip
F
50 dots are O

2
punched on the R
tape in one second

CHAPTER
M
3
ticker-tape 4
inclined runway
4 trolley
Figure 2.6
3 4
Each time the metal pin moves down, The ticker tape is attached to a trolley which moves on a
it makes a dot on the carbonised ticker F4/2/6
bench or runway. As the trolley moves, it pulls the tape
tape which passes underneath it. through the ticker-timer. A trail of dots is punched on
the ticker tape at equal time intervals. Thus, the dots on
the tape form a complete record of the motion of
the trolley.

Analysing Motion on a Single Strip

The type of motion of the trolley can be inferred from the distance between the dots on the ticker
tape pulled by the trolley.
direction of motion

The distance between two neighbouring dots is equal.  The object is moving at a constant/uniform velocity.
F4/2/14(a)

direction of motion direction of motion

The distance F4/2/14(b)


between two neighbouring The distance between
F4/2/14(c)two neighbouring dots is
dots is small. greater.
 The object is moving slowly.  The object is moving at a greater velocity.

direction of motion direction of motion

The distance between two neighbouring The distance between two neighbouring dots
F4/2/14(d) F4/2/14(e)
dots increases. decreases.
 The velocity increases.  The velocity decreases.
 The object is accelerating.  The object is decelerating.

43 Forces and Motion


To Find the Time Interval of Motion and Velocity of an Object SPM
’05/P3/(B)

1 The time for 1 dot-space, or 1 tick of time is the time interval between one carbon dot and
the next one on the ticker tape.
dot
number 0 1 2 3 4 5 678 910 20 30 40 50

direction
of one tentick
motion of time
First 10-tick Second 10-tick Third 10-tick Fourth 10-tick Fifth 10-tick
strip strip strip strip strip

Figure 2.7
F
O Since the vibrating pin makes 50 dots in 1 4 Therefore, one 10-tick of time
2

R second, therefore: = 10 × 0.02 s = 0.2 s


∴ 10-tick = 0.2 s
CHAPTER

M Time for moving 50 dot-spaces = 50 ticks = 1 s


1 5 Similarly, one 5-tick of time and one 2-tick of
4 ∴ 1 tick = s = 0.02 s
50 time can be calculated as shown in Table 2.2.
2 A 10-tick of time is the time interval from dot 6 With the quantities of time interval and
number 0 to dot number 10 on the tape. displacement, we can calculate the velocity of
3 The next 10-tick of time is the time taken to an object.
move from dot number 10 to Fdot number 20.
O
2

R Table 2.2
CHAPTER

M
10-tick strip Time taken to move from A to B Constant velocity,
A 4 = 10 × 0.02 s s 8 cm
direction of motion B v = =
1 2 3 4 5 6 7 8 9 10 = 0.2 s t 0.2 s
∴ 10-tick = 0.2 s = 40 cm s–1
8 cm
5-tick strip Time taken to move from P to Q Average velocity,
P direction of motion = 5 × 0.02 s s 6 cm
Q v = =
1 2 3 4 5 = 0.1 s t 0.1 s
6 cm
∴ 5-tick = 0.1 s = 60 cm s–1

2-tick strip Time taken to move from R to S Average velocity,


R = 2 × 0.02 s s 5 cm
direction of motion S v = =
1 2
= 0.04 s t 0.04 s
∴ 2-tick = 0.04 s = 125 cm s–1
5 cm

To Find the Uniform Acceleration or Deceleration of an Object

7
A trolley is moving down a runway. A strip with six The ticker-timer vibrates at a frequency of 50 Hz.
dot-spaces as shown in Figure 2.8 is obtained. Find the acceleration of the trolley.
Solution
1 2 3 4 5
direction
Frequency, f = 50 Hz
of motion A B P Q ∴ 1 tick = 0.02 s
0.5 cm
Average velocity from A to B: u = = 25 cm s–1
1.5 cm 0.02 s
0.5 cm
1.5 cm
Average velocity from P to Q: v = = 75 cm s–1
Figure 2.8 0.02 s

Forces and Motion 44


Time taken (from the midpoint of AB to the Change in velocity
________________
Acceleration, a =
midpoint of PQ) to produce the change in velocity    Time taken
= 5 × 0.02 s ____ v – u
= 0.1 s =
  t
Watch out! It is 5-tick, not 6-tick. The average
velocity happens somewhere at the midpoints (75 – 25) cm s–1
______________
=
of AB and PQ respectively. Therefore, the time     0.1 s
taken for the change in velocity is from the
midpoint of AB to the midpoint of PQ. ________ 50 cm s–1
=
  0.1 s
= 500 cm s–2
= 5 m s–2
Alternative method F
O

2
The time taken, t to produce the change in velocity can also be found as shown below. R

CHAPTER
First, label the time as an interval of 0.02 s at the dots as shown in the figure below. M

direction
of motion
0 0.02 s 0.04 s 0.06 s 0.08 s 0.10 s 0.12 s

t
0.01 s 0.11 s

∴ t = 0.11 s – 0.01 s = 0.1 s

Making a Tape Chart 2 On the tape, lines are drawn across dots
number 0, 10, 20 and so on, from the start of
1 Figure 2.9 shows a ticker tape obtained in an the first clear dot, to mark off in sections 10
experiment. dot-spaces long.
dot 3 The 10 dot-spaces are labelled in order. The tape
number 01 2 3 4 5 6 7 8 9 10 20 30
is then cut at the lines to form 10-tick strips.
1 2 3
direction 4 The 10-tick strips are pasted in order side by
of motion one tentick
of time side on paper, preferably a graph paper, for
easy measurement, to form a tape chart as
Figure 2.9
shown in Figure 2.10.
Velocity, v (or distance moved per tentick) v

8
7
6
5
4
3
2
1

Time in ticks t
0 10 20 30 40 50 60 70 80
0 0.2 0.4 0.6 0.8 1.0 1.2 1.4 1.6 Time in seconds

Figure 2.10
5 The length of each 10-tick strip is the distance the time taken for each strip is the same, i.e.
moved in one 10-tick of time, that is, 0.2 s. 0.2 s.
6 The length of the strip represents velocity. The 7 As such, the tape chart is practically a velocity-
longer the strip, the greater the velocity since time graph where the vertical axis is the

45 Forces and Motion


velocity (distance moved per 10-tick), while length actually represents the velocity since the
the horizontal axis indicates the time since time taken for each strip is the same.
each strip starts 0.2 s after the one before. 9 Tape charts can also be made of strips with
8 Some may prefer to label the vertical axis as 5 dot-spaces or 2 dot-spaces.
length of the strips, but remember that this

Activ To determine displacement, velocity and acceleration of a trolley


ity 2.1
Apparatus/Materials Calculation
Trolley, runway, ticker-timer, 12 V power supply, (a) Displacement = x1 + x2 + x3 + x4 + x5 + x6
F ticker tape, cellophane tape and wooden block.
O where x is the length of the strip.
(b) Time taken = 6 × 10-tick
2

R Arrangement of apparatus
= 6 × 0.2 s
CHAPTER

M
ticker tape ticker-timer
= 1.2 s
4 trolley
Displacement
12 V a.c. inclined Average velocity = ____________
power runway
Time taken
supply wooden
(x1 + x2 + x3 + x4 + x5 + x6) cm
= _________________________
block

Figure 2.11 1.2 s
F
Procedure O (c) Average velocity for the 1st strip:
2

R x1
___
1 The apparatus is set up as shown
M in Figure 2.11. u= cm s–1
CHAPTER

0.2
2 The inclination of the runway is set so that the
4 it is released.
trolley will roll down freely after Average velocity for the 6th strip:
3 A length of ticker tape is passed through the x6
v = ___ cm s–1
ticker-timer and attached to the trolley. 0.2
4 The ticker-timer is switched on and the trolley is
released. ime taken for the change in velocity, t
T
5 The ticker tape obtained is then cut into 6 pieces = (6 – 1) × 0.2 s
of 10-tick strips. = 5 × 0.2 s
6 The strips are pasted side by side on a graph = 1.0 s
paper to form a tape chart.
Results Acceleration of the trolley is calculated from the
velocity (distance moved per tentick) formula:
v–u
5 a=
x6
   t
6
4
x5
5
Discussion
3
The length of strip increases uniformly.
x4
2
4
Thus, the trolley moves down the runway at a
x3 constant or uniform acceleration.
3
1
x2
2 Conclusion
x1 1
The average velocity and acceleration of a trolley are
thus determined.
Activity 2.1

0 10 20 30 40 50 60 Time in ticks
0 0.2 0.4 0.6 0.8 1.0 1.2 Time in seconds
t

Figure 2.12
F4/2/10

Forces and Motion 46


To prepare a friction-compensated runway
In Activity 2.1, there are two main forces acting on the trolley along the runway, i.e., the component weight of the trolley
down the runway and the frictional force of the runway on the trolley (please refer to Section 2.9 on page 118).
(a) If the slope is not steep enough, (b) If the slope is too steep, the trolley (c) The slope is adjusted until the trolley
after a slight push, the trolley moves moves down the runway by itself. goes down at uniform velocity, after
a short distance and then stops. The dots get farther apart. a slight push. The dots on the tape
The dots on the tape get closer. are equally-spaced. This is a
friction-compensated runway.
Physics Blog
direction of motion
direction of motion
direction of motion
trolley given a slight push
stop trolley goes down on its own F
trolley given a slight push
velo O

2
city cons
Key : incr tant v R
eas elocit
ing y

CHAPTER
component weight friction M

8
F4/2/11b F4/2/11c
A student carried out an experiment using a trolley Average velocity
and a ticker-timer that vibrates at a frequency of ____________________
Total distance travelled
=
50 Hz. Figure 2.13 shows a tape chart consisting of­     Time taken
10-tick strips that he obtained. 30 cm
______
= =  30 cm s–1
velocity (distance moved per tentick)
1.0 s   
4 = 0.30 m s–2
10 (c) Average velocity for the 1st strip:
3 Key :
____ 2 cm
start u = =  10 cm s–1
component
weight
8 0.2 s 
2 trolley given a slight push friction
6
Average velocity for the 5th strip:
1 ______
stop 10 cm
v = =  50 cm s–1 Watch out, not 5 × 10-tick!
4  0.2 s  The time taken for the
2
Time taken, t change in velocity is from
= 4 × 10-tick the midpoint of the 1st strip
to the midpoint of the 5th
0 = 4 × 0.2 s =  0.8 s strip.
0 10 20 30 40 50 Time in ticks
0 0.2 0.4 0.6 0.8 1.0 Time in seconds OR:
t Time taken, t From the time axis.
0.1 0.9
=  (0.9 – 0.1) s =  0.8 s
Figure 2.13 Change in velocity
_________________
∴ Acceleration, a =
Find F4/2/12    Time taken
(a) the total distance travelled,

____ v – u
(b) the average velocity, =
(c) the acceleration of the trolley.   t

______________ (50 – 10) cm s–1
Solution =
     0.8 s
(a) The total distance travelled = 50 cm s–2 = 0.5 m s–2
(from dot number 0 to number 50)
Note: Since the motion is of uniform acceleration, the
= (2 + 4 + 6 + 8 + 10) cm
average velocity in (b) can also be calculated
= 30 cm u+v
(b) Total time taken from the formula –v  = .
= 5 × 10-tick
The total time taken   2
is 1 s if you label the –1
= 5 × 0.2 s time axis in seconds. –v  = (10 + 50) cm s  = 30 cm s–1
= 1.0 s   2

47 Forces and Motion


In the questions, the vertical axis may not be labelled as As explained earlier, the distance between dots, or the
velocity or distance moved per 10-tick (or 5-tick or length of a strip, represents the velocity of a moving
2-tick). However, the way to solve the problems is still object. Therefore, the increase in distance between dots
the same. and the increase in length between successive strips
represent the increase in velocity. If the increment is
uniform, then the increase in velocity is uniform, that is,
the object is moving with uniform acceleration.
(a) In the figure below, the increase in distance
9 between the successive dots is equal (i.e., 0.2 cm).
Therefore, the acceleration is uniform.
Figure 2.14 shows a tape chart for a trolley which
F
O
moves up on an inclined plane. Determine the direction of motion
2

R deceleration of the trolley.


1.0 cm 1.2 cm 1.4 cm 1.6 cm 1.8 cm 2.0 cm
CHAPTER

M Length of strips (cm)


1
4 2 (b) In the figure below, the increase in length between
6 3 the successive strips is equal (i.e., 1 cm). Therefore,
5 4 the acceleration is uniform. (Alternatively, if a
4 5 straight line can be drawn across all the top
3 midpoints of the strips, the acceleration is uniform)
2 F
Velocity (cm per tentick) Velocity
1
O
2

R 8
7
CHAPTER

0 M Time (s)
0.04 0.08 0.12 0.16 0.20 0.24 6
t 5
4 4
0.02 0.22
3
Figure 2.14 2
1
Time (s)
Solution Time
0 0.2 0.4 0.6 0.8 1.0 1.2
The time of each 2-tick strip = 2 × 0.02 s
= 0.04 s (c) In the figure below, there is no increase in length
6 cm between the successive strips. Therefore, the
u =    =  150 cm s–1 acceleration is zero (the object is moving with
0.04 s
constant or uniform velocity).
1 cm
v  =    =  25 cm s–1
0.04 s Velocity (cm per tentick) Velocity
Time taken to produce the change in velocity, t 6
= 5 × 0.04 s
= 0.20 s = 5 × 2-tick
Time(s)
OR: t = (0.22 – 0.02) s Time
If the time axis is 0 0.2 0.4 0.6 0.8 1.0
= 0.20 s labelled.
Acceleration, a
(d) In the figure below, the decrease in length between
v – u
____ the successive strips is equalVelocity
(i.e., 2(cm
cm). Therefore,
= per tentick)
  t the deceleration is uniform.
(25 – 150) cm s–1 8
= 0.2 s Velocity (cm per 5-tick) 7
Velocity
16 6
–125 cm s–1 14
= 0.2 s 12 Velocity (cm per tentick) 5
Veloc
10
8 4
= – 625 cm s–2 6
3
= – 6.25 m s–2 4
2
6
Time(s) 2
∴ Deceleration = 6.25 m s–2 0 Time
0.1 0.2 0.3 0.4 1
Time (
0 0.2 0.4 0.6 0.8 1.0 1.2
0 0.2 0.4 0.6 0.8 1.0 Time
Forces and Motion 48
F4/2/34
The Equations of Linear Motion

For linear motion with uniform acceleration, a, the displacement or Summary of the equations for
distance travelled, s, in a particular direction is given by: linear motion with uniform
s = Average velocity × Time taken acceleration:
 v = u + at
∴ s = 1 (u + v)t 1
 s = (u + v) t
  2 2
1 2
 s = ut + at
2
rearrange rearrange
v = u + at a = v – u t = v – u  v 2 = u2 + 2as
 t  a
substitute substitute s = displacement F
u = initial velocity O

2
v = final velocity R
1 1 a = uniform acceleration
( )

CHAPTER
s = (u + v)t 1 s = (u + v) t M
2 s = u + v t 2 t = time interval
1 2
= (u + u + at)t 1 (v – u) 4
2 = (u + v) a
2
1
= (2ut + at 2) (v 2 – u2)
2 = 1
2 a
1 2 2 2
s = ut + at 2as = v – u
2
v 2 = u 2 + 2as

There are 5 physical quantities in the equations of 1


linear motion with uniform acceleration. (II) For s = (u + v) t:
2
Each of the four equations of motion involves 4 physical
quantities. The velocity of a rocket travelling at 2000 m s–1
To solve numerical problems, you need to know 3 increases to 6000 m s–1 after moves through a
quantities before you can find the value of the fourth distance of 80 km. Calculate the time for the rocket
quantity. In the beginning, you may be unsure of which to reach this velocity.
equation to choose to solve the problem. You may start 2000 m s–1 6000 m s–1
by trial and error. After some practice, you will be able to
select the required equation easily.
80 km
The table below shows some examples.

(I) For v = u + at: Solution


A car accelerates from 20 m s–1 with an acceleration u = 2000 m s–1, v = 6000 m s–1,
of 2 m s–2. s = 80 km = 80 000 m, t = ?
What is the velocity after 8 seconds? Unable to
 ? ×? calculate t
Solution 1 because only
First trial: s = ut + at2 2 quantities
2
u = 20 m s–1, a = 2 m s–2, t = 8 s, v = ? are known.

Unable to   ?
?  × 1 3 quantities
calculate v, Second trial: s = (u + v) t
First trial: v 2 = u 2 + 2as because only 2 are known.
2 quantities are Can be used
(2000 + 6000) to find t.
?   known. 80 000 = t
2
Second trial: v = u + at
v = 20 + 2(8) t = 20 s
3 quantities are
= 36 m s–1 known. Can be
used to find v.

49 Forces and Motion


10 12
Starting from rest, a sprinter reaches his top velocity By applying the brakes, a driver reduces the
in 3 seconds. He runs a distance of 24 m in the velocity of his car from 20 m s–1 to 10 m s–1 after
3 seconds. What is his acceleration? a distance of 30 m. Calculate the deceleration of
(Assume his acceleration is uniform.) the car.
Solution
u = 0, s = 24 m, t = 3 s, a = ? Solution
1 u = 20 m s–1, v = 10 m s–1, s = 30 m, a = ?
Applying s = ut + at 2:
2 Applying v 2 = u2 + 2as:
1 102 = 202 + 2a(30)
F 24 = 0(3) + a(3)2
O 2 100 – 400
a =
2

R 2(30)
a = 2 × 24
CHAPTER

M = –5 m s–2
9
Deceleration = 5 m s–2
4 = 5.3 m s–2

F 1 SPM
Clone
’07
O
2

11 R The figure below shows the route taken by a van


CHAPTER

M
from town P to town S.
Salina is driving at a velocity of 10 m s–1.
Seeing a cow in front, she brakes 4 to stop her car. 3 km
Q R
If the deceleration of the car is 2 m s–2, what is 1 km
the distance the car covers before it comes to a S
halt?
5 km

Solution
P
u = 10 m s–1 v=0

What is the displacement of the van?


F4/2/14a
A 4.0 km C 7.5 km
s=?
B 5.0 km D 9.0 km
Comments
u = 10 m s–1, v = 0, a = –2F4/2/13
m s–2, s = ?
2 2 Displacement is a vector quantity. It is the shortest
Applying v = u + 2as:
distance that links the initial position of an object
0 = 102 + 2(–2)s
to its final position. In the above question, the
4s = 100
100 displacement is the distance PS. Join P to S, and use
s = Pythagoras’ theorem to calculate the displacement.
4
= 25 m Q 3 km R
1 km 3 km 1 km
S

4 km
PS = 3 2 + 4 2
= 5 km

P
Don’t forget to insert the negative sign for deceleration
when doing the calculation. Answer  B
F4/2/14b

Forces and Motion 50


2.1

1 During a class activity, Hashim walks 20 m due east. 6 When a dart is blown from a blowpipe of 1.2 m
He reverses his direction and walks 12 m. After that, length, it travels at a speed of 15 m s–1. Find the
he reverses his direction again and walk for another time taken for the dart to travel in the barrel.
10 m. If the total time taken is 30 s, what is his
7 Daniel drives his car at a constant velocity of
speed and velocity?
20 m s–1. He steps on his brakes to reduce the
2 A speedboat moves due north for 12 km before velocity of the car to 10 m s–1 after travelling 30 m.
turning east for 8 km. Later, the speedboat moves Calculate the deceleration and the further distance
south for 6 km. What is the displacement of covered before the car stops.
the speedboat from its original position? Find its
average velocity, in m s–1, if the total time taken is 8 An aeroplane needs a velocity of 33 m s–1 for
30 minutes. take-off. F
3 Find the acceleration or deceleration for the tape O

2
v = 33 m s–1
R
charts below. The ticker-timer in use vibrates at a

CHAPTER
M
frequency of 50 Hz.
u=0
(a) Velocity (cm per 10 ticks) 4

s
10

If the plane accelerates at 3 m s–2, calculate the


minimum length of the runway needed by the plane
2 to reach the speed for take-off.
Time
9 Velocity
A baseball pitcher
(cm per holds a baseball in his hand for
10 ticks)
3 m before the baseball is thrown at a velocity of
(b) Length of strips (cm) 39 m s–1.
10
12
3.0 m
10
8
6
4 39 m s–1
2
Time
2
Time

4 A sports car accelerates from rest and covers a


distance of 90 m in 6 s. What is its acceleration?
5 A cyclist starts from rest and reaches a velocity of Find the acceleration of the baseball before it is
20 m s–1 in 8 s. Calculate his acceleration. released.

2.2 Analysing Motion Graphs

2.2 Analysing Motion Graphs

Motion Graphs

1 Graphs can be useful in studying motion. They show the changes in


the motion of an object with time.
2 There are two main types of linear motion graphs:
(a) the displacement-time graph
(b) the velocity-time graph

51 Forces and Motion


Displacement-time Graphs

1 A displacement-time graph (s-t graph) is a graph that shows how the


Relationship between the
displacement of an object varies with time.
position of an object,
2 Figure 2.15 shows a student cycles at a constant velocity from position
displacement, s, and velocity, v:
A to reach position B, which is 300 m away, in 200 seconds. He rests
s = (+)
for 100 seconds at position B and then cycles back to position A using
v = (+)
the same straight path. He reaches position A after another 200
(object on the
seconds. right of O and
N
zero velocity — at rest moving
displacement (m)
due east)
positive velocity O
F — moving in a fixed direction
negative velocity
O — moving in opposite direction s=0 v=0
B B
2

300 and stops at


R s = (–) s = (+)
the starting point
CHAPTER

M v = (–) (object on the


(object right with
4 ∆y
on the respect to O)
I II III left of
A ∆x A O and v = (–)
time (s) moving (object moving
O 100 200 300 400 500
due due west)
F
    Figure 2.15 west)
O
F4/2/15
2

R
CHAPTER

4 • In Section III of the graph:


From conclusion 1:
• In Section I of the graph: Velocity = Gradient
To find the velocity, use the formula: (0 – 300) m
Change in displacement =
Velocity = (500 – 300) s
Time taken
= – 1.5 m s–1
300 m
Velocity, v = The negative sign shows that the direction of
200 s motion is opposite to its original direction.
= 1.5 m s–1 Take note that velocity is a vector quantity.
Gradient of the graph • At t = 500 s, the graph intersects the t-axis.
∆y The displacement at this moment is zero, that is,
= the student has returned to the original position.
∆x
(300 – 0) m
=
(200 – 0) s
= 1.5 m s–1 • In Section II of the graph, a horizontal line is drawn from
t = 200 s to t = 300 s. During this period, the student
remained at position B, which is 300 m away from position
A, the origin, with a velocity of zero.

Conclusion 1 Conclusion 2
On a displacement-time On a displacement-time graph,
graph, the gradient of the a horizontal line (gradient = 0)
graph is equal to the shows that an object is
velocity of the object. stationary, i.e., not in motion.

Forces and Motion 52


Velocity-time Graphs SPM
’03/P1
SPM
’04/P1
SPM
’05/P1
SPM
’07/P1
SPM
’08/P1
SPM
’09/P1

1 A velocity-time graph (v-t graph) is a graph that shows the variance in the velocity of an object against
time.
2 A car starts from rest and accelerates for 20 seconds until it reaches a velocity of 30 m s–1. The driver
maintains this velocity for 20 seconds. The velocity of the car is then reduced until it stops at t = 60 seconds.
3 The graph in Figure 2.16 shows how the velocity of the car changes against time.
v = 0 m s-1 v = 30 m s-1 v = 30 m s-1 v = 0 m s-1

t=0 t = 20 s t = 40 s t = 60 s
starts stops
v (m s-1)

F
constant velocity O

2
30 To determine the acceleration of the car R

CHAPTER
• From conclusion 1: M
acceleration deceleration Acceleration, a
(positive ∆y ∆y (negative 4
I II III = Gradient
gradient) gradient)
(0 – 30) m s–1
= (60 – 40) s
∆x ∆x
t (s) The negative
0 20 40 60
= –1.5 m s–2 sign indicates
∴ The deceleration of deceleration.
                      FigureF4/2/17
2.16 the car is 1.5 m s–2.

To determine the acceleration of the car


• Using the formula:
Change in velocity
Acceleration =  Conclusion 3
Time taken
v–u A negative gradient indicates deceleration.
Acceleration, a =
t
(30 – 0) m s–1
= 20 s • Distance travelled from t = 20 s to t = 40 s:
Distance = Velocity × Time
= 1.5 m s–2 = 30 m s–1 × 20 s
∆y = 600 m
• Gradient of the graph =
∆x • Area under the graph (for section II)
(30 – 0) m s–1 = 30 × 20
= = 600 unit2
(20 – 0) s
1.5 m s–2
=

Conclusion 4
Conclusion 1
On a velocity-time graph, the area under
On a velocity-time graph, the gradient of the the graph is numerically equal to the distance
graph represents the acceleration of the object. travelled.

The car travels at a constant velocity of 60 m s–1


from t = 20 s to t = 40 s. A horizontal line is
shown in the graph. This method of calculating the distance travelled
can be applied to any velocity-time graph,
whether the velocity is constant or not.
For example, in Section I of the graph, the area
1
Conclusion 2 of the shaded triangle (= × base × height)
2
On a velocity-time graph, a horizontal line equals 300. So, the car travelled a distance of
(gradient = 0) represents a constant velocity. 300 metres in the first 20 second of its motion.

53 Forces and Motion


13
Figure 2.17 shows the velocity-time graph of a Solution
motorcycle travelling along a straight road between (a) The line OA shows that the motorcycle accelerates
two traffic lights. uniformly from 0 m s–1 to 12 m s–1.
The line BC shows that the motorcycle decelerates
uniformly before coming to a rest.
velocity (m s–1)
(b) The motorcycle is moving at a constant velocity
for 10 s (line AB of the graph).
(c) Distance between the two traffic lights
A B = Area under the graph
12 –
1
F = (10 + 25) × 12 = 210 m
O 2
2

R (d) From 0 s to 5 s:
CHAPTER

M
12 – 0
C Acceleration = = 2.4 m s–2
4 time (s) 5–0
O 5 10 15 20 25
From 5 s to 15 s:
Figure 2.17
Acceleration = 0 because the
object is moving
(m From
velocity s–1) 15 s to 25 s: at constant
(a) Explain the motion of the F motorcycle as
Acceleration velocity
represented by the lines OAO and BC on the – A
­ 0B– 12
12 = = – 1.2 m s–2
2

graph. R
25 – 15
CHAPTER

(b) What is the time interval Mduring which the Thus, the acceleration-time graph is as shown below.
motorcycle is moving at a constant velocity?
4 C –2
(c) What is the distance between the two traffic O 5 10 acceleration 15 20 25
(m s time
) (s)

lights? 2.4
(d) Sketch an acceleration-time graph to represent
time (s)
the motion of the motorcycle between the two
–1.2
traffic lights.

Non-uniform Velocity

displacement, s (m) 1 Figures 2.18(a) and (b) show the


positions of a ball falling from rest
50
55
24
27
60
30
3

6
5
10
and its corresponding displacement-
t=0 time graph.
21 9

18 12
15

45 15

40 20
35 25
30

s1 2 The ball covers a longer distance in


55
60
5
gradient is steeper the second second as compared with
⇒ greater velocity
30
27 3

50
24

10

t=1s the first second. The ball is moving


21 9

18 12
15

45 15

40
35
30
25
20
gradient
s2
= velocity with non-uniform velocity.
(at t = 1 s)
3 To determine the ball’s instantaneous
velocity, for example, at t = 1 s or
Δs t = 2 s, a tangent must be drawn at the
50
55
24
27
60
30
3

6
5
10
Δt
respective points on the graph, as
t=2s
21 9

shown in Figure 2.18(b).


18 12
15

45 15

time, t (s)
40 20
35 25
30

O 1 2
4 The gradient of the tangent is equal
(a) (b) to the velocity.
5 However, the stone is falling with
                          Figure 2.18
F4/2/18 uniform acceleration.

Forces and Motion 54


Non-uniform Acceleration

acceleration constant velocity


1 Figure 2.19 shows an
athlete running a 100 m
track and the corresponding
velocity-time graph of his
motion.
2 The athlete increases his
velocity, v (m s-1)
constant velocity velocity (accelerates) until
the maximum velocity.
smaller gradient gradient = 0 He maintains the
smaller acceleration zero acceleration maximum velocity to
F
finish the race. O

2
gradient of tangent
3 The acceleration of the R
athlete can be determined

CHAPTER
Δv = acceleration at this instant M
by drawing tangents at the
Δt 4
respective points and
time, t (s) determining the gradients
O t1 t2
Figure 2.19
of the tangents.

F4/2/19

A comparison between the displacement-time graph and the velocity-time graph:

Displacement-time graph Velocity-time graph


displacement
velocity

A
time time

The object moves at a Non-horizontal


The object moves at a constant acceleration.
constant velocity. straight line

Represents the velocity of


Gradient Represents the acceleration of the object.
the object.

The object is stationary. Horizontal line The object moves at a constant velocity.

The object returns to its Intersection on


The object stops.
original position. the time-axis

Positive ⇒ The object moves


in a specific Sign of the Positive ⇒ Acceleration
direction. gradient Negative ⇒ Deceleration
Negative ⇒ The object moves (positive or
in the opposite negative)
direction.

Area under the Numerically equals the distance travelled by the


No significance
graph object

55 Forces and Motion


14
The velocity-time graph in Figure 2.20 shows a particle (a) 10 s (from t = 0 s to t = 10 s, the velocity is
starting from rest and travelling east. positive).
v ( m s –1 )
(b) 4 s (from t = 16 s to t = 20 s, the velocity is
negative).
B
20 (c) Distance travelled while moving towards the
east, S1 = Area of triangle ABC
10
1
C P R = × 10 × 20
A t ( s) 2 
5 10 16 18 20
= 100 m
–10
F Q Distance travelled while moving towards the west,
O S2 = Area of triangle PQR
2

R
Figure 2.20 1 = × 4 × 10
CHAPTER

M 2 
(a) How long does the particle travel towards the east?
4
= 20 m
(b) How long does the particle travel towards the west?
(c) Find the average speed and the average velocity. Total distance
Average speed =
Solution Time taken
100 + 20
To understand the above situation better, please refer =
F
to the figure below which demonstrates the positions 20
O
6 m s–1
=
2

of the particle that vary with time. R


CHAPTER

100 m M Final displacement


v=0 v = 20 m s–1 v=0 Average velocity =
A B 4 C Time taken
t=0s t=5s t = 10 s
100 – 20
=
N
v=0 v = –10 v=0 20
R Q P = 4 m s–1 to the east
t = 20 s t = 18 s t = 16 s
(Please note that in the situation discussed, the
20 m
particle is always to the east of its original
Note:  The particle is at rest from t = 10 s to t = 16 s. position.)
F4/2/21a

Area under velocity-time graph


v
An object moving at a decreasing acceleration is
moving at a decreasing velocity.

A
t
B
The velocity of an object with a decreasing acceleration
is always increasing. However, the rate of increase is
Area A (above the t-axis) getting smaller.
= Distance travelled by the moving object in its original
direction
Area B (below the t-axis)
=Distance travelled by the moving object in the
opposite direction
Total distance travelled = Area A + Area B
Final displacement = Area A – Area B

Forces and Motion 56


2 SPM
Clone
’11

The diagram shows the velocity-time graph of a toy A 0 m C 9m


car in motion. B 6 m D 15 m
velocity(m s-1) Comments
6 Displacement = Area above the time-axis – Area below
4 the time-
2 axis
0 time (s)
1 1
= (6)(2 + 3) – (2)(6)
-2 2 2
-4
= 15 m – 6 m F
= 9 m O

2
-6
R

CHAPTER
M
What is the displacement of the toy car in 6 s? Answer:  C
4

2.2
s (m)
1 The figure 3 The figure below shows the velocity-time graph of a
shows the 20
motorcycle starting from rest and travelling north.
displacement-
15
time graph v (m s–1)
10
of a moving
5 12
particle.
t (s)
O 3 10 15

15 20
(a) What is the velocity of the particle in the initial O t (s)
6 10 13
period of 3 seconds?
(b) How long is the particle stationary?
(c) At what point in time does the particle return to –10
its original position?
(d) Calculate (a) What is the deceleration from t = 10 s to t = 13 s?
(i) the average speed, and (b) What is the displacement of the motorcycle
(ii) the average velocity of the moving particle. during the first 13 s?
(c) For how long is the motorcycle travelling towards
2 The the south?
v (m s–1)
velocity- (d) What is the final displacement of the motorcycle
time graph at t = 20 s?
shows the 12 (e) What is the average velocity of the motorcycle for
movement the whole journey?
of a particle. 4 A sports car starting from rest, accelerates uniformly
t (s)
to 30 m s–1 over a period of 20 s. The car maintains
O 3 6 10 the velocity for 30 s. The velocity is then reduced
(a) What is the total distance, in m, travelled by the uniformly to 20 m s–1 in 10 s and then brought to
particle in 10 seconds? rest after another further 10 s.
(b) For how long is the particle moving with constant Draw a velocity-time graph to represent the journey
velocity? as described above. From the graph, find
(c) Calculate the ratio of acceleration : deceleration. (a) the acceleration of the car for the first 20 s,
(d) Calculate the average velocity of the particle. (b) the distance travelled, and
(c) the average velocity over the time described.

57 Forces and Motion


2.3 Understanding Inertia

2.3 Understanding Inertia


SPM SPM
Concept of Inertia ’05/P1 ’09/P1

1 All objects tend to continue with what they are doing.


2 Newton's first law of motion (also known as the Law of Inertia)
states that:

Every object continues in its state of rest or uniform speed in


a straight line unless acted upon by an external force.

F 3 The tendency of an object to maintain its state of rest or uniform


O motion in a straight line is called inertia.
2

R 4 Only an external force (or a non-zero net force) can cause a change to
CHAPTER

M
the state of motion of an object (either at rest or moving at the same
4 speed in a straight line).
5 The external force is a non-zero net force, if more than one external
force act on the object.
SPM
Situations Involving Inertia ’09/P1

F
O
1 When a boy is riding a bicycle that runs over a stone, he is
2

R
thrown forward and off the bicycle.
CHAPTER

M
Explanation
4
The stone abruptly stops the motion of the bicycle, but the
inertia of the boy keeps him in the forward motion and
throws him forwards.
  
F4/2/23
A cardboard is placed on the rim of a glass and a coin is
2
placed on top of the centre of the glass. When the
cardboard
cardboard is pulled away quickly, the coin resting on the
coin
cardboard drops straight into the glass.
Explanation
The inertia of the coin maintains its state of rest. When the
card is pulled away, the coin falls into the glass due to gravity.
Note:
If the card is pulled away slowly, the frictional force
between the coin and the card causes the coin to accelerate
so that it moves together with the card. However, if the card
is pulled quickly, the time is too short for the friction to
cause any appreciable movement of the coin.
3 If thread Y is pulled slowly, thread X will snap. If thread Y is
thread X pulled suddenly, then thread Y will snap.
(can withstand 10.5 N)
Explanation
When thread Y is pulled slowly, the additional force to the
weight (10 N)
weight causes thread X to snap when the tension exceeds
thread Y snaps
the breaking force. When thread Y is pulled abruptly, the
weight maintains its state at rest due to inertia. Thread Y
thus stretches and snaps. (If the weight does move, it will
stretch and snap thread X instead.)

Forces and Motion F4/2/24 58


4 When a stationary bus starts to move forward with an
acceleration, the passengers are thrown backwards.
Explanation
The passengers in the bus are originally in a stationary
state. When the bus starts to move forward with an
acceleration, the inertia of the passengers keeps them in
their position. Thus, the passengers are thrown backwards.

F
O

2
R

CHAPTER
M

5 When a moving bus stops suddenly, the passengers lurch 4


forwards.
Explanation
The passengers are in a state of motion when the bus is
moving. When the bus stops suddenly, the inertia of the
passengers keeps them in motion. Thus, the passengers
lurch forwards.

6 A weight is suspended by a fine thread and rests at point P.


When the weight is raised and then released, the thread
fine
thread
snaps when it passes point P.
thread
snaps Explanation
The fine thread is able to support the weight when it is
stationary. When the weight is allowed to fall, the weight
P
maintains its inertia of moving downwards when it passes
point P. The inertia of the weight causes an additional force
weight to exert on the thread, thus causing it to snap.

F4/2/25
7 If a book is pulled out very quickly from the middle of a
BO
OK

BO

pile of books, the books above it will drop instead of


OK

BO
OK

moving along with it.


Explanation
The inertia of the books above keeps them in their original
position.
F4/2/26
Again, if the book is pulled out slowly, the books above it will
move together with the book as explained in situation 1 .

59 Forces and Motion


8 The decorative item hanging from the rear-view mirror
swings backwards when a stationary car starts to move or
accelerates.
Explanation
The inertia of the decorative item keeps it in its original
position while the car moves forwards.

9 The decorative item swings forwards when the moving car


comes to a sudden halt.
Explanation
F
O
The inertia of the decorative item keeps it in motion when
2

R the car stops suddenly.


CHAPTER

F object will not change its state of motion unless forced to do so.
• Inertia = ‘Laziness’ literally, such that an
O
2

R
CHAPTER

4 I am in
I am at rest, motion.
I will always I will always
stay at rest. move at
constant
speed in a
straight line.

   
• Force is needed to change its velocity, i.e.,
(a) to get the object to start moving, or
(b) to make the object move faster, slower or change the direction of motion.

Hard-boiled or raw?
On a table are two eggs, one raw and one hard-boiled.
How can the two eggs be distinguished?

Solution
The answer is to spin the egg.
Spin each egg on its side. It is much easier to spin the hard-boiled egg.
A hard-boiled egg is solid whereas a raw yolk floats in the liquid white
in a raw egg. When a hard-boiled egg is spun, the solid contents turn
together with the shell. When a raw egg is spun, the yolk and the liquid
white tend to remain at rest because of inertia. The reluctance to spin
along with the shell makes the raw egg harder to spin.

Forces and Motion 60


SPM SPM
Relationship between Mass and Inertia ’04/P1 ’07/P1, P2

1 A shopper in a supermarket observes that it is always easier to start


moving an empty trolley than a full trolley.

Supertankers: Large crude oil F


carriers O

2
R

CHAPTER
Massive oil tankers carry M

(a) Empty trolley    (b) Full trolley crude oil ranging from 150 to


4
300 thousands tons. With full
Figure 2.21
power on, a tanker with a
very large inertia takes a long
2 Similarly, it is easier to stop an empty trolley than a full one if both are time to accelerate to its
moving at the same speed towards the shopper. maximum speed. The tanker
3 The more mass an object has, the harder it is to change its state of might need to travel a
motion. distance of 10 km before
For example, coming to a stop even with
(a) It is more difficult to start moving a bucket filled with sand. the engine set into reverse
(b) When both the buckets are swinging and an attempt is made to stop condition (propeller in
them at the lowest point of the swing, it is more difficult to stop the reverse rotation).
bucket filled with sand. It is not an easy job to turn
the tanker around since the
inertia of the crude oil is
ceiling
enormous. Supertanker
rope of officers need special training
the same in the handling of heavily
length It is harder
to start it
loaded ships.
sand moving from
rest

It is harder
to stop it
empty bucket filled here with
bucket with sand your hand

Figure 2.22
F4/2/28

(c) This shows that the bucket with more mass offers a greater resistance
to change from its state of rest or from its state of motion.
4 By the same reasoning, it is harder to start a bowling ball moving and
harder to stop it than a hollow rubber ball of the same size.
5 Thus, an object with a larger mass has a larger inertia.
Mass is a measure of the inertia of a body.
6 However, inertia is a phenomenon. It has no unit even though it is
closely related to mass.

61 Forces and Motion


2.1
SPM SPM
’04/P3(A) ’06/P3(B)
Relationship between inertia and mass
Situation   Procedure  
Mrs Tan and her son Siao Yang are sitting on two 1 One end of the hacksaw blade is clamped by a
similar swings. G-clamp to a leg of a table as shown in Figure 2.24.
2 A 50 g plasticine ball is fixed to the free end of
the blade.
3 The free end of the blade is displaced horizontally
and released so that it oscillates. The time for 20
F complete oscillations, t20, is measured using a
O stopwatch. This step is then repeated. The average
2

R of t20 is calculated. Then, the period of oscillation,


CHAPTER

M
Figure 2.23 t
T is determined by using T = 20 .
4 Mr Tan gives each of them a push while they are at 20
rest and later tries to stop them in the same position. 4 Steps 2 and 3 are repeated with plasticine balls of
Mr Tan finds that it is harder to push as well as to masses 75 g, 100 g, 125 g and 150 g.
stop Mrs Tan’s motion. 5 A graph of T against m is drawn.
Inference   F
Tabulation of data  
The inertia of an object depends onO its mass.
2

R
Hypothesis   M Table 2.3
CHAPTER

An object with a larger mass has a larger inertia Mass of Time of 20 oscillations, Period of
(represented by a longer period of 4oscillation). load, t20 (s) oscillation,
Aim   t
m (g) t t Average T = 20 (s)
To investigate the relationship between mass and 1 2 20
inertia using an inertia balance  50
Variables    75
(a) Manipulated : mass of plasticine, m 100
(b) Responding : period, T
(c) Fixed : the stiffness of a hacksaw blade 125
and the distance of the centre of 150
the plasticine from the clamp
Graph  
Notes  
A larger inertia means it is more difficult for the T(s)

load to start or stop, thus making the time for one


oscillation, T longer. Therefore, the period of
oscillation represents the inertia.
Apparatus/Materials  
Hacksaw blade, G-clamp, stopwatch, and plasticine
balls of mass 50 g, 75 g, 100 g, 125 g and 150 g. m (g)
Arrangement of apparatus   Figure 2.25
Experiment 2.1

Conclusion  
G-clamp
The graph of T against m in Figure 2.25 shows that
hacksaw blade the period increases with the mass of the load i.e. an
plasticine object with a larger mass has a larger inertia.
The hypothesis is valid.
Figure 2.24

Forces and Motion 62


Effects of inertia

Animals such as dogs and If the bottom of the handle of a hammer is


cats shake their bodies knocked downwards against a hard surface,
vigorously to dry their the loose head of the hammer tightens in its
wet fur. The droplets of wooden handle. The hammer head continues
water on the fur tend to with its downward motion after the handle
continue in motion when has come to a stop. This causes the upper
the fur are reversed in part of the wooden handle to slot deeper into
direction during shaking. the hammer head.
As a result, water droplets F
are separated from the fur O

2
and fall off. R

CHAPTER
M

The chilli sauce in a bottle


Applications can be forced out easily by
When chased by a bull, run in a zigzag pattern. of the turning the bottle upside
The larger inertia of the bull makes it more concept of down and giving it a quick
difficult for the bull to turn around inertia downward shake, followed by
continuously. a sudden stop. The sauce
continues its downward
movement due to its inertia
when the bottle is stopped
causing the sauce to be
forced out of the
Sumo wrestlers are heavy. bottle.
This is an advantage because the When the branch of an
larger inertia forms a greater apple tree is shaken, the
resistance to his opponent who is apples fall to the ground.
trying to topple him. The apples which are
stationary tend to remain at
rest when the branch is shaken.
As a result, the stalks are
strained and the apples break away
from the branch. Droplets of water on a
wet umbrella can be
spun off if the umbrella
is rotated vigorously
and stopped it
abruptly. Owing to
inertia, water droplets
on the umbrella
continue to move even
though the umbrella has
stopped spinning.

63 Forces and Motion


Ways of Reducing the Negative Effect of Inertia SPM
’07/P2

1 If a car crashes while travelling at a fixed velocity of 30 m s–1, for


example, the inertia of the passengers causes them to continue moving
at 30 m s–1 until a force acts to change this speed. This is a
dangerous situation. Upon impact, the passengers will crash into
the parts of the car immediately in front of them and suffer injuries
(Figure 2.26).
2 Ways to reduce the negative effect of inertia in a car:

(a) Safety belt


Seat belts secure the driver
F and passengers to their seats.
O When the car stops suddenly,
2

R
the seat belt provides the
CHAPTER

M
external force that prevents Figure 2.26
F4/2/30
4 the driver or passengers from
being thrown forwards.

(b) Airbag system


Airbags mounted on the
F dashboard or steering wheel
O inflate automatically when a
2

R
collision occurs. This prevents
CHAPTER

M
the driver or passengers from
4 crashing into the dashboard.

3 Lorries that carry heavy loads utilise the following features to reduce the negative effects of inertia:

(a) Strong structure behind the driver’s cabin If a loaded lorry stops abruptly, its heavy load, for
strong iron structure
example, timber logs, will continue to move
forward towards the driver’s cabin because of its
massive inertia. A strong iron structure between
the driver’s cabin and the load ensures the driver’s
safety.

(b) Subdivision of the mass


F4/2/31 to reduce its inertia The figure shows a liquid product being carried in
three separate compartments instead of one. This
reduces the effect of inertia of the liquid on the
walls of each container if the lorry stops suddenly.

(c) Fastening of the objects to the carrier Big items such as furniture or large electrical
appliances (the objects) must be securely fastened
to the lorry (the carrier) so that the objects become
part of the carrier. This will ensure that the objects
and the carrier always move and stop together.
The objects will not fall off the lorry when the lorry
starts moving, or move forwards when the lorry
stops suddenly.

Forces and Motion 64


3 SPM
Clone
’10

The diagram shows two trolleys P and Q with two R S


identical wooden blocks R and S placed on them.
A Pushed to the right Pushed to the left
Trolley P moves and collides with the stationary
trolley Q. B Pushed to the right Pushed to the right
C Pushed to the left Pushed to the right
R S
D Pushed to the left Pushed to the left

P Q
Comments
The collision of the trolleys causes trolley P to slow
Direction of movement of P Stationary down while trolley Q to start moving. Due to F
inertia, R continues to move fast to the right, and O

2
R
Key : Right appeared to be pushed to the right while S, reluctant

CHAPTER
Left M
to move along with trolley Q, appeared to be
pushed to the left. 4
What happens to wooden block R and S?
Answer  A

2.3

1 In a bus moving with a uniform speed in a straight 4 A circus strongman slams a hammer and breaks a
line, a boy drops a steel sphere from rest outside the brick over the hand of a clown. The clown feels no
window. He observes that the steel sphere drops pain. Why?
vertically downwards. Explain. (Caution: Please do not attempt this at home.)
2 A durian is originally placed in a lorry as shown in
Figure (a). When the lorry starts to move with forward
acceleration, the durian rolls backwards. Later, when
the lorry stops suddenly, the durian rolls forwards as
shown in Figure (b). Explain the observations.
hammer

hand brick
on floor
(a)
(a)
(a)
5 Suria runs along a track from P to R through Q while
holding a pail full of water. At which points of the
track will more water likely to be spilt? Give your
explanation.

(b)
(b)
(b) Q
3 The figure below shows two blocks of the same
dimensions but of different materials. One is heavier R
F4/2/32
than the other. WithoutF4/2/32
lifting the two blocks, explain
one way to identify the heavier block.

F4/2/34

65 Forces and Motion


2.4 Analysing Momentum

2.4 Analysing Momentum

What is Momentum?
1 If a loaded lorry and a car are moving at v

the same speed, it is more difficult for


the lorry to stop (Figure 2.27).
2 This is because the lorry possesses a
physical quantity, momentum, more v
than the car.
3 All moving objects possess momentum. Figure 2.27
F 4 Activity 2.2 will help you to gain the idea
O of momentum by comparing the effects
2

R of stopping two objects.


CHAPTER

4
Activ To compare the effects of stopping two objects in motion
ity 2.2

Apparatus/Materials One steel ball and one wooden ball of the same diameter, 2 slabs of plasticine.
Arrangement of apparatus F
steel
O
2

R Note
steel
CHAPTER

M steel
Note
Although both balls
4 100 cm wood
The steel ball have the same
released from a diameter, the steel
greater height strikes 50 cm 50 cm ball has a greater
the plasticine at a plasticine mass than the
greater velocity. wooden ball.
          (a)            (b)
Figure 2.28

(b) Two objects of different masses moving at


(a) Two objects of the same mass moving atF4/2/96
different velocities the same velocity
Procedure Procedure
1 A steel ball is first released from a height of 1 A steel ball and a wooden ball of the same
50 cm and then from 100 cm above a slab of diameter are released from a height of 50 cm
plasticine as shown in Figure 2.28(a). above a slab of plasticine as shown in Figure
2 The depths and sizes of the cavities caused 2.28(b).
by the steel ball on the slab are observed and 2 The depths and sizes of the cavities formed
compared. are observed and compared.
Observations Observations
The depth and size of the cavity caused by the The depth and size of the cavity formed by the
steel ball released from a greater height is deeper steel ball is deeper and larger.
and larger.
Activity 2.2

Conclusion
The moving balls produce an effect on the plasticine which is there to stop the motion. The greater the mass
or the velocity of the moving object is, the greater is the effect (the depth and size of the cavity), the greater is
the momentum.

Forces and Motion 66


Linear Momentum

1 Activity 2.2 also shows that it is always harder to stop a massive object Momentum and inertia are
moving at a high velocity. not the same.
2 The above activity serves to explain a concept in physics called
Momentum  Inertia
momentum.
3 The linear momentum, p, of a m Inertia depends solely on
mass, m, moving at a velocity, mass while momentum as
v, is defined as the product of v
‘mass in motion’ depends on
mass and velocity. both mass and velocity.
Figure 2.29
F4/2/36 F
Momentum = Mass × Velocity O
Unit = kilogram × metre per second

2
∴  p = mv = kg × m s–1
R

CHAPTER
M
4 The unit of momentum is kg m s–1.
5 Momentum is a vector quantity with the same direction as velocity. 4

6 If the direction to the right is denoted as positive, an object moving to


the right possesses a positive momentum while an object moving to
the left will have a negative momentum.

15
A ball of mass 0.8 kg strikes a wall at a velocity of (a) Take the direction to the right as positive.
10 m s–1 and rebounds at 6 m s–1. Momentum of the ball before striking the wall, p1 
What is its momentum = mv
(a) before it strikes the wall, and = 0.8 × 10
(b) after the rebound? = 8 kg m s–1
Solution 10 m s–1 (b) Momentum of the ball after rebound, p2 
= mv
= 0.8 × (– 6)
= – 4.8 kg m s–1
Since direction to the
right is positive, direction
6m s–1 to the left is negative.

Conservation of Momentum 4 To be precise, the Principle of conservation of


momentum is true for a closed system.
A closed system is one where the sum of
1 The term conservation is used if the total
external forces acting on the system is zero.
amount of matter or quantity remains the same
5 The principle shall be discussed in two
before and after the occurence of an event.
situations as shown in Table 2.4.
2 The Principle of conservation of momentum
states that: Table 2.4
A collision An explosion
The total momentum of a system The total momentum of the The sum of the
is constant, if no external force objects before a collision momentums
acts on the system. equals that after the remains as
collision. zero after an
3 An example of an external force is friction. explosion.

67 Forces and Motion


SPM SPM
Collisions ’06/P1 ’08/P1

There are two types of collisions.

Collisions

Elastic collisions Inelastic collisions

Two objects collide and move apart after a Two objects combine and stop, or move together
collision. with a common velocity after a collision.
F m1 m2 m1 m2 m1 m2 m1 m2
O u1 u2 v
u1 u2 v1 v2
2

R
CHAPTER

M
•  Momentum is conserved. •  Momentum is conserved.
4 •  Total energy is conserved. •  Total energy is conserved.
•  Kinetic energy is conserved. • Kinetic energy is not conserved:
The total kinetic energy after the collision is less
than the total kinetic energy before the collision.

Formula: F Formula:
m1 u1 + m2 u2 = m1 v1 +Om2 v2 m1 u1 + m2 u2 = (m1 + m2) v
2

R
CHAPTER

Most collisions are inelastic as a significant amount of kinetic energy is converted to other forms of energy
during the collision. However, the collisions involving two objects that bounce off one another with little
deformation during collision are approximate elastic collisions. Collisions between air molecules are elastic.
Examples of approximate elastic collisions are as follows:

Approximate elastic collisions

PhysicsBlog
Collision of steel balls of equal mass Collision of two snooker balls of equal mass
Before collision After collision Before collision After collision
stationary stationary

P
QR S T P QP
RQS RS
T T

• Ball P is pulled to the side and then released so as to • Ball P is hit and moves with velocity u. It is observed
fall back and strike ball Q. It is observed that ball P that ball P stops when it collides with ball Q which
stops, but ball T swings out to the same height from moves away with velocity u.
which the ball P was released. • This shows that ball Q, after the collision, has the
• This shows that ball T possesses the same amount of same amount of momentum and kinetic energy as
momentum and kinetic energy as P before it struck ball P before the collision.
ball Q.

Forces and Motion 68


Activ To verify the principle of conservation of momentum in
ity 2.3 (a) elastic collisions, and (b) inelastic collisions

Apparatus/Materials
Ticker-timer, 12 V a.c. power supply, runway, 4 trolleys, wooden block, ticker tape, cellophane tape,
and plasticine.

(A)  Elastic collision


Arrangement of apparatus
spring-loaded piston
ticker-timer trolley A
ticker tape trolley B
mA
mB
F
friction-compensated
runway
O

2
R

CHAPTER
M
12 V a.c. wooden
power supply block 4

Figure 2.30
Procedure
1 The apparatus is set up as shown in Figure 2.30.
2 The runway is adjusted so that it is friction-compensated.
3 Two trolleys of equal mass are used. Trolley A with a spring-loaded piston is placed at the higher end of the
runway while trolley B is placed halfway down the runway.
4 A ticker tape is attached to trolley A and another to trolley B. Both ticker tapes are allowed to pass through the
ticker-timer.
5 The ticker-timer is switched on and trolley A is given a slight push so that it moves down the runway at
a uniform velocity and collides with trolley B.
6 After the collision, the two trolleys move separately.
7 From the ticker tapes, the velocities of trolleys A and B before and after the collision are calculated.
8 Assuming that the mass of each trolley is 1 unit, the momentum before and after the collision is calculated
and recorded in a table.
9 The experiment is repeated using:
(a) 1 trolley to collide with 2 stationary stacked trolleys,
(b) 2 stacked trolleys to collide with 1 stationary trolley.
Results
1 Ticker tapes obtained:
(a) Trolley A
after during before
collision collision collision

direction
of motion
x2 x1
x2 x1
vA = uA =
0.2 0.2

(b) Trolley B
after during before
collision collision collision
Activity 2.3

direction
of motion
x3 uB = 0
x
vB = 3
0.2

Figure 2.31

69 Forces and Motion


Tabulation of data

Table 2.5

Before collision After collision


Initial total Final total
mA mB uA momentum, vA vB momentum,
mA uA mA vA + mB vB
1 1
1 2
2 1
F
O
2

R
CHAPTER

M Disscussion
4 1 The spring-loaded piston acts as a ‘springy buffer’ in the collision in order to make the trolley bounce off the
other one.
2 Strictly speaking, this collision is not a perfect elastic collision as part of the kinetic energy of the colliding
trolley changes to sound or heat energy during the collision.

F
Conclusion O
2

From Table 2.5, it is found that: R


CHAPTER

Total momentum
4
before collision = Total momentum after collision

The principle of conservation of momentum is verified.

(B)  Inelastic collision


Arrangement of apparatus

ticker-timer
ticker plasticine
tape
trolley A
trolley B
12 V friction-compensated
a.c. power runway
supply

wooden block

Figure 2.32

Procedure
1 Some plasticine is pasted onto trolleys A and B (both without a spring-loaded piston) as shown in Figure 2.32.
2 A ticker tape is attached to trolley A only.
3 The ticker-timer is switched on. Trolley A is given a gentle push so that it moves down the runway to collide
with trolley B which is stationary halfway down the runway.
4 After the collision, trolley A attaches itself to trolley B and they move together.
Activity 2.3

5 From the ticker tape obtained, the velocity of trolley A before the collision, and the common velocity of
trolleys A and B after the collision are determined. The initial velocity of trolley B is zero.
6 The experiment is repeated using:
(a) 1 trolley to collide with 2 stationary stacked trolleys,
(b) 2 stacked trolleys to collide with 1 stationary trolley.

Forces and Motion 70


Results
Ticker tape obtained: during
collision
before
after
collision
collision
direction
of motion
x2 x1
x2 x1
v = u =
0 .2 0.2

Figure 2.33
Tabulation of data
Table 2.6 F
O
Before collision After collision

2
R
Initial Initial Initial total Final Final Final total

CHAPTER
M
mass, velocity, momentum, mass, velocity, momentum,
mA u (mA u) (mA + mB) v (mA + mB) v 4

1 1+1=2
1 1+2=3
2 2+1=3

Conclusion
From Table 2.6, it is found that:
Total momentum before collision = Total momentum after collision
The principle of conservation of momentum is verified.

16
An astronaut of mass 90 kg moves at a velocity of 6 m s–1 and bumps into a stationary astronaut
of mass 100 kg. How fast do the two astronauts move together after collision?
90 kg 100 kg v
6 m s–1 at rest 100 kg
90 kg

(a) Before collision (b) After collision



Figure 2.34

Solution
This is an inelastic collision.
Total momentum before collision = Total momentum after collision
m1 u1 + m2 u2 = (m1 + m2)v
(90 × 6) + (100 × 0) = (90 + 100)v
540
Activity 2.3

v is the common velocity


v = of the two astronauts.
190
= 2.8 m s–1
The two astronauts move at the speed of 2.8 m s–1 after the collision.

71 Forces and Motion


–1 –1
3ms 7ms

17
A 50 kg skater is moving due east at a speed of 3 m s–1 before colliding into another skater of mass
60 kg moving in the opposite direction at a speed of 7 m s–1. After the collision, the two skaters hold
on to each other. In which direction will they move? What is the speed of the two skaters?
–1 –1
3ms 7ms

v=?

F
O
2

R
(a)  Before collision (b)  After collision
CHAPTER

M v=?
Figure 2.35
4
Solution
Total momentum before collision = Total momentum after collision
m1 u1 + m2 u2 = (m1 + m2)v
50 × 3 + 60 × (–7) = (50 + 60) × v
F 150 – 420 = 110v
v = –2.5 m s O –1
v is the common velocity
2

R
of the two skaters.
CHAPTER

Momentum to the east takenMas positive, so


momentum to the west is negative.
4
The two skaters will move to the west at a speed of 2.5 m s–1.

18
A trolley of mass 3 kg moving at a velocity of 2 m s–1 collides with another trolley of mass 0.5 kg
which is moving at a velocity of 1 m s–1 in the same direction. If the 0.5 kg trolley moves at a
velocity of 2.5 m s–1 in the same direction after the collision, what is the velocity of the 3 kg trolley?
Solution

2 m s–1 1 m s–1 v=? 2.5 m s–1

3 kg 0.5 kg 3 kg 0.5 kg


(a)  Before collision (b)  After collision
The collision is elastic.
Total momentum before collision = Total momentum after collision
m1 u1 + m2 u2 = m1 v1 + m2 v2
3 × 2 + 0.5 × 1 = 3 × v + 0.5 × 2.5
6.5 = 3v + 1.25
3v = 6.5 – 1.25
5.25
v =
3
= 1.75 m s–1
The 3 kg trolley moves at a velocity of 1.75 m s–1 in its original direction.

Forces and Motion 72


SPM
Momentum and Explosions ’04/P2

1 Rifle
2
The explosion creates a backward momentum on
the rifle. This causes the rifle to recoil backwards.
Total momentum of the rifle
and the bullet is zero as they ard
kw tum
are stationary. bac men v1
mo riflev 2 forward momentum
on on bullet
m1
m2

1 F
When the rifle is fired, the explosion of
O
the gunpowder forces the bullet out of

2
the barrel. A momentum in the forward R

CHAPTER
direction is created. M

4
(a)  Before explosion (b)  After explosion
Figure 2.36
F4/2/38

2 Air escapes from a deflating balloon


upward
momentum
m2
balloon v2 2
Total momentum The balloon shoots upwards,
stationary of the balloon is moving with an upward momentum.
(u = 0) zero as it is
stationary. 1
v1 Air has mass and moves with a velocity.
This creates a momentum in the
m1 downward direction.
downward
momentum

(a) Before explosion


(a)  Before explosion (b) After explosion
(b)  After explosion
Figure 2.37

3 An explosion is a closed system which does not involve any external force—that is, the total
momentum is conserved in an explosion.
∴ Total momentum before explosion = Total momentum after explosion
0 = m1v1 + m2v2
Rearranging the formula:
m1v1 = –m2v2
where v1 and v2 are of opposite directions.
If we ignore the direction,
m1v1 = m2v2
as the two momenta have the same magnitude.

• Momentum to the left =  Momentum to the right


Bear in mind that the two velocities
• Momentum upwards =  Momentum downwards are in opposite directions.
• Momentum forwards =  Momentum backwards

73 Forces and Motion


Activ To verify the principle of conservation of momentum in an
ity 2.4 explosion
Apparatus/Materials
4 trolleys, 2 wooden blocks, a hammer, and a metre rule.
Notes
The positions of the wooden blocks are adjusted so that each trolley collides with the corresponding
wooden blocks at the same time, t. From the equation d = vt, the magnitude of the velocity v is
directly proportional to the distance d, i.e., v  d (if t is constant). Thus, the distance d travelled by the
trolley represents the velocity of the trolley.
Arrangement of apparatus Procedure
F
O (a) Before explosion 1 The apparatus is arranged as shown in Figure
2

R 2.38 (a).
release pin
CHAPTER

M wooden
trolley B trolley A block 2 Two trolleys A and B of equal mass are placed in
4 contact with each other on a smooth surface. The
mB mA spring-loaded piston in trolley B is compressed.
3 The release pin on trolley B is given a light tap to
dB dA release the spring-loaded piston which then
pushes the trolleys apart. The trolleys collide with
(b) After explosion F the wooden blocks.
O
spring- 4 The experiment is repeated and the positions
2

vB loaded R v
piston A of the wooden blocks are adjusted so that both
CHAPTER

M
trolleys collide with them at the same time.
4 5 The distances dA and dB are measured and
recorded.
Figure 2.38 6 The experiment is repeated using
(a) 1 trolley with 2 stacked trolleys,
(b) 3 stacked trolleys with 1 trolley.

Tabulation of data
The results of the experiment are recorded in Table 2.7.

Table 2.7
Before explosion After explosion
Initial total Mass of Mass of Velocity of Velocity of Final total
momentum trolley A, trolley B, trolley A, trolley B, momentum,
mA mB dA (–dB) mA dA + mB (–dB)
0 1 1 0
0 1 2 0
0 3 1 0
Discussion
1 Total momentum before explosion =  0 (because both trolleys are stationary)
Total momentum after explosion =  mAdA + mB (–dB) as d represents v,
Activity 2.4

2 Table 2.7 shows that mA dA + mB (–dB) = 0 and v is a vector.


∴ Total momentum after explosion =  Total momentum before explosion
Conclusion
Momentum is conserved in an explosion.

Forces and Motion 74


19
Jane and John go ice skating. With their skates on,
Jane and John push against each other on level ice.
Jane, of mass 50 kg, moves away at a velocity of
3 m s–1 to the right. What is John’s velocity if he is
75 kg?
Solution
This is a closed system since the external force, i.e.,
friction, is negligible.
Let the velocity of John be v.
F
Total momentum = Total momentum
Figure 2.39 O
after explosion before explosion

2
R
m1 v1 + m2 v2 = 0

CHAPTER
M
50 × 3 + 75v = 0
–150 4
v = _____ The minus sign indicates that
  75 John moves to the left,
= –2 m s–1 opposite to Jane’s motion.

Alternative Method
omentum to the right = Momentum to the left
M
m1 v1 = m2 v2
Just ignore the sign as the two
50 × 3 = 75 × v skaters are moving in opposite
v = 2 m s–1 directions.

∴ John moves to the left at a velocity of 2 m s–1.

20
Figure 2.40 shows trolley A, with a weight attached, placed in contact with trolley B on a smooth surface.

weight attached to trolley A


release pin

A B

d1 = 1 d2 d2
3

Figure 2.40

When the release pin of trolley B is tapped lightly, the Solution


spring-loaded piston pushes the two trolleys to move Applying m1 d1 = m2 d2:
in opposite directions. The two trolleys touch the 1 m1 is the total
wooden blocks simultaneously. Trolley A moves one- m1 × d2 = m × d2 mass of trolley A
3 and the weight.
third the distance moved by trolley B. Given that the
two trolleys are of equal mass, m kg, find the mass of m1 = 3m
the weight, in terms of m, that is attached to trolley A. The mass of the weight, m′
= 3m – m
= 2m

75 Forces and Motion


21
Harfeez alights a boat at a v m s–1
velocity of v m s–1 and lands
on the dock. v' = ?
The boat bounces backwards
with a speed of v′.
If the masses of Harfeez and
the boat are m and 3m
respectively, find the speed
of the boat (ignoring friction Figure 2.41
F due to water).
O
2

R Solution
CHAPTER

M
m1 v1 + m2 v2 = 0
4 (m × v) + (3m × v′) = 0
3mv′ = –mv
1
v′ = – v
3
1 –1
The speed of the boat = v m s F
3
O
2

R
CHAPTER

4
Application of the Conservation of Momentum

1 Rocket

1 A rocket carries liquid 5

hydrogen and liquid


oxygen. 5 In accordance with the
principle of conservation of
momentum, the rocket gains
a forward momentum and
2 The mixture of hydrogen 1 moves forwards at high
fuel and oxygen burns liquid velocity.
vigorously in the hydrogen
(as fuel)
combustion chamber.
1
liquid
oxygen

3 The gases formed


expand rapidly and are
forced to discharge 2
through the exhaust combustion 4 A backward momentum
chamber
nozzle at a high is created.
velocity.
3
hot exhaust
gases at
high velocity
4

Figure 2.42
F4/2/41
Forces and Motion 76
2 Jet engine
1 2 3 4
Air from the atmosphere is drawn into In the combustion chamber, The hot gases formed expand rapidly
the engine and compressed by a kerosene fuel burns vigorously and are forced out of the nozzle at
compressor before it is forced into the with the compressed air. high speed through the turbine which
combustion chamber at high pressure. rotates the compressor.

3 combustion 4 turbine blade


2 compressor
chamber

5 F
1
jet of
O

2
air exhaust R

CHAPTER
intake gases M

nozzle

6 moves forwards fuel injector

6
In accordance with the principle of the conservation of 5
momentum, a forward momentum for the engine is produced. The ejected high-speed exhaust gases create
The plane thus flies forwards. a backward momentum.

Figure 2.43

Demonstrations to Show the Principle of Propulsion in Jet fuel


Engines
injector and Rockets
6
The following demonstrates the principle in rockets and jet engines. moves
forward

Jet engine Rocket


2 5 1 5 upward
thread
jet of cylindrical 1
1
3 exhaust weight plastic air
front gases air bottle intake

water
3 plastic tube
bicycle rubber stopper 2
pump
nozzle
elastic back tripod stand
band downward
4 turbine blade 3 combustion 4 2 compressor
chamber
1 A weight is placed on the trolley. 1 One-third of a plastic bottle is filled of water.
F4/2/43b
2 As the thread is cut, the weight is
F4/2/43a 2 The bottle is then plugged
F4/2/42tightly with a rubber stopper with a
thrown to the back with a plastic tube.
momentum. 3 Air is pumped into the bottle.
3 This in turn creates a forward 4 When the pressure of the air in the bottle is sufficient, it will
momentum which causes the push the stopper out together with the water. A downward
trolley to move forwards. momentum is produced.
5 This in turn creates an upward momentum which moves the
plastic bottle upwards.

77 Forces and Motion


• The principle of the conservation of momentum occurs in nature — the squid uses it to propel itself in the water.
The squid moves forward by discharging a jet of water from its body. An equal and opposite momentum created thus
propels the squid in the opposite direction.

F • The shower of burning fragments from an exploding fireworks launched into the sky is governed by the principle of the
O F4/2/44athat the total momentum is conserved.
conservation of momentum. The symmetrical pattern indicates
2

R
CHAPTER

F
• The large volume of water that rushes out from a water hose with a very high speed has a large momentum. In
O
accordance with the principle of theR conservation of momentum, an equal and opposite momentum is created causing
2

the fireman to fall backwards. Thus,M


several firemen are needed to hold the water hose.
CHAPTER

4 SPM
Clone
’11

The diagram shows three identical theme park coins P Q R


on a straight line, with Q and R touching each other.
A Moves Stationary Stationary
P is given a velocity and collides Q.
B Stationary Stationary Moves
Q R C Moves Moves Stationary
token
P token
D Moves Stationary Moves
token

Comments
The momentum of P is totally transferred to R
What will be observed?
through Q.
Therefore, P stops and R moves with the velocity of
P before collision while Q remains stationary.
Answer  B

Forces and Motion 78


2.4

1 A pigeon of mass 120 g is flying at a velocity of 5 A butterfly rests on a leaf floating on the surface of a
2 m s–1. What is its momentum? pond.

5 cm s–1
2 A bull of mass 250 kg is moving at a momentum of
750 kg m s–1. Find its velocity.

3 During training, Othman fires a pistol of 1.5 kg mass. 3 cm s–1

The butterfly then starts moving to the tip of the leaf


at a speed of 5 cm s–1 relative to the water. The leaf, F
stationary in accordance with the principle of the conservation O

2
of momentum, moves at 3 cm s–1 relative to the R

CHAPTER
water in the opposite direction. If the mass of the leaf M
(a) Before is 8 g, determine the mass of the butterfly.
(a)  Beforeshooting
shooting 4
6 Boat A and boat B are moving at a speed of 2 m s–1
and 1 m s–1 respectively before the two collide head
on. The masses of boats A and B (including the
passengers) are 150 kg and 250 kg respectively.
30 g

1.5 kg

(b) After shooting


(b)  After shooting

A bullet with a mass of 30 g is released at a (a)  Before collision


velocity of 300 m s–1. What is the recoil velocity of
the pistol? 0.5 m s–1 v=?

4 Hizam and his son Jamal are at an ice rink.

A B

(b)  After collision


If boat A bounces back with a velocity of 0.5 m s–1,
what is the velocity of boat B?
7 Sau Fei and Siew Ling, each with a mass of 60 kg and
49.5 kg respectively, are standing at rest on an ice
rink. Sau Fei throws a ball of mass 0.5 kg towards Siew
Ling.

Jamal with a mass of 20 kg is moving at a velocity of


2 m s–1 while Hizam with a mass of 60 kg, is directly
behind Jamal and moving at 6 m s–1. Hizam decides
to pick Jamal up and continues moving without
stopping. Determine the final velocity of Hizam and What is the recoil velocity of Sau Fei if the velocity of
Jamal. the ball is 8 m s–1? What is the velocity of Siew Ling
after she receives the ball?

79 Forces and Motion


2.5 Understanding the Effects of a Force

2.5 Understanding the Effects of a Force

What is Force?

F
F A force is a push
or a pull.
Pressing a switch

F
Lifting objects
F
F F
O
2

R F
CHAPTER

M
COKE
4 Pulling off the ring of a soft drink tin Stretching a chest expander Kicking a football

1 When you push or pull on an object, you need to know


(a) the strength or magnitude of your force, and
F4/2/45
(b) the direction in which you are pushing or pulling.
F
2 Therefore, force is a vector quantity since it has both magnitude and
O

10
5
2
0

9
3

6
direction.
2

spring balance
R
3 A spring balance which isM used to measure the weight (or the
CHAPTER

gravitational pull on the object) can be used to measure the magnitude


of a force. For example, the4 force pulling a wooden block can be
measured as shown in Figure 2.44. Figure 2.44

The Effects of a Force

The shape of a tube of toothpaste A spring lengthens or compresses A plastic ruler can be bent when
changes when you press on it. when you stretch or compress it. a force is exerted on it.


1 A force can change the shape of an object (deformation of an object). F4/2/46c

F4/2/46a F4/2/46b

Effects of force

2 A force can change the original state of motion (either at rest or in motion) of an object.
(a)  To move a stationary object (b)  To stop a moving object (c) To change the direction of
motion of an object


A pushing force is required to F4/2/46e
Friction will stop the bicycle A tennis player hits a fast-moving
F4/2/46f
move a F4/2/46d
stalled car. when the boy stops pedalling. tennis ball to return it to his opponent.

Forces and Motion 80


The effects of force are applicable in the games below.
(a)  Football
1
3 Ben kicks the football to Sani from a corner
Hazuri manages to catch the ball. during a football match. He needs to apply
He needs to apply a force to stop the ball. a force to make the stationary ball move.
Also, as his boot is in contact with the ball,
the force applied deforms the ball.

Hazuri
Ben F
O

2
R

CHAPTER
M
Sani
4

2
Sani's header produces a force on the ball to change its direction.

F4/2/47a
(b)  Baseball

3 2 1
If the ball is not hit, it is The force from the bat deforms the ball A pitcher applies a force when
stopped by the catcher. and reverses the direction of the ball. he throws the baseball.
F4/2/47b

Relationship between Acceleration, Mass and Force

1 Consider two identical speedboats A and B at thrust is applied, speedboat P accelerates


rest on the sea. Different engine thrusts are slower than Q.
applied to accelerate the boats. However, 3 Thus, the three quantities: acceleration, mass
speedboat A which applies more engine thrust and force, are related.
increases the velocity at higher rate. 4 The relationship between the three quantities
2 Consider two identical speedboats, P and Q can be determined in the following experiments.
with P heavily loaded. When the same engine

81 Forces and Motion


2.2

Relationship between acceleration and force applied on a constant mass


Situation elastic cord attached to the trolley and stretched to a

fixed length represents one unit of force acting on the
Figure 2.45(a) shows car A and car B of the same
trolley.
mass at the same starting line. Car B is a sports car.
The engine capacity of sports car B is much bigger Apparatus/Materials
than car A. (A car with a bigger engine capacity can  
provide greater engine thrust.) Trolley, 3 identical elastic cords, runway, ticker-
Figure 2.45(b) shows that sports car B has built up a timer, carbonised ticker tape, cellophane tape, 12 V
higher velocity than car A after 3 seconds. a.c. power supply and a wooden block.
F
O Arrangement of apparatus  
2

R B (greater engine capacity)


B
CHAPTER

M
ticker-timer stretched elastic cord (1 unitgreater engine
of force)
ticker tape thrust (force)
4 A
A friction-compensated
a.c. power runway
supply
trolley

wooden block
(a)  t = 0 F
(a)
O F4/2/48
2

B
R greater
CHAPTER

M engine two stretched elastic cords (2 units of force)


thrust [same extension as (a)]
A (force)
4

(b)  t = 3 s
Figure 2.45
(b)
Can you make an inference about this situation?
Figure 2.46
Inference  
The acceleration of an object depends on the force Procedure  
which acts on it. 1 A friction-compensated inclined runway is
Hypothesis prepared.
  2 The apparatus is then set up as shown in Figure
When the mass of an object is constant, the greater 2.46(a).
the force applied, the greater its acceleration. 3 The ticker-timer is switched on and the trolley is
Aim pulled down the runway by an elastic cord

attached to the hind post of the trolley.
To investigate the relationship between acceleration
4 The elastic cord is stretched until the other end
and the force applied on a constant mass.
is level with the front end of the trolley. The
Variables length is maintained as the trolley runs down the

(a) Manipulated : force applied, F runway.
(b) Responding : acceleration of trolley, a 5 The ticker tape obtained is cut into strips of
Experiment 2.2

(c) Fixed : mass of trolley, m, and the 10-tick. A tape chart is constructed and the
stretching force of an elastic cord acceleration, a, is determined.
6 The experiment is repeated with 2, and 3 elastic
Notes   cords to double and triple the pulling force to the
The force in this experiment is the stretching force in same constant extension as when one elastic cord
an elastic cord used to pull the trolley. A length of is stretched.

Forces and Motion 82


Results  
Ticker tape chart:
Velocity (cm per tentick) Velocity (cm per tentick) Velocity (cm per tentick)

the gradient
is tripled
the gradient
is doubled

(a) One elastic cord (b) Two elastic cords (c) Three elastic cords
(a) (1 unit of force)             (b) (2 units of force)             (c) (3 units of force)
F
Figure 2.47 O

2
R
1 The gradient of the line passing through the dot at the top of each successive strip increases with the number

CHAPTER
M
of cords.
2 This indicates an increase in acceleration when the force is increased. 4
Tabulation of data   Graph  
Table 2.8 1 The graph of a against F is a straight line passing
through the origin as shown in Figure 2.48.
Units of force Acceleration, This shows that: a  F (if m is constant).
applied, F a (m s–2) acceleration, a
1 a1
2 a2
3 a3
force, F
O
Figure 2.48
Conclusion  
The acceleration of an object is directly proportional to the force applied if the mass is constant.
The hypothesis is valid.

2.3 acceleratio

Relationship between acceleration and the mass of an object under a constant force
Situation

Figure 2.49(a) shows two similar lorries, A and B in front of a traffic light. When the light turns green, both drivers
step on the accelerator simultaneously with the same pressure to provide the same engine thrust, F. O
Figure 2.49(b) shows that within 3 seconds, the empty lorry has built up a higher velocity than the heavy one.
Experiments 2.2 & 2.3

B
B

full-loaded F B
lorry
A
A
empty
lorry F A


(a) 
t = 0 (b)  t = 3 s
Figure 2.49

F4/2/49
83 Forces and Motion
Inference 5 The elastic cord is stretched until the other end

is level with the front end of the trolley. The
The acceleration of an object depends on its mass. length is maintained as the trolley runs down the
Hypothesis   runway.
When the force applied on an object is constant, the 6 The ticker tape obtained is cut into strips of
greater the mass of the object, the smaller its 10-tick. A tape chart is constructed and the
acceleration. acceleration, a, is determined.
Aim 7 The experiment is repeated using 2 trolleys (with
  a second trolley stacked on the first trolley) and
To investigate the relationship between acceleration 3 trolleys. The elastic cord is stretched to the
and the mass of an object under a constant force. same fixed length as in the first experiment.
Variables   Results
F

(a) Manipulated : mass of trolley, m Ticker tape chart:
O
(b) Responding : acceleration of trolley, a
2

R Velocity (cm per tentick)


(c) Fixed : force applied by an elastic cord, F
CHAPTER

M
Notes  
4
(a) The mass in this experiment is represented by the
number of identical trolleys used.
(b) The constant force is applied by stretching the
elastic cord with the same extension for each (a) One trolley (1 unit of mass)
repetition of the experiment. F Velocity (cm per tentick)
O
Apparatus/Materials  
2

R
CHAPTER

Ticker-timer, 12  V a.c. power Msupply, 3 trolleys, the gradient


is halved
elastic cord, runway, wooden block, ticker tape and
4
cellophane tape.
Arrangement of apparatus  
(b) Two trolleys (2 units of mass)
ticker-timer stretched elastic cord
ticker tape Velocity (cm per tentick)

friction-compensated runway
a.c. power
supply the gradient is
trolley one third of (a)
wooden block

(a)
stretched elastic cord
(c) Three trolleys (3 units of mass)
F [same extension as in (a)]
Figure 2.51

2 units of mass
The gradients of the lines joining the uppermost dots
for successive strips decrease as the number of
trolleys used increases. This indicates that
acceleration decreases as mass increases.
(b)
Tabulation of data  
Figure 2.50
Table 2.9
Procedure  
Mass of Inverse
1 A friction-compensated inclined runway is Acceleration,
trolley, m of mass,
prepared. 1 a
Experiment 2.3

(Number of
2 The apparatus is then set up as shown in Figure 2.50. m (m s–2)
trolleys)
3 A ticker tape is attached to the trolley and passed
through the ticker-timer. 1 1.00 a1
4 The ticker-timer is switched on and the trolley is 2 0.50 a2
pulled down the inclined runway by an elastic 3 0.33 a3
cord attached to the hind post of the trolley.

Forces and Motion 84


Graph 1
  The graph of a against shows a straight line
m
acceleration, a passing through the origin as shown in Figure 2.52.
1
This shows that:   a  (if F is constant).
m

Conclusion  
The acceleration of an object is inversely
O
proportional to its mass when the force acting
Figure 2.52 on it is constant.
The hypothesis is valid.
F
O

2
R

CHAPTER
M

Newton's Second Law of Motion 22

1 From Experiment 2.2:   a  F A force of 10 N acts on an object of mass 5 kg on a


smooth floor. Find its acceleration.
1
From Experiment 2.3:   a  a=?
m
10 N
The two results are combined.
F
a 
m
k is a constant. Figure 2.54
or  F  ma
∴  F = kma Solution
2 The unit of force is Newton, N. F = 10 N, m = 5 kg
3 In order to make the formula as simple as possible, F = ma
we make k = 1 by defining a force of 1 N as: 10 = 5a
a = 2 m s–2
1 N is the force which gives a mass of 1 kg
an acceleration of 1 m s–2.

1 kg a = 1 m s-2
1N Fnet = ma is the mathematical expression of Newton's
second law of motion.
This law states:
Figure 2.53 The net force on an object is proportional to the rate of
change of momentum.
Change in momentum
F = kma Force
(net) 
Time
1 N = k × 1 kg × 1 m s–2 mv – mu The net force on an
Experiment 2.3

k = 1 Fnet  object causes the


t object with a mass
   ∴ F = ma (v – u) of m to change its
Fnet  m velocity from u to v.
t

Newton's second Fnet  ma v–u


a=
law of motion Fnet = kma t

85 Forces and Motion


23
–152
A car of mass 1200 kg travelling at 15 m s–1 comes a = 2 × 30
to rest over a distance of 30 m. Find Retardation is
deceleration.
(a) the average retardation, and a = – 3.75 m s–2
(b) the average braking force. ∴  Retardation = 3.75 m s–2
Solution (b) F = ma Negative sign indicates
= 1200 × (–3.75) retardation force or
u = 15 m s–1, v = 0 m s–1, s = 30 m opposing force.
= – 4500 N
(a) Using the formula v2 = u2 + 2as:
0 = 152 + 2a(30) Average braking force = 4500 N

F
O
2

SPM SPM SPM SPM


R Balanced Forces and Unbalanced Forces ’03/P1 ’05/P1 ’07/P2(C) ’08/P1
CHAPTER

M
1 In general, there may be several forces acting on the object (whether a single force or several
4
on the mass, whether parallel or anti-parallel, forces are acting on it).
or in different directions.
2 Thus, the force, F, must be replaced with the Balanced Forces
net or resultant force when there are several
forces acting on the mass. 1 When the forces acting on an object are
F
O
balanced, they cancel each other out (that is,
2

R to use ΣF = ma
Some prefer
net force = 0).
Fnet = ma 2 The object then behaves as if there is no force
CHAPTER

where ΣFMis the sum of the


force vectors. acting on it.
4
3 Since Fnet = 0, the acceleration of the object,
where a is in the direction of the net or a = 0. Thus, the object remains at rest or moves
resultant force. at constant velocity when there is no net force
3 However, for simplicity, F = ma is always used, acting on it. This is Newton’s first law of motion.
bearing in mind that F is the net force acting 4 Examples of balanced forces:

(a) Balanced forces on a stationary gymnast (b) Balanced forces on a car moving at a constant velocity
R

air resistance, G
thrust, T

T = G + Fr
friction, Fr
weight, W W=R
W
W=R

reaction force, R
from beam • There are 3 horizontal forces acting on a car moving
at a constant velocity. The forward thrust, T,
provided by the car engine is balanced by the
frictional force on the wheels and the air resistance.
• The weight of the gymnast, W, is balanced by the
F4/2/143
reaction force, R, from the beam. T = G + Fr
•  The two forces are of equal magnitude but opposite
in direction. • The weight of the car, W, is vertically balanced by
• Without the beam (that is, no reaction force), the the reaction force, R, from the road.
gymnast will fall to the ground because of her
weight. W=R

Forces and Motion 86


Balanced forces (Fnet = 0, a = 0)

F1 F2
F net = 0
(as no force
acting on it)
F1 = F2 F4/2/53a
From Fnet = ma:
0 = ma
∴  a = 0 (since mass, m cannot be zero)

Object in motion F
Object at rest (v ≠ 0, and the object is moving O
(v = 0 m s–1)

2
at constant velocity) R

CHAPTER
M

For example, For example,

5N
2 cm s–1
PHYSICS
200 N Ffriction

Ffriction
Linda pushes a book on a table with a force of 5 N.
F4/2/53c
Zamhari pushes a heavy cupboard with a force of
F4/2/53b The book moves with a uniform velocity of 2 cm s–1.
200 N, but the cupboard does not move. Find the frictional force acting on the book.
Find the frictional force acting on the cupboard. Solution
Solution Using Fnet = ma: because the book
Using Fnet = ma: because the But Fnet = 0 since a = 0 moves with a
cupboard does uniform velocity
But Fnet = 0 since a = 0 5 – Ffriction = 0
not move
∴  200 – Ffriction = 0 Ffriction = 5 N
Ffriction = 200 N (The frictional force here is known as dynamic
(The frictional force here is known as static friction) friction)

Note:  In both situations, the frictional forces are equal to the forces applied.

SPM
Effect of Balanced Forces and Unbalanced Forces on an Object ’08/P2(A)

Balanced forces (Fnet = 0, a = 0)

lift from wings, L 1 Balanced forces on an aircraft allow it to


move at constant velocity at a constant
altitude.
drag or air resistance, G
2 The engine thrust is balanced by the drag due to
engine thrust, T
air resistance while the weight of the aircraft is
T = G
weight, W
W = L
balanced by a lift from the wings. The lift from
the wings is discussed in Chapter 3.

87 Forces and Motion


Unbalanced forces (Fnet ≠ 0, a ≠ 0)
lift from wings, L

engine thrust, T

drag or air
weight, W resistance, G

1 When the forces acting on an object do not cancel out each other, a net force known as unbalanced force is
acting on the object.
2 Unbalanced forces produce an acceleration to the mass on which the forces are acting.
F 3 However, the object will accelerate in the direction of the net force.
O 4 When an airplane is moving at a constant velocity, if the pilot increases the engine thrust, the forces acting
2

R
horizontally are no longer balanced. There is a net force forwards and the plane will accelerate in the
CHAPTER

M
forward direction.
4

24
5000 N
Figure 2.55 shows Solution
upward
a small rocket of F
force from Since the upward force is greater than the downward
mass 300 kg at the O
engine
force by 2000 N, the rocket accelerates upwards.
2

R 2000 N
point of take-off. Neglecting the mass of the exhaust gases, the upward
CHAPTER

M resultant
Find its initial mass
upward launching acceleration is given by:
300 kg
acceleration. 4 force Fnet = 5000 N – 3000 N
weight = 2000 N
Fnet = ma
3000 N F 2000
a = mnet =
  300
               Figure 2.55 =  6.7 m s–2
F4/2/55b

25
John pushes a 12 kg carton with a force of 50 N. • When a floor is smooth, frictional force is nil.

2 m s–2 F E velocity increasing


IL (a ≠ 0)
AG
FR
F1 = 50 N
Ffriction = 0 smooth surface
Fnet = 50 – Ffriction

• When an object on a rough floor moves at uniform


frictional force, Ffriction
velocity with a horizontal force acting on it, friction
is equal in magnitude to the applied force (but
Figure 2.56 acts in the opposite direction).
F4/2/55a
If the carton moves with an acceleration of 2 m s–2,
what is the frictional force acting on the carton? constant velocity
F (a = 0)
E
Solution GIL
FRA
Fnet = ma
Ffriction rough surface
50 – Ffriction = 12 × 2
F = Ffriction
F=F
Ffriction = 50 – 24 = 26 N friction

Forces and Motion 88


26 5 SPM
Clone
’05

A shopper pushes a trolley with a force of 20 N. The Two forces F1 and F2 act on a wooden block which is
trolley with a mass of 5 kg, moves at a uniform placed on a table. The friction between the table and
velocity of 1 m s–1. He then increases his force to the block is 3 N.
accelerate the trolley. What force should he apply in
order to give the trolley an acceleration of 2 m s–2? F2 F1

Solution
table

20 N Fʹ = ?
Which pair of forces F1 and F2 will accelerate the block?
1m s–1
a=2m s–2 F
F1 (N) F2 (N) O

2
R
A  4  7

CHAPTER
(a) (b) M
F4/2/57 B  8  5
For (a): Ffriction = 20 N 4
C  6  4
For (b): Fnet = ma The trolley is moving D  9  5
F′ – Ffriction = ma with uniform velocity,
F′ – 20 = 5 × 2 i.e. the force applied
equals friction acting Comments
F′ = 30 N on the trolley.
The block will move with acceleration if the forces
acting on it are unbalanced.
If the difference between the forces F1 and F2 is
27 greater than 3 N, then there is an acceleration.
Mr Brown whose mass is 70 kg, performs as a human Answer D
cannonball at a circus. He is propelled from a 1.6 m
long cannon. He is in the barrel of the cannon for 1.2 s.
6 SPM
Clone
’07

The figure below shows a car of mass 1200  kg


moving at an acceleration of 2 m s–2.

acceleration
Figure 2.57
Find the average net force exerted on him.
Solution If the frictional force acting on the car is 750 N, find
First, we need to find the acceleration of Mr Brown its engine thrust.
in the barrel. Then, apply F = ma to find the net force A 750 N C 2400 N
on him. B 1350 N D 3150 N
u = 0 m s–1, s = 1.6 m, t = 1.2 s Solution
1
Using s = ut + at 2: Use Fnet= ma since two forces are acting on the car.
2
Let T be the engine thrust of the car.
1 T – Friction = ma
1.6 = 0 + a (1.2)2
2 T – 750 = 1200 × 2
a = 1.6 × 2  =  2.22 m s–2 T = 2400 + 750
1.44 = 3150 N
Fnet = ma = 70 × 22.2 = 1554 N Answer D

89 Forces and Motion


Newton’s Third Law of Motion
Newton’s third law of motion states that:

If object A exerts a force, F on object B, then object B will exert an equal but opposite force, –F on
object A. In other words:
To every action there is an equal but opposite reaction.

Everyday phenomena that are governed by Newton’s (b) When a man paddles with a backward force, +F
third law of motion: Physics
Blog (action), the reaction force, –F, pushes the boat
(a) When a boy presses on the wall with a force, F, the forwards.
wall presses on his hands with a normal reaction
F
force, –F.
O
2

R
reactions, –F
CHAPTER

M
action,+F
reactions,
4
–F action, +F

F4/2/60b
(c) The principle used in rockets and jet engines can also
be explained by Newton’s third Iaw of motion. The
F action that pushes the exhaust gases out through the
Note: The action and reaction forces of Newton’s third
F4/2/60a O
law act on different objects whereas two nozzle results in a forward force (reaction force) that
2

R
balanced forces act on the same propels the rocket or jet engine forwards.
M object.
CHAPTER

2.5

1 What force is needed so that an object with a mass of 6 (a) When a 2 kg block is pushed with a force of
3 kg has an acceleration of 2 m s–2? 12 N to the right, it accelerates with an
2 A wooden box of mass 2 kg is placed on a smooth acceleration of 3 m s–2. In which direction does
plane. If a force of F is applied to the box, it moves at friction act? Find its magnitude.
an acceleration of 3 m s–2.
a = 3 m s–2
Find the acceleration of another box with mass of
6 kg if the same force is exerted on it.
12 N
3 Puan Zaitun pushes a trolley of mass m kg with a force 2 kg
of 30 N. The trolley moves with a uniform velocity of
1 m s–1. When she doubles her force, the trolley
accelerates at 2 m s–2. Find the mass of the trolley.
F4/2/62
4 Faizal rides a bicycle at a constant speed of 16 m s–1. (b) An additional force of 20 N to the left then acts
He stops pedalling and the bicycle stops completely on the block. In which direction does friction
after 6 s. Given the total mass of Faizal and his now act? Find the new acceleration of the
bicycle is 72 kg, find the average opposing force on block.
Faizal and his bicycle.
5 A race car of mass 1200 kg accelerates from rest to a
–1 12 N 20 N
Clone velocity of 72 km h in 8 s.
SPM
2 kg
’07
(a) Find the acceleration of the car.
(b) Find the net force acting on the car.

F4/2/63

Forces and Motion 90


2.6 Analysing Impulse and Impulsive Force

Impulse and Impulsive Force

• When a player kicks a F4/2/64a


football, his boot is in
contact with the ball for a time t. F
O
• During the time t, an average force F acts on the • When a tennis racket hits an oncoming tennis ball,

2
F4/2/64b R
ball which makes the ball fly off with a the tennis racket delivers a large force that acts on

CHAPTER
M
momentum. the ball for a short time t. The ball bounces off in
• Thus, the force F, acting for a period of time t, the opposite direction. Again, there is a change in 4
produces a change in momentum to the ball, momentum.
since the ball with a mass of m acquires velocity v • The change in momentum is due to the force F
after the time t. acting on the object for a time t.

F = ma
F=m
( )v–u
t
substitute a=
v–u
t

SPM
Ft = mv – mu mv – mu ’07/P1
  F = t

Impulse =  Change in momentum


Change in momentum
Impulsive force =
  Time taken
unit = N s or unit = kg m s–1
unit = N
• Impulse is defined as the product of a     
force, F and the time interval, t during which • Impulsive force is defined as the rate of change
the force acts. of momentum during a collision or explosion.
Inpulse = Force × Time
=F×t • Impulsive force is the large force produced in a
• Impulse is a vector quantity and has the same collision or explosion that happens in a short
direction as the force that causes the change in time of impact.
momentum.

•  Both formulae can be used to solve numerical problems.


•  Both are vector quantities.

91 Forces and Motion


Rebound and Impulse

1 An object might rebound from a wall, or stick to it without rebounding after striking it.
2 In which situation will the wall exert a greater impulse? Look at Example 28.

28
A tennis ball and a piece of mud with the same mass Solution
(0.060 kg) which are moving at 9 m s–1 strike a wall. Take the direction to the right as positive.
The mud sticks to the wall while the ball rebounds at
6 m s–1. Find the impulse on each object. For the mud: For the tennis ball:
Impulse Impulse
9 m s-1
F = Change in momentum = Change in momentum
O mud = mv – mu = mv – mu
2

R 9 m s-1
= 0 – 0.06 × (–9) = 0.06 × 6 – 0.06 × (–9)
CHAPTER

M
tennis ball = 0 + 0.54 = 0.36 + 0.54
4 6 m s-1 = 0.54 N s = 0.90 N s
Figure 2.58

3 Thus, a greater impulse is exerted


F on an object if it rebounds after a collision.
O
2

29 R
CHAPTER

M
Figure 2.59(a) shows a 2 kg wooden block initially at If the force acts for 3 s,
4
rest on a smooth surface. A force of 8 N is applied on (a) what is the impulse on the block?
the wooden block. (b) what is the velocity of the wooden block after 3 s?
u=0 v Solution
8N 8N
2 kg 2 kg
(a) Impulse = F × t = 8 × 3 = 24 N s
t=0s t=3s (b) Impulse = Change in momentum
Ft = mv – mu
(a) (b)
24 = 2v – 0
Figure 2.59
v = 12 m s–1

30
A player spikes an oncoming volleyball moving Solution
towards him at a speed of 5 m s–1 to reverse its direction (a) Take the direction away from the hand as positive.
at a speed of 20 m s–1. The mass of the ball is 0.36 kg. Impulse = Change in momentum
5 m s-1 20 m s-1
= mv – mu
= 0.36 × 20 – 0.36 × (–5)
= 0.36 × 20 + 0.36 × 5
F
= 9Ns
mv – mu
(b) F =
t
9
Figure 2.60 300 =
t
(a) Find the impulse on F4/2/164
the ball. 9
(b) If the average force acting on the ball is 300 N, t =
300
how long is the time of contact between the hand
and the ball? = 0.03 s

Forces and Motion 92


Effects of Time on Impulsive Force SPM
’09/P1
SPM
’09/P2/(A)

1 From F = mv – mu 1
t F
t
= Change in momentum ⇒ t small, F large
Time of impact
⇒ t large, F small
If the change in momentum is constant, then:
From F  1
t
2 A student throws a raw egg at a high speed at a wall, and another egg
against a towel held by his friends. In which case will the egg break? F
O

2
towel
R

CHAPTER
egg
egg M

egg
egg 4
pile of towel

The egg is stopped by the wall in a very short time The egg thrown against a towel falls on a few towel
interval, resulting in a large force which causes it towels stacked below and does not break.
to shatter. The movement of the egg is stopped in a longer
time interval, resulting in a small force.

In both cases, the eggs are stopped. Thus, the change in momentum is the same. pile of towe

3 To understand the effect of time in a collision, look at the following example.

Save your knees!


Joginder (mass, m = 50 kg) jumps down from a wall. He lands on a cement ground at a velocity of 6 m s–1.

Joginder bends his knees upon landing. Joginder didn’t bend his knees upon landing.
The time taken to stop his motion is 1.0 s. The time taken to stop his motion is 0.05 s.

u = 6 m s–1
u = 6 m s-1
v = 0 m s–1
v = 0 m s-1
Solution
Take the downward direction as positive. Solution
m = 50 kg, u = 6 m s–1, v = 0 m s–1, t = 1.0 s Take the downward direction as positive.
mv – mu m = 50 kg, u = 6 m s–1, v = 0 m s–1, t = 0.05 s
F =
t The negative sign indicates the mv – mu 50 × 0 – 50 × 6
50 × 0 – 50 × 6 F= =
= force acts as an opposing force t 0.05
1 that reduces the momentum
of the object. = –6000 N
= –300 N
The impulsive force acting on Joginder’s leg is 6000 N!
This force is great enough to break Joginder’s legs.
The impulsive force that acts on Joginder’s leg is 300 N.

This example clearly shows that an effective way to reduce the impulsive force is to lengthen the collision time.
u = 6 m s–1

v = 0 m s–1
93 Forces and Motion
Increasing the Impulsive Force by Reducing the Time of Impact

In all the sports shown below, the time of impact with


It should be noted that
the ball is very small and the impulsive force
impulsive forces always
produced is large.
exist in pairs. In the figure
on the right, the change in
momentum of the tennis
ball produces a large
impulsive force on the
racket which reacts to give
rise to an equal but
F2 F1
FF opposite impulsive force to
OO the ball (this is in
A footballer kicking A golfer driving a
2

RR a football golfball with a club accordance with Newton’s


CHAPTER

MM third law of motion).


Both the racket and the
44
tennis ball are deformed
temporarily due to the
large force being exerted on
A batter hitting a A tennis player hitting a each other.
baseball tennis ball
F
O
2

R
CHAPTER

M
A massive hammer head
moving at a fast speed is
4
brought to rest upon hitting
The effects of
t time ont the
the nail. The large change in
small small

F F
momentum within a short
time interval produces a
large impulsive force which large
magnitudelarge
la
of
drives the nail into the wood. the impulsive
force

pile driver

In
pestle
pile construction,
tion
the pile driver
is raised to a
certain height
before it is
An expert in karate can released. The
mortar
split a thick wooden slab momentum
with his bare hand which acquired by Food such as chillies and
is moving at a very fast the massive onions can be pounded using a
speed. The momentary pile driver is greatest just before it mortar and pestle (both made
contact between the hits the pile. The subsequent from stone). The pestle is
fast-moving hand and change in momentum that brought downwards at a fast
the wooden slab occurs in a short time interval speed and stopped by the
produces a large results in a large impulsive mortar in a very short time.
impulsive force which force which drives the pile into This produces a large
splits the wooden slab. the ground. impulsive force which crushes
the food.

Forces and Motion 94


94
Reducing the Impulsive Force by Increasing the
Time of Impact In sports, the effects of
impulsive forces are reduced
to prevent injuries to
athletes. Thick mattresses
Polystyrene and cardboard egg with soft surfaces are used
containers are stiff but in events such as the high
compressible. They will jump and pole-vaulting so
absorb and reduce impulsive that the time of impact
force by lengthening the on landing is extended,
time of impact. thus reducing the
resultant impulsive force.
F F
polystyrene
O O

2
R R

CHAPTER
The use of padding in certain sports equipment M M
like baseball gloves, goalkeeping mitts, pole-
vaulting pits, boxing gloves, and gymnastic mats 4 4
cardboard egg carton
is to prevent injuries to players by reducing the
impulsive force.

The effects of
t
(a) Baseball glove (b) Boxing glove

time on the
large
magnitude of
the impulsive F small
In baseball, a player must catch
the ball in the direction of the
force motion
m
mo
caught
c
cau
of the ball. If the ball is
by stopping it in its path,
the
t impulsive force acting on the
hand
h
ha will be considerable.
Moving
M
Mo his hand backwards
when
wh catching the ball
prolongs
pr the time for the
momentum change to occur so
mo
the impulsive force is reduced.

Playgrounds are covered with a coarse


fabric material which prolongs thee time When a boxer sees that
of impact when the children fall,l, thus his opponent’s fist is
reducing the impulsive force. going to hit his head, he
will move his head
backwards or duck.
This will increase the
stopping time, hence
reducing the average
force on his head since
the momentum change
coarse fabric
material will be longer.

95 Forces and Motion


31
A tennis player hits an oncoming 0.060 kg tennis ball Students are always tempted to find the acceleration, a
with a velocity of 60 m s–1. The ball bounces off in the and then use F = ma to find the impulsive force.
opposite direction at 90 m s–1. mv – mu
By using F = , you save the step of finding a,
t
90 m s-1
which is not necessary. Try using the latter formula if the
125 N question mentions the impulsive force, and
acceleration is not stated.
60 m s-1

F
7 SPM
Clone
’07
O
2

R Figure 2.61 The front and rear sections of a car are designed to
CHAPTER

M crumple easily. What is the reason?


Find the time of impact between the racket and the A To increase the impact time
4 ball if the impulsive force acting on the ball is 125 N. B To increase the impulse
Solution C To decrease the momentum
D To increase the frictional force
Take the direction to the right as positive.
F = 125 N, m = 0.060 kg, v = 90 m s–1, u = –60 m s–1 Comments
F
mv – mu Change in momentum
F = O Impulsive force =
2

t R Time
0.06 × 90 – 0.06 × (–60)
CHAPTER

M
When the time of impact is prolonged, the impulsive
125 =
t force is reduced.
4
5.4 + 3.6
t = Answer A
125
= 7.2 × 10–2 s

8 SPM
Clone

32 ’09

Tiger Woods hits a golf ball of mass 0.045 kg at a An athlete bends his legs upon landing in a long
velocity of 30 m s–1. jump event as shown in the Figure 2.63.

m = 0.045 kg

30 m s–1

Figure 2.62

If the time of impact is 0.005 s, what is the average


impulsive force applied on the ball by the club?
Solution Figure 2.63
m = 0.045 kg, u = 0, v = 30 m s–1, t = 5 × 10–3 s
The purpose of bending his legs is to reduce the
Impulsive force, F = mv – mu A impulse on his feet
t
B impulsive force on his legs
(0.045 × 30) – (0.045 × 0)
= C velocity before landing
5 × 10–3 D time of impact between his legs and the sand.
= 270 N

Forces and Motion 96


Comments landing to lengthen the time of impact; reducing the
The impulsive force is inversely proportional to the impulsive force.
time of impact. The athlete bends his legs upon
Answer B

2.6

1 A force F acts on a 6 kg object at rest on a smooth 6 A baseball of mass 0.14 kg moving at 40 m s–1 is
surface. If the velocity of the object increases to struck by a bat and rebounds at 60 m s–1.
2 m s–1 in 6 s, what is the value of F ? F
2 An object of mass 2 kg is acted on by a force which O

2
R
causes the velocity of the object to increase from 40 m s-1 60 m s-1

CHAPTER
M
1 m s–1 to 9 m s–1. What is the impulse on the object?
3 The figure shows a helicopter dropping a box of mass 4
60 kg which touches a sandy ground at a velocity of
12 m s–1. The box takes 2 seconds to stop after it (a) (b)
touches the ground.
Given that the time of contact is 5 × 10–2 s, find the
force exerted on the ball.

7 In a crash test, a car of mass 1500 kg crashes into a


wall at 15 m s–1. It rebounds at 2 m s–1. 40 m s-1

15.0 m s–1

(a)  Before collision

2 m s-1

What is the magnitude of the impulsive force on the


box? F4/2/77

4 Beckham kicks a ball with a force of 1500 N. The (b)  After collision
time of contact of his boot with the ball is 0.008 s. If the collision time is 0.18 s, find the force exerted
What is the impulse delivered to the ball? If the mass on the car.
of the ball is 0.5 kg, what is the velocity of the ball?
5 A 0.045 kg golf ball strikes a wall at a speed of 8 A pole-vaulter (mass, m = 50 kg) falls onto a foam
30 m s–1 and rebounds at a speed of 20 m s–1. mattress which exerts a force of 250 N on him over a2 m s-1
time interval of 2.0 s.
30 m s-1

u=?
foam 250 N
20 m s-1 mattress

What is the impulse on the ball? If the force on the


ball is 500 N, find the contact time of the ball with
the wall. fo
Find his velocity just before landing on the mattress. ru

97 Forces and Motion


2.7 Being Aware of the Need for Safety Features in Vehicles

lmportance of Safety Features in Vehicles SPM


’07/P1

1 It is important to increase the time interval Safety seat belts


of collision to reduce the impulsive force in • Prevents the passenger from being
thrown forward or out of the car. Slows
an accident. This concept was explained in down the forward movement of the
Section 2.6. passenger when the car stops abruptly.
2 Figure below explains the safety features in
the design of vehicles. Headrest
• To push the driver's head so that it moves
F together with the rest of his body when a
O stationary car is knocked from behind and
2

R Shatter-proof windscreen pushed forward suddenly. This will prevent


• Prevents the windscreen
CHAPTER

M a severe whiplash injury to the neck.


from shattering and reduce
injuries of passengers caused
4 by the shards of glass.
.
Automatic airbag
• Acts as a cushion
for the head and
body in an F
accident and thus
O
prevents injuries to
2

the driver and front R


CHAPTER

passenger. M

Tyres with tread


• Drains away water in
its grooves when the
car is on a slippery
road. The car has
more contact on
the road.

Strong steel struts


• Prevents the collapse of the front and back doors
of the car into the passenger compartment.
Bumper Also gives good protection from a side-on collision.
• Absorbs the impact
in minor accidents,
thus preventing
damage to the car. Anti-lock braking system (ABS)
• Prevents the wheels from locking when the
brakes are applied suddenly.
• Allows a car to stop quickly in slippery road
conditions and prevents the car from skidding.

Padded dashboard
• Increases the time interval of collision,
thereby reducing the impulsive force Crumple zone
produced during impact. • Increases the time interval of impact so that the resultant impulsive
force is reduced. Therefore, only little energy is transferred to the
inside of the car, and the passengers sustain minimal bodily injuries.
• The car is less likely to rebound upon impact. This reduces the
momentum change or impulse.

Forces and Motion 96


98
2.8 Understanding Gravity
SPM SPM SPM A stroboscopic photograph is
Acceleration Due to Gravity ’05/P1 ’08/P1 ’09/P1
a photograph that shows the
images of an object in motion.
1 Figure 2.64 shows a stroboscopic photograph of two steel spheres The images are taken at
(of different sizes) falling under gravity. The two spheres are dropped regular time intervals.
simultaneously from the same height.
2 The time intervals between two successive images are the same.
3 Two inferences can be made from the photograph.

Inference 2 F
Inference 1 s1 O

2
The two spheres are falling R

CHAPTER
M
The two spheres are falling with the same acceleration.
s2
with an acceleration. The two spheres are at the 4
The distance between two same level at all times.
successive images of each s3 Thus, a heavy object and a
sphere increases, showing light object fall with the
that the two spheres are same gravitational
falling with increasing acceleration.
s4
velocity, i.e., falling with an In other words, gravitational
acceleration. acceleration is independent
of mass.

Figure 2.64

Free-falling Object 3 Practically, a free fall can only take place in a


vacuum.
1 A free-falling object is an object falling under 4 The demonstration in Table 2.10 shows the
the force of gravity only. difference between a fall in air and a free fall in
2 A free-falling object does not encounter other a vacuum of a coin and a feather. Both the coin
forces like air resistance or friction that would and the feather are released simultaneously
oppose its motion. from the same height.
Table 2.10
In air In vacuum

to vacuum
pump
coin tap
feather


The air resistance to the feather is greater than the In a vacuum, both the coin and the feather would
air resistance to the coin. The feather falls with a fall with the same acceleration regardless of their
smaller acceleration and takes a longer time to reach shapes or masses.
the ground.

99 Forces and Motion


Acceleration of Gravity

1 Figure 2.65 shows that all objects are pulled towards the centre of the
Earth by the force of gravity.
2 The objects will fall with an acceleration of 9.8 m s–2 due to the pull of
this gravitational force.
3 Since this acceleration is due to the force of gravity, it is called the
gravitational acceleration.
4 The value of gravitational acceleration is important in physics and is Figure 2.65 Gravitational
F4/2/80 force
specially denoted by the symbol, g. is always acting
5 However, the value of g varies slightly according to location. towards the centre
6 The value of g depends on: of the Earth
F
O (a) Latitude
2

R Generally, the value of g increases with latitude.


CHAPTER

M
g = 9.832 m s –2
4
North Pole (90° N)
rp = distance from the North Pole to
rp < re the centre of the Earth
re = distance from the Equator to the
re F centre of the Earth
g = 9.780 m s –2
Equator (0°) O  
2

R
CHAPTER

M
    
4 Figure 2.66(a)
F4/2/81a
The elliptical shape of the Earth causes the gravitational acceleration to vary with latitude.
The value of g is smaller at the equator than at the poles because the surface of the Earth is further
from the centre of the Earth at the equator than at the poles.
(b) Height above sea level
The greater the height above the sea level is, the smaller is the value of g.
g1 < g2
g2
Mount Everest

New Delhi (same latitude as


Mount Everest but nearer to
the centre of the earth)

Figure 2.66(b)
F4/2/81b

1 Air resistance depends on the shape or cross-


sectional area of the falling object.
The figure shows that a paper, repeatedly folded and
sealed tightly with a cellophane tape, falls faster than
crumpled
a loosely crumpled piece of paper (together with paper
cellophane tape) of the same mass. folded
2 Air resistance also increases with the velocity of paper
Physics
the falling object. An objectBlogmoving at a higher
velocity encounters greater air resistance than a
similar object moving at a lower velocity.
F4/2/82

Forces and Motion 100


Activ To determine the value of gravitational acceleration, g
ity 2.5
Apparatus/Materials
Ticker-timer with carbonised tape, power supply, retort stand, slotted weights with holder, G-clamp, cellophane
tape and soft board.
Arrangement of apparatus
G-clamp

ticker-timer
ticker
tape
F
O

2
12 V a.c. R
power supply ticker tape

CHAPTER
M
stool folded and
stapled
weight here 4

G-clamp
soft board

Figure 2.67
Procedure 2 Calculation of the value of g:
Initial velocity, u
1 A ticker-timer is clamped to a retort stand with a
x cm x
G-clamp and placed on a tabletop as shown in = = cm s–1
2 × 0.02 s 0.04  
Figure 2.67.
2 One end of a carbonised ticker tape Final velocity, v
(approximately 1.5 m in length) is attached to a y cm y
= = cm s–1
weight holder with a total mass of 200 g. 2 × 0.02 s 0.04  
3 The other end of the ticker tape is passed Time taken, t = (6 – 1) strips × 0.04 = 0.20 s
through the ticker-timer. Gravitational acceleration, g is calculated by
4 A soft board is placed on the floor below the substituting the values of u, v and t into the
weight to stop its fall. v–u
5 The ticker-timer is switched on and the weight formula g = .
t
is released so that it falls squarely onto the soft
board. Discussion
6 Six strips are cut off from the middle section of 1 If the activity is repeated with weights of mass
the ticker tape with each strip containing 2 dot- 250  g, 300 g, etc, the calculated values of g
spaces. would be the same. This shows that gravitational
7 A tape chart is constructed. From the chart, the acceleration, g is not affected by the mass of the
acceleration of gravity is calculated. object.
Results 2 It is found that the value of the gravitational
acceleration, g obtained from this activity is less
1 Ticker tape chart:
than the standard value of 9.8 m s–2. This is
because the weight (object) is not actually free
falling. It is affected by the following:
y cm
(a) Air resistance during its fall.
(b) There is friction between the ticker tape and
Activity 2.5

x cm the ticker-timer.
Conclusion
t A free-falling object falls with gravitational
Figure 2.68 acceleration, g of 9.8 m s–2.

101 Forces and Motion


Weight and Gravitational Acceleration SPM
’09/P1

1 The weight of an object is defined as the force


of gravity which is exerted on it by Earth.
2 From the formula: Since acceleration is proportional to the force acting,
a student may think that a sphere with a weight 10
F = force and times the weight of a smaller sphere would fall with
F = ma
a = acceleration an acceleration 10 times that of the lighter one.
Since weight, W, is the force of gravity acting
on an object of mass, m that makes it fall with It is true that the acceleration of an object is directly
an acceleration, g, therefore, using the proportional to the force acting on it. However, at the
corresponding terms, we get: same time, acceleration is also inversely proportional
F to mass. Therefore, the effect of greater force is
O F = m a cancelled by the larger mass of the heavy sphere.
2

R
So all objects in free fall have the same acceleration
↓ ↓
CHAPTER

M
regardless of their masses.
W = m g Unit is the same
4
as force, N.
∴ Weight of object
∴= Mass of object × Acceleration due to gravity

FF
Gravitational Acceleration Oand
O Gravitational Field Strength
22

RR
1 A gravitational field is theMMregion in which an object experiences a
CHAPTER
CHAPTER

force due to gravitational attraction.


44
2 The gravitational field strength is defined as the ratio of the weight
to the mass of the object or weight per unit mass. 10 spring balance
20
measures the
Weight W gravitational
30

40

3 Gravitational field strength = = Unit = N kg–1 50


attraction
Mass m
60

70 (weight) on
the object
Rearranging the formula,

W = m × Gravitational field strength m

Compare this with the formula W = m × g: mg

W = m  ×  Gravitational field strength Earth


W  =  m  ×  g

∴  Gravitational field strength = g Figure 2.69

33
g = Gravitational
An object of 8 kg weighs 120 N on a planet. acceleration
Find the gravitational field strength on the planet. g = Gravitational field
strength
Solution
∴  g = 10 m s–2
W = mg
= 10 N kg–1
120 = 8g
120 N W = mg holds true when g
g =
8 kg is given in m s–2 as well as
N kg–1.
= 15 N kg–1

Forces and Motion 102


34
Calculate the weight of an astronaut with a mass of 100 kg
(a) on Earth,
1
(b) on the Moon where the gravitational acceleration is of that on Earth,
6
(c) in outer space.
[Take g = 10 m s–2]
Solution
Situation (a)  Surface of the Earth (b)  Surface of the Moon (c)  Outer space
F
O

2
R

CHAPTER
M

4
  Mass, m 100 kg 100 kg 100 kg
  Weight, W
W = mg Gravitational acceleration, g′ In the outer space, there is
= 100 × 10 1 no gravitational pull on the
= g
= 1000 N 6 astronaut.
1 ∴  g = 0
= × 10 W = mg
6
= 100 × 0
= 1.67 m s–2
= 0 N (no weight)
W = mg′
= 100 × 1.67
= 167 N

Mass and Weight


Notice that in Example 34, the weight, W varies but the mass, m of an object remains the same.
The differences between mass, m and weight, W:

Mass, m Weight, W

The mass of an object is the amount of The weight of an object is the force of
matter in the object. Definition gravity on the object.
The weight of an object varies with the
The mass of an object is constant
Changing of value magnitude of gravitational field strength, g
everywhere.
of the location.

A scalar quantity Physical quantity A vector quantity

A base quantity Type of quantity A derived quantity

kilogram (kg) newton (N)


SI unit

103 Forces and Motion


SPM SPM
’03/P1 ’07/P1

Remember our equations of motion with constant Some recommend changing the acceleration, a in the
acceleration: equations to g, but this is not necessary.
Simply bear in mind that while solving problems involving
•  v = u + at
free fall, the acceleration, a in the equations has a value of:
1 •  a = 10 m s–2 (for downward motion)
•  s = ut + at 2
2 ⇒  Since the object is accelerating downwards.
•  v2 = u2 + 2as •  a = –10 m s–2 (for upward motion)
⇒  Since the object is moving with deceleration.

F 36 SPM
’04/P1
O
Since W = mg,
2

R A coconut takes 1.5 seconds to fall to the ground.


the weight of 25 000 km
CHAPTER

M
an object What is
4 depends on mass weight (a) its speed when it strikes the ground?
20 000 km
the 1 kg g 1 N
1
(b) the height of the coconut tree?
gravitational
mass weight Solution
field strength, 15 000 km 1 kg 2N
g which g2 up = 0 Coconut falls from rest.
decreases as F ap = 10 m s–2
the distance of 10 000 km O
mass weight
2

the object 1 Rkg 10 N Coconut accelerating


CHAPTER

from the M downwards.


5 000 km g3
centre of the
4
Earth
increases.
Earth h
g1 < g2 < g3

35 t = 1.5 s
v=?
Wei Bo is a basketball player. His vertical leap is
( a) v = u + at
0.75 m. What is his take-off speed?
= 0 + 10 × 1.5
Solution = 15 m s–1
1
(b) s = ut + at2
2
1
h = 0 + × 10 × 1.5 × 1.5 = 11.3 m
2

0.75 m 37
After winning a game, a pitcher throws a baseball
s = 0.75 m vertically up with a velocity of 30 m s–1.
a = –10 m s–2 Wei Bo is moving upwards. (a) What is the time taken for the ball to reach the
v = 0 maximum height?
v2 = u2 + 2as At the highest point, (b) What is the speed of the baseball when it returns
0 = u2 + 2 × (–10) × 0.75 Wei Bo is at rest to his hands?
momentarily.
u2 = 15 (c) How long is the ball in the air before it comes
u = 3.87 m s–1 back to his hands?

Forces and Motion 104


Solution v2 = 900
(a) Consider the motion B v = ±30 m s–1+ve
from A to B (motion Since the ball is moving downwards (direction of

motion is opposite to its initial direction),
up):
u = 30 m s–1 ∴ v = –30 m s–1 –ve
a = –10 m s–2 The speed of the ball is 30 m s–1.
At the (c) When the ball returns to his hands, the
v = 0 highest A C
v = u + at point, displacement, s, is zero.
0 = 30 + (–10)t v = 0 u = 30 m s–1, a = –10 m s–2, s = 0
t = 3 s 1
s = ut + at2
2
The ball takes 3 s 1
0 = 30t + (–10)t2 when the ball
to reach the 2 is thrown. F
maximum height. O

2
5t2 – 30t = 0 R
(b)
When the ball returns to his hands, the

CHAPTER
t(t – 6) = 0 M
displacement, s, is zero. ∴ t = 0 when the ball returns 4
Consider direction upwards as positive. or t = 6 to his hands.
u = 30 m s–1, a = –l0 m s–2, s = 0
v2 = u2 + 2as ∴ The time taken for the ball to return to his
= 302 + 2(–10)(0) hands is 6 s.

Problems Involving F = ma and W = mg


(a) From Example 37, by neglecting air resistance, a ball
thrown at 30 m s–1 takes 3 seconds to move up. Lift
This is true as the deceleration is 10 m s–2. The
velocity decreases by 10 m s–1 for each second.
So after 3 seconds, the velocity is zero. reading on the scale shows
the normal reaction force
30 m s–1 20 m s–1 10 m s–1 0 m s–1 0
W = true weight = mg
1 s 1 s 1s W R = normal reaction force
exerted on the girl by the
(b) The velocity when the ball returns to its original platform of the scale
position is of the same magnitude as the initial R
velocity of the ball, but in the opposite direction.
weighing machine
(c) The time to descend to the original position is the
same as the time to ascend. Therefore, an
alternative method to find the total time in the air is Figure 2.70
F4/2/85
shown as follows. 1 When a girl stands on the platform of a weighing
Time to reach the highest point = 3 s scale, there are two forces acting on her:
∴  The total time in the air = 2 × 3 = 6 s (a) the girl’s weight, W (= mg) acting down­
at rest wards, and
t=3s 0 m s-1 0 m s-1 t=3s (b) the upward normal reaction force, R exerted
on her feet by the platform of the scale.
t=2s 10 m s-1 –10 m s-1 t=4s 2 The reading of the scale gives the value of the
normal reaction force, R.
3 Table 2.11 shows a different situation in the lift.
t=1s 20 m s-1 –20 m s-1 t=5s [Please take note that when the lift is
accelerating upwards or downwards, the
reading on the scale gives the ‘apparent’ weight,
t=0s 30 m s-1 –30 m s-1 t=6s
which is equal to the normal reaction force on
the feet of the girl.]

F4/2/84 105 Forces and Motion


Table 2.11
Situation Explanation As force vector

(a) Lift at rest or moves up or down at a constant Velocity is zero or constant R


velocity ⇒a=0

cable Applying Fnet = ma,


R
R – mg = ma
reading shows the
normal reaction R – mg = 0
force, R R = mg
0
acceleration, a = 0 mg
F
O
F net = 0 ∴ Reading on the weighing
The girl feels ‘normal’.
2

R scale gives the true weight.


F4/2/86b
CHAPTER

4 weighing machine
mg

F4/2/86a

FF
(b)  OO of a m s–2
Lift moves up at an acceleration Lift accelerates upwards R1
22

RR ⇒ Resultant force, Fnet on the girl


CHAPTER

M
CHAPTER

M
R1 cable
is upwards
44 ∴ R1 > mg
acceleration
=a
Applying Fnet = ma,
0 R1 – mg = ma
R1 = mg + ma

∴ Reading on the weighing scale mg


is increased by ma.
weighing machine The girl feels ‘heavier’.
F4/2/87b
mg

F4/2/87a

(c)  Lift moves down at an acceleration of a m s–2 Lift accelerates downwards R2


⇒ Resultant force, Fnet on the girl
cable
is downwards
∴  mg > R2

Applying Fnet = ma,


R2 0 mg – R2 = ma mg
R2 = mg – ma
The girl feels ‘lighter’.
acceleration
=a ∴ Reading on the weighing scale F4/2/88b
is decreased by ma.
weighing machine
mg

F4/2/88a

Forces and Motion 106


SPM
38 ’06/P1

A box of mass 1.6 kg is suspended from a spring Solution


balance hanging from the ceiling of a lift.
The reading on the spring balance
= The tension in the spring, T
10

(a) If the lift is stationary:


20

30

40

T = mg
50

60

70

= 1.6 × 10
T = 16 N
(b) When the lift is accelerating upwards: F
O
T1 – mg = ma

2
R
T1 = 1.6 × 10 + 1.6 × 2 Apparent weight (the

CHAPTER
W = mg M
= 19.2 N reading on the scale)
Figure 2.71 increases when object
accelerates upwards. 4
F4/2/203
What is the reading on the spring balance if
(a) the lift is stationary? (c) When the lift is accelerating downwards:
(b) the lift moves upwards at an acceleration of mg – T2 = ma
2 m s–2? T2 = 1.6 × 10 – 1.6 × 3 Apparent weight (the
= 11.2 N reading on the scale)
(c) the lift moves downwards at an acceleration of decreases when object
3 m s–2? accelerates downwards.
[Take g = 10 m s–2]

Help! I am floating!
What is the apparent weight of a girl on a weighing scale Solution
in a lift if the cable of the lift suddenly breaks? The girl, the weighing scale and the lift will free-fall if the
cable suddenly breaks.

⇒  a = g
R = mg – ma
= mg – mg
0
= 0

The apparent weight is zero and the girl experiences


‘weightlessness’. She feels as if she is ‘floating’.

Pulley System

1 A frictionless pulley serves to change the direction of a force.


2 The tension, T that results from pulling at the ends of the string or rope
has the same magnitude along its entire length.

(A)  A force pulling a mass over a pulley


In this situation, the tension T, is equal to the pulling force F, even if
the rope is slanting.

107 Forces and Motion


 boy is pulling a bucket filled with sand. The mass of the bucket with the sand is
A
3 kg. Find the tension in the rope if the bucket is
i(i)  stationary, or
(ii)  moving up with a constant velocity of 2 m s–1.
Solution
Whether the bucket is stationary or moving up with a constant velocity,
the acceleration, a = 0.
Applying Fnet = ma,
T – mg = ma
T – (3 × 10) = 0
T = 30 N
F
O  he boy increases his force to move the bucket upwards with an acceleration of
T
2

R
2 m s–2. Find the applied force, F.
CHAPTER

M
Solution
4
Since the bucket is moving upwards, T > mg.
Applying Fnet = ma,
T – mg = ma
T – (3 × 10) = 3 × 2
T =
F 6 + 30
O
= 36 N
2

R
Applied force, F = 36 N
CHAPTER

(B)  A pulley with two masses 4


1 The heavier mass will accelerate downwards while the lighter
one will accelerate upwards with the same magnitude.
2 The tension is not equal to the weight of either mass.

39
Two masses of 5 kg and 3 kg are connected to a rope Applying Fnet = ma,
which passes over a frictionless pulley. Mg – T = Ma
50 – T = 5a ……(i)

For the 3 kg mass, it moves upwards (T > weight mg).


Applying Fnet = ma,
T – mg = ma
T – 30 = 3a ……(ii)
(i) + (ii): 50 – 30 = 8a
8a = 20
a = 2.5 m s–2
Acceleration, a = 2.5 m s–2
Figure 2.72
Substitute the value of a into equation (ii).
Find the tension in the rope and the acceleration of T – 30 = 3 × 2.5
the 3 kg mass when the 5 kg mass is released. T = 30 + 7.5
Solution = 37.5 N
When the 5 kg mass is released, it moves downwards
Tension in the rope, T = 37.5 N
at an acceleration of a (weight Mg > T).

Forces and Motion 108


9 SPM
Clone
’07
For Example 39, if you need to find the acceleration
only, you can solve the problem by considering the two A stone falls from a height of 20 m. What is the time
masses together. taken for the stone to reach the ground?
The weight of 50 N overcomes the weight of 30 N. A 1.4 s C 2.8 s
Applying Fnet = ma, B 2.0 s D 3.6 s
50 – 30 = (5 + 3)a Since both the
20 = 8a masses are Solution
a = 2.5 m s–2 moving together. u = 0 m s–1, a = 10 m s–2, s = 20 m
1
Applying s = ut + at 2,
2
40 1
20 = 0 + (10)t 2
F
2 O

2
A 2 kg weight is connected by a rope to a 3 kg R
5t2 = 20

CHAPTER
wooden block. The rope passes over a smooth pulley M
t 2 = 4––
as shown in Figure 2.73.
t = 4 4
= 2 s
Answer B

10 SPM
Clone
’09

Figure 2.73 The diagram shows a


coconut falling from a
The weight is then released. Find the tension in the tree.
rope if a friction of 5 N acts against the wooden block. Which of the following
Solution graphs correctly shows
the relationship between
the gravitational force, F
and the mass, m, of the
coconut?

A
F C F

For the 2 kg weight, motion is downwards. m m


Applying Fnet = ma,
20 – T = 2a ……(i) B
F D F
F4/2/209
For the wooden block, motion is to the right.
Applying Fnet = ma,
T – 5 = 3a ……(ii)
m m
(i) + (ii):   20 – 5 = 5a
5a = 15 Comments
a = 3 m s–2
Substitute a into equation (ii). Gravitational force (weight), w = mg (i.e. F = mg).
T – 5 = 3 × 3 Since g is constant, so F  m.
T = 9 + 5 The corresponding graph is a straight line passing
= 14 N through the origin.
∴  Tension in the rope = 14 N Answer  C

109 Forces and Motion


2.8

1 An object weighs 60 N on the surface of the Earth If the distance between the water surface and his
where the gravitational field strength is 10 N kg–1. hands is 9 m, what is
What is the weight of the object on a planet where (a) the time, and
the gravitational field strength is 18 N kg–1? (b) the velocity
2 Mustafa steps off a 3.0 m high diving board and when the stone strikes the water surface?
drops to the water below. Find his velocity when his 5 A ball is thrown vertically upwards with a velocity of
leg touches the water. 20 m s–1. At what height will the ball be when its
3 Alvin tosses a coin up. The coin takes 1.2 s to return velocity reduces to half?
to his hand.
6 A student of mass 50 kg stands on a weighing scale
F in a lift. If the reading on the scale becomes 550 N,
O is the lift accelerating upwards or downwards?
2

R Find the magnitude of the acceleration of the lift.


CHAPTER

M
7 Find the tension in the rope and the acceleration of
4 the 2 kg weight in each of the following figures.
(a) (b)

Find F
(a) the initial velocity of the coin, O
2

(b) the maximum height of the coin. R


CHAPTER

M
4 A man releases a stone into a well.
4

8 A steel sphere is dropped from a height of 9 m on


SPM
the surface of the Moon. Given that the gravitational
Clone
’03 acceleration on the Moon equals
1
of Earth’s
6
gravitational acceleration, find the time taken by the
sphere to reach the surface of the Moon.

2.9 Analysing Forces in Equilibrium


SPM SPM
Forces In Equilibrium ’07/P1 ’08/P1

1 Forces in equilibrium or balanced forces have


been discussed in Section 2.5. air resistance, G
2 When forces are in equilibrium, the net
force, Fnet or resultant force, FR is zero. The
object will either be W=G

(a) at rest, or
(b) in motion with constant velocity.
3 Examples of two forces in equilibrium are constant
(a) a skydiver falling at a constant velocity, velocity
weight, W
(b) a book resting on a table.
       (a) A skydiver falling at constant velocity

Forces and Motion 110


normal reaction force, R
Addition of Forces and Resultant Force
R=W
1 In Figure 2.76(a), the two forces of 3 N and
4 N, pushing a wooden block of mass 2 kg on
a smooth surface in the same direction cause
weight, W = mg the block to accelerate at 3.5 m s–2.
(a) a = 3.5 m s-2

(b)  Book on a table


Figure 2.74

4 In this section, we shall study more about the


F
equilibrium of forces, including an object at O
rest on an inclined plane where three forces

2
R
are in equilibrium. The resultant force on the a = 3.5 m s-2

CHAPTER
M
object is zero.
4
normal reaction force, R (exerted by
surface on the object)
smooth surface
n

tio
rough fric (b) Same acceleration (effect) when a force of 7 N
surface
replaces the two forces 3 N and 4 N
Figure 2.76
weight, W = mg 2 Figure 2.76(b) shows that if the two forces are
replaced with a single force of 7 N, it will still
Figure 2.75 accelerate at 3.5 m s–2.
3 Thus, the effect of pushing the block with 7 N
5 A tilted surface is called an inclined plane. is the same as pushing it with 3 N and 4 N in
6 To understand better how three forces work in the same direction, i.e., the block accelerates at
equilibrium, we need to understand 3.5 m s–2.
(a) the resultant force of two forces, and 4 For the example discussed, 7 N is the resultant
(b) the resolution of a force. force of the combined forces of 3 N and 4 N.
5 The resultant force is defined as a single
force that will produce the same effect as the
two or more combined forces that it replaces.
To Find the Resultant Force
SPM
(I)  Two Parallel Forces ’09/P1

The resultant force is obtained by simple arithmetic.

Forces acting in the same direction Forces acting in opposite directions

= =

Subtract the magnitude of the smaller force by the


larger one (to find the difference in magnitude
Add the magnitudes of the two forces.
between the two forces).
The resultant force is in the same direction as the
The resultant force is in the direction of the larger
two forces.
force.
Resultant force, F = F1 + F2
Resultant force, F = F2 – F1

111 Forces and Motion


(II)  Two Non-parallel Forces (forces at an angle to each other acting at a point) Resultant force, FR
the tail of F1 to the

1 Simple arithmetic cannot be applied to find the resultant force of two


non-parallel forces.
For example, how to determine the resultant force of the two forces F1
and F2 acting at an angle θ to each other as shown in Figure 2.77?
2 Instead, we can determine the resultant force by drawing scaled Figure 2.77
diagrams using the two methods below.
SPM
Method (I) ’07/P1

The Triangle Method (Tail-to-Tip Method)


F Figure 2.78 shows a scaled diagram of the triangle of forces constructed to determine the resultant force of the
O two forces, F1 and F2, acting at an angle θ to each other (Figure 2.77).
2

R First, choose a suitable scale for the two forces, for example, 1 cm : 20 kN.
CHAPTER

M Then, follow the steps below to determine the resultant force. Complete the triangle by
connecting the tail of F1 to
4 Draw F2 according to the the tip of F2.
Draw the force F1 according scale, with its tail starting B B
4
to the scale. from the tip of F1.
1 3 FR
F2 F2
θ θ
O A F O O
F1 F1 A F1 A
O 2 5
2

R With the help of a protractor, To find the resultant force,


CHAPTER

M measure θ at A. measure the length OB and


the angle AOB.
4
Figure 2.78
F4/2/93

Method (II)

The Parallelogram Rule (Parallelogram of Forces)


Figure 2.79 shows a scaled diagram of the parallelogram of forces constructed to determine the resultant of two
forces, F1 and F2 acting at an angle θ to each other (Figure 2.77).
First, choose a suitable scale for the two forces, for example, 1 cm : 50 N.
Then, follow the steps below to determine the resultant force.
With the help of a protractor,
Complete the parallelogram Draw the diagonal of the
measure θ , then draw F 2 so
using a pair of compasses. parallelogram.
that the tails of both forces F 1
and F 2 start at O. 3 4
2 B
C B C
C

F2 FR
F2 F2

θ θ θ
A O A
O O A F1
F1 F1
5
1 This diagonal represents the
Draw the force, F 1 according magnitude and direction of the
to the chosen scale. resultant force, F R.

Figure 2.79
Note that the tails of both forces F1 and F2, and the tail of the resultant force, FR are all at the same point, O and
FR is in between F1 and F2.
F4/2/94
[Note: More students prefer Method (II) to Method (I).]

Forces and Motion 112


41
An ocean liner is towed into harbour by two tugs, A and B, exerting forces of 12 kN and 10 kN
respectively and the tow-ropes making an angle of 68°. Find the resultant force acting on the liner.

12 kN

68°

10 kN B
F
O

2
Figure 2.80 R

CHAPTER
M

Solution 4

Use a scale of 1 cm : 2 kN (if a scale of l cm : 1 kN is used, the diagram would be too large).

4
Complete the parallelogram by
using a pair of compasses.

Arc with a radius of 5 cm


C B and the centre of circle at C.

Arc with a radius of 6 cm


and the centre of circle
at A.

3
Draw a line, OC, of 5
6 cm to represent Draw the diagonal OB.
the 12 kN force 6 cm FR
from point O.
F2

6
Measure the angle after
drawing diagonal OB.

2 68°
Measure 68º from OA
38° F1
using a protractor.
O 5 cm A
1
Draw a line, OA, of 5 cm to
represent the 10 kN force.

The resultant force, FR, is represented in both magnitude and direction by the diagonal OB.
Length of OB = 9.2 cm
∴  Magnitude of the resultant force, FR = 9.2 × 2 kN
= 18.4 kN
∠AOB = 38°
The ocean liner is pulled forward by a resultant force of 18.4 kN at an angle of 38° to the 10 kN force.

Note:  There is no need to draw the scaled diagram following the original directions of F1 and F2 as long as the
angle between them is correctly measured.
[Note: If you prefer to solve the problem by calculation, please refer to Example 48 in page 117.]

113 Forces and Motion


(III)  Two Perpendicular Forces Figure 2.81 shows the two forces (F1 and F2)
together with the resultant force, FR using the
This is a special situation for two non-parallel forces parallelogram rule.
acting on an object at a right angle to each other. The resultant force is obtained using Pythagoras’
theorem.

•  Resultant force, FR = F12 + F22


FR
F2 F2 F
• tan θ = 2
F1

θ
F1 Note: You can also solve the problem by drawing a scale
F diagram as discussed in section (II).
O Figure 2.81
2

R
CHAPTER

4 42
Samy and Heng Gee pull a crate with forces of 70 N Solution
and 90 N respectively. A sketch of the forces is drawn.
Heng Gee
F
O
2

R 70 N FR
CHAPTER

M
F2 = 70 N θ
4
90 N

F1 = 90 N Resultant force, FR = 702 + 902


= 114 N
70
tan θ = = 0.7778
Samy The incorrect
90 way to determine the resultant force
using the parallelogram rule:
Figure 2.82 tan θ = 37.9°
D force is 114 N C
The resultant which acts in a direction
Find the resultant force on the crate due to these two 37.9° from the original 90 N force.
forces.
F2 FR
BD represents the
resultant force.

A F1 B

The incorrect way to determine the resultant force Remember:


using the parallelogram rule: To find the resultant force using the parallelogram
rule, the tails of the three forces FR, F1 and F2 must
D C
originate from the same point.
D C
F2 FR
BD represents the
resultant force. F2 FR
AC represents the
resultant force.
A F1 B

A F1 B

Remember:
Forces and Motion 114
To find the resultant force using the parallelogram
rule, the tails of the three forces FR, F1 and F2 must
originate from the same point.
Resolution of Forces SPM 5 The magnitudes of the vertical and horizontal
’03/P2
components can be determined with
1 Since two forces can combine to give a single knowledge of simple trigonometry. We need to
resultant force, therefore by reversing the know the magnitudes only since the direction
process, a single force can also be resolved or is already known.
broken up into two components. Refer to Figure 2.83(b).
2 This reverse process is known as the
resolution of forces. For ∆ OAB: For ∆ OBC:
3 Usually, a force is resolved into components Fx Fy
cos θ = sin θ =
which are perpendicular to each other. F F
4 Figure 2.83(a) shows a single force, F. Applying
the parallelogram rule, the force F is resolved ∴ F = F cos θ ∴ F = F sin θ
x y
    F
into a pair of perpendicular components: the
O
vertical force, Fy and the horizontal force, Fx.

2
R

CHAPTER
M
C B
θ
4
Fy = F sin θ
F Fy F Horizontal component, Fx = F cos θ, and vertical
component, Fy = F sin θ, are not fixed formulae.
θ θ It depends on the position of θ.
O Fx A Fx = F cos θ Generally, you can take the component at the given
angle θ as F cos θ and the other component as F sin θ.
(a) (b) (c) Please study Example 43 carefully.
Figure 2.83

43
Find the horizontal and vertical components for each To find the vertical component:
of the following forces. Fy = F sin θ
= 80 × sin 60°
120 N = 80 × 0.866
60°
80 N = 69.3 N upwards
(b)
60° 120 sin 60°

     (a) (b) 120 N


60°
Figure 2.84
120 cos 60°
Solution
(a)
θ is 60° from vertical line.
80 N
80 sin 60° To find the horizontal component:
60°
Fx = F sin θ
= 120 × sin 60°
80 cos 60°
= 120 × 0.866
= 103.9 N to the left
θ is 60° from horizontal line.
To find the horizontal component: To find the vertical component:
Fx = F cos θ Fy = F cos θ
= 80 × cos 60° = 120 × cos 60°
= 80 × 0.5 = 120 × 0.5
= 40 N to the right = 60 N downwards
* Please take note of the position of the angle .

115 Forces and Motion


44
A boy is pulling a toy car with a force of 6 N as shown Solution
in Figure 2.85.
6 sin 30°
6N

30°

6 cos 30°

The horizontal component, Fx = 6 cos 30°


F Figure 2.85 = 6 × 0.866 = 5.2 N
O Since the toy car is in dynamic equilibrium (a = 0), the
Find the horizontal component of the force. If the toy
2

R horizontal force on the toy car is balanced by the friction.


car is moving with constant velocity, what is the
CHAPTER

M ∴  Friction = 5.2 N
friction against the toy car?
4

45 SPM
’03/P1

Santhiran pulls a 5 kg crate on the Ffloor with a force of 35 sin 25°


O
35 N. The friction against the box is 8 N. 35 N
2

R
25°
CHAPTER

M
8N 35 cos 25°
4 8N
35N

25°
8N
Since the crate moves along the floor, only the
horizontal forces need to be considered.
Figure 2.86
Applying Fnet = ma,
What is the acceleration ofF4/2/97
the crate? 35 cos 25° – 8 = 5a
Solution 23.7 = 5a
a = 4.7 m s–2
First, resolve the pulling force into its horizontal and
vertical components.

46
A gardener pushes a (c) If the gardener pulls the handle with the same
250 kg concrete roller force in a reverse direction, what is the total
with a force of 200 N downward force on the ground then?
directed from the handle. 33° (d) In which direction, pushing or pulling, would the
200 N
The handle is at an roller be more effective in levelling the grass?
angle of 33° to
Solution
the horizontal.
Figure 2.87 Resolve the force into horizontal and vertical components.
F4/2/98
(a) Find the horizontal and vertical components of the
force.
(b) What is the weight of the concrete roller?
What is the total downward force on the ground?

Forces and Motion 116


(a) To find the horizontal component: Resultant vertical force on the ground
Fx = F cos θ = Weight + Vertical component
= 200 × cos 33° = 2500 + 109 = 2609 N
= 200 × 0.8387 (c) When the gardener pulls the roller:
= 168 N to the right The vertical component of the force
To find the vertical component: = 109 N upwards
Fy = F sin θ Resultant vertical force on the ground
= 200 × sin 33° = Weight – Vertical component
= 200 × 0.5446 = 2500 – 109 = 2391 N
= 109 N downwards (d) When the gardener pushes the roller, the net
(b) Weight, W = mg = 250 × 10 downward force is greater. This makes it more
= 2500 N effective in levelling the grass.
F
O

2
47 R

CHAPTER
M
Eric and Darren are pulling a trolley on which their What is
sister, Tina is sitting. Both Eric and Darren pull the (a) the total of the forces towards the left? 4

rope with a force of 50 N, which makes an angle of (b) the acceleration of Tina and the trolley?
30° with the direction of motion.
Solution
Resolving the tension forces into components.
a
50 N 50 sin 30°
50 cos 30°
30° 40 N
40 N
30°
50 cos 30°
50 N 50 sin 30°

(a) We need only consider the horizontal components


of the forces.
Total forces to the left
= 2 × 50 cos 30° – 40
Figure 2.88 = 2 × 50 × 0.866 – 40 = 46.6 N
(b) Fnet = ma
The total mass of Tina and the trolley is 30 kg and the 46.6 = 30a
friction between the trolley and the floor is 40 N. a = 1.6 m s–2

To Find the Resultant Force of Two Forces at an Angle  to Each Other by Calculation

48
Find the resultant force 8N
First resolve the 8 N force into its vertical and
of the two forces as 120° horizontal components.
shown in Figure 2.89. Total vertical force = 8 sin 60° = 6.93 N
12 N
Total horizontal force to the right = 12 – 8 cos 60°
                      Figure 2.89 = 12 – 4 = 8 N
Solution F4/2/239 ∴ FR = 82 + 6.932 = 10.6 N
8N 8 sin 60° FR 6.93
tan θ = ____
6.93 N
60° 120°
8
12 N 8 cos 60° 12 N 8N
θ = 41°
∴ The resultant force is 10.6 N at an angle of 41° to
12 N.
F4/2/240

117 Forces and Motion


The Effective Components of a Weight on an Inclined Plane

1 Consider a box resting on an inclined plane at an angle θ to the horizontal.


2 The weight of the box is W = mg. The weight can be resolved into two perpendicular
components:
(a) the component vertical or perpendicular to the plane = mg cos θ,
(b) the component down or parallel to the plane = mg sin θ.

F θ
O
2

R θ W = mg
CHAPTER

F θ sθ
sin θ mg co
mOg
2

R W = mg
θ
CHAPTER

4
Figure 2.90

3 To solve problems involving inclined planes, weight is replaced with its two perpendicular
components. F4/2/99

Acceleration of an object on a smooth


Object in equilibrium on a rough inclined plane
inclined plane

• The figure below shows a box at rest on an • The figure below shows a box on a smooth
inclined plane. inclined plane.
normal reaction force, R
• For an object on a smooth inclined plane, the
vertical Rcomponents of the forces are balanced.
g
n vin n
oHowever, the force downne the plane is not balanced.
tm a pla
tio
F fric
n, jec ith n th
ict
io ob wn w ratio oo
fr do cele sm R
in θ ac a θ
gs sin vin
g
m mg mo h an ne
mg cos θ ctmg t
i cosnθ pla
je th
θ θ ob wn w ratio oo
do cele sm
ac a θ
• Since the box is at rest: sin
mg
The net force perpendicular to the plane = 0 mg cos θ
θ
R – mg cos θ = 0
Applying Fnet = ma,
The net force parallel to the plane = 0
mg sin θ = ma
Ffriction – mg sin θ = 0
a = g sin θ
Therefore: F4/2/100
If θ = 30°, If θ = 60°,
R = mg cos θ a = 10 sin 30° a = 10 sin 60°
Ffriction = mg sin θ = 5 m s–2 = 8.66 m s–2
∴ The greater the inclined angle, the greater its
acceleration.

Forces and Motion 118


49
A carton of mass 5 kg is at rest on an inclined plane making an angle of 15° with the horizontal.
Find the frictional force and the normal force acting on the carton.
Solution
l
ma
F nor FR
g
5k
t ion
fric

° 15°
15 50 cos 15°
s in 50
15°
W = mg 50 15°
F
O

2
Friction, FR = mg sin θ Fnormal = mg cos θ R
= 5 × 10 × sin 15°

CHAPTER
= 5 × 10 × cos 15° M
= 50 × 0.2588 = 50 × 0.9659 4
= 12.9 N = 48.3 N

50
Figure 2.91 shows a horizontal force, F acts on a wooden block of weight 8 N placed on a smooth plane inclined at
30° to the horizontal.
ne
pla
o th
F smo

30° W=8N

Figure 2.91

What must the magnitude of F be to prevent the block from sliding down?
Solution
Resolving the force, F and W(= mg) parallel to the plane.
(The vertical component does not play a role because when the block is in motion, it will only move along the
plane.)
°
30
F os
Fc °
30° 30
in
30° 30° m gs
os sin
F c mg 30° W = mg 30°
Forces along the plane

F cos θ = mg sin θ
F cos 30° = 8 sin 30°
8 × 0.5
F =
0.866°
= 4.6 N

119 Forces and Motion


Three Forces in Equilibrium

Problems involving three forces in equilibrium can be solved either by:


(a) Method A: Resolution of forces
(b) Method B: Drawing a closed triangle of forces

51
A 6 kg weight is suspended from a hook in the ceiling
as shown in Figure 2.92. A horizontal string pulls the T1
F supporting string so that the latter makes an angle of
O 50° T2
50° with the horizontal. Find the tensions in both
2

R
strings.
CHAPTER

4
6 kg

60 N

Figure 2.92
F
Solution
O
2

Method A:  Resolution of forcesR Method B:  By drawing a closed triangle of forces
CHAPTER

M
The slanting force, T1 is resolved as shown. Starting with any one of the three forces, a triangle
4 of forces is drawn. Let’s say you start with OA
which represents the weight, 60 N.
T1 sin 50°
T1 a Draw the line OA with an arrow, to represent
T2 T1 cos 50° T2 60 N.
50°
b Starting from A, draw line AB which represents
60 N 60 N T2.
c Then, starting from B, draw the line BO which
represents T1 to complete the triangle. The
directions of the three forces follow an
Since the weight is in equilibrium, therefore:
anticlockwise direction. However, if you draw
Vertically:
in sequence 60 N, T1 and T2, you’ll get a triangle
of forces that show a clockwise direction.
Total force upwards = Total force downwards O O O
a b c
T1 sin 50° = 60
60 T1
T1 = 60 N 60 N
sin 50° 60 N
50°
60 A A B A B
= T2 T2
0.7660
After drawing the triangle, use simple trigonometry
= 78.3 N
to solve the problem.
Horizontally: 60 60
= sin 50° = tan 50°
T1 T
2
Total force to the left = Total force to the right 60 60
T1 = T2 =
T1 cos 50° = T2 sin 50° 50°
tan
T2 = 78.3 × 0.6428 60 60
= = 78.3 N = = 50.3 N
= 50.3 N 0.7660 1.192

Forces and Motion 120


52
Joan and her sister lift a pail of water of total mass
5.6 kg. Each of them exerts a force of F at 25° to the F cos 25° F cos 25°
25° 25°
vertical. Find the magnitude of F. F F F sin 25° F sin 25°

56 N 56 N
F F
25°
25°
We need to consider the vertical components of the
forces only.
F
Since the pail of water is in equilibrium:
O
Total force upwards = Total force downwards

2
R
2F cos 25° = 56

CHAPTER
M
W = mg 56 W = mg
F = 4
2 cos 25° = 5.6 × 10
Figure 2.93
F4/2/102 = 56 N
= 30.9 N
Solution Note: Method B is not suitable for this question as
Resolve F into its vertical and horizontal components. there is no right angle involved.

53
A lamp of weight 25 N is supported by two ropes as shown
in Figure 2.94. F
25°
F
Given that the tension in rope A is 20 N, find 25°
(a) the angle θ,
(b) the tension in rope B.
Solution
Resolve the forces into their components.
W = mg

T F4/2/102
T cos θ 20 cos 30° Figure 2.94
30° 20 N
θ

T sin θ 20 sin 30°

25 N 25 N

(a) For horizontal component: (b) Substitute θ = 52.5° into equation (i).
T sin θ = 20 sin 30° F4/2/253
T sin 52.5° = 10
T sin θ = 10 ……(i) 10
T =
sin 52.5°
For vertical component:
T cos θ + 20 cos 30° = 25 = 12.6 N
T cos θ = 25 – 20 cos 30° ∴ Tension in rope B = 12.6 N
T cos θ = 7.68 ……(ii)
T sin θ 10
(i) ÷ (ii): T cos θ = 7.68

tan θ = 1.302
θ = 52.5°

121 Forces and Motion


Each elephant in the diagram below is pulling a rope Vertical component:
attached to a mass of 10 kg. The elephants pull with a 2 × 5000 cos θ = 100
force of 5000 N. 100
cos θ = 2 × 5000

= 0.01
5000 N
θ = 89.4°
θ θ 5000 N The angle between the rope, 2θ = 2 × 89.4° = 178.8°

It is impossible to make the rope horizontal.


10 kg
See the explanation below.
F Let T = tension in this rope.
O Find the angle between the ropes. 2T cos θ = 100
2

R Can the ropes be pulled to become horizontal? Supposing θ = 90° The rope is
horizontal.
CHAPTER

M 100
Solution T =
2 cos θ
4 Resolve the forces into their components:
50
5000 cos θ 5000 cos θ =
cos 90°
θ Value cannot be
θ
50 determined.
5000 N 5000 N =
0
5000 sin θ 5000 sin θ
F
Note: You may try to use θ = 89.99999° and see what is
O
the value of tension in the rope, T.
2

100 N 100 N
R
CHAPTER

F4/2/103 4

11 SPM
Clone
’04

The figure below shows a poster hanging on a wall. Comments


nail First, indicate the directions of tension forces along
tension,T1
the string. Since the poster is in equilibrium, the
tension,T2 three forces should then form a closed triangle of
forces.

T1
T2

T1

weight, W
W
T2
Which of the following triangles of forces shows the
forces, T1, T2 and W acting on the poster? W

A B C D
Remember:
Draw the second force from the tip of the first force.
F4/2/259
Then, draw the third force from the tip of the second
force to complete the triangle.
Answer  A

Forces and Motion 122


12 SPM
Clone
’06

A mirror of weight 12 N is hung on the wall using a Solution


string as shown below. 3
A
Measure 30°, same
reasoning as 2 . Then
30° draw line AC to intersect
1 with BC.
Draw a vertical
T T line of 6 cm (to T
represent the
weight of 12 N).
60° 60°
6 cm
(12 N) C
F
O

2
R

CHAPTER
T M
2
30° Measure 30° using a 4
protractor (since the string
Draw a scale drawing of a triangle of forces to makes 30° to the vertical).
Draw a line BC.
determine the tension, T in the string. B
(Use the scale 1 cm : F4/2/104a
2 N)
Measure the length of BC.
BC = 3.4 cm F4/2/104b
∴ Tension, T = 3.4 × 2 N = 6.8 N

2.9

1 Two forces of 60 N and 80 N act on an object.­ 4 In a circus, a monkey is pulling his chain with a
Find forward force of 20 N as
(a) the greatest and the smallest forces that can be shown in the figure.
exerted,
(b) the resultant force if the force 80 N is acting to
the right and is at right angles to the 60 N force,
(c) the resultant force if the force 80 N is acting to
the right and is at 120° to the 60 N force.
2 Two astronauts push a
800 kg satellite with 45 N

forces of 35 N and 45 N What force does the clown need to apply to the
which make an angle of chain to prevent the monkey from moving forward?
35 N
60° with each other. 5 A footballer is injured and requires traction on his
Find the resultant force foot as shown in the figure below.
and the acceleration
acquired by the satellite.

3 Aunty Anne is
ironing with a
force of 6 N as
shown in the
figure.

What is the effective force that pushes the iron If the force of the traction is 125 N, what is the mass,
forwards? m required to pull the rope?

123 Forces and Motion


6 A boy exerts a force of F to pull a box of mass 2 kg 8 A wooden block is placed on top of a smooth
up an inclined plane which makes an angle of 30° runway of length 2 m. The angle of inclination with
with the floor. the floor is 30°. Find
(a) the time and
(b) the velocity of the block
when it reaches the bottom of the runway.

F
9 A bird of mass 0.6 kg stands on the midpoint of a
tion horizontal rope. The rope sags and an angle of 150°
Fric N is formed.
= 3
30°

Given that the friction acting on the box is 3 N, find 150°


F
O (a) the normal reaction force, R acting on the box,
2

R (b) the component of the weight down the plane,


CHAPTER

M (c) the value of F if the box is moving up the plane


with an acceleration of I m s–2.
4 Find the tension in the rope.

7 A 3.5 kg flowerpot is supported by two ropes as


shown in the figure. The rope attached to the wall is 10 A traffic light of mass 15 kg is suspended from
horizontal and the rope attached to the ceiling two cables as shown in the figure.
makes an angle of 30° with the horizontal.
F
40° 50°
O
40°
2

R 50°
CHAPTER

M T2
T1

m = 15 kg

Find the tension in each rope. Find the tension in each cable.

2.10 Understanding Work, Energy, Power and Efficiency


2.10 Understanding Work, Energy, Power and Efficiency
SPM
Work ’03/P1

1 Work is done when a force causes an object to move in the direction of the force.

F F
s
F

s

(a)  A manF4/2/105a
pushing a bicycle (b)  A weightlifter lifting a barbell
Figure 2.95 F4/2/105b

Forces and Motion 124


2 The work done, W is defined as the product 4 One joule is the work done when a force of
of the force, F and the displacement, s in the one newton moves an object over a distance of
direction of the force. one metre, in the direction of the force.
Displacement in the 1m
Work done = Force × 1N
direction of the force

∴ W = F × s
Figure 2.96
F4/2/106a

3 The SI unit of work is the joule, J. ∴  1 J = 1 N × 1 m = 1 N m


Work is a scalar quantity.

Work = Force × Displacement (in the direction of the force) F


O

2
R

CHAPTER
M
1   Force, F in the same direction as the 2   Force, F at an angle  to the displacement, s
displacement, s 4
F sin θ
F F
F F θ θ F cos θ

s s

The object moves over F4/2/106


a distance
b of s, in the The object does notF4/2/106
move in c the direction of F.
direction of the force. Instead, the horizontal component of the force,
F cos , moves the object on the surface of the floor.
W = F × s Work done, W
Horizontal Displacement in the
= component × direction of the force

= F cos θ × s

∴ W = Fs cos θ

54
55
A worker in a hypermarket pushes a trolley full of
goods with a horizontal force of 120 N for a Man Ling pulls a luggage with a force of 30 N that is
displacement of 15 m. What is the work done by him? applied along the handle at an angle of 40° to the
horizontal. How much work has she done after
Solution walking a distance of 15 m horizontally?
Solution
120 N 120 N

N N
30 30
15 m 40°

The force and the displacement


F4/2/107 are in the same
15 m
direction.
∴  Work done, W = F × s
= 120 × 15 Work done, W = Fs cosF4/2/108
θ
= 1800 J = 30 × 15 × cos 40° = 345 J

125 Forces and Motion


SPM
No work is done when … ’08/P1

1   Force, F does not move 2 Force, F is at a right angle (perpendicular) to


(a) James stands still for the displacement, s
30 minutes holding some F = 20 N
A waiter walks a distance of 5 m holding a tray of
books of 20 N. What is the food that weighs 10 N. How much work has the
work done on the books? waiter done to the tray?
direction of motion
W = 20 N
F = 10 N

F
O (b) Nicholas pushes a concrete wall in his
2

R
classroom with a force of 20 N for 30 minutes.
CHAPTER

M weight = 10 N
How much work does he do on the wall?
4 wall
5m

20 N
Solution
The waiter exerts an upward force of 10 N while he
F
O is holding the tray. When he walks forward a
2

R distance of 5 m, the tray is not displaced upwards or


CHAPTER

M downwards – that is, the displacement in the direction


Solution of the force is zero.
4
Work done, W = F  × 0 = 0
    Work done, W = F × s
= 20 × 0 Alternative method
=0 Use the formula: W = Fs cos θ
Both James and Nicholas will get tired but there F = 10 N, s = 5 m, θ = 90°
is no work done on the books or the wall as the ∴  W = Fs cos θ = 10 × 5 × 0 = 0
force does not move. This means that there is no work done to the tray.

Work Done against the Force of Gravity


56
1 An upward force must be applied to lift an object
Auntie Sarimah mops
of weight, mg newton, to a height of h metres.
the floor by using a
force of 9 N at an angle 4m
F (= mg)
of 60° to the floor. 60°
How much work has
she done after mopping 9N
a total distance of 4 m?
h
weight
= mg

                       Figure 2.97
Figure 2.98
Solution 2 The work done is the same as the force
The angle between the force and displacement is 60°. multiplied by the distance, in the direction of
Work done, W = Fs cos θ the force–that is,
= 9 × 4 × cos 60° where F is equal to and
Work done = F × h opposite in direction to
= 18 J W = mg × h the weight, mg.

Forces and Motion 126


57
A carton of weight, mg newton, is lifted to the top of a Solution
flight of stairs. (a) Work done
Displacement in the
= Force × 
F direction of the force
W =  mg × h
=  mgh
h

Reminder
Work done ≠ mg ×  or mg × b because the
mg
force, F (= mg) acts vertically over a height h. F
b
The work done is not dependent on the path O

2
Figure 2.99 taken i.e.  or b, but on the vertical height R

CHAPTER
M
reached.
(a) What is the work done? F4/2/111
(b) If the mass of the carton is 2 kg and the height of 4
the stairs is 60 cm, calculate the work done by the (b) If m = 2 kg, g = 10 m s–2, h = 60 cm = 0.6 m
worker. [Take g = 10 m s–2] ∴  The work done, W =  mgh
F
= 2 × 10 × 0.6
=  12 J
h


mg SPM
Energy ’06/P1
b

1 Energy is defined as the capacity to do work.


2 The SI unit of energy is the joule, J. It has the same unit as work.
3 Energy is a scalar quantity. F4/2/111

Gravitational potential Elastic potential energy is


Heat energy is a type of
energy stored in an object Kinetic energy is the the energy stored in a
energy associated with
due to its position. energy that a body has due compressed/stretched spring
temperature.
to its motion. or elastic material.

FORMS OF ENERGY

Chemical energy is a stored


Sound energy is the energy which is released Nuclear energy is the Electrical energy is the
energy created when an during a chemical reaction. energy released by a energy of electric charges
object vibrates. Batteries and food store nuclear reaction. or electric currents.
chemical energy.

127 Forces and Motion


Energy Transfer and Work Done

1 Energy is transferred from one object to another when work is done.

A weightlifter displaces a barbell from the ground to a A pitcher throws a baseball towards a batter.
height above his head. Work is done to lift the barbell.

F
O Chemical energy  Kinetic energy
2

R Chemical energy  Potential energy


The work done transfers the chemical energy in the
CHAPTER

M
The chemical energy (stored in food) in the F4/2/113
pitcher’s body to the baseball. The chemical energy
4 weightlifter is transferred to the barbell which is converted to kinetic energy due to the motion of
possesses energy due to its position, that is, the the ball.
gravitational potential energy.


F
2 Work done is the medium forO transferring energy from one object to another.
2

R
CHAPTER

M
Potential Energy
4
1 The potential energy of an object is defined as the energy stored in the object because of its
position or its state.
2 There are two types of potential energy.

1   Gravitational potential energy 2   Elastic potential energy


Example: A ball raised above the floor Example: A wound coil-spring in a toy car

final position

(a) Before winding (a) After winding


ball
Elastic potential energy will be discussed in
mass = m kg initial position Section 2.12.

SPM
Gravitational Potential Energy ’04/P1 F

1 A boy is lifting a box of mass m kg at a


constant velocity to a height, h metres above
the Earth’s surface.
2 Since the box is moving with constant
velocity, its acceleration is zero. mg
3 Thus, the net force acting on the box is zero. h
This means that the upward force, F is equal to
the weight of the box (= mg).
F = mg (in magnitude)
Figure 2.100
F4/2/114
Forces and Motion 128
4 Since the force moves a displacement of h to
lift the box, work is done by the force. 58
Displacement A pole-vaulter of mass 55 kg clears the bar at 6.0 m.
Work done, W = Force × in the direction What is his potential energy at the highest point?

of force [Take g = 10 m s–2]
= F × s Solution
= mg × h
= mgh

5 What has happened to the work done?


It has changed into one form of energy which, 6.0 m
in this case, is the gravitational potential F
energy, Ep stored in the box because of its O

2
vertical position above the ground. R

CHAPTER
M
W  Ep
Ep = W 4
= mgh Potential energy, Ep =  mgh
= 55 × 10 × 6
∴ Ep = mgh =  3300 J

6 Figure 2.101 shows a ball of mass m displaced


from position B to C. The h represents the Kinetic Energy
vertical height.
1 Kinetic energy, Ek, is the energy possessed by
an object due to its motion.
m kg
2 Consider a force F acting on a trolley of mass
m kg on a frictionless plane over a distance of s.
C The force accelerates the trolley from an initial
velocity, u (= 0 m s–1) to a velocity, v m s–1.
F
s
h

B A Figure 2.102

Figure 2.101 3 What happens to the work done, W = F × s?


Since
m kg the trolley possesses kinetic energy, Ek
(because of its motion), which is acquired
When the ball is displaced from position B to because of the work done, we can reasonably
C
C, work is done. conclude that the kinetic energy of the trolley
Work done, W = F s cos θ F s is transferred from the work done.
=F×h h

= mgh θ Ek = Work done


=F×s F = ma
θ is the angle betweenBthe force A= mas
vector and the displacement vector. v 2 = u2+ 2as
Vertical height, h = s cos θ (v 2 – u2) (v 2 – u2)
=m as =
2 2
1 2
= mv u=0
7 Therefore, an object raised to a height, h 2
possesses a gravitational potential energy,­ 1 2
Ep = mgh, which is independent of the path ∴ Ek = mv
2
taken by the object.

129 Forces and Motion


59
A pitcher throws a baseball of mass 145 g with a
m m
speed of 20 m s–1. What is its kinetic energy?
v v' = 2v

Solution
Momentum p = mv p = mv
Kinetic energy, Ek = 2 mv
= 2p
1
= mv 2 u=0 Kinetic Ek = 1 mv2
E  = 1 m(v)2
2 energy
2 2
1 2
= 2 m(2 v)
1
= × 0.145 × 20 × 20 145 g = 0.145 kg
F
O
2 = 4 1 mv2

2 ( )
2

R = 29 J = 4 Ek
CHAPTER

F
O
2

Work is equivalent to energy as they are


R interchangeable.
(a) Work done = Energy acquired, and
CHAPTER

M
(b) Energy = Work done
4

Principle of Conservation of Energy

1 Figure 2.103 shows a coconut of mass, m kg 3 As the coconut falls to the ground, the
falling from a height of h metres to the gravitational potential energy, Ep decreases
ground. while the kinetic energy, Ek increases due to its
increasing velocity.
4 However, the sum of kinetic and potential
energy remains constant as the coconut falls.
5 When the coconut reaches the ground, all its
100% Ep
initial potential energy has been changed into
kinetic energy.
6 This is a typical example of the conservation
of energy.
h 50% Ep 7 The Principle of conservation of energy
50% Ek states that:

Energy cannot be created or


destroyed. It can be transformed
100% Ek
from one form to another, but the
total energy in a system is constant.
Figure 2.103

2
Before falling, the coconut possesses 8 Figure 2.104 shows another example of the
gravitational potential energy, EP = mgh. Since conservation of energy by an athlete during a
the coconut is at rest, its kinetic energy, Ek is zero. pole-vaulting event.

Forces and Motion 130


N
bar Ep(gravitational)

M
P

v Ep(elastic)

Ek
Ek(kinetic) L
Q
J K

F
O
Ek of running athlete → Elastic, EP of the bending pole → Gravitational potential energy, EP → Ek of athlete

2
R

CHAPTER
Figure 2.104  Transformation of energy during a pole-vaulting event M

Solution 4
60
1 1
(a) Kinetic energy, Ek = mu2 = × 2 × 102
A durian falls from a height of 6 m. What is the 2 2
velocity of the durian just before it strikes the = 100 J
ground? [Take g = 10 m s–2] (b) The kinetic energy is used to overcome the
Solution frictional force.
Initial kinetic Work done to
According to the Principle of conservation of energy: =
energy overcome friction
Kinetic energy gained = Potential energy lost
Ek = F × s
1 F = F rictional force
mv 2 = mgh 100 = F × 5
2 m is the
mass of F = 20 N
1
× v 2 = 10 × 6 the durian. (c) Heat energy gained = Kinetic energy lost = 100 J
2
v = 120
= 10.95 m s–1
62
The velocity of the durian before it strikes the ground
is 10.95 m s–1. Figure 2.106 shows a v=0

gymnast bouncing on a
trampoline. He moves
upwards at a velocity
of 8 m s–1. h=?
61 How high will he reach? u = 8 m s–1
A moving 2 kg object has an initial velocity of [Take g = 10 m s–2]
10 m s–1. It comes to a stop on a rough surface after
travelling a distance of 5 m.
10 m s–1
stop
Figure 2.106
2 kg

frictional force
rough surface Solution
5m
Potential energy gained = Kinetic energy lost
Figure 2.105 1
mgh = mv 2
Calculate 2
(a) the kinetic energy of the object, 1
10 × h = × 8 × 8
(b) the frictional force acting on the object, 2
(c) the heat energy produced. h = 3.2 m

131 Forces and Motion


63
At a children’s playground, Calvin of 30  kg mass Solution
climbs up a concrete slide of 2.3 m height. He slides (a) Change in potential energy
down the slope that has a length of 5 m. At the end of = mg∆h
the slope, which is 0.3 m above the ground, his = 30 × 10 × (2.3 – 0.3)
velocity is 1 m s–1. = 600 J
1
(b) Kinetic energy, Ek = mv 2
2
1
= × 30 × 1 × 1
2
F = 15 J
O (c) Let F be the frictional force. According to the
2

R
Principle of conservation of energy:
CHAPTER

M
Potential Kinetic Work done
+
4
energy lost = energy gained against friction
Figure 2.107
600 = 15 + (F × 5)
(a) What is his change in potential energy? 5F = 600 – 15
(b) What is his kinetic energy at the end of the slope? F = 585
(c) Find the average frictional F force against his   5 F = F rictional force
motion along the slope. [Take Og = 10 m s–2] = 117 N
2

R
CHAPTER

To pull or to lift?
The figure (a) shows a worker pulling a load of mass (c) Explain why the work done in (a) is greater than the
120 kg along a 3 m inclined wooden plane onto a lorry. work done in (b) even though the gain in gravitational
The tension in the rope is 600 N. potential energy of the load is the same.

Solution
worker lorry
(a) Pulling:
rope
Work done, Wa = F × s
600 N
load = 600 × 3
= 1800 J
3 m 0.9 m (b) Lifting vertically:
road
Work done, Wb = mg × h
(a) = 120 × 10 × 0.9
= 1080 J
(c) The work done in situation (a) is greater than that in
situation (b) because extra work is required to
overcome the friction between the load and the
0.9 m
inclined plane. The extra work done changes into
heat energy which dissipates to the surroundings.
(b) Extra info
(a) What is the work done by the worker to pull the load Then, why does the worker choose method (a)?
onto the lorry? Obviously, this is because he only needs to apply a lesser
(b) Instead of pulling, if the worker lifts the load vertically force to pull the load (600 N < 1200 N).
up a height of 0.9 m onto the lorry as shown in figure It is easier to raise a heavy load by pulling it along an
(b), how much work does he need to do? inclined surface than lifting it vertically. This is the function
[Take g = 10 m s–1] of an inclined plane.

Forces and Motion 132


Activ To show the principle of conservation of energy
ity 2.6
Apparatus/Materials Final velocity of the trolley and the weight just
before the weight hits the soft board, v
Ticker-timer with tape and power supply, thread, x
trolley, slotted weights with holder, smooth pulley =
  5 × 0.02 s
on a clamp, metre rule, triple beam balance, trolley
x
runway, soft board, and cellophane tape. = cm s–1
  0.1
2 Calculation:
Arrangement of apparatus Mass of trolley = M
Mass of weight = m F
ticker-timer Height of weight before its release = h O

2
trolley thread Final velocity of trolley and weight = v R

CHAPTER
pulley M
ticker (a) Loss of potential energy of the weight
tape
=  mgh 4

weight (b) Gain in kinetic energy of the trolley and


1
the weight = (M + m) v2
soft board 2
Figure 2.108 It is found that:
F4/2/293 1
(M + m) v2 = mgh
2
Procedure
1 The mass of the trolley, M is determined using Discussion
the triple beam balance. 1 The loss of potential energy of the weight is not
2 A 150  g weight (m) is connected by a thread, converted to kinetic energy of the weight only.
which passes over a smooth pulley, to the front Part of it is converted to kinetic energy of the
of the trolley. trolley.
3 A friction-compensated runway is prepared and
the apparatus is set up as shown in Figure 2.108. 2 As the trolley and the weight are moving with
4 The height of the weight from the soft board, h the same speed, the total kinetic energy gained
is measured with a metre rule. 1
= (M + m) v 2.
5 The ticker-timer is switched on and the trolley is 2
released.
6 From the ticker tape obtained, the velocity of Conclusion
the trolley and the weight just before the weight
When the weight falls, the loss of its gravitational
hits the soft board is determined.
potential energy is equal to the gain in kinetic
energy of the trolley and the weight. Energy is
Results therefore conserved. The principle of conservation
of energy is thus verified.
1 Analysis of the ticker tape:

after the weight hits before hitting Note


the soft board, the the soft board, the In practice, you may find that
trolley moves at a trolley moves with
constant velocity increasing velocity 1
(M + m) v2 < mgh.
2
Activity 2.6

direction
of This is because part of the potential energy lost is
motion
converted to sound energy and does work against
air resistance. The principle of conservation is still
Figure 2.109 true if these are included.

133 Forces and Motion


Power SPM 3 The larger units are the kilowatt (kW) and the
’04/P1
megawatt (MW).
1 Power, P is the rate at which work is done, or 1 kW = 1000 W or 103 W
the rate at which energy is transformed. 1 MW = 1 000 000 W = 106 W
4 Power is a scalar quantity as both the work
done and energy are scalar quantities.
Power, P = Work done
Time taken 5 Another unit of power is the horsepower (hp),
Energy transformed which is commonly used in electrical appliances
or P= such as air conditioners.
Time taken
1 hp  =  746 W
2 The SI unit of power is the watt (W). 1 hp    3 kW
4
F 1 W  =  1 J =  1 J s–1
O 1 s 
Power depends on 2 factors:
2

R  Work or energy
P= W = E
CHAPTER

M  Time
t t
4
1 If t is constant: 2 If W or E is constant:
P∝W 1
P∝ t
Work done ↑, Power ↑
Time taken ↓, Power ↑
For example, F
Two librarians A and B are transferring
O books from For example,
2

the floor to a tabletop. Librarian A carries


R one book Two cars A and B of the same mass accelerate until
CHAPTER

to the table while librarian B carriesMtwo books. 80 km h–1. As both cars have the same mass, both
cars do the same amount of work to accelerate to
4
the same velocity.
t=5s
v = 80 km h–1
A
A

B B

t=0 t = 10 s
stationary v = 80 km h–1

Car A takes 5 s to reach the velocity while car B


In 10 s, librarian B does more work (lifting 20 takes 10 s to do so.
books as compared to 10 books by librarian A). Car A takes a shorter time to complete the work.
⇒ Librarian B generates more power. ⇒ Car A is more powerful.

Activ To measure the rate at which work is done by a person


ity 2.7
Apparatus/Materials
Weighing scales, stopwatch and ruler.
Arrangement of apparatus
Activity 2.7

(a) (b) (c)


Figure 2.110

Forces and Motion 134


Procedure Work done by the person in climbing the flight of stairs
= Weight × Vertical height of stairs
1 The mass of the person, m is determined using a
= mgh
weighing scale.
= mgnx
2 The time, t taken by the person to climb a flight
Work done
of stairs is measured with a stopwatch. Power generated by the person =
Time taken
3 The number of steps, n is counted and the height
of each step, x (in metre) is measured using a mgnx
=
ruler. t
Conclusion
Calculation The above method can be used to measure or
compare the power outputs of two or more persons.
Mass of person = m kg F
Note:
Number of steps = n O
The above method measures only the initial power

2
R
Height of each step = x m of the person. In the process of climbing up a flight

CHAPTER
M
of stairs (say, 200 steps) without stopping, the time
Time taken = t s
taken to climb every 10 steps increases as the 4
Vertical height of stairs, h = nx m person becomes tired and hence his/her power
Weight of person = mg N, where g = 10 N kg–1 progressively decreases.

64
A student who is undergoing National Service Work done, W = mgh
training has a mass of 45 kg and takes 6 s to climb a = 45 × 10 × 5.76
flight of stairs of 36 steps. If each step is 16 cm high, = 2592 J
how much power is generated by the student?
[Take g = 10 m s–2] Power generated by the student, P
Work done
Solution =
Time taken
First, find the vertical height, h of the stairs. 2592
h = nx =
6
= 36 × 0.16
n = number of stairs = 432 W
= 5.76 m

Power and Velocity


65
A stuntman (mass 60 kg) in a movie climbs up onto Power, P can also be expressed in terms of force,
the roof of a building of 56 m high. If the power F and velocity, v.
generated by the stuntman is 2.1 kW, how long does Power, P
it take him to complete this task? [Take g = 10 m s–2] Work done
= Work done = Force × Displacement
Solution Time taken

Power generated by the stuntman = Work done = Force × Displacement


Time taken Time
2100 = mg × h = Force × Displacement
t Time
= 60 × 10 × 56 = Force × Velocity
Displacement
Change the unit of t Velocity =
Time
kilowatt to watt. 33 600 ∴ P=F×v
2.1 kW = 2100 W t=  
2100
t = 16 s

135 Forces and Motion


66
A car is moving at a constant velocity of 30 m s–1. Solution
Since the car is moving at constant velocity,
Constant velocity
Fnet = ma = 0 ⇒a=0
engine
thrust

friction = 700 N Engine thrust, F = Total resistance force


= 700 N
Figure 2.111
Power required, P = F × v
If the total resistance force (road friction and air = 700 × 30
F
O
resistance) acting on the car is 700 N, what is the = 21 000 W
power needed by the car to maintain this velocity?
2

R = 21 kW
CHAPTER

Efficiency of Various Devices

1 Heat engines such as a diesel or a petrol engine, and electric motors are
two common devices that convert F energy from one form into another,
usually mechanical energy. O
2

R
2 The pie chart in Table 2.12 shows the relationship between the total
CHAPTER

M
energy input and total energy output of an efficient device and an
inefficient device respectively.4

Table 2.12
Efficient device Inefficient device
wasted energy useful energy
output output

energy useful energy wasted


input energy output input energy output

input output input output



F4/2/115a F4/2/115b

Energy input =  Total energy output


=  Useful energy output + Wasted energy output

In any device, not all A portion of the energy output is


the energy input is converted into heat energy as a result
converted into useful of friction or air resistance. This heat is
energy output or lost to the surroundings and cannot
useful work done. be used to produce useful work.

Forces and Motion 136


3 Efficiency compares the useful energy output to the energy input.
The efficiency of a device:

Useful energy output The efficiency of a device can also be calculated in


Efficiency =   × 100% terms of power.
Energy input

Eo (useful) E (useful)
Efficiency = o × 100%
=   × 100% Ei
Ei 
Eo The useful energy
output and the


t
=   × 100% energy input terms
Ei are each divided
t by time.
If efficiency = 100%, then:
Useful energy output = Energy input Useful power output F
⇒ Perfect device ∴  Efficiency = × 100% O
Power input

2
R

P (useful)

CHAPTER
M
= o × 100%
Pi
4
However, this perfect situation is not
achievable, that is, efficiency < 100%.

4 Table 2.13 compares the efficiency of a petrol engine and efficiency of an electric motor.
Table 2.13
Less efficient device (Petrol engine) Efficient device (Electric motor)
wasted energy
(thermal energy) wasted energy
(75 J) (20 J)

input input useful energy


energy useful energy
output energy output
100 J 100 J (80 J)
(25 J)
efficiency = 25% electric motor efficiency = 80%
petrol engine
  
A large portion of the heat produced at high The resultant heat which originates from friction or
Wasted energy
temperature in the engine is not converted into the heating of the coils in the motor is only a small
(Internal energy
kinetic energy of the car.
F4/2/116a portion of the energy input. of the car and
other energy)

Input energy Useful output energy


100 J (Kinetik energy of
the motion of the car

To increase the efficiency of a device, we need to reduce the wasted energy output.
F4/2/116b
Case Energy input = Useful energy output + Wasted energy output Efficiency
(I) 100 J 60 J 40 J 60
= × 100%
100
= 60%

(II)   80 J 60 J 20 J 60
= × 100%
80

= 75%

We need only 80 J to The more efficient


produce the same device is able to
useful energy output. Same useful reduce the wasted Efficiency
energy output energy output. improved.

137 Forces and Motion


67 13 SPM
Clone
’10

A petrol engine has a work output of 96 kJ per The diagram shows a weight which is connected to
minute. What is the power input if the efficiency of a wooden block through a frictionless pulley with
the engine is 20%? an inelastic string.
Solution
Inelastic string
96 000 J Frictionless table
Frictionless
Power output =
60 s pulley

= 1600 W
P (useful)
F
Efficiency = o × 100% Wooden block
Pi
O Weight
Floor
2

R 1600
20% = × 100%
CHAPTER

M
P
i
4 160 000 What is the energy change in the system after the
Power input, Pi = weight is released?
20
A Potential energy of the wooden block ➞
= 8000 W
Kinetic Energy of the weight
B Potential energy of the weight ➞ Kinetic
F
O
energy of the wooden block
C Potential energy of the weight ➞ Kinetic
2

R
68 energy of the wooden block and weight
CHAPTER

M
D Potential energy of the weight and wooden
A crane lifts a load of 500 kg to a4height of 120 m in block ➞ Kinetic energy of wooden block
16 s.
Comment
When the weight moves downwards, its potential
energy is converted to the kinetic energy of both
the weight itself and the wooden block (since both
120 m are moving)
Answer  C

Figure 2.112

If the power input is 45 000 W, what is the efficiency


of the motor used in the crane?
[Take g = 10 N kg–1]
Solution
Useful energy output = mgh
= 500 × 10 × 120
= 600 000 J Power
Energy input = Power × Time Energy
=
Time
= 45 000 × 16
= 720 000 J
E (useful)
Efficiency = o × 100%
Ei

= 600 000 × 100%
720 000
= 83.3%

Forces and Motion 138


14 SPM
Clone
’11

A ball bearing P is released at the top of a smooth plane as shown in the figure.
Smooth plane
Ball bearing P

0.6 m
Q

What is its velocity at Q? F


A 2.5 m s –1 O

2
B 3.5 m s –1 R

CHAPTER
C 4.9 m s –1 M

D 5.8 m s –1 4
Comments
The potential energy of the ball bearing at P is converted to the kinetic energy at Q.
Solution
Kinetic energy at Q = Potential energy at P
1 2
mv = mgh where m is the mass of the ball bearing
2
v = 2gh
= 2 × 10 × 0.6
= 3. 5 m s–1

Answer  B

2.10

1 The figure shows a weightlifter lifting a barbell of 80 kg.



SPM
Clone
What is the work done on the barbell?
’06

1.8 m
1.7

2 A waiter carries a tray full of dishes weighing 30 N from the kitchen to a customer who is 5 m away.
What is the work done on the tray and dishes?
3 What is the work done by a workman carrying a bag of cement of mass 30 kg up a flight of stairs with
20 steps, if each step is 15 cm in height?

139 Forces and Motion


4 A nurse pushes a patient in a wheelchair with a force 9 Saidin, a fireman, slides down a pole when the fire
of 30 N for a distance of 25 m. alarm sounds. He reaches the ground at 5 m s–1.
Given that his mass is 60 kg, find the friction that
acts against his motion as he slides down the pole.

30 N 30 N

25 m 3.0 m

What is the work done by the nurse?


F 5 The figure shows two trucks pulling a ship in a canal
O of length 1.6 km. The tension in each cable is
2

R 5000 N.
CHAPTER

M
10 Simon rides his bicycle down a slope of 5 m vertical
4 height without pedalling his bicycle. The velocities of
the bicycle before and at the end of the slope are
2 m s–1 and 8 m s–1 respectively.
u = 2 m s–1

F
O
2

R
v = 8 m s–1
CHAPTER

Find the total work done on the M


ship.
5m
6 A fireman of a mass of 70 kg climbs
4
a 10-m ladder
to rescue a girl.

Given that the mass of Simon and his bicycle is


70 kg, find
(a) the total mechanical energy before Simon
moves down the slope,
30° (b) the work done by Simon against friction along
the slope.
11 A sprinter runs at a steady speed of 8 m s–1 against a
resistance force of 35 N. What is his power?
12 In a hypermarket, a machine can lift a crate of
500 kg up onto a shelf as shown in the figure. The
If the ladder is inclined at an angle of 30° to the wall
F4/2/305 time taken for the machine to do this work is
as shown in the figure, how much work has he done?
1 minute.
7 A 7 kg bowling ball is rolling at 3 m s–1. What is its
kinetic energy?
8 The figure shows a girl on a swing. She is released
from a height of 1.5 m.
2m
500 kg

initial
position
(a) What is the minimum power needed?
v=?
(b) If the machine F4/2/309
is operating at an efficiency of
1.5 m
80%, what is its power input?
13 A pump can force 300 kg of water to a height of
0.3 m
12 m in 20 s. Find its useful power. If the power
input is 2 kW, what is the efficiency of the pump?
Find her maximum velocity.

Forces and Motion 140


2.11 Appreciating the Importance of Maximising the Efficiency of Devices

2.11 Appreciating the Importance of Maximising the Efficiency


of Devices
1 It has been explained in Section 2.10 that the If the useful energy output (or work done) is
efficiency of diesel and petrol engines is low as constant, then:
a large portion of energy is lost as heat or 1
Efficiency ∝
sound energy. Energy input
2 The efficiency of an electric motor is high but
the efficiency of an electrical generator at a ∴ Efficiency ↑, Energy input ↓
power station is low (about 30%).
3 The main sources of energy of an electrical 8 With higher efficiency, fuel-consuming
generator in a power station are fossil fuels devices will require less fuel to produce the F
such as coal, petroleum, and natural gas. same amount of work and hence, the O

2
4 Similarly, vehicles (for transportation) derive R
availability of existing sources of fossil fuels

CHAPTER
M
their energy from petrol and natural gas. Thus, will be prolonged.
fossil fuels are of utmost importance to us. 9 The same reasoning is applicable to other 4
5 It must, however, be noted that fossil fuels are devices. Since less energy input is required to
not renewable and these sources of energy will produce the same amount of work or useful
be depleted some day. energy output, the cost of operating the device
6 Apart from looking for alternative sources of is reduced (we need to pay for energy
energy or increasing the source of renewable consumption).
energy, efforts must be made to maximise the 10 Besides conserving resources, maximising the
efficiency of fuel-burning devices. efficiency can also reduce the heat output or
7 From the formula of efficiency: sound production which might be detrimental
Useful energy output to the environment.
Efficiency = × 100%
Energy input

Ways of Increasing the Efficiency of


Devices
Electrical Devices life span is much longer since it does not
have a filament which can burn out when it
Light Fittings overheats.
1 Replace filament light bulbs with fluorescent 3 Use a lamp with a reflector so that the
lamps which have a higher efficiency, preferably illumination can be directed to specific areas.
compact fluorescent bulbs. In this way, the required illumination can be
obtained with a smaller number of light bulbs.

Air Conditioners
1 Choose a brand/model with a high efficiency.
 2 Generally, a higher capacity unit has a higher
efficiency but the choice of an air conditioner
    is determined by the user’s requirement and
Filament light bulb Compact fluorescent bulbs the size of the room. Central air conditioners
  (Efficiency: 2%) (Effeciency: 7–9%) have high efficiencies.
Figure 2.113 3 Ensure that windows are closed and gaps
under doors are sealed when using an air
2 The illumination efficiency of a fluorescent conditioner so that the temperature in the
bulb is four times that of a filament bulb. Its room can be maintained.

141 Forces and Motion


Refrigerators 2 When less water is being used, the
1 Choose the capacity of a refrigerator according consumption of electricity to heat up the water
to the size of the family. is correspondingly reduced.
2 The refrigerator must be installed away from 3 A front-loading washing machine uses
sources of heat and direct sunlight. The less detergent compared to a top-loading
distance from the wall must be more than machine.
3 cm to ensure there is good air flow.
3 The doors of the refrigerator must always be
shut tight.
4 Use a refrigerator with a manual defrost device
Operation of Electrical Devices
— such a refrigerator can save as much as
1 The management of electrical devices includes
30–50% electrical energy when compared to a
F
ensuring that the devices are in good operating
refrigerator with an automatic defrost device.
O condition.
However, defrosting must be carried out
2

R 2 If electrical devices function efficiently, it


periodically to maintain the efficiency of the
CHAPTER

M increases their life span.


refrigerator.
3 Devices must be inspected regularly and
4 5 In terms of energy efficiency, it is more
serviced.
economical to use a large capacity refrigerator
4 For example, the filter in an air conditioner
than a smaller capacity unit.
and the fins of the cooling coil of a refrigerator
Washing Machines must be periodically cleaned to ensure there
1 Use a front-loading machine,
F which is is good air flow and to maintain their efficiency.
more economical in waterO and electricity
2

consumption. R
CHAPTER

M
SPM
2.12 Understanding
4 Elasticity ’08/P2(B)

2.12 Understanding Elasticity


Meaning of Elasticity

1 Figure 2.114 shows that the shape and the size of an object change
when an external force is applied on it.

when when
force is applied force is removed





when when
force is applied force is removed






Figure 2.114

2 When the external force is removed, the object returns to its original
shape and size.
3 The property of an object that enables it to return to its original
shape and dimensions (sizes) when an applied external force is
removed is called elasticity.
4 Rubber and plastic are examples of elastic materials.
5 Soap or plasticine is deformed when an external force is applied on
them. The deformation is permanent even after the external force is
removed — that is, these materials are non-elastic.

Forces and Motion 142


force of
Forces between Atoms repulsion
force of repulsion
increases when
(positive)
x decreases
1 The property of elasticity is caused by the equilibrium distance
existence of forces of repulsion and attraction
between molecules in solid materials. distance of
separation between
2 For simplicity, we will discuss elasticity in xo
2 atoms, x
relation to metals only.
force of attraction
3 Forces of repulsion and attraction between decreases with distance, x
force of force of attraction
atoms are always present in metals in a solid attraction increases with
state. (negative) distance, x
4 In the absence of an applied external force Figure 2.115
on a metal rod, atoms vibrate about their
F
equilibrium positions. The atoms are acting between the two atoms is a force of O

2
separated by a distance called the equilibrium repulsion. When the external force is R
distance, xo. At this equilibrium distance, the

CHAPTER
removed, the repul­sive force pushes the atoms M
force of attraction is balanced by the force of back to their ori­ginal position and the metal
4
repulsion — the resultant force is therefore rod returns to its original length.
zero. 7 When a stretching force is applied to the two
5 The graph in Figure 2.115 shows the ends of the metal rod, its length increases. As
relationship between the resultant force and a result, the separ­ation distance between the
the distance between two atoms, x. two atoms is increased—that is, x > xo. The
6 When a compressive force is applied to the resultant force acting between two atoms is a
two ends of the rod, the length of the rod is force of attraction. When the external force is
reduced and hence the separation distance removed, the force of attraction pulls the
between the two atoms is reduced—that is, atoms back to their original position and the
x < xo. The resultant force (or effective force) metal rod returns to its original length.

Equilibrium
F F F F

MetalMetal rod under


Metal rod is compressed, length rod
no in equilibrium
external force Metal rod is stretched,
of rod decreases. condition. length of rod increases.
F4/2/120b
F4/2/120d


F4/2/120f

x < xo Arrangement of atoms x > xo

xo
F F F F

Metal rod under compression: Metal rod under tension:


Equilibrium condition:
F4/2/120c
1 Force of repulsion takes 1 Force of attraction takes
F4/2/120e 1 Force of repulsion F4/2/120g
effect. effect.
2 When the compressive force = Force of attraction
2 When the stretching force is
is removed, the force of 2 Resultant force = 0
removed, the force of attraction
repulsion between the atoms between the atoms pulls the
pushes the atoms back to atoms back to their equilibrium
their equilibrium positions. positions.

143 Forces and Motion


2.4

Relationship between force and extension of a spring


Situation
Azizi and Kamaruzzaman, who are 1 year old and
3 years old, are sleeping in their sarong cradles.
The springs of both cradles are identical and new.
Observe the babies and the springs in the Figure
2.116. What inference can you make?

F Inference
O
The extension of the spring depends on the weight or
2

R
force acting on it.
CHAPTER

M (a) Azizi (1 year old) (b) Kamaruzzaman


(3 years old)
4
Hypothesis
Figure 2.116
The greater the stretching force, the longer the
extension of the spring.

F
Aim O
2

To investigate the relationship between


R a stretching force (weight) and the extension of a spring.
CHAPTER

Variables 4
(a) Manipulated : weight of load, F
(b) Responding : spring extension, x
(c) Fixed : force constant of the spring, k

Apparatus/Materials
Spring, pin, slotted weight, weight holder, retort stand with clamp, metre rule, and plasticine.

Arrangement of apparatus

metre rule zero mark

clamp

spring

pin as pointer
retort
stand o o
extension, x
plasticine

Experiment 2.4

weight holder
weight

(a) (b)
Figure 2.117

Forces and Motion 144


Procedure
1 The metre rule is clamped vertically to the retort stand, with its ‘0’ mark at the top.
2 The spring is suspended from the clamp.
3 The pin is attached to the bottom end of the spring with plasticine.
4 The initial position of the pin, o is noted.
5 A weight holder of mass 50 g is hooked onto the bottom of the spring. A slotted weight of mass 50 g
is added to the weight holder. The total weight acting on the spring is noted and the new position of
the pin,  is recorded.
6 The extension of the spring is calculated from the formula: x =  – o
7 Step 5 is repeated with the addition of 50 g each time until a maximum of 300 g.
All the respective readings are noted and recorded.
F
Tabulation of data O

2
Original position of the spring = o cm R

CHAPTER
M
Table 2.14
4
Load, Stretching Position Spring
m (kg) force, of pin, extension,
F = mg  (cm) x = ( – o )
(N) (cm)
0.10 1.0
0.15 1.5
0.20 2.0
0.25 2.5
0.30 3.0

Graph
The graph of spring extension, x against stretching force, F is drawn.
x (cm)

F (N)
O

Figure 2.118
Conclusion
The straight line in the x-F graph passes through the origin. This shows that the extension of the spring,
x is directly proportional to the stretching force, F.
The hypothesis is valid.

Hooke’s Law
Experiment 2.4

1 Hooke’s law states that the extension of a spring is directly


proportional to the stretching force acting on it provided the
elastic limit of the spring is not exceeded.
2 In Experiment 2.4, the spring returns to its original length when
the load is removed — that is, the spring is elastic.

145 Forces and Motion


3 As the addition of weights continues (the stretching force increases), a situation will be reached
where there is permanent extension to the spring when the load is removed.
4 The elastic limit of a spring is the maximum stretching force which can be applied to the
spring before it ceases to be elastic, i.e. permanent extension or permanent stretching is
produced.

SPM SPM
’04/P2 ’09/P1
Graph of Stretching Force, F against Spring Extension, x

1 Figure 2.119 shows a graph of stretching force, F against the extension of a spring, x.
F
Q P
E
F F = kx spring not
O spring obeying obeying Hooke’s law
Hooke’s law
2

R
CHAPTER

4
x
O R

Figure 2.119

2 Section OE of the graph is a straight


F line passing through the origin.
Thus, the stretching force is Odirectly proportional to the extension of the spring (that is, the
2

R
extension is directly proportional to the stretching force) and Hooke’s law is obeyed.
CHAPTER

M
3 Beyond point E (EQP), the graph takes the form of a curve — that is, the stretching force does
not vary directly with the extension
4 and Hooke’s law is no longer applicable.
4 If the load is removed at point Q, the spring will contract but with a permanent extension,
represented by line OR.

Force Constant or Spring Constant

1 Figure 2.120 shows the linear portion of a graph of the stretching force against the extension of
a spring.
F

a x
O

Figure 2.120

2 The graph is a straight line passing through the origin. Hence:


F    x
F = kx
where k is a constant known as the force constant or spring constant.
3 From the formula : F = kx
F (N)
k =
x (m)
∴  The unit of k is N m .
–1

Forces and Motion 146


4 From Hooke’s law: F = k x
Equation of straight line: y = m x, where m is the gradient.
∴k=m
k= b
a
= gradient of the F-x graph
5 The value of the force constant is a measure of the stiffness of the spring.

Force Constant and Stiffness of Spring

To understand the meaning of the force constant, look at the following examples: F
O

2
R
(a) A load of 2.5 N extends spring A by 20 cm. (b) A load of 15 N extends spring B by 20 cm.

CHAPTER
M
Applying F = kx, Applying F = kx,
F F 4
k = k =
x x
2.5 N 15 N
= =
0.2 m 0.2 m
= 12.5 N m–1 = 75 N m–1

F (N) Spring A requires 12.5 N to be


B
g extended by 1 metre whereas
in
75
spr
stiff spring spring B requires 75 N for the same
ng A extension.
spri
12.5 ∴ Spring B is stiffer than spring A.
soft spring

x (m)
O 1       

Force constant, k = Gradient of graph


1   A larger value of k indicates a stiffer spring.
2   A steeper graph indicates a stiffer spring.

Force and compression of spring



Hooke’s law is also applicable in the case of the
compression of a spring where the compression of the
spring (a decrease in length) is directly proportional to
the compressive force.
F

x = 0 – 

Physics Blog
147 Forces and Motion

F4/2/121
69
A spring has an original length of 20 cm. When a load When 200 g is added, load =  300 + 200
of mass 300 g attached to it, the length of the spring is =  500 g
extended to 26 cm. Calculate the force constant, in =  0.5 kg
N m–1, of the spring. What is the length of the spring Stretching force =  mg
when the load is increased by 200 g? = 0.5 × 10
[Take g = 10 N kg–1] =  5 N
Solution
From the formula: F = kx
m = 300 g = 0.3 kg
Stretching force =  mg Extension, x = F
F k
O = 0.3 × 10 = 3 N 5
=
2

R Extension, x =  26 – 20 50
=  6 cm = 0.06 m
CHAPTER

M
= 0.1 m
From the formula: F = kx
4 = 10 cm
F
k =
x
∴  Length of the spring =  20 + 10
3
= =  30 cm
0.06
= 50 N m–1F
O
2

R
CHAPTER

Application of Hooke’s Law on Springs


4 From Hooke’s Law, for a spring under two separate
forces F1 and F2 (or loads W1 and W2) with
Many questions do not need the calculation of the corresponding extensions x1 and x2:
force constant, k. F1 = kx1
The following method explained in these examples F2 = kx2 The stretching force is
is suggested. F1 x1 proportional to the
Dividing, = extension of the spring.
F2 x2

In the situations for load–extension:


W1 x1
=
0
W2 x2
1 2 m1 x1
Since W = mg, we obtain: =
m2 x2
x1 = 1 – 0
x2 = 2 – 0 Therefore:
F1 F1 x1 F1 1 – 0
–– = –– –– = ––––––––
F2 F2 x2 F2
m1
2 – 0
m1 x1
m2 or –– = –– m1 1 – 0
m2 x2 or –– = ––––––––
m2
2 – 0
Figure 2.121 • Used when one
F4/2/122 extension or both • Used only when
extensions are given, needed to find the
0 = the original length of a spring or easily determined original length or
1 = length of spring for F1 and m1 from the information reading on the scale
2 = length of spring for F2 and m2 given. when no load is
• Used more often. attached (please refer
to Example 72).

Forces and Motion 148


70 71
The original length of a spring is 10 cm. With a load A 30 cm spring is compressed to 25 cm when a load
of mass 300 g, the length of the spring is extended to of 300 g is placed on it. What additional load is
16 cm. required to compress the spring to 22 cm?
(a) What is the length of the spring with a load of Solution
400 g?
(b) What load is required to extend the spring to m1 x1
Applying –– = –– ,
19 cm? m2 x2
Compression:
300 5 x1 = 30 – 25 = 5 cm
Solution –––– = –– x2 = 30 – 22 = 8 cm
m2 8
m1 x1
(a) Applying –– = –– , 8 × 300 F
m2 x2 ∴ m2 = ––––––– O
5

2
x1 = 16 – 10 R
300 6 = 6 cm
–––– = ––

CHAPTER
= 480 g M
400 x2
Additional load =  480 – 300 =  180 g
6 × 400 4
∴ x2 = –––––––
300
= 8 cm 72
∴ Length of spring = 10 + 8
= 18 cm Figure 2.122 shows the two positions of a pin when a
spring is loaded with two different weights.
Alternative methods
ruler
m 1 – 0
 (i) ––1 = ––––––––
m2  – 
2 0 10 cm 10 cm
300 16 – 10
–––– = –––––––– 15 cm 15 cm
400 2 – 10

3 6 20 cm 20 cm
–– = ––––––––
4 2 – 10 300 g
25 cm 25 cm
32 – 30 = 6 × 4
500 g
32 = 54
2 = 18 cm (a) (b)
Length of spring = 18 cm Figure 2.122
(ii) 300 g  →  6 cm Extension What is the reading of the pin when the spring is
= (16 – 10) cm without a load?
∴ 100 g  →  2 cm = 6 cm
⇒ 400 g  → 4 × 2 cm Solution
= 8 cm
Let 0 = reading of the pin when the spring has no
Length of spring = 10 + 8
load.
= 18 cm
m1 1 – 0
Applying –– ––––––––
m2 = 2 – 0 ,
m x
(b) Applying ––1 = ––1 ,

m2 x2
300 20 – 
x1 = 16 – 10 –––– = –––––––––0
300 6 500 25 – 0
–––– = –– = 6 cm
m2 9 x2 = 19 – 10
= 9 cm 3 20 – 
9 × 300 –– = –––––––––0
∴ m2 = ––––––– 5 25 – 0
6
75 – 30 = 100 – 50
= 450 g 20 = 25
∴ A load of 450 g is required. 0 = 12.5 cm

149 Forces and Motion


Factors Affecting the Stiffness of a Spring

Activ To investigate the factors that affect the elasticity of a spring


ity 2.8
Apparatus/Materials
A pair of steel springs, P and Q of different lengths, a pair of steel springs, R and S with wires of different
diameters (different thickness), a pair of steel springs, T and U with coils of different diameters, a steel spring, V
and a copper spring, W of the same dimension, pin, slotted weights, weight holder, retort stand with a clamp,
metre rule and plasticine.
F
O Procedure
2

R
 1 The apparatus is set up as shown in Figure 2.117 (Experiment 2.4).
CHAPTER

M
2 Steps 1 to 6 of Experiment 2.4 are repeated using the pair of springs of different lengths.
4 3 A force-extension (F-x) graph is plotted for each spring for comparison.
4 Steps 1 to 3 are repeated with springs of different thickness, springs with coils of different diameters and
springs of different type of materials. Please take note that in each case, all other physical factors remain
constant while one particular factor is being investigated.
F
Results and Inferences O
2

R
CHAPTER

M Table 2.15

4 Thickness of
Length of spring Coil diameter Type of material
spring wire
Steel springs Steel springs Steel springs Same dimensions

P Q R S T U V W
Factors
investigated

shorter smaller
larger
(thicker) smaller larger steel copper
longer spring spring

F (N) F (N) F (N) F (N) F (N) F (N) F (N)
spring P spring R spring R spring T spring T spring V spring T

spring Q
F-x graph spring S spring S spring U spring U spring W sprin

O x (cm) O
O x (cm) O O x (cm) x (cm) O x (cm) O
x (cm) x

The shorter spring, P The spring with the The spring with the
F4/2/123 The steel spring, V is
Conclusion is stiffer. wire of a larger coil of a smaller stiffer than the copper
Activity 2.8

diameter (thicker), diameter, T is stiffer. spring.


R is stiffer.

Forces and Motion 150


System of Springs
0
Identical Springs

Note: For each case below, when load = W, spring extension = x (applied to x

a spring alone). W
Note: o = length of original spring
Figure 2.123
                            Table 2.16

Series arrangement Parallel arrangement

F
0 0 0 0 O

2
R

CHAPTER
M
x x 1x 11
xx 1x
2 22 2
W W 4

0 0

x x

W W
            
The same load is applied to each spring. The load is shared equally among the springs.
Tension in each spring = W W
Tension in each spring =
Extension of each spring = x 2
Total extension = 2x x
Extension of each spring =
Length of spring system 2
= Total original length + Total extension x
Length of spring system = o +
= 2o + 2x 2
If n springs are used: If n springs are used:
The total extension = nx x
The total extension =
n

73
Figure 2.124 shows a Solution
series arrangement of First, find the extension of a single spring (very
three identical springs. important).
The original length of
each spring is 16 cm. Extension, x1 = 20 – 16 = 4 cm
p m1 x1
With a load of 200 g, Applying –– ––
m2 = x2 ,
the length of each
spring becomes 20 cm. 200 4
–––– = ––
What is the length p 300 x2
of the system shown 4 × 300
300 g ∴ x2 = ––––––– = 6 cm
in the figure? 200

Figure 2.124 ∴ Total extension = 3 × 6 cm = 18 cm
p = Total original length + Total extension
= (3 × 16 cm) + 18 cm = 66 cm

151 Forces and Motion


74
Figure 2.125 shows a system For spring P :
of 6 identical springs. m1 x1 x1 = 15 – 12
S T U –– = –– = 3 cm
The original length of each m2 x2

spring is 12 cm. When a load 200 3
of 200 g is added, the length –––– = ––
y 300 x2
of each spring extends to Q R

15 cm. 200x2 = 3 × 300
What is the length y of the x2 = 4.5 cm
spring system? For spring Q or R :
P
F x 4.5
Extension = 2 = 2
O
2

300 g
R
Extension     = 2.25 cm
CHAPTER

M
Figure 2.125
For spring S, T or U :
4 x 4.5
Solution Extension = 3 = 3
First, find the extension of a single spring, x.
x Extension     = 1.5 cm
For 2 springs in parallel, extension =
2
F
x
For 3 springs in parallel, extension
O =
3
2

R S T U 12 + 1.5
CHAPTER

4
extension = x
3
Q R 12 + 2.25

extension = x
2

P 12 + 4.5

extension = x
300 g
W

∴  y = Total original length + Total extension


Length of spring system = 12 × 3 + (4.5 + 2.25 + 1.5)
= Total original length + Total extension = 36 + 8.25 = 44.25 cm

Non-identical Springs Arranged in Series

75
Springs A and B are two non-identical springs. Spring
10 cm
A
12 cm
B A has an original length of 10 cm which stretches to
15 cm 18 cm A 12 cm with a load of 300 g. Spring B has an original
length of 15 cm and extends to 18 cm with a load of
300 g 500 g. What is the length y of the spring system with a
y
500 g B
load of 800 g?

800 g

Figure 2.126

Forces and Motion 152


Solution For spring B:
x1 = 18 – 15
m1 x1 500 3 = 3 cm
= 
Applying –– ––
m = x , 800 b
2   2
For spring A: Extension, b =  800 × 3 =  4.8 cm
300 2 x1 = 12 – 10 500
=  = 2 cm
y = Total original length + Total extension
800 a
= (10 + 15) + (5.3 + 4.8)
Extension, a =  800 × 2 =  5.3 cm = 35.1 cm
300

Elastic Potential Energy 5 Work done, W


=  Average force × Displacement, x
F
1 Elastic potential energy is the energy =  0 + F × x O

2
stored in a spring when it is extended or 2 R

CHAPTER
M
compressed. =  1 Fx
2 The elastic potential energy is a result of the 2 4
work done to extend or compress the spring. = Area of the shaded triangle under the graph
3 Figure 2.127(a) shows a spring with an of force against extension
original length of o being extended and
Substitute F = kx into the formula.
compressed with a force that is increased from
zero to F. The extension and compression Work done, W = 1 kx × x  =  1 kx2
2 2
have the same value x. Figure 2.127(b) shows
the graph of force against the extension/ ∴ Elastic potential energy, Ep is given as:
compression of the spring.
EP = 1 kx2 because
2 EP = W
o

x 76
spring
F
compressed
x A force is increased from 0 to 30 N and applied to a
spring, compressing it by 12 cm as shown in Figure
2.128. force, F (N)

F
spring extended 30

(a)
force F4/2/329(a)
compression (cm)
12

F Figure 2.128

Area = 1 Fx
Find
2 (a) the force constant, k of the spring,
= Work done
= Elastic Potential Energy (b) the elastic potential energy stored.
Solution
Change the unit
O x extension/ compression (a) k = gradient of F-s graph of compression
30 N to metres.
(b) = –––––––
0.12 m = 250 N m
–1
12 cm = 0.12 m
Figure 2.127
(b) Ep = Area under the F-s graph
4 The work done on the spring is not equal
1
to F × x, because the force is not constant but is = –2 × 0.12 × 30 = 1.8 J
increasing from zero to F.

153 Forces and Motion


The graph in Figure 2.130 shows the relationship
77 between the applied force F, which increases non-
A spring with a 300 g sphere attached to it is uniformly with the extension, x of the elastic band of
extended by 5 cm. The force constant of the spring is a catapult. The mass of the stone is 25 g.
200 N m–1. (a) What is the work done to stretch the elastic band
of the catapult?
(b) What is the elastic potential energy stored in the
elastic band?
5 cm 300 g (c) If the stone is released, what is its velocity as it
leaves the catapult?

smooth surface
Solution
F (a) The number of squares covering the area under
Figure 2.129
O the F–x graph are estimated and multiplied by the
2

R value of each square. A partial square, whose


What is the elastic potential energy stored in the
CHAPTER

M
spring? What is the maximum velocity of the sphere area is greater than half the area of a full square,
4 when the stretching force is released? is counted as a complete square. Conversely, a
square that is less than half filled is not counted.
Solution
Number of squares = 28
Extension, x = 5 cm = 0.05 m
1 Value of 1 square = 0.5 N × 0.02 m = 0.01 J
Elastic potential energy, Ep = kx2
2 F ∴  Work done, W = 0.01 J × 28 = 0.28 J
1 O (b) Ep = Work done = 0.28 J
2

= ×R200 × 0.052 (c) Ek = Ep = 0.28 J


2 M
CHAPTER

1
= 0.25 J ∴ mv2 = 0.28
2
Maximum kinetic energy gained 4
= Elastic potential energy lost 1
× 0.025 × v2 = 0.28
1 2 Velocity, v is 2
mv = 0.25 maximum when
2
the spring is not v2 = 0.28 × 2
1 under tension or 0.025
× 0.3 × v2 = 0.25
2 compression, i.e., = 22.4
when the original
2 × 0.25 length is restored. v = 4.7 m s–1
v2 =
  0.3
v = 1.666
= 1.29 m s–1 79
A spring is compressed with a force of 300 N.
If the potential energy stored is 6 J,
78 (a) what is the compression of the spring?
elastic band
(b) determine the force constant of the spring.
stone Solution
x 1
(a) Elastic potential energy, Ep = Fx
F 2
1
F (N)
6 = × 300 × x
2
3.0
2.5
Compression, x = 0.04 m = 4 cm
2.0 (b) Use the formula F = kx,
1.5 F
1.0 k =
x
0.5
x ( cm ) 300
2 4 6 8 10 12 14 16 = =  7500 N m–1
0.04  
Figure 2.130

Forces and Motion 154


Uses of the Property of Elasticity

Weighing Spring shock Baby hammock Mattress and


apparatus absorbers cushion
A weighing apparatus Spring shock absorbers The spring attached to The springs in a
uses the elastic pro­perty are mounted on the the baby hammock mattress or cushion
of a spring. When a load wheels of cars and enables it to bounce undergo many cycles of
is being weighed, the motorcycles to absorb vertically when the compression during its
spring is either extended, impact and reduce hammock is displaced use yet the mattress or
or compressed, in direct vibrations resulting down­wards slightly. cushion always returns
proportion to the load from uneven road The bounce helps babies to its original shape.
whose value is regis­tered surfaces. to fall asleep. This is due to the
on the linear scale of the elasticity of the F
O
apparatus. If there is no springs.

2
R
load, the spring in the

CHAPTER
M
weighing apparatus
returns to its original 4
length which
corresponds to the zero
reading on the scale.

Uses of springs
Voltmeter, ammeter, in daily life Toys
galvanometer, spring
operated clocks and
watches The toys below move
using the power of
The potential energy springs. The work done
stored in the spiral in compressing the
springs is converted into spring is converted into
kinetic energy which Tyre pressure elastic potential energy,
Door closer gauge
turns the hands of the which is then converted
clock and the pointer of This device has a spring When the nozzle of the into the kinetic energy of
the meter until the system and is mounted gauge is fixed on to the the toys, thus making the
springs return to their on the door to close it valve of the tyre, the air toys move.
original state. automatically. When pressure in the tyre pushes
the door is open, the a piston which compresses
elastic potential energy a spring inside the gauge.
stored in the compressed This allows us to read the
spring closes the door. amount of pressure in the
tyre.
door closer

155 Forces and Motion


The material of trampoline has The elasticity of a springboard
The pole in a pole-vaulting event enables a diver to jump high in
uses the property of elasticity to an elastic property to help a
gymnast bounce upwards. the air before diving into the
F help the pole-vaulter to leap over pool.
O the bar.
2

R
CHAPTER

M
Other elastic material
4

F
O
2

R
As the bow bends, the elastic
CHAPTER

M
twine of the bow is stretched to The elasticity of the strings of a The double clips and clothes
store the elastic potential energy 4 tennis or a badminton racket pegs make use of the spring
which propels the arrow when it allows a ball or shuttlecock to between the clips to hold things
is released. bounce off them. tightly.

15 SPM
Clone
’11

Figure (a) and (b) shows the set up to study the elasticity of spring X and Y and the corresponding force-length graphs.
Force
X

Length of
      spring
(a) (b)

Which comparism about spring X and Y is correct? Comment Force


X
A X is stiffer than Y As shown in the diagram,
B To stretch 1 cm, work done needed on X is less for the same force
than on Y applied, spring X Y
C The original length of X is longer than Y extends less than spring
F‘
Y. Therefore, spring X
is stiffer than spring Y Length of
spring
Answer  A

Forces and Motion 156


2.12

1 A spring of length 20 cm is extended to 26 cm


when pulled by a force of 10 N. What is the length
of the spring when it is compressed by a force of p
20 N?
2 The figure below shows a spring which is compressed. 450 g

m 500 g
m
(b)

What is the length of the spring system in Figure (b)?


12 cm 10 cm All the springs are identical.
F
6 The figures show a spring system comprising O

2
identical springs. R
Calculate the value of m if the original length of the

CHAPTER
M
spring is 15 cm.
3 The figure shows the pointer reading of a spring. 4
When an additional load of 200 g is placed in the
pan, the pointer reads 25 cm.

20 cm
300 g 25 cm
600 g 300 g

What is the reading of the pointer when the total


load is removed? Each spring is of length 18 cm and extends to 22
cm when subjected to a load of 200 g. What is the
4 M and N are two non-identical springs each
length of the spring system in the figure?
measuring 12 cm. When subjected to loading, their
respective lengths are as shown in the figure (a) 7 Figure (a) shows a spring of length 20 cm
below. compressed to a length of 15 cm by a load of 250 g.

15 cm N 250 g
M
18 cm m
y
300 g
15 cm 12 cm
500 g

600 g

(a) (b) (a)


What is the length y of the spring system if M and N
250 g
are arranged in series and the load applied is 600 g? m
5 Figure (a) shows two springs arranged in parallel,
each of length 9 cm, subjected to a load of 600 g.
15 cm 12 cm

12 cm
(b)

600 g
What is the value of m in Figure (b) if three iden­tical
(a) springs are compressed to a length of 12 cm?

157 Forces and Motion


8 The figure is a graph of extension, x against load, m 9 The figure shows a sphere of mass 20 g pushed
for a spring. against one end of a spring on a smooth surface.
The original length of the spring is 18 cm and its
x (cm)
force constant is 12.5 N m–1.
a
12 cm 12 cm 20 g

compression
force
O m (g) smooth surface
600 800

(a) What is the elastic potential energy stored in the


(a) What is the value of the force constant, k? spring?
F (b) What is the value of a? (b) What is the maximum velocity reached by the
O (c) What is the potential energy stored when the sphere when the compressive force on the
2

R spring is extended by a cm? spring is removed?


CHAPTER

F
O
2

1. Speed is the rate of change of distance.


R the collision is less than the kinetic energy before
CHAPTER

M
Distance travelled the collision.
Speed =
Time taken (b) In an elastic collision, momentum, total energy
4 and kinetic energy are conserved.
2. Velocity is the rate of change of displacement.
Displacement 10. Acceleration is proportional to the applied force but
Velocity = inversely proportional to the mass.
Time taken
11. Impulse is the product of a force and the time
3. Acceleration is the rate of change of velocity.
it acts.
v–u
a= Impulse = Ft
t
12. Impulse is equal to the change in momentum.
4. Equations of linear motion with constant acceleration: Ft = mv – mu
v = u + at 13. The impulsive force is reduced by prolonging the
1 time of impact.
s = (u + v)t
2 14. When forces are in equilibrium, the object will be at
v 2 = u2 + 2as rest or move with constant velocity.
1 15. A force can be resolved into vertical and horizontal
s = ut + at 2
2 components.
5. On a displacement–time graph, the gradient of the 16. Work is the product of a force and the distance
graph represents the velocity of the object. moved in the direction of the force.
6. On a velocity–time graph, 17. Energy is the capacity to do work.
(a) the gradient of the graph is equal to the (a) Potential energy, Ep = mgh
1
acceleration of the object. (b) Kinetic energy, Ek = mv 2
2
(b) the area under the graph is numerically equal to
the distance travelled. 18. Power is the rate at which work is done, or the rate at
7. Inertia is the tendency for an object to maintain its which energy is transformed.
state of rest or uniform motion in a straight line. Work done
Power =
8. Momentum of a moving object is the product of its Time taken
mass and its velocity. Useful power output
9. The principle of conservation of momentum states 19. Efficiency = × 100%
Power input
that the total momentum of a system is conserved,
20. Hooke’s Law, F = kx
if no external force acts on the system. 1
(a) For inelastic collision, momentum and total Elastic potential energy, Ep = kx 2
2
energy are conserved but the kinetic energy after

Forces and Motion 158


SPM Exam Practice 2

2
Multiple-choice Questions

2.1 Linear Motion


1 Puan Halimah pushes her trolley from P to U via Q, R, S, T and U as shown in 4 Which features of a graph is used
Diagram 1. to determine acceleration?
A The slope of a displacement– F
time graph O

2
B The slope of a velocity–time R

CHAPTER
M
graph
C The area under a 4
displacement–time graph
D The area under a velocity–time
graph

2.3 Inertia
5 A body stays at rest, or continues
SPM
Clone to move with a constant velocity,
’09 unless an external force acts on it.
1.1 Significant Figure Which of the Newton’s laws is
related to the above statement?
Diagram 1 A Newton’s first law of motion
B Newton’s second law of
motion
Find her displacement from P.
2.2 Motion Graphs C Newton’s third law of motion
A 10 m
B 17 m 3 In Diagram 3, graphs X and Y are 6 A loaded ocean tanker is more
C 23 m the velocity-time graphs for car V difficult to stop than an empty one
D 77 m and car W respectively. because it
A possesses more kinetic energy
2 Cars P and Q start from rest
B sinks more into the water
simultaneously and accelerates at
C possesses more inertia
acceleration a and 3 m s–2
D encounters a smaller frictional
respectively.
force 1.1 S

2.4 Momentum and


Conservation of
Momentum
Diagram 3
7 A golfer swings a club to hit a golf
ball. The club continues to swing
Diagram 2
Which of the following areas while the golf ball flies off with a
After 2 seconds, P is ahead of Q represents the distance between 1.1 high velocity.
Signifi Which of the
cant Figure
by 6 m. Find a. the two cars at time t? following quantities is conserved?
A 4.5 m s–2 A OPQ A Kinetic energy
B 5.0 m s–2 B OQR B Potential energy
C 5.5 m s–2 C OPQR C Momentum
D 6.0 m s–2 D OPR D Impulse

159 Forces and Motion


8 Diagram 4 shows two trolleys Which of the following tape charts
2.6 Impulse and Impulsive
P (mass = 2 kg) and can be expected if the student
Q (mass = 1 kg) placed together. Force repeats the experiment using a
P and Q move in opposite 11 An egg falls on a thick mattress. It falling weight of 200 g?
directions when the release pin is does not break because the mattress A
struck. A reduces the time of impact.
B prolongs the time of impact.
C reduces the kinetic energy of
the egg.
D reduces the egg’s impulse.
12 Diagram 6 shows Nazli catches an
incoming softball of mass 0.14 kg
Diagram 4
F by moving his hand backwards.
O Which of the following statements B
2

R is correct?
A Kinetic energy of P
CHAPTER

M
= Kinetic energy of Q
4 B Momentum of P
= 1 × Momentum of Q 1.1 Significant Figure
2
C Speed of P
= 1 the speed of Q
2 F Diagram 6
O C
The velocity of the ball reaching
2

D Velocity of P R
= 2 times velocity of Q his glove is 10 m s–1. If the time
CHAPTER

M
taken to stop the ball is 1.2 s, what
4 is the force exerted on his hand?
A 0.9 N C 2.0 N
2.5 The Effects of a Force B 1.2 N D 8.6 N
9 The graph in Diagram 5 shows the
velocity-time graph of a car.
2.7 Safety Features in Vehicles
13 Which of the following is not a
safety feature of a car?
15 The mass of an object in Kuala
A Safety belts
Lumpur is 20 N. Compare its
B Spacious compartments
mass and weight on Mount
C Air bags
Everest.
D Padded dashboard

Mass Weight
Diagram 5
2.8 A  2 kg 20 N
Which part of the graph indicates Gravity
B  2 kg 20 N
that the resultant force on the car 14 The tape chart shown in Diagram C 2 kg  20 N
is zero? 7 is obtained in an experiment D 2 kg  20 N
A PQ C RS carried out in a laboratory.
B QR D ST
1.1 The falling weight used is
Significant Figure
10 A force of 9 N pushes an object of 100 grams. 1
16 The weight of a student is the
mass 3 kg on a floor. If the object 3
moves at a uniform velocity of weight of his father. If the total
2 m s–1, what is the magnitude of 1.1
weight
Signifiof theFigure
cant student and his
the friction, in N? father is 600 N, what is the mass,
in kg, of the student?
A 1
3 A 15
B 3 B 20
C 6 C 30
D 9 Diagram 7 D 150

Forces and Motion 160


1.1 Significant Figure
At which point of the path does
2.9 Forces in Equilibrium 2.12 Elasticity
the stone possess minimum
17 FR1 and FR2 are the resultant forces kinetic energy? 24 A spring with an original length of
of the forces shown in Diagrams A P 20 cm is stretched to 26 cm.
8(a) and (b) respectively. B Q The force is F N and the elastic
C R potential energy stored is Ee. The
D S spring is then compressed so that
its length is now 17 cm. What is
the force and the elastic potential
21 Diagram 11 shows a pulley
energy stored in the spring?
system with P and Q of masses
(a) (b)
2 kg and 1 kg respectively.
Force Elastic
Diagram 8 applied potential F
What can be said about FR1 and FR2? energy stored O

2
A FR1  FR2 1 1 R
A F E
4 4 e

CHAPTER
B FR1 = FR2 M
C FR1  FR2 1 1
B F E 4
2 4 e
18 The following situations show
equilibrium of forces except 1.1 Significant Figure 1 1 1.1 S
C F E
SPM
Clone A a coconut falling from a tree. 2 2 e
’08 B a ship floating at rest in the sea. D F Ee
C a car moving up a hill at Diagram 11
constant speed.
D a lamp hanging at rest in a hall. What is the total kinetic energy of
P and Q when Q rises 2 m?
A 5 J 25 Diagram 12 shows three identical
B 10 J springs, P, Q and R, supporting a
2.10 Work, Energy, Power and C 20 J load of 600 g. The original length
D 40 J of each spring is 10 cm.
Efficiency
19 A motor takes 5 s to lift a load of
SPM
6 kg to a height of 0.8 m. 22 Pak Hashim with a mass of
Clone
’04 50 kg climbs a coconut tree
motor to a height of 6 m to pick a
coconut in 1 minute. What
is his power?
load A 5 W
B 25 W
C 50 W
0.8 m D 300 W

Diagram 9
2.11 The Importance of
What is the power of the motor?
A 9.6 W C 240 W Maximising the Efficiency
1.1 of cant
Signifi Devices
Figure
B 96 W D 375 W
23 The efficiency of a device can be Diagram 12
20 A boy throws a stone in the
increased if
direction as shown in Diagram 10.
A the useful power output is What is the total length, y, of the
increased while the power spring system if P, Q and R are
input is maintained. arranged in series and a load of
B the useful power output is 300 g is attached to spring R?
maintained while the A 27 cm
power input is increased. B 39 cm
C the useful power output and C 48 cm
Diagram 10 power input are increased. D 57 cm

161 Forces and Motion

1.1 Significant Figure


Structured Questions
1 A sprinter runs along a 100-m track as shown in 3 (a) Bakri carries a pail of sand up a flight of stairs of
Diagram 1. 1.2 m as shown in Diagram 3. The weight of the
pail of sand is 100 N.
100 m

t=5s

24 m

F                         Diagram 1
O
(a) The sprinter reaches his highest velocity
2

R
5 seconds after starting from rest. He covers a
CHAPTER

M
displacement of 24 m in the 5 seconds. What is Diagram 3
4 his average acceleration? [2 marks]
(b) What is the average net force that is needed to What
100 mis the work done on the pail of sand by

achieve the acceleration in (a) if the mass of the Bakri? [2 marks]


sprinter is 60 kg? [2 marks] t=5s (b) Two elephants pulled the same pail of sand to the
(c) Find the total time it takes the sprinter to same height using two ropes and two pulleys as
complete 100 m if he maintainsF his
24 m highest shown in Diagram 4.
velocity. O
2

[ R 2 marks]
CHAPTER

M
2 A boy scout of a mass of 45 kg runs at a speed
of 6 m s–1 before grabbing a 4rope and swinging F4/2/145
upward.

Diagram 4
(i) How much work did the two elephants do
on the pail of sand? [1 mark]
(ii) If the angle between the two ropes is 178°,
determine the tension in each rope.
[2 marks]
(iii) Is it possible to pull the ropes such that they
are horizontal to the floor? Explain your
answer.
Diagram 2 [3 marks]

4 Diagram 5 shows two blocks, P and Q, of masses 2m


(a) What is the change in energy? [1 mark]
and 3m respectively tied with a string. The spring
(b) How high will the boy swing upward?  [2 marks]
which is attached to block P is compressed to 12 cm.
[Take g = 9.8 m s–2]
(c) The boy releases his hold on the rope at the
highest point of the swing.
(i) How does the boy fall? Tick your answer.

Vertically
In a parabolic path Diagram 5

[1 mark] (a) The force constant, k, for the spring is 5000 N m–1.
(ii) State his velocity on landing. [1 mark] Explain the meaning of 5000 N m–1.[1 mark]
(iii) The boy bends his knees and takes 1.2 s to (b) Given the original length of the spring is 20 cm,
stop his fall. Find the impulsive force on his find the elastic potential energy stored in the
legs. spring. [2 marks]

Forces and Motion 162


(c) The string is burnt and P and Q are pushed apart from each other.
(i) Name the principle of physics in this motion. [1 mark]
(ii) If Q moves at a speed of 6 m s–1, determine the speed of P.[2 marks]
(d) Determine the value of m.[2 marks]

Essay Questions
5 Diagram 6 shows two eggs of the same mass 6 (a) What is meant by force constant of a spring?
released from the same height. One egg falls on a  [1 mark] F
concrete floor and breaks while the other one falls on (b) Diagram 8 shows an increasing force applied to O

2
a thick folded towel and remains unbroken. the right against two springs. R

CHAPTER
M

Diagram 6

(a) (i) Define momentum. [1 mark]


(ii) Relate the outcome for each material to the
change in momentum. Then, deduce a Diagram 8
relevant physical concept. [5 marks]
Draw a force-distance graph to show how the
(b) Explain the changes in energy of the unbroken
force varies with the distance moved. The force
egg from the moment it is released until it
starts from zero and increases as it moves to the
touches the towel. [4 marks]
right. [2 marks]

Explain the shape of your graph. [2 marks]


(c) Mr Arsenal applies a stretching force of 380 N to
pull a chest expander (consists of 2 springs) from
deck 30 cm to 40 cm.

truck wheel
Diagram 7

(c) Diagram 7 shows a boy in action on his


stakeboard. Diagram 9
You are asked to investigate and suggest how the
skater can improve his performance by (i) Find the force constant of each spring.
considering the following aspects: (ii) What is the stretching force needed to pull
•  Skater’s attire another chest expander of three springs
• Structure of the skateboard (deck and truck) [identical to the springs in (i)] to 52 cm?
•  Wheels of the skateboard (c) Table 1 shows 4 toy pistols with different
•  Bearings fitted to the wheel [10 marks] specifications. You are required to determine the
most suitable toy pistol that can shoot the dart the
furthest.

163 Forces and Motion


Table 1

k Characteristics of the dart:


•  Rough body
•  Density = 1.6 g cm–3
P ρ = 1.6 g cm–3


(a) (b)
Characteristics of the dart:
•  Smooth body
•  Density = 0.8 g cm–3
F
O Q
2

R
CHAPTER

M

4 (a) (b)
Characteristics of the dart:
•  Rough body
•  Density = 1.2 g cm–3

R F
O
2

R

CHAPTER

M
(a) (b)
4 Characteristics of the dart:
•  Smooth body
•  Density = 2.0 g cm–3
S


(a) (b)
Study the specifications of all four toy pistols from the following aspects:
–  The force constant of the spring, k –  The density of the plastic dart
–  Compression of the spring –  The body of the dart
Explain the suitability of each aspect.
Justify your choice. [10 marks]

Experiment
1 A student carries out an experiment to study the relationship between the speed, v, of a trolley and the distance of
compression, e, of a spring that will push the trolley down the track after the compression of the spring is released.
A friction-compensated runway and a ticker-timer with its accessories are set up as shown in Diagram 1. The trolley is
pushed back to compress the spring with a compression, e, of 1.0 cm. The trolley is released. It moves at a constant
velocity, v, down the runway, after it is pushed by the compressed spring.

a.c. power supply

Diagram 1

Forces and Motion 164


The procedure is repeated with compression values of e, 2.0 cm, 3.0 cm, 4.0 cm and 5.0 cm.
The ticker tapes are shown in Diagrams 3(a), (b), (c), (d) and (e) respectively.
Diagram 2 shows the parts of the actual size ticker tape obtained in the experiment.
The velocity, v, of each compression can be calculated by using the formula:
x
v= cm s–1
                     0.2
in which x is the distance covered in an interval of 10 ticks. An example is shown in Diagram 2.

Diagram 2  Part of a ticker tape at a constant velocity


F
O

2
R

CHAPTER
M

4
e = 1.0 cm
(a) 


e = 2.0 cm
(b) 


e = 3.0 cm
(c) 


e = 4.0 cm
(d) 


e = 5.0 cm
(e) 
Diagram 3
(a) In the experiment described above, identify
(i) the manipulated variable, [1 mark]
(ii) the responding variable, [1 mark]
(iii) the constant variable. [1 mark]
(b) (i) Using the method shown in Diagram 2, measure x of every ticker tape in Diagram 3 with a ruler.
x
(ii) Using the formula v = , calculate the value of v for every ticker tape.
0.2
Tabulate your results for e, x and v. [7 marks]
(c) On the graph paper provided, draw a graph of v against e. [5 marks]
(d) Based on your graph, state the relationship between v and e. [1 mark]

COMPANION WEBSITE 165 Forces and Motion


Online Tests
FORM 4

3
CHAPTER

Forces and Pressure

SPM Topical Analysis


Year 2007 2008 2009 2010 2011
Paper 1 2 3 1 2 3 1 2 3 1 2 3 1 2 3
Section A B C A B A B C A B A B C A B A B C A B A B C A B
Number of questions 7 1 1 – – – 8 1 – – – 1 7 1 1 – 1 – 8 – 1 – – – 5 – – 1 1 –

ONCEPT MAP

FORCES AND PRESSURE

Pressure

Gas pressure and


Pressure in
atmospheric
liquids
pressure
F
P=
A

P = ρgh Instruments for measuring


gas pressure and
atmospheric pressure

Pascal’s Archimedes’ Bernoulli’s


principle principle principle

Hydraulic Buoyant Pressure in


systems force moving fluids

• Hydraulic brake • Submarine • Aerofoil


• Hydraulic jacks • Hot-air balloon • Bunsen burner
• Hydrometer • Carburettor

COMPANION WEBSITE
166 Learning Objectives
3.1 Understanding Pressure

What is Pressure?

Applications/phenomena of pressure in our daily life:

F
O

3
R

CHAPTER
M

(a) Water pressure in a (b) High air pressure in a (c) Low pressure under the feet
fountain forces the water car tyre helps it to prevents the wearer from
spray high in the air. support a heavy load. sinking into soft ground.

Pressure

1 Pressure is defined as a perpendicular force Hence, the pressure at the base of a high-heeled
acting on one unit area of a surface. shoe is greater than that of a flat shoe because
normal force, F
the cross-sectional area of a high-heeled shoe is
surface smaller.
area

Figure 3.1

Pressure = Normal force


2000 Pa
Area (a) The base of a high-heeled shoe is small; as a result, the
F
P= pressure that acts on the ground is greater.
A

F = Force acting perpendicularly on a surface,


in newtons (N)
A = Surface area, in square metres (m2)
2 The SI unit of pressure is pascal (Pa). It is
20 Pa
equivalent to a force of 1 newton acting on a
surface area of 1 square metre. (b) The base of a flat shoe is large; as a result, the pressure
that acts on the ground is less.
1 Pa = 1 N m–2 Figure 3.2
4 The pressure on a surface also depends on the
3 The pressure exerted by a force on a surface direction the force is applied. If a force acts at a
depends on the force as well as the area of the known angle to a surface, only the component
surface. The pressure of a given force increases of the force which is perpendicular to the
as the surface area decreases. surface is used to determine the pressure.

167 Forces and Pressure


1
Figure 3.3 shows a cardboard box placed on a soft If F is the weight of the cardboard box, then:
ground in two different orientations. F 24 N
Pressure, P1 =
weight = 24 N A1 = 6 m2
= 4 N m–2
weight = 24 N = 4 Pa
Case (b): the area is
A1 A2 larger, hence
the pressure
is less 24 N
(a) (b)
P2
F Figure 3.3
O
3

R Calculate the pressure exerted on the soft ground in each


case.
CHAPTER

M
The surface area in contact, A2 = 3 m × 4 m
4 Solution the area is = 12 m2
Case (a): smaller, F 24 N
Pressure, P2 =
A2 = 12 m2
hence the
pressure is 24 N
more
= 2 Pa
P1
Note: In case (b), the force is spread over a larger area,
so that the pressure, force on each square metre,
is reduced.
The surface area in contact, A1 = 3 m × 2 m = 6 m2

The figure below shows a student pressing a drawing pin The pressure exerted by the thumb on the head of
with a force of 15 N into a piece of wood. The surface the drawing pin, P1
area of the head of the pin is 1 cm2 and the cross- F 15 N
= =
sectional area of the tip of the pin is 0.01 cm2. A1 1 × 10–4 m2
thumb = 1.5 × 105 N m–2
= 1.5 × 105 Pa
(b) F = 15 N,
Cross-sectional area of the tip of the drawing pin,
head of drawing pin A2 = 0.01 cm2 = 1 × 10–6 m2
The pressure of the tip of the drawing pin on the
wood, P2
wood tip of drawing pin
F 15 N
= = = 1.5 × 107 Pa
A2 1 × 10–6 m2
Calculate
(a) the pressure exerted by the student’s thumb on the Insight
head of the drawing pin, The pressure exerted by a given force increases as the
(b) the pressure of the tip of the drawing pin on the wood. surface area decreases. It is to be noted that the pressure
Picture the problem exerted at the tip of the
drawing pin is 100 times
The same force is applied in either case. The difference is greater than the pressure pressure P1
the size of the areas over on the force is applied. exerted at the head of the
150 000 Pa
Solution drawing pin! This pressure
drives the pin into the wood.
(a) F = 15 N,
Surface area of the head of the drawing pin, pressure P2
A1 = 1 cm2 = 1 × 10–4 m2 15 000 000 Pa

Forces and Pressure 168


Applications of Pressure in Our Everyday Life

In our everyday life, there are situations which require high pressure and those which require low
pressure.

F
P=
A

Application of high pressure Application of low pressure


P when F or A P when F or A
F
O

3
R

CHAPTER
M

Reducing area to increase pressure Increasing area to reduce pressure 4

The foundation of a
building has a large
horizontal base area in
contact with the ground.
This reduces the pressure
beneath so that the
The area of the sharp edge of a knife’s blade is
building does not sink
extremely small. The high pressure thus produced
further into the ground.
from an applied force enables the knife to cut
through material. Similarly, a pair of scissors with
its two sharp edges cuts with ease.
An ice-skate has a narrow metal
blade fixed to its sole. The high
pressure exerted by the edge of
the blade melts the ice allowing
the skater to glide smoothly The large soles of snow shoes reduce the pressure
across the surface of the ice. on the snow in order to prevent the wearer from
The sole of a football boot has sinking. That is why a polar bear has wide paws.
sharp studs which come into The large tyres of a tractor have a
contact with the ground at a large area of contact with the
high pressure. This gives the ground. The reduced pressure on
player a better grip of the the surface of the soil or road
ground as he dribbles the ball. prevents the tractor from sinking
into the ground.

An elephant is able to move on soft ground or mud


A nail can easily penetrate a wall because under the because the large surface area of its feet reduces the
small area of the tip of the nail, the pressure is very pressure on the ground. With all four feet on the
high. In the same way, a sharp arrow can easily ground, the total surface area is very large. Hence,
pierce a target and a woodpecker is able to pierce less pressure is exerted on the ground. This also
and chip the bark off a tree with its sharp beak. accounts for the broad base of a crocodile.

169 Forces and Pressure


1 SPM
Clone
’08

A man placed some durians in a plastic bag and


carried the bag. After a while, the man noticed that
the plastic bag is torn. Why?
Comments
The surface area of the durians that is in contact with
the plastic bag is small. As a result, the pressure
exerted by the durians on the plastic bag is great.
This is why the plastic bag was easily torn.
F
O
3

R
CHAPTER

M
3.1
4
1 The weight of a wooden block with dimensions 4 The mass of a car is 1000 kg. If the area of contact
0.2 m × 0.3 m × 0.8 m, as shown in the figure, is between the tyres and the road surface is 25 cm2,
12 N. calculate the pressure exerted by each tyre on the
Calculate surface of the road.
(a) the maximum 5 A boy tries to pop a balloon by using his finger and
pressure, 0.2 m then a needle.
0.8 m
(b) the minimum needle
pressure 0.3 m
that the block exerts
on the floor.
2 Azmi placed a block of copper, with dimensions
1 cm × 2 cm × 5 cm, on his palm. What is the
minimum pressure exerted on his palm? (a) (b)
[Copper density = 500 kg m–3]
(a) Find the pressure exerted on the balloon if he
3 Wen Hui has a mass of 47 kg. She wears a pair of presses the balloon with a force of 1.8 N using
high-heeled shoes that cover 405 cm2 of floor area. (i) his finger; assuming the area of his fingertip
(a) What is the average pressure (in kPa) that her is 1.2 × 10–4 m2,
shoes exert on the floor? (ii) a needle; assuming the area of the needle tip
(b) How does the pressure change when she puts all is 2.4 × 10–7 m2.
her weight on the heel of one shoe with a (b) What conclusion can be drawn from the answers
surface area of contact of 1.5 cm2? to (a)(i) and (ii)?

3.2 Understanding Pressure Pressure in Liquids


in Liquids 1 A liquid in a container exerts pressure due to
A diver experiences discomfort in his body and ears the distribution of its weight. It also exerts
when he dives deep underwater. pressure on any object in it.
Why does this happen? 2 The pressure at any point in a liquid acts in all
This is because water directions. It also increases with depth because
exerts a pressure on P the deeper
the body of the diver. the point, the
greater the pressure acts in all directions
weight of the pressure
increases
liquid above with depth

it.
Figure 3.4 Figure 3.5

Forces and Pressure 170


The force acting on the surface area A is the
Deriving the Formula for Pressure in a Liquid
weight of the liquid column above it.
Weight of liquid column
1 Figure 3.6 shows a column of liquid of density Pressure, P =
ρ in the form of a cuboid with a depth, h below Area of the base of liquid column
the surface of the liquid, and base area, A. = W
A
liquid
A = Area of base Ahρg
surface =
h = Depth of base A
uniform
column
g = Gravitational
field strength ∴ P = hρg
liquid
density, ρ ρ = Density of
liquid 4 The SI unit of pressure is pascal (Pa), which is F
equivalent to 1 N m–2, if h is measured in O

3
R
Figure 3.6 metres (m), ρ in kg m–3, and g in N kg–1.

CHAPTER
M
2 The force acting on the base of surface area,
4
A is the weight of the column of liquid above it.
3 The formula for the pressure at a depth, h in
the liquid can be derived as follows:
Volume of liquid column, V = Ah
Mass of liquid column, m = ρV Note that in deriving the formula for pressure, the
= Ahρ expression for the surface area, A is cancelled out.
Weight of liquid column, W = mg This shows that the pressure in a liquid does not
= Ahρg depend on the surface area.

3.1

Variation of pressure with the depth and the density of a liquid

Aim
To investigate the relationship between the pressure in a liquid and
(a) the depth of the liquid
(b) the density of the liquid

Case A Case B
Hypothesis The pressure in a liquid increases when The pressure in a liquid is directly
the depth of the liquid increases. proportional to the density of the
liquid.
Variables
(a) Manipulated Depth of liquid, y Density of liquid, ρ
(b) Responding Pressure in liquid, h Pressure in liquid, h
(c) Fixed Density of liquid, ρ Depth of liquid, y

Note
The pressure at a depth, y, of the liquid is represented by the difference between the water levels in a manometer, h.
Apparatus/Materials
Measuring cylinder, thistle funnel, rubber tube, manometer, and metre rule.

171 Forces and Pressure


(A) Relationship between the pressure in a liquid Conclusion
and the depth of the liquid It is observed that the difference between the water
levels in the manometer, h increases as the depth, y of
Arrangement of apparatus
the thistle funnel is increased.
rubber tube This shows that the pressure in a liquid increases
metre
rule with depth.
measuring The hypothesis is valid.
cylinder
water
thistle funnel
thin piece
manometer (B) Relationship between the pressure in a liquid
of rubber water and the density of the liquid
F Procedure
O Figure 3.7
1 The apparatus is set up as shown in Figure 3.7.
3

R
2 The thistle funnel is lowered into the water to a
CHAPTER

M
Procedure depth, y = 50.0 cm and the difference between the
4
1 The measuring cylinder is completely filled with water levels in the manometer, h is measured with
water. a metre rule.
2 The thistle funnel is connected to the manometer 3 The thistle funnel is withdrawn and cleaned.
with a rubber tube. The apparatus is set up as 4 Steps 2 and 3 are repeated with different liquids
shown in Figure 3.7. in the measuring cylinder as follows:
3 The thistle funnel is lowered into the water to a (a) alcohol (density = 800 kg m–3)
depth, y =10.0 cm. The difference between the (b) glycerin (density = 1300 kg m–3)
water levels in the manometer, h is measured with 5 A graph of h against density, ρ is drawn.
a metre rule.
4 Step 3 is repeated with values of depth, Tabulation of data
y = 20.0 cm, 30.0 cm, 40.0 cm, and 50.0 cm.
Table 3.2

Tabulation of data Liquid Density, h (cm)


ρ (kg m–3)
Table 3.1
Alcohol 800 hmin
Depth, y (cm) h (cm) Water 1000
10.0 hmin Glycerin 1300 hmax
20.0
30.0
Graph
40.0
50.0 hmax

Graph
h (cm)
O

Figure 3.9
Experiment 3.1

Conclusion
The graph of h against ρ shows that the pressure at a
y (cm) particular depth in a liquid is directly proportional
O
to the density of the liquid.
Figure 3.8 The hypothesis is valid.

Forces and Pressure 172


Characteristics of Pressure in a Liquid

Characteristics of pressure in a liquid

The pressure in a liquid increases The pressure in a liquid acts in all directions
with depth If the thistle funnel, bent at different angles as
The pressure of water is highest at the shown in the figure below is placed at the same
lowest point of the cylinder as the depth y as in Experiment 3.1, the manometer will
water spurts out the furthest from this indicate the same reading h. A liquid exerts a
point. pressure on every surface in contact with it
water supply regardless which way the surface is facing. F
weakest jet of water O

3
R
h1

CHAPTER
h2 M

h3 strongest jet 60 4
of water
50
40
30 h

y 20
10

F4/3/17
The pressure at two points at the same level in
the same liquid are equal F4/3/18
When the container is drilled with several identical The pressure in a liquid does not depend on the
holes at the same height, water will spurt out at area of its surface
equal speeds to reach equal distances from the
circumference of the base.

h
water level water
P1 P2 h
P3 water
jet of P1 P2
water

identical holes
at the same level P1 = P2
P1 = P2 = P3 sinceF4/3/20
h is the same
  

The pressure in a liquid does not depend on the shape or


size of the container
In the figure below, the diameters and shapes of the tubes are
different. The pressure at the same depth is the same since the
PA = PB
PC = PD water levels in each tube is the same.
C D

A B

F4/3/19
173 Forces and Pressure
4 The water at the bottom of the dam is at a
Applications of Pressure in Liquids
higher pressure than at the top. Hence, the wall
Public Water Supply System of the dam has to be thicker at the bottom to
sustain this greater water pressure.
1 Figure 3.10 shows a public water supply system. dam
water
reservoir tank

water water
pump house tank
B
river pump Figure 3.12
A
F
O PA > PB since A is at a greater depth.
3

R
CHAPTER

M Transfer of medicine in a saline solution to a patient


Figure 3.10
by intravenous infusion
4
2 The water from a river or dam is pumped into a
reservoir located at a high elevation.
3 In the process, the water is filtered and
chlorinated to kill bacteria. h
4 The reservoir is placed at an elevated location
so that the water will have sufficient pressure
to flow to consumers located at lower levels.
5 Normally, every home has a water tank above
the roof for storage and to provide a constant
Intravenous infusion is a method of transferring liquids
water pressure. High rise buildings have to such as medicine in a saline solution into the blood
operate their own pumps to deliver the water stream of a patient. A bottle is hung at an elevated
to tanks at the higher level. position so that the liquid solution has sufficient
6 The water pressure, as we normally call it, is pressure to flow into the veins of the patient.
actually the pressure difference between A and B.

Construction of Dams

1 Dams are built across rivers to stop their flow The pressure of water at a depth of approximately 4000 m
under the sea is about 400 times greater than the
and to form lakes for the supply of water.
pressure at sea level. Pressure of this magnitude will crush
2 Apart from supplying water, the high water human beings and even ships unless they are protected
pressure at a dam is also used to drive turbines by special equipment or made with special materials.
for the generation of hydroelectricity.
dam
reservoir
generator

turbine

Figure 3.11
(a) (b)
3 The turbine is placed at a much lower level
than the water in the reservoir. The difference The figures show that the deep-sea diving vessel and
the diver’s suit are built using special metal and glass to
in height causes a high pressure at the turbine
withstand the enormous pressure of sea water at great
and this pressure is strong enough to drive the depths.
turbine and generate electricity.

Forces and Pressure 174


2 2 SPM
Clone
’08
A submarine has a surface area of 300 m2.
The density of sea water is 1050 kg m–3. The figure below shows a reservoir on a hill. The
(a) At what depth under the sea, will the additional reservoir supplies water to a water tank on a tall
pressure on the submarine be 5 × 105 Pa? building.
(b) If the submarine is at the depth determined in (a), tank on a
calculate the force exerted by sea water on the 4m reservoir tall building
surface of the submarine.
[Take g = 10 N kg–1]
X
26 m
Solution
16 m
F
O

3
R
Calculate the water pressure at X.
F4/3/35

CHAPTER
M
(Density of water = 1000 kg m–3)
4
Comments
The water pressure here is actually the pressure
difference due to the difference in height between
(a) From the pressure formula, P = hρg:
the reservoir and the tank.
5 × 105 N m–2 = F4/3/24
hρg
h= 5 × 105 N m–2 Answer
(1050 kg m–3)(10 N kg–1) Difference in height = 30 – 16
= 47.62 m = 14 m
(b) Force, F F Applying P = hρg:
P=
= Pressure × Surface area A P = 14 × 1000 × 10
= 5 × 105 × 300 ∴ F = PA = 1.4 × 105 N m–2
= 1.5 × 108 N

3 SPM
Clone
’09
3
The figure shows a pond with two fish P and Q.
A coin sinks to a depth of 5 m in a liquid of density
800 kg m–3. At what depth, in a different liquid of
water
density 600 kg m–3, will the coin be subjected to the P
same pressure?
Solution
Q

h1 = 5 m P1 800 h2 P2 600
50 kg m–3 kg m–3
50
What is the correct relation of the water pressures on
fish P and fish Q?
Since the pressures are the same, then:
P1 = P2 Answer
h1 ρ1g = h2 ρ2g Water pressure on fish Q > Water pressure on fish P.
hρ Pressure of water increases with the depth of water.
h2 = 1 1 Presure of liquid, P = ρgh
ρ2
5 × 800 where ρ = density of liquid
= g = gravitational field strength
600
= 6.67 m h = depth of liquid

175 Forces and Pressure


3.2

1 An air bubble is at a depth of 3 m below the surface 3 A plumber noticed that the bath in Raj’s home was
of a lake. What is the pressure of water on the air not functioning satisfactorily as the water pressure at
bubble? the shower head was too low.
[Take density of water = 1000 kg m–3]
2 The figure shows a dam with a depth of 25 m.

water

F
O
3

R dam
(a) What is the water pressure at the shower
CHAPTER

M
(a) Calculate the water pressure at the bottom of the head?
4 dam when the water is at its maximum level. (b) What must the plumber do to increase
[Take density of water = 1000 kg m–3, the water pressure at the shower head to
g = 10 N kg–1] 15 000 Pa?
(b) Explain why the width of the wall of the dam is [Take density of water = 1000 kg m–3,
much larger at the base than at the top. g = 10 N kg–1]
F
O
3

R
CHAPTER

3.3 Understanding4 Gas


Gas Pressure
Pressure and Existence of Gas Pressure
Atmospheric Pressure
1 Figure 3.14 demonstrates how gas pressure can
In 1654, Otto von Guericke, the mayor of the be understood in terms of moving molecules.
German town of Magdeburg, put together two A weighing machine with its scale pan inverted
large, hemispherical, copper cups that fitted to is used. A stream of marbles is poured over the
form a hollow spherical ball. He then pumped out inverted scale pan. The pointer on the scale
the air from within the ball. When he had done shows a constant reading.
this, the two cups were held together so firmly that
it was impossible to pull them apart even with two
teams of eight horses hitched to the hemispheres!
marble
Do you know what caused the cups to hold
together so strongly?
inverted scale pan
rubber washer copper
hemisphere
0
10 1

9 2 weighing
machine
8 3

7 4
6 5

Figure 3.13
Note: The stream of marbles represents
moving molecules of gas.

Figure 3.14

Forces and Pressure 176


2 The irregular impact of a large number of Factors Affecting the Air or Gas Pressure
marbles produces a fairly constant average
force on the scale pan, as shown by the reading 1 Pressure increases when the density of gas
on the scale. increases.
3 The irregular impact of the marbles simulates
elastic gas molecules in a container bouncing
off its walls. As the molecules bounce off the
container walls, they exert an average force on
the walls which is known as gas pressure.

Kinetic Theory of Gases Density


Density Density
Density
Pressure
Pressure Pressure
Pressure
F
1 The kinetic theory of gases is based on the Figure 3.16 O

3
following assumptions: R
2 Pressure increasesF4/3/27
F4/3/27temperature increases
when
(a) The molecules in a gas move freely in

CHAPTER
M
(kinetic energy of molecules increases).
random motion and possess kinetic energy.
4
(b) The forces of attraction between the
molecules are negligible.
(c) The collisions of the molecules with each
other and with the walls of the container
are elastic collisions.
2 The molecules of a gas in a container move in
all directions to fill the entire space of the Temperature Temperature
Pressure Pressure
container until they collide with its walls.
Figure 3.17

F4/3/28
Atmospheric Pressure

Existence of Atmospheric Pressure

1 According to the kinetic theory of gases,


gases consist of molecules which are far apart
and in random motion at high speeds.
Figure 3.15 Particles of a gas constantly collide 2 The gas molecules possess mass and experience
F4/3/26
with the walls of the container the gravitational pull. The result is that gases
have weight.
3 The collisions of the gas molecules with the 3 The atmosphere is a thick layer of air that
walls of the container are elastic collisions and surrounds the Earth.
the molecules rebound with the same speed 4 The atmosphere exerts a pressure called
which results in a change in momentum for atmospheric pressure which is caused by the
each molecule. weight of the thick layer of air above the
4 The total change of momentum when the gas Earth’s surface.
molecules collide with the walls of the 5 Atmospheric atmospheric pressure is
container in one second produces a force pressure acts all around us
which acts on the walls of the container. on every object
5 By the definition of Pressure = Force , the gas on the surface
Area of the Earth.
pressure on the wall is thus obtained by
dividing the force with the total area of the
walls in the container.
6 In short, the pressure in a closed container is
caused by the collision of the gas particles on
the wall of the container. Figure 3.18

177 Forces and Pressure


To Show the Existence of Atmospheric Pressure

Table 3.3 shows certain phenomena which are the result of atmospheric pressure.

Table 3.3
Activity Observation
A glass is filled to the brim with water and covered with a piece
of cardboard. The glass of water is swiftly inverted. It is noted
glass water that the water remains in the glass as shown in the diagram.
cardboard The explanation for this phenomenon is that the atmospheric
pressure outside acts on the surface of the cardboard pushing it up.
atmospheric
pressure The resultant force on the cardboard is greater than the weight of
F
O the water in the glass.
3

R air is driven cold An empty metal tin is filled with some water and the tin is heated
out of the can water
CHAPTER

M until the water boils. The tin is then capped and cooled down
by heating

4 tin
with tap water. As the steam condenses, the pressure inside the
atmospheric metal tin decreases. The external atmospheric pressure, which is
boiling pressure
water higher, crushes the tin.
the evacuated tin
collapses under
the influence of F
atmospheric pressure
O
3

F4/3/30 R
CHAPTER

4
It is difficult to pour out milk from a tin with one small When an additional hole is made, atmospheric air fills the
hole. However, when an additional hole is made, milk can space in the tin, so the pressure inside and outside are
be easily poured out. Why? balanced. Milk then flows out easier as there is no
external pressure against it.
atmospheric
partial vacuum air flows in

atmospheric milk
pressure

Solution F4/3/31a
As milk is poured out, the pressure in the tin decreases.
The atmospheric pressure, which is higher, acts against F4/3/31b
the milk flowing out.

Mercury Barometer

1 A mercury barometer consists of a thick-walled vacuum


glass tube, which is closed at one end, as
shown in Figure 3.19.
2 The tube is completely filled with mercury and atmospheric
inverted several times to remove air bubbles. pressure
mercury
The tube is then completely filled again with
mercury.
(a) (b) (c)
3 After all air has been removed, the open end of
the glass tube is inverted into a container of Figure 3.19 Mercury barometer
mercury. F4/3/32

Forces and Pressure 178


4 The mercury column drops until it reaches a height of about 76 cm
above the lower surface. The space between the top of the mercury and
the end of the tube should contain no air; it is a complete vacuum. The vertical height of the
(Actually, the space contains a little mercury vapour.) column of mercury in a
5 The column of mercury in the tube is supported by the atmospheric mercury barometer will not
pressure and its height depends on the magnitude of the atmospheric vary when the glass tube is
pressure. tilted as shown in Figures
3.19(b) and (c). If the
mercury is replaced by water,
Magnitude of Atmospheric Pressure and Its Units for example, a longer tube
(more than 10 m long) will
be required because the
density of water is very much
atmospheric lower than mercury. F
760 mm pressure O

3
R

CHAPTER
M
mercury
(density = 13 600 kg m–3)
4
Figure 3.20
F4/3/47
1 Since the atmospheric pressure at sea level can support a vertical
column of mercury 76 cm or 760 mm high, we can, for convenience,
express mm Hg as a unit of pressure.
1 standard atmospheric pressure (1 Patm) = 76 cm Hg or
= 760 mm Hg
1 bar = 1000 millibar
also known as one atmosphere = 1.01 × 105 N m–2
Take g = 9.8 N kg–1 1 millibar = 101 N m–2
2 In unit N m–2 (pascal) or N cm–2:
1 Patm = 76 cm Hg
= hρg 105 N
Patm = ––––––
= 0.76 × 13 600 × 9.8 1 m2
5 –2 105 N
= 1.01 × 10 N m = –––––––
104 cm2
= 105 N m–2 (approximately) = 10 N cm–2
= 105 Pa
3 In unit bar or millibar:
The unit of pressure used for meteorological purposes is called the bar.
1 Patm = 1.01 × 105 N m–2 = 1 bar
4 In unit m water:
What is the height of a water column the atmosphere can support? h = 10 metres

h
Patm

water

Figure 3.21
F4/3/48
5 –2
1.01 × 10 N m = hρg
Density of water = 1000 kg m–3
= h × 1000 × 9.8 g = 9.8 N kg–1 Figure 3.22 Using a water
∴ h = 10.3 m barometer to
= 10 m (approximately) measure
F4/3/33the
∴ 1 Patm = 10 m water atmospheric pressure

179 Forces and Pressure


Applications of Atmospheric Pressure
Rubber Sucker
Drinking Straw When a person sucks air is
on the straw, the forced out
atmospheric
pressure in the straw pressure
The external decreases.
atmospheric partially
pressure, vacuumed
(low pressure)
which is
greater, will
then act on
the surface of atmospheric
the water in pressure When the rubber sucker is The surrounding
F the glass pressed onto a smooth surface, atmospheric pressure,
causing it to usually a glass or tiled surface, which is much higher,
O
rise through
3

R liquid the air in the rubber sucker is acts on the rubber


the straw. forced out. This causes the space sucker, pressing it
CHAPTER

M
between the surface and the securely against the
sucker to have low pressure. wall.
4

Syringe
A rubber syringe is used to draw a liquid from a container.

When the plunger is


pulled upwards, the
When the
pull up air pressure in the
plunger is
syringe is reduced,
pushed push down air pressure and a partial vacuum
down, the air decreases plunger
is created.
pressure
inside the air pressure atmospheric pressure
syringe increases The surrounding
increases. atmospheric pressure,
The air is which is much higher,
forced out of air is
forces the liquid up
syringe. forced out
into the syringe.

down
Lift Pump
up up
A lift pump is used to
pump water out of a
A plunger A A
well or to a higher level. spout
The greatest height to atmospheric
B B
B B
which water can be pressure
pumped is 10 m. This is
equivalent to the
atmospheric pressure.
upstroke downstroke upstroke

When the handle is lifted, valve A When the handle is When the handle is next lifted, valve A closes and valve B opens.
closes and valve B opens. The pushed down, valve The atmospheric pressure, acting on the surface of the water,
atmospheric pressure, acting on the B closes and valve forces water past valve B into the cylinder. Simultaneously, the
surface of the water, causes water to A opens. Water water above the plunger is lifted and flows out through the spout.
flow past valve B into the cylinder as flows above the This process is repeated until the required amount of water has
shown. plunger. been pumped out.

Forces and Pressure 180


Atmospheric Pressure and Altitude
Vacuum Cleaner
1 Atmospheric pressure decreases with altitude, or
The principle of atmospheric pressure is used in a the height above sea level.
vacuum cleaner to remove dust particles. 2 At higher altitudes, the density and the temperature
of the air are lower. As a result, the frequency of
When a vacuum cleaner is switched on, the rotating fan collisions of the molecules is lower. Hence,
sucks out the air from space A. Space A then becomes a
partial vacuum.
atmospheric pressure is lower.

dust bag
partially height (km)
vacuumed Pressure at sea
fan level is 1 atmosphere
A or 1 atm.
1 atm = 105 N m –2 or 105 Pa
F
air dust O

3
R
dust jet plane

CHAPTER
motor M
Mount Everest
atmospheric pressure
atmospheric 4
Sea pressure (103 Pa)
The atmospheric pressure outside, which is greater, then
level
forces air and dust particles into the filter bag. This traps
the dust particles but allows the air to flow through an exit Figure 3.23
at the back.

4
Siphon
h
A rubber tube can be used to siphon liquid from
a container on a higher level to another on a
lower level. For example, we can remove petrol
from the petrol tank of a vehicle, or dirty water
from an aquarium. 76 cm
1

atmospheric
pressure
container A Figure 3.24
Find the height of a simple mercury barometer
at the top of a mountain where the atmospheric
h
pressure is 0.6 × 10 5 N m–2.
P
[Given: Hg = 13 600 kg m–3 and g = 9.8 N kg–1]
container B
Solution
P=h g
0.6 × 10 5 = h′ × 13 600 × 9.8
The tube is first filled with the liquid and one end is
placed in the liquid in container A. The other end is h′ = 0.45 m
placed in container B on a level which must be lower than = 45 cm
the surface of the liquid in container A.

The pressure in the rubber tube at point P


= Atmospheric pressure + Pressure of h cm of liquid column
The higher atmospheric pressure pushes water into the
As the pressure at point P is greater than the atmospheric
tube.
pressure, the liquid flows out at P.

The liquid in container A can be completely drained or the


The pressure in the rubber tube decreases as the water flows flow stops as soon as the liquid surface in container A
out and a partial vacuum is created. reaches the same level as P at the tube.

181 Forces and Pressure


Total Pressure below the Surface of a Liquid
6
1 The formula for liquid pressure, P = hρg, is
used to determine the additional pressure as a Figure 3.26 shows a mercury barometer at sea level.
result of the weight of the liquid at any point
below the liquid surface.
2 The total pressure acting at a depth, h, below A
76 cm
the liquid surface is equal to the pressure of the 46 cm
weight of the liquid (P) and the atmospheric
pressure acting on the liquid surface. 10 cm
atmospheric pressure, Patm B C 50 cm

F D
O water pressure, P Figure 3.26
3

R
h
CHAPTER

M (a) Find the total pressure,


F4/3/59 in cm Hg,
(i) at A, (ii) at B and C.
4
(b) Calculate the total pressure at D in
Total pressure acting on the fish (i) cm Hg, (ii) unit N m–2.
=F4/3/34
Patm + hρg [Given ρHg = 13 600 kg m and g = 9.8 m s–2]
–3

Figure 3.25 Solution


F
O (a) (i) PA = 76 – 46
3

R = 30 cm Hg
CHAPTER

M (ii) PB = PC
= 76 + 10
4 It is always easier
= 86 cm Hg
5 to solve the
(b) (i) PD = 76 + 50 problem by
Calculate the total pressure on the body of a diver at = 126 cm Hg expressing the
(ii) PD = hρg total pressure in
a depth of 450 m under water. cm Hg before
[Take density of sea water = 1200 kg m–3, = 1.26 × 13 600 × 9.8 calculating hρg.
gravitational field strength = 10 N kg–1, and = 1.68  105 N m–2
atmospheric pressure = 105 N m–2]
Solution
Instruments for Measuring Atmospheric
Pressure

Mercury Barometer

• This was explained vacuum


on pages 178 and
179. mercury
h
atmospheric atmospheric
pressure pressure

Pressure due to the


weight of sea water
Total pressure = hρg Figure 3.27 A simple mercury barometer
= Patm + hρg F4/3/36
= 105 + (450)(1200)(10) • At sea level, or 1 atmosphere pressure, mercury
= 105 + 5 400 000 level, h is at 76 cm Hg.
= 5 500 000 N m–2 • When atmospheric pressure decreases at high
= 5500 kPa altitudes, the mercury level drops.

Forces and Pressure 182


Fortin Barometer

• A Fortin barometer is a type of mercury barometer which has a higher


accuracy.
• The atmospheric pressure is measured in mm Hg.
main scale
vernier scale
Gives a more accurate
reading of the
screw to adjust atmospheric pressure.
vernier scale

brass tube
F
barometer tube O

3
mercury
R

CHAPTER
M

ivory pointer 4
glass
mercury
To eliminate the zero error:
Adjusts the screw until the
screw to adjust
mercury level
pointer touches the surface
of the mercury.
Figure 3.28 Fortin barometer

Aneroid Barometer

• An Aneroid barometer (Figure 3.29) is a more convenient form of


The Aneroid barometer,
barometer because it does not use any liquid. besides being used for
To prevent the pressure and altitude
pointer metal case from measurement, is also used as
collapsing. pointer
fine chain a weather glass to forecast
4
lever system the weather. Rain clouds form
6 5
pivot in large areas of lower
lever pressure air, so a fall in the
barometer reading often
metal case chamber means that bad weather is
coming.
metal case A partially evacuated metal case in the
form of a flat cylinder with flexible walls.
(a)
(b)
Figure 3.29
F4/3/43

• The chamber expands and contracts in response to changes in atmospheric


pressure.
• The movement of the chamber walls is transmitted by a mechanical lever
system which moves a pointer over a calibrated scale.
• The Aneroid barometer can be used as an altimeter (to determine altitude)
by mountaineers or by pilots to determine an airplane’s altitude. This is
possible because the atmospheric pressure decreases with altitude. The
scale can be calibrated to give readings of altitude equivalent to a range of
values of atmospheric pressure.

183 Forces and Pressure


Instruments for Measuring Gas Pressure

Manometer

• A manometer consists of a U-tube filled with a • To measure the pressure of a gas, the arm B is
liquid (mercury, water, or oil) with a certain density. connected to the gas pipe and the gas pressure acts
• The manometer is used to measure the difference on the surface of the liquid in arm B.
in pressure between the two sides of the U-tube as • If the gas pressure is greater than the atmospheric
shown in Figure 3.30(b). pressure, the liquid in arm B will be pushed
Patm
Patm downwards. Under equilibrium conditions, the
Patm gas pressure at B is equal to the pressure at C because
supply both B and C are at the same level.
F
O • The pressure of the gas, P is the sum of atmospheric
3

R pressure (Patm) and the pressure due to the difference


CHAPTER

M in the height, h of the liquid in the manometer.


(a) (b)

4 Figure 3.30 Manometer ∴


F4/3/44 P = Patm + hρg
• When the manometer is not connected to the gas
supply, i.e. when both arms are open to the where h = manometer reading,
atmosphere, the liquid levels in both arms A and B ρ = density of liquid,
are equal as shown in Figure 3.30(a). g = gravitational field strength
F
O
3

R
Bourdon Gauge
CHAPTER

M Patm
Patm
gas
• Figure 3.31 shows a Bourdon gauge. 4 It consists
Patm • The supply
unit of measurement used in the Bourdon
basically of a coil of flattened copper tube with an gauge is pascal. Bourdon gauges are normally
oval cross section connected to a lever system. connected to gas cylinders to give an indication of
the quantity of gas in the cylinders.
• When the gas supply is connected, the pressure in
pointer (a) (b)
the gas acts to straighten the copper coil.
lever • The movement of the copper coil is transferred to
system coiled tube
the lever system which actuates a pointer to move
F4/3/44

gas supply
scale across a scale which has been calibrated to give
readings of pressure.
• Bourdon gauges are more robust than manometers
Figure 3.31 Bourdon gauge and more suitable for measuring higher pressures. But
they have to be calibrated before they can be used.

4 SPM
Clone
’08

A housewife used a plunger to unblock a blocked sink Explain how the plunger works.
pipe as shown in the diagram below.
Answer
When the housewife pulled the handle of the plunger,
a partial vacuum is created inside the rubber sucker.
The greater atmospheric pressure below the blockage
will push or dislodge the blockage, thus allowing the
water to flow again.

Forces and Pressure 184


7
Figure 3.32 shows a manometer containing mercury Solution
that has a density of 1.36 × 104 kg m–3. (a) Pressure at point B
The manometer is connected to a gas supply. Atmospheric Pressure due to
= +
pressure column AB
= 76 cm Hg + 6 cm Hg
gas
= 82 cm Hg
supply = (0.82 m)(1.36 × 104 kg m–3)(10 N kg–1)
= 111 520 N m–2
= 1.12 ×105 Pa
Pressure at C F
Figure 3.32 O
= Pressure at B (at the same level)

3
R
= 82 cm Hg

CHAPTER
(a) Calculate the pressure at points B and C in units of M
cm Hg and Pa. = 1.12 × 105 Pa
(b) Pressure of gas supply 4
[Assume g = 10 N kg–1, atmospheric pressure
= 76 cm Hg] = Pressure at point C
(b) What is the pressure of the gas supply? = 82 cm Hg
(c) If the manometer is made from a smaller diameter = 1.12 × 105 Pa
tube, what will be the effect on the height of the (c) The height of the mercury column does not
mercury column AB? Explain your answer. change because pressure is independent of the
diameter of the manometer tube.

3.3

1 What is the total force acting on the bottom of a pool The air in tubes X and Y is sucked out through tube Z
with dimensions 50 m × 100 m, at a depth of 6 m? which is then clipped. Determine the density of
Take the density of water as 1000 kg m–3 and glycerol if the density of water is 1000 kg m–3.
atmospheric pressure as 105 N m–2. 4 (a) The atmospheric pressure at sea level is
2 The figure shows a measured with a mercury barometer. If the
U-tube used to reading obtained is 75 cm, what is the
determine the atmospheric pressure, in Pa?
density of liquid X. liquid (b) Based on the information given below, calculate
When liquid X is X the pressure at a point 20 m below the surface
poured into one of the sea.
arm, the water Gravitational field strength = 10 N kg–1
level in the other Density of mercury = 1.36  104 kg m–3
water
arm rises. Density of sea water = 1025 kg m–3

Based on the information shown in the figure, 5 A U-tube is filled with mercury. Liquid P is poured into
determine the density of liquid X if the density of the X-arm of the U-tube and water is poured into the
water is 1000 kg m–3. Y-arm until the levels of mercury in both arms are the
same, as shown in the figure below.
3 The figure shows Hare’s apparatus used to determine
the density of glycerol.
liquid 12 cm
clip 8 cm
water

mercury

Determine the density of liquid P.


water glycerol water glycerol [Density of water = 1000 kg m–3]

185 Forces and Pressure


3.4 Applying Pascal’s Principle A Basic Hydraulic System

Liquid pressure can be used to transfer energy from 1 Figure 3.35 shows a basic hydraulic system.
one place to another. Have you ever wondered how It operates on Pascal’s principle.
this idea is being adopted in some of the machines INPUT OUTPUT
such as hydraulic garage lift, hydraulic car jacks and input force, F1 piston K moves
is pressed on piston J upwards, object
excavator? is lifted

piston J piston K
F2
area, A1 area, A2
F1

F
liquid
O
3

R
CHAPTER

M
high pressure is transmitted through the liquid

4 Figure 3.35 A basic hydraulic system

Figure 3.33 Excavator 2 In this hydraulic system,


F4/3/47 a small force, F1 is
applied to the small piston J resulting in a large
force, F2 at the large piston K. The pressure, due
Transmission of Pressure in a Liquid to the force F1, is transmitted by the liquid to
F
O the large piston.
1 The molecules in a liquid are quite close and in
3

R F
contact with each other. As a result, liquids are 3 Applying pressure, P = 1 , this pressure is
CHAPTER

M
A1
practically incompressible.
2 When the piston is pushed in4 by a force, F, a transmitted through the liquid and acts on the
pressure is applied to the water. base of the large piston.
F Force on the large piston, F2 = P × A2
Pressure =
A F
= 1 × A2
where A is the cross-sectional area of the piston. A1
3 Figure 3.34 shows that water will spurt out The large force pushes the load upwards.
through the openings with equal velocity
Also:
when the piston is pushed into the flask.
glass sphere with holes of uniform size
F2 A
= 2
piston of cross-sectional area, A F1 A1
Output force Output piston area
force, F =
Input force Input piston area
water

4 Because the surface area, A2, of the piston K is


Figure 3.34 much larger when compared to the surface
F4/3/46 area, A1, of the piston J, the resultant force, F2,
4 The applied pressure is transmitted uniformly is much larger than the applied force, F1. This
throughout the water in accordance with shows that in applying Pascal’s principle, a
Pascal’s principle. large force can be produced by a small force.
5 Pascal’s principle states that pressure exerted 5 Hydraulic systems act as a force multiplier
on an enclosed liquid is transmitted equally A
throughout the liquid. where 2 is the multiplying factor.
A1
6 Pascal’s principle is also known as the principle
For example, if A2 = 5A1,
of the transmission of pressure in a liquid.
then F2 = 5F1
7 Pascal’s principle is important for the
A A2
understanding of hydraulics, the study of the since F2 = F1 × 2 where ––– =5
A1 A1
transfer of forces through liquids.

Forces and Pressure 186


6 A hydraulic system must not contain any air bubbles in any portion of its hydraulic fluid
system. The presence of air bubbles in the hydraulic fluid system will reduce the efficiency of the
system as part of the applied force will be used up to compress the air bubbles.

8
A basic hydraulic system has small and large pistons Solution
with cross-sectional areas of 2 cm2 and 50 cm2 (a) F1 = 20 N
respectively. A1 = 2 cm2 = 2 × 10–4 m2
20 N F1
Pressure transmitted, P1 =
A1
2 cm2 20
= F
50 cm2 2 × 10–4 O

3
= 100 000 N m–2 R

CHAPTER
(b) P2 = P1 = 100 000 N m–2 M

Figure 3.36 A2 = 50 cm2 4


= 5 × 10–3 m2
When a force of 20 N is applied to the small piston, Force on the large piston, F2
calculate F4/3/63 = P2 × A2
(a) the pressure transmitted in the hydraulic fluid, = 100 000 × 5 × 10–3
(b) the force acting on the large piston. = 500 N

Another View on Hydraulic Systems (in equilibrium)


Since the system is in equilibrium,
W1 W2
A1 A2 pressure at X = pressure at Y
X Y
PX = PY
F1 F2
W1 W or ––– = –––
= 2 A1 A2
A1 A2
also, W1 = m1 g and W2 = m2 g
Figure 3.37
F4/3/64 m m
∴ 1 = 2 Weight = mass × g
A1 A2

9
Figure 3.38 shows a hydraulic system in equilibrium. Given that the area of the output piston is 5 times the
20 kg area of the input piston, what is the mass, m needed to
m balance the load of 20 kg?

input output Solution


A1 A2 m1 m2
=
A1 A2 where A2 = 5A1

m 20
=
A1 5A1
Figure 3.38 m = 4 kg
F4/3/48

Applications of Pascal’s Principle in Everyday Life

1 A hydraulic system is a device in which a small applied force can give rise to a larger force.
2 The principle of the hydraulic system is widely used in jacks, vehicle brake systems, hydraulic
presses and heavy machinery.

187 Forces and Pressure


Hydraulic Jacks
• A hydraulic jack is used to lift a heavy load
such as a car when a tyre is being changed. When the handle is pressed down, valve A closes and the
small piston forces hydraulic fluid through valve B to the
large cylinder. The pressure transmitted results in a large
load force on the load.
large piston


handle

large When the handle is raised, valve B closes and hydraulic


cylinder fluid flows from the buffer tank through valve A into the
valve release valve small cylinder. The handle is pressed up and down
B repeatedly until the load is sufficiently raised.


F A
buffer tank
O
3

R The large piston can be lowered at the end by opening the


small cylinder
CHAPTER

M
release valve to allow all the hydraulic fluid to flow back
Figure 3.39 Hydraulic jack into the buffer tank.
4

Hydraulic Brakes
• Hydraulic brakes are used in cars, lorries, and When the brake pedal is pressed, the piston of
motorcycles. F the master cylinder applies a pressure on the
• In a hydraulic brake system, a liquid,
O known as the brake fluid. This pressure is transmitted via a
3

R
brake fluid, is used to transmit pressure from the system of pipes to each cylinder of the wheel.
CHAPTER

M
brake pedal to all the wheels of the vehicle.
Rear wheel


brake
4
shoes brake
drum The pressure at the cylinders in the wheels
return cause a pair of pistons to push a pair of
brake spring
piston
friction pads to press against the surface of
pedal the brake discs or brake drums. The
master
cylinder frictional force between these brake
brake components causes the vehicle to slow
pads wheel
cylinder down and stop.

brake
disc
piston
When the brake pedal is released, a spring
Front wheel
brake fluid
restores the brake discs to their original
positions.
Figure 3.40 Hydraulic brake
F4/3/49
Hydraulic Pumps
• Hydraulic pumps are used in a car workshop The machine is equipped with a small cylinder connected
to raise cars. to a large cylinder. Both cylinders are filled with oil.


air compressor

ground When compressed air is introduced into the small


cylinder, the compressed air exerts a pressure on the
oil
surface of the oil.


small cylinder large cylinder


This pressure is transmitted by the oil to the large
Figure 3.41 A hydraulic pump cylinder causing the large piston to produce a force large
F4/3/50 enough to lift a car.

Forces and Pressure 188


5 SPM
Clone
’07

Sunny squeezes the toothpaste out from a tube as Which principle explains the situation?
shown below. A Archimedes C Pascal
B Bernoulli
Comments
The pressure applied by the hand is transmitted
toothpaste throughout the toothpaste in the tube. This causes the
toothpaste to be pushed out from the tube.
Answer C
F
F4/3/51 O

3
3.4 R
3.5 Applying Archimedes’

CHAPTER
M

1 The figure shows a simple hydraulic system. Principle


4
20 N force Steel is denser than water so we might expect heavy
steel ships to sink. However, they can still float. Why?
I can float!

small large
piston piston

liquid

I sink to the
The cross-sectional areas of the small cylinder and the bottom of
large cylinder are 5 × 10–4 m2 and 8 × 10–3 m2 the sea.
respectively. A force of 20 N is applied to the small
piston.
(a) State the relationship between the liquid Figure 3.42
F4/3/52
pressures in the small cylinder and the large If you try to lift a heavy object under water, you
cylinder. Name the principle in physics which find it surprisingly light and much easier to lift
defines this relationship. than when it is out of the water. Why?
(b) Calculate the force acting on the large piston.
Oh! Why does it
2 The figure shows a simple hydraulic system. Hey! This is light! become so heavy now?

20 N

A B
area area
0.1 m2 0.5 m2
Figure 3.43
Archimedes (287–212 F4/3/53
B.C.) was a Greek scientist
Assuming there is no friction in the system, who first discovered that an object submerged in a
(a) what is the pressure at A? liquid is acted on by an upward buoyant force
(b) what is the pressure at B? (or upthrust). The buoyant force is due to the
(c) what is the upward force produced? surrounding liquid which causes the object to
(d) what is the effect on the hydraulic system if
weigh less in the liquid. Archimedes realised that
(i) the cross-sectional area of the large piston
submerged objects always displace liquid upwards
is increased?
(ii) the cross-sectional area of the small piston (supposedly when he got into the bathtub, the
is decreased? level of water rose). Later, he did show that the
upthrust is equal to the weight of water displaced.

189 Forces and Pressure


Buoyant Force and Flotation

Note
20 N
The reading in Figure 3.44(a)
T1 gives the actual weight which is
tension T1
T1
= actual weight equal to the gravitational pull
15 N
that always exists whether the
actual weight stone is in the air or water.
(= gravitational tension T2
pull)
T2
= apparent weight
However, in Figure 3.44(b),
mg FB T2 water displaced some water is displaced. But
F the loss in weight due to the
buoyant force, FB
O
gravitational pull water displaced is 5 N.
3

R is still acting
mg
CHAPTER

M
(a) (b)
4
Figure 3.44
F4/3/54
1 In Figure 3.44, when a piece of stone is lowered into the water, the following observations are made.
(a) The stone experiences a reduction in weight. The weight of the stone in water is less than
its weight in air. The apparent loss in weight of the stone is caused by the buoyant force of
the surrounding water on theF stone.
O
3

Apparent Rloss in the weight of the stone


CHAPTER

= Weight M
of the stone in air – Weight of the stone in water
4
(b) The stone displaces a volume of water.
Volume of water displaced = Volume of the submerged part of the stone

(c) From Figure 3.44, the apparent loss in weight is due to the buoyant force.
Therefore:
Buoyant force, FB = Actual weight – Weight in water = 20 N – 15 N = 5 N
2 Archimedes’ principle states that an object, when it is completely or partially immersed in a
fluid, is acted on by a buoyant force, which is equal to the weight of the fluid displaced.

What Causes the Buoyant Force?


liquid surface liquid surface
1 Figure 3.45 shows a simple
P3 = P4
way of explaining the P1 P5 = P6
F1 = ρ h1gA
h1 h1
existence of a buoyant force. h2
P2 > P1
h2

The principle is also true for P6


P4
P3
an irregular object. P5
base base
area = A area = A
P2
F2 = ρh2gA
(a) The difference in pressure (b) Buoyant force
between P1 and P2 causes = F2 – F1
the buoyant force F4/3/55

Figure 3.45

2 Figure 3.45 shows a rectangular block submerged in a liquid of density, ρ. The pressure of the
liquid on the underside of the block, P2 is greater than the pressure on the upper surface of the
block, P1. The pressures acting on the sides of the block, being equal and opposite, cancel out
each other. That is, P3 = P4 and P5 = P6.

Forces and Pressure 190


3 P2 is greater than P1 because liquid pressure increases with depth. This
means that the force on the under side is greater than the force on the
upper side as F = P × A.
Archimedes’ principle:
4 The difference between the forces acting on the upper surface and the Buoyant force
lower surface is the net force acting upwards.
= Reduction in weight of
This net force is known as the buoyant force.
object
Force acting on the upper surface, F1 = P1 A = h1 ρgA
Force acting on the underside, F2 = P2 A = h2 ρgA = Weight of fluid displaced
Net force acting upwards
= Buoyant force
A(h2 – h1)
= F2 – F1 =V
= h2 ρgA – h1 ρgA = Volume of cuboid
= ρgA (h2 – h1) = Volume of liquid displaced F
Pressure due to liquid = hρg O
= ρgV

3
ρV = m Buoyant force due to liquid R
= mg = Mass of liquid displaced

CHAPTER
= ρVg M
= Weight of liquid displaced
5 Therefore, the buoyant force is equal to the weight of liquid 4
displaced, as according to Archimedes’ principle.

Activ To investigate the relationship between the buoyant force


ity 3.1 on an object and the weight of liquid displaced

Apparatus/Materials 4 The stone is then lowered into the Eureka can


Eureka can, a piece of stone, beaker, thread, spring until it is completely immersed and is freely
balance and triple-beam balance. suspended in the water without it touching the
bottom of the can as in Figure 3.46(b). The water
Arrangement of apparatus
displaced flows via the spout into the beaker.
5 The spring balance reading is noted. This is the
weight of the stone in water, W2.
6 A triple-beam balance is used to determine the
N mass of the beaker and water displaced.
spring
balance N Results
Weight of stone in air = W1
Weight of stone in water = W2
stone
Buoyant force on the stone = W1 – W2
Eureka can
Weight of empty beaker = m1g
water Weight of beaker and water displaced = m2 g
beaker beaker
Weight of water displaced = (m2 – m1)g
(a) (b) Finding: W1 – W2 = (m2 – m1)g
Figure 3.46
Discussion
Procedure (a) The loss in weight of the stone immersed in
1 The Eureka can is placed on a table and a beaker water is due to the buoyant force of the water on
is placed below the spout of the can as shown in it.
Figure 3.46(a). Water is poured into the can until (b) From the results, it is found that the loss in
it spills out through the spout. weight of the stone is equal to the weight of the
Activity 3.1

2 The beaker is replaced with an empty beaker water displaced, i.e., (m2 – m1)g.
which has been weighed on a triple-beam balance.
3 A stone is suspended from the spring balance and
its weight in air, W1, is obtained from the reading Buoyant force on the stone
on the spring balance. = Weight of water displaced by the stone

191 Forces and Pressure


10
A stone of volume 9 cm3 weighs 0.36 N in air. To sink or to rise?
Find its apparent weight in a liquid of density
0.8 g cm–3. liquid surface
liquid
rise
Solution
FB
Buoyant force, FB FB FB
ρ = 0.8 g cm–3
= ρVg = 800 kg m–3
= 800 × 9 × 10–6 × 10 float
V = 9 cm3
= 0.072 N = 9 × 10–6 m3
W
∴ Apparent weight W W
F
= Actual weight – Buoyant force
O sink
= 0.36 – 0.072
3

R
= 0.288 N
CHAPTER

M
• When an object which is held stationary in a liquid is
4 F4/3/56
released, it will normally either sink to the bottom or
rise to the surface of the liquid.
11 • If the weight (W) > buoyant force (FB), the object will
sink to the bottom.
The weight of a metal cylinder is measured at • If the weight (W) < buoyant force (FB), the object will
F (b) and (c).
positions as shown in Figures 3.47(a), rise to the surface.
O
3

R
CHAPTER

4 Buoyancy
W
1 Buoyancy is actually an application of
Archimedes’ principle.
FB3
2 When a piece of wood of a higher density than
(c) water is placed in water, it sinks and displaces
liquid
some water.
FB2
(b)
3 As it sinks, more and more water is displaced.
FB1 This increases the buoyant force since FB is
(a)
equal to the weight of water displaced.
Figure
F4/3/753.47 4 The wood will sink until the buoyant force is
(a) Compare the values of FB1, FB2 and FB3. equal to its weight.
(b) The reading of the spring balance at position (c)
is higher than when the cylinder is at position (b). W

Explain why. FB FB
Solution W > FB
W = FB (weight of
(a) FB1 = FB2 because the same volume of liquid is water displaced)
displaced by the metal cylinder. Figure 3.48
FB3 < FB1 (or FB2) because a smaller volume of F4/3/77
liquid is displaced. 5 A floating object displaces its own weight of
∴ FB3 < FB1 = FB2 the fluid in which it floats.
(b) Since:
Apparent weight = Actual weight – Buoyant force
Weight of floating object
∴ Apparent weight is more when the buoyant
= Weight of fluid displaced Since W = mg
force is less. ∴ Wm
Therefore, the reading of the spring balance for Mass of floating object
(c) is higher. = Mass of fluid displaced

Forces and Pressure 192


6 Any changes in the density of the surrounding liquid will affect the level at
which an object floats. Figure 3.49 shows a cuboid floating at different
depths when placed in liquids of different densities.
cuboid

oil water paraffin


0.18 g cm–3 1.0 g cm–3 1.8 g cm–3

Figure 3.49 The same cuboid floats at different levels in liquids of different densities
F
12 O

3
R

CHAPTER
M
Figure 3.50 shows a wooden cube floating in water. Solution
When a stone is placed on the wood, it sinks a further In both situations, the weight of object 4
0.2 m. stone = weight of water displaced.
(a) Weight of wood
0.6 m 0.8 m
= Weight of water displaced
water = ρVg V = 0.6 m × 1 m2
= 0.6 m3
base area = 1 m  1 m = 1000 × 0.6 × 10
= 6000 N
Figure 3.50
F4/3/79 (b) Weight of stone
Find the weight of the = Weight of additional water displaced
(a) wood and = ρVg
∆V = 0.2 m × 1 m2
(b) stone. = 1000 × 0.2 × 10 = 0.2 m3
[Density of water = 1000 kg m–3] = 2000 N

Applications of Archimedes’ Principle

Submarine
1 Figure 3.51 shows a submarine which can float or submerge in water.

air air
purged air rising
buoyant
floating force
buoyant
buoyant compressed force
force air forces the
water out
weight

weight
water enters weight
ballast tank
diving

• A submarine floats in the sea • When water enters the ballast tank • When compressed air forces water
because the buoyant force of a submarine, the weight of the out of the ballast tank, the weight of
acting on the submarine is the submarine becomes greater than the submarine becomes less than
same as the weight of the the buoyant force. Thus, the the buoyant force. Thus, the
submarine. submarine dives into the sea. submarine rises.
F4/3/57a F4/3/57b F4/3/57c
Figure 3.51

193 Forces and Pressure


Hot-air Balloon
1 The operation of a hot-air balloon or gas balloon is based on Archimedes’
principle. buoyant
force
2 The air in the balloon is heated up by gas burner to over 100 °C.
3 The air in the balloon expands and a large volume of surrounding air is displaced.
4 Hot air has a lower density than the surrounding air. When the buoyant force is
greater than the weight of the balloon, the balloon starts rising up.
5 When the height increases, the density of surrounding air decreases. Thus, the
buoyant force will decrease. The balloon descends when the buoyant force is weight
less than the weight of the balloon.
6 It remains stationary in the air when the buoyant force is equal to the weight of Figure 3.52
the balloon. The forces are at equilibrum.
F 7 By varying the temperature of the air in the balloon, the balloon’s altitude can be controlled.
O
3

R
CHAPTER

M Ship
4 Plimsoll line ship is sufficiently large to have a weight equal to
the weight of the ship.
LTF 2 Although a ship is constructed of metal, which has
LF
LT a greater density than water, its shape is hollow so
LS that the overall density of the ship is less than the
LW L R
LWNA F weight sea water. As a result, the buoyant force acting on
O the ship is large enough to support its weight.
3

R 3 The density of sea water varies with location.


air buoyant
Key
CHAPTER

space M force To ensure that a ship is loaded to within safety


TF : Tropical fresh water
F : Fresh water limits, the Plimsoll line marked on the body of the
T : Tropical salt water 4
S : Salt water in summer
ship acts as a guide.
W : Salt water in winter 4 A ship will submerge deeper in fresh water
WNA : Winter in North Atlantic because the density of fresh water is less than sea
F4/3/59 water. This is why a ship must displace more water
Figure 3.53 to obtain sufficient buoyant force to support its
weight.
1 A ship, though very heavy, floats in the sea. This is
5 A ship can also float higher in cold seasons
because the volume of sea water displaced by the
because of the higher density of cold water.

Hydrometer
1 A hydrometer is an instrument used to measure equal the weight of the hydrometer and so it sinks
the relative density of liquids such as milk or lower. However, the hydrometer floats higher in a
acid in accumulators. liquid of higher density.
2 It consists of a tube with 0.97
a bulb at one end. Lead smallest
stem measurement
shots are placed in the scale largest
bulb to weigh it down measurement
and enable the
hydrometer to float
vertically in a liquid.
1.15 lead
float shots
liquid of a liquid of a
higher density lesser density
lead shot
Figure 3.55
F4/3/60
Figure 3.54 Hydrometer
4 On the basis of the above mentioned law of
3 In a liquid of lower density, a greater volume of flotation, the scale on the stem can be calibrated to
liquid must be displaced for the buoyant force to give readings for density in units of g cm–3.

Forces and Pressure 194


6 SPM
Clone
SPM
Clone
’07 ’09

The figure below shows an iceberg floating in the sea. Which of the following statements is correct?
A Weight of iceberg = Weight of sea water displaced
B Density of iceberg = Density of sea water displaced
Iceberg C Volume of iceberg = Volume of sea water displaced
Comments
The Law of flotation (Archimedes’ principle) states
that when an object floats in a liquid, it displaces its
own weight of liquid.
Sea
Answer A F
O

3
R

CHAPTER
M

Cartesian Diver 3.5 4

1 An object is suspended from a spring. Its weight in


air is 65 N. When the object is completely submerged
cork in water, the reading on the spring balance is 30 N.

water

gas jar

hollow
Cartesian diver

F4/3/62a
Figure 3.56F4/3/62b
Cartesian diver F4/3/62c

1 A Cartesian diver is a body whose average


(a) What is the buoyant force on the object in
density can be varied. It is a small hollow
water?
figure made of thin glass with an open-ended (b) Determine the volume of water displaced by the
tail as shown in Figure 3.56. It will sink, object.
remain stationary or rise according to whether [Density of water = 1000 kg m–3]
its average density is greater, equal or less than 2 The figure below shows a hydrometer which
the density of water. consists of a bulb and a stem of a smaller diameter.
2 The diver floats when it is full of air. However, the Explain why the hydrometer is marked as shown.
diver can be made to sink by pressing the cork. smallest measurement
3 Pressure on the cork increase the pressure
inside the jar. As a result, water is forced to flow
into the hollow Cartesian diver through the
largest measurement
open-ended tail compressing the air in it.
4 The diver now consists of a mixture of glass,
water and air. The overall weight of the diver is
lead shot
greater than the buoyant force and it sinks.
5 When the pressure is released, the compressed
air forces the water out through the open- 3 An empty gas balloon has a mass of 500 kg. The
balloon is filled with 2000 m3 of helium gas, at
ended tail. This reduces the weight of the diver
standard temperature and pressure (STP). If the
and it rises.
densities of air and helium, at STP, are 1.3 kg m–3
and 0.18 kg m–3 respectively, calculate the resultant
force acting on the balloon. [Take g = 10 N kg–1]

195 Forces and Pressure


3.6 Understanding Bernoulli’s Principle
Bernoulli’s Principle 1 For a fluid at rest, the pressure is the same at all
How does a huge and massive airplane fly in the points on the same level. However, this is not
air? We can explain this using Bernoulli’s principle. true when the fluid is moving.
Similarly, a fast moving racing car achieves part of 2 The pressure in a moving fluid depends on its
its stability and balance by taking advantage of the flow velocity.
difference in the pressures of moving air which is 3 Bernoulli’s principle states that:
based on Bernoulli’s principle. In a steady flow of a fluid, the pressure of the
fluid decreases when the velocity of the fluid
increases — and the converse is also true.
4 Air, being fluid, has the same characteristics as
F
water and other liquids, and obeys Bernoulli’s
O
principle.
3

R
5 Table 3.4 shows a few observations based on
CHAPTER

M
Bernoulli’s principle.
4 (a) Aeroplane (b) Racing car 6 Bernoulli’s principle is very important as it is
Figure 3.57 used in the design of airplanes, boat hulls, fan
blades and cars.

F
O
3

R Table 3.4
CHAPTER

M
Observation Explanation
4
1 • A thin sheet of paper is held in a horizontal plane in front of the lips.
Air is blown across the top of the paper. The free end of the paper will
rise.
• The flow of air at high speed creates a region of low pressure across the
top of the paper. The still air beneath the paper is at a higher pressure and
original a net upward force lifts the paper.
paper position

2 • A filter funnel is inverted and a ping-pong ball is held under it.


When air is blown through the funnel, the ping-pong ball does not drop
when released, but is held up beneath the funnel.
blowing
• The air flows around the ping-pong ball at high speed and creates a
region of low pressure. The air beneath the ball is at atmospheric
filter funnel pressure and a net upward force holds up the ping-pong ball.
fast flow ping-pong ball
of air
stationary air
atmospheric
pressure

3 F4/3/91 • Air is blown vigorously between two sheets of paper held vertically.
The two sheets of paper will be pressed together by the outside air.
• The air, blown at high speed, creates a region of low pressure in the
space between the two sheets of paper. The outside air, which is at a
higher pressure (atmospheric), presses the two sheets together.

Forces and Pressure 196


Bernoulli’s Principle Applied to Fluid Flow in Tubes

• Water flows into a Bernoulli’s principle:


tube from A to C. In a steady flow of fluid
• The water level (liquids and gases), the
water
decreases from A to pressure in the fluid is low
A B C C because water when the velocity of the fluid
highest lowest flows from a high is high and vice versa.
water water
pressure pressure pressure region to a
Figure 3.58
low pressure region.
F4/3/64
• Water flows faster
F
in a narrow tube O

3
than in a wide tube R

CHAPTER
as the same amount M
of water flows It is observed that the water
level in the region of 4
A C through them.
B • In contrast, the constriction in Figure 3.60
flow the slowest flow fast flow is slow
water pressure at B differs from that in
Figure 3.59. The reason is
highest water lowest water higher water is the lowest.
pressure pressure pressure that in Figure 3.59, the
flowing liquid experiences a
Figure 3.59 lower pressure whereas in
F4/3/65 Figure 3.60, the flowing air
flow is the slowest flow is fast flow is slow • When air is blown experiences the lower
through the tube as pressure.
highest air lowest air high
pressure pressure air pressure
shown, the narrow
section, B, has a
B lower pressure.
A C • The external
atmospheric
atmospheric pressure pushes the
pressure water in B to its
highest level.
water

Figure 3.60
F4/3/66

Applications of Bernoulli’s Principle

Bunsen Burner

1 Figure 3.61 shows the basic components of a Bunsen


burning of a burner.
mixture of gas 2 When the burner is connected to a gas supply, the gas
and air
flows at high velocity through a narrow passage in the
lower speed, burner, creating a region of low pressure.
higher pressure, 3 The outside air, which is at atmospheric pressure, is
higher speed, air from the atmosphere
lower pressure flows in drawn in and mixes with the gas.
4 The mixture of gas and air enables the gas to burn
gas supply
completely to produce a clean, hot, and smokeless
Figure 3.61 Bunsen burner flame.
F4/3/67

197 Forces and Pressure


Aerofoil Insecticide Sprayer

1 The flight of an airplane is based on the principle 1 Figure 3.64 shows how an insecticide sprayer
regarding the effect of the flow of air around its functions.
wings, which are in the form of an aerofoil. plunger
nozzle
2 Figure 3.62 shows an aerofoil with a leading plunger movement
compressed
air spray
edge that is rounded and a pointed trailing edge.
The top surface is arched and the bottom surface
is relatively flat. air hole metal
cylinder tube

atmospheric
pressure
insecticide
F
O Figure 3.64 Insecticide sprayer
3

R
2 When the plunger isF4/3/72
pushed in, the air in the
CHAPTER

M
cylinder flows out at a high velocity through a
4 small nozzle.
lift 3 The flow of air at high velocity creates a region
air flows faster,
low pressure of low pressure just above the metal tube. The
higher atmospheric pressure (from the air hole)
aerofoil acts on the surface of the liquid insecticide,
F
causing it to rise up through the metal tube.
O
4 The insecticide leaves the top of the metal tube
3

R
air flows slower, high pressure
as a fine spray through the nozzle.
CHAPTER

Figure 3.62 Aerofoil 4


F4/3/68
3 When a plane wing in the form of an aerofoil
moves through air, the flow of air over the top Carburettor
has to travel faster to cover a longer distance and
1 Figure 3.65 shows a carburettor. It is a device
creates a region of low pressure. The flow of air
which controls the speed of a car engine.
below the wing is slower resulting in a region of
It regulates the quantity of petrol and air that
higher pressure.
enters the engine.
4 The difference between the pressures at the top
low pressure
and underside of the wing causes a net upward choke area accelerator
force, called lift, which helps the plane to take- petrol
to
from fuel
off. tank air engine
cylinder
5 In addition being used on airplanes, inverted
aerofoils are also used as spoilers on racing cars. float
mixture of
petrol and air
In this case, the downward force helps to petrol
stabilise the car at high speeds.
Figure 3.65 Carburettor
F4/3/71
2 The air flows past a choke valve, which controls
the quantity of air, into a narrow section of the
carburettor where the air velocity increases.
3 According to Bernoulli’s principle, the high
velocity of the air in the narrow section creates a
region of low pressure. As a result, the petrol
which is at a higher atmospheric pressure is
forced out of a jet as a fine spray to form a
combustible mixture with the air.
4 The mixture of petrol and air is then drawn into
Figure 3.63 the engine cylinders to be burnt.

Forces and Pressure 198


Hydrofoil Other examples

1 A hydrofoil uses the same principle as an 1 canvas roof air moving P2


aerofoil. P1 > P2
2 A hydrofoil is a boat with wing-like foils
P1
mounted on struts below the hull.
(a) At rest (b) Moving fast
boat
The roof is flat The roof bulges upward
since the air because the pressure in
water flow faster, F4/3/101
low pressure inside and the car is greater
outside the (window is closed and
strut
car is at rest. the air in car is at rest). F
water flow slower,
hydrofoils high pressure O
Figure 3.67

3
R
2 Curve Ball

CHAPTER
Figure 3.66 M
F4/3/70 faster air,
lower deflection force 4
pressure
3 When the boat moves, the water above the foil
moves faster than the water below it. As a result,
a lift is produced.
4 At a certain speed, when the lift produced by the
hydrofoils equals the total weight of the hull and
the cargo, the hull comes out of the surface. slower air,
higher
5 This results in a great reduction in drag and a pressure
corresponding increase in speed. (a) Without spin (b) With spin
The ball does notF4/3/102(a)The spin causes the
curve as the speed air to move slower
of air is equal on on one side but
both sides. faster on the other.
7 SPM
Clone
’08
spinning
Explain why the roof of a house can be lifted during ball

strong winds.
Comments (c) The unbalanced pressure results in a net force
which curves the ball.
When a strong wind is blowing, the air above the
Figure 3.68
roof of a house is moving at a fast speed, thus
creating a region of low pressure, according to F4/3/102(b)
Bernoulli’s principle. The air below the roof is
13
stationary and therefore has high pressure. The
higher pressure of air below the roof of the house Why does a skier
pushes the roof upwards. As a result, the roof of the lean his body
house can be lifted when a strong wind is blowing. forward while
jumping?

low pressure roof is lifted Figure 3.69


Solution
The skier leans his body forward so that the air
high moves faster above than the air below him. This
pressure
causes a dynamic lift force and makes him stay
longer in the air.

199 Forces and Pressure


3.6

1 The figure shows the cross section of the wing of an airplane with its flap extended after take-off.

flap

wing

(a) Mark the region of low pressure with the letter L, and the region of high pressure with the letter H.
(b) Why is region L a region of low pressure?
F (c) State the underlying principle of this phenomenon.
O
(d) How does the difference between the pressures in regions L and H help the airplane to take-off?
3

R
(e) What is the function of the flaps during take-off?
CHAPTER

F
O
3

1. Pressure is the force acting normally R on one unit 11. At sea level, atmospheric pressure will support a
area of a surface or the ratio of force
M to area. 76 cm high column of mercury.
CHAPTER

12. The standard value of atmospheric pressure at sea


4 level in SI units is about 105 Pa.
Force, F
Pressure, P = 13. Examples of the applications of atmospheric pressure
Area, A
include drinking straws, rubber suckers, syringes, lift
pumps, vacuum cleaners and siphon.
2. Pressure is measured in pascals in SI units. 14. Atmospheric pressure decreases with altitude.
3. A liquid exerts pressure because it has weight. 15. The instruments for measuring atmospheric pressure
The pressure at any point in a liquid acts in all are the mercury barometer, the Fortin barometer
directions. and the aneroid barometer.
4. Examples of the application of pressure in everyday 16. The instruments for measuring gas pressure is the
life include knives, scissors, ice skates, nails, the manometer and the Bourdon gauge.
foundations of buildings, snow shoes and tyres on 17. Pascal’s principle states that in a confined fluid,
tractors. externally applied pressure is transmitted uniformly in
5. Pressure in a liquid, P = hρg all directions. This principle is used in the working of a
h = Depth of liquid hydraulic jack, the pneumatic tyre and other similar
ρ = Density of liquid devices.
g = Gravitational field strength 18. Archimedes’ principle states that when a body is
6. The pressure in a liquid increases when partially or totally immersed in a fluid, the upthrust
(a) its depth increases, (buoyant force) on the body is equal to the weight of
(b) the density of the liquid increases. fluid displaced.
7. Examples of the applications of pressure in liquids 19. The law of flotation states that the weight of the
include the public water supply systems and the liquid displaced by a floating body is equal to the
construction of dams. weight of that body.
8. Gas pressure is the result of the random impact of a 20. Examples of the applications of Archimedes’ principle
large number of gas molecules against the walls of its include submarines, ships and hydrometers.
container. 21. Bernoulli’s principle states that in a steady flow of
9. Gas pressure increases when a fluid, the pressure of the fluid decreases when
(a) the density of the gas increases, the velocity of the fluid increases and the converse is
(b) its temperature increases. true.
10. Atmospheric pressure is the pressure exerted by the 22. Examples of the applications of Bernoulli’s principle
weight of the air above any point on the earth’s include Bunsen burners, aerofoils, carburettors and
surface. insecticide sprayers.

Forces and Pressure 200


3
Multiple-choice Questions

3.1 Pressure
wrist cuff
heart
dam
1 Diagram 1 shows a cuboid with the level of heart
dimensions 4 cm × 5 cm × 10 cm water
and a density of 2.8 g cm–3. F
O

3
Diagram 4 R

CHAPTER
A The design of this structure is M
Diagram 2
cuboid stronger.
A Blood pressure depends on B Water pressure increases with 4

base
the level of the blood. depth.
B Blood pressure is normal when C The cost of construction is lower.
the cuff and heart are at the D Water pressure acts uniformly
Diagram 1 same level. in all directions.
C Blood is able to flow easily to
What is the pressure acting on the
the wrist.
base of the cuboid?
A 0.28 Pa C 280 Pa 5 The pressure of sea water at the 3.3 Gas Pressure and
B 2.8 Pa D 2800 Pa bottom of the sea is 1.2 × 108 N m–2. Atmospheric Pressure
Determine the depth of the sea.
2 If you walk on soft snow wearing 8 A fish is at a depth of 50 m in the
[Density of sea water = 1020 kg m–3]
skis, you are less likely to sink in. sea. Calculate the total pressure
A 1.18 × 104 m
This is because acting on the fish.
B 1.18 × 105 m
A your weight depends on the [Density of sea water
C 1.18 × 106 m
area of the shoe which is in = 1150 kg m–3, gravitational field
D 1.18 × 109 m
contact with the ground. strength = 10 N kg–1, atmospheric
B the pressure under your shoe 6 Diagram 3 shows a U-tube filled pressure = 105 N m–2]
depends on the area of the with two liquids, X and Y which are A 3.67 × 105 N m–2
shoe which is in contact with immiscible. The heights of the B 4.82 × 105 N m–2
the ground. liquid columns of X and Y from C 5.75 × 105 N m–2
C your mass depends on the level P are 8 cm and 4 cm D 6.75 × 105 N m–2
area of the shoe which is in respectively.
contact with the ground. 9 Diagram 5 shows a rubber
sucker with a hook pressed
3 What is the pressure on a boy’s against a smooth wall and then
feet if his mass is 40 kg and the level P released.
total area of the soles of his shoes
wall wall
is 180 cm2? Diagram 3
rubber
sucker
A 18.2 kPa
Which of the following statements
B 19.1 kPa
is true?
C 19.8 kPa
A The density of X is less than the
D 22.2 kPa
density of Y.
B The liquid pressure is higher in hook
column X than in column Y
3.2 Pressure in Liquids at level P. Diagram 5
4 When blood pressure is measured, C The densities of liquids X and Y The rubber sucker sticks to the
the cuff of the sphygmomanometer are equal. wall because
is wrapped round the upper arm 7 Diagram 4 shows the design of a dam. A the atmospheric pressure is
and must be held at the level of the Why does the thickness of the more than the pressure inside
heart. Why? dam increase with its depth? the sucker.

201 Forces and Pressure


B the atmospheric pressure is B low atmospheric density. 17 Hydraulic machines are able to
less than the pressure inside C low atmospheric pressure. use liquid pressure to transfer
the sucker. D low atmospheric volume. energy from one place to another
C the atmospheric pressure is because
14 Diagram 8 shows a manometer
equal to the pressure inside A liquid pressure acts equally in
containing mercury of density
the sucker. one direction.
1.36 × 104 kg m–3, connected to a
B liquids are incompressible.
gas supply.
Questions 10 and 11 refer to Diagram 6 C liquids do not have a fixed form.
which shows a mercury barometer.

vacuum
glass tube
3.5 Archimedes’ Principle
5 cm
mercury 18 Diagram 11 shows a metal block
gas supply
F being lowered slowly into a beaker
O of water which is placed on the
3

R mercury mercury
bath
pan of a weighing scale.
CHAPTER

M Diagram 8
Diagram 6 thread
4 Calculate the pressure of the gas.
10 The height, h will be decreased if [Gravitational field strength, metal
A the glass tube is lowered g = 10 N kg–1, atmospheric
further into the container. pressure = 76 cm Hg] beaker
B the atmospheric pressure A 1.1 × 105 Pa C 2.8 × 105 Pa weighing
decreases. B 2.3 × 105 Pa D 3.0 × 105 Pa scale
C the quantity of mercury in the
container is decreased. 15 Diagram 9 shows air trapped by a
drop of mercury inside a capillary
11 If a small amount of air is Diagram 11
tube. The capillary tube is placed
introduced into the vacuum in three different positions as Which of the following graphs
space, which of the following shown in (a), (b) and (c). shows the variations of the
changes will occur? reading on the weighing scale
A Height, h increases. mercury with the depth, x?
B Height, h decreases. P3 A scale reading C scale reading
C Height, h remains unchanged. air
P1
12 Diagram 7 shows a manometer P2
attached to a container containing
(a) (b) (c)
gas.
Diagram 9 O O

gas manometer
Which comparison of the air B D
scale reading scale reading
pressure trapped, P1, P2 and P3 is
true?
A P1 > P2 > P3 C P1 < P2 < P3
container
B P1 = P2 = P3 D P1 = P2 > P3
Diagram 7
O O
Which of the following statements
is true about pressure of the gas 3.4 Pascal’s Principle 19 A block with a density of
and the atmospheric pressure?
16 Diagram 10 shows a hydraulic press. 1000 kg m–3 is carefully lowered
A Gas pressure is equal to
into a beaker of pure water.
atmospheric pressure. load
10 N The metal block will
B Gas pressure is greater than
area A sink to the bottom of the
atmospheric pressure. = 2 cm2
area
= 100 cm2 beaker.
C Gas pressure is less than
B float with a larger portion of its
atmospheric pressure. liquid
volume above the water
13 Mountain climbers need to be Diagram 10 surface.
extremely wary of altitude sickness, The mass of a load, which can be C float below the water surface.
which can cause death. One of the lifted by a force of 10 N, is D float with a larger portion of its
causes for altitude sickness is A 5 kg C 25 kg volume below the water
A low atmospheric temperature. B 10 kg D 50 kg surface.

Forces and Pressure 202


20 An anchor made of steel has a Which of the following graphs same length as shown in
weight of 6000 N. The ship is correctly shows the variation in the Diagram 13.
floating in fresh water of a density velocity of the gas flowing along
of 103 kg m–3. What is the apparent the pipe with its distance, x?
string
weight of the anchor when it is A C
below the surface of the water? velocity velocity
ping-pong ball
[Density of steel = 8.0 × 103 kg m–3]
A 750 N C 4750 N Diagram 13
B 4500 N D 5250 N
What would you observe when air
O O
is blown between the two ping-
3.6 Bernoulli’s Principle pong balls?
B D A C
21 Diagram 12 shows a gas in steady velocity velocity
F
flow through a tube with a
O
constriction.

3
R
tube

CHAPTER
M

B D
O O 4

O distance, x 22 Two ping-pong balls are


Diagram 12 suspended on strings of the

Structured Questions
1 Diagram 1(a) shows a piece of paper hanging from (i) Compare the air speeds at P and Q.
one end of a support. When air is blown across the (ii) Relate the speed of air to the pressure of air.
upper surface of the paper, the position of the papers (iii) Mark the water levels in both the vertical
changes as shown in Diagram 1(b). tubes in Diagram 2 while air flows through
air flow the horizontal tube.

2 (a) Explain why a piece of stone sinks in water while a


boat filled with stones floats on water.
(b) A metal container has a base area of 0.02 m2.
paper Diagram 3(a) shows a cross section of the empty
paper
container where the portion submerged is 4 cm.
Diagram 3(b) shows the same container with an
support
support object placed in it. The portion submerged is now
6 cm. The density of the liquid is 1020 kg m–3.
(a) (b)
Diagram 1 metal metal
container container
(a) (i) Compare the positions of the paper in object
Diagrams 1(a) and (b).
(ii) Explain your answer in (a)(i).
(b) Name the principle that causes the observation in liquid liquid

Diagram 1(b). (a) (b)


(c) Diagram 2 shows an experiment carried out by a Diagram 3
student.
(i) Calculate the increase in the volume of the
liquid displaced, after the object is placed in
air flow P Q the container.
(ii) Determine the mass of the object.
tube
(iii) If the container with the object is placed in
distilled water, what changes would occur
with respect to the portion that is
water
submerged? Explain your answer.
(iv) State the principle involved in the situation
Diagram 2 in (b)(iii).

203 Forces and Pressure


Essay Questions
3 (a) Diagram 4 shows a hydraulic pump is used in a (c)
car workshop to raise a car.
10 N Load
Big
air compressor piston
small (1.0 m2)
piston
ground (0.1 m2) fluid
F
oil
In a simple hydraulic system, the cross-section
area of the small piston and the big piston are
small cylinder large cylinder
0.1 m2 and 1.0 m2 respectively. A force of 10 N
Diagram 4 is applied to the small piston. Calculate
F
O (i) Name and state the principle used in the (i) the force multiplier factor,
3

R function of a hydraulic pump. [2 marks] (ii) the maximum load which can be raised by
CHAPTER

M (ii) Explain how the hydraulic pump can be the big piston. [4 marks]
used to raise the car when a force F is
4 4 (a) Diagrams 6 shows a compression balance with its
applied on the small piston [4 marks]
scale pan inverted.
(b)

Rear wheel
brake
shoes brake
drum
return
spring
brake piston
pedal master
cylinder
brake
pads wheel
cylinder
brake
disc
piston
brake fluid Diagram 6
Front wheel
Diagram 5 The impact of a large number of marbles falling
Diagram 5 shows the hydraulic brake system randomly on the pan produces a fairly constant
used in a car. You are required to investigate the reading on the balance. This demonstration is the
characteristics of hydraulic brake system as simulation of the production of gas pressure
shown in table 1. based on the kinetic theory of gas.
Based on the kinetic theory of gas, describe how
Table 1 this simulation can be used to explain the gas
pressure on the wall of a container. [5 marks]
Cross-section
Boiling (b) You are asked to investigate the features of several
Hydraulic Type of area of piston
point of manometers shown in Table 1.
brake brake fluid Master Wheel
brake fluid Table 1
cylinder cylinder
P Compressible High Small Big Manometer Features of the U-tube
P
Q Incompressible High Small Big
Diameter: 1 cm
to gas supply
R Compressible Low Big Small Limbs’ separation: 5 cm
40 cm
S Incompressible Low Big Small
Liquid used: Water
Explain the suitability of each characteristic of the
Q
hydraulic brake system. Determine the most
Diameter: 1 cm
effective hydraulic brake to be used in a car brake to gas supply
Limbs’ separation: 5 cm
system. Give reasons for your choice. [10 marks] 40 cm

Liquid used: Mercury

Forces and Pressure 204


F
F
Manometer Features of the U-tube (c) Diagram 7 shows a manometer which is
connected to a gas supply. The atmospheric
R pressure is 76 cm Hg.
Diameter: 1 cm
to gas supply
Limbs’ separation: 15 cm
40 cm

Liquid used: Mercury


S
Diameter: 3 cm
to gas supply
Limbs’ separation: 5 cm
40 cm to gas
supply
Liquid used: Mercury F
F4/3/127
T Diagram 7 O

3
to gas 1 cm
Diameter: R
to gas supply supply (i) What is the pressure at the point A?

CHAPTER
Limbs’ separation:40
5 cm M
[1 mark]
20 cm F4/3/128
(ii) Find the pressure, in pascals, of the gas 4
to gas supply. [3 marks]
Liquid used: Mercury supply
(iii) The tap is open and the mercury is allowed
You are asked to measure40the cmexcess pressure of to flow out until the level B falls to the 60 cm
a gas supply. The excess pressure is about one mark of the ruler. State the new level of A.
atmospheric pressure, i.e., 76 cm Hg. [Density of mercury = 1.36 ×
F4/3/129
Explain the suitability of each feature20incmTable 1 104 kg m–3 and gravitation field strength =
and then determine the most suitable 10 N kg–1] [1 mark]
manometer to measure the excess pressure of a
gas supply. [10 marks]

Experiment
1 Diagram 1 shows depth, h (cm) Graph of h against m
the arrangement cylinder
container
of apparatus by a
student to water
investigate the
relationship
between the
mass of lead
shots, m and the lead shot

immersion depth,
h of the cylinder. Diagram 1
The student
placed different masses of lead shots in the cylinder
and measured the corresponding immersion depths.
A graph of immersion depth, h against mass of lead
shots, m was then drawn as shown in Diagram 2.
(a) Based on the graph in Diagram 2,
(i) state the relationship between m and h. O
[2 marks] mass, m (g)
(ii) what is the value of h, if the cylindrical Diagram 2
container is empty? [2 marks]
(iii) calculate the gradient of the graph. area of the cylindrical container, in cm2, calculate
[4 marks] the value of A. [2 marks]
(b) Using the value of the gradient of the graph (c) State one principle in physics connected with this
obtained in (a)(iii) and the relationship, experiment. [1 mark]
1 (d) State one precaution to be taken when
A= , where A is the cross-sectional
gradient conducting this experiment. [1 mark]

COMPANION WEBSITE 205 Forces and Pressure


Online Tests
FORM 4

4
CHAPTER

Heat

SPM Topical Analysis


Year 2007 2008 2009 2010 2011
Paper 1 2 3 1 2 3 1 2 3 1 2 3 1 2 3
Section A B C A B A B C A B A B C A B A B C A B A B C A B
Number of questions 5 1 – – – – 5 1 – – – – 5 1 – 1 – – 3 1 – – – – 4 1 – – 1 –

ONCEPT MAP

Gas
HEAT laws

Relationship between
Thermal Specific heat Specific gas pressure P,
equilibrium capacity latent heat temperature T and
volume V

Heat Temperature Heating Change


and cooling of state Boyle’s law
PV = constant

Q Q
c= = Charles’ law
mθ m
Thermometers V
= constant
T
Specific Specific
heat latent
capacity heat of Pressure law
of a solid fusion P
= constant
T

Specific Specific
heat latent
capacity heat of
of a liquid vaporisation Absolute zero

kelvin scale

COMPANION WEBSITE
206 Learning Objectives
4.1 Understanding Thermal
Equilibrium
hot pebble
(temperature, T2)
Rajan drops a hot pebble into a pail of water. After
some time, he finds that the hot pebble cools while
the water warms up until both reach the same same
temperature, T
temperature. What process had actually taken place water
(temperature, T1)
between the pebble and the water? T1 < T < T2

Figure 4.1

temperatures of both bodies, A and B are


Thermal Equilibrium
equal, thermal equilibrium has been reached.
F
3 The two bodies, A and B are said to be in
1 When two bodies A and B, which are at O
thermal equilibrium if the rates of heat

4
different temperatures, are placed in contact R
transfer from A to B, and vice versa, are

CHAPTER
M
with each other as shown in Figure 4.2, heat is
equal—i.e. the net rate of heat transfer
transferred from the hotter body, A to the 4
between the two bodies is zero.
colder body, B.
4 A and B are said to be in dynamic equilibrium
hot cold
A B A B as energy exchange still goes on between the
two objects.
5 Two bodies which are in thermal equilibrium
are at the same temperature irrespective of
shape, mass, size or type of surface.
temperature A > temperature B temperature A = temperature B

• Rate of transfer of • Rate of transfer of


energy from A to B is energy from A to B
higher than rate of equals rate of transfer What is thermal contact?
transfer of energy of energy from B to A. • Two objects are in thermal contact if heat can flow
between them.
from B to A. • There is no net heat
• Thermal contact can occur even with no physical
• TA decreases whereas transfer between A
contact. For example, when you warm your hands
TB increases. and B, TA = TB. near a fire.
(a) Net transfer of heat (b) At thermal
energy is from A to B equilibrium
Figure 4.2 Mechanism of thermal equilibrium

2 The temperature of the body A decreases as a Thermal equilibrium is a state in which


result of the loss of heat from its particles while ⇒ there is no net flow of heat between two objects.
the temperature of the body B increases ⇒ temperature of the two objects are equal.
when its particles receive heat. When the

Activ To show thermal equilibrium


ity 4.1
Apparatus/Materials Arrangement of apparatus
Plastic basin, beaker, thermometers, hot water and
cold water. thermometer
Activity 4.1

Procedure cold
water plastic basin
1 The apparatus is set up as shown in Figure 4.3.
hot
2 A beaker of cold water is placed inside the basin water
of hot water.
Figure 4.3

207 Heat
3 The temperature changes of both the hot and The temperature of the cold water increases while
cold water are observed. the temperature of the hot water decreases.
Observation The changes in temperatures continue until both the
water in the beaker and the basin have reached the
thermometer same temperature.
Discussion
cold The temperature of the water in the beaker is the
water plastic basin same as the temperature of the water in the basin
hot when thermal equilibrium is reached. Since the
water
temperature does not rise or fall, there is no net heat
Figure 4.4 flow between the portions of water.
F
O
Examples of Thermal Equilibrium Measuring the temperature of a patient
4

SPM
R ’07/P1
1 A doctor places a
CHAPTER

M
Cooking using an oven clinical thermometer
4 1 An oven has a thermostat to keep its under the tongue of
temperature constant. his patient having a
2 When food is placed in an oven, the food fever.
absorbs heat from the oven thereby causing its 2 After sometimes,
temperature to rise. thermal equilibrium
3 This process continues until thermal is reached, the clinical
equilibrium is reached, i.e. the temperatures of Figure 4.6
thermometer shows
the food and oven are equal. the temperature of the patient.

Cooling with a refrigerator Change in the Physical Properties of


1 The same principle is used in a refrigerator Materials with Temperature
except that its temperature is kept at a constant
low temperature by using a thermostat. 1 The physical property of a substance which is
2 When food at room temperature or higher is sensitive to and varies linearly with changes in
placed in a refrigerator, it releases its heat to the temperature of the material is known as
the cooler environment in the refrigerator thermometric property.
until thermal equilibrium is reached. 2 An important application of the thermometric
3 The temperature of the food decreases until it property of a substance is in the construction
becomes the same as the temperature inside the of a thermometer to measure temperature.
refrigerator. This helps to keep the food fresh.

Making a cup of warm drink Various types of thermometers are constructed based
on the four thermometric properties of substances that
The milk is are sensitive to changes in temperature. The table
too cold! The coffee is
just nice. below shows examples of physical properties that are
utilised by various types of thermometers.
Thermometer Thermometric property
The coffee is Gas thermometer Gas pressure
too hot!
Mercury Volume of mercury
thermometer
Figure 4.5 Resistance Electrical resistance of
thermometer platinum
Activity 4.1

1 There is thermal equilibrium between the cold


Thermocouple Electrical potential difference
milk and the hot coffee after the two drinks are
thermometer (e.m.f.) between the two
mixed. junctions of two different
2 Similarly, this is how you would prepare a cup metal wires
of warm water.

Heat 208
this increase in volume, the mercury rises
Liquid-in-glass Thermometer
higher in tube B which has a smaller
Mercury Thermometer diameter. Thus, a thermometer with a
finer capillary tube is more sensitive than a
1 This is a type of liquid-in-glass thermometer thermometer with a capillary tube of
whose function depends on the expansion of larger diameter.
mercury with temperature. (b) A smaller bulb
A smaller bulb contains less mercury. It
glass tube small bulb
will absorb heat from the surroundings
in a shorter time and thus respond faster
bulb with thin mercury narrow to a temperature change. Hence, the
glass wall thread capillary tube
thermometer is more sensitive.
Figure 4.7 Mercury thermometer F
-10 0 10 20 -10 0 10 20
O
2 Mercury is used in thermometers because

4
smaller bulb larger bulb R
(a) it is a good conductor of heat,

CHAPTER
M
(b) it has a high boiling point, Figure 4.9
(c) it expands uniformly when heated, (c) A glass bulb with a thinner wall 4
(d) it is opaque (does not allow light to pass The wall of the glass bulb is made thin
through) and it can be seen easily. to enable a quick heat transfer from the
3 Mercury freezes at a temperature of –39 °C surroundings to the mercury. Hence, the
and is not suitable for measuring temperatures thermometer is more sensitive.
below this temperature. The upper temperature -10 0 10 20 -10 0 10 20
limit of a mercury thermometer is 500 °C.
4 A mercury thermometer consists of a thin- thinner wall thicker wall
walled glass bulb which is connected to a glass Figure 4.10
capillary tube of uniform diameter. The glass
bulb is filled with mercury. Calibration of a Thermometer
5 The mercury in the bulb expands when
temperature rises, moving up in the tube, thus 1 A thermometer must be calibrated before it
increasing the length of the mercury thread. can be used to measure temperature.
6 The increase in the length of the mercury thread 2 To produce a temperature scale for a thermometer,
is determined by the increase in temperature. two standard (reference) temperatures must
7 The space above the mercury column in the first be selected. The two standard temperatures
capillary tube is evacuated (a vacuum) so must be such that they can be reproduced in a
that no air is present to interfere with the laboratory anywhere in the world.
thermometer reading. 3 In the Celsius temperature scale, the first
8 The sensitivity of a mercury thermometer can standard temperature is the melting point of
be increased by using the following: pure ice which is 0 °C. This is also called the
(a) Capillary tube with a finer bore ice point (lower fixed point).
Figure 4.8 shows two thermometers with 4 The second standard temperature is the boiling
the same volume of mercury at low point of pure water which is 100 °C. This is
temperature. also called the steam point (upper fixed point).
larger smaller
5 Note that both temperatures are taken from
diameter same
diameter
increment the same material, water, at a fixed pressure of
same
volume
in volume 1 atmosphere.
of mercury
of mercury

low high
temperature
temperature
Mercury expands less compared to the same volume of
Figure 4.8 alcohol for a given temperature rise. For this reason, a
mercury thermometer must be constructed with a
When the temperature rises, the increase capillary tube of smaller diameter than that of an alcohol
in volume of the mercury is the same in thermometer to achieve the same sensitivity.
both cases. However, to accommodate for

209 Heat
Calibration of a Mercury-in-glass Thermometer in Degree Celsius

A mercury-in-glass thermometer is calibrated using the following procedures.

To determine the ice point To determine the steam point

upper fixed
point, 100 °C
lower fixed
melting
point, 0 °C steam
ice
distillation
filter flask
funnel
F
boiling
O beaker water
4

R
CHAPTER

M
water

4
Figure 4.11 Figure 4.12
1 The bulb of an uncalibrated thermometer is placed 1 The thermometer is placed in steam above boiling
in melting ice cubes in a filter funnel. water in a boiling flask.
2 Mercury level decreases until thermal equilibrium 2 Mercury level increases until thermal equilibrium
is achieved between thermometer and ice. is achieved between thermometer and steam.
3 The level of the mercury is marked on the stem as 3 The level of the mercury is marked on the stem as
0 °C. The length of mercury column, 0 is 100 °C. The length of mercury column, 100, is
measured. measured.

The space between the 0 °C and 100 °C is divided 1 When determining the ice point 0 °C, pure ice
into 100 equal gradations so that each gradation must be used since the presence of impurities will
lower the melting point.
represents a temperature rise of 1 °C. The calibrated
2 When determining the steam point 100 °C, the
thermometer can now be used to measure water must be boiled at standard atmospheric
temperatures between 0 °C and 100 °C. pressure as a change in pressure will change the
boiling point of water. The presence of impurities will
also affect the boiling point of water but it has no
effect on the temperature of the steam produced.

If gradations are not marked on the thermometer,


1
the temperature of a substance, θ can be determined
using the following formula. A mercury thermometer is used to measure the
100 temperature of a liquid P. The length of the mercury
column at the ice point and steam point are 5 cm and
40 cm respectively. When the thermometer is immersed
0 °C in the liquid P, the length of the mercury column is
100 °C
0 23 cm. What is the temperature of the liquid P?
θ 40 cm

0 °C θ 100 °C
Figure 4.13

θ – 0 5 cm
Temperature, θ = × 100 °C
100 – 0 23 cm
Figure 4.14

Heat 210
Solution
Ice point = 0 °C, 0 = 5 cm
Steam point = 100 °C, 100 = 40 cm
θ = 23 cm Beware
∴ Temperature of the liquid P, θ Students often forget to subtract 5 cm, i.e. the length of
the mercury column at the ice point and wrongly
θ – 0 calculate the temperature of the liquid as:
= × 100 °C
100 – 0 23
θ= × 100 °C
(23 – 5) 35
= × 100 °C
(40 – 5) θ = 65.7 °C
= 51.4 °C

F
O

4
R
4.1 4.2 Understanding Specific

CHAPTER
M

1 Define ‘the thermometric property of a substance’. Heat Capacity 4


2 The ice point and steam If we look around us, we observe that some
cm
point are marked on an
substances heat up faster than others. For example,
uncalibrated mercury steam 30
thermometer as shown in point A metals can usually be heated up faster than non-
20 metals. Also, some substances can maintain their
the figure. B
(a) What is the ice 10 temperatures for a longer period of time than others
point
temperature of C 0 which lose their heat faster. Water is a substance
(i) the ice point? which can keep its heat and maintain its temperature
(ii) the steam point? for a long period of time. Do you know why?
(b) What is the temperature, in °C, if the end of the
mercury thread is at
(i) point A? (ii) point B? (iii) point C? Concept of Heat Capacity
(c) Explain why it is impossible to record the
temperature of point C with a mercury 1 Kassim places 2 pebbles of different masses
thermometer? and a glass of water under the hot afternoon
3 The figure below shows the graph of a length of Sun.
mercury in a column,  against the temperature
reading on a thermometer. Sun
length of mercury,  (cm)

35

27

2 temperature, θ (°C)
O θ 100 small large
pebble pebble water (same mass
as the large pebble)
What is the temperature, θ of a liquid measured with
this thermometer? Figure 4.15
4 (a) What is meant by thermal equilibrium? After 5 minutes, he finds that:
(b) Two bodies of different temperatures are placed
(a) the small pebble is hotter than the large
in contact with each other. After a while, it is
found that the temperatures of the two bodies
pebble, and
are equal. Explain this observation. (b) the large pebble is hotter than water of the
(c) Explain the principle of operation of a same mass.
thermometer based on the concept of 2 When an object is heated, its temperature
thermal equilibrium. State the thermometric increases as the heat energy absorbed increases
property that is utilised in this thermometer. the kinetic energy of the molecules and makes
the molecules move faster.

211 Heat
3 However, the increase in temperature depends 4 Activity 4.2 shows the situations where you can
on: observe and compare the increase in temperature
(i) the quantity of heat absorbed, when a fixed quantity of heat is supplied to
(ii) the mass of the object and (i) two different masses of water and
(iii) what the object is made of. (ii) two different liquids of the same mass.

Activ To observe and compare the change in temperature when a


ity 4.2 fixed quantity of heat energy is supplied to (a) two different
masses of water, (b) two different liquids of the same mass
Apparatus/Materials Observation
Weighing scale, 1000 m glass beakers, immersion When 500 g of water is used,
F heater, thermometer, stirrer, water, cooking oil and (a) the rate of temperature increase is lower and
O stopwatch. (b) the temperature rise is smaller.
4

R
CHAPTER

M (A) Two different masses of water


(B) Two different liquids of the same mass
Arrangement of apparatus
4 Procedure
thermometer 1 Steps 1 to 9 in Activity (A) are repeated by
to power supply
immersing two immersion heaters (with the
immersion heater same power) in 200 g of water and 200 g of
cooking oil separately.
water Observation
(a) The temperature rise is greater for the 200 g of
Figure 4.16 cooking oil.
(b) The rate of increase in temperature for cooking
Procedure oil is higher.
1 A beaker containing 200 g of water is prepared Discussion
using a weighing scale.
Since the heater is switched on for the same period
2 The apparatus is set up as shown in Figure 4.16.
of time, the amount of heat energy supplied is the same.
3 A 50 W immersion heater is placed in the beaker.
4 The initial temperature of the water, θ1 is Precautions
recorded. (a) The water or oil must be stirred continuously to
5 The immersion heater is switched on and the ensure that heat is uniformly distributed.
stopwatch is started. The water is continuously (b) When the immersion heater is switched off,
stirred. the water or oil temperature continues to rise.
6 The temperature of the water is recorded every The water or oil is stirred until the highest
minute. temperature is reached before recording its value.
7 After 5 minutes of heating, the heater is Conclusions
switched off. The highest temperature reached When the amount of heat supplied is fixed, the
by the water, θ2 is recorded. increase in temperature depends on
8 The temperature rise, θ = θ2 – θ1, is calculated. (a) its mass. The greater the mass, the smaller the
9 The above procedure is repeated with 500 g of increase in temperature
water. (b) what each object is made of.

3 The heat capacity of the 500 g water is greater


Heat Capacity, C
because it needs more heat to increase its
temperature by 1 °C.
1 Heat capacity, C is defined as the amount of
4 Similarly, the oil has a smaller heat capacity as
Activity 4.2

heat energy needed to increase the temperature


it needs less heat to increase its temperature by
of an object by 1 °C.
1 °C compared with the 200 g water.
2 For Activity 4.2(A), which object (the 200 g
5 The unit of heat capacity is J °C–1.
water or the 500 g water) has a greater heat
6 Figure 4.17 shows a boy drinking hot soup
capacity?
with a spoon.

Heat 212
1 °C is almost 5 times the heat energy required
to raise the temperature of the aluminium by
1 °C (Figure 4.18).
1 kg water

Figure 4.17
( )
If he accidentally spills a spoonful of soup amount of
heat supplied
onto his hand, he would experience only a
slight pain because the quantity of heat (a) 4200 J is required to raise the temperature of
transferred to his hand is small. F
1 kg of water by 1 °C O
7 However, if he spills the whole content of the

4
R
bowl of soup onto himself, he would suffer

CHAPTER
M
serious injuries because the quantity of heat in
the whole content of the bowl is quite large. 4
8 This observation shows that although the soup ( )
in the spoon and the bowl are at the same amount of
temperature, the soup in the bowl contains heat supplied
more heat. The soup in the bowl is said to have
(b) 900 J is required to raise the temperature of
a larger heat capacity.
1 kg of aluminium by 1 °C
9 The larger the mass of a body, the larger its
Figure 4.18
heat capacity.
6 This is because the value of the specific heat
Specific Heat Capacity, c capacity of water is higher than that of
aluminium.
1 The heat capacity of an object is the quantity 7 Table 4.1 shows the values of specific heat
of heat energy required to increase its capacities of various substances.
temperature by 1 °C. Each object has its own
heat capacity. Table 4.1
2 The specific heat capacity of a substance is
the quantity of heat energy required to Substance Specific heat capacity, c
increase the temperature of 1 kg of the (J kg–1 °C–1)
substance by 1 °C or 1 K. Solids
3 The symbol for specific heat capacity is c and Ice 2100
the unit is J kg–1 °C–1 or J kg–1 K–1. Aluminium 900
4 The formula that relates heat energy, Q to
specific heat capacity, c is: Concrete 800
Glass 700
heat gained or increase in
heat absorbed temperature Iron 500
Copper 400
Liquids
Q = mc θ
Water 4200
Alcohol 2500
heat lost or decrease in
heat released temperature Paraffin 2100
Mercury 140
where m is the mass of the substance, in
kilograms. (a) Liquids generally have higher specific
5 If 1 kg of water and a 1 kg block of aluminium heat capacities than metals. Mercury is an
are heated individually, the heat energy exception.
required to raise the temperature of water by

213 Heat
(b) When a solid and a liquid of equal mass
are heated separately to achieve the same
temperature rise, the liquid requires more Mercury is a metal which exists in the liquid state at
heat energy compared to the solid. room temperature.
(c) An object with a lower value of specific
heat capacity will reach a particular
temperature faster when heated as it
requires less heat to raise its temperature
by 1 °C. Heat Absorbed and Heat Released
(d) Similarly, an object with a lower specific • Consider an object of mass m and specific heat
capacity c.
heat capacity cools faster due to its lower
• When the temperature increases by θ °C (from t1 °C
amount of heat stored and a substance
to t2 °C), it gains an amount of heat energy given by
F with a higher specific heat capacity takes a
O
Q = mcθ.
longer time to cool. • When the temperature of the same object decreases
4

R
by θ °C (from t2 °C to t1 °C), it releases an amount of
CHAPTER

M
heat energy given by Q = mcθ.
4

• The differences between heat capacity (C) and specific heat capacity (c)

copper water

100 g 2 kg 2 kg
c = 400 J kg–1 °C–1 c = 400 J kg–1 °C–1 c = 4200 J kg–1 °C–1
C = 40 J °C–1 C = 800 J °C–1 C = 8400 J °C–1

Specific heat capacity Heat capacity

Symbol c C

General unit Relates to a material: J kg–1 °C–1 Relates to a particular object: J °C–1

Value Fixed for a material regardless of its mass Not fixed, depends on the mass of the object

Formula Q = mcθ Q = Cθ

Relationship
C = mc
between c and C
• Derivation of the formula, Q = mcθ a 1 °C temperature rise in a mass of 1 kg of the
1 Suppose Q is the amount of heat energy required to substance.
produce a θ °C temperature rise in a mass of m kg of 4 Statement 3 is equivalent to the definition of specific
a substance. heat capacity, c.
Q Q
2 –– is the amount of heat energy required to produce c = –––
m mθ
a θ °C temperature rise in a mass of 1 kg of the ∴ Q = mcθ
substance. Q
5 Also, from c = ––– :
Q mθ
3 ––– is the amount of heat energy required to produce J
mθ Unit c = –––––– = J kg–1 °C–1 (or J kg–1 K–1)
kg °C

Heat 214
Relationship between Heat Content Solution
and Mass Specific heat capacity of liquid X, c
= 3800 J kg–1 °C–1
1 Figure 4.19 shows a burning sparkler which Heat required to raise the temperature of 1 kg of
can be used to show the relationship between liquid X by 3 °C = mcθ
the mass of an object and its heat content. = 1 × 3800 × 3 = 11 400 J

sparks

3
The temperature of 10 kg of a substance rises by
burning 55 °C when heated. Calculate the temperature rise
stem F
when 22 kg of the substance is heated with the same
O
quantity of heat.

4
Figure 4.19 R

CHAPTER
M
2 A spark does not cause injury when it strikes Solution
the skin of a child because the heat stored in a The substance is heated by the same quantity of heat 4
spark is very little due to its small mass, even Q in both cases. Therefore the following relationship
though it is at a very high temperature. is obtained:
3 However, if you touch the burning stem, it can m1cθ1 = m2cθ2
cause quite a severe burn on your fingers m1θ1 = m2θ2
because of its much larger mass compared 10 × 55 = 22 × θ2
with the mass in a spark. Specific heat capacity, c
θ2 = 10 × 55 appears on both sides of
4 Similarly sparks that strike a welder during 22 the equation.
welding do not cause injury to him. = 25 °C
5 Figure 4.20 shows two similar bowls, P and Q,
one full while the other 1 filled with hot soup
3
In questions involving changes in heat and
at the same temperature.
temperature of a particular substance, students
should not worry if the value of the specific heat
capacity, c, is not given. This is because the value of c
will cancel out each other.
P Q

Solving Problems Involving Specific


Heat Capacity
P Q 1 According to the law of conservation of
Figure 4.20 energy, energy can neither be created nor
6 After 15 minutes, the soup in bowl Q falls to destroyed. Heat energy is obtained by
room temperature while the soup in bowl P is conversion from other forms of energy.
still warm. 2 This law is used in solving problems related to
7 All these indicate that the bigger the mass of an heat energy.
object, the more heat it can store. Examples:
(a) Electrical energy from an electric heater
2 Electrical energy Heat energy
A liquid X of mass 1 kg requires 3800 J of heat Q = Pt Q = mcθ
energy to raise its temperature by 1 °C. State the
value of the specific heat capacity of liquid X. ∴ Pt = mcθ
Calculate the total heat energy required to raise the where P = power of heater, in watts (W)
temperature of 1 kg of liquid X by 3 °C. t = time of heating, in seconds (s)

215 Heat
(b) The kinetic energy of a moving object is
transformed into heat energy when it is 6
brought to rest by frictional force
A bullet travelling at 200 m s–1 hits a sand bag.
Kinetic energy Heat energy The temperature of the bullet rises by 50 °C.
1 Assuming that all the kinetic energy of the bullet is
Ek = — mv 2 Q = mcθ converted into heat energy which heats up the bullet,
2
calculate the specific heat capacity of the bullet.
1
∴ — mv 2 = mcθ Solution
2
Heat energy = Kinetic energy
(c) Potential energy of a falling object 1
mcθ = –– mv 2
F
2
Potential energy Heat energy 1
O –1– v2 –– × (200)2
4

R Ep = mgh Q = mcθ 2 2–––––––––


c = –––– =
CHAPTER

M θ 50
4 ∴ mgh = mcθ = 400 J kg–1 °C–1

4 Mixing Two Similar Substances


A 100 W electric heater is used to heat 4 kg of olive
oil for 10 minutes. Assuming that there is no loss
of heat, calculate the temperature rise of the olive 7
oil. The specific heat capacity of olive oil is
1890 J kg–1 °C–1. 200 g of hot water at 80 °C is mixed with 100 g of
cold water at 20 °C. What is the final temperature of
Solution the water?
Heat energy = Electrical energy
mcθ = Pt Solution
Pt
θ = ––– 200 g hot water
mc
× 10 × 60
= 100
––––––––––––– 80 °C
4 × 1890 temperature
= 7.9 °C fall heat
= (80 – θ) °C lost
θ °C
temperature
rise heat
5 = (θ – 20) °C gained
20 °C θ °C

The water temperature at the top of a 200 m high


100 g cold water
waterfall is 20 °C. What is the water temperature at
the bottom of the waterfall?
Heat lost by the hot water = m1c1(θ1 – θ)
(Take g = 10 m s–2 and c = 4200 J kg–1 °C–1)
= 0.2 × c1 × (80 – θ)
Solution Heat gained by the cold water = m2c2(θ – θ2)
Heat energy = Gravitational potential energy = 0.1 × c2 × (θ – 20)
mcθ = mgh Heat energy lost = Heat energy gained
gh 0.2 × c1 × (80 – θ) = 0.1 × c2 × (θ – 20)
Change in temperature, θ = ––– c is the same for similar substances and cancels out.
c
10 × 200 ∴ 0.2 × (80 – θ) = 0.1 × (θ – 20)
= ––––––––– 16 – 0.2θ = 0.1θ – 2
4200
= 0.48 °C 0.1θ + 0.2θ = 16 + 2
∴ The water temperature at the bottom of the waterfall 0.3θ = 18
∴ = 0.48 °C + 20 °C = 20.48 °C θ = 60 °C

Heat 216
Mixing of Two Dissimilar Substances

8
A copper sphere of mass 500 g is heated in the flame of a Bunsen burner for several minutes before being
immersed into 2 kg of water at 30 °C. The final temperature of the water is 50 °C.
Calculate the temperature of the sphere before it was dropped into the water.
The specific heat capacities of copper and water are 400 J kg–1 °C–1 and 4200 J kg–1 °C–1 respectively.
Solution
Heat lost by the copper sphere
= 0.5 400 (θ1 – 50)
F
temperature O
fall Heat gained by the water = 2 4200 (50 – 30)

4
copper R
500 g = (θ 1– 50) °C
Heat energy lost = Heat energy gained

CHAPTER
θ 1°C M
0.5 400 (θ1 – 50) = 2 4200 (50 – 30)
water 200θ1 – 10 000 = 168 000 4
temperature 50 °C
2 kg
30 °C
rise
= (50 – 30) °C
200θ1 = 178 000
θ1 = 890 °C

Activ To determine the specific heat capacity of a liquid (water)


ity 4.3
Apparatus/Materials 4 The apparatus is set up as shown in Figure 4.21.
Weighing scale, 50 W electric heater (immersion 5 The electrical power rating of the heater, P is
heater), thermometer, power supply, tissue paper or recorded before it is placed in the water.
felt cloth, polystyrene sheet, stopwatch, stirrer, 6 The initial temperature of water, θ1 is read on
beaker, and water. the thermometer.
7 The power supply is switched on and the
Arrangement of apparatus stopwatch is started simultaneously.
8 While the water is being heated, it is stirred
stirrer
continuously to ensure uniform heating of the
thermometer
water.
tissue paper/ 9 After heating for t seconds, the heater is
power felt cloth switched off. The highest reading on the
supply
thermometer, θ2 is recorded.
polystyrene sheet
10 The temperature rise, θ = θ2 – θ1 is calculated.
electric heater Results/Calculation
Figure 4.21 Electrical power of heater = P watts
Heating time = t seconds
Procedure
Electrical energy supplied by the heater = Pt joules
1 An empty beaker with a stirrer is weighed with a Mass of beaker with stirrer = m1 grams
triple-beam balance and its mass, m1 is recorded. Mass of beaker containing water and stirrer
2 The beaker is filled with water until it is three- = m2 grams
quarters full. The beaker and stirrer are weighed m
Mass of water = m2 – m1 = m gram = kg
1000
Activity 4.3

again with a triple-beam balance and the mass,


m2 is recorded. Initial temperature of water = θ1 °C
3 The beaker is wrapped with tissue paper or felt Final temperature of water = θ2 °C
cloth and placed on a polystyrene sheet to prevent Temperature rise = θ2 – θ1 = θ °C
heat loss to the surroundings and the table. Heat energy absorbed by the water = mcθ joules

217 Heat
Assuming there is no heat loss to the surroundings: obtain a value of c (water) = 5000 J kg–1 °C–1
Heat supplied = Heat absorbed instead of a standard value of 4200 J kg–1 °C–1.
Pt = mcθ 2 This is because the experimental value of the
Pt temperature rise, θ is less than expected due to
Specific heat capacity, c = ___
mθ some heat loss to the surroundings. Besides, the
Discussion beaker is also heated up in the process, further
1 The value of the specific heat capacity of water, reducing the expected rise in temperature.
c determined in the experiment is larger than the 3 This method is applicable for other liquids that
standard value of c. For example, one may are not poisonous or volatile.

Activ To determine the specific heat capacity of a solid (aluminium)


F ity 4.4
O
4

R 11 The temperature rise, θ = θ2 – θ1 is calculated.


Apparatus/Materials
CHAPTER

M
Aluminium cylinder, weighing scale, 50 W electric Results/Calculation
4 heater, thermometer, power supply, felt cloth, Electric power of heater = P watts
polystyrene sheet, stopwatch, and some lubricating Heating time = t seconds
oil. Mass of aluminium cylinder = m kg
Arrangement of apparatus Initial temperature of the aluminium cylinder = θ1
Final temperature of the aluminium cylinder = θ2
electric heater Temperature rise = θ2 – θ1 = θ
thermometer
Electrical energy supplied by the heater = Pt
felt cloth
power supply Heat energy absorbed by the aluminium cylinder = mc θ
lubricating oil On the assumption that there is no heat loss to the
aluminium cylinder surroundings:
polystyrene sheet Heat supplied = Heat absorbed
Pt = mcθ
Pt
Figure 4.22 Specific heat capacity, c = ___

Procedure
Discussion
1 An aluminium cylinder with two cavities is 1 The aluminium cylinder is wrapped in felt cloth
weighed with a triple-beam balance and its to reduce heat loss to the surroundings and the
mass, m is recorded. polystyrene sheet acts as a heat insulator to
2 The electrical power of the heater, P is recorded. avoid heat loss to the surface of the table.
3 The electrical heater is then placed inside the 2 The value of the specific heat capacity of
large cavity in the centre of the cylinder. aluminium, c determined in the experiment is
4 The thermometer is then placed in the small larger than the standard value of c. For example,
cavity of the aluminium cylinder. one may obtain a value of c = 1050 J kg–1 °C–1
5 The aluminium is wrapped in felt cloth which instead of the standard value of 900 J kg–1 °C–1.
acts as an insulator. This is because the experimental value of the
6 A few drops of lubricating oil are added to both temperature rise is less than the expected
cavities to ensure good thermal contact (for temperature rise due to some heat loss to the
better heat transfer). surroundings. In substituting the experimental
7 The apparatus is set up as shown in Figure 4.22. Pt
8 The initial temperature of the aluminium value of θ in the formula, c = , where Pt and

Activity 4.3 & 4.4

cylinder, θ1 is read off the thermometer.


9 The electric heater is switched on and the m are constant, the smaller value of θ results in a
stopwatch is started simultaneously to measure larger value of c.
the time taken for heating. 3 The temperature reading on the thermometer
10 After heating for t seconds, the heater is continues to rise after the electrical heater is
switched off. The highest reading, θ2 on the switched off because the temperature of the
thermometer is recorded. heater is still higher than the aluminium cylinder.

Heat 218
Auntie Susan poured 220 g of coffee at 75 °C and 12.0 g of milk at 10 °C into a ceramic
cup of mass 110 g which has a specific heat capacity of 1100 J kg–1 °C–1 at room
temperature (27 °C).
milk
What is the final temperature of the coffee when thermal equilibrium is reached?
coffee
Assume that there is no heat loss to the surroundings and that the specific heat
capacities of coffee and milk are the same as that of water which is 4200 J kg–1 °C–1.

Picture the problem


F
Specific heat capacity of coffee = specific heat capacity of milk = 4200 J kg–1 °C–1
Let the final temperature of the coffee, when thermal equilibrium is reached, be θ.
Heat lost by coffee:
F
Qcoffee = mcoffee ccoffee (75 – θ) 75 °C heat lost O

4
Heat gained by milk: temperature fall by coffee R
= (75 – θ) °C

CHAPTER
Qmilk = mmilk cmilk (θ – 10) θ °C M
Heat gained by cup: temperature
4
Qcup = mcup ccup (θ – 27) temperature rise rise = (θ – 27) °C
= (θ – 10) °C heat
27 °C gained
Assuming no heat loss to the surroundings:
10 °C
Heat lost by coffee = Heat gained by milk + Heat gained by cup
⇒ Qcoffee = Qmilk + Qcup

Solution
(0.220)(4200)(75 – θ) = (0.012)(4200)(θ – 10) + (0.110)(1100)(θ – 27)
1095.4θ = 73 071
θ = 66.7 °C

1 SPM
Clone
’07

In the figure below, a beaker of hot water is added to The significance of specific heat capacity
a beaker of cold water.
When c is low • Heats up easily (i.e. at a faster
(metals or alloy) rate).
• Cools off easily.

• Heats up and cools off slowly


hot When c is large (takes a longer time to change
water (water, plastic its temperature).
and wood) • Can store up more heat (which
cold water warm water can be released later) after being
heated up to a certain temperature.
Which of the following is true if there is no heat loss
to the surroundings?
A Heat lost by hot water < Heat gained by cold water
B Heat lost by hot water = Heat gained by cold water
C Heat lost by hot water > Heat gained by cold water
Which cools off faster after a sunny afternoon, the
Comments
field in the Bukit Jalil Stadium or the surrounding car
When hot water is added to cold water, hot water parks?
losses heat while cold water gains heat. If there is no
heat lost to the surroundings (including the container Solution
of the cold water), the heat lost by the hot water is • The soil has a smaller specific heat capacity than the
equal to the heat gained by the cold water. concrete.
• So, the field in the Bukit Jalil Stadium cools off faster
Answer B than the car park after a sunny afternoon.

219 Heat
Sea breeze
• The movement of cool air from the sea to the Do you know that countries that are close to the sea
land as a result of hot air that rising from the land do not experience large daily temperature changes
in the day time. because of the presence of sea and land breezes?
During day time,
On the other hand, countries or regions of barren land
heat energy from the
Cooler air Sun is absorbed by which are very far from the sea experience very large
Hot air above the
above the the sea and the land. temperature differences daily.
land rises.
sea moves
towards
the land.
4.2

1 A factory uses methylated spirit as fuel to heat F


its water tank which contains 50 kg of water. If O

4
5 g of methylated spirit burns to release 90 kJ R

CHAPTER
of heat, what is the minimum mass of M

warm land
methylated spirit required to heat the water
4
tank from 27°C to 60°C? [Specific heat capacity
sea of water, c = 4200 J kg –1 °C–1]
2 The figure below shows two spheres, A and B
each of mass 0.5 kg being heated to 100 °C in a
beaker of water. Sphere A is made of copper and
• Sea has a higher specific
• Land has a lower specific heat capacity. sphere B is made of aluminium. Two beakers of
heat capacity. • Temperature increase the same size, X and Y are each filled with 1.0 kg
• Temperature increase faster. slower. of water at the same temperature of 25°C.
• Land warmer than sea. • Sea is cooler than land. Sphere A is placed in beaker X and sphere B in
beaker Y. The water in both beakers is stirred
slowly and the temperature is measured when
thermal equilibrium in each beaker is reached.
Land breeze
• The movement of cool air from the land to the
sea as a result of hot air rising from the sea during
aluminium
the night.
copper
1 kg water
A B at room temperature
During night time, heat is lost Cooler air from the land
xxxxxxxxxxxxxxxx 25 ºC
from the sea and the land. moves towards the sea.

A B
Hot air
above the
sea rises.

The table below shows the values of the specific


heat capacities of water, aluminium and copper.

Substance Specific heat capacity,


warm sea
land c (J kg –1 °C –1)

Water 4200

Aluminium 900
• Land has a lower • Sea has a higher specific
specific heat capacity. heat capacity. Copper 390
• Cools off faster. • Cools off slower.
• Land is cooler than sea. • Sea is warmer than land.

221 Heat
(a) Explain the meaning of ‘the specific heat capacity (b) Explain the relationship between specific heat
of water is 4200 J kg–1 °C–1’. capacity and heat capacity.
(b) Compare the equilibrium temperatures reached (c) Calculate the energy required to heat 2 kg of
in beaker X and beaker Y. Explain why there is a copper to raise its temperature by 15 °C.
difference. [Specific heat capacity of copper = 380 J kg–1 °C–1]
(c) Calculate the equilibrium temperature reached in (d) 1000 cm3 of water at 80 °C is mixed with
beaker Y. 2500 cm3 of water at 30 °C. The mixture is
(d) What is the assumption made in your calculation stirred. What is the final temperature of the
for (c)? mixture? [Density of water = 1 g cm–3]
3 (a) Explain the meaning of ‘specific heat capacity’ (e) Why is the handle of a cooking pot made of
and ‘heat capacity’ of an object. plastic which has a high specific heat capacity?

F
O 4.3 Understanding Specific Latent Heat
4

R
CHAPTER

M
Latent Heat
4
What observations and inferences can be made
from this experience?

iced
drink Change of Phase and Latent Heat

1 Matter can exist in three phases or states, that


is, solid, liquid and gas. For example, water
Figure 4.23 molecules exist in the solid phase as ice, liquid
phase as water, and gas phase as steam.
You are given a glass of cool drink, with lots of ice 2 A change in the phase of matter from one
cubes. You can feel that the drink is cold (at 0 °C if phase to another occurs at a specific
you measure its temperature with a thermometer), temperature and pressure. A change in phase
and it remains at that temperature as long as there normally occurs with the absorption or the
are some unmelted ice. After all the ice cubes have release of heat followed by a change in the
melted, the temperature of the drink begins to rise. volume and density of the substance.

Activ To observe the changes in temperature when heat is supplied to


ity 4.5 (a) a solid at its melting point (b) a liquid at its boiling point

Apparatus/Materials 2 The apparatus is set up as shown in Figure 4.24.


Beaker, ice cubes, thermometer, Bunsen burner, 3 The initial temperature of the ice cubes is
tripod stand, stirrer, and wire gauze. recorded.
4 The Bunsen burner is lit and the stopwatch is
Arrangement of apparatus
started simultaneously.
stirrer 5 The reading on the thermometer is recorded
every 1 minute until all the ice cubes have
ice hot boiling changed from the solid phase to the gaseous
water water
phase (water begins to boil). The time when the
ice begins to melt and the time when the water
begins to boil are noted.
(a) (b) (c) 6 The phase of the ice is observed and tabulated
Activity 4.5

Figure 4.24 throughout the heating process.


7 The contents of the beaker are stirred from time
Procedure to time to ensure uniform heating. (Precaution
1 Ice cubes are placed in a beaker until it is three- step)
quarters full.

Heat 222
8 The recording of the readings on the 2 The thermometer showed 0 °C before the
thermometer is continued for a few minutes heating process started. Heat absorbed by the
after the water starts boiling. ice is used to melt the ice cubes. The volume of
9 The readings are tabulated and a graph of ice decreases while the volume of water
temperature against time is plotted. increases. However, the temperature remains at
0 °C.
Tabulation of data
3 After all the ice cubes have melted, the
Table 4.2 temperature of water begins to rise as the water
absorbs heat from the Bunsen burner. The
Time, t (s) Temperature, θ (°C) Phase temperature of water rises until it reaches
0 0 Solid 100 °C, at which point the water starts boiling.
60 . 4 After reaching the boiling point, the temperature
F
. . . of the water remains at 100 °C. Steam is formed O
and the volume of water decreases while the

4
. . R
water boils.

CHAPTER
. . . M

. 100 Gas 5 Thus, we can conclude that the transfer of heat


during a change of phase (from solid to liquid or 4
from liquid to gas) does not cause a change in
Graph temperature because heat absorbed is used to
temperature, θ (°C) overcome the force of attraction between the
molecules of solid and liquid (or liquid and gas).
liquid 6 On the other hand, when there is no change
+ of phase, the heat absorbed increases the
gas
100 temperature of the substance being heated
because heat absorbed has been transformed into
on uid

kinetic energy and cause a rise in temperature.


ly
liq

solid 7 The heat absorbed or released when a


+
liquid time, substance undergoes a change of phase
0 t (s) without a change in temperature is called the
Figure 4.25 latent heat of the substance.

Discussion Conclusion
1 From the graph in Figure 4.25, it is noted that During the process of melting and boiling, the
there is no change in temperature when heat is respective temperatures remain constant although
supplied to melting ice and boiling water. the supply of heat is continuous.

Latent heat is the total energy absorbed or released when


Latent heat absorbed a substance completely changes its physical state at a
constant temperature.
Fusion Vaporisation • Latent heat of fusion is the heat absorbed when a solid
(melting) (boiling)
melts at constant temperature.
• Latent heat of vaporisation is the heat absorbed when
latent heat latent heat of
of fusion
a liquid changes into vapour at constant temperature.
vaporisation

Solid Liquid Gas Note:


Activity 4.5

Solidification Condensation
(freezes) (condenses) A solid can also be converted directly into a gas.
This process is known as sublimation.
Latent heat released

223 Heat
F

4
R
O

M
CHAPTER 4
Heating Curve

Heat
Explanation of the Temperature-Time Graph Based on the Kinetic Theory of Matter

The heating curve in Figure 4.26 shows the temperature variation of a substance being heated at a fixed rate.

• Molecules in a solid are arranged in regular • The solid begins to melt. • The liquid starts to boil.
patterns and are held together tightly. • Latent heat of fusion is absorbed by the solid to provide the • Latent heat of vaporisation is absorbed by the liquid. This
• When the solid is heated, the molecules gain energy to overcome the forces of attraction between the molecules. provides the liquid with the energy to do work to overcome the
energy and vibrate about their fixed positions at • The molecules are freed from their fixed positions and are able to forces of attraction between the molecules and to work against the
greater speeds and greater amplitudes. move freely. atmospheric pressure as the liquid expands into gas.
• The kinetic energy of the molecules increases • The energy absorbed does not increase the kinetic energy of the • As the heat absorbed does not increase the kinetic energy of the
and the temperature of the solid rises. molecules, so the temperature remains constant during melting. molecules, the temperature remains constant during boiling.

• The liquid temperature rises.


Temperature • The heat supplied increases • The heat supplied increases
(°C) the kinetic energy of the liquid the kinetic energy of the gas
molecules which begin to and its temperature rises.
move faster and randomly. F • All the liquid has turned into
vapour.
• The gas molecules are now
completely free to move.

224
Gas

During the heating process,


D E the time, t2 required for a
boiling point complete change of phase
Liquid + Gas from liquid to gas is greater
than the time, t1 that is
Liquid required for a complete
B C
melting point change of phase from solid to
Solid + Liquid liquid. The reason is that the
A latent heat of vaporisation is
Solid
greater than the latent heat of
fusion. More energy is used
by liquid molecules to change
solid is heating solid is melting liquid is heating liquid is boiling gas is heating into a gas due to the
Time (s)
0 additional energy required to
melting begins melting completes boiling begins boiling completes do work against the external
t1 t2 atmospheric pressure.

Figure 4.26
Cooling Curve

Figure 4.27 shows the cooling curve of a substance in a gaseous state when it is being cooled at a
fixed rate and undergoes a change of phase from gas to liquid to solid.

• When the gas is cooled, • When the gas reaches the


heat is released to the condensation temperature (point B),
surroundings and the it begins to condense into a liquid.
kinetic energy of the • The temperature remains constant.
molecules decreases. • Latent heat of vaporisation is
• The temperature of the • When the liquid reaches the
released to the surroundings.
gas decreases. freezing point, it begins to freeze.
• The molecules become more
closely packed.
• The average kinetic • The temperature remains
Temperature (°C) energy of the liquid constant.
molecules decreases. • Latent heat of fusion is released.
• The temperature keeps
falling.
A

• The molecules are closely


packed.
• All the liquid has solidified.

225
• The temperature
Gas
decreases.

B C
condensation
point Gas + Liquid

Liquid

D E
freezing
point Liquid + Solid

Solid
F

gas is cooling gas is condensing liquid is cooling liquid is freezing solid is cooling
Time (s)

condensation begins condensation completes freezing begins freezing completes

Figure 4.27

Heat
CHAPTER 4
F

4
R
O

M
Specific Latent Heat

Q
=
m
Q = latent heat absorbed or
Specific latent heat,  released by the
• Amount of heat required to change substance
the phase of 1 kg of the substance m = mass of the substance
at a constant temperature.
F
O
4

R
CHAPTER

4 Q = m
Unit: If heat is supplied
J kg–1 electrically to
change the phase of
a substance:
Pt = m

Specific latent heat of fusion Specific latent heat of vaporisation

• Amount of heat energy required to change 1 kg • Amount of heat energy required to change
of a substance from a solid phase to a liquid 1 kg of a substance from the liquid phase to
phase without a change in temperature. the gaseous phase, without a change in
temperature at its boiling point.
fusion
vaporisation
absorbed
latent heat absorbed
of fusion
latent heat
released of vaporisation

solidification released
1 kg of solid 1 kg of liquid
1 kg of liquid condensation 1 kg of gas
• Recall that the temperature of an object depends
on the average kinetic energy of the particles of • During vaporisation, the heat absorbed by the
the object. In the process of melting, the heat liquid is used to overcome the forces of attraction
absorbed by the solid is used to overcome the between its molecules which disperse as
forces of attraction between the molecules. The molecules of gas and to do work against the
average kinetic energy of the molecules thus does external atmospheric pressure as it expands into
not increase and therefore, melting occurs at a a gas.
constant temperature. • The average kinetic energy of the molecules does
• During solidification, heat is released as the not increase and, therefore, vaporisation occurs at
particles move back into their fixed positions in a constant temperature.
the regular structure.

Heat 226
Activ To determine the latent heat of fusion of ice
ity 4.6
Apparatus/Materials Mass of ice melted by heat from the surroundings:
Pure ice, electric immersion heater, filter funnel, ma = (m3 – m1) kg
beaker, stopwatch, weighing balance, power supply, Mass of ice melted by the electric immersion heater
retort stand, and clamp. and heat from the surroundings:
mb = (m4 – m2) kg
Arrangement of apparatus
∴ Mass of ice melted by the electric immersion
to power heater:
immersion supply 12 V
heater A immersion m = (mb – ma) kg F
heater B
ice Electrical energy supplied by the electrical O
ice

4
immersion heater B: R
filter funnel

CHAPTER
filter funnel M
E = Power × Heating period
beaker A = Pt
beaker B 4
water
water Heat energy absorbed by the ice during melting:
Control experiment
(a) Set A (b) Set B
Q = m
Assuming that there is no heat loss to the
Figure 4.28 surroundings:
Electrical energy Heat energy absorbed
=
supplied by the melting ice
Procedure
Pt = m
1 The apparatus is arranged as Set A and Set B as
Pt
shown in Figure 4.28. ∴ Specific latent heat of fusion of ice,  =
2 The mass of each of the two empty beakers, A m
and B is determined using the weighing balance.
3 Each of the two filter funnels is filled with ice Discussion
cubes. 1 The purpose of Set A, the control experiment, is
4 The immersion heater in Set A, the control to determine the mass of ice melted by the heat
experiment, is not connected to the power from the surroundings.
supply. The purpose of Set A is to determine the 2 The immersion heater must be fully immersed
mass of ice melted by the surrounding heat. in the ice cubes to avoid or reduce heat loss.
The heater in Set B is switched on. (Precaution step)
5 When water starts to drip from the filter funnels 3 The stopwatch is not started simultaneously
at a steady rate, the stopwatch is started and the when the immersion heater B is switched on
empty beakers, A and B are placed beneath the because this heater requires some time before it
filter funnels. reaches a steady temperature. At this point, the
6 After a period of t seconds, the heater B is rate of melting of the ice will be steady.
switched off. The masses of both beakers of 4 The value of the specific latent heat of fusion
water, A and B are determined using the of ice,  obtained in this experiment is higher
weighing balance. The time taken is measured than the standard value because part of the
using the stopwatch. heat supplied by the heater is lost to the
surroundings. This loss of heat causes the value
Results/Calculation of m to be lower than the expected mass and
Mass of empty beaker A = m1 kg hence the calculated value of  is higher.
Activity 4.6

Mass of empty beaker B = m2 kg


Conclusion
Mass of beaker A + water = m3 kg
The specific latent heat of fusion of ice can be
Mass of beaker B + water = m4 kg determined using the electrical method.

227 Heat
9 10
The specific latent heat of fusion of ice is 900 kJ of heat is required to melt a block of copper at
336 000 J kg–1. What is the quantity of heat required its melting point. If the specific latent heat of fusion
to melt 2.5 kg of ice at 0 °C? of copper is 500 000 J kg–1, calculate the mass of the
copper block.
Solution Solution
Heat required, Q = m From the equation, Q = m,
= 2.5 × 336 000 900 000 = m × 500 000
= 840 000 900 000
m = _______ = 1.8 kg
= 840 kJ 500 000
F
O
4

R
CHAPTER

M 11
4 Students must note that the
This problem involves the specific heat capacity, c and the specific latent heat, .
melting point of ice is
What is the heat required to change 1.2 kg of ice at –5 °C to water at 30 °C? 0 °C, i.e. ice will change into
[Specific heat capacity of ice = 2100 J kg–1 °C–1; water at 0 °C.
Specific heat capacity of water = 4200 J kg–1 °C–1; The problem involves
Specific latent heat of fusion
F of ice = 336 000 J kg–1] 3 stages:
O (i) Heat supplied to ice
Solution
4

R from –5 °C to 0 °C
CHAPTER

M (involves the specific


heat capacity of ice).
4 (ii) Heat supplied to melt
ice water
θ1 = –5 °C θ4 = 30 °C the ice at 0 °C (involves
the specific latent heat
absorbs Q Q3 absorbs
heat 1
heat of fusion of ice).
(iii) Heat supplied to water
absorbs heat
from 0 °C to 30 °C
Q2
(involves the specific
heat capacity of water).
ice water
θ2 = 0 °C θ3 = 0 °C

Heat energy required to heat the ice from –5 °C to 0 °C:


Q1 = mcθ
Do not calculate from –5 °C to 30 °C because at
= 1.2 × 2100 × [0 – (–5)] 0 °C, ice will melt and the value of the specific heat
= 12 600 J capacity of ice is not applicable from 0 °C to 30 °C.

Heat energy required to melt the ice at 0 °C: If a problem involves:


Q2 = m (a) an increase or a decrease
Students often forget to calculate this step. in temperature of a
= 1.2 × 336 000 Remember that the specific latent heat of fusion is
involved when a solid changes to a liquid. substance only, use
= 403 200 J
Q = mcθ.
(b) a change in phase of a
Heat energy required to heat the water from 0 °C to 30 °C: substance only, use
Q3 = mcθ Q = m.
= 1.2 × 4200 × (30 – 0) = 151 200 J (c) both an increase or a
Total heat energy required, Q decrease in temperature
= Q1 + Q2 + Q3 and a change in phase of
= (12 600 + 403 200 + 151 200) J the substance, use both
= 567 000 J Q = mcθ and Q = m.

Heat 228
12
A copper sphere of mass 200 g is heated in boiling water and then transferred to a block of ice at 0 °C.
How much ice will be melted?
[Specific heat capacity of copper = 380 J kg–1 °C–1;
Specific latent heat of fusion of ice = 3.36 × 105 J kg–1]
Solution
Use the mixture method. Let m′ be the mass of ice melted.
Heat released by
= Heat absorbed by ice
heat heat copper sphere
released absorbed
by copper by ice
mcθ = m′
0.2 × 380 × (100 – 0) = m′ × 3.36 × 105 F
O
m′ = 0.023 kg

4
R
boiling water, ice at 0 °C ice melted ∴ 0.023 kg of ice will be melted.

CHAPTER
100 °C M

F4/4/44
4

Activ To determine the specific latent heat of vaporisation of water


ity 4.7
Apparatus/Materials Results/Calculation
Beaker, electric immersion heater, pure water, Electrical power of heater = P watts
electronic weighing balance, stopwatch, power Time period of boiling = t seconds
supply, retort stand, and clamp. Electrical energy supplied by the heater, E = Pt
Arrangement of apparatus Mass of water evaporated, m = m1 – m2
Heat absorbed by water during vaporisation, Q = m
to power supply
Assuming that the heat loss to the surroundings is
negligible and that all the heat supplied by the
clamp heater is absorbed by the water,
immersion heater Q =E
retort m = Pt
stand water boiling ∴ Specific latent heat of vaporisation of water,
electronic weighing balance Pt
=
Figure 4.29 m
Discussion
Procedure
1 The immersion heater must be fully immersed
1 The apparatus is set up as shown in Figure 4.29. in the water to avoid or reduce direct heat loss to
2 A beaker is placed on a platform of an electronic the surroundings. (Precaution step)
weighing balance. 2 The value of the specific latent heat of
3 An electric immersion heater (rated P watts) is vaporisation of water,  obtained in this
fully immersed in the water and held in this experiment is higher than the standard value
position by a retort stand. because part of the heat is lost to the
4 The electric heater is switched on to heat the surroundings. This loss of heat causes the value
water to its boiling point. of m to be less than the expected value and
5 When the water starts to boil at a steady rate, the hence the calculated value of  is higher than
stopwatch is started and the reading on the the standard value.
electronic balance, m1 is recorded.
Activity 4.7

6 The water is allowed to boil for a period of Conclusion


t seconds. The specific latent heat of vaporisation of water is a
7 At the end of the period of t seconds, the reading constant, which can be determined by electrical
on the electronic balance, m2 is recorded. means.

229 Heat
13
The specific latent heat of vaporisation of water is 2 260 000 J kg–1.
What is the quantity of heat released when 5 kg of steam condenses to water?

Solution
Heat energy released, Q = m
= 5 × 2 260 000 J kg–1
= 11 300 kJ
= 1.13 × 104 kJ
F
O
4

R
CHAPTER

M
14
4
Figure 4.30 shows the temperature-time graph for a (b) Melting point = 55 °C
substance X, of mass 2.5 kg, being heated using a Boiling point = 95 °C
300 W heater.
(c) The specific latent heat of vaporisation of
temperature (°C) F substance X is greater than its specific latent heat
O
of fusion. This is because more heat energy is
4

R
required by liquid molecules to change to gas
CHAPTER

M
95 molecules since additional energy is required to
55 4 do work against the atmospheric pressure when
the liquid molecules expand into gas.
27
(d) From point A to point B, the temperature of the
0 time
2 4 7 15 (min) solid rises by 28 °C in a time of 2 minutes.
Energy absorbed by the solid substance X
Figure 4.30
= Energy supplied by the heater
Based on the above graph, mcθ = Pt
(a) what are the states of the substance represented by 2.5 × c × 28 = 300 × 2 × 60
the regions AB, BC, CD, DE and EF? c = 514.3 J kg–1 °C–1
(b) state the melting point and boiling point of the
substance X. (e) The time taken for the change in phase from a
(c) why is the period of time for DE greater than that solid to a liquid = 2 minutes (region BC)
for BC? Energy absorbed during melting
(d) determine the specific heat capacity of the solid = Energy supplied by the heater
substance X. m = Pt
(e) calculate the specific latent heat of fusion of 2.5 = 300 × 2 × 60
substance X.  = 14 400 J kg–1
(f) calculate the specific latent heat of vaporisation of
substance X. (f) The time taken for the change in phase from a
Solution liquid to a gas is 8 minutes (region DE).
(a) Region AB = solid Energy absorbed during vaporisation
Region BC = solid and liquid = Energy supplied by the heater
Region CD = liquid m = Pt
Region DE = liquid and gas 2.5 = 300 × (8 × 60)
Region EF = gas  = 57 600 J kg–1

Heat 230
15
A 500 W electric heater is used to boil water. Solution
What is the time required to reduce the mass of water Use the energy equivalence method.
by 80 g after the water has reached its boiling point? Electric energy = Heat energy
[Specific latent heat of vaporisation of water Pt = m
= 2.26 × 106 J kg–1] m 0.08 × 2.26 × 106
t = ––– = ––––––––––––––– = 362 s
P 500
= 6 minutes 2 seconds

16 F
O
What is the mass of steam at 100 °C required to heat Final temperature of mixture = 100 °C

4
R
1 kg of ice until it boils? Let the mass of steam condensed = m′

CHAPTER
M
[Specific heat capacity of water = 4200 J kg–1 °C–1; Heat released by steam during condensation
[Specific latent heat of fusion of ice = 3.36 × 105 J kg–1; = m′ = m′ × 2.26 × 106 4
[Specific latent heat of vaporisation of water Heat absorbed by ice
[= 2.26 × 106 J kg–1] = Q1 + Q2 = m + mcθ
= 1 × 3.36 × 105 + [1 × 4200(100 – 0)]
Solution
= 756 000 J
1 kg ice Heat released by steam = Heat absorbed by ice
absorbs absorbs
0 °C heat heat 2.26 × 106m′ = 756 000
Q1 Q2 m′ = 0.335 kg
water at 0 °C water at 100 °C ∴ Mass of steam required is 0.335 kg.

Applications of Specific
F4/4/48 Latent Heat in Steam for Cooking
Daily Life
1 Water has a large specific latent heat of
Water as a Coolant vaporisation.
2 When 1 kg of steam condenses to water, an
1 When 1 kg of ice melts, its large latent heat amount of 2.26 × 106 J kg–1 of heat is released.
of 3.36 × 105 J kg–1 is absorbed from the 3 This property enables steam to be used for
surroundings. This property makes ice a suitable cooking by the method of steaming. Food such
substance for use as a coolant to maintain other as fish, cakes, buns etc, can be cooked by
substances at a low temperature. steaming. The latent
2 Drinks can be cooled by adding in several heat released directly
cubes of ice. When the ice melts, a large onto the food enables
amount of heat (latent heat) is absorbed and the food to be cooked
this lowers the temperature of the drink. at a higher rate.
3 The freshness of foodstuffs such as fish and 4 Latent heat released
meat can be maintained by placing them in from condensing
contact with ice. With its large latent heat, ice is steam can also be used
able to absorb a to heat beverages such
large quantity of as coffee in coffee-
heat from the making machines. Figure 4.32
foodstuffs as it
melts. Thus, food
can be kept at a
Do you know why our bodies feel cool after sweating?
low temperature
This is because latent heat of vaporisation is absorbed
for an extended from the body when sweat evaporates.
period of time. As a result, the body is cooled by the removal of heat.
Figure 4.31

231 Heat
2 SPM
Clone
’07

1 kg of ice (at 0 °C) is heated until it changes into 1 kg of steam through a heating process as shown in the diagram below.
R S T U
Q1 Q2 Q3
1 kg ice 1 kg water 1 kg water 1 kg steam
at 0 °C at 0 °C at 100 °C at 100 °C

Q1, Q2 and Q3 are the heat energies absorbed during the heating process.
(a) Name the heat energy, Q1 absorbed by the ice from process R to process S.
(b) Explain why the temperature at process R and process S is the same even though heat is absorbed.
(c) Calculate the heat energy, Q2 absorbed from process S to process T.
F (d) Sketch the graph of temperature against time to show the change of phase from process R to process U.
O
Solution
4

R
CHAPTER

M (a) Specific latent heat of fusion is absorbed from (d) Temperature (°C)
process R to process S.
4
(b) The heat is not used to increase the kinetic energy
but to overcome the forces of attraction between 100
the water molecules.
(c) Q2 = mcθ
= 1 × 4200 × (100 – 0) 0 Time (s)
= 420 000 J

4.3

1 A P watt immersion heater is used to melt ice at 0 °C. When the water starts to drip at a steady rate, an
If 89.3 g of ice is melted in 5 minutes, what is the empty dry beaker is placed under the filter funnel in
value of P? place of the first beaker and the stopwatch is started.
[Specific latent heat of fusion of ice After a period of 10 minutes, the beaker is removed
= 3.36 × 105 J kg–1] and the mass of water collected is determined.
(a) The mass of water collected in the period of 10
2 1 kg of ice is mixed with 1 kg of water at 25 °C in a
minutes is 200 g and the power rating of the
closed insulated container. What is the percentage of
heater used is 60 W.
ice remaining?
(i) Calculate the total heat supplied by the
[Specific heat capacity of water = 4200 J kg–1 °C–1; heater in 10 minutes.
Specific latent heat of fusion of ice (ii) Based on the results in (i), determine the
= 3.36 × 105 J kg–1] specific latent heat of fusion of ice.
3 The figure below shows one set of apparatus used in (b) What are the other sources of heat other than
an experiment to determine the specific latent heat that from the heater?
of fusion of ice. (c) Compare the value of the specific latent heat of
fusion obtained in the experiment with the
to power supply
standard value of 3.36 × 105 J kg–1. Give a reason
immersion heater
ice for the difference between the experimental and
standard values.
filter funnel (d) Propose a method to increase the accuracy of
the experiment.
(e) The figure shows an error which reduces the
beaker accuracy of the experiment. State the error.
water (f) Given that the specific latent heat of fusion of ice
is 3.4 × 105 J kg–1 and the specific heat capacity
of water is 4.2 × 103 J kg–1, determine the time
After the immersion heater is switched on, water is taken by a 500 W heater to change 0.5 kg of ice
allowed to drip from the filter funnel into a beaker. to boiling water at 100 °C.

Heat 232
4.4 Understanding the Gas Laws

Existence of Gas Pressure

1 There are many phenomena that involve gas pressure in daily life.
The existence of gas pressure that is commonly experienced is atmospheric pressure.
2 The existence of gas pressure can be observed in the following activities.
Table 4.3

Activity Observation
Try to compress a balloon between the palms of your hands.
The balloon is not flattened and a resistance is felt due to the air pressure in F
O
the balloon.

4
R

CHAPTER
M

4
air pressure A ruler is placed underneath a sheet of newspaper with a portion of the ruler
protruding out over the edge of the table. When the protruding end of the ruler
is forced down, it is observed that it is difficult to lift up the sheet of
newspaper. This is because the atmospheric pressure exerts a force on the
sheet of newspaper.

A dented ping-pong ball is placed in hot water. It is observed that the ping-
hot
pong ball reverts to its original spherical shape. This is because an increase in
water the air pressure inside the ping-pong ball pushes the wall of the ball back to its
original shape.

3 The activities show the existence of gas pressure. Why does gas pressure exist? This question
can be explained by the kinetic theory of matter as discussed in Chapter 3.

Activ To study the behaviour of the molecules of a gas using a model


ity 4.8
Apparatus/Materials Procedure
An electrically operated three-dimensional kinetic 1 The three-dimensional kinetic theory model as
theory model. shown in Figure 4.33 is switched on.
Arrangement of apparatus 2 The speed of the motor is adjusted so that the
polystyrene disc is held at the middle of the tube
perspex by the impact of the ball bearings.
tube
3 The motion of the ball bearings is observed.
polystyrene
4 The voltage of the power supply is slowly
disc, P increased. The changes in the motion of the ball
bearings and the position of the polystyrene disc
small phosphor-bronze are observed.
ball bearings
5 A similar polystyrene disc Q is added to the
disc P [Figure 4.34(c)]. The changes in the
motor-driven
vibrator motion of the ball bearings and the position of
the disc are observed.
Activity 4.8

Notes
to low
voltage (a) Ball bearings represent gas molecules.
d.c. supply (b) The impact of the ball bearings represents the
Figure 4.33 impact of the gas molecules.

233 Heat
(c) The speed of the motor represents the Discussion
temperature of the gas. 1 The molecules of a gas are in random motion,
(d) The volume of the space between the rubber and exert pressure on the walls of the container.
sheet and polystyrene disc represents the 2 (a) When the speed of the piston is increased, the
volume of the trapped gas. gas molecules move faster. The gas is thus at
Observation a higher temperature as the average kinetic
energy of the gas molecules increases.
P increasing increasing
‘temperature’ ‘pressure’
(b) The polystyrene disc moves to a higher
P
of gas of gas Q position indicating an increase in the
P
volume of gas, as shown in Figure 4.34(b).
(c) We infer that when the temperature of a
rubber fixed mass of gas increases, the volume of
F sheet
O the gas increases provided the pressure of
(b) (a) Initial condition (c)
4

R the gas is constant.


Figure 4.34 3 (a) When we add another similar disc Q to disc
CHAPTER

M
1 When the three-dimensional model is switched P, we increase the pressure of a gas.
4
on, the ball bearings move in random motion. (b) The two discs move down the tube, reducing
2 When the voltage is increased (speed of the piston the volume of the gas, as shown in Figure
increased), disc P is pushed to a higher position 4.34(c).
and the ball bearings move at a faster rate. (c) We infer that when the pressure on a fixed
3 When disc Q is added to disc P, disc P moves to mass of gas increases, the volume of the gas
a lower position. decreases provided the temperature is constant.

The Gas Laws Boyle’s law and the kinetic theory of gases

1 The relationship between the pressure, volume When the temperature of a gas is kept constant,
and temperature of a gas is given by the gas laws. the average kinetic energy of the particles of gas
2 The gas laws include: remains unchanged. When the gas is
(a) Boyle’s law which states the relationship compressed (that is, its volume is decreased),
between the pressure and volume of a the number of particles per unit volume of gas
given mass of gas at a constant increases. Therefore, the frequency of collisions
temperature. between the particles and the walls of the
(b) Charles’ law which states the relationship container increases resulting in a corresponding
between the temperature and volume of a increase in the pressure of the gas.
given mass of gas at a constant pressure.
(c) Pressure law which states the Conclusion
relationship between the pressure and
When the volume of a gas is reduced, the
temperature of a given mass of gas at a
pressure of the gas increases.
constant volume.

load
Boyle’s Law
load

1 Boyle’s law states that the pressure of a fixed


mass of gas is inversely proportional to its gas
volume provided the temperature of the gas is molecules
kept constant.
∴ P ∝ 1 (at constant temperature)
Activity 4.8

V
or PV = constant
P1V1 = P2V2

Heat 234
2 The graphs of a gas obeying Boyle’s law are shown in Figure 4.35.
P PV
P
PV is constant
hyperbolic while V varies
curve a straight
line through
the origin
1 V
V O
O O V
(a) P against V 1 (c) PV against V
(b) P against
V
Figure 4.35

4.1
F
The gas laws — Boyle’s law O

4
R
Hypothesis 1

CHAPTER
6 A graph of P against is drawn. M
The volume of the balloon will decrease. The volume V
of a gas inversely proportional with its pressure. 4
Tabulation of data
Aim
Table 4.4
To investigate the relationship between the pressure,
P and volume, V of air at a constant temperature (to 1
verify Boyle’s law) Volume, V (cm3) (cm–3) Pressure, P (Pa)
V
Variables
100
(a) Manipulated : volume of air in a syringe, V
(b) Responding : pressure of trapped air, P 90
(c) Fixed : mass and temperature of air inside 80
a syringe (an airtight syringe is used) 70
Apparatus/Materials 60
A 100 cm3 syringe, rubber tube, and Bourdon gauge.
Arrangement of apparatus Graph
1 P (Pa)
The graph of P against :
syringe rubber tube V

force
piston air Bourdon 1
gauge (cm-3)
O V
Figure 4.36
Figure 4.37
Procedure
1 The apparatus is set up as shown in Figure 4.36. Discussion
2 A rubber tube is connected to the nozzle of the The accuracy of the experiment can be increased by
syringe. The piston of the syringe is adjusted until using the shortest possible length of rubber tube. This
the volume of air in the syringe is 100 cm3 at will minimise the volume of air in the tube which
atmospheric pressure. 1
3 The other end of the rubber tube is connected to a should not be included in the calculation of .
V
Bourdon gauge and the pressure of the air in the
syringe is read from the gauge. Conclusion
The graph shows that the pressure, P is directly
Experiment 4.1

4 The piston of the syringe is pushed in until the


enclosed volume is 90 cm3. The volume of air is 1 1
proportional to , i.e. P ∝ .
read from the scale on the syringe. The pressure V V
on the Bourdon gauge is recorded. This means that the pressure, P varies inversely with
5 Step 4 is repeated for enclosed volumes of the volume, V.
80 cm3, 70 cm3, and 60 cm3. The hypothesis is valid.

235 Heat
17
Figure 4.38 shows a glass capillary tube, closed at one When the tube is held vertically with its open end
end, with a drop of mercury trapped inside. at the top:
trapped air atmospheric
pressure
atmospheric
pressure
8 cm 2 cm mercury

Figure 4.38
When the tube is in the horizontal position, the length 2 cm

F of the air column in the glass tube is 8 cm. The length trapped
O of the mercury thread is 2 cm. What is air
4

R (a) the pressure of the column of trapped air when the


CHAPTER

M glass tube is held horizontally?


(b) the length of the column of trapped air when the Final pressure of the column of trapped air, P2
4
glass tube is held vertically with its open end at the Atmospheric Pressure of
= pressure + mercury thread
top?
[Atmospheric pressure = 76 cm Hg] = 76 cm Hg + 2 cm Hg
Solution = 78 cm Hg
(a) When the tube is held horizontally, the pressure of From Boyle’s law: P1V1 = P2V2
the trapped air is the same as the atmospheric PV
pressure outside. V2 = 1 1
P2
∴ Pressure of the trapped air, P = 76 cm Hg
76 × 8
(b) Initial pressure of the trapped air column, =
P1 = 76 cm Hg 78
Initial volume of the trapped air, Vl = 8 cm = 7.8 cm
∴ The length of the column of trapped air when
The length of air column represents
the volume of trapped air. the tube is held vertically is 7.8 cm.

18
A bicycle pump of uniform cross section has a length The volume of air in the pump is proportional to the
of 30 cm and contains air at atmospheric pressure. length of the pump, which has a uniform cross
If the piston is pushed inwards a distance of 12 cm to section.
pump air into a tyre, what is the pressure of the air in Initial volume, Vl = 30 cm
the pump at this instant? Initial pressure, P1 = 1 atm
Final volume, V2 = 30 – 12
= 18 cm
Solution
Final pressure, P2 = ?
From Boyle’s law:
P1V1 = P2V2
PV
12 cm P2 = 1 1
30 cm V2
1 × 30
=
18
(a) (b)
= 1.67 atm

Heat 236
The Pressure Law The pressure law and the
kinetic theory of gases
1 The pressure law states that the pressure of a
fixed mass of gas is directly proportional to its
absolute temperature (in kelvin), provided
the volume of the gas is kept constant.
∴ P ∝ T (at constant volume)
or P = constant
T

P1 P
= 2
T1 T2
F
2 The graphs of a gas, which obey the pressure When the temperature of a gas in a container O

4
law, are shown in Figure 4.39. rises, the average kinetic energy of the particles R

CHAPTER
of gas increases and the particles move faster. M
P (Pa)
Hence, the gas molecules strike the walls of the
4
container
(a) at a higher frequency,
(b) more vigorously (with a larger momentum).
O O 273 373
T (K) The above two factors increase the rate of the
change of momentum of the gas. As a
(a) P against θ in Celsius (b) P against T in kelvin consequence, the force and hence the pressure
scale scale increases.
Figure 4.39
Conclusion
The pressure of a gas increases with its
temperature.

4.2
The gas laws — Pressure law
Hypothesis Arrangement of apparatus
The pressure of a gas increases when its temperature rubber
rises. thermometer tube Bourdon
stirrer gauge
Aim beaker
ice
To investigate the relationship between the pressure, dry air
P and temperature, T of a gas at constant volume (to round-bottomed flask
water bath
verify the pressure law)
heat
Variables
Figure 4.40
(a) Manipulated : temperature of the trapped air, θ
Procedure
Experiment 4.2

(b) Responding : pressure of the trapped air, P


(c) Fixed : mass and volume of gas 1 The apparatus is set up as shown in Figure 4.40.
2 The round-bottomed flask is immersed in a
Apparatus/Materials beaker of water containing ice. The mixture of
Round-bottomed flask, mercury thermometer, water and ice is stirred continuously until the
Bourdon gauge, Bunsen burner, tripod stand, wire temperature of the water bath is steady. The
gauze, retort stand, stirrer, and ice. temperature, θ and the pressure, P are recorded.

237 Heat
3 The water bath is heated slowly until the pressure, P (Pa)
temperature of the water reaches 10 °C.
4 The Bunsen burner is moved away and the water
bath is stirred until the temperature of the air is in
temperature, θ (°C)
equilibrium with the temperature of the water –273 O 100
bath. The temperature and pressure of the air are
Figure 4.41
recorded.
5 The above steps are repeated for temperature (b) A graph of P against T (in kelvin) is shown in
increments of 10 °C until the water bath reaches Figure 4.42.
a temperature of 90 °C. pressure, P (Pa)
6 The graph of air pressure, P against temperature,
θ is drawn. A temperature scale ranging from
F –300 °C to 100 °C is used.
O
temperature, T (K)
4

R O 273
Tabulation of data
CHAPTER

M
Figure 4.42
4 Table 4.5 Discussion
1 If the pressure continues to drop as shown in
Temperature, θ (°C) Air pressure, P (Pa) Figure 4.41, the gas would exert no pressure at
10 P1 all at absolute zero. This happens at a
20 P2 temperature of –273 °C. If the temperature
30 . –273 °C, which is the lowest temperature
40 . theoretically obtainable, is used as the origin of a
50 . scale called the kelvin scale, the graph is a
60 . straight line passing through the origin as shown
70 . in Figure 4.42.
80 . 2 The temperature measured on this scale is called
90 . absolute temperature and the kelvin, K is the
unit of absolute temperature.
Conclusion
Graph The pressure of a fixed mass of gas at constant
(a) A graph of P against θ (°C) is shown in Figure volume, is directly proportional to its absolute
4.41. The graph intercepts the temperature axis temperature.
at –273 °C. The hypothesis is valid.

kelvin scale Celsius scale Increase of 100 °C = Increase of 100 K


∴ Increase of 1 °C = Increase of 1 K
100 θ °C 100 Summary:
gradations gradations +273
θ °C TK
Experiment 4.2

–273
Example: 30 °C = (30 + 273) K = 303 K
130 °C = (130 + 273) K = 403 K
From 0 °C → 100 °C, the increase is 100 °C. Conversely: 373 K = (373 – 273) °C = 100 °C
From 273 K → 373 K, the increase is 100 K. 313 K = (313 – 273) °C = 40 °C

Heat 238
19
A closed flask contains gas at a temperature of 95 °C and a pressure of 152 kPa.
If the temperature rises by 24 °C, what is the new pressure of the gas?
Solution
Initial pressure, P1 = 152 kPa
Initial temperature, T1 = 273 + 95 = 368 K
Final pressure, P2 = ?
Final temperature, T2 = 273 + 95 + 24 = 392 K
Using the pressure law:
P1 P
= 2 F
T1 T2
O

4
T R
P2 = P1 × 2 = (152)(392) = 161.9 kPa

CHAPTER
T1 M
368
4

Charles’ Law Charles’ law and the kinetic theory of gases


1 Charles’ law states that the volume of a fixed load
mass of gas is directly proportional to its piston
absolute temperature (kelvin), provided the gas volume
pressure of the gas is kept constant. molecules increases
V ∝T
V = constant
T

V1 V
∴ = 2
T1 T2 (a) (b)

In a fixed mass of gas at constant pressure, the


2 The following graphs show the characteristics
frequency of collisions between the gas molecules
of a gas which obeys Charles’ law.
and the walls of the container is constant.
When the temperature of the gas rises, the
average kinetic energy of the gas molecules
increases. This causes the gas molecules to move
faster and as a result, the gas molecules collide
with the walls of the container more vigorously
O
and at a greater frequency. These two factors
–273 100
increase the rate of change of momentum.
(a) V against θ in Celsius scale The force and correspondingly, the pressure
acting on the walls of the container increases.
Since the volume of the gas is allowed to
increase freely, the gas expands, with its pressure
decreasing, until the internal gas pressure is
equal to the initial externally applied pressure.

Conclusion
O 273 373
The volume of a gas, at constant pressure,
(b) V against T in kelvin scale
increases with temperature.
Figure 4.43

239 Heat
4.3

The gas laws — Charles’ law


Hypothesis Procedure
The volume of a fixed mass of gas at constant 1 The apparatus is set up as shown in Figure 4.44(b).
pressure increases when its temperature is increased. 2 The air sample under investigation is trapped
inside the capillary tube by a bead of concentrated
Aim sulphuric acid which acts as an index.
To investigate the relationship between the temperature, 3 The capillary tube is mounted on a 30-cm ruler
T and the volume, V of a gas at constant pressure (to such that the bottom end of the column of air is
F verify Charles’ law) aligned with the ‘0’ mark on the ruler.
O 4 Water and ice are poured into the beaker until the
4

R Variables
column of air is fully immersed under water. The
CHAPTER

M
(a) Manipulated : temperature of trapped air, θ water is stirred until the water temperature falls to
4 (b) Responding : length of air column, x 0 °C. The length of the air column, x and the
(c) Fixed : atmospheric pressure and mass of temperature, θ are recorded.
trapped air 5 The water is heated and continuously stirred. The
values of x and θ are recorded for each temperature
Notes increment of 10 °C until a temperature of 90 °C is
(a) The length of the air column, x represents the reached.
volume of air trapped inside the capillary tube. 6 A graph of length of air column, x against
(b) The pressure of the trapped air is equal to the sum temperature, θ is drawn.
of the atmospheric pressure and the pressure due
to the weight of concentrated sulphuric acid. Tabulation of data

Apparatus/Materials Table 4.6


Capillary tube, tall beaker, thermometer, Bunsen Temperature, θ (°C) Length of air column,
burner, tripod stand, wire gauze, retort stand, mercury x (cm)
or concentrated sulphuric acid, stirrer, ruler, and ice.
10
Arrangement of apparatus 20
30
glass concentrated 40
capillary sulphuric
tube acid index 50
60
trapped 70
air
80
(a) 90
stirrer
30 cm ruler
thermometer Graph
rubber band
capillary tube (a) A graph of x against θ is shown in Figure 4.45.
ice
length of air column, x (cm)
concentrated sulphuric
acid index
trapped air
Experiment 4.3

wire
gauze Bunsen burner

–273
table
–250 O 100 temperature, θ (°C)
(b)
Figure 4.44 Figure 4.45

Heat 240
(b) If the Celsius scale is replaced by a kelvin Discussion
scale, a straight line graph through the origin
The graph shows that the length of the air column, x
is obtained as shown in Figure 4.46.
(volume of gas) is directly proportional to the
length of air column, x (cm)
absolute temperature, Τ (K).
Conclusion
The volume of a fixed mass of gas at constant
temperature, T (K) pressure is directly proportional to the absolute
O 273 373 temperature (in kelvin) of the gas.
Figure 4.46 The hypothesis is valid.

F
O

4
Absolute temperature scale and absolute zero • Absolute temperatures are stated in units of kelvin. R

CHAPTER
M
• In accordance with Charles’ law, the graph of volume • The absolute zero is the zero point on the kelvin
against temperature of a gas is linear. If the graph is temperature scale (absolute scale). It is equivalent to 4
extrapolated, the graph will intersect the temperature –273 °C.
axis at –273 °C as shown in Figure 4.43(a). ∴ 0 K = –273 °C
• The temperature of the gas at –273 °C is the lowest t
• The relationship between the absolute scale and the
temperature theoretically achievable. This temperature
Celsius scale is: T = (θ + 273) K, where T is the
of –273 °C is called absolute zero measured on the
temperature on the kelvin scale and θ is the
absolute temperature scale.
temperature on the Celsius scale.
• The absolute temperature scale is also known as the
kelvin temperature scale which is used in studies on – 273 °C – 50 °C 0 °C 50 °C 100 °C
gas laws.
0K 223 K 273 K 323 K 373 K

20
A balloon is filled with 150 cm3 of gas at 35 °C.
The balloon is immersed in a beaker of water and the water is heated so that the
pressure in the balloon remains constant. What is the volume of the balloon
when its temperature reaches 50 °C?
Solution

150 cm3

50 °C
35 °C

Initial volume, V1 = 150 cm3


Initial temperature, Tl = 273 + 35 = 308 K
Final volume, V2 = ? Reminder:
Final temperature, T2 = 273 + 50 = 323 K Do not forget to change the
V V temperature of the gas from
Applying Charles’ law: 1 = 2 the Celsius scale to the
T1 T2
absolute (kelvin) scale for
VT 150 × 323 your calculations.
V2 = 1 2 = = 157.3 cm3
T1 308

241 Heat
Applications of Gas Laws
21
A constant pressure of 100 kPa is applied to a volume Bicycle Pump
of air in a cylinder. The initial height of the piston is
1 When the piston of the pump is pushed into
20 cm from the base of the cylinder.
the cylinder, the air inside is compressed.
2 According to Boyle’s law, the pressure of the
air in the cylinder increases.
3 The pressure of the air in the cylinder is greater
T2
than the air pressure in the tyre. The valve of
h1 T1 h2
the tyre therefore opens to allow air to flow
into the tyre.
F temperature temperature 4 When the piston is retracted, the pressure in
O 300 K 400 K
the cylinder decreases. The valve closes and the
4

R
Figure 4.47 outside air at the higher atmospheric pressure
CHAPTER

M
flows into the cylinder.
4
What is the final height of the piston if the
piston
temperature of this system is increased from an initial pushed in piston
retracted
temperature of 300 K to a final temperature of bicycle
400 K? pump air
high low pressure
moves in
Solution pressure region
region
V1 V2
Applying Charles’ law: =
T1 T2 valve
opens tyre
valve high
VT closes pressure
V2 = 1 2 tyre
T1
Figure 4.48 Application of Boyle’s law to the operation of a
T h represents the
bicycle pump
h2 = 2 × h1 volume V of the
T1 trapped air.
400 Tyre
= × 20
300 1 When a car is moving, the tyres of the car are
= 26.7 cm subjected to frictional force and continuous
compression. This causes the temperature of
the air in the tyres to increase.
2 According to the pressure law, the air pressure
in the tyre increases when its temperature
increases. The pressure of the air in the tyre
increases until the optimum pressure with a
The gas laws and important formulae:
value between 220 kPa and 280 kPa is reached.
Boyle’s law P Pressure law
Hot-air Balloon
P1V1 = P2V2 P1 P2
= 1 When the air in a hot-air gas balloon is heated
T1 T2
at atmospheric pressure, its temperature
increases.
V 2 According to Charles’ law, the volume of a gas
T
increases when its temperature is increased at
constant pressure.
Charles’ law 3 When the volume of the air displaced by the
balloon is increased, the buoyant force on the
V1 V2 balloon increases—Archimedes’ principle.
=
T1 T2 4 The balloon rises when the buoyant force on the
balloon is greater than the weight of the balloon.

Heat 242
3 SPM
Clone
5 SPM
Clone
’06 ’07

The air pressure in a tyre is 205 kPa when the A plastic bottle which containing hot air initially is
temperature is 26 °C. What would the pressure be if put into a basin of ice as shown in the figure below.
the temperature increases to 36 °C?
(Assume the volume of the air in the tyre is constant.)
A 198 kPa C 236 kPa
B 212 kPa D 287 kPa
ice
Solution hot air
P1 P
Applying the pressure law: = 2
T1 T2
F
Study the figure carefully. The situation can be O
P1 = 205 kPa

4
R
explained by
T1 = 26 °C T2 = 36 °C

CHAPTER
M
A Boyle’s law
= (26 + 273) K = (36 + 273) K
B Pressure law 4
= 299 K = 309 K
C Charles’ law
205 P
∴ = 2 Comments
299 309
205 The volume of the plastic bottle is reduced when its
P2 = × 309 temperature drops, which is in accordance with
299
Charles’ law.
= 212 kPa
Answer C
Answer B

4 SPM
Clone
4.4
’07
1 (a) State Boyle’s law and Charles’ law.
Which of the following P – T graphs correctly shows (b) An air bubble of volume 12 cm3 at a pressure of
the relationship between pressure, P and temperature, 350 cm Hg is released from a diver near the
T, of a gas at constant volume? bottom of the sea. Determine the volume of the
A P (N m–2) CP bubble when it reaches the surface of the water.
P (N m )
–2

[Atmospheric pressure = 76 cm Hg]


2 Diagram (a) shows a column of air trapped in a
narrow capillary tube held vertically. The atmospheric
pressure at room temperature is P cm Hg.
T (K) T (K) 2

B P (N m–2) D mercury
P (N m–2)

air air mercury


1 (b)
capillary
tube
T (K)
T T
T (K)
(a)
Comments
Pressure law states that the pressure of a gas, P, at (a) State the pressure of the air trapped in the
constant volume is proportional to its absolute capillary tube, in terms of P cm Hg as shown in
diagram (a).
temperature, T (i.e. in Kelvin scale).
(b) What will be observed if the capillary tube is held
Answer B in a horizontal position as shown in diagram (b)?

243 Heat
(c) If 1 = 10 cm, 2 = 11 cm and P = 76 cm Hg, temperature. The glass tube G contains dry air
what is the pressure of the trapped air when the trapped by a thread of mercury. Take note that there
tube is held vertically? is a small gap in the electrical circuit at C.
(d) State two assumptions you made in the (a) Briefly explain what happens when the
calculations in (c). temperature in the tank rises until it exceeds the
3 mercury thread battery set temperature.
(b) What will happen to the position of the mercury
C
G alarm thread when the atmospheric pressure
increases?
cylindrical (c) What is the effect of an increase in atmospheric
tank
C pressure, as stated in (b), on the temperature at
which the audible alarm will be activated?
The figure shows an instrument designed to operate (d) If the volume of air in the tube is 5 cm3 at 27 °C
F and the atmospheric pressure is constant, what
O
on the principle of the expansion of a gas. Its function
will the temperature be when the volume of the
4

R is to give an audible alarm when the temperature


inside a cylindrical tank exceeds a set (fixed) trapped air reaches 7 cm3?
CHAPTER

1. A body is said to be in thermal equilibrium if there is 9. Latent heat, L, is the quantity of heat absorbed or
no net heat transfer within it, or between it and its released when a substance changes its physical phase
surroundings. at constant temperature.
2. Thermometer is an instrument used to measure the 10. Specific latent heat, , is the heat absorbed or
temperature of a substance, by making use of the released per unit mass of a substance in the course
physical property of a substance that varies with of its change of phase at constant temperature.
temperature (thermometric property). The SI unit of specific latent heat is J kg–1.
3. A liquid-in-glass thermometer make use of the
expansion of a liquid to measure temperature. Q
Specific latent heat,  =
4. Calibration of a thermometer is based on the melting m
point of pure ice at 0 °C as lower fixed point and the
boiling point of pure water at 100 °C as the upper in which Q = Heat absorbed or released
fixed point. m = Mass of substance
5. Heat capacity, C, is the quantity of heat required to 11. Specific latent heat of fusion is the quantity of heat
raise the temperature of an object by 1 °C. It is absorbed to change 1 kg of a substance from solid to
meausred in joules per degree celsius, J °C–1 or joules liquid without change in temperature at its melting
per kelvin, J K–1. point.
6. Specific heat capacity, c, of a substance is the 12. Specific latent heat of vaporisation is the quantity of
quantity of heat required to raise the temperature of heat absorbed to change 1 kg of a substance from the
1 kg of the substance by 1 °C. In SI units, it is liquid to gas without change in temperature at its
measured in J kg–1 K–1 or J kg–1 °C–1. boiling point.
7. The formula that relates quantity of heat, Q to specific 13. Boyle’s law states that the pressure, P, of a given
heat capacity, c is: mass of gas is inversely proportional to the volume, V,
at constant temperature.
Q Q = mcθ
c= or P1V1 = P2V2

14. Charles’ law states that the volume, V, of a given


in which m is the mass of a substance and θ is the mass of gas is directly proportional to its absolute
change in temperature of a substance. temperature, T, provided the pressure of the gas is
8. Examples of the applications of specific heat capacity: kept constant.
• Water as coolant in a car engine
• Water as heating agent in keeping house warm V1 V
• Household utensils = 2
T1 T2
• Formation of sea breeze and land breeze

Heat 244
15. Pressure law states that the pressure, P, of a given 16. The temperature of the gas at –273 °C is known as
mass of gas is directly proportional to its absolute absolute zero from which the absolute temperature
temperature, T, provided the volume of the gas is kept scale starts. Absolute temperature scale also known as
constant. kelvin scale. The SI unit of temperature is kelvin, K.
17. The relationship between the kelvin scale and celsius
P1 P scale is T = (θ + 273) K.
= 2 18. Examples of the applications of gas laws include
T1 T2
bicycle pump, tyre and hot-air balloon.

4 F
O

4
R

CHAPTER
M
Multiple-choice Questions
4

4.1 light calorimeter containing a 5 Which of the following liquids A, B,


Thermal Equilibrium liquid at 30 °C. Clone C or D, can be used in a liquid-in-
SPM

’04 glass thermometer to measure


1 A metal sphere S at 100 °C is
SPM
dropped into a liquid L at 25 °C. temperatures ranging from –30 °C
Clone
’03 Thermal equilibrium is reached to 80 °C?
when metal block (80°C)
Freezing Boiling
A material of S = material of L liquid (30°C) Liquid
point (°C) point (°C)
B temperature of S Diagram 2
= temperature of L A –55 98
C specific heat capacity of S What is the temperature of the B –19 254
= specific heat capacity of L metal block after it has reached
thermal equilibrium with the liquid? C 0 100
D specific latent heat of S
A Less than 30 °C. D 37 118
= specific latent heat of L
B Higher than 80 °C.
2 Diagram 1 shows two aluminium C Between 30 °C and 80 °C. 6 An ungraduated mercury-in-glass
SPM
blocks, P and Q, in physical contact.
Clone D Same as room temperature. thermometer is placed beside a
’05
ruler. At 0 °C, the length of the
4 Diagram 3 shows cold water being
mercury column is found to be
SPM
added to hot water to obtain a
P
Clone
’07
15.0 cm. At 100 °C, the length of
Q 1.1 beaker of water which is lukewarm.
Significant Figure the mercury column becomes
cold 27.5 cm. When the thermometer
water is in contact with a liquid, the
Diagram 1
length of the mercury column is
The initial temperatures of blocks 22.0 cm. What is the temperature
P and Q are 90 °C and 40 °C hot lukewarm
water water of the liquid?
respectively. Which statement is A 31 °C C 70 °C
correct when they are in thermal Diagram 3 B 56 °C D 76 °C
equilibrium?
Assume there are no heat losses 7 Diagram 4 shows a mercury-in-
A Temperature of P is higher than
to the surroundings, which of the glass thermometer. The distance
that of Q.
following statements is correct? between the –10 °C and 110 °C
B There is no net heat flow
A Heat gained by the cold water markings is 30 cm.
between P and Q.
is more than heat lost by the
C The rate of temperature 18 cm
hot water.
change in P is faster than that
B Heat gained by the cold water –10°C 110°C
in Q. 30cm
3
0 cm
is less than heat lost by the hot
D The amount of heat energy in Diagram 4
water.
P and that in Q are the same.
C Heat gained by the cold water At which temperature is the end of
3 Diagram 2 shows a metal block at is equal to heat lost by the hot the mercury thread 18 cm from
SPM
temperature of 80 °C placed in a
Clone water. the –10 °C mark?
’06

245 Heat
A 50 °C C 62 °C 12 100 kg of a substance cools from becomes heat energy when the
B 60 °C D 72 °C 60 °C to 20 °C. If its specific heat lump strikes the ground, what is
capacity is 4.0 J kg–1 °C–1, how the increase in temperature of the
8 When taking the body
much heat is lost by the lead? [g = 10 m s–1]
SPM temperature of a patient, the
Clone
’07 clinical thermometer is placed
substance? A 0.4 °C
A 400 J C 4000 J B 1.0 °C
under her tongue for some time.
B 700 J D 16 000 J C 4.0 °C
What is the concept employed?
D 10.0 °C
A Specific heat capacity 13 400 g of a particular liquid at
B Specific latent heat 80 °C is mixed with m g of the 18 Water is a suitable cooling agent
C Thermal equilibrium same liquid at 30 °C. If the SPM
Clone used in the cooling system of a car
D Thermal radiation maximum temperature which can ’06 engine. This is because water
be achieved by the mixture is SPM
Clone A evaporates rapidly
50 °C, what is the value of m? ’08 B is rust-resistant
F 4.2
O
Specific Heat Capacity A 240 C 640 C has a high density
4

R 9 The specific heat capacity is defined B 600 D 1000 D can absorb a lot of heat
CHAPTER

M as the quantity of heat required to 14 A stone of 0.3 kg at 6 °C is placed


A change the temperature of in a beaker of 0.6 kg water at
4 4.3
1 kg of a material. 28 °C. The equilibrium Specific Latent Heat
B change the temperature of a temperature is 26 °C. What is the 19 Which of the following occurs
material by 1 °C. average specific heat capacity of when a naphthalene ball
C change the temperature of the stone, in J kg–1 °C–1? (mothball) melts when heated?
1 kg of a material by 1 °C. [Specific heat capacity of water
F at its A Temperature increases
D melt 1 kg of a material = 4200 J kg–1 °C–1]
O B Temperature decreases
melting point. A 210 C 840
4

R C Heat is released
10 Diagram 5 shows two M B 420 D 2800 D Heat is absorbed
CHAPTER

solid
SPM spheres X and Y of equal mass, 15 A 2 kW heater is used to heat
Clone
’04 which are being heated4 in a water Questions 20 and 21 are based on
100 kg of water. What is the Diagram 6
bath. The temperatures of the two maximum possible temperature
spheres are the same initially. rise which can occur in 5 minutes? 20 Diagram 6 is a graph showing the
[Specific heat capacity of water SPM
Clone heating curve of a substance.
= 4200 J kg–1 °C–1] ’03
1.1 Signifi cant Figure Temperature (°C)
X Y 2000  300
A Y
4200  100 °C
Z
W
stove 2000  100 X
B
4200  5 °C V 1.1 Sign
Diagram 5 4200  100
C
X is observed to become hot 2000  300 °C Time (s)

slower than Y. This observation is 4200  300 Diagram 6


D
due to the difference in 100  2000 °C SPM At which stage is the substance in
Clone
A density ’05 the solid and liquid state the same
B melting point 16 A 240 V, 300 W electrical heater is time?
C specific heat capacity placed in an insulated metal block A VW C XY
D specific latent heat M of mass 3.0 kg. After 1 minute B WX D YZ
32 seconds of heating, the
11 Heat energy is supplied at the 21 At which stages are specific latent
thermometer records a
same rate to 50 g of oil and to heat absorbed?
temperature change from 30 °C to SPM
Clone
50 g of water in similar containers. ’06 A VW and WX
50 °C. What is the specific heat
Which of the following explains B XY and YZ
capacity of metal M, in J kg–1 °C–1? SPM
Clone
why the temperature of the oil ’08 C VW and XY
A 368 C 613
rises more quickly? D WX and YZ
B 460 D 1104
A Oil is less dense than water.
B Oil is more dense than water. 17 A lump of lead of specific heat 22 Some molten wax in a test-tube
C Oil has a smaller specific heat capacity 130 J kg–1 °C–1 is dropped was allowed to cool and the
capacity than water. from the top of a tall building of temperature was observed at half-
D Oil has a larger specific heat height 52 m. Assuming that all the minute intervals. Diagram 7 is a
capacity than water. initial potential energy of the lead graph showing the results.

Heat 246
Temperature (°C) A control the melting rate of ice. cylinder without any friction. No
B determine the mass of ice gas escapes.
melted by heat from the
surroundings.
C verify that the ice used in the
X
Time
(minute)
experiment does not contain
2h
impurity. gas
Diagram 7 D calculate the average value for h (30°C)

The temperature stayed the same the latent heat of fusion of the
during the period X because the ice. (a) (b)
A melting point of the wax was Diagram 10
equal to the temperature of
the surroundings. The initial height of the piston is
B solid wax had a lower specific 4.4 h cm. As the cylinder is heated, F
The Gas Laws
heat capacity than the molten. the piston moves upwards. The O

4
C molten wax gave out latent 26 An air bubble is released by a new height of the piston is 2h cm, R

CHAPTER
heat when it changed phase. Clone diver, as shown in Diagram 9.
SPM
as shown in Diagram 10(b). The M
’03 initial temperature of the gas is
D molten wax absorbed latent
2m 30 °C. If the pressure remains 4
heat from the surroundings
8m air bubble constant, what is its final
when it changed phase. (6 cm3)
temperature?
23 How much heat is required to A 15 °C C 333 °C
change 100 g of ice at 0 °C into B 60 °C D 606 °C
water at 20 °C? Diagram 9
[The specific heat of water is 29 Which of the following graphs
If the initial volume of the air shows the relationship between
4200 J kg–1 °C–1 and the latent heat bubble is 6 cm3 when it is 8 m
of fusion of ice is 3.34  105 J kg–1.]
SPM
Clone volume, V, and absolute
below the water surface, what is its ’07 temperature, T, of a gas at
A 8.4 kJ volume when it is 2 m below the
B 41.8 kJ constant pressure?
water surface? Assume the A C
C 8400 kJ atmospheric pressure is equivalent V (cm3) V (cm3)
D 4.18  104 kJ to a height of 10 m of water.
24 Heat must be removed from A 4 cm3 C 10 cm3
1.1 SignificantTFigure
200 g of water at 25 °C to change B 9 cm3 D 18 cm3 O (K) O T (K)
it into ice at 0 °C. If the process 27 A car tyre was tested before being B D
takes 20 minutes, what is the rate SPM driven on a motorway. The V (cm3) V (cm3)
of heat transfer? Clone
’04 pressure of the tyre was 220 kPa
[The specific heat of water is and the temperature was 7 °C.
nificant Figure
4200 J kg–1 °C–1 and the latent heat At the end of the journey, the O T (K) O T (K)
of fusion of ice is 3.34  105 J kg–1.] pressure was found to be
A 73 J s–1 242 kPa. If the volume of the air in 30 Diagram 11 shows a cylinder which
B 1.5  103 J s–1 the tyre remained constant, which contains a gas. The piston is held
C 4.4  103 J s–1 expression gives the temperature Clone fixed and the cylinder is heated.
SPM

D 7.3  103 J s–1 of the air in the tyre at the end of ’08

25 Diagram 8 shows the the journey?


cylinder
arrangement of the apparatus for
an experiment to determine the
A 
242  7
220
°C gas

latent heat of fusion of ice.


to power
B  220242 7  °C heat
Diagram 11
supply
C  242220 280
 – 273 °C Which of the following statements
is correct?
A The gas molecules move faster.
control experiment
D  220242 280
 – 273 °C B The gas molecules are further
apart.
Diagram 8
28 Diagram 10(a) shows a cylinder C The number of gas molecules
A set of the apparatus is used as which contains gas at the same increases.
the control. The control pressure as the air outside. The D The pressure of the gas
experiment is required to piston moves freely along the remains unchanged.

247 Heat
Structured Questions
1 (a) In what state is the substance in the regions
SPM
Clone
(i) PQ? [1 mark]
’10 Polymer handle water
(ii) QR? [1 mark]

copper pot (iii) RS? [1 mark]

hot plate (b) Using kinetic theory of matter, explain why there is
no increase in temperature in the region QR even
Diagram 1
though the substance is heated. [2 marks]

(c) What is the melting point of the pure substance?


Diagram 1 shows a copper pot is used to boil water
F [1 mark]
O
with a hot plate.
4

R (a) What is the meaning of specific heat capacity? (d) From the information given,
(i) calculate the power of the heater, in W,
CHAPTER

M [1 mark]
[2 marks]
4 (b) Give reason for the suitability of the characteristics
of the pot to be used for heating up the water (ii) calculate the specific latent heat of fusion, ,
rapidly. Consider the following in your explanation. of the substance. [2 marks]
(i) material for making the pot
(ii) material for making the handle of the pot
3 In Diagram 4, dry air is trapped in the horizontal
(iii) mass of the pot
F capillary tube by a column of mercury which occupies
O
(c) The copper pot is Rheated by an electric hot plate a 25 cm length of the tube. At 290 K, h, the length of
4

of power 1kW. Calculate the time taken to achieve the air column, is 20 cm. The other end of the tube is
CHAPTER

M
a temperature rise of 50°C open to the atmosphere, the atmospheric pressure
mass of pot = 2.0 kg
4 being 75 cm Hg.
mass of water = 2.5 kg
Specific heat capacity of water = 4200 J kg–1°C–1 25 cm 20 cm
Specific heat capacity of copper = 390 J kg–1°C–1
open
end

2 Diagram 3 shows a graph of temperature plotted mercury trapped air


against time for 0.8 kg of a pure substance, which is
Diagram 4
being heated at a constant rate over the range from
–20 °C to 60 °C. The specific heat capacity of the
substance is known to be 1000 J kg–1 °C–1 when it is The capillary tube has a constant cross-sectional area,
in the liquid state. A.
(a) The open end is now raised until the tube is
Temperature (°C) vertical.
60 S (i) What is now the pressure on the trapped
50
air? [1 mark]
Q R
40 (ii) Calculate the length of the capillary tube
30 occupied by the trapped air now. [2 marks]
20
(b) With the tube still vertical, the temperature is
10
increased to 348 K. Calculate the length of
0 the tube occupied by the trapped air now.
–10 [2 marks]
P
–20 Time (s)
0 200 400 600 800 1000 1200 (c) With the tube returned to the horizontal position
Diagram 3 at 348 K, calculate the length of the tube
occupied by the trapped air now. [2 marks]

Assume no heat is lost from the apparatus and the (d) State one assumption you made in the above
heat capacity of the vessel is negligible. calculations. [1 mark]

Heat 248
Essay Questions
4 Diagrams 5(a) and (b) show the melting and freezing heater thermostat upper radiator coolant
core hose recovery
of naphthalene when heat is transferred. system
bypass
thermometer thermometer hose
80 80 heater
hoses
70 70

cooling
jackets
naphthalene naphthalene
water conical flask radiator
water pump
Bunsen F
burner (b)
O
Diagram 7

4
(a) (b) R

CHAPTER
Table 1 shows the characteristics of liquids which M
Diagram 5 can be used in the cooling system of a car.
4
The melting point of naphthalene is 80 °C.
Characteristic
(a) What is meant by melting point?
[1 mark] Specific heat Freezing Boiling Rusting
(b) (i) Using Diagrams 5(a) and (b), compare the Liquid capacity point point rate on
processes which take place in both (J kg–1 °C–1) (°C) (°C) metal
situations and the heat transfer involved in J 5000 20 110 High
the processes. Relate the processes, the K 4600 –15 120 Low
heat transfer, the change in temperature to
deduce a relevant physics concept. L 3800 15 95 Medium
[5 marks] M 3000 5 95 Medium
(ii) Name the type of heat involved in both N 200 –20 320 Low
situations. [1 mark]
(c) A test-tube contains 0.3 kg of liquid naphthalene Table 1
at 100 °C. If the average rate of heat loss is
(i) What is meant by specific heat capacity?
120 J min–1, calculate the time required for it to
[1 mark]
cool down to 80 °C before it solidifies. The specific
(ii) You are asked to investigate the
heat capacity of naphthalene is 1700 J kg–1 °C–1 in
characteristics of the liquids in Table 1 which
the liquid state. [3 marks]
could be used in the cooling system of a car.
(d) Ismail is a mobile fishmonger. He sells fishes and
Explain the suitability of each characteristic in
other seafood by driving them in his van to
Table 1 and hence, determine which liquid
different residential areas. Diagram 6 shows the
is most suitable to be used in the cooling
container he uses to keep the seafood.
system. Justify your choice.
[10 marks]
(b) Table 2 gives details about the cooling system of a
car engine.
Cooling system
Total energy generated by the engine in 1 hour
8.1  107 J
Breakdown of energy generated
wooden box • 40% mechanical energy
Diagram 6 • 60% heat
Mass flow rate of cooling liquid circulating
Using the appropriate physical concepts, suggest
and explain suitable ways and modifications to 200 kg in 1 hour
keep the seafood fresh for a longer period of time. Temperature of the cooling liquid
[10 marks] • 36 °C when it enters the engine
5 (a) Heat generated in the car engine has to be • 91 °C when it leaves the engine
removed effectively to avoid overheating. Diagram
7 shows the cooling system of a car engine. Table 2

249 Heat
Using the data in Table 2, determine (iii) the specific heat capacity of the cooling
(i) the output power of the engine, liquid, assuming that all the heat energy is
[2 marks] removed by the cooling liquid. [3 marks]
(ii) the amount of heat generated in one hour, (c) Explain how the heat is removed from the engine
[1 mark] by the cooling system. [3 marks]

Experiments
1 A student carries out an experiment to investigate the (d) Determine the mass, m, of the heated liquid if the
relationship between the temperature, θ, of a liquid final temperature, θ, is 29 °C. Show on the graph
when heated and its mass, m. The same immersion how you determine the value of m. [3 marks]
heater is used to heat up different masses of the liquid (e) State one precaution that should be taken during
F
for a fixed amount of time. The final temperature, θ, of this experiment. [1 mark]
O
the liquid after being heated is recorded.
4

R 2 Jeremy was playing a game of ping-pong (table


The results of the experiment are shown in a graph of
CHAPTER

M tennis) with his friend and they are left with only one
1
θ against shown in Diagram 1. ping-pong ball. After playing for a while, they found
4 m
that the ping-pong ball had a slight dent in it.
1 Diagram 2 shows the ping-pong ball with a slight dent.
Graph θ of against m
θ (°C) The ping-pong ball was being restored by Jeremy
31 pouring some hot water on it as shown in Diagram 3.

F
30 O
4

R
CHAPTER

29 M hot
y = 6x + 26 water
4
28

27
Diagram 2 Diagram 3
26 The ping-pong ball is airtight, so when it is heated, the
air expands and the plastic softens just enough to
1 –1 allow the dent to be blown out.
25
0 0.1 0.2 0.3 0.4 0.5 0.6 0.7 0.8 m (kg )
Using the information on Jeremy’s observation;
(a) State one suitable inference. [1 mark]
Diagram 1 (b) State one appropriate hypothesis that could be
investigated. [1 mark]
(a) The initial temperature, θ0, of the liquid can be
(c) Design an experiment to investigate the
determined from the intercept of the θ-axis.
hypothesis stated in (b).
Show on the graph how you determine θ0 and
With the use of apparatus such as thermometer,
write the value below.
capillary tube, beaker, rule and other apparatus,
θ0 = _________________
describe one experiment to investigate the
(b) State the relationship between θ and m.
hypothesis stated in (b).
(c) The specific heat capacity, c, of the liquid is given
In your description, clearly state the following:
by the equation:
(i) The aim of the experiment
1.8 × 104
c= (ii) The variables in the experiment
k
(iii) The list of apparatus and materials
where k is the gradient of the graph. (iv) The arrangement of the apparatus
(i) Calculate the gradient of the graph of θ (v) The procedure used in the experiment.
1 Describe how to control the manipulated
against .
m variable and how to measure the
Show on the graph how you determine the responding variable.
gradient. [3 marks] (vi) The way to tabulate the data
(ii) Calculate the value of the specific heat (vii) The way to analyse the data
capacity, c, of the liquid. [2 marks] [10 marks]

Heat 250 COMPANION WEBSITE


Online Tests
FORM 4

5
CHAPTER

Light

SPM Topical Analysis


Year 2007 2008 2009 2010 2011
Paper 1 2 3 1 2 3 1 2 3 1 2 3 1 2 3
Section A B C A B A B C A B A B C A B A B C A B A B C A B
Number of questions 5 1 – – – 1 5 1 – 1 – – 5 1 – – – – 4 1 – – – 1 5 1 – – – 1

ONCEPT MAP

Light

Reflection Lenses

Refraction Total internal


Laws of reflection reflection
Convex Concave
Laws of refraction
Image formed in (Snell’s law) Critical
• plane mirror angle, c • Focal length, f
• convex mirror • Focal point, F
• concave mirror • Object distance, u
Refractive index, n
sin i Natural • Image distance, v
= phenomena &
sin r
Applications Real depth applications of
of mirrors = total internal Power of lens
Apparent depth
reflection
=
1 P= 1
sin c f

Magnification
Phenomena due
to refraction m= v
u

Lens formula
Ray diagrams 1 = 1 + 1
f u v

Characteristic
of images The uses of lenses
in optical devices

COMPANION WEBSITE
Learning Objectives 251
5.1 Understanding Reflection of Light

Reflection of Light

1 A plane mirror is a flat smooth surface which reflects most of the light falling on it.
2 A well polished copper surface and water surface are also mirrors.

The line drawn at a right angle to the mirrror


surface where the incident ray strikes is called
the normal.

The reflected ray is the ray


F reflected by the plane mirror.
O The incident ray is the ray
5

R of light directed onto the


plane mirror.
CHAPTER

M
the vertical plane on which the

normal
incident ray, normal and
4

ray
reflected ray lie
inc

ted
ide

lec
nt

ref
ra

r The angle of reflection, r is the


y

i
angle between the reflected ray
and the normal.
O
The angle of incidence, i is plane mirror
the angle between the incident
ray and the normal.

Figure 5.1 Plane mirror

3 How the reflection of light works is described under the laws of reflection.
4 These laws are applicable to all types of reflecting surfaces at the points of incidence.

Laws of reflection

a b
The incident ray, the reflected ray, and The angle of incidence is equal to
the normal all lie in the same plane. the angle of reflection, i.e., i = r.

1
A ray of light is directed onto a plane mirror as shown The path of a ray of light deviates from the original
in Figure 5.2. Calculate the angle θ. direction of the ray when the ray is reflected. What is
the angle of deviation?
angle of
deviation Solution
θ = 90° – 55° Angle of reflection
th
pa

θ = Angle of incidence
= 35°
al

= 55°
gin
ori

normal ∴ Angle of deviation


°55 = 35° + 35°
Note: = 70°
Angle of deviation
= 2θ

Figure 5.2

Light 252
2 If a screen is placed behind the plane mirror,
Characteristics of an Image Formed in a
no image is projected on the screen.
Plane Mirror SPM SPM SPM
’05/P2(A) ’06/P1 ’09/P1 3 An image, which cannot be formed on a
Figure 5.3 shows how a plane mirror forms an screen, is called a virtual image.
image of a light bulb. 4 A virtual image is produced at the place where
the reflected rays appear to intersect.
plane
object mirror image SPM
Lateral Inversion of an Image ’03/P1
d d
1 In Figure 5.4, the woman is brushing her hair
3 with a brush in her right hand in front of a
plane mirror but she appears to be using her
P P F
a left hand if you look at her image in the mirror.
O

5
b R

CHAPTER
virtual rays M
1 2 (broken lines)
a 4

observer
Figure 5.4
Figure 5.3 2 This phenomenon is called lateral inversion
(a left-right reversal).
1
3 However, the image is still upright (no top-
Rays of light a and b from point P of the bottom vertical reversal).
object are reflected by the plane mirror before 4 Figure 5.5 shows how the letter ‘F’ is reversed
entering the observer’s eyes. in the virtual image.


card with plane


letter ‘F’ mirror
2 image
The reflected rays, when extended backwards 1
behind the mirror appear to intersect at P′ which 2
forms the corresponding image of P. This 3
4
happens to all the points of the object and a 1
3
complete virtual image is then formed. 2
4


observer
3
Figure 5.5 Lateral inversion of a card with the
Characteristic of image in a plane mirror:
letter ‘F’
(a) The image is virtual.
(b) The image is upright but laterally inverted. 5 Besides the orientation, the order of letters is
(c) The size of the image is the same as the also reversed in a virtual image. Figure 5.6
object. shows a fire engine with the word ‘ ABMOB ’.
(d) The distance between image and mirror is the The letters are placed in
same as distance between object and mirror. this way so that drivers
in front can see the
letters in the right order,
i.e., ‘BOMBA’ and the
Virtual Image correct orientation in
their rear-view mirrors.
1 From Figure 5.3, it can be seen that the image
of the light bulb appears to be behind the
mirror. Figure 5.6 Bomba

253 Light
1
O Draw a perpendicular line from O to the mirror
through N. Mark IN = ON to locate the image, I.

3
O I Draw broken
4 lines behind the
plane mirror Draw the rays, d N
mirror to
c and d from c
represent virtual
the object, O to rays.
How to draw two rays of light from an object, O to the the mirror.
observer’s eye in the above figure to show the plane mirror
formation of the image, I. b
a
Note:
F • Drawing the rays from the object, O first would result
O 2
in an inaccurate diagram. Draw the rays, a and b from
5

R the image, I to the eye.


• If the position of the image, I is given, start from
CHAPTER

M
step 2 .
4

The figure shows Nicolas who is 1.8 m tall standing in


A
front of a mirror which is as high as he is. He can see his
P H
full length image in the mirror. 12 cm
M
What is the shortest length of mirror that will still show his E
full length, if we know that his eyes are 12 cm from the
top of his head? image
168 cm
N

mirror
Q B C

Solution
The problem can be solved if a diagram is drawn.
Assuming MN is the shortest length of mirror used, AM and NB can be cut off.
P A

12 cm θ3
X θ4 M θ3 = θ4
E 1
∴ AM = PX = PE
2
= 6 cm Note:
X = midpoint of PE
168 cm θ1
Y N mirror Y = midpoint of EQ
θ2 θ1 = θ2
1
∴ NB = YQ = EQ
2
= 84 cm
Q B

MN = 180 – (6 + 84) = 90 cm
Conclusion
1 The shortest length of the mirror can be half of the height of a person.
2 The bottom edge of the mirror must be placed at a position half way between the eyes and the feet.

Light 254
2 SPM
’04/P1
SPM
’07/P1 SPM
’04/P1

A dancer stands at a distance, d, in front of a plane


The figure (a) shows a ray of light reflected by a
mirror. What is the distance ‘travelled’ by the image if plane mirror. What is the angle of deviation of the
(a) the dancer moves 2 m towards the mirror? reflected ray in figure (b), if the mirror is rotated 10°
(b) the plane mirror is moved 2 m towards the clockwise about point O?
N‘
dancer? N Q
P P Q
Solution Q‘
This question is best solved by means of diagrams. 30° 30° 40° 40°
(a)
d d 10°
O O F
(a) (b) O

5
R
2m 2m

CHAPTER
M
Students give the answer as 10°, i.e. the angle of
rotation of the mirror. 4

Distance moved by image = 2 m Before rotation:


Angle of incidence = Angle of reflection
(b) d d = 30°
∴ ∠POQ = 60°
After rotation of 10°:
Angle of incidence = 40° = 2 × angle of
rotation of
∴ Angle of reflection = 40° the mirror
Now, ∠POQ′ = 80°
Now, ∠QOQ′ = 80° – 60°
2m
= 20°

3
d–2 d–2 x
Rahimah and Johan are standing at a distance of 2 m
(d – 2) + (d – 2) + x = 2d from a big plane mirror. Rahimah is at a distance of
∴ Distance moved by image, x = 2d – 2d + 4 3 m from Johan. What is the distance between Johan
= 4m and Rahimah’s image?
Solution
Rahimah’s image

1 In general:
2m
Stationary Observer Image also a
mirror moves by a m moves by a m
Stationary Mirror moves Image moves
observer by a m by 2a m
2m

2 By the same reasoning


(a) if the dancer runs at a speed of 2 m s–1 Applying Pythagoras’
towards the mirror, her image is also theorem: 3m
Johan
Rahimah
running at 2 m s–1, a = 3 + 4 = 5 m
2 2
(b) if the mirror is moved towards the dancer ∴ The distance between Johan and Rahimah’s
at a speed of 2 m s–1, her image is moving image is 5 m.
at 4 m s–1.

255 Light
Uses of Apparatus and Instruments Based on the Principle of Reflection of Light

Mirror periscope
• A mirror periscope is used to view objects in an elevated position from behind an obstruction.
• It can be built by mounting two plane mirrors in a cardboard tube as shown in Figure 5.7.
• The two mirrors are set parallel and facing each other so that the angle of incidence at each mirror is 45°.

moon
45
The rays of light are
The rays of light 45 rotated 90° by each
from the object are 45
mirror.
FF

al
reflected by the

rm
OO

no
cardboard tube
upper mirror. obstruction
5

RR
MM
CHAPTER

al
rm
no
44 observer
image
The reflected rays
are subsequently
45
reflected by the
The resultant image is lower mirror.
virtual and upright. Figure 5.7 Mirror periscope

• The periscopes used in submarines have glass prisms instead of mirrors but they operate on the
same principle.

Kaleidoscope

• A kaleidoscope is a simple optical device with coloured glass chips that form designs and patterns.
• It consists of two parts:
(a) A viewing tube with an eyepiece at one end.
(b) An object box at the opposite end of the tube. This box has coloured chips of glass sandwiched
between two glass discs. The outer disc allows light to enter.

object box Look through the • The tumbling of coloured chips


eyepiece at this end. presents the viewer with
varying beautiful symmetrical
pattern because of the
vie reflections in the mirrors.
win
g tu
be • If the angle between the two
mirrors is 60°, then on looking
down the tube, five images are
Light enters through this end, reflecting off the formed.
mirrors. • The object and five images
axis of the tube form a symmetrical pattern of 6
coloured chips act sectors.
as objects

plane mirrors within


a glass disc in the
subject box

Figure 5.8

Light 256
Reflection of Light on Curved Mirrors
Optical testing
• A plane mirror is used in optical testing because
the objects can be laterally inverted.
laterally inverted
illuminated letters

(a)
reflecting
mirror letters are in correct surface
mounted orientation and order
on the and appear at a distance
wall two times the distance FF
between the man and OO
the mirror pole, P

5
C RR
M

CHAPTER
Figure 5.9 M
object
• The images of the illuminated letters can be read 44
correctly in the mirror and appear far behind the
(b) Concave mirror: reflecting surface ‘caves’ inwards
mirror than they really are.
• This characteristic reduces the space or size of room
needed. reflecting
surface

Anti-parallax mirror in ammeters pole, P C


SPM
’06/P2(A)
(or voltmeters)
object
• Figure 5.10 shows an ammeter with a strip of plane
mirror behind the pointer. The mirror acts as an (c) Convex mirror: reflecting surface bulges outwards
anti-parallax mirror.
Figure 5.12
mirror strip
1 The most common type of curved mirrors is a
spherical mirror.
2 Spherical mirrors are formed when the shell of a
pointer
hollow sphere with the centre C is cut as shown in
Figure 5.12(a).
3 If the inside or concave surface of the cut-off part
Figure 5.10 of the sphere is reflecting, the mirror is concave.
4 If the outside or convex surface of the cut-off part
• A parallax error occurs when the eye sees both the
of the sphere is reflecting, the mirror is convex.
pointer and its image.
5 If both sides are reflecting, for example, a shiny steel
• The correct reading is obtained by making sure that
spoon, then it behaves as a convex mirror if the
the eye is vertically above the pointer, so that the
object is facing the outer surface and a concave
image of the pointer is hidden behind it in the
mirror if the object is facing the inner surface.
mirror.
image
image
inner surface
(concave mirror)

pointer pointer

outer surface
(convex mirror)

(a) Image seen on (b) Image is (c) Image seen on


Figure 5.13
the left side of hidden behind the right side of
the pointer (✗) the pointer (✓) the pointer (✗)
6 In a laboratory, convex and concave mirrors are
made of glass coated with reflecting material such as
Figure 5.11 Correct way to take reading mercury.

257 Light
SPM SPM
Common Terminology and Differences between a ’04/P2(B) ’08/P1

Concave Mirror and a Convex Mirror

Concave mirror (converging mirror) Convex mirror (diverging mirror)

converging light rays reflecting surface reflecting surface

virtual rays

principal axis P C
P principal axis F
C F focal virtual focal point
point
diverging
F light rays
O
5

R
CHAPTER

4 Common terminology of spherical mirrors

 Centre of curvature, C The geometric centre of a sphere of which the concave or convex mirror is a part.

 Pole of mirror, P The centre point on the spherical mirror.

 Principal axis A line which passes through the centre of curvature, C and the pole of a spherical
mirror, P.

 Radius of curvature, r Distance between the pole, P and the centre of curvature, C.
(= CP)

 Focal point, F A point to which all rays parallel to the principal axis converge or appear to diverge
from, after reflection by the mirror.

 Focal length, f The distance between the focal point, F and the pole of the spherical mirror, P.

 Aperture of mirror The portion of the surface of the mirror that reflects light.

 Object distance, u Distance of object from the pole of the mirror, P.

 Image distance, v Distance of image from the pole of the mirror, P.

Differences

Concave mirror Convex mirror

• Rays parallel to the principal axis converge to a • Rays parallel to the principal axis appear to
point, called the real focal point, on the diverge from a point behind the mirror. This
principal axis. point which lies on the principal axis is called
the virtual focal point.
• PF = Focal length, f • PF = Focal length, f
= Distance between the real focal point and = Distance between the virtual focal point
the pole of the mirror and the pole of the mirror
Example: f = +20 cm Example: f = –20 cm

Light 258
Relationship between the Radius of Curvature, r and the Focal Length, f

Consider a ray AM, parallel to and near the principal axis.


nor equal angles
equal angles ma
l
r
A M i M
i A
al r
norm
C F principal axis P F
principal P C
axis f f
r r

(a) Concave mirror (b) Convex mirror


Figure 5.14
F
At any point on the reflecting surface (say M), the angle of incidence equals the angle of reflection. O

5
In both cases, it can be shown that: R

CHAPTER
M
Focal length, f = 1 × Radius of curvature, r Relationship
2 between f 4
Focal length, f = 1 r or r = 2f
and r.
2

Determining the Position and Characteristics of an Image with a Ray Diagram

1 A ray diagram can be used to determine the position and characteristics of an image for various
positions of an object.
2 There are three important rays to be drawn in a ray diagram as shown in the following table.
Concave mirror


C F P C F P C F P

 A ray parallel to the principal  A ray through F is reflected  A ray through C is reflected
axis is reflected to pass through F. parallel to the principal axis. along its own path.
Convex mirror
② ③

P F C P F C
P F C

 A ray parallel to the principal axis  A ray towards F is reflected  A ray towards C is reflected
is reflected as if it came from F. parallel to the principal axis. along its own path.

3 Any two of the rays ,  or  can be used to determine the position and the size of the image.
4 Take note that FP = 1 CP (f = 1 × radius of curvature, r).
2 2
Therefore, always mark the positions of C and F on the principal axis with F in between C and P.
5 (a) A real image is formed at the intersection of two real rays originating from a point on the object
(refer part C on page 260).
(b) A virtual image is formed at the point where
(i) two virtual extended rays intersect (refer part A on page 260),
(ii) one real and one virtual extended rays intersect (see Figure 5.58 on page 289).

259 Light
SPM SPM
Images Formed by a Concave Mirror ’05/P1 ’07/P1

Position of the object Ray diagram Position and characteristics of the image
A SPM
’08/P2(A)
virtual
extended rays Image, I is behind the mirror
I
• virtual
Object, O is 3
O • upright
between F and P C F P • magnified
(u < f) Application: make-up mirror
1

F
O
B
5

R
O Image, I is at infinity
CHAPTER

M 3
C F P • virtual
4 • upright
Object, O is at F 1
y
(u = f) nit • magnified
nfi
toi
lel
ra y Application: reflector in torchlight
ral
pa

C
O 1 Image, I is beyond point C
Object, O is C F P • real
between F and C 2
• inverted
( f < u < 2f ) • magnified
I

D
O 1
Image, I is at C (v = 2f )
2
• real
C P
Object, O is at C F • inverted
(u = 2f ) • same size as the object
I
Application: reflector in projector

E
O 1
Image, I is between C and F
C
2
F
• real
P
Object, O is • inverted
beyond C (u > 2f) I • diminished in size

F pa
fro rallel Image, I is at F (v = f )
m
dis light
tan • real
to
2 b jec
Object at infinity t • inverted
C F P
(u > 2f) 3 • diminished in size
I Application: reflecting telescope

Light 260
Images Formed by a Convex Mirror

1 virtual 1
extended
virtual Concave mirror (can form a real or virtual image)
extended
3 rays rays
3 4
O I F C O I F C
u f u f 5
1 2 3 4 5
principal
(a) Object at a point (b) Object at a point axis
(a) where u > f where u < f C F P
Figure 5.15 1
2
For all positions of an object: 3 F
(a) the positions of its images are O

5
(i) behind the mirror, 1 As the object moves nearer to the concave mirror R
from infinity to F, the inverted real image moves away

CHAPTER
(ii) v < f. M
(b) the characteristics of its images are from the mirror (starting from F). At the same time,
the image becomes larger. 4
(i) virtual,
(ii) upright, 2 At C, v = u = 2f, the image is of the same size as the
(iii) diminished. object.
3 As the object moves nearer to the mirror from F, the
upright virtual image also moves nearer to the mirror.
At the same time, the image becomes smaller (but is
4 still larger than the object).

Norfaza stands at a distance of 1.5 m from a concave


mirror which has a radius of curvature of 2.0 m. What
are the characteristics of her image? Convex mirror (can form a virtual image only)

Solution
As the object distance, u is between f and 2f, the
1 2 3 4 5
5
4 3
image is inverted and magnified. 2
1

principal F F
axis

As the object moves nearer to the convex mirror from


(a) Chee Wai is a scout. He has three mirrors—a plane
infinity, the image also moves nearer to the mirror from F.
mirror, a concave mirror, and a convex mirror.
At the same time, the image becomes larger (but is still
Which mirror can be used to ignite a match?
smaller than the object).
Explain your choice.
(b) When you hold a shiny steel spoon with its back
facing you at a distance of approximately half a Key:
metre from your eyes, you will see an upright image
of yourself, but if the other side of the spoon is
1 … 5 positions of object
facing you, your image is inverted. Explain. 1 … 5 positions of corresponding images

(i) Upright image (ii) Inverted image General Information


1 The image formed on the same side of a
Solution curved mirror as the object is real and
(a) He must use a concave mirror which can reflect the inverted.
sun’s rays to converge on the match to light it. 2 The image formed on the opposite side of a
(b) The spoon is first a convex reflector in (i) and then a curved mirror is virtual and upright.
concave reflector in (ii).

261 Light
Widening the Field of Vision by Using a Convex Mirror SPM 2 When a convex mirror is used, the field of
’06/P1
vision is widened.
1 Figures 5.16(a) and (b) show the fields of 3 For this reason, convex mirrors are used in
vision of an observer who is looking through a many apparatus.
plane mirror and a convex mirror of the same
size respectively.

Images formed P
by a convex
P′
mirror are always
smaller.
field of vision wider field of vision
F Q Q′
O (a) Plane mirror (b) Convex mirror
5

R
Figure 5.16
CHAPTER

4
Application and Construction of Apparatus Which Utilise Concave Mirrors and Convex Mirrors
SPM
’06/P1
Applications of
concave mirror

Reflector Make-up Mirror


1 Concave parabolic mirrors are used as reflectors 1 For this purpose, a
in torchlights and car headlights. wide-aperture
Magnifying Mirror concave mirror
1 Concave mirror in with a large radius of
dentist’s mirror curvature is used.
magnifies the image
so that the dentist
can get a closer look
at the teeth.
2 The images
reflector
produced are
upright.
Figure 5.20
beam of parallel light rays
2 The user must be
Figure 5.17
positioned between
2 The light bulb is fixed in position at the focal point the focal point and
of the concave mirror to produce a beam of the pole of the
parallel light rays so that the light rays will Figure 5.19 mirror.
maintain a uniform intensity for a greater distance. 3 In this way, the
3 Other applications of this principle are the slide image formed is
projectors and telescopes. virtual, upright and
magnified.

(a) Slide projector (b) Telescope


Figure 5.18

Light 262
Uses of convex mirrors

Rear-view Mirrors Safety Mirrors


Convex mirrors are used as Wide-angle view Mirrors Convex mirrors are also mounted
rear-view mirrors in motor Convex mirrors are hung from at sharp corners of roads so that
vehicles to give drivers a wide- the corners of supermarket drivers are able to see oncoming
angle view of the vehicles ceilings to provide a wider field cars on their blind side
behind them. of vision of the activities taking
place in the shopping areas.

F
O

5
Figure 5.23 R
Figure 5.21

CHAPTER
M
Figure 5.22
4

1 SPM
Clone
’05

A boy stands 3 m from a plane mirror in a room. A (c) The time shown by the wall
wall clock is hung on the opposite wall 2 m away clock is 9.00 a.m.
from the boy. Draw the hands of the clock on
the diagram to show what the
boy observes.

Answer
wall clock (a) The image of the clock is 5 m behind the plane
plane mirror mirror. Therefore, the distance between the boy
and the image of the clock is (3 + 5) = 8 m
2m 3m
(b) The boy’s movement does not affect the image of
the clock. So the new distance between the boy
The boy looks at the mirror and sees the image of the
and the image is (2 + 5) = 7 m
clock.
(c) Since the image formed by a
(a) What is the distance between the boy and the
plane mirror is laterally inverted,
image of the wall clock?
the image of the clock shows
(b) The boy then walks 1 m towards the plane mirror.
3 p.m.
What is the distance between him and the image
of the clock at this new position?

2 SPM
Clone
’10

An object is placed 30 cm in front of a concave A Virtual, diminished, upright


mirror of focal length, f, 15 cm. B Virtual, same size, upright
Concave mirror C Real, diminished, inverted
Object D Real, same size, inverted
Comment
ƒ
The object distance is 30 cm which is equal to 2f
30 cm 15 cm (2 × 15 cm). At this distance, a real and inverted
image of the same size as the object is formed at the
same place as the object.
What are the characteristics of the image formed? Answer D

263 Light
5.1

1 The diagram shows the positions of a girl and a the mirror is rotated 15° anticlockwise at point O, the
SPM flower pot in front of a plane mirror.
Clone reflected ray OB is turned to position OB’.
’09 What is the value of ∠BOB’?
plane mirror N
B'

B
A
60
15
O
F
O 3m 7 A bee flies in a direction perpendicular to a plane
4m
5

R mirror at a speed of 0.8 m s–1. What is the distance


CHAPTER

M What is the distance between the flower pot and the travelled by the bee towards its image in 5 seconds?
image of the girl? 8 The figure shows a point light source positioned at
4
2 the focal point, A of a concave mirror.

light source
observer
plane mirror A B

Draw on the above diagram to show the position


and appearance of the image as seen by the
observer’s eye. What must be done so that the light reflected by the
3 The diagram below shows a girl looking at a plane concave mirror can converge to point B?
mirror. A white board is behind her. 9 The radius of curvature of a concave mirror is 20 cm.
At what distance must an object be placed to produce
(a) an image at infinity?
(b) an image of the same size?
plane
WET mirror
10 A ladybird flies horizontally in front of a concave
mirror at a distance beyond the centre of curvature,
C. The path of the ladybird is parallel to the plane of
the mirror. The ladybird travels from the north to the
south. Explain the movement of the image on the
Draw and show the image observed by the girl. concave mirror.
4 Aziz stands at a distance of 6 m from a large plane 11 A dentist is examining Brenda’s teeth. The form of
mirror. What is the distance that he has to walk to be the apparatus used is ____________ mirror.
at a distance of 3 m from his image? 12 The figure shows the formation of an image, I on a
5 The figure shows two plane mirrors placed at an screen of an object, O positioned in front of a
angle of 100°. A ray of light is directed onto one of concave mirror.
the mirrors at i = 60°. Find the angle of reflection, θ P Q
on the second mirror.
card
normal
M N I
60
O principal axis
concave
B screen mirror
θ 100
What happens to the image if a card is used
A O (a) to cover the upper half of the object (at position
P)?
6 The figure shows a ray of light directed onto a plane (b) to cover the upper half of the mirror (at position
mirror at an angle of incidence of 60°. When Q)?

Light 264
5.2 Understanding
Refraction of Light
Refraction of Light SPM
’05/P1

1 When a slab of glass is held in front of a pen, as


shown in Figure 5.24, it would appear as if the Figure 5.24 The glass block refracts light
pen is broken. normal
2 The ‘broken pen’ illusion occurs because light 1 angle of
Ray of light
is bent by the glass. This bending effect is travels at a incidence
called refraction of light. certain velocity.
incident ray Light travels faster
3 Refraction of light is the change in direction F
Optically less
O
dense medium.

5
or bending of the light as it enters from one medium A R
2

CHAPTER
medium to another. Refraction medium B Light travels slower M
occurs, ray of Optically
light crosses denser medium. 4
the boundary
It is caused by the change in the velocity of between two
light as it propagates from one medium to mediums and angle of refracted ray
changes refraction
another of different optical densities. direction.

3
Ray of light travels
A medium in which the velocity of light is at a new velocity.
lower is an optically denser medium.
Figure 5.25
SPM
Two Ways for a Ray of Light to Bend ’08/P2(A)

normal 1
Ray of light travels from air
incident (less dense) to glass (denser):
ray i • The ray is bent towards
i>r the normal.
air
glass • After entering the glass, the
speed of light decreases.
r This causes the ray of light
original to bend towards the normal.
path • From Fast to Slow, bend
2 Towards the normal
Ray of light travels from glass (FST rule).
(denser) to air (less dense): refracted
• The ray is bent away from ray
the normal.
• After emerging from the , Note
glass, the speed of light i
increases. This causes the • i’ = r
ray of light to bend away • r’ = i
from the normal. , ,
i <r
• From Slow to Fast, bend ,
Away from the normal r
(SFA rule). emergent
original ray
normal path
Figure 5.26

incident ray

When a ray of light crosses the boundary between two different mediums
at a right angle air
• No refraction occurs, so the ray does not bend. glass
• However, the speed of light still changes because the nature of medium
determines the speed of light.
• i = 0° and r = 0°

265 Light
Refractive Index and the Speed of Light 5 Since c > v, n > 1 for all mediums
1 Light travels at a speed of 3 × 108 m s–1 in a 6 A refractive index has no unit as it is a
vacuum. comparison of two speeds.
2 This is the highest possible speed of light.
c 1
3 When a ray of light passes from a vacuum into 7 Since n = , n (c = constant).
v v
a transparent material or medium, its speed is
reduced. Therefore, refractive index, n (or optical
4 The refractive index, n of the medium is density) is inversely proportional to the speed
defined as the ratio of the speed of light in a of light in the medium. The higher the value of
vacuum to the speed of light in the medium. n of a medium, the slower the speed of light in
the medium and vice versa.
F
O Speed of light in vacuum c 8 The following table shows the refractive
n = =
5

R Speed of light in the medium v indices of various types of materials and their
CHAPTER

M corresponding speeds of light.


4

Medium Refractive index, n Speed of light, v (cm s–1)


Vacuum 1.00 3 × 108 = c
(highest speed of light)
Air = 1.0003 ≈ 1.0 2.999 × 108
refractive Ice 1.31 2.29 × 108
index, n
4
or Water 1.33 ≈ 2.26 × 108
3
optical
density Perspex 1.49 2.01 × 108
increases 3
↓ Crown glass 1.52 ≈ 1.97 × 108
2
speed of
light Light flint glass 1.58 1.90 × 108
decreases Dense flint glass 1.66 1.81 × 108
Diamond 2.42 1.24 × 108
Rutile 2.91 1.03 × 108
Gallium phosphide 3.50 0.86 × 108
(highest optical density)

Note n > 1 for all mediums

Optical Density and Physical Density


5
1 Optical density and physical density, ρ
The refractive index of paraffin is 1.44.
(
ρ = mass
volume )
are different parameters. What is the speed of light in paraffin?

2 A material with a greater optical density than Solution


water does not imply it has a greater physical c
n=
density than water. v
3 For example, paraffin is optically denser than 3 × 108
water but is less dense than water as a liquid. v= = 2.08  108 m s–1
1.44

Light 266
2 The following table shows the comparison
6 between Snell’s law and the refractive index of
a medium.
When light travels from air to a medium, its speed is
decreased by 25%. What is the refractive index of the
medium? Refractive index of a
Snell’s law
medium
Solution
sin i sin i
Speed of light in the medium, v = 75%c • = constant • n=
sin r sin r
3
∴ v= c • True for any two • True only when
4
c mediums vacuum or air is
Refractive index, n = involved
v
• True when light • True only when light F
c O
= travels from medium 1 travels from vacuum

5
3 R
c to medium 2 or (or air) to the medium

CHAPTER
4 M
vice versa concerned
= 1.33 4
i r
medium 1 imedium 1
Laws of Refraction e.g. medium 2 vacuum/air
medium 2
sin i e.g. medium
sin r r sin i i
r
= 1.5 sin r
the plane where the = 1.5
i
incident ray, refracted
ray and normal lie F4/5/72
i r i = angleF4/5/73
in vacuum/air
r medium 1 medium 1
e.g.
r = angle in medium
medium 2 medium 2
sin i
r sin r i
= 0.87

F4/5/72
Figure 5.27
1 There areF4/5/71
two laws of refraction. 7 SPM
’06/P1
2 When a ray of light travels from one medium
to another, Figure 5.28 shows a ray of light passing from air to a
(a) the incident ray and the refracted ray are medium with a refractive index, n.
on the opposite sides of the normal at the
point of incidence, and all three lie on the
same plane. 45°
sin i
(b) the value of is a constant, where i is air
sin r
medium
the angle of incidence and r is the angle of 25°
refraction. This second law of refraction is
also known as Snell’s law.
Figure 5.28
F4/5/74

Snell’s Law and Refractive Index Calculate the value of n.


Solution
1 When a ray of light travels from vacuum (or
air) at an angle of incidence, i and refracted sin i (angle in air)
n =
into a medium at an angle of refraction, r, it sin r (angle in medium)
can be shown that the refractive index of the sin 45°
=
medium is given by: sin 25°
= 0.7071
0.4226
n = sin i (in vacuum)
sin r (in medium) = 1.67

267 Light
5.1
Refraction of light
Hypothesis 5 A ray of light from the ray box is directed along
the 20° line. The emerging ray from the side RS is
(i) The greater the angle of incidence, i, the greater
drawn.
the angle of refraction, r.
6 Step 5 is repeated with other angles of incidence.
(ii) The sine of the angle of incidence, i, is directly
7 The glass slab is removed. The refracted rays are
proportional to the sine of the angle of refraction,
drawn by joining the points of incidence to the
sin i
r. The value of is a constant, which is the corresponding points of emergence of the rays
sin r
of light. A protractor is used to measure the
value of the refractive index, n, of the glass. respective angles of refraction.
F
O Aim sin i
8 The values of sin i, sin r, and (which is the
5

R sin r
(i) To investigate the relationship between the angle
CHAPTER

M
of incidence and the angle of refraction. refractive index of the glass, n) are calculated and
4 sin i
(ii) To determine the Frefractive index of glass. tabulated. The average value of sin r is obtained.
O
Variables
5

R 9 A graph of r against i and a graph of sin i against


CHAPTER

(a) Manipulated : M
angle of incidence, i sin r are plotted.
(b) Responding : angle of refraction, r
4 Tabulation of data
(c) Fixed : type of glass (refractive index)
Apparatus/Materials Angle of Angle of n
Ray box, glass slab, white paper, and protractor. incidence, refraction, sin i sin r sin i
=
i (°) r (°) sin r
Arrangement of apparatus
20
emergent ray 30
40
S R
50
refracted
N
60
ray glass
r block
Graph
P O Q
ray box 60° r (°)r (°)
sin isin i
50° i normal emergent ray
40°
30° S R
20° M
incident rays 0°
refracted
N glass
ray r
Figure 5.29 block
O O P O O
O i (°) i (°) Q sin rsin r
ray box
Procedure 60°
(a) normal (b)
50° i
1 The outline of the glass slab is traced onto a sheet 40°
30°
Figure 5.30
M F4/5/77
F4/5/77
20° 0°
of white paper and labelled as PQRS. incident rays
Discussion
2 The glass slab is removed. Point O is marked on
1 sin i
the side PQ such that OP ≈ PQ. From the formula: n =
3 sin r
Experiment 5.1

3 A protractor is used to draw and mark the normal, sin i = n sin r


MN, through point O, and lines forming angles of Compare it with the equation of a straight line.
incidence of 20°, 30°, 40°, 50° and 60° with the y = mx
normal. n = m (= gradient)
4 The glass slab is placed back on its outline on the The refractive index of glass can be determined from
paper. the gradient of the graph of sin i against sin r.

Light 268
Conclusion line passing through the origin. Therefore, sin i
is directly proportional to sin r.
1 (a) The graph of r against i shows that as i
increases, r also increases. However, i is not sin i
2 The value of = constant, thus Snell’s law is
directly proportional to r because the graph is sin r
verified.
not a straight line graph.
(b) The graph of sin i against sin r shows a straight The hypotheses are valid.

8 SPM
’07/P1

Figure 5.31 shows a ray of Solution


light travelling from air to sin i c F
water at an angle of incidence 60° (a) n= (b) n= O
sin r v

5
of 60°. air R
sin 60° c

CHAPTER
water sin r = M
1.33 v=
r n
0.8660 4
= 3 × 108
Figure 5.31 1.33 v=
1.33
Calculate = 0.6511
r = 40° 37′ = 2.26 × 108 m s–1
(a) the angle of refraction, r,
(b) the speed of light in water. The speed of light in water
[Refractive index of water = 1.33] is 2.26 × 108 m s–1.

Refractive Index and Bending Ability of SPM


’08/P1
a Medium

A ray of light passes 1 The refractive index is an indication of how


from a medium to the ray of light bends when it enters the surface
air as shown in the 55° of a medium from the air.
figure. Calculate 2 A material with a higher refractive index has a
the refractive air greater bending effect on light because it
index of the slows down the light more and it bends the ray
medium.
36° of light more towards the normal. It causes a
larger angle of deviation of the ray of light.
sin i sin 36° 0.5878
n= = = = 0.72
sin r sin 55° 0.8192 F4/5/79

Reminder:
(1) The value of n must be greater than 1. 45° 45°
sin i air air
(2) n = is true for light passing from air into
sin r water glass

the medium, so the direction of travel of the θ1 θ2


light ray must be reversed.
sin i The
n= θ1, θ2 = angle of deviation
sin r principle of
Experiment 5.1

sin 55° reversibility n (glass) > n (water)


= 55° of light F4/5/81
sin 36° θ2 > θ1
states that
0.8192 air the paths ∴ A ray of light bends more in glass than in water
= because n (glass) > n (water).
0.5878 of light
= 1.39 36° rays are i.e. greater n → greater refraction
reversible. → light bends more

F4/5/80
269 Light
1 In the course of conducting the experiment on
Snell’s law, you would have noticed that the
Dispersion incident ray is parallel to the emergent ray, i.e.,
Dispersion is the separation of white light into its ∠i = ∠a.
component colours by a prism. 2 The effect of the whole glass slab on the incident
ray is that the incident ray is moved aside but its
white screen direction of travel is still maintained.
Red
slit prism
Orange
Yellow 3 Reflection occurs on both the top and bottom
Green spectrum
Blue
Indigo
boundaries where the intensity of the reflected
white sunlight Violet
ray is weak.
The seven colours are: Red, Orange, Yellow, Green,
Blue, Indigo and Violet. F4/5/83 Real Depth and Apparent Depth SPM
’04/P1
SPM
’05/P1
F When a narrow beam of white light passes through a
O glass prism, the seven colours are refracted at different 1 The refraction of light gives us a false
5

R impression of depth. Water in a deep pool


angles. Amongst the seven colours, the red with the
CHAPTER

M
highest velocity in glass is the least deviated or appears shallower than it really is.
4 refracted while the violet with the lowest velocity is
the most deviated. As Fa result, the refractive index of incoming light
O
violet light is greater than the refractive index of red
5

light. The seven coloursR fall on a white screen to form a


CHAPTER

M
continuous spectrum.
apparent
normal 4 normal I
depth of
the bottom
vio i
red i let
lig lig
ht ht actual depth
ori ori
gin gin of the bottom
al al
rrr r pa rv pa
th th
(a)
bends less bends more F4/5/86a
towards the normal towards the normal observer

(a) (b)
rr > rv F4/5/85 air
sin i apparent
From the formula: n = real
depth, d
sin r depth, D I
∴ nred < nviolet water

(b) Refraction makes a swimming pool look


shallower than it really is
Figure 5.33

2 Figure 5.33 shows two rays of light, OA and OB


Effect of the Whole Glass Slab on the Incident Ray leaving a point at the bottom of the pool. The
weak rays are refracted away from the normal at the
reflected ray water-air boundary before entering the
weak observer’s eye.
refracted incident
ray i i ray 3 To the observer, the emergent rays seem to
glass block with originate from a point, I which is above the
parallel sides object, O. As a result, the bottom of the pool
r
internally
rr appears closer to the surface of the water.
reflected refracted ray
ray 4 From Figure 5.33, the following terms are
defined.
original path
a emergent ray parallel (a) Apparent depth, d
to incident ray The distance of the image from the
lateral
displacement surface of the water (or the boundary
Figure 5.32 between the two mediums involved).
F4/5/82
Light 270
(b) Real depth, D
The distance of the object from the surface of the water (or the
boundary between the two mediums involved).
5 The refractive index, n of a material can also be expressed in terms of Higher n
the real depth and apparent depth as follows: ↓
slower light of speed
Real depth, D ↓
Refractive index, n = more bending
Apparent depth, d
(if i is constant)

6 This formula is true only if the observer’s line of view is from the air lower apparent depth
and perpendicular to the medium-air boundary. (if real depth is constant)

F
O

5
R
5.2 SPM

CHAPTER
’07/P3(B) M

Apparent depth and real depth 4

Hypothesis 2 Another pin, P is mounted on a piece of cork and


clamped to a retort stand.
The greater the real depth, the greater the apparent
3 The beaker is filled with water to a height of
depth.
8.0 cm (real depth, D = 8.0 cm).
Aim 4 Pin P is adjusted until it coincides with the image
To investigate the relationship between the apparent of pin Po by using the non-parallax method.
depth and the real depth 5 The apparent depth, d of image of pin Po below
Variables the water surface is recorded.
6 The experiment is repeated using D = 10.0 cm,
(a) Manipulated: real depth, D
12.0 cm, 14.0 cm and 16.0 cm.
(b) Responding : apparent depth, d
(c) Fixed : water (refractive index) Tabulation of data
Notes
The real depth is the depth of the water. The apparent Real depth, D (cm) Apparent depth, d (cm)
depth is the distance from the position of the pin, P 8.0
(where there is no parallax between the pin and the
10.0
image of the pin, Po) to the surface of the water.
Apparatus/Materials 12.0
14.0
Tall beaker, cork, 2 long pins, metre rule and retort
stand with clamp. 16.0
Arrangement of apparatus
Graph
observer

metre rule d (cm)


tall beaker

water
image of pin, P0 cork
D (cm)
pin P
Figure
F4/5/88 5.35
Experiment 5.2

pin P0

Figure 5.34 Conclusion


The graph of d against D shows that the apparent
Procedure
depth increases as the real depth increases.
1 A long pin, Po is placed in a tall beaker. The hypothesis is valid.

271 Light
9 SPM
’03/P2(A)
SPM
’05/P1 10
The depth of a pool is 2 m. What is the apparent A coin rests at the bottom of a liquid in a container.
4 When viewed from above, the coin appears to be
depth if the refractive index of water is ?
3 raised up to a distance equal to one-third the depth of
the liquid. What is the refractive index of the liquid?
Solution
Real depth, D Solution
Refractive index, n =
Apparent depth, d Apparent depth, d = D –
1
D
3
4 2
=
3 d Apparent depth, d =
2
D
3
F 3
d = 2× D D
O 4 Refractive index, n = = = 1.5
d
5

R 2
= 1.5 m DD
CHAPTER

M 3
4
The apparent depth of the pool is 1.5 m. The refractive index of the liquid is 1.5.

Archer fish applies Physics!


Concept: Light does not refract or bend when it crosses a boundary of two mediums perpendicularly.
The law of Physics is applied when an archer fish shoots a powerful jet of water to knock its prey off the branch
where it is resting. When the prey is in its line of vision, it shoots at the prey.
bug
bug
appears
here bug

(a) There is an occurrence of refraction. (b) There is no refraction. (c) Shoot!


This is not the right position to shoot from. This is the right position.
The prey is not at the place where it seems Ready.
to be.

SPM
’09/P1

‘You should throw your spear at the position below the


fish you see. What you see is the image of the fish at its
apparent depth. The fish is, in reality, below this position’,
image of fish the instructor explains.
However, what must the instructor do if he wishes to
shoot a fish with a laser pistol?

Solution
Shoot at the fish as seen. A laser ray will refract at the
surface of the water and travel in the same direction as
A diving instructor is teaching his students the art of the actual position of the fish.
spearing fish in the sea.

Light 272
Effects of the Refraction of Light
Bending of a Ruler

5
3
The submerged portion of the To an observer, the emergent rays
ruler appears to be at position of light appear to originate from
RI, thus giving the impression the position of the virtual image, I.
that the ruler is bent.

P
ruler F
O

5
observer
R

CHAPTER
water M
1 2
Water is optically When the rays of light, OA and 4
denser than air. OB emerge from the surface of
the water, they are refracted
away from the normal.
4
The rays of light from all the
points along the length RO of the
ruler are similarly refracted.
Figure 5.36

Broken Pencil

2 3
Rays of light from portion of pencil Rays of light from the portion of pencil
above the water travel directly to the submerged in water are refracted when
observer’s eyes without any refraction. they emerged from water.

pencil placed
to the left
of the centre, C
C C

1
image of A pencil which
the is placed to the
submerged
portion of left of the
the pencil centre, C of a observer
thin glass filled 4 observer
This causes the image of 5
with water The further the pencil is
seems to be the pencil appears to be
broken. placed away from C, the
broken. more the broken effect is.
(i) (ii)
(a) Side view of a pencil in a glass of water (b) Top view of the submerged portion of the pencil and its image

Figure 5.37

273 Light
Atmospheric Refraction and the Setting Sun

1
al Atmosphere consists of layers of air with density or
light in straight rm atmosphere
line no density refractive index increasing downwards.
3 1 increases
cu 3 downwards
2
pat rved 2
h
co du ht l i g
ref ntinu e to The rays of light from the sun travel in straight lines
rac ous
tion before entering the atmosphere.

(a) 3
oval-sh F4/5/97 After the rays of light have entered the atmosphere,
above aped sun a line of
the ho p
rizon pears vision the rays of light are bent continuously downwards.
F 7 As the density increases slowly and continuously,
atmosphere
O the path of light becomes a curve. This is known as
5

horizon 6
R atmospheric refraction.
observer
er path
CHAPTER

M
8b short th
round
ng er pa 4
4 sun 8a lo
5
F When an observer sees the sun touching the sea, the
4 O whole sun has actually already set.
5

sun below the horizon R


5
CHAPTER

M (b)
The rays from the sun travel in straight lines until
4 F4/5/98 they reach the top of the atmosphere.

6
They then curve downwards until they reach the
(c) observer’s eyes.
Figure 5.38 Setting sun appears to be oval-shaped
7
8 By then, the line of vision is above the horizon. The
The setting sun also looks flattened (or oval in shape) sun thus appears to be above the horizon.
because the light from the lower part has travelled a
longer path in the atmosphere before reaching the
observer’s eyes. More refraction has occured and as a Note:
result, the lower part of the sun is raised more than the By the same reasoning, refraction also causes
upper part making it appear oval. early sunrise. In effect, day time is lengthened.

Twinkling of the Stars

4 The fluctuation of the local air densities changes


light
from a the path of light rapidly.
star the Earth's 5 Thus the starlight enters the eyes for one moment,
atmosphere
and does not at another.
Earth 6 This constant but random shifting results in the
twinkling of stars.
7 The moon and planets do not twinkle because
Figure 5.39 their apparent sizes are not affected by the
F4/5/100
1 Stars twinkle because we view them through thick small changes in atmospheric refraction which
layers of turbulent (moving) air in the Earth’s does not alter the amount of light that enters
atmosphere. If an observer is in outer space, he the eyes. This is one way of distinguishing stars
does not see the stars twinkle. from planets.
2 The movement of air changes the densities or the 8 A horizontal star twinkles more than an
refractive indices of the layers of the atmosphere. overhead star as the light from the former travels
3 Light refracts as it enters the regions of different through more layers of air before it reaches the
densities. eyes of an observer.

Light 274
3 SPM
Clone 5.2
’10

1 The figure shows


The word Physics appears as shown in the diagram
a ray of light travelling
under a magnifying lens. from medium 1 to
medium 2.
Which medium is
optically denser?
Physics Give an explanation for your answer.
2 The figure shows a ray of
light passing from a glass
into a liquid which has
the same refractive F
Which of the following light phenomena explains index as glass. O

5
the situation? R
What will happen to

CHAPTER
A Reflection
M
the incident ray when it enters the liquid?
B Refraction Direction of travel: __________________ 4
C Total internal reflection Speed of light: _____________________
D Diffraction 3 Figures (a) and (b) show two coins placed at the
bottom of two liquids of different refractive indices,
Comments where n1 > n2.
The magnifying lens magnifies an object when it is
placed at a distance less than the focal length. The
refraction of light due to the lens causes the
magnification of the image.
Answer B

(a) (b)
4 SPM
Clone
’10
Which coin will appear closer to the surface of the
liquid?
A light ray enters water from normal 4 The figure shows Jackson
air as shown in the diagram. SPM standing by the side of a
Clone
’05 swimming pool where the
What is the refractive index of 40°
water? air
depth of water is 1.2 m.
A 0.75 29°
61° water Jackson seems to be
shorter than he actually is.
B 1.33
What is the apparent depth of the pool,
C 1.38 given that the refractive index of water is 1.33?
D 1.58
5 The figure shows a ray
Comments of light passing from
Identify the angle of incidence and angle of air into a transparent
refraction first before applying the Snell’s Law. medium.
What is the refractive
Solution index of the medium?
Angle of incidence, i = 40° 6 The figure shows a ray of light passing from the air
Angle of refraction, r = 29° into a transparent material. The ray is refracted.
Applying Snell’s Law
sin i
n(water) = ––––
sin r
sin 40 = 1.33
= ––––––
sin 29
Answer B
Calculate the refractive index of the material.

275 Light
5.3 Understanding Total Internal Reflection

Total Internal Reflection

1 Total internal reflection is the total reflection of a beam of light at the boundary of two mediums,
when the angle of incidence in the optically denser medium exceeds a specific critical angle, c.
2 The boundary between the two mediums acts like a perfect plane mirror where total internal reflection
occurs.
SPM SPM SPM SPM SPM
Critical Angle and Total Internal Reflection ’05/P1 ’06/P1 ’07/P1 ’08/P1 ’09/P1

F 1 The following flow chart shows four situations in which a ray of light passes from glass
O into air—a medium of lower refractive index, as the angle of incidence is increasing.
5

R
CHAPTER

M
• Angle of incidence, i is small (say 10°). • Angle of incidence, i is increased.
4 • Produces a strong refracted ray and a weak reflected ray. • The angle of refraction is also increased and the
F
• Intensity of Intensity of refracted ray is closer to the glass-air boundary.
= O + Intensity of • Intensity of refracted ray decreases while
incident ray refracted ray
5

R reflected ray
intensity of reflected ray increases.
CHAPTER

4
(a) i is small (b) i is bigger but < c
normal normal
ay
dr
refracted ray ra cte
(strong) r ref 3
r 3 air
air glass re
fle
glass 2 reflected ray 1 i<c ct
(weak) ed
1 i<c ra
2 y
incident
incident ray ray
• Refraction and internal reflection • Refraction and more internal reflection
F4/5/114
F4/5/113

(d) i > c (c) i = c


normal normal

r = 90°
air air refracted ray
glass c glass 3
re

1 i>c 2
fle

1 i=c
cte

reflected 2
incident
d

ray ray incident


ra

ray
y

• Total internal reflection • Critical internal reflection


F4/5/116 F4/5/115

• Angle of incidence, i > critical angle, c. • Angle of incidence, i reaches a certain


• No refraction occurs (no refracted ray emerges to the air). value called the critical angle, c (c = 42°
• All the incident light is reflected back into the glass. for glass).
Total internal reflection occurs when the angle of • The refracted ray travels along the glass-air
incidence exceeds the critical angle, c. boundary.

2 The critical angle is the angle of incidence in the optically denser medium for which the angle
of refraction in the less dense medium is 90°.
3 The conditions required for the occurrence of total internal reflection are:
(a) the light ray must travel from an optically denser medium to a less dense medium.
(b) the angle of incidence must be greater than the critical angle, that is, i > c.

Light 276
11
Do you know why total internal reflection does not Calculate the critical angle for
occur when light travels from an optically less dense (a) a glass with refractive index, n = 1.5,
medium to a denser medium? (b) water with refractive index, n = 1.33.
As i increases, Solution
r also increases.
sin 90°
But r < i for all (a) = n
values of i. sin c
1 1
sin c = = = 0.6667
n 1.5
c = 41.8° ≈ 42° F
(a) 1 1 O
(b) sin c = = = 0.7519

5
n 1.33 R

CHAPTER
M
When i = 90°, c = 48.8° ≈ 49°
r = 42° (critical
angle for glass). 4
glass So, r can never
air be 90° i.e. total 12 SPM
’03/P1
internal reflection
never happens.
A ray of light passes through a transparent medium
as shown in Figure 5.41.
(b)

Relationship between Critical Angle, c


and Refractive Index, n

1 When a ray of light passes from glass into


air with an angle of incidence equal to the Figure 5.41
critical angle, c, the refracted ray makes an Calculate
angle of 90° with the normal as shown in (a) the refractive index of the medium,
Figure 5.40(a). (b) the angle of incidence, i.
Solution
(a) Critical angle, c = 140° – 90° = 50°
1 1
n= = = 1.31
sin 50° 0.7660
(b)

(a) (b)
Figure 5.40

2 By the principle of reversibility of light, we


have the path of ray as shown in Figure
5.40(b). Angle of refraction, r = 180° – 140°
3 Formula for refractive index: = 40°
n = sin i n =
sin i
=
sin i
sin r sin r sin 40°
sin 90° sin 90° = 1
= sin i = 1.31 × sin 40°
sin c = 1.31 × 0.6428
1 = 0.8421
∴ n = or sin c = 1 i =57.4°
sin c n

277 Light
13
Figure 5.42 T Given that the critical angle of water is 49°, which
shows a ray of path of the ray of light is correct?
light passing U Solution
from water
O air To determine the right answer, first we must know the
into air. V
40° 40° water
value of the angle of incidence.
inc
ide Angle of incidence = 90° – 40° = 50°, which is > 49°
nt
W ray P (the critical angle of water) and thus total internal
reflection occurs.
Figure 5.42 The correct path is POW.
F
O F4/5/123
Activ
5

R (a) To show the effect of increasing the angle of incidence on the angle of
ity 5.1
CHAPTER

M refraction when light travels from one medium to a less dense medium
(b) To obtain the critical angle and refractive index for glass
4 6 The refractive index, n is calculated using the
Apparatus/MaterialsF
O glass block, 360°-protractor 1
Ray box, semicircular formula, n = .
5

sin c
(or a scaled circle). R
CHAPTER

M
Arrangement of apparatus Results/Observation
4 1 (a) When the angle of incidence, i is less than
B
the critical angle, c:
0° refracted ray Part of the light beam will be refracted on
r
Or crossing the glass-air boundary and part of it
C 90° 90° D
i
semicircular will be reflected within the glass block.
glass block
incident ray
protractor (b) When the angle of incidence, i increases, the

angle of refraction, r also increases. The
A
refracted ray is getting closer to the air-glass
reflected ray boundary. At the same time, the intensity of
the refracted ray decreases.
ray (c) When the angle of incidence, i is equal to
box the critical angle, c:
Figure 5.43 Studying total internal reflection
F4/5/124 The refracted ray travels along the length of
Procedure the glass-air boundary.
1 The semicircular glass block is placed on a (d) When the angle of incidence, i exceeds the
360°-protractor (or a scaled circle) as shown in critical angle, c:
Figure 5.43. The midpoint, O of the straight No refraction occurs and all the light energy
edge should coincide with the interception point is totally reflected internally within the glass
of the two diameters of the protractor. block.
2 A narrow beam of light from the ray box is 2 The critical angle measured is 42°.
directed at O, where AB is the normal, at an 1
3 The refractive index of glass, n = = 1.49
angle of incidence, i = 10°. The refracted ray sin 42°
and the reflected ray emerging from the glass Conclusion
block are observed. 1 The angle of refraction in the air increases when
3 The angle of incidence is increased and the changes the angle of incidence in the glass is increased.
on the refracted ray and reflected ray are observed. 2 The critical angle of glass, c is obtained when
4 The angle of incidence is increased until the the angle of refraction in the air is 90°.
Activity 5.1

refracted ray travels along the length of the 3 Total internal reflection occurs when the angle
air-glass boundary. The angle of incidence when of incidence exceeds the critical angle.
this occurs is measured. The angle is known as 4 The refractive index of glass can be calculated
critical angle, c. 1
using the formula, n = .
5 The angle of incidence is further increased. sin c

Light 278
Students think that the
angle of incidence at
P is 45°.

Given that the critical normal


angle, c of glass is 42°, 60°
60° P
complete the ray diagram Students must first draw 60°
shown. 60°
the normal at P and then
complete the ray diagram as
normal
shown beside.
60° F4/5/125
P Please take note that the angle F4/5/127
Students think that the of incidence at P is 60° and
angle of incidence at therefore the angle of reflection
P is 45°. is also 60°. F
O

5
R

CHAPTER
F4/5/126 M

4
Students must first draw
the normal at P and then
SPM SPM
Effect of
complete the arayPrism
diagramonasRays of Light ’03/P1 ’07/P2(A)
shown beside.
Please take note that the angle
of incidence at P is 60°When
and two parallel rays of light are directed to a prism, total internal
therefore the angle of reflection
reflection occurs and various effects can be observed as shown.
is also 60°.

(a) The two parallel rays of light are deflected (b) The two parallel rays of light are inverted.
by 90°. An inverted image is seen.

A
observer
B

A B Figure 5.45

Figure 5.44

(c) The two parallel rays of light are deflected by 180°. The two rays are inverted and an inverted image is
seen.
(c) (i) 1 prism (ii) 2 prisms

A A
B B

B
A B
observer
observer A

Figure 5.46

Figure 5.47

279 Light
observer
Na
Natural
aturall Phen
Phenomena
nome ena an
and
nd thee Use
Uses
es
off Totall Internal
Interrnal Reflection
Reflecction

Road mirage
1 While driving a car on a hot day, you may see a pool of water on the road in front of you. But as you
FF approach that particular spot, you find that the pool does not really exits. What causes this to happen?
OO
2 This phenomenon is called a mirage. It is caused by refraction and total internal reflection.
5

RR
MM
3 Mirages normally occur during the daytime when the weather is hot.
CHAPTER

4 The occurrence of a mirage can be explained as shown in Figure 5.48.


44
cloud X

cool air observer


warm air
6 density
decreasing
g hot air Y

rays travel through cool and


warm layers of air and undergo
total internal reflection
pool of virtual image 1
water seen by • The air above the road surface
observer's
b ' eye consists of many layers.
Figure 5.48 Mirage • The layer of air nearest to the road is
hot and the upper layers get progres-
5 6 sively cooler.
Total internal reflection occurs in this The observer sees the image of the • As a result, the density and refractive
layer and the ray of light bends in an sky and the clouds as a pool of index of the air decreases from the top
upward curve towards the eye of the water on the surface of the road. downwards the road surface.
observer.

4 3 2
Finally, at the layer of air closest to As the ray passes through each A ray of light, XY
Y originating from the sky
the road surface, the angle of layer, the angle of incidence and travelling downwards is refracted and
incidence exceeds the critical angle. increases with each layer. curves away from the normal.

A mirage can also be observed at sea near the geographical pole.

image
In this area, the upper layers of air are hot
when compared to the lower layers of air.
The rays of light from a ship are refracted
air away from the normal until total internal
rm
wa reflection occurs. After that, the rays of
ship cool air light are refracted in a curve, bending
observer towards the normal until they reach the
eyes of the observer.
The image is inverted and appears to be
in the sky.

Light 280
1
sunlig
A rainbow ht refraction and dispersion Sunlight that passes through a
raindrop experiences two refractions
total internal
The formation of a rainbow is reflection
and one total internal reflection.
a phenomenon as a result of
refraction, dispersion and refraction
2
and dispersion
total internal reflection. The first refraction disperses the
sunlight into its component colours
white lig
ht while the second refraction causes
Sun from the red different drops
sun water
wate
er further dispersion.
orange in different
droplet
drop
plet
yellow positions send
green each colour to
red light 3
blue the observer The red light from the upper layers
from this drop FF
is sent to the
e indigo
violet and the violet light from the lower OO
observer violet layers of the raindrops are seen

5
RR
red separately at different angles.
M

CHAPTER
M
violet water
droplet
44
violet light 4
d The overall result is a curved, bow-
re from this drop
is sent to the shaped rainbow with the red on
observer the outer most edge and violet on
the lowest edge.
observer

Figure 5.49
5 49

Double Rainbows This is a secondary rainbow w and is formed when


Sometimes, a larger rainbow is seen outside a sunlight is reflected twice internally before emerging
rainbow and is parallel to it. from the raindrop.
second reflection
su
Secondary: nli
gh
violet on top, t
Primary: red
on top, violet red at bottom
raindrop
at bottom red
d
violet first reflection

As a result, the colours are in reverse order and are not


as bright as those in the primary rainbow.

Fish’s eye view surface acts as a perfect mirror, which allows the
fish and diver to see objects in the water and the
1 A fish or a diver is able to see an object above the objects around obstacles.
water surface because the rays of light from the 3 A fish sees the outside world within a 98-degree
object are refracted to the eyes of the fish or diver. cone. Beyond the 98-degree cone, total internal
2 Due to total internal reflection, part of the water reflection occurs and the fish sees light reflected
from the bottom of the pond.

refraction

49°° total
internal
98°
reflectio
on
reflection
fish
h sees
s th
the
outside world
within
hin a 98-degree cone
Figure 5.50

281 Light
from the top P
Prism periscope SPM
’06/P2 (A)
SPM
’04/P2 (A)
of the object First total
to
otal internal
Perpendicular rays of light 45°
45° reflecti on occurs.
reflection
P 45° enter the prism perpendicu- 45°
45°
larly, so there is no refraction m Q
from the bottom R
rays from O prism of the object
object occurs.
R
Q

T Rays of light exit


S
prism 45° W without refraction.

Seco total internal


Second S 45°
45 T
V
reflec
reflection occurs. W
4 °
45
45°
F
O Figure 5.51 Prism periscope V
5

R 1 The construction of a periscope is based on the 5 The image produced ed is upright, virtual
virtu
ual and is of
CHAPTER

M effects of a prism on rays of light. the same size as thehe object.


2 The periscope consists of two right-angled prisms 6 The advantages of the prism periscope
perisscope when
4
made of crown glass. F They are arranged as shown compared to a mirrorror periscope:
in Figure 5.51. O (a) The image is brighter because all the light
5

3 The critical angle ofR crown glass is 42º. energy is reflected.


cted.
CHAPTER

M
4 Total internal reflection takes place on the (b) The image is clearer
earer because no multiple images
hypotenuse of both prisms as the angles of are formed as in
n a mirror periscope.
4
incidence at these faces are 45º, which is greater 7 A periscope can bee used to view objects behind
than the critical angle of 42º. F obstacles. In submarines,
marines periscopes are used to
O observe ships on the surface of the sea.
5

R
CHAPTER

M
Applications
4 of total internal
Optical fibres SPM SPM
’03/P1 ’08/P2 reflection
cable bundle
cladding
(optically less core
sheath denser) (optically denser)

Figure 5.52
1 An optical fibre is a very thin, flexible rod made 7 These fibres are used to transmit information in
of special glass or transparent plastic with a telephone and data signals at high speed, thus
diameter of approximately 0.01 mm. enabling multiple telephone messages and data ta to
2 A cable of optical fibre consists of many fine optical be sent simultaneously.
fibres bundled together. 8 The advantages of using optical fibres es over
oveer
3 Rays of light entering one end of the fibre electrical wires in telecommunications:
undergo repeated total internal reflections until (a) Much more information can be transmitted mittted
they emerge at the other end of the fibre. This also because almost the entire light energy (which
hicch
happens in a bent fibre. carries information) undergoes total internal
rnaal
4 This property enables optical fibres to be used in a reflection in the optical fibre.
fibre-optic system such as an endoscope, a medical (b) There is no electromagnetic interference, e,
instrument that is used to view the internal parts of resulting in clearer connections.
the human body. (c) There is no electrical resistance.
5 A doctor can view an organ within the body via one (d) There is no danger of electrocution if a cable
end of the endoscope, which is inserted into the breaks.
body. (e) The fibres are cheap, light and can be easily
6 The use of optical fibres is also important in handled.
telecommunications.

Light 282
Prism binoculars Ray of light exits
observer without refraction.
focusing
wheel
Second total A
internal reflection.
piece
eyepiece First total internal
reflection.

B Third total internal


reflection.
B F
O
objective Fourth total Ray of light enters

5
lens internal reflection. R
perpendicularly.

CHAPTER
M
ray of light
Figure 5.53 Prism binoculars 4

1 The construction of the prism binoculars is final image which is upright and not laterally
also based on the effects of a prism on rays inverted.
of light. 6 Without prisms, the final image observed
2 Figure 5.53 shows a cross section of a pair of through a pair of binoculars (a telescope in
binoculars that reveals the two prisms inside it. normal adjustment) will be inverted.
3 A ray of light undergoes two total internal 7 The benefits of using prisms in binoculars:
reflections at prisms A and B respectively. (a) An upright image is produced.
4 The magnifying power of binoculars is made (b) The distance between the objective lens
enchanced by lengthening the path between and the eyepiece is reduced. This makes
Applications the two lenses. the binoculars shorter as compared to a
of total 5 The two prisms are arranged with their telescope which has the same magnifying
internal hypotenuses parallel but diagonally power.
reflection perpendicular to each other. This produces a

SPM
Sparkling
ng diamonds
d ’08/P2(C)

1 The critical angle for a diamond in air is about


24.4º.
2 This means that total internal reflection can occur
easily as the critical angle is small.
3 When a diamond is cut, the facets (faces) are
angled so that they produce total internal
reflection. Any ray which strikes the surface of the
(a) bottom facet at an angle greater than 24.4° will be
totally reflected back into the diamond.
incident
4 These rays eventually exit from the top surface after
ray critical angle multiple total internal reflections of white light.
This gives the diamond its ‘sparkle’.
5 The critical angle for diamond-water interface is
.4°
24

rnal greater than 24.4° (c = 33.3°).


l inte
tota ction 6 Therefore, a diamond placed in water is not as
refle
‘sparkling’ as in air since less total internal
(b) reflection occurs.

Figure 5.54

283
283 Light
5 SPM
Clone 5.3
’06

Figures (a) and (b) show two rays of light striking at 1 When total internal reflection occurs, the boundary
between the two mediums acts as a perfect
different angles of incidence at the flat glass-air
______________.
boundary of a semicircular glass block.
N N
2 Two conditions for the occurrence of total internal
reflection:
(a) The ray of light travels from an ____________
p
air air medium to a _____________ medium.
glass glass s (b) The angle of incidence _______________ the
q r critical angle.
3 The figure shows three rays of light from a light bulb
F placed underwater.
O
5

R (a)(I) (II)
(b)
CHAPTER

M
Which is the critical angle of the glass? 49°
4 A p F4/5/112 C r
F
B q O D s water
5

R
Comments light bulb
CHAPTER

M
Critical angle is the angle of incidence in the
4
optically denser medium when the refracted ray Given that the critical angle of water is 49°, complete
F4/5/150
the paths of the three rays of light.
travels along the boundary of the two mediums. In
the above diagrams, the angle is r. 4 Given that the critical angle of glass is 42°, complete
the following diagrams to show the path of each ray
Answer C after it strikes the prism.
(a) (c)

6 SPM
Clone 45°
’09 30°

The critical angle for a liquid is 47°.


Which diagram shows the occurrence of total
internal reflection? F4/5/153
(b) F4/5/154 (d)
A C
46° 46° 50° 50°
air air 42°
60°
liquid liquid

F4/5/151
5 DiamondF4/5/152
has a very high refractive index, n = 2.4.
B D
What is the critical angle of diamond?
air air 6 The figure shows the path of a ray of light in a glass
liquid liquid slab.
46° 46° 50° 50°

39°

Comments
The condition for occurrence of total internal
reflection in this situation are:
(1) light travels from liquid to air, and
(2) the angle of incidence, i  47°.
Answer D What is the refractive index of the glass?

Light 284
5.4 Understanding Lenses

Convex Lenses and Concave Lenses

1 Thin lenses are divided into


(a) convex lenses, (b) concave lenses.

Differences
Convex lenses Concave lenses

• Convex lenses are also known as converging • Concave lenses are also known as diverging
lenses or positive lenses. lenses or negative lenses. F
O
• Convex lenses are thicker in the centre than at • Concave lenses are thinner in the centre than at

5
R
the edge. the edge.

CHAPTER
M
• Cross-sectional views of common types of • Cross-sectional views of common types of
convex lenses: concave lenses: 4

• Concave lenses refract incident rays of light,


• Convex lenses refract incident rays of light, which are parallel to the principal axis, so that
which are parallel to the principal axis, so that they appear to diverge from a point located on
they converge to a point on the principal axis. the incident side of the principal axis. This point
This point is called the real focal point. is called the virtual focal point.

Common terminology of convex and concave lens

principal axis

focal
plane

(a) Convex lens (b) Concave lens

1 Optical centre, O
 The optical centre, O is the geometric centre of a lens. Rays of light travelling
through the optical centre pass through the lens in a straight line.
principal axis
2 Principal axis
 The principal axis is a straight line which passes through the optical centre, O
at right angles to the plane of the lens. focal
3 Focal point, F
 The focal point, F is a point on the principal
plane axis
(Principal focus) (a) to which incident rays of light travelling parallel to the principal axis, converge
after refraction through a convex lens.
(b) from which incident rays of light travelling parallel to the principal axis appear
to diverge after refraction through a concave lens.

4 Object distance, u Distance of the object from the optical centre, O.

5 Image distance, v Distance of the image from the optical centre, O.

6 Focal plane A plane through the focal point and perpendicular to the principal axis.

7 Focal length, f Distance between the principal focus, F and the optical centre, O.
From the above figures, focal length, f = OF.
The focal length of a convex lens is positive whereas the focal length of a concave
lens is negative (by convention).

285 Light
Power of Lenses SPM
’06/P2(B)

original path A thin lens is a lens whose


converged ray thickness is negligible
compared to its focal length.
principal axis F

f = 10 cm
Spherical lens and
(a) Convex lens cylindrical lens
A spherical plano-convex lens
ray is made by cutting through a
ed
i verg portion of a sphere.
F d
original path
O
5

R
CHAPTER

M principal axis F

4
f = 10 cm

(b) Concave lens A cylindrical plano-convex


Figure 5.55 lens is made by cutting
through a side of a cylinder.
1 The power of a lens is a measure of its ability to converge or diverge
an incident beam of light.
2 Power of a lens is defined as the reciprocal of the focal length, in
metres.

1 100
Power of a lens, P= or P =
f (in m) f (in cm)
How do lenses bend light?
3 The unit of power of a lens is m–1 or dioptre, D.
4 The power of a lens is 1 dioptre if its focal length is 1 metre.
5 The power of a convex lens is taken to be positive and that of a
concave lens is taken to be negative.

(a) Convex lens


14
Find the power of each of the following lenses.
(a) Convex lens, f = 20 cm
(b) Convex lens, f = 5 cm
(c) Concave lens, f = –50 cm (b) Concave lens
Solution Lenses are made of glass,
(a) Power, P (c) Power, P plastic, or other transparent
1 1 materials. Each section of a
= = lens can be thought of as
0.2 – 0.5
being made up of several tiny
= +5 D = –2 D prisms, refracting (bending)
(b) Power, P light as it goes in and again as
1 it comes out. Expensive
=
0.05 lenses have special coatings
to reduce the colour
= +20 D
spreading of the prisms.

Light 286
Activ To determine the focal point, F and investigate the relationship
ity 5.2 between the focal length, f and thickness of a thin lens

Apparatus/Materials 7 The optical centre, O is marked and the focal


Ray box, plate with 5 slots, two cylindrical convex length, OF is measured.
lenses (of different thickness), two cylindrical Results/Observation
concave lenses (of different thickness), white paper,
and a metre rule.
F F
Arrangement of apparatus f f
plate with 5 slots
(i) Thin lens (ii) Thick lens
ray box F
cylindrical (shorter focal length)
convex lens O
(a) Convex lens

5
F cylindrical R
concave

CHAPTER
f lens
M

4
F F

Figure 5.56 f f

(i) Thin lens (ii) Thick lens


Procedure (shorter focal length)
1 The ray box is adjusted to direct 5 parallel rays, (b) Concave lens
through the 5 slots in the plate, to the cylindrical Figure 5.57 Thick lens is able to bend rays of light more
biconvex lens. The rays of light are seen to
converge to a point on the principal axis, i.e. the
Discussion
focal point, F.
1 From the above observation, it can be seen that
2 The principal axis is drawn. The focal point, F
the thin lens has a longer focal length, f.
and the optical centre of the lens, O are marked.
1
3 The distance OF which is equal to the real focal 2 From the equation for power of a lens, P = ,
f (m)
length of the convex lens, f is measured.
4 The observation is repeated using a thicker it is found that a thin lens with a longer focal
cylindrical biconvex lens. length has a lower power.
5 The above steps are repeated with a cylindrical Conclusion
biconcave lens. The rays of light are seen to (a) A thin lens with a longer focal length, f has a
diverge from a point on the principal axis, i.e. the lower power.
focal point, F. (b) A thick lens with a shorter focal length, f has a
6 The divergent rays are marked and the biconcave higher power.
lens is removed. The rays of light are extended to (c) In the case of a convex lens, a thick lens has a
the back until they intersect at a point on the stronger converging effect, i.e., the incident
principal axis, i.e. the focal point, F. beam of light converges nearer to the lens.

Estimating the focal length of a convex lens The focal length of a convex lens can be estimated as follows.
convex lens (a) The convex lens is mounted on a holder and placed
screen on a table as shown in the figure.
(b) The lens is turned to face a distant object, say a tree.
distant (c) The position of the screen is adjusted until a sharp
Activity 5.2

object inverted image is formed on the screen.


f
(d) The distance of the screen from the optical centre of
lens holder
the lens is measured. This distance is the estimated
focal length, f.
How can the focal length of a lens, f be quickly estimated?

287 Light
(a) (b)
f F O F
F
principal axis
F'

focal plane f f

• All parallel incident rays at an angle to the principal axis • A lens has two focal points, one on each side of the
will focus on a point F ′ on the focal plane. lens since light can pass through the lens in either
F direction.
O • The two focal lengths are the same.
5

R
CHAPTER

Ray Diagrams

1 We can use ray diagrams to determine the position and characteristics of the images
formed by an object at any position.
2 Ray diagrams can be completed by drawing any two of three rays of light from the tip of the
object. The image is located at the point of intersection of the two rays drawn.

Rules for ray diagrams

Convex lens Concave lens


SPM
’05/P1

image
object O principal axis
object image O principal axis

front back
front back

1 A ray of light which passes through the optical 1 A ray of light which passes through the optical
centre, O of the lens does not deviate from its centre, O does not deviate from its path.
path. 2 A ray of light parallel to the principal axis is
2 A ray of light parallel to the principal axis is refracted and appears to come from the focal
refracted and passes through the focal point, F. point, F on the same side of the lens or diverge
3 A ray of light which passes through the focal from the focal point, F.
point, F is refracted parallel to the principal 3 A ray of light which travels towards the focal
axis. point, F on the other side of the lens is refracted
parallel to the principal axis.

Any two of the three rays 1 , 2 or 3 are


sufficient to determine the position and
characteristics of the image.

Light 288
Images Formed by Concave Lenses

1 Images formed by concave lenses do not depend on the position of Common eye diseases:
Myopia (short-sightedness)
the object with respect to the lens.
• Clear vision of near objects.
2 An image formed by a concave lens is always • Blurry vision of far objects.
(a) virtual,
(b) upright, Hypermetropia (long-
(c) smaller than the object, sightedness)
(d) located between the lens and the object, i.e. v < u, and • Blurry vision of near
objects.
(e) the image distance is less than the focal length, i.e. v < f.
• Clear vision of far objects.
(v = f when the object is at infinity)
3 Figure 5.58 shows two situations where an object placed at two Nowadays, many people
prefer to use contact lenses F
different positions, produces the same kind of image. O
rather than wear spectacles.

5
R
These are lenses placed in

CHAPTER
M
direct contact with the eyes.
2 4

object F image
observer

(a) Object distance, u > f

Contact lenses can give more


2
natural vision.
object

F image

observer

(b) Object distance, u < f


Figure 5.58

4 However, the image formed by concave lens approaches the lens as the
object approaches the lens. Figure 5.59 shows this effect.

O1 O2 O3
I3
I2
I1

2F F

Figure 5.59

5 At the same time, the image becomes larger but is still smaller than the
object.
6 The concave lens is used in spectacles for myopia (short-sightedness).

289 Light
Images Formed by Convex Lenses SPM
’04/P1
SPM
’05/P2(C)
SPM
’07/P1
SPM
’09/P1

Object Ray Position Characteristics


distance diagram of image of image

(a) Distant Image distance: • Real


object 1 v=f • Inverted
⇒u 3
• Diminished in size
F
= infinity F • On the opposite side of
image
the object

F
O
5

R (b) Object Image distance: • Real


CHAPTER

M
distance is object 2 f < v < 2f • Inverted
4 more than F 2F • Diminished in size
2f ⇒ u > 2f 2F F image • On the opposite side of
the object

(c) Object Image distance: • Real


distance is object 2 u = 2f • Inverted
equal to 2f F 2F • Same size as the object
⇒ u = 2f 2F F image
• On the opposite side of
the object

(d) Object Image distance: • Real


object
distance is v > 2f • Inverted
between f F 2F
• Magnified
and 2f 2F F
image • On the opposite side of
⇒ f < u < 2f the object

(e) Object image Image is at • Virtual


2
distance is infinity • Upright
equal to f object F v = infinity • Magnified
⇒u=f F • On the same side as the
object

observer

(f) Object Object distance: • Virtual


distance is v>u • Upright
image
less than f object F • Magnified
⇒u<f F • On the same side as the
object
observer

Light 290
Application

• Objective lens Concave lens (can form virtual images only)


of a telescope
objective
lens

F
O

5
• Lens of a camera R

CHAPTER
M
• Lens of the
human eye 4
As the object is gradually moved nearer to the
concave lens from infinity, the image also moves
nearer to the lens from F. At the same time, the image
become larger (but is still smaller than the object).

Convex lens (can form real or virtual images)


• Photocopying
machine

• Projector lens
• Objective lens of
a microscope 1 As the object is gradually moved nearer to the
convex lens from infinity to F, the inverted real
image moves away from the lens (starting from
F). At the same time, the image becomes larger.
2 When u = v = 2f, the image is of the same size
as the object.
3 As the object is gradually moved nearer to the
• Eyepiece of an lens from F, the upright virtual image also
astronomical moves nearer to the lens. At the same time, the
telescope image becomes smaller (but is still larger than
the object).
Keys:
eyepiece positions of object
positions of corresponding images

• Magnifying lens
• Spectacle lens General Information
to correct 1 The image formed on the opposite side of the
lens as the object is real and inverted.
hypermetropia
2 The image formed on the same side of the lens
(long-sightedness) as the object is virtual and upright.

291 Light
The upper portion of the inverted image disappears.
I
O

I
O
A convex lens is used to produce a real and inverted
image. What is the effect on the image produced
F
when the upper portion of the lens is covered by a
The size and position of the image are the same as
O card?
before. The brightness of the image, however, is
5

R
reduced.
CHAPTER

4 SPM
Linear Magnification ’06/P1
15
u v v
P
S
P u
An object 3 cm in height is placed at a distance of
6 cm from a convex lens. A real image is formed at a
R O I R O distance of 20 cm from the lens. Calculate
(a) the linear magnification, and
S (b) the height of the image.
(a) Real image (b) Virtual image
Solution
Figure 5.60
v 20
Take note that in both situations, ∆OPR is similar (a) Linear magnification, m = =
u 6
to ∆OSI.
1
=3
∴ IS = OI 3
RP OR
Image height
Image height, IS = hi (b) m =
Linear magnification, m = Object height
Object height, RP ho
20 Image height
OR =
6 3
Image distance, OI
Linear magnification, m = =v ∴ Height of image = 3 ×
20
Object distance, OR u 6
hi v = 10 cm
∴ m= =
ho u

Linear magnification, m = v .
u object object

object v v
v u u
u
image image

image
(b) If v = u, then m = 1 (c) If v < u, then m < 1
(a) If v > u, then m > 1 ⇒ Image is the same size as the ⇒ Image is diminished in size.
⇒ Image is magnified. object.

Light 292
16
Draw a ray diagram on a piece of graph paper to Scale: Horizontal axis – 1 cm : 10 cm
determine the characteristics of the image, image Vertical axis – 1 cm : 5 cm
distance, and height of the image of the following Image distance, v = 6 × 10 cm = 60 cm
objects. Use a suitable scale for the diagrams. Height of image = 4 × 5 cm = 20 cm
(a) An object 10 cm high on the principal axis is The image is real, inverted, and magnified.
placed at a distance of 30 cm from a convex lens
(b)
of focal length 20 cm.
(b) An object 20 cm high on the principal axis is
placed at a distance of 50 cm from a concave lens
of focal length 20 cm.
F
Solution F I O

5
R
(a)

CHAPTER
M

1 cm
4

1 cm
I
O F F
Scale: Horizontal axis – 1 cm : 10 cm
Vertical axis – 1 cm : 10 cm
Image distance, v = 1.4 × 10 = 14 cm
1 cm Height of image = 0.6 × 10 = 6 cm
1 cm
The image is virtual, upright, and diminished in
size.
F

Lens Equation (Relationship between u, v and f )


1 For a thin lens (a lens whose thickness is negligible compared to its focal length), the object
distance, u, image distance, v, and the focal length, f, are related by the equation:

1 + __
__ 1 = __
1
u v f

2 When applying the equation, take note of the following:

Convex lens Concave lens


Object distance, u • Positive (+) • Positive (+)
Image distance, v • Positive (+) • Negative (–)
⇔ Real image ⇔ Virtual image
• Negative (–)
⇔ Virtual image
Focal length, f • Positive (+) • Negative (–)
Example: f = 10 cm Example: f = –15 cm

3 The object distance, u, for a convex lens and a concave lens is always positive (at SPM level).
4 The image distance, v, for a concave lens is always negative (at SPM level).
5 The lens equation can be used to determine the position of the image and the linear
magnification.

293 Light
17
An object of height 3 cm is placed 3 × 15
∴ Height of image = = 1.5 cm
(a) at 30 cm, 30
(b) at 5 cm The image is real and at a distance of 15 cm from
from a convex lens of focal length 10 cm. the lens on the opposite side of the object. The
Determine the position and size of the image in height of the image is 1.5 cm.
each case. (b) u = 5 cm, f = 10 cm, v = ?
1 1 1 A negative sign
Solution + = shows that the
5 v 10 image is virtual.
(a) u = 30 cm, f = 10 cm, v = ?
1 1 1 1 1 1 –1
+ = = – =
F
u v f v 10 5 10
O
1 1 1 3–1 v = –10 cm
5

R = – =
v 10 30 30 Image height
CHAPTER

M 10
2 1 m= =
= = Object height 5
4
30 15 10
∴ Height of image = × 3 = 6 cm
∴ v = +15 cm 5
Image height v The image is virtual, 10 cm from the lens on the
m= =
Object height u same side as the object and 6 cm high.

18 19
An object is placed at a distance of 30 cm from a A light bulb is placed to the left of a lens of focal
concave lens of focal length 15 cm. The image length 20 cm. The image formed is enlarged 5 times
formed has a height of 2 cm. Determine the position and it is to the right of the lens.
of the image, the linear magnification, and the height (a) What is the type of lens used?
of the object. (b) What is the object distance?
Solution
Solution
(a) A convex lens, because the image is formed on
u = 30 cm, f = –15 cm (concave lens), v = ? the opposite side of the lens as the object–that is,
1 1 1 the image is real.
+ =
u v f v
(b) Linear magnification, m =
1 1 1 u
+ =
30 v –15 v
= 5
1 1 1 (b) Linear magnification, u
= –
v –15 30 ∴ v = 5u
–3 –1 1 1 1
= = + =
30 10 u v f
v = –10 cm 1 1 1
v 10 1 + =
Linear magnification, m = = = u 5u 20
u 30 3 5+1 1
Image height =
= m 5u 20
Object height 6 1
=
2 1 5u 20
=
Object height 3 5u = 20 × 6
∴ Height of object = 3 × 2 120
∴ Object distance, u = = 24 cm
= 6 cm 5

Light 294
Activ To investigate the relationship between the object distance (u),
ity 5.3 image distance (v), and focal length (f ) of a lens
Apparatus/Materials Procedure
Cardboard with a cross-wire in a triangular cut-out, 1 The apparatus is set up as shown in Figure 5.61.
light bulb, lens holder, convex lens, metre rule, and 2 The focal length of the lens as supplied is
white screen. 1
recorded and the value of is calculated.
f
Arrangement of apparatus 3 The object is placed at a distance of approximately
3f from the lens. The screen is moved back and
sharp image
cardboard convex forth until a sharp image is formed on it.
cross-wire with
triangular
lens 4 The object distance, u and the image distance, v F
white
light bulb cut-out screen are measured and recorded. O

5
5 The object distance, u is decreased in steps until u R

CHAPTER
M
e
imag ce,
is approximately 1.2f. Six sets of values of u and v
n
dista are obtained and tabulated in a table. 4
ct v
obje nce,
d a t 1 1
is
u 6 The respective values of and are calculated.
u v
to power supply
1 1
Figure 5.61
7 A graph of v against u and a graph of against
u v
are drawn. The intercepts on both axes, which
commence from the origin, are determined for the
second graph.

Results and tabulation of data


Focal length of the lens, f = ______ cm
1
= ______ cm–1
f
Experimental data:

Object distance, Image distance, 1 1 1 1


(cm–1) (cm–1) + (cm–1)
u (cm) v (cm) u v u v
30.0
25.0
20.0
15.0
12.0

Graph
(a) Graph of v against u: 1 1
(b) Graph of against :
v u
v (cm)
1 –1
v (cm )

p
Activity 5.3

u (cm)
(0, 0)
1 (cm–1
)
(0, 0) q u
Figure 5.62
Figure 5.63

295 Light
Discussion linear equation y = mx + c, (where gradient,
1
1 It is found that the value of ( u1 + 1v ) is a m = –1 and c = p = ):
f
1 1 1
constant, which is equal to the calculated value = (–1) +
1 1 1 1 v u f
of , i.e., + = .
f u v f 1 1 1
∴ + =
1 1 u v f
2 (a) The graph of against is a straight
v u Conclusion
line with a gradient of –1. 1 From the table and the graph of v against u
(b) The value of p, the intercept on the (Figure 5.62), it is found that the image
y-axis and the value of q, the intercept on the distance, v, decreases as the object distance, u,
F 1 increases.
O
x-axis are each equal to the value of .
f 2 The relationship between u, v, and f:
5

R
1 1 1 1 1
CHAPTER

M (c) From the graph against , in terms of the + =


v u u v f
4

To focus an image means to adjust the position of either The figure shows a straight wire placed along the
the object, lens or screen so as to form a sharp image principal axis of a convex lens of focal length
on the screen. After focusing, the distance between the 10 cm. P and Q are at 20 cm and 15 cm respectively
lens and the screen is the image distance, not the focal from the optical centre of the lens.
length. The image distance is equal to the focal length
only when the object is a distant object.
ant

straight wire
20 SPM
’08/P1
P Q

A student conducted
an experiment using
a convex lens. An ant takes 30 seconds to travel from P to Q. What is
F
The graph obtained the speed of the image of the ant?
is as shown in Figure
Solution
5.64.
What is the focal When the ant is at P:
length of the convex The position of its image is 20 cm from the optical
centre (u = 2f, v = 2f ).
lens?
When the ant is at Q:
Figure 5.64 1 +1 = 1
15 v 10
Solution
1 = 1 – 1
1 1 1 v 10 15
From the formula: + =
u v f
1 1 1 = 3–2 = 1
When = 0, = From the graph, 30 30
u v f
1 v = 30 cm
1 when = 0,
0.1 = u The distance travelled by the image of the ant in
f 1 30 seconds = 30 – 20 = 10 cm
Activity 5.3

= 0.1
1 v
f = Speed = Distance
0.1 Time
= 10 cm
Speed = 10 = 1 cm s–1
The focal length of the convex lens is 10 cm. 30 3

Light 296
SPM SPM SPM
The Uses of Lenses in Optical Devices ’04/P2(A) ’08/P1 ’09/P2(A)

Magnifying Glass (Simple Microscope)

P'

F
image F object
u<f magnifying glass

F
O

5
virtual, upright and Figure 5.65 R
magnified image

CHAPTER
M

Near Point and Far Point 4


1 A convex lens acts as a magnifying glass (simple microscope) when • The near point, D is the
the object is placed at a distance of less than the focal shortest distance from
length, f. the eye that can focus the
2 The two rays from the top of the object, P do not converge after passing image of an object on the
retina. The average value of
through the lens. However, when extended backwards, the two rays the near point is 25 cm for
intersect at P′ to form the image of P. an adult with normal vision.
3 The same principle applies to every point of the object. However, this distance
A virtual, upright and magnified image is formed. increases with age.
4 The magnification, M of the magnifying glass is given by: • The far point is the
greatest distance from
Image distance the eye that the image of
M= an object can be focused
Object distance
on the retina. The far point
of the normal eye is infinity.

7 SPM
Clone
’11

An object is placed in front of a convex lens as shown in the diagram.

Object

2F F F 2F

What are the characteristics of the image formed?


A Virtual, upright, enlarged C Real, inverted, same size
B Real, inverted, diminished D Real, inverted, enlarged
Comments
When an object is placed in between f and 2f, the image formed is real, inverted and enlarged.
Answer D

297 Light
Compound Microscope

3
objective eyepiece eyepiece
lens construction
1 line
2
object Fo Fe I1 Fe
2Fo Fo objective
lens
final fo observer
virtual
image
I2 fe

4
F Figure 5.66
O
5

R
Definition How it works
CHAPTER

4 A compound microscope is an optical instrument


used to view very small or fine objects. 1
An object is placed at a distance that is slightly
further than the focal point of the objective
lens, i.e. between fo and 2fo.

Structure
1 A compound microscope consists of two convex
lenses of high power. 2
(a) The lens nearer the object is called the The first image, I1 formed by the objective lens
objective lens, with focal length, fo. is real, inverted, and magnified. The first
(b) The lens nearer the eye is called the image becomes the object for the eyepiece.
eyepiece, with focal length, fe, where fo < fe.
2 The objective lens has a higher power as it has
a shorter focal length compared to the eyepiece.
3 The total distance between the objective lens and
the eyepiece should be greater than ( fo + fe). 3
The eyepiece functions as a magnifying lens.
The eyepiece is adjusted so that image, I1 is
positioned at a distance less than fe and hence
the eyepiece which acts as a magnifying lens
further enlarges the image I1.
Magnification, M

Height of image I2, hi


M=
Original height of object, ho
Height of Height of 4
image I1, h image I2, hi The final image, I2 is virtual, inverted with
= ×
Original height Height of respect to the object, and much larger than the
of object, ho image I1, h object. The eyepiece is normally adjusted so
that the final image is at the near point (about
∴ M = mo × me
25 cm) from the observer’s eye for a clearer
where mo is the magnification produced by the
view.
objective lens and me is the magnification produced
by the eyepiece.

Light 298
Astronomical Telescope SPM
’05/P2(C)
SPM
’06/P2(B)
SPM
’09/P2(A)

focal
plane
objective lens f eyepiece
rays of o fe
light c objective
1 from a construction line lens
distant Fo eyepiece
object b Fe 2
Fo Fe observer
I1

is
, I2
im age nfinity
l i
fina ed at
3 form
F
Figure 5.67 O

5
R

CHAPTER
M
Definition How it works
4
An astronomical telescope is an optical instrument
that is used to view objects at a great distance, 1
such as planets and stars. Parallel rays of light from the distant object
converge at the focal plane of the objective lens
to form the first image, I1 which is real,
inverted, and diminished.

2
Structure The first image, I1 becomes the object for the
1 An astronomical telescope consists of two eyepiece. The first image, I1 is at the focal
convex lenses. The objective lens whose power plane of the eyepiece and hence the final
is low has a long focal length, fo. The eyepiece image, I2 is virtual, inverted, and magnified
whose power is high has a short focal length, fe. with respect to the original object.
Therefore, fo > fe.
2 The distance between the objective lens and the
eyepiece is (fo + fe), i.e. the focal point of the
objective lens, Fo is coincident with the focal 3
point of the eyepiece, Fe. Both lenses have the The final image, I2 is formed at infinity.
same focal plane.

Why are a low powered lens and a high powered lens


used in an astronomical telescope?

Magnification Solution
fo
The magnification of the telescope in a normal From the formula: M = (where fo > fe)
fe
adjustment, i.e. with image formed at infinity, is
given by: The value of M is large if fo is large. The objective lens
Focal length of objective lens, fo must have a low power so that fo is large. Conversely, M
M= is large if fe is small. Therefore, the eyepiece must have
Focal length of eyepiece, fe a high power so that fe is small.

299 Light
Comparison between an astronomical telescope and a compound microscope.
Similarities

• Consists of two convex lenses.


• The first image is real, inverted, and acts as the object for the eyepiece.
• The eyepiece acts as a magnifying lens.
(For an astronomical telescope in normal adjustment, the first image is at the common focal plane of the
objective lens and the eyepiece.)
• The final image is virtual, inverted, and magnified.
F
O
5

R Differences
CHAPTER

M
Aspects Compound microscope Astronomical telescope
4
Type of lens Two high powered convex lenses A low powered convex lens and a
high powered convex lens
Focal length fo < fe fo > fe
First image First image is magnified First image is diminished
Position of final image At the near point of the observer’s eye At infinity
Distance between lenses Greater than fo + fe Equal to fo + fe
fo
Linear magnification, M M = mo × me M=
fe

Condition of the eye when using Ciliary muscles are contracted. Ciliary muscles are at ease and the
the instrument Eye is strained. eye is relaxed.

Method of drawing a ray diagram for a microscope. Step 2


Step 1 eyepiece
objective
lens construction
eyepiece 3 line
1 Fo Fe Fe
objective lens
O Fo
2 observer
3
1 Fo Fe Fe
4
O Fo I1
2
Draw the construction line  from the top point of the first
image I1 through the optical centre of the eyepiece. Extend
ray  and the construction line  until they intersect
where the top point of the final image I2 is formed. The
Show the positions of Fo and Fe on both sides of each
construction line is drawn without an arrow. It is the
lens. Draw ray  through the optical centre of the
construction line necessary to locate the final image, I2.
objective lens until the eyepiece. Then draw ray 
through Fo, refracted parallel to the principle axis, refracted
through Fe before entering the eye. Then, complete ray Step 3
. The intersection point of rays ,  and  is the top Complete rays  and . The extended lines for rays 
point of the image I1. and  should meet the intersecting point of ray  and
construction line .

Light 300
Method of drawing a ray diagram for an astronomical telescope.
Figure 5.67 is referred to.

Step 1 Show the positions of Fo and Fe on both Step 3 Draw ray ,


c which is parallel to 
a and ,
b to

sides of each lens. Draw ray a through the be refracted by the objective lens to pass
optical centre of the objective lens until it through the point of intersection of a and

meets the eyepiece. .


b

Then draw ray  b through F , refracted and


Step 2 o
Step 4 Draw rays  a and  c refracted by the
travels parallel to the principle axis before eyepiece and travel parallel to ray b before
being refracted by the eyepiece and travels entering the eye. Extend rays ,a b and 
c to
through Fe. infinity.

Camera F
O

5
R

CHAPTER
M
Focusing ring
4
Focusing of the image on the film is done by adjusting the focusing ring.
very closer
distant object
object film
F F
F

focal length, f increased distance

(a) For a very distant object, the focusing ring adjusts the (b) For a closer object, the focusing ring adjusts the
lens towards the film so that the film is located at the lens away from the film so that v > f.
focal point, F of the lens.

Convex lens
A convex lens is used to form an
image which is real, inverted, and
diminished in size, on the film and image
in this respect, the object distance object
screw
is more than twice the focal length
of the camera lens. Film
Figure 5.68
Light sensitive film is
contained in a light-
Diaphragm tight compartment at
The diaphragm controls the diameter of the the back of the
aperture, which also determines the amount camera.
of light passing through the lens.
Shutter
diaphragm
When the button is pressed, the shutter opens to
allow light from the object to pass through the lens
to the film. The amount of light passing through the
lens is controlled by the time of exposure. For
moving objects, a short exposure time is required.
wide aperture narrow aperture

301 Light
Slide Projector

A slide projector is an optical device which consists of several convex lenses and a concave mirror.
screen
fan concave condenser
motor mirror lens

4
The slide or film,
which acts as the
object of the
bulb slide or film is projection
projection lens is at
F
2 inserted here lens a distance between f
O The mirror and 2f from the
heat filter
5

reflects the rays, concave projection lens. The


R F4/5/206a
mirror slide
which pass projection
CHAPTER

M or slide must be
bulb lens
through the film placed in an
4
optical centre inverted position
towards the (upside down) in
condenser lens. inverted order to obtain an
object magnified, upright image on
r condenser real and
lens upright image the screen.

1 F Figure 5.69
A bright bulb is placed at the centre of O 6
5

R F4/5/206b The image on the screen is


curvature of the concave mirror, so thatM
CHAPTER

rays of light from the bulb fall normally on real, upright, and
the surface of the concave mirror. 4
magnified.

3 5
The condenser consists of two plano-convex lenses, with a heat The projection lens forms the image
filter in between. The heat filter is required to protect the slide or on the screen. It is mounted in a
film from the heat of the bulb. The direct light from the bulb and sliding tube. The position of the
the reflected light from the concave mirror are combined and projection lens must be adjusted until
converged evenly onto the whole of the slide, so that the slide is a sharp image is formed on the
brightly and evenly lit. screen.

21
A student stands at a distance of 1.6 m in front of a camera with a focal length
of 5.0 cm. To obtain a sharp image on the film, how far must the lens be from
the film?
Solution
1 1 1
+ =
u v f
1 1 1
+ =
160 v 5
1 1 1 32 – 1 31
= – = =
v 5 160 160 160
v = 5 .16 cm
The distance between the lens and the film is 5.16 cm.

Light 302
22
Sharif observes an ant with the help of a magnifying lens ∴ The distance of the ant from the
of focal length 20 cm. The magnifying lens is held near 1
magnifying glass is 11 cm.
his eyes. If the image of the ant is at a distance of 25 cm 9
from the lens, what is the distance of the ant from the v
magnifying lens and what is the magnification? Magnification, M =
u
Solution 25
=
1 1 1 1
+ = 11
u v f 9
1 1 1 9
+ = v = –25 = F
u –25 20 The image 4
formed is virtual. O
1 1 1 1

5
= + =2 R
u 20 25 4

CHAPTER
M
5+4 9 1
= = ∴ The image of the ant is magnified 2 times. 4
100 100 4
1
u = 11 cm
9

23
The height of a person on a slide is 1.5 cm. The slide is placed at a distance of 16 cm from a projection
lens of focal length 15 cm. What is the height of the image of the person on the screen?
Solution
1 1 1 v Height of image
+ = =
u v f u Height of object
1 1 1 240 Height of image
+ = =
16 v 15 16 1.5
1 1 1 16 – 15 1 240 × 1.5
= – = = ∴ Height of image =
v 15 16 240 240 16
v = 240 cm ∴ Height of image = 22.5 cm

Constructing a Simple Optical Device that uses Lenses

Activ To construct a simple astronomical telescope


ity 5.4
Apparatus/Materials Arrangement of apparatus lamp at far
end of the
Convex lens of large diameter with a power of objective laboratory
+2.5 D, convex lens of small diameter with a power lens
of +20 D, translucent paper, and a lamp.
translucent
paper
Procedure
1 A convex lens of power +2.5 D (fo = 40 cm) is eyepiece
Activity 5.4

used as the objective lens.


2 A convex lens of power +20 D (fe = 5 cm) is
used as the eyepiece.
observer

Figure 5.70
obser
303 F4/5/207 Light
3 A bright lamp is mounted on a table at the end of objective lens
the laboratory. Manila frame
4 The objective lens and eyepiece are each
mounted on a lens holder which can slide on an
eyepiece objective
aluminium bar. lens
5 The objective lens is pointed in the direction of cardboard
the bright lamp. tube
eyepiece
6 The position of the translucent paper is adjusted
to receive a sharp image of the lamp. Manila frame
7 The eyepiece is moved by sliding it along the
aluminium bar until the first image on the (a) (b)
translucent paper can be seen clearly. Figure 5.71
F 8 The translucent paper is removed. The (b) The length of the tubes should be
O apparatus that has been set up can function as an 40 + 5
5

R
astronomical telescope. ( ) + 2 cm (i.e. average of the two focal
2
CHAPTER

M
9 You need to shift the position of your eye until
lengths plus 2 cm).
4 you get a wide field of view. This is the position
(c) The objective lens and eyepiece are each
of the eye ring of the telescope.
glued to a frame made of Manila card with a
10 This apparatus can be used outside the
circle slightly less than the diameter of the
laboratory to view distant objects.
lenses. You may need masking tape to fix the
11 The distance between the lenses are measured
lenses to the Manila frames.
and recorded. The distance is normally equal to
(d) The framed lenses are glued to the end of the
the sum of the focal lengths. F
O telescope: tubes as shown in Figure 5.71(b). An
12 The actual setting up of an astronomical
5

R astronomical telescope is thus constructed.


(a) Two cardboard tubes with the smaller one
(e) You can slide the two tubes in and out until
CHAPTER

M
able to slide in the larger one are used. The
you have a clear image.
tubes are the body of the telescope.
4

Activ To construct a compound microscope


ity 5.5
Apparatus/Materials 2 A piece of graph paper, which is lit up by a
Convex lens of large diameter with a power of +20 D, bright lamp is used as the object.
convex lens of small diameter with a power of +14 D, 3 The objective lens and eyepiece are each
translucent paper, graph paper, and a lamp. mounted on a lens holder which can slide on an
aluminium bar.
Arrangement of apparatus 4 The position of the translucent paper is adjusted
graph paper
to receive a sharp image of the graph paper.
translucent 5 The eyepiece is moved by sliding it along the
paper objective
eyepiece aluminium bar until the first image on the
lens
observer
translucent paper can be seen clearly.
6 The translucent paper is removed. The
apparatus that has been set up can function as a
compound microscope.
7 You need to shift the position of your eye until
Activity 5.4 & 5.5

you get a wide field of view. This is the position


Figure 5.72 for the eye ring of the microscope.
8 The distance between the two lenses is measured.
Procedure The distance between the lenses is always greater
1 A lens of power +20 D (i.e. fo = 5 cm) is used as than the sum of the two focal lengths.
lamp
the objective lens and a lens of power +14 D 9 The actual setting up of an compound
(i.e. fe = 7.1 cm) acts as the eyepiece. microscope:

Light 304
objective lens The construction of a compound
Manila frame microscope is similar to that of
an astronomical telescope except
the length of the tubes should be
eyepiece
2 × 7.1 cm = 14.2 cm (double the
objective
focal length of the eyepiece).
cardboard
lens
tube

Manila frame eyepiece

(a) (b)
Figure 5.73
F
O

5
R

CHAPTER
M

To find the magnification of a telescope 1 Draw six equally spaced lines on a green board and 4
label the lines from 1 to 6.
2 With one eye looking at the lines through the
telescope and the other eye looking directly at the
view through group of lines, a view as shown in the figure is
a telescope
obtained.
3 The space between lines 2 and 3 is compared with
the line spacing on the green board.
4 The magnification of the telescope is 5.

What is the effect on the image produced when the From the formula:
objective lens and the eyepiece in an astronomical f
M = o (where fo > fe)
telescope are interchanged? fe
(The distance between the lenses is maintained.)
Interchanging the lenses will result in fo (new) < fe (new)
Solution and the final image will be diminished.
The final image is virtual, inverted, and positioned at infinity.

8 SPM
Clone
’06

An image is formed by a convex lens as shown in the Solution


diagram below. You need to find the image distance first, which is not
9 cm equal to 30 cm.
v = 30 cm – 9 cm
= 21 cm
3 cm
v height of image
object Applying = :
u height of object
image
21 height of image
=
30 cm
9 3
Activity 5.5

3  21
If the object is of height 3.0 cm, what is the height of Height of image =
9
the image? F4/5/169
A 10.0 cm C 7.0 cm = 7.0 cm
B 8.6 cm D 3.3 cm Answer C

305 Light
24 5.4

The distance between two lenses of an astronomical 1 A pencil 6 cm in length is placed vertically at a
telescope in a normal adjustment is 85 cm. If the distance of 40 cm from a convex lens of focal
power of the objective lens is 1.25 D, what is the length 10 cm.
power of the eyepiece? (a) Calculate
(i) the position of the image,
Solution (ii) the height of the image.
Distance between 2 lenses = fo + fe = 85 cm. The student (b) State the characteristics of the image formed.
must determine the focal length of the objective lens, 2 The figure shows the formation of an image by a
fo and then determine fe by finding the difference. convex lens. The object distance is 20 cm and the
1
Power of objective lens, Po = distance between the object and the image is 50 cm.
F fo 50 cm
O
1 1
fo =
5

R = = 0.80 m = 80 cm
Po 1.25
CHAPTER

M object
fe = 85 – 80 = 5 cm = 0.05 m 20 cm image
4
1 1
∴ Power of eyepiece, Pe = = = 20 D
fe 0.05
Calculate
(a) the linear magnification,
9 SPM
Clone
(b) the focal length of the lens.
’09
3 The figure shows a beam of light passing through
A student is given an eyepiece of focal length 5 cm two convex lenses. The distance between the two
and four objective lens P, Q, R and S as shown in the lenses is 45 cm.
table below. 45 cm
P R
Lens Focal length Magnification Diameter of the
of lens, f objective lens
m= o
fo (cm) fe (cm)

P 10 2.5
Q 10 5.0 40 cm

R 50 2.5 What is the power of lens R?


S 50 5.0 4 A convex lens with a focal length of 15 cm is used
to form an image which is inverted and is 3 times
(a) Calculate and fill in the values of m for lenses P, the size of the object. What is the object distance?
Q, R and S.
(b) Which two lenses produce the largest image? 5 An object is placed at a distance of 9 cm from a
Explain why. concave lens with a focal length of 12 cm.
(c) Which two lenses produce the brightest image? (a) Calculate
Explain why. (i) the position of the image,
(d) Hence, choose the most suitable objective lens for (ii) the linear magnification.
construction of an astronomical telescope. (b) State the characteristics of the image.
6 An object is placed at a distance of 20 cm from a
Solution lens. The image formed is upright and half the size
10 of the object.
(i) For P and Q, m = =2
5 (a) What type of lens is used?
50 (b) What is the focal length of the lens?
For R and S, m = = 10
5 7 Rajan intended to build an astronomical telescope
(ii) Lens R and lens S as both have long focal length for use at a normal adjustment. The two lenses to
(to enable the telescope to have a magnification be used have a power of 2.5 D and 14 D
of 10). respectively.
(iii) Lens Q and lens S as both have large diameter to (a) Which lens is used as the objective lens?
allow more light to enter into the telescope. (b) Calculate the distance between the lenses and
(iv) Lens S. the magnification of the telescope.

Light 306
1. Laws of reflection: (b) Angle of incidence, i  the critical angle, c
(a) The incident ray, the reflected ray and the normal 1
8. Refractive index, n =
all lie on the same plane. sin c
(b) The angle of incidence, i, is equal to the angle of 9. Focal length, f
reflection, r. = Distance from optical centre to focal point
2. Characteristics of the image formed in a plane mirror: 1
(a) Same size as the object 10. Power of a lens, P =
f(in m)
(b) Image distance = Object distance
11. Images formed by a convex lens:
(c) Virtual and upright
(a) For u  f: real and inverted F
(d) Laterally inverted O
(b) For u  f: virtual and upright
3. Images formed by a convex mirror:

5
12. Images formed by a concave lens: R
virtual, upright and diminished

CHAPTER
M
virtual, upright and diminished
4. Images formed by a concave mirror:
v
(a) For u  f: real and inverted 13. Linear magnification, m = 4
u
(b) For u  f: virtual and upright
5. Refraction is the bending of light when it passes from 14. Differences between an astronomical telescope and a
a medium to another. This is due to the change in compound microscope:
velocity of light when it passes from one medium to
another. Astronomical Compound
Aspects
6. Refractive index of a medium, n telescope microscope
sin i Type of lens Low powered High powered
=
sin r objective lens objective lens
c and high and eyepiece
=
v powered eyepiece
Real depth, D Distance
= = fo + fe  (fo + fe)
Apparent depth, d between lenses
7. Conditions for occurrence of total internal reflection: Linear fo
(a) A light ray passes from a optical denser medium magnification M= M = mo × me
fe
to a less dense medium

5
Multiple-choice Questions
between the incident and Which ofl the following diagrams
5.1 Reflection of Light reflected ray is 120°?
l shows the correct observation in
1 Diagram 1 shows a light ray travelling A 30° C 60° the mirror?
perpendicular to a plane mirror. B 45° D 120° A l l
l l
2 Diagram 2 shows a number card
placed vertically on a plane mirror. B
l l
card F4/5/218
l l l C
Diagram 1
plane l l F4/5/218
mirror
What is the angle of rotation D
l
l N so that the angle
about Diagram 2
l l
l l
307 Light
F4/5/218 l l
F4/5/218

F4/5/21
3 Diagram 3 shows 5.2 placed vertically at the centre of
the reflection of a Refraction of Light the glass. Diagram 6(b) shows the
ray of light on a 8 Diagram 4 shows a ray of light side view as seen by an observer.
plane mirror. travelling from glass to air at an
angle of incidence of 40°. water pencil
pencil water
Diagram 3 glass
glass
p
What is the angle of incidence? air A B C
A 30° C 50° glass (a) (b)
B 40° D 130° 40°
Diagram 6
4 Which of the following is not a Which position, A, B or C does the
F characteristic of an image in a observer stand?
O plane mirror? Diagram 4
12 Diagram 7 shows a light ray passing
5

R A Upright Determine the value of p.


B Inverted through mediums P, Q and R.
CHAPTER

M
[Refractive index of glass = 1.5]
C Virtual A 74.6° C 30.0°
4 D Same size as the object B 60.0° D 15.4° 60° medium P

5 An object is placed at a distance of 9 In Diagram 5, a thick line is drawn 130° medium Q


20 cm in front of a concave mirror. on a piece of white paper. A glass
A sharp image is formed on the 120° medium R
block of a thickness of 12 cm is
screen next to the object. What is placed on it. The line seemed to
the focal length of the concave Diagram 7
rise by 3.8 cm.
mirror? Which of the following is true
A 5 cm C 15 cm regarding the refractive indices of
B 10 cm D 20 cm P, Q and R?
A nP  nQ  nR
6 In which of the following mirrors
B nP  nQ  nR
are the images always virtual and
C nP = nR  nQ
diminished?
D nP = nR  nQ
A A convex mirror
B A plane mirror
C A concave mirror
5.3 Total Internal Reflection
7 Which diagram correctly shows
SPM
Clone the reflection of light from a Diagram 5 13 A ray of light strikes the surface of an
’08 concave mirror? ice prism as shown in Diagram 8.
What is the refractive index of the
A glass?
A 1.46 C 2.16
45° ice
F B 1.52 D 3.16 ray of light prism

10 The refractive indices of water and


Clone glass are 1.3 and 1.5 respectively.
SPM

’08 Based on the above information,


B Diagram 8
which light ray, A, B, C or D in the
diagram is correct? If the critical angle of ice is 50°,
F
which of the following diagrams
shows the correct path of the ray
normal
of light?
C A C
F water
glass

D
D A C
B B D
F
11 Diagram 6(a) shows the top view
of a glass of water with a pencil

Light 308
14 A ray of light strikes a surface of a 18 The image of a distant building is A Between P and R
transparent prism formed by a convex lens on a B Between Q and R
normally. screen placed 10 cm away. A card is C Between R and S
The onward 40° then placed right in front of the lens
23 A lit lamp with its shape as shown
path of the ray to cover 25% of the area of the
in Diagram 13 is placed in front of
of light is lens. What happens to the image?
a convex lens.
shown in A 25% of the image will
convex
Diagram 9. disappear. lamp lens
Diagram 9 B The new image is 7.5 cm from
the lens.
What is the refractive index of the
F4/5/227 C The image is still complete but screen
transparent material? less bright.
A 1.15 C 1.56 D The image is still complete but
B 1.31 D 1.63 its sharpness is reduced by F
15 The refractive index of ‘flint’ glass 25%. observer O

5
R
is 1.62. What is its critical angle? 19 The distance between the object Diagram 13

CHAPTER
M
A 33.6° C 41.8° and the image formed by a
B 38.1° D 48.3° The lamp, lens and screen are
F4/5/185
convex lens with a linear parallel to one another. 4
16 The paths of a light ray in magnification of 1 is p cm. What is A sharp image is formed on a
mediums P and Q with refractive the focal length of the lens? screen placed at the opposite side
indices, n and n are shown in 1 1 of the lens. Which of the following
A p C p
Diagram 10. 8 4 is the correct image on the screen
1 1 as seen by an observer?
B p D p
5 2 A C

20 Two rays of light pass through a


convex lens as shown in Diagram 11.

B D
B C
A
D
Diagram 11
Diagram 10 24 Table 1 shows lenses P, Q, R and
Which of the points, A, B, C or D, S with their respective focal
Which of the comparisons is
is the focal point? lengths.
correct?
A n  n C n  n 21 An object is placed in front of a Lens P Q R S
B n = n concave lens with a focal length of
Focal length,
20 cm. The image formed is 5 cm
f (cm) 5 7 20 40
in front of the lens. What is the
5.4 object distance? Table 1
Lenses
A 5.0 cm C 12.5 cm
B 6.7 cm D 16.7 cm Which of the following pairs of
17 It is given that u, v and f are object
lenses you would choose to form
distance, image distance and focal
22 Diagram 12 shows points P, Q, R a compound microscope?
length. Which formula is correct in
and S on the principal axis of a A P and Q C Q and R
finding the focal length of a lens, if
convex lens. Where would you B P and S D R and S
the values of u and v are known?
place an object in order to produce
production of u and v 25 The distance between the two
A f= a real and magnified image?
sum of u and v lenses in a telescope in normal
sum of u and v adjustment is 63 cm. The power of
B f= the eyepiece is 8 times the power
production of u and v P Q R S
of the objective lens. Calculate the
C f = production of u and f focal length of the eyepiece.
v + sum of u and v
2f A 5 cm C 9 cm
Diagram 12 B 7 cm D 52 cm

309 Light

1.1 Significant Figure


Structured Questions
1 Diagram 1 shows two boys, P and Q standing 6 m in (b) Diagram 2(b) shows a light ray travelling towards
front of a plane mirror. The length of the mirror and the centre, O of a circular glass block.
the distance between P and Q are both10 m. (i) What is the angle of incidence at the
air–glass edge? [1 mark]
(ii) Complete the path of the light ray.
[2 marks]
(c) Diagram 3 shows two light rays entering a semi
circular glass block.

F
O
5

R Diagram 1
CHAPTER

M N is the mid point of the mirror.


(a) Draw on Diagram 1 the position of the image of P
4
which Q sees. [2 marks]
(b) P moves towards N and reaches N after 5 seconds.
(i) How is this movement seen by Q? [1 mark]
(ii) What is the velocity of the image of P? Diagram 3
[2 marks] (i) Draw the normal at point P and at point Q.
(c) If P and Q move to the right at a speed of 1 m s–1, [2 marks]
how long can P observe the image of F Q after both (ii) Complete the path of each light ray.
start moving? O [2 marks] [2 marks]
5

R
2 (a) Diagram 2(a) shows a light ray
M entering a 3 Diagram 5 is the structure of an optical instrument
CHAPTER

rectangular glass block perpendicularly. SPM used in a submarine as shown in Diagram 4.


Clone

4 ’06 optical
instrument P

(a) Diagram 4

ray of light
45°
glass prism F4/5/188 (New) from the
45°
object

observer A
(b)
Diagram 2
(i) Explain why the light ray does not bend Diagram 5
through the glass block. [1 mark]
(ii) Which is correct? Tick your answer. (a) Name the optical P.
instrument
F4/5/189 [1 mark]
(b) Instrument P is a structure with two glass prisms.
Since refraction does not occur, This enables the observer to view objects above
there is no change in velocity of the surface of the sea water.
light in the glass block. (i) Diagram 5 shows the position of one of the
The speed of light decreases prisms. Draw in box A the position and
because the nature of the medium orientation of the second prism.
determines the speed of light. [1 mark]
(ii) Explain why the prisms are placed as such in
[1 mark] (b)(i). [1 mark]

Light 310
(c) (i) In Diagram 5, complete the path of the ray (b) How far must the object be moved to obtain an
of light from the object to the observer’s image which is of an equal size as the object?
eye. [1 mark] What is the distance of the screen from the lens
(ii) State one characteristic of the image then? [2 marks]
observed. [1 mark] (c) The object is moved upwards 6 cm in 2 seconds
(d) Determine the critical angle of the glass prism, in a parallel direction to the plane of the screen.
given that its refractive index is 1.52. What is the speed of the image on the screen?
[2 marks] [2 marks]
(d) When a concave lens is placed between the
4 Diagram 6 shows a point object, O placed on the left
convex lens and the screen, the image becomes
side of a convex lens. The lens is moved until an
blurred.
image 3 times the size of the object is formed on the
(i) Draw the rays of light to complete Diagram
screen. The distance between the object and screen is
7. [2 marks]
60 cm.
F
O
convex lens concave

5
screen R
lens

CHAPTER
M
convex lens screen
4
O

60 cm

Diagram 6 60 cm
(a) the image distance, v in terms of the
(i) State F4/5/235 Diagram 7
object distance, u.
(ii) Calculate the focal length of the lens. (ii) State two ways to restore the sharpness of
[3 marks] the image. [2 marks]

Essay Questions
5 (a) Differentiate between a real image and a virtual image. [1 mark]
(b) Diagram 8 and Diagram 9 show the positions of an object placed in front of a convex lens.
F is the focal point of the lens.

Diagram 8 Diagram 9

(i) Based on Diagram 8 and Diagram 9, compare the object distances of the object. [1 mark]
(ii) Based on Diagram 8 and Diagram 9, compare the positions of the images. [1 mark]
(iii) Based on Diagram 8 and Diagram 9, relate the characteristics of images to the object distances.
[2 marks]
(c) Draw a ray diagram to show the formation of an image in a compound microscope. [5 marks]

311 Light
(d) Diagram 10 shows the ray diagram for two convex Table 1
lenses used in an astronomical telescope.
Periscope Prism Arrangement Features of
used of the two the optical
prisms in the materials
tube used to make
the prisms
n = 1.3
Low scattering
45° and low
P
Diagram 10 absorption of
45° F4/5/241
light F4/5/241

Explain how you would design a prism binoculars


F n = 1.5
O
with an additional two prisms. Draw a diagram to
show your design and, in your explanation, Low scattering
5

R 45° 45°
emphasise the following aspects: Q and low F4/5/242
F4/5/242
CHAPTER

M
(i) the material of the prisms 45° 45° absorption of
4 (ii) the shape of the prisms light
(iii) the arrangement of the prisms
What are the advantages of these binoculars over n = 1.5
an astronomical telescope if you are looking at the Low scattering
R 60° 60°
and low
scenery from the Petronas Twin Tower in Kuala F4/5/243
Lumpur?
30° 30° absorptionF4/5/243
of
[10 marks]
light
F
O
n = 1.3
Low scattering
5

R
6 (a) What is meant by the ‘refractiveM index of a S 60° 60° and low f4/5/(244a,b)f
CHAPTER

transparent material’ in terms of the speed of 60° 60° 60° 60° absorption of
light? Hence, explain why the 4value of the light
refractive index, n is always more than 1.0.
(b) Diagram 11 shows two opaque cups, A and B, n = 1.5
with different levels of water. A similar coin is 45° 45° High scattering
placed at the bottom of each cup. T and high
45° 45°
absorption of
P Q light
f4/5/(245a
f4/5
R S
n = 1.5
Low scattering
U 60° 60° and high
image
image
water
30° 30° absorption of f4/5/(246a
f4/5
water
light
cup A cup B

Diagram 11
F4/5/240
Study the specifications of all the periscopes from
When the coin is observed from positions P and Q the following aspects:
(same level), the images in both cups are as — the shapes of prisms
shown. However, at positions R and S (same level — the arrangements of the prisms
and same distance from the cups), the image of — the features of the material used to make the
the coin in cup A cannot be seen, but the image in prisms
cup B can be seen. Explain the suitability of the aspects.
Explain, by drawing appropriate rays of light, why Justify your choice.
the observer can see the coin in cup B but not in (d) A light ray travelling in air enters a transparent
cup A. medium at an incident angle of 50°. If the light ray
(c) Table 1 shows the specifications and arrangements of is deviated by 20° from its original path,
two prisms used in the construction of a periscope. determine
You are required to determine the most suitable (i) the refractive index of the medium,
periscope for a submarine. (ii) the critical angle of the medium.

Light 312
Experiments
1 An experiment is conducted to investigate the relationship between the magnification of an image and the
object distance from a thin lens. A transparent ruler with graduations, in mm, is placed in a ray box.
The ruler acts as the object for a convex lens of a focal length, f. The convex lens is placed at an object distance,
u = 12.0 cm from the ruler as shown in Diagram 1. The screen is adjusted until a sharp image is formed on
the screen.

F
O

5
Diagram 1 R

CHAPTER
M

The experiment is repeated with different object distances, u = 15.0 cm, 18.0 cm, 21.0 cm, and 24.0 cm.
4

Diagrams 2 and 3 show the ruler and its varying images in the experiment.
[Diagrams 2 and 3 are actual size drawings.]

0 10 20 30 40 50 60

)f4/5/(244a,b) Diagram 2

a,b)
5/(245a,b)

(a) u = 12.0 cm

5/(246a,b)
a,b)

(b) u = 15.0 cm

(c) u = 18.0 cm

313 Light
2 A student conducts an experiment to investigate the
relationship between the object distance, u and the
10 20 30 40 50 60 image distance, v for a convex lens. The student uses
0 different values of the object distance, u and records
the corresponding image distance, v when a sharp
image is formed on a screen. The student then draws
a graph of v against u, as shown in Diagram 4.
(d) u = 21.0 cm Graph of v against u

F
10 20 30 40 50 60
O 0
5

R
CHAPTER

4 (e) u = 24.0 cm
Diagram 3

(a) For the experiment described, identify


(i) the manipulated variable,
(ii) the responding variable,
(iii) a constant variable. F [3 marks]
O
5

R
(b) What are the characteristics of the images formed?
CHAPTER

M
[2 marks]

(c) The magnification of the image is given4in the formula:


Size of image
m =
Size of object
Diagram 4
For each object distance, u, record the value of the
size of the object, x, and the measured size of the (a) Based on the graph in Diagram 4,
image, y. You can take part of or the whole ruler as (i) extrapolate the graph and determine the
your object. The size of the object and the value of the image distance, v when the
corresponding size of the image are measured object distance, u = 35.0 cm. Show on the
accordingly. graph paper how you obtain the value of v
from the extrapolated portion of the graph.
Calculate the corresponding magnification,
[2 marks]
y
m= . (ii) calculate the gradient of the graph at
x u = 20.0 cm. [3 marks]
Tabulate the results for u, x, y, and m. (b) Draw a straight line, v = 2u over the graph in
[5 marks] Diagram 4. From the point of intersection, (u’, v’)
of the two graphs, calculate the value of the focal
(d) Based on your table, draw a graph of length using the equation below:
magnification, m against the object distance, u on
a graph paper. [5 marks] 1 1 1
+ =
u’ v’ f
(e) From your graph, state the relationship between [3 marks]
magnification, m and the object distance, u. (c) State one precaution to be taken in the
[1 mark] experiment. [1 mark]

Light 314
FORM 5

1
CHAPTER

Waves

SPM Topical Analysis


Year 2007 2008 2009 2010 2011
Paper 1 2 3 1 2 3 1 2 3 1 2 3 1 2 3
Section A B C A B A B C A B A B C A B A B C A B A B C A B
Number of questions 6 1 – 1 – – 6 1 – 1 1 – 6 1 – – 1 – 8 2 1 – 1 – 7 1 – – – 1

ONCEPT MAP

WAVES

Oscillating Types of
Damping system waves

Resonance
Transverse waves Longitudinal waves
C R C R

Quantities
Graphs
• Amplitude, A
• Period, T Water Light Electromagnetic Sound
• Frequency, f waves waves (EM) waves waves
• Wavelength, λ
• Speed, v Displacement-
time graph Electromagnetic
spectrum
Relationships
• v = fλ Phenomena
1 T
• Reflection • Sources
• f= • Properties
T • Refraction
• Diffraction • Applications
Displacement- • Interference • Detrimental effects
distance graph ax
λ=
D

Applications of • Loudness and


λ reflection of amplitude
sound waves • Pitch and frequency

COMPANION WEBSITE
Learning Objectives 315
1.1 Understanding Waves

Motion of Waves

1 In Figure 1.1, a girl is moving to and fro on a


swing. This type of motion is called an
oscillation.
2 An oscillating or vibrating motion in which a
Figure 1.1 An oscillating system produces waves
point or body moves back and forth along a
line about a fixed central point produces waves.
3 An oscillating or vibrating system acts as the
source of waves which transfer energy from one 4 Table 1.1 shows various types of waves and the
point to another (without transferring matter). sources that produced them.
Table 1.1

Type of wave Source of wave Type of wave Source of wave


Light Vibration of electrons Water wave Disturbance (or vibration)
in an atom. on a still water surface.
F
O
1

R
CHAPTER

M
Sound Vibration of mechanical Electromagnetic wave Oscillation (or vibration)
5 bodies such as guitar between the electric field
strings or tuning fork. and the magnetic field.

2 However, the particles of the medium do not


Propagation (Travelling) of Waves
travel in the direction of the wave.
1 When a wave travels through a medium, the 3 A wave transfers energy and the momentum
particles of the medium vibrate about their from the source of the wave (the oscillating or
equilibrium positions. vibrating system) to the surroundings.

Activ To demonstrate that waves transfer energy without


ity 1.1
SPM
’08/P2
transferring matter

Apparatus/Materials 2 The candle is lit and the movements of its flame


Radio, candle and matches. is observed.
3 Then, the radio is turned on and the volume of
Arrangement of apparatus
the sound is gradually increased until a change in
the movement of the flame becomes noticeable.
Discussion
The flame vibrates when the radio is turned on.
candle This observation shows that the propagation of the
sound waves from the vibration of the cone of the
speaker transfers energy (or momentum) to the
Activity 1.1

Figure 1.2 flame and causes it to vibrate.


Procedure Conclusion
1 A candle is placed about 10 cm from the speaker Waves transfer energy from a vibrating system
of a radio. without transferring matter.

Waves 316
The total energy carried by a wave depends on its amplitude and frequency.
A wave with a large amplitude or a high frequency carries a larger amount of energy.
Tsunami is a huge ocean wave produced by an underwater earthquake, landslide or
volcanic eruption. These waves may reach enormous dimensions and have sufficient
energy to travel across entire oceans. The 2004 Indian Ocean earthquake that occurred in
26 December 2004, in Sumatra, Indonesia was one of the deadliest natural disasters in
history, killing over 225 000 people in 11 countries.

Types of Waves

1 There are two types of waves. 2 A slinky spring can be used to demonstrate the
(a) Transverse wave propagation of transverse and longitudinal
(b) Longitudinal wave waves.
SPM
Transverse wave Longitudinal wave ’09/P1

Differences

A wave in which the vibration of A wave in which the vibration of F


particles in the medium is at right angles Definition particles in the medium is along (parallel O

1
(perpendicular) to the direction of the to) the direction of the propagation of R

CHAPTER
propagation of the wave. the wave. M

SPM direction coloured coloured thread


’08/P2 direction of thread direction
of wave of wave direction of
propagation vibration vibration
propagation
side to side C R C R C R
movement fixed fixed
end end
Key:
backwards and C: compression
forwards movement R: rarefaction
• When a stretched slinky spring placed Model of
on a horizontal surface (floor) with one slinky spring • When a slinky spring is vibrated back
end fixed and the other end free is and forth along the direction of the
moved sideways (from left to right and propagation of the wave at a fixed rate, a
vice versa) on the floor, a transverse longitudinal wave is produced.
wave is then produced. • The coloured piece of thread, which
• A coloured piece of thread (representing represents the particles of the medium,
a particle of the medium), which is tied vibrate in the direction of the
to the spring, is observed to be moving propagation of the wave.
in a direction (left to right)
perpendicular to the direction of
propagation of the wave.

• Water waves Examples of


• Sound waves
• Light waves wave

Similarities
• Produced by vibration or oscillation
• Transfer energy or momentum without transferring matter
• Propagated by vibration or oscillation of the particles of the medium

317 Waves
Wavefronts

1 A wavefront is a line or plane on which the vibrations of every point


on it are in phase and are at the same distance from the source of the The lines that join the points
wave. along the troughs of the
2 Points in a wave are in phase if they vibrate in the same direction with waves are also wavefronts.
the same displacement.

Plane Wavefronts

• Figure 1.3 shows the production of plane water waves when a wooden bar crests
vibrates vertically with a constant frequency on the surface of the water.

wavefront
troughs

direction
wave direction of propagation
wavefront of wave
crest
S U W
F Q
plane wavefronts
O
Q S U W
1

P R T V
R
trough
CHAPTER

M
Figure 1.3 Plane wavefronts
5 direction of
• Lines PQ, RS, TU and VW are straight lines along the respective crests of propagation
P R T V
the waves. These lines are called wavefronts.
(a) Plane wavefronts

Circular Wavefronts
direction of
• When we use our fingertip to touch the surface of water repeatedly, circular propagation
waves are produced as shown in Figure 1.4.

wavefront direction
of propagation
of wave
circular
wavefronts
circular
O
wavefronts
direction (b) Circular wavefronts
of propagation
direction of of wave
propagation Figure 1.5 The direction of
of waves
propagation is
Figure 1.4 Circular wavefronts always at right angles
to the wavefront
• The wavefront at any point is perpendicular to the direction in which the
wave is advancing, i.e., the direction of propagation is normal to the
tangent at that point on the wavefront.

3 Figure 1.5 shows that the wavefronts are perpendicular to the


direction of propagation.

Waves 318
Amplitude, Period and Frequency of a Wave
1
1 The amplitude, A, of a vibrating system is the
In an experiment, Aziz observes that a simple pendulum
maximum displacement from its equilibrium
completes 30 oscillations in 48.0 seconds. What is
position. It is a measure of the height of the
(a) the period of oscillation?
wave crest or the depth of the wave trough.
(b) the frequency of oscillation?
2 In Figures 1.6 (a) and (b), the distance OP or
OQ is the amplitude, where O is the equilibrium Solution
position of the vibrating system. Time taken
(a) Period, T =
Number of complete oscillations
spring = 48.0
30
string
= 1.6 s
P
(b) Frequency, f
Number of complete oscillations
bob mass =
O Time taken
P Q
= 30 1 1
O Q 48 or f = T = 1.6 = 0.625 Hz
(a) Simple pendulum (b) Weighted spring = 0.625 Hz F
O
Figure 1.6

1
R

CHAPTER
3 The period, T, of a vibrating system is the time M
taken to complete an oscillation. 5
4 A complete oscillation may be referred as the
movement of a vibrating system from one The relationship between T and f is shown by the
extreme position to the other and back to the following graphs.
same position. T (s) TT
(s)
5 In the two vibrating (oscillating) systems shown
in Figure 1.6, a complete oscillation is:
(a) from P → Q → P or Q → P → Q,
(b) from O → P → Q → O or O → Q → P → O. ƒ (Hz) 1
(Hz -1)-
(Hz
6 If a vibrating system makes n complete 0 0 ƒ

oscillations in a time of t seconds, the period of T is inversely T is directly


proportional to f 1
oscillation, T of the system is t seconds. proportional to
f
n
ƒ (Hz) ƒT
∴T= Time taken, t
Number of complete oscillations, n
The SI unit of period is second. 1
7 The frequency, f, is the number of complete
1 (s-1)
oscillations made by a vibrating system in one 0 T 0
T (s)
second.
f is directly fT = 1
f = Number of complete oscillations, n 1
Time taken, t proportional to
T
The unit of frequency is hertz (Hz) or s–1.
8 From the formulae of T and f , the relationship
between period, T and frequency, f is:
SPM
T= 1 or f= 1 Displacement–time Graph of a Wave ’04/P1
f T SPM
’07/P1
1 The sinusoidal graph in Figure 1.7 is a
∴ T is inversely proportional to f and vice graph of displacement, s against time, t
versa. of a load on a spring.

319 Waves
displacement, s (cm)
Solution
spring
(a) Amplitude, A = 5 cm
C T G
t = t1 A a (b) Period of oscillation, T = 0.04 s
a P Q 1
t2 time, t (s) (c) Frequency of oscillation, f =
t=0 O 1 T
O
a load, m t1 D F ∴f = = 25 Hz
0.04
t = t2 B –a
E

Figure 1.7 Displacement-time graph


Displacement–distance Graph of a Wave
(a) At t = 0 s, the load m is at its equilibrium
SPM
position, O. The load then oscillates to the 1 Figures 1.9 (a) and (b) show the ’06/P1
highest position A at t = t1. Section OC shows propagation of a water wave and a SPM
’07/P1
the increase in displacement with time. sound wave. SPM
(b) Section CD represents the return motion of crest λ
’08/P2

the load, m from position A to its equilibrium


position O where its displacement is zero.
This position is represented by point D on
the horizontal axis.
(c) When the load m moves from position O
F to position B, its displacement is negative trough λ
O
as it is below the equilibrium position O.
1

R (a)
Section DE shows the negative displacement
CHAPTER

M
of the load with time.
C R C R C R C
5 (d) The return motion of the load m from C : compression
position B to O is represented by section R : rarefaction
EF.
(e) The rest of the graph represents the
continuous movement of the load.
2 From the graph of s against t in Figure 1.7, the λ λ
following information is obtained. (b)
(a) Amplitude, A = a cm
(b) Period of oscillation, T is the time interval
displacement, s (cm)
between points:
(i) O and F, (ii) C and G or (iii) P and Q. λ
A

2 O distance, d (cm)

Figure 1.8 shows the displacement-time graph of the –A


λ
oscillation of a mass on a spring.
displacement, s (cm)
(c)
Figure 1.9
5
2 When a wave is propagating through a medium
such as water or air, the particles of the medium
0 0.02 0.04 0.06 time, t (s) vibrate about their equilibrium positions.
3 The displacement, s of each particle of the
–5
medium at different distances can be shown in
Figure 1.8 a displacement–distance graph as shown in
From the graph, Figure 1.9 (c).
(a) state the amplitude, 4 The wavelength, λ is the distance between
(b) find the period of the oscillation, two successive points of the same phase in a
(c) calculate the frequency of the oscillation. wave.

Waves 320
For example:
Relationship between Speed (v),
(a) the distance between two successive crests
Wavelength (λ) and Frequency (f)
or two successive troughs in a water wave,
(b) the distance between two successive The relationship between speed, wavelength and
compressions or two successive rarefactions frequency can be obtained by relating the SI units
in a sound wave. of the quantities.
The SI unit of wavelength, λ is metre (m). From the units of v = [m s–1], λ = [m] and f = [s–1]:
[m s–1] = [m] × [s–1]
3 ∴ v = fλ
Figure 1.10 shows a displacement-distance graph of a Thus: Velocity = Frequency × Wavelength
wave.
displacement, s (cm)
4
4
A wave of frequency 120 Hz has a wavelength of
0
5.0 m. What is the speed of the wave?
3 6 9 12 15 18 distance, d (cm)
Solution
–4
f = 120 Hz and λ = 5.0 m F
Figure 1.10 Speed of wave, v = fλ = 120 × 5 = 600 m s–1 O

1
R
Find the amplitude and the wavelength of the wave.

CHAPTER
M
Solution
5 5
Amplitude, A = 4 cm
Wavelength, λ = 12 cm The displacement-distance graph in Figure 1.11
shows the motion of a transverse wave. The source of
the wave produces 10 complete waves in one second.
displacement, s (cm)
1 SPM
Clone
’10
6
Figure below shows an oscillating pendulum.
0
25 distance, d (cm)

–6

Figure 1.11
Pendulum
Z bob Calculate
X
Y (a) the amplitude,
(b) the wavelength, and
What is the correct path for one complete (c) the speed of the wave.
oscillation? Solution
A X→Y (a) Amplitude, A = 6 cm
B X→Y→Z
C X→Z→Y (b) Wavelength, λ = 4 × 25 cm
5
D X→Z→X 1λ
= 20 cm 4
λ
Comments (c) Frequency, f = 10 Hz
One complete oscillation is the motion from one Speed, v
extreme position to the other and back to the = fλ
position. = 10 × 20
25 cm
= 200 cm s–1 5 = 25 cm
Answer D 4
λ

321 Waves
SPM 9 The characteristics of resonance can be
Damping and Resonance ’04/P1
demonstrated with a Barton’s pendulum
system as shown in Figure 1.14.

string wire

retort
stand
D
A B
X B
C weight
O
A
Figure 1.12 Oscillation of a simple pendulum E

1 In an oscillating system such as the oscillation Figure 1.14 Barton’s pendulum


of a simple pendulum, the oscillation does not
continue with the same amplitude indefinitely (a) When pendulum X oscillates, all the other
except when the system is oscillating in a vacuum. pendulums are forced to oscillate. It is found
2 The amplitude of oscillation of the simple that pendulum B oscillates with the largest
pendulum will gradually decrease and become amplitude, that is, pendulum B resonates.
zero when the oscillation stops. The decrease (b) The natural frequency of a simple
in the amplitude of an oscillating system is pendulum depends on the length of the
F called damping. pendulum. Note that pendulum X and
O 3 An oscillating system experiences damping pendulum B are of the same length.
1

R
when its energy is drained out as heat energy. Therefore, pendulum X causes pendulum
CHAPTER

M B to oscillate at its natural frequency.


(a) External damping of the system is the
loss of energy to overcome frictional 10 Some effects of resonance observed in daily life:
5
forces or air resistance. (a) The tuner in a radio or television enables
(b) Internal damping is the loss of energy you to select the programmes you are
due to the extension and compression of interested in. The circuit in the tuner is
the molecules in the system. adjusted until resonance is achieved, at the
frequency transmitted by a particular
amplitude, A (cm) station selected. Hence a strong electrical
F
O
signal is produced.
1

amplitude gradually decrease (b)


R The loudness of music produced by musical
M instruments such as the trumpet and flute
CHAPTER

is the result of resonance in the air.


5
O time, t (s)
(c) The effects of resonance can also cause
damage. For example, a bridge can collapse
Figure 1.13 Damping when the amplitude of its vibration
4 Damping in an oscillating system causes increases as a result of resonance.
(a) the amplitude, and
(b) the energy of the system
to decrease but the frequency, f does not change.
5 To enable an oscillating system to go on Tacoma Narrows Bridge Disaster
continuously, an external force must be applied A well known example of the effects of resonance is the
to the system. collapse of the suspension bridge, Tacoma Narrows in
6 The external force supplies energy to the system. USA in 1940. The action of the wind caused the bridge
Such a motion is called a forced oscillation. to vibrate with a large amplitude as a result of resonance.
7 The frequency of a system which oscillates
freely without the action of an external force
is called the natural frequency.
8 Resonance occurs when a system is made to
oscillate at a frequency equivalent to its natural
frequency by an external force. The resonating
system oscillates at its maximum amplitude.

Waves 322
1.1

1 A mass on a spring oscillates with a frequency of


An error in sketching a displacement-time graph of a 4 Hz. What is the time taken by the mass to
damped oscillating system. complete 120 oscillations?
2 A wave of frequency 240 Hz has a wavelength of
displacement, s (cm) 6 m. What is the speed of the wave?
amplitude gradually decrease 3 A mass on a spring oscillates with a frequency of
3 Hz. Calculate
(a) the number of oscillations in 10 s,
time, t (s) (b) the period of oscillation.
Period changed, T1 ≠ T2 4 A dipper produces plane waves in a ripple tank at a rate
∴ Frequency varies
of x waves per second. The wavelength of the plane
waves is λ cm. What is the wavelength of the plane
waves if the dipper produces 2x waves per second?
displacement, s (cm)
5 The figure below is the displacement-time graph of a
amplitude gradually decrease
wave.
displacement, s (cm)

time, t (s) F
T1 T2 4
O

1
time, t (s)
R
Period is a constant, T1 = T2

CHAPTER
0 3.0 M
∴ Frequency is a constant
–4
5

Calculate
2 SPM
Clone
(a) the amplitude, (c) the wavelength,
’04
(b) the frequency,
A system is oscillating at its natural frequency in a if the speed of the wave is 8 cm s–1.
vacuum. What happens to the total energy of the 6 The figure shows the form of a transverse wave
oscillating system after two hours? produced by a slinky spring.
A It decreases C It remains constant
B It increases 11 cm
Comments
In a vacuum, an oscillating system will not undergo 36 cm
damping. Hence, it will oscillate with a constant
amplitude indefinitely. What is
(a) its amplitude? (b) its wavelength?
Answer C
7 The diagram shows three spring systems.

Q
P S T
3 SPM
Clone
’06
m R
The process of reduction in the amplitude of frequency = f 1 m
oscillation is known as m frequency = f 3
A resonance frequency = f 2
B damping
C modulation The spring systems oscillate with frequencies f1, f2
D rectification and f3 respectively. If springs P, Q, R, S and T are
identical, what is the relationship between the
Answer B frequencies f1 , f2 and f3 ?

323 Waves
1.2 Analysing Reflection of Waves
The phenomenon of water
Reflection of Waves waves can be investigated
using a ripple tank. The water
1 Reflection of a wave occurs when a wave strikes an obstacle. The wave waves are produced by a
undergoes a change in the direction of propagation when it is reflected. vibrating bar on the water
2 The incident wave is the wave before it strikes the obstacle, whereas surface. The water acts as a
the reflected wave is the wave which has undergone a change in lens to produce a pattern of
direction of propagation after reflection. bright and dark regions on a
piece of white paper placed
normal
incident wave reflected wave under the tank when light
passes through it. The
diagram below shows the
working principle of a ripple
r
i tank.

plane reflector crest


light from lamp

i = angle of incidence (the angle between the direction of propagation of the incident trough
wave and the normal) water
F
O r = angle of reflection (the angle between the direction of propagation of the reflected
1

R wave and the normal)


white
CHAPTER

M
Figure 1.15 Laws of reflection of waves paper

5
3 The phenomenon of reflection of waves obeys the Laws of reflection dark bright dark bright dark
where:
(a) The angle of incidence, i is equal to the angle of reflection, r.
(b) The incident wave, the reflected wave and the normal lie in the
same plane which is perpendicular to the reflecting surface at the
point of incidence.
F
Reflection of Water Waves O
1

R da
CHAPTER

Activ To investigate the reflection of plane waves


ity 1.2 5

Apparatus/Materials Arrangement of apparatus


Ripple tank, plane
lamp
reflector, white piece of
rubber band
paper, wooden bar, motor
lamp, motor, sponge wooden
spherical dipper
and mechanical bar water

stroboscope. ripple tank


(a) plane waves
reflector
turning the
stroboscope

shadow observer
Activity 1.2

stroboscope is used to
‘freeze’ the motion of
the wave
(b) circular waves white piece of paper

Figure 1.16 Investigation of the reflection of plane waves

F5/1/12
Waves 324
Procedure
1 A ripple tank is filled with water and is set up as shown in Figure 1.16. The tank is levelled so that the
depth of water in the tank is uniform to ensure water waves propagate with uniform speed.
2 All the inner surfaces of the ripple tank are lined with a layer of sponge to prevent reflection of the
water waves from the edges.
3 The lamp above the tank is switched on and a large piece of white paper is placed below the tank.
4 A metallic plane reflector is placed at the centre of the tank. The motor with a wooden bar attached is
switched on to produce plane waves which propagate towards the reflector.
5 The pattern (on the white paper) of the reflected waves produced by the vibrating wooden bar is
observed with the help of a mechanical stroboscope. The incident waves and the reflected waves are
sketched.
6 Steps 4 and 5 are repeated with the reflector repositioned so that the wave is incident at angles,
i = 20°, 30°, 40°, 50° and 60° on the reflector as shown in Figure 1.17.

plane waves

direction of normal
propagation i
plane
reflector F
crest Figure 1.17 O

1
R
light from lamp

CHAPTER
M

Results trough
5
water
Table 1.2

Pattern of reflected waves Characteristic of waves


(i) white
• Angle of incidence, i = 0°
reflected wave
paper Angle of reflection, r = 0°
• Wavelength, frequency and speed of wave do not
ark bright dark bright
direction of reflected waves
dark change after reflection.
direction • Direction of propagation of wave changes.
of incident
waves
λ λ
incident wave

(ii) SPM
i (°) 20 30 40 50 60 ’09/P1
incident wave
r (°) 20 30 40 50 60

direction of
incident • Angle of incidence, i = Angle of reflection, r
O
waves • Wavelength, frequency and speed of wave do not
change after reflection.
• Direction of propagation of the wave changes.
reflected direction of
wave reflected
waves

Conclusion
The angle of reflection is equal to the angle of incidence.
The laws of reflection are obeyed.

325 Waves
4 SPM
Clone
’04
Water waves in motion are not easily observed by the
naked eye. A mechanical stroboscope is used to study Which of the following figures shows the correct
the propagation of water waves. The moving water pattern for the reflection of water waves?
waves appear to be stationary when the frequency of A C
the stroboscope is the same as the frequency of the
water waves. The frequency of the water waves can be
x x
determined as follows: y y
Frequency of water wave, f = nf′
n = number of slits on the stroboscope
f′ = frequency of the stroboscope
B D

x x
x x
6
A water wave of frequency 20 Hz appears stationary
F when observed through a stroboscope with 4 slits.
O What is the frequency of rotation of the stroboscope?
Comments
1

R
Solution
CHAPTER

M
For reflection of waves, the angle of incidence is
Number Frequency of equal to the angle of reflection.
5 Frequency of wave = ×
of slits stroboscope The frequency, wavelength and speed of reflected
20 = 4 × f ʹ′ waves are unchanged.
f ʹ′ = 5 Hz Answer D

Reflection of Sound Waves

Activ To investigate the reflection of sound waves


ity 1.3
Apparatus/Materials Procedure
Two cardboard tubes, stopwatch, a slab of soft wood, 1 The apparatus is set up as shown in Figure 1.18.
a wooden board with a smooth surface and a 2 The angle of incidence, i = 30° is measured with a
protractor. protractor.
Arrangement of apparatus 3 The stopwatch is started.
wooden board 4 The position of the cardboard tube B is adjusted
cardboard tube B until a loud ticking sound of the stopwatch is
cardboard tube A i r heard.
5 The angle of reflection, r at this position of the
cardboard tube B is measured.
Activity 1.3

6 Steps 2 to 5 are repeated with angles of incidence,


i = 40°, 50°, 60° and 70°.
stopwatch
soft wood observer 7 The results are tabulated.
Figure 1.18 Investigation of the reflection of sound

Waves 326
Tabulation of data

Table 1.3
Angle of incidence, i (°) 30 40 50 60 70
Angle of reflection, r (°) 30 40 50 60 70

Discussion
The sound waves from the stopwatch experience a reflection after striking the wooden board.
The slab of soft wood placed along the normal serves as a barrier to prevent the sound of the stopwatch
from reaching the observer directly.

Conclusion
The angle of incidence, i, is equal to the angle of reflection, r.
The laws of reflection are obeyed.

F
Reflection of Light O

1
R

CHAPTER
M

Activ To investigate the reflection of light


ity 1.4 5

Apparatus/Materials
Plane mirror, ray box, plasticine, protractor, white piece of paper and a sharp pencil.

Arrangement of apparatus
plane mirror on a
piece of white paper

60o P
50o
40o
ray box 30o O
20o i r
N
Q
plasticine

Figure 1.19 Investigation of the reflection of light

Procedure
1 A straight line, PQ is drawn on a sheet of white paper.
2 A normal line, ON is drawn from the midpoint of PQ.
3 Using a protractor, lines at angles of incidence of 20°, 30°, 40°, 50° and 60°, with the normal, ON are
drawn.
4 A plane mirror is erected along the line PQ.
5 A ray of light from the ray box is directed along the 20° line. The angle between the reflected ray and
the normal, ON is measured.
6 Step 5 is repeated with the angle of incidence, i of 30°, 40°, 50° and 60°.
7 The results are tabulated.

327 Waves
Tabulation of data
Table 1.4
Angle of incidence, i (°) 20 30 40 50 60
Angle of reflection, r (°) 20 30 40 50 60

Discussion
The incident ray must be as narrow as possible to obtain a narrow and thin reflected ray.
It can be done by adjusting the lens in the ray box (or a laser pen can be used instead).

Conclusion
The angle of incidence, i, is equal to the angle of reflection, r.
The laws of reflection are obeyed.

F
O
In the reflection of waves, the speed (v), wavelength (λ) and frequency (f) of the waves do not change.
However, the direction of propagation and velocity of the reflected waves change (unlike speed, velocity takes the
1

R
direction into account) because it is a vector quantity.
CHAPTER

incident wave

plane reflector
direction
of i
incident r Angle of incidence, i ≠ Angle of reflection, r
waves

normal reflected
wave

incident wave

plane reflector
direction of
incident waves i
Angle of incidence, i = Angle of reflection, r
λ1 r
Activity 1.4

λ1 = λ2
∴ Wavelength, λ does not change
λ2 ∴ as the depth of water is uniform
normal reflected
wave

Waves 328
Applications of Reflection of Waves in Telecommunications
Daily Life Infrared waves from the remote control of an
electrical equipment (television or radio) are
Safety
reflected by objects in the surroundings and
(a) The rear-view mirror and side mirror in a car received by the television set or radio.
are used to view cars behind and at the side
while overtaking another car, making a left or
right turn and parking the car. The mirrors
reflect light waves from other cars and objects
into the driver’s eyes.

rear-view Figure 1.23


mirror side
mirror

1.2

Figure 1.20 Mirrors of a car 1 The diagram shows the stationary pattern of plane
waves seen through a stroboscope with 6 slits F
(b) The lamps of a car emit light waves with rotating at a frequency of 5 Hz. O

1
minimum dispersion. The light bulb is placed R

CHAPTER
at the focal point of the parabolic reflector of 6.0 cm M
the car lamp so that the reflected light waves
5
are parallel to the principal axis of the reflector.
Parallel light waves have a further coverage. direction of propagation

What is
(a) the wavelength of the water wave?
(b) the speed of the water wave?
Figure 1.21 Motor car lamps emit parallel light rays
2 State whether each of the following quantities
changes or remains constant when water waves are
Defence reflected.
(a) Frequency : _________________
(b) Velocity : _________________
reflection by plane mirror (c) Wavelength : _________________
incident light from object
(d) Speed : _________________

3 A wave is incident on a plane reflector PQ.

P O Q
reflection by plane mirror
observer in the 50°
submarine

Figure 1.22 Periscope

A periscope is an optical instrument. It can be


constructed using two plane mirrors for viewing
objects beyond obstacles. The light waves from an
object which is incident on a plane mirror in the What is the angle of incidence if the reflector PQ is
periscope are reflected twice before entering the rotated 15° clockwise about point O?
eyes of the observer.

329 Waves
1.3 Analysing Refraction of Waves
Any type of wave can be refracted, which means a change of direction.
Refraction occurs when the speed of a wave changes, as it moves from one
medium to another.

Refraction of Plane Water Waves SPM


’09/P2

1 Water waves undergo refraction (bending) when its speed changes.


Refraction is accompanied by a change in speed and wavelength of
the waves. The relationship between v
2 Water waves travel faster (with higher velocity, v) on the surface of deep and λ of a water wave in
water than they do on shallow water. Thus, if water waves are passing deep and shallow water can
from deep water into shallow water, they will slow down. This decrease be obtained from the
in speed will also be accompanied by a decrease in wavelength. The formula:
change in speed of the wave causes refraction. v = fλ
where frequency of the
Vd Vs Vd wave, f, is a constant in both
cases.
F side
view deep shallow water deep λ2
O water water
1

R
λd λs
CHAPTER

M
Key: Vd > Vs, λ d > λ s v2
v1
5
Figure 1.24 Water waves travel slower in shallow water

3 After refraction, the wave has the same frequency, but a different shallow
speed, wavelength and direction. deep
λ1
4 When a water wave travels from deep water into shallow water, the
v1 > v2, λ1 > λ2
wave is refracted towards the normal. Conversely, the wave is refracted
away from the normal when the water wave travels from shallow
F water v
f=
= constant
into deep water. The effects of refraction of water waves Oare shown in λ
1

Figures 1.25 (a) and (b). R v v


∴ 1= 2
CHAPTER

M λ1 λ2
5 v is directly
proportional
to λ.
normal normal
v
deep shallow shallow deep
water water water water v ∝λ
(a) Waves transmitted from deep (b) Waves transmitted from
water into shallow water shallow water into deep water
O λ
Figure 1.25
5 Table 1.5 shows the changes in velocity, wavelength and frequency of a
water wave when it travels from deep water region to shallow water region.
Table 1.5

Water region
Aspect
Deep water Shallow water
Velocity Increase Decrease
Wavelength Increase Decrease
Frequency Unchanged Unchanged

Waves 330
Activ To investigate the refraction of water waves
ity 1.5
Apparatus/Materials of the tank to create an area of shallow water in
Ripple tank, wooden bar, Perspex plates of different the tank.
shapes, mechanical stroboscope and white paper.

Arrangement of apparatus Perspex


plate
lamp
rubber band
motor
Figure 1.27
wooden water
bar
ripple tank 4 The dipper with the wooden bar attached is
Perspex
switched on to produce plane waves.
plate 5 The directions of the water waves in the areas of
deep and shallow water are observed with a
stroboscope.
6 Steps 3 to 5 are repeated with the orientation of
shadow the trapezium plate changed so that the wave is F
incident at an acute angle on a side of the plate O

1
as shown in Figure 1.28. R

CHAPTER
M
white paper
5
Figure 1.26 Investigation of the refraction of water waves Perspex
plate
F5/1/18
Procedure
1 A ripple tank is set up as shown in Figure 1.26.
2 The tank is filled with water and the legs of the Figure 1.28
tank are adjusted until the depth of the water in 7 Steps 3 to 5 are repeated using Perspex plates in
the tank is uniform. the shapes of a triangle, convex lens and concave
3 A Perspex plate in the shape of a trapezium, as lens.
shown in Figure 1.27, is immersed in the centre

Tabulation of data
Table 1.6
Position Observation
(a) Trapezium Perspex plate with the vertical normal
side of the plate facing the incident wave.

deep shallow deep

(b) Trapezium Perspex plate with the slant side shallow


of the plate facing the incident wave.

deep deep

(c) Triangular Perspex plate


Activity 1.5

deep shallow deep

331 Waves
Position Observation
(d) Perspex plate in the shape of a convex lens
converge to
SPM focus
’07/P1
plane
incident
waves F

(e) Perspex plate in the shape of a concave lens F5/1/19


SPM
plane incident ’09/P1
waves
F divergent
waves

F5/1/20
F Discussion 3 If the wave strikes the Perspex plate at a certain
O 1 Refraction occurs when a water wave passes angle of incidence, the water wave is refracted.
1

R from one area to another area with a different 4 The water wave is refracted towards the normal
CHAPTER

M if the wave travels from a deeper area to a


depth of water.
2 If the wave strikes the Perspex plate at an angle shallower area, and vice versa.
5
of incidence of 0°, the direction of propagation Conclusion
of the wave remains unchanged. The water wave The direction of propagation of a wave changes
is not refracted, i.e., the angle of refraction is if the angle of incidence of the wave is not zero.
zero. The shape of the refracted wave depends on the
shape of the area of shallow water over which the
waveF is passing.
O
1

R
CHAPTER

5
5 SPM
Clone
’06

The figure shows a ripple tank with a sloping base. Which of the following wave patterns is produced?
Plane waves are produced by a straight edged dipper A C
at P. The waves move towards Q.

straight dipper
B D
P Q

water

Comments
ripple tank
Water waves moving over regions where the depth of
Activity 1.5

water is decreasing experience a decrease in their


speed and wavelength. The wave pattern shows the
F5/1/61 wavefronts becoming closer.
Answer B

Waves 332
7
Figure 1.29 shows water ripples in two areas of Solution
water with different depths. The observation is made (a) Frequency of dipper
with a stroboscope with 3 slits. The frequency of the = Number of slits × Frequency of stroboscope
stroboscope is 4 rotations per second. = 3 × 4 = 12 Hz
plane dipper 2.4 cm (b) Area of deep water:
6 3λ1 = 6 cm
Wavelength, λ1 =
3
= 2 cm
Area of shallow water:
shallow 2.4 3λ2 = 2.4 cm
water Wavelength, λ2 =
3
6 cm = 0.8 cm
deep water
(c) Area of deep water:
Speed, v1 = fλ1
Figure 1.29 = 12 × 2
Calculate = 24 cm s–1
(a) the frequency of the dipper, Area of shallow water: F
(b) the wavelength in the deep area and in the shallow Speed, v2 = fλ2 O

1
area, = 12 × 0.8 R

CHAPTER
M
(c) the speeds of the waves in the two areas. = 9.6 cm s–1
5
8
A plane wave has a wavelength of 2 cm and a velocity Substituting in the relationship:
of 8 cm s–1 as it moves over the surface of shallow v1 v
water. When the plane wave moves into an area of = 2
λ1 λ2
greater depth, its velocity becomes 12 cm s–1. What is
8 12
(a) the wavelength and =
(b) the frequency 2 λ2
of the wave in the area of greater depth? 4λ2 = 12
Solution λ2 = 3 cm
v
(a) Area of shallow water: (b) Frequency of wave, f =
λ
v1 = 8 cm s–1 and λ1 = 2 cm 8
Area of deeper water: =
2
v2 = 12 cm s–1 and λ2 = ? = 4 Hz
v
For refraction, frequency, f (= ) remains the same. The frequency of the wave is the same in all the areas.
λ

6 SPM
Clone
’07

Which of the following characteristics of waves,


Water waves in areas of deep and shallow water appear
remains constant when the waves are refracted ? stationary when observed at the same time through a
A Wavelength C Frequency stroboscope. This proves that the frequencies of the
B Speed D Direction of propagation water waves in both areas are the same.
Comments
Refraction of waves will result in change of
wavelength, speed and direction of propagation
except its frequency.
Answer C

333 Waves
ray and the refracted ray are from the same
Refraction of Light
source (ray box), the frequency remains the
same. Hence, the wavelength of the ray in the
1 A swimming pool seems much shallower than glass is shorter than the ray in the air.
it actually is; a spoon appears bent when part
of it is in water and a boy’s legs look shorter Refer Form 4, Chapter 5, Unit 5.2.
when immersed in a pool. All these effects are
due to the refraction of light.
2 Figure 1.30 shows that a light ray is bent or Refraction of Sound Waves
refracted when passing from air to glass.
speed
ray box sound
ir, s lower
cool a
P normal

i
Q warmer air, faster sound speed
glass
r block warmer ai
r, faster soun
d speed

F refracted ray bent normal cool air, slower sound speed


towards the normal emergent ray bent
O
away from the normal
1

R Figure 1.31 Effects of refraction of a sound wave


F5/1/22
CHAPTER

M PQ = incident ray QR = refracted ray


1 It is easier to hear distant sounds on a cold
i = angle of incidence r = angle of refraction
5 night compared with a hot day. This is due to
Figure 1.30 Refraction of light ray by a glass block
F5/1/21
the effects of the refraction of sound waves.
3 When a ray propagates from one medium to 2 At night-time, the layers of air close to the
an optically denser medium, the ray refracts ground are cooler than the layers further from
towards the normal. Conversely, a ray the ground.
propagating from one medium to an optically 3 Cool air is denser than warmer air. Sound
less dense medium is refracted away from the travels
F at a slower speed in cold air. As a result,
normal. the
O sound waves are refracted in the path of a
1

4 The speed of the light decreases as it propagates curve


R (due to total internal reflection) towards
CHAPTER

in the glass block, causing it to alter the M


the ground instead of disappearing into the
direction of propagation. Since the incident upper layers of the air.
5

Activ To investigate the refraction of sound waves


ity 1.6
Apparatus/Materials Procedure
Audio signal generator, loudspeaker, balloon filled 1 The apparatus is set up as shown in Figure 1.32.
with carbon dioxide, microphone and cathode-ray
2 The experiment is started without the balloon.
oscilloscope.
3 The audio signal generator and the cathode-ray
Arrangement of apparatus
oscilloscope are switched on. The wave form
cathode-ray oscilloscope displayed on the screen of the oscilloscope is
audio signal observed and drawn.
generator loudspeaker
4 A balloon filled with carbon dioxide is placed
microphone between the audio signal generator and the
Activity 1.6

CO2 oscilloscope.
balloon 5 The wave form displayed on the screen is
observed and drawn.
Figure 1.32 Investigation of the refraction of
sound waves

Waves 334
Results A sound wave is refracted towards the normal when
the wave passes from the air to the carbon dioxide in
the balloon. This is because carbon dioxide is
denser than air and the speed of sound in carbon
dioxide is reduced. When the sound wave emerges
(a) without the balloon (b) with the balloon from the balloon, the wave is refracted away from
Figure 1.33 the normal. The balloon acts as a biconvex lens
F5/1/23
which converge the sound waves to the microphone.
The wave form displayed on the oscilloscope shows
that the amplitude is larger when the balloon is
placed between the audio signal generator and the Conclusion
oscilloscope. The larger amplitude indicates that a Sound waves are refracted when they travel from
louder sound is received by the microphone. one medium to another of different density. The
sound waves are refracted away from the normal
Discussion
after passing through the balloon filled with carbon
loudspeaker
loud sound received dioxide. The result is a sound wave with a larger
by microphone
amplitude.
CO2
microphone

balloon F
Figure 1.34 Balloon acts as a biconvex lens O

1
R

CHAPTER
M

If the balloon is filled with a less dense gas such as nitrogen or helium, the sound wave will be refracted away
from the normal when it passes from the air to the balloon. The balloon will act as a concave lens in this case.

1.3

1 Complete the following table using the words, 3 In the following figure, ABCD is an area of deep water.
‘increase’, ‘decrease’ or ‘unchanged’ as applicable. Plane waves with a velocity of 2 cm s–1 propagate
from a shallow area to a deeper area.
Water wave passes from
Characteristic an area of shallow water to
an area of deeper water A 5 cm B
1 cm
Speed
Wavelength
Frequency

2 If a plane wave is incident on an area of shallow D C


water with a plano-concave shape as shown below,
draw the pattern of the water waves when passing
through and after passing the area. What is the frequency and speed of the waves in the
area ABCD?
Activity 1.6

4 The following figure shows water waves from the sea


advancing towards a bay and a cape.
(a) Why are the speed and wavelength of waves in
the middle of the sea almost uniform?

335 Waves
parallel water waves
1.4 Analysing Diffraction
sea of Waves
1 Diffraction of waves is a phenomenon in
water waves
which waves spread out as they pass through
follow the an aperture or round a small obstacle.
shape of the
beach
2 The effect of diffraction is obvious only if
cape
(a) the size of the aperture or obstacle is
bay
small enough,
beach (b) the wavelength is large enough.
3 Characteristics of diffracted waves:
(b) Why do the distances between the wavefronts (a) Frequency, wavelength and speed of
decrease as the waves approach the beach? waves do not change.
(c) Why is the water in the bay calmer compared to (b) Changes in the direction of propagation
the water at the cape? and the pattern of the waves.

F
Diffraction of Water Waves
O
1

R
Activ To investigate the pattern of diffracted water waves
ity 1.7
CHAPTER

5
(I) Fixed wavelength
Apparatus/Materials
Ripple tank, metal bars, mechanical stroboscope and a piece of white paper.
Arrangement of apparatus
lamp
F
O
1

R
CHAPTER

M
wooden bar motor
5
rubber band
metal bar
water
aperture slit
ripple tank

shadow

white paper

Figure 1.35 Investigation of the diffraction of water waves

Procedure 3
F5/1/24a The motor (vibrator) is switched on and the
Activity 1.7

1 A ripple tank is filled with water and is set up as rheostat is adjusted so that the wooden bar
shown in Figure 1.35. produces plane waves with a wavelength of
2 Two pieces of metal bars are positioned to form a approximately 4 cm.
slit of width 10 cm at a distance of 5 cm from the 4 The pattern of the waves before and after passing
vibrating wooden bar. the slit is observed and drawn.

Waves 336
5 Steps 2 to 4 are repeated with different widths of (II) Fixed size of aperture
the slits: 6 cm and 2 cm respectively.
6 The two pieces of metal bars are replaced with a Procedure
single metal bar of length 10 cm placed 1 A ripple tank is filled with water and is set up as
centrally at a distance of 5 cm from the vibrating shown in Figure 1.35.
wooden bar. 2 Two pieces of metal bars are positioned to form
7 Steps 3 and 4 are repeated with different lengths a slit 5 cm wide at a distance of 5 cm from the
of metal bar: 6 cm and 2 cm respectively. vibrating wooden bar.
3 The vibrator is switched on. The frequency of
Results/Discussion the water wave is gradually increased by
adjusting the rheostat.
Table 1.7 4 The patterns of the waves passing through the
slit at different frequencies are observed and
(I) Fixed wavelength drawn.
(a) Large slit (b) Small slit
λ λ Results/Discussion

Table 1.8
a a
(II) Fixed size of slit F
O
(a) Waves of high (b) Waves of low

1
R
frequency (short λ). frequency (long λ).

CHAPTER
M

λ λ
Observation Observation 5
If λ  a, the effect of If λ  a, the effect of
diffraction is not diffraction is very
a a
obvious. obvious.
The waves are bent The waves are circular
only at the edges and appear to originate
after passing the slit. from the small slit.
Observation Observation
(c) Large obstacle (d) Small obstacle At a higher frequency, At a lower frequency,
the wavelength of the the wavelength of
water wave is short. the water wave is long.
If λ  a, the effect of If λ  a, the effect of
diffraction is not diffraction is very
obvious. obvious.

Conclusion
It is observed that the effect of diffraction of a water
Observation Observation wave becomes obvious as the frequency of the wave
The effect ofF5/1/24c F5/1/24c
The effect of diffraction decreases (wavelength increases).
diffraction is not is obvious. The waves
noticeable. recombine after passing
the obstacle.
Activity 1.7

Conclusion
As the size of the aperture or obstacle decreases, the
effect of diffraction becomes obvious.

337 Waves
9
In an experiment on the diffraction of water waves, a dipper with a frequency of 8 Hz produces waves with a
wavelength of 2 cm. If the frequency of the dipper is 10 Hz, what is the wavelength of the water waves produced?
Solution
Using the formula: v = fλ where fλ is constant as
f1 λ1 = f2 λ2 the velocity of the wave
is constant in a ripple
8 × 2 = 10 × λ2 tank of uniform depth.
λ2 = 1.6 cm

Diffraction of Light

1 Light is diffracted if it passes through a narrow slit comparable in size


to its wavelength.
However, the effect is not obvious as the size of the slit increases. This is
because the wavelengths of light are very short.
2 Two activities below show how diffraction of light waves and patterns of
F diffraction may be observed.
O (a) Small slit
1

R
CHAPTER

M
observer
5 bulb single slit
plate

Figure 1.36 Diffraction of light through a small slit

(b) Small pin hole

F
observer O
1

bulb R
black card
CHAPTER

M
Figure 1.37 Diffraction of light through a small pin hole
5
3 Diffraction of light is hardly noticeable compared with diffraction of
sound waves and water waves because the wavelength of light is very
short (approximately 10–7 m). Monochromatic light can be
4 Light waves will be diffracted if produced from visible light
(a) light is propagated through a pin hole or a tiny slit where its size is using a colour filter.
similar to that of the light wavelength (around one hundredth of a
millimetre or less).
(b) the light source is monochromatic, i.e., light of one colour, and
therefore of one wavelength only. Have you met our
Oh, my new teacher Ms Song?
students
Diffraction of Sound Waves like me!

Very often, we can hear people talking behind a doorway. Also, we can
hear an aeroplane flying over our house when we stay indoors. All Yes, she is
these are the phenomena of diffraction of sound waves. Sound can very kind...
bend around corners (diffract) like water waves and light waves.
Figure 1.38 We can hear people
talking behind a
doorway

Waves 338
Activ To investigate the diffraction of sound waves
ity 1.8
Apparatus/Materials 4 The position of the radio is changed and the
Radio effect on the sound of the radio is listened to.
Arrangement of apparatus Results
The student is able to hear the sound of the radio
although it is behind the wall (beyond her vision).
Discussion
The sound of the radio spreads around the corner of
the wall due to diffraction of sound.

student diffraction of sound


radio
S building
Figure 1.39 Investigation of diffraction of sound
F5/1/26 R Key:
R: radio
Procedure S: student

1 A radio is placed near a corner of a wall. sound waves

2 A student is requested to stand on the other side F


of the corner of the wall so that the radio is Figure 1.40 O

1
R
beyond her vision. Conclusion diffrac

CHAPTER
M
3 The radio is turned on and the student is The sound is able to spread around a space or an of sou

requested to listen to the radio. obstacle. 5

sound
waves
Sound waves are more easily diffracted in comparison to light waves because the
wavelength of sound waves is much longer than the wavelength of light waves.

The effect of diffraction of a wave is obvious if


(a) the size of the aperture or obstacle is small (a  λ), Wavelength decreases after diffraction, (i.e., λ1 > λ2).
(b) the frequency of vibration is low (frequency of a wave is inversely proportional λ1
to its wavelength). λ2

Wavelength decreases after diffraction, (i.e., λ1 > λ2). Wavelength remains unchanged after diffraction,
λ1 (i.e., λ1 = λ2).
λ2 λ1
λ2
Activity 1.8

Wavelength remains unchanged after diffraction,


(i.e., λ1 = λ2). 339 Waves
λ1
λ2
7 SPM
Clone
’03

The figure below shows an arrangement of the Which pattern will appear on the screen?
apparatus for an experiment investigating a light A C
phenomenon.

screen

B D

single slit
sodium plate
lamp Comments
to power supply
Light diffraction is the light phenomenon investigated
in this experiment. In light diffraction, we should
observe a wider middle fringe with narrower fringes
on both sides.
F
O Answer C
1

R
CHAPTER

5
Transmission and reception of radio
waves
Diffraction allows radio signals to spread to radio
transmitter
areas behind an obstacle. Radio signals in
amplitude modulated (AM) waves of long
wavelength (ranging from 186 – 560 m)
can spread around large obstacles such as
receiver
tall buildings and hills. Thus, a radio
broadcast can be received with clarity.
Radio signals in frequency modulated (FM)
waves which have short wavelengths (in the
range 2.8 – 3.4 m) can pass through small
obstacles such as tunnels and bridges.

1.4

1 The effects of diffraction of water waves are clearly


visible if the width of the gap is _______________ .
2 When the width of the gap or size of the obstacle
increases, the effect of diffraction becomes port
_________________ .
3 When the width of an aperture is small or approaches sea wave
the dimension of the wavelength of the water wave, wall barrier
the pattern of the diffracted waves appears like
_________________ waves.
Draw the pattern of the waves after passing through
4 The following diagram shows the construction of a
the opening between the walls. How does the wall
wall at the sea front of a port to protect the port from
protect the port?
being hit by large waves.

Waves 340
3 Constructive interference occurs when the
1.5 Analysing Interference crests or troughs of both waves coincide to
of Waves produce a wave with crests and troughs of
maximum amplitude.
SPM
Principle of Superposition ’08/P1 4 Destructive interference occurs when the
crest of one wave coincides with the trough of
1 The Principle of superposition states that at the other wave, thus cancelling each other
any instant, the wave displacement of the with the result that the resultant amplitude
combined motion of any number of interacting is zero.
waves at a point is the sum of the displacements 5 Figure 1.42 shows the occurrence of constructive
of all the component waves at that point. interference and destructive interference.
2 Figures 1.41 (a), (b) and (c) show the
combined amplitude produced when two
waves, both of amplitude a, from opposite + =
directions meet.
in phase maximum amplitude

(a) Constructive interference (two waves are in phase)


+ =

out of phase zero amplitude F


O

1
(a) Superposition of two crests (b) Destructive interference R

CHAPTER
M
Figure 1.42 Result of interference (two waves 180° out
of phase) 5

6 An antinode is a point where constructive


interference occurs, whereas a node is a point
where destructive interference occurs. From
Figure 1.43, it can be seen that the antinodal
line joins all antinodes while the nodal line
(b) Superposition of two troughs
joins all nodes.
antinodal line
(constructive interference)

nodal line
(destructive interference)
nodal line
antinodal line
(c) Superposition of a crest and a trough antinodal
line
Figure 1.41 Principle of superposition
wave crest
3 The interference pattern is the result of the
superpositions of waves. wave trough

SPM
Interference of Waves ’08/P2

S1 S2
1 Interference is the superposition of two
waves originating from two coherent sources. waves sources
Sources which are coherent produce waves of
Key:
the same frequency (f), amplitude (a) and are
• Maximum crest waves (two crests meet)
in phase.
× Zero amplitude waves (trough meets crest)
2 The superposition of two waves emitted from
coherent sources gives either constructive or ° Maximum trough waves (two troughs meet)
destructive interference. Figure 1.43 Interference of waves

341 Waves
Activ To investigate the interference of water waves
ity 1.9
Apparatus/Materials Procedure
Ripple tank, spherical dippers, metre rule and 1 A ripple tank is set up with two spherical
mechanical stroboscope. dippers in contact with the surface of the water.
Arrangement of apparatus 2 The distance between the two spherical dippers
(I) Different wavelengths (λ) is fixed at 4 cm.
to electrical supply 3 The motor and the rheostat is adjusted to operate
motor
the motor at low frequency.
wooden bar
4 The pattern of the interference of the waves is
observed with a stroboscope and the pattern is
S2 drawn.
5 Steps 3 and 4 are repeated with the vibrator
S1 operating at a higher frequency. The waves
produced have a shorter wavelength.

Figure 1.44 Investigation of interference of waves

F
O Observations
1

R Table 1.9 Pattern of interference of waves at different wavelengths, λ


CHAPTER

M
(a) Low frequency (large λ) (b) High frequency (small λ)
5

S1 nodal line
S1
nodal line
a x a x

nodal line nodal line


S2
F S2
O
1

R
D
CHAPTER

M D

As λ increases, x increases 5
As λ decreases, x also decreases

Discussion shorter, the distance between two adjacent nodal


1 Table 1.9 shows that when the two waves are lines or antinodal lines, x, is smaller.
combined, they produce waves of larger Conclusion
amplitude, as shown by the regions which are The distance between two consecutive nodal lines
bright (where the crests of the two waves or antinodal lines (x) increases when the wavelength
coincide) and the darker regions (where the of the water waves (λ) from the source increases.
troughs coincide). This part of the pattern of ⇒x∝λ
waves shows the crests and troughs which are x
reinforced as a result of constructive interference.
2 In some regions where the water is still, without
any ripples, the crests and troughs of the two
waves coincide and eliminate each other x∝λ
Activity 1.9

(destructive interference). Since the amplitude


is zero, there is no wave motion.
λ
3 The results in Table 1.9 shows that when the O
frequency is higher, i.e., the wavelength is Figure 1.45

Waves 342
(II) Different distances between the two sources of waves (a) SPM
’09/P3
Procedure
1 The ripple tank is set up as shown in Figure 1.44.
2 The distance between two spherical dippers, a, is fixed at 4 cm.
3 The motor is switched on and the rheostat is adjusted to obtain waves with medium wavelength.
4 The interference pattern is observed with a stroboscope and the pattern is drawn.
5 Steps 3 and 4 are repeated with the distance between the two spherical dippers reduced to 2 cm.

Observations
Table 1.10

(a) a = 4 cm (b) a = 2 cm
nodal line

S1 nodal line
S1
a x
x
a
S2 nodal line S2

nodal line
D F
O
As a becomes larger, x becomes smaller As a becomes smaller, x becomes larger

1
R

CHAPTER
M
Table 1.10 shows that when the distance between the two sources, x x
a, decreases, the distance between two consecutive nodal lines 5
increases.
Conclusion
The distance between two consecutive nodal (or antinodal) lines, x,
is inversely proportional to the distance between the two sources. O
a
O
1
(a) (b) a
1
⇒x∝ Figure 1.46
a

Relationship between λ, a,
between , a,xxand
andDD

antinodal line antinodal line


antinodal line
Key:
a = distance between two coherent sources
λ = wavelength
D x = distance between two consecutive nodal
(or antinodal) lines
D = distance from the two sources to the point
of measurement of x
S1 a S2
antinodal antinodal antinodal
trough crest line line line

Figure 1.47 Interference of water waves


F5/1/102
From the interference pattern (Figure 1.47) and the Combining the two relationships:
factors that influence the interference pattern in λ D

Activity 1.9, we found that: ⇒x∝


a
Activity 1.9

1 λ
x ∝ λ and x ∝ ⇒x=D where D is a constant
a a
S a S2
λ = –ax
1

trough∴ — crest
D
F5/1/102
343 Waves
8 SPM
Clone
’11

Diagram shows the pattern of interference produced Node Antinode


by two coherent sources of water waves. A P Q
B Q R
P R
C Q S
crest
Q
S D R P
trough
Comments
Node is a point of intersection of a crest and a trough
coherent sources
Antinode is a point of intersection of two crests or
Which of the following pairs of points is correct ? troughs
Answer A

single-slit
Interference of Light Waves SPM
’08/P2 plate double-slit
magnifying
plate
lens
S2
1 Interference of light waves, like that of water S1

F waves and sound waves, also requires two sodium-vapour


O
coherent sources. lamp
1

R interference
2 Waves emitted from two coherent sources scale
CHAPTER

M pattern
have the same frequency (or wavelength) and
5 are in phase. Figure 1.48 Young’s double-slit experiment
3 Light emitted by a single source consists of (c) Interference occurs as a result of the
waves which extend over a wide range of superposition of the two light waves
wavelengths and are not in phase. Because of originating from S1 and S2. A pattern
this, it is difficult to have two sources of light consisting of a series of parallel and
which are coherent. alternating bright and dark fringes is formed.
4 In 1801, Thomas Young produced two coherent (d) The bright fringes are regions where
light sources in his experiment now referred to constructive interference occurs, whereas
as Young’s double-slit experiment. the dark fringes are regions of destructive
5 Principle of Young’s double–slit experiment: interference.
(a) Yellow light emitted by a sodium-vapour (e) Figure 1.49 shows the interference pattern
lamp has a very narrow frequency band. obtained in Young’s double-slit experiment.
For all practical purposes, it can be con-
sidered as a monochromatic light which
is light of only one frequency or wavelength.
(b) Slits S1 and S2 give rise to two coherent light
sources since the light passing through
them are from the same monochromatic Figure 1.49 Interference pattern
light, the sodium-vapour lamp. (f) The widths of the bright and dark fringes
are the same.

Activ
ity 1.10
To study the interference pattern of light waves and to
ax
determine the wavelength of light using the formula λ =
D
Apparatus/Materials
Activity 1.10

Sodium-vapour lamp, single-slit plate, double-slit plate, metre rule, retort stand and a translucent screen.
Procedure
1 The apparatus is set up as shown in Figure 1.50. The experiment is carried out in a dark laboratory
room.

Waves 344
2 The light beam from the lamp is directed towards ∴ Wavelength of red light, λ
the double-slit plate. The translucent screen is ax
adjusted to obtain a clear interference pattern. =
D
3 The interference pattern on the screen is 5 × 10–4 × 3.2 × 10–3
observed and drawn. =
3
4 The distance across 11 consecutive bright = 5.33 × 10–7 m
fringes (that is, 10x) in the interference pattern is
measured. Discussion
5 The distance D is determined from the experiment
and the wavelength of yellow light is calculated.
single-slit plate double-slit plate translucent screen S1
x
a
lamp
S2 x
lamp
eye λ
D dark fringe
25 cm
bright fringe
double slit
Figure 1.52 Interference pattern
F
1 The interference pattern consists of a series of O

1
alternating bright fringes and dark fringes. R
The bright fringes are regions, called the

CHAPTER
M
antinodes, where constructive interference
5
occurs. The dark fringes are the nodes where
destructive interference occurs.
Figure 1.50 To study the interference of light waves:
2 To determine the wavelength of light, λ, in
Young’s double-slit experiment
Young’s double-slit experiment, three quantities
Results must be determined: separation between the two
slits, a; distance between the screen and the
32 mm double slit, D, and the distance between two
consecutive bright fringes or dark fringes, x.
The wavelength is then determined with the
formula:
ax
λ=
D

Figure 1.51 Interference pattern 3 Interference of light occurs when an incident


light wave passes through a single slit to a
Sample calculation: double slit S1 and S2. An interference pattern is
produced as a result of the superposition of two
Separation between the slits, a
light waves emerging from the double slits.
= 0.5 mm
The sizes of the two slits must be equal to or
= 5 × 10–4 m
smaller than the wavelength of the light wave to
From Figure 1.51, the distance across 11 consecutive produce a clear interference pattern.
fringes = 32 mm
∴ Distance between two consecutive bright fringes, Conclusion
Activity 1.10

x = 3.2 mm Interference is a phenomenon experienced by light


= 3.2 × 10–3 m 10 x = 32 mm
waves. The interference pattern produced by this
Distance between the double slit and screen, D experiment can be used to determine the wavelength
= 3.0 m of monochromatic light.

345 Waves
11
• A double-slit glass insert is
A monochromatic light source with a wavelength of 600 nm is used in a normally coated with
Young’s double-slit experiment. A series of interference fringes is produced on graphite powder to enable
a screen placed at a distance of 2.5 m from the double slits. What is the distance it to absorb light.
between the double slits if the distance between two consecutive dark fringes • White light is refracted into
is 1.2 mm? 7 major colours by a glass
prism. White light can be a
Solution source of monochromatic
Given: D = 2.5 m light (one colour and one
x = 1.2 mm wavelength) with the use
= 1.2 × 10–3 m of colour filters.
λ = 600 nm
= 6 × 10–7 m 1 nm = 10–9 m
ax:
Using the formula λ = __
D
λD
∴ a= x

6 × 10–7 × 2.5
F =
O 1.2 × 10–3
1

R = 1.25 × 10–3 m
CHAPTER

M = 1.25 mm
5
∴ The distance between the two slits is 1.25 mm.

Activ
ity 1.11
To investigate the factors affecting the interference pattern of
light waves

Apparatus/Materials 5 Steps 2 to 4 are repeated using a blue filter and a


Vertical filament lamp, single-slit plate, double-slit yellow filter respectively to replace the red filter.
plate, colour filters (red, blue and yellow), metre Results
rule and translucent screen. Table 1.11
Arrangement of apparatus
Colour Distance,
double-slit Interference pattern
single-slit plate plate screen filter x (mm)
diffraction xr

a interference Red xr
lamp
colour
filter D xy

Figure 1.53 To investigate the interference of light Yellow xy


Procedure
1 The apparatus is set up, as shown in Figure 1.53, xb
in a dark laboratory room.
2 A red filter is placed between the lamp and the Blue xb
Activity 1.11

single-slit plate.
3 The lamp is switched on. The interference pattern
formed on the screen is observed and drawn. Observations
4 The distance across 11 consecutive bright Distance xr > xy > xb , where λr > λy > λb.
fringes is measured.

Waves 346
Discussion ax
4 From the formula, λ = :
1 A colour filter is used to produce D
monochromatic light, that is, light with one x∝λ
colour and one value of wavelength. x
2 Red, yellow and blue light have very short
wavelengths. It is observed that the distance
between two consecutive bright fringes, x
decreases as the wavelength of the light
decreases. λ
O
3 At a fixed distance between the double slit and
the screen (D), the interference pattern of red Figure 1.54 Relationship between x and λ
light has fringes which are spaced further apart Conclusion
with a lesser number of fringes compared to An increase in the wavelength of a monochromatic
the fringes in the interference pattern of blue light source results in an increase in the distance
light. between two consecutive bright fringes or dark
fringes.

F
O
12

1
R

CHAPTER
M
In a Young’s double-slit experiment, the distance between the double slits and the screen is 4.0 m and the
separation of the two slits is 0.5 mm. 5
Calculate the distance between two consecutive bright fringes for
(a) violet light with a wavelength of 4.0 × 10–7 m,
(b) red light with a wavelength of 7.0 × 10–7 m.
Solution
Given: D = 4.0 m and a = 0.50 mm = 5 × 10–4 m
Use the formula:
λD
Use the formula: x = a

(a) Violet light: (b) Red light:


4 × 10–7 × 4 7 × 10–7 × 4
x = x =
5 × 10–4 5 × 10–4
= 3.2 × 10–3 m = 5.6 × 10–3 m
= 3.2 mm = 5.6 mm

The distance across 6 consecutive bright fringes is 12 mm. bright fringe


1 2 3 4 5 6

Distance between 2 consecutive bright fringes, x = 12 = 2 mm


Activity 1.11

x x x x x
12 5x
Distance between 2 consecutive bright fringes, x = = 2.4 mm
5

347 Waves
visible light spectrum

red orange yellow green blue indigo violet

wavelength decreases
largest λ shortest λ

13
The wavelength of light can be determined with a Solution
double-slit plate. The diagram below shows the 7x = 35 mm
pattern of interference fringes obtained in a Young’s x = 5 mm
double-slit experiment. The separation distance of the = 5 × 10–3 m
two slits is 0.40 mm and the distance between the D = 3.0 m
screen and the double-slit plate is 3.0 m. a = 0.40 mm
= 4.0 × 10 –4 m
F
O
λ = ax
1

R
D
CHAPTER

M
4.0 × 10–4 × 5 × 10–3
5 35 mm =
3
Calculate the wavelength of light used in the
= 6.67 × 10–7 m
experiment.

Interference of Sound Waves

1 Like other types of waves, sound waves can also give rise to interference effects.
2 Coherent sound waves interfere with each other to produce areas of louder sound and softer sound.
3 Constructive interference occurs at the areas of louder sound whereas destructive interference
occurs at the areas of softer sound.

Activ
ity 1.12
To investigate the interference of sound waves

Apparatus/Materials Procedure
Loudspeakers, audio signal generator and metre rule. 1 The apparatus, as shown in Figure 1.55, is set up
on a table located in an open space (such as the
Arrangement of apparatus
area outside of the laboratory building) to avoid
audio signal generator
disturbance by reflected waves.
2 Using a metre rule, the distance between the two
loudspeakers, a, is fixed at 1 m.
3 The audio signal generator is switched on and
loudspeakers
its frequency is adjusted to a certain level.
4 A student (observer) is requested to walk along
Activity 1.12

a=1m
D a straight path at a distance of 2 metres from the
loudspeakers as shown in Figure 1.55.
straight line path
Results
Figure 1.55 Investigation of the interference of The student (observer) hears alternating loud and
sound waves soft sounds as she walks along the straight path.

Waves 348
Discussion x = distance between two consecutive loud or
1 The alternating loud and soft sounds are caused soft sounds,
by interference of the sound waves. The two D = distance between the straight path and the
loudspeakers are sources of two coherent sound loudspeakers.
waves as they are connected to the same audio From the formula of wave interference, the
signal generator. The loud sounds are produced distance between the two consecutive loud or
by constructive interference, whereas the soft Dλ
soft sounds is given by x = a .
sounds are produced by destructive interference.
The value of x increases when
audio signal generator
(a) the distance of the path from the sources of
the sound (loudspeakers), D is increased,
loudspeakers (b) the frequency of the audio signal generator
is decreased, i.e., the wavelength of the
sound wave, λ, is increased,
L : loud sound (c) the distance between the two loudspeakers,
L S L S L S L SL S L S : soft sound a, is reduced.
x

Figure 1.56 Interference pattern of sound waves Conclusion


The interference of sound waves occurs when the
2 The wavelength of a sound wave is given by the two coherent sound waves interact on the basis of F
formula λ = ax , where the principle of superposition to produce a pattern O

1
D R
of constructive interference (loud sound) and

CHAPTER
M
a = distance between the two loudspeakers, destructive interference (soft sound).
5

x x x x

D λ f a
O O O O
1 1
x∝D x∝λ x∝ x∝a
f

14
In an experiment on the interference of sound waves, Using v = fλ:
two loudspeakers are placed at a distance of 1.5 m
from each other. They are connected to an audio signal λ = v = 300 = 0.6 m
f 500
generator to produce coherent sound waves at a
frequency of 0.5 kHz. Calculate (b) D = 5 m, a = 1.5 m and λ = 0.6 m
(a) the wavelength of the sound wave if the speed of
sound is 300 m s–1, Using λ = ax :
D
(b) the distance between two consecutive soft sounds
at a perpendicular distance of 5 m from the source λD
Using x = a
Activity 1.12

of the sound.
Solution Using x = 0.6 × 5
1.5
(a) v = 300 m s–1,
f = 0.5 kHz = 500 Hz Using = 2 m

349 Waves
Interference of waves

Type of
wave Water wave Sound wave Light wave
Parameter

antinodal dark fringes


line L x screen
S
x
a L
antinodal S x
a a
line L
Diagram S
nodal line
D

antinodal loudspeakers D
line double-slit plate
D L = loud sound
S = soft sound single-slit plate

ray box

F
O Wavelength, λ The wavelength of water The wavelength of sound The wavelength of light depends
1

R waves is influenced by the waves is influenced by the on its colour.


CHAPTER

M frequency of the vibrator. frequency of the audio signal


generator.
5

D Distance between the Distance between the plane of Distance between the
spherical dippers and the the two loudspeakers and the double-slit plate and the screen
position marked x is path along which interference
measured can be detected

a Distance between the two Distance between the two Distance between the two slits
spherical dippers loudspeakers on the double-slit plate

x Distance between two Distance between two Distance between two


consecutive antinodal lines consecutive positions where consecutive bright fringes or
or two consecutive nodal lines a loud sound is heard dark fringes

ax
The formula, λ = __ can be used in the interference of all three types of waves.
D

1.5

1 From the points labelled in the diagram, list the nodes and the antinodes.

R
S
Q
T crest
P U V

trough

Waves 350
2 A plane wave approaches two small slits as shown in Sketch a graph to show the relationship between a
the diagram. and x.

7 In an experiment of light interference using red light


(λ = 6.5 × 10–7 m), two consecutive bright fringes
formed on the screen are 0.6 mm apart. When the
experiment is repeated using monochromatic light P,
it is found that the two consecutive bright fringes
(a) Draw the pattern of the water waves passing formed are 0.4 mm apart. What is the wavelength of
through the two slits. the light P?
(b) Mark the points where constructive interference
occurs and draw the antinodal lines in the diagram. 8 The diagram shows a Young’s double-slit experiment.
(c) Mark the points where destructive interference A yellow filter is placed between the light source and
occurs and draw the nodal lines in the diagram. the slits.
3 The diagram below shows the interference pattern
produced by two sources of water waves, S1 and S2.
colour double-slit
filter plate
white screen

S1
F
3.0 cm O

1
white R
1.0 cm
light

CHAPTER
source M

S2 5

The pattern of interference is drawn. The experiment


is repeated using blue filter followed by green filter.
The figures below are the patterns of interference
20 cm
which are not arranged in order. Write down the
Calculate the wavelength of the water wave using the correct colour filter for the respective patterns of
measurements in the diagram. interference.
4 In a Young’s double-slit experiment, the slits are
0.2 mm apart and the screen is 2.5 m away from the
slits. If the distance between five consecutive bright
fringes is 2.4 cm, what is the wavelength of the
monochromatic light?
5 A monochromatic light wave with a wavelength of (a) (b) (c)
450 nm is propagated through a double slit. Fringes
are formed on the screen that is 4.0 m away from the 9 The diagram below represents the fringe pattern
slits. If the distance between two consecutive bright obtained in a double-slit experiment when a
fringes is 1.6 mm, what is the separation of the slits? monochromatic light was used. The two slits were
6 In a Young’s double-slit experiment, the distance 0.05 cm apart and the distance from the double-slit
between the double slit and screen is fixed. As the plate to the screen was 2.5 m.
separation of double slit, a, varies, the distance
between two adjacent dark fringes, x also varies.
double-slit
white screen
plate
dark fringes

x
a
21 mm

monochromatic D dark fringes


light source Calculate the wavelength of the monochromatic light
used in the experiment.

351 Waves
the air in it using a vacuum pump. During this
1.6 Analysing Sound Waves process, the sound of the bell becomes less and
less audible (fainter).
The Nature of Sound Waves
dry cell
1 Sound waves are longitudinal waves which
require a medium for its propagation.
2 Sounds are produced by vibrations such as the
vibrations of the cone-shaped diaphragm of rubber band
a loudspeaker, guitar strings, column of air in a
musical instrument and a tuning fork.
3 Figures 1.57 (a) and (b) show the production
of sound due to the vibrations of a tuning fork electric bell

and the diaphragm of a loudspeaker. air is


glass jar
being removed

direction of propagation
of the sound waves vacuum pump
C R C R C R
λ λ Figure 1.58 Sound waves cannot be
propagated in a vacuum
F tuning fork
O (a) Vibration of tuning fork 10 Sound waves can propagate through solids,
1

R liquids and gases. The sound that we hear is


CHAPTER

M R C R C R usually propagated through the air. Sounds


direction of propagation
of the sound waves can also be heard while swimming; sound
5
loudspeaker waves can also travel through walls, ceilings,
(b) Vibration of diaphragm of a loudspeaker doors and roofs.
11 Investigations carried out using audio waves
Key: C = compression
R = rarefaction
and speakers show that the human ear is
λ = wavelength capable of hearing sounds with frequencies in
the range of 20 Hz to 20 000 Hz. Nevertheless,
Figure 1.57 Sound waves F
the ability to hear sounds in the upper part of
O
the
R frequency range declines with age.
1

4 When a tuning fork or the diaphragm of a


12 The
M following table shows the frequencies of
CHAPTER

loudspeaker vibrates, layers of air vibrate and


the sound energy is propagated through the air different types of sound waves.
5
around it in the form of waves. Table 1.12
5 The particles of air are compressed when the
prong of the tuning fork or the diaphragm Infrasonic Audio / Sonic Ultrasonic
of the loudspeaker moves forwards and < 20 Hz 20 Hz – 20 kHz > 20 kHz
rarefaction of the air occurs when the prong or
diaphragm moves backwards.
6 As the prongs of the tuning fork vibrate
backwards and forwards, a series of rarefactions
and compressions is produced in the air.
7 The air particles vibrate back and forth along
the direction of propagation of the sound
wave. Such waves are known as longitudinal
waves.
8 Sound waves cannot travel in a vacuum because Bats navigate in the dark Dolphins can make sound
there is no medium (particles) through which by emitting ultrasound from 7 to 120 kHz. Some
the sound energy can be propagated. pulses of 100 kHz and scientists believe that
9 Figure 1.58 shows an experiment to prove that detecting the echoes of dolphins can communicate
sound waves cannot propagate in a vacuum. A these pulses reflected by with each other with such
vacuum is created in the glass jar by removing obstacles. ultrasound.

Waves 352
Speed of Sound

1 The speed of sound, v, in a medium can be determined using the


formula v = fλ, where λ is the wavelength and f is the frequency of the
sound wave.
The SI units of v, f and λ are m s–1, Hz and m respectively.
2 The speed of sound in solids is greater than the speed of sound in
liquids which is greater than the speed of sound in gases.
3 Table 1.13 shows the speeds of sound in different mediums.

Table 1.13
Medium Speed of sound (m s–1)
Air (0 °C) 330 Sound waves travel faster in
solid  liquid  gas.
Water (0 °C) 1400
Concrete 5000

4 The speed of sound is unaffected by pressure. If the atmospheric


F
pressure changes, the speed of sound in air remains constant. O
5 The speed of sound increases with temperature. At the peak of high-

1
R
altitude mountains, the speed of sound is less than that at sea level.

CHAPTER
M
This is not due to the lower pressures but because of the lower
temperatures at the peak of mountains. 5

6 Figure 1.59 shows a comparison between the speed of sound in air and
the speeds of an airplane and a car.

supersonic airplane

sound in air (10 oC)

sound in air The loudness of a sound


(–40 oC)
wave is directly proportional
car to the square of the
speed amplitude of the sound
wave.
330 m s–1
⇒ Loudness of sound
Figure 1.59 Comparison of speeds ∝ (amplitude)2
loudness loudness

Loudness and Amplitude of Sound

1 The loudness of a sound is considered to be high or otherwise


depending on the hearing ability of a person. amplitude

2 Loudness is influenced by the amplitude of the sound wave.


loudness loudness

(a) Soft (b) Loud


2
amplitude (amplitude)

Figure 1.60 Idea of the loudness of sound

353 Waves
Activ
ity 1.13
To investigate the relationship between the amplitude and the
loudness of sound

Apparatus/Materials 5 The loudness of the sound and the form (shape)


Audio signal generator, microphone and cathode- of the sound wave displayed on the screen of the
ray oscilloscope. oscilloscope are observed.
Arrangement of apparatus Results SPM
’09/P1
Table 1.14
cathode-ray oscilloscope
Wave Amplitude of Loudness
audio signal
generator loudspeaker
form sound wave of sound
microphone
small low

medium medium
Figure 1.61
Procedure large high
1 The apparatus is set up as shown in Figure 1.61.
2 The cathode-ray oscilloscope and its internal
timebase unit is switched on. Conclusion
F The amplitude is the height of the crest of the wave
O
3 The audio signal generator is switched on and
the frequency of the sound wave is adjusted to a form displayed on the screen of the oscilloscope.
1

R
suitable level. The sound waves with low amplitude is soft,
CHAPTER

M
4 The loudness of the sound is varied from a low whereas the sound waves with large amplitude
5 to a high level gradually. is loud.

Pitch and Frequency of Sound

1 The pitch of a sound or a musical note is an indication of how high or low the sound is.
It is a subjective judgement which varies with different individuals.
F
2 The pitch of a sound is determined by its frequency: a highO pitch corresponds to a high frequency.
1

R
CHAPTER

(a) Low pitch


(a) Low pitch (b) High pitch
(b) High pitch
Figure 1.62 Idea of the pitch of sound

Variations in the pitch and loudness of sound. pitch loudness


increased increased

Pitch Loudness
Activity 1.13

original
signal

pitch loudness
reduced reduced

Waves 354
Activ
ity 1.14
To investigate the relationship between the pitch and the
frequency of sound
Apparatus/Materials
Audio signal generator, microphone and cathode-ray oscilloscope.
Procedure
1 The apparatus is set up as shown in Figure 1.61.
2 The cathode-ray oscilloscope and its internal timebase unit is switched on.
3 The audio signal generator is switched on and the loudness is adjusted to a suitable level.
4 The frequency of the sound is varied from low to high gradually.
5 The pitch of the sound that is heard and the form of the wave displayed on the screen of the oscilloscope
are observed.
Results Discussion
Table 1.15 The frequency of the sound is represented by the
number of waves displayed on the screen of the
Wave form Frequency Pitch of
cathode-ray oscilloscope.
of sound sound
At a fixed amplitude of a sound wave, the pitch
low low increases when the frequency of the audio signal
generator is increased. F
O

1
R
medium medium Conclusion

CHAPTER
M
The pitch of a sound increases as its frequency
increases. 5
high high

Applications of Sound Waves


• Sounds with frequencies which change randomly are
known as noise. Cleaning with Ultrasonic Waves

1 A goldsmith uses high frequency sound waves


to dislodge dirt particles adhering to jewellery
and precious stones.
2 Dentists use ultrasonic waves to remove
plaque from teeth.
• The same tuning fork, when it is struck with a large
or smaller force, produces sounds of the same pitch. Locating an Object under Water
The magnitude of the force applied to the tuning fork
has no influence on the pitch of the sound produced 1 SONAR (Sound Navigation and Ranging) is a
(the frequency is the same) but the amplitude is
system used to detect underwater objects or to
increased.
determine the depth of the water by means of
an echo.
2 During World War I, sonar had been used to
detect the location of shoals of fish.
3 Sonar equipment emits a high frequency
Activity 1.14

tuning fork
sound signal which is reflected by the object in
the water. The reflected sound wave is received
(a) Struck the fork softly (b) Struck the fork by the sonar equipment. The time taken for the
hard echo to return is used to determine the
distance of the object below the water surface.

355 Waves

pitch loudness
increased increased
Calculating Distances Using the Reflection of
Sound Waves

1 The phenomenon of the reflection of sound


waves is used to determine the distance

sound waves
between two objects using the formula d = tv ,
2
produced by
echo a sonar where d is the distance to be determined, t is
the time interval between the instant the
sound is emitted to the instant the reflected
shoal of fish sound returns to the ranging device.
net 2 A mountain climber uses this principle to
estimate the width of the valley.
Figure 1.63
F5/1/32
d

Disintegration of Kidney Stones

F
1 Previously, kidney stones were removed by an
O operation. Presently a method which does not reflected
1

R require an operation has been introduced. sound


CHAPTER

M 2 Sound waves of high energy are directed at the


wall
kidney stones to destroy them in the cavity of
5
the kidney. The disintegrated particles are
removed during urination by the patient. Figure 1.64 Determination
F5/1/140 of distance

F
15 O
1

R
CHAPTER

In Figure 1.65, two vertical walls P and Q are 60 m Solution


M
apart. A boy standing at K, 27 m from the wall P, claps vt
his hands once. Using
5 the formula d = :
2
2d
∴ t=
v
60 m t1 = time taken between the clap and the first echo
K
from wall P
P Q t2 = time taken between the clap and the first echo
from wall Q
27 m
2 × 27
t1 = s
330
Figure 1.65
2 × (60 – 27)
t2 =
330
If the speed of sound in air is 330 m s–1, what is the 2 × 33
t2 = s
time interval between the first and the second echo 330
that he hears? 2 × 33 2 × 27
The time interval = –
60 m 330 330
12
K =
P Q 330
27 m = 0.036 s

Waves 356
9 SPM
Clone
’11

Figure below shows sound waves produced by a Comments


loudspeaker. The sound propagates in the air at the v = 330 m s-1,  = 0.8 m
speed of 330 m s-1. Using the formula v = f
v
f =
0.8 m

330
=
0.8
= 412.5 Hz
The frequency of sound wave is Answer C
A 206.25 Hz C 412.5 Hz
B 264 Hz D 825 Hz

1.6

1 A school teacher shows a method of measuring the (b) Which wave form shows a sound wave with the
speed of sound to a group of students. Two lowest pitch? Explain your answer.
microphones A and B and a balloon are positioned in 3 Two speakers, separated by a distance of 2.0 m, F
line as shown in the diagram below. produce sound waves with a frequency of 400 Hz. O

1
If the speed of sound is 330 m s–1, calculate the R

CHAPTER
M
2.5 m 1.5 m perpendicular distance from the two speakers, given
A B that the distance between consecutive loud and soft
5
balloon switch sounds is 60 cm.
4 A student is standing at a distance of 45 m from a wall.
switch
He gives a loud clap and the echo is heard after 0.3 s.
millisecond
electronic timer

Each microphone is connected to an electronic timer


via a sound activated switch. When the sound waves
from the explosion of the balloon reaches the
microphone A, the electronic timer is activated,
whereas the electronic timer is switched off when the
45 m
sound waves reach the microphone B. After the
balloon is exploded, the electronic timer registered a Calculate the speed of sound in air.
reading of 6 ms.
5 A student is standing between two cliffs A and B at
(a) Write 6 ms in units of s.
a distance of 400 m and 150 m from each cliff.
(b) What is the distance travelled by sound in 6 ms?
The student gives a loud scream.
(c) Calculate the speed of sound in air.
2 A microphone is connected to a cathode-ray
oscilloscope. Three types of sound waves are received
by the microphone. The wave forms A, B and C
displayed on the screen of the cathode-ray oscilloscope
are obtained as shown in the diagrams below.
B A

150 m 400 m

Calculate the time interval between the two echoes


he hears if the speed of sound in air is 300 m s–1 .
A B C
6 An audio signal generator produces sound waves in
(a) Which wave form represents a sound wave of the air. The wavelength is λ when the frequency is f.
large magnitude? Explain your answer. Find the wavelength when the frequency is 3f.

357 Waves
1.7 Analysing 3 Electromagnetic waves carry energy and
Electromagnetic Waves momentum which may be imparted when
they interact with matter.
Electromagnetic Spectrum 4 Electromagnetic waves consist of a series of
waves whose frequencies and wavelengths
1 When you watch television, listen to the radio extend over a broad range.
or cook something in a microwave oven, you 5 Waves in the electromagnetic spectrum vary in
are using electromagnetic waves. size in terms of wavelength from very long
2 Electromagnetic waves are propagating waves radio waves to very short gamma rays.
in space with electric and magnetic 6 Visible light waves are the only
components. These components oscillate at electromagnetic waves we can see. We see these
right angles to each other and to the direction waves as the colours of the rainbow. Each
of propagation as shown in the Figure 1.66. colour has a different wavelength. Red has the
longest wavelength and violet has the shortest
magnetic field (B)
wavelength. When all the waves are seen
together, they make white light.
electric field (E)
7 When white light shines through a prism or
through water vapour, the white light is
F separated into the colours of the visible light
O propagation direction spectrum.
1

R wavelength (λ)
8 The electromagnetic spectrum is the range of
CHAPTER

M
frequencies and wavelengths over which
5 electromagnetic waves are propagated.
Figure 1.66 Electromagnetic wave The spectrum is shown in Figure 1.67.

F
O
1

R
CHAPTER

M
SPM
5 ’09/P1
buildings human tennis ant cell bacteria virus DNA water atomic
ball molecule nucleus
about the
size of....

longer wavelength (m) shorter


4 3 2 –1 –2 –3 –4 –5 –6 –7 –8 –9 –10 –11 –12
10 10 10 10 1 10 10 10 10 10 10 10 10 10 10 10 10 10–13

105 106 107 10 8 109 1010 1011 1012 1013 1014 1015 1016 1017 1018 1019 1020 1021
lower frequency (Hz) higher

radio waves infrared visible ultraviolet X-rays gamma rays


light
LW, MW, SW, VHF UHF microwave

Figure 1.67 Electromagnetic spectrum


F5/1/37
Waves 358
Sources of Electromagnetic Waves at a specific energy level. When a charged
electron moves from an orbit with a particular
1 Matter is made up of elementary particles energy level to one of a lower energy level,
called atoms. electromagnetic waves are emitted.
2 Every atom has a nucleus at its centre which is 4 Electromagnetic waves are also produced when
surrounded by orbiting electrons. a charged particle (electron or nucleus)
3 Electrons are charged particles and they move oscillates.
around the nucleus in orbits, each of which is

SPM
Properties of Electromagnetic Waves ’07/P1

Transverse
waves Do not require a medium
to propagate and can
Energy is transferred travel in a vacuum
by the waves

Show characteristics The magnetic and electric F


Properties of field components of the O
of polarisation
electromagnetic

1
wave oscillate at right R

CHAPTER
waves angles to each other and M

The waves are electrically to the direction of 5


neutral propagation of the wave

Undergo the same In a vacuum, the waves Obey the wave equation,
phenomena as light: travel at the speed of c = fλ,
reflection, refraction, light, where c is the velocity of
diffraction and interference c = 3 × 108 m s–1 light, f is the frequency
of the wave and λ is the
wavelength

1 In vacuum, c is a constant for all electro- 10 SPM


Clone

magnetic waves. The formula, c = fλ shows that ’03

the frequency, f is inversely proportional to Photograph below shows the device used in scanning
the wavelength, λ. of baggage in an airport.
2 The velocities of an electromagnetic wave in
other mediums are different (less than) from
its velocity in vacuum.

Some graphs of c = fλ, where c is a constant.


Which of the following electromagnetic waves is
used in the process of scanning ?
c
A Gamma ray C Radio wave
B Infrared D X-ray
Answer D

359 Waves
Applications and Detrimental Effects of Electromagnetic Waves

Radio waves Microwave Infrared


Infrared
Range of
wavelength, (m) 10–1 – 10 4 10–3 – 10–1 10–6 –– 10
10 10–3

• Radio transmitter • Radar transmitter


transmitter •• Hot
Hot objects
objects
• Lightning • Stars •• Stars
Stars (e.g.
(e.g. Sun)
Sun)
• Stars • Microwave oven
oven •• Flame
Flame
Source

• For broadcasting and • Satellite transmissions •• Night


Night vision
vision system
system
wireless • Radar system •• Thermal
Thermal imaging
imaging
communications (navigation, weather •• Remote
Remote control
control for
for
• UHF (Ultra High forecast) TV/VCR
TV/VCR
Frequency) radio wave • Traffic speed camera
– television and • Global Positioning
F mobile phone System (GPS)
111

O
R
CHAPTER

M •• Burglar
Burglar alarm
alarm system
system
Application •• Auto-focusing
Auto-focusing
5 (camera)
(camera)
• VHF (Very High
High
Frequency) radio wave • Cooking (microwave
– local radio and oven)
wireless
telecommunications
(e.g. aircraft, ships, •• Data
Data transmission
transmission
taxi and police) F
O through
through optical fibre
1

R network
CHAPTER

5
•• potential
potential risk to • internal heating of • over-heat
health, causing
health, body tissue • skin burns
Danger
Danger
migraine, headache
migraine, • cataracts

9 ‘03

The figure below shows an electromagnetic spectrum.


spectrum.
Comments
Comments
K L Ultraviolet M N Microwave The electromagnetic spectrum in
order of increasing wavelength:
What are the waves at K, L, M and N?
gamma ray, X-ray, ultraviolet,
K
K LL M
M N
N visible light, infrared, microwave,
A
A X-ray gamma ray infrared visible
visible light
light radio wave
B
B gamma ray X-ray visible light
light infrared
infrared
C
C X-ray visible light infrared radio
radio wave
wave Answer B
Answer B
D
D gamma ray visible light radio wave infrared
infrared

Waves 360
Visible light Ultraviolet X-rays Gamma rays
9

• Stars (e.g. Sun) • White hot objects • X-ray tube • Radioactive substance
• Very hot objects • Mercury lamp • Stars • Cosmic rays
• Lamp/bulb • UV lamp • Stars
• Fire • Stars (e.g. Sun)

• Vision • Detection of fake • Radiotherapy • Radiotherapy (cancer


• Photosynthesis in banknotes • Medical imaging treatment)
plants (X-ray photograph) • medical imaging

F
O
• Sterilisation (drinking R

CHAPTER
M
• Photography water and medical
• Photocell equipment)
5

• Airport security check • Sterilisation (food


(luggage) and medical
• Detection of cracks in equipment)
building structures • Pest control in
agriculture
• Production of vitamin
D in human body
• Production of
fluorescent effects

• increase rates of • skin cancer • cell damage • cell damage


premature skin aging • sunburn • cancer • DNA mutation
• skin cancer • blindness • cancer
• damage to the retina

1.7
1 Arrange the following electromagnetic waves in order (a) Name the waves labelled A, B, C and D.
of increasing frequencies: (b) Name an electromagnetic wave with
infrared radiation, X-rays, radio waves (i) a very low frequency,
ultraviolet rays, visible light (ii) very low wavelength.
2 State three differences between electromagnetic (c) State two properties of electromagnetic waves.
waves and sound waves. (d) Name an electromagnetic wave which can be
3 The chart below shows the electromagnetic sensed by humans.
spectrum in order of increasing wavelengths. (e) State one application for each of the following
electromagnetic waves:
γ-ray A B visible light C microwave D (i) γ-ray (iii) microwave
(ii) visible light

361 Waves
1. Vibration or oscillation is a back and forth motion 11. The four phenomena of waves are reflection,
about an equilibrium position. refraction, diffraction and interference of waves.
2. In an oscillation, 12. For reflection of waves, angle of incidence is equal to
(a) period (T) is the time taken for a complete oscillation. angle of reflection and the reflected wave has the
(b) frequency (f ) is the number of complete same speed, same frequency and same wavelength
oscillations in one second. as the incident wave.
(c) amplitude (a) is the maximum displacement 13. Refraction of wave is the change in the velocity of the
from equilibrium position. wave as it propagates from one medium to another
1 medium.
3. Period and frequency are related by a formula, f = .
T 14. Diffraction is the spreading of waves after passing
4. Two types of waves are longitudinal wave and through a narrow gap or a small obstacle.
transverse wave. 15. Waves diffract more if the gap is narrower or the
(a) In a longitudinal wave, particles in the medium wavelength is longer (lower frequency).
vibrate in a direction parallel to the direction of 16. Interference is the effect produced by the
wave propagation. superposition of waves from two coherent sources,
Example: sound wave resulting in constructive interference and destructive
F (b) In a transverse wave, particles in the medium interference.
O vibrate in a direction perpendicular to the 17. Constructive interference occurs when two crests or
1

R direction of wave propagation. two troughs meet, whereas destructive interference


occurs when a crest meets a trough.
CHAPTER

M Examples: water wave and light wave


5. A wave motion can be illustrated by a displacement- ax
18. The interference formula: λ =
5 D
time graph and a displacement-distance graph.
6. Speed of a wave (v) is the distance travelled by the 19. Sound waves require a medium for propagation and
wave in a second. it cannot travel in vacuum.
7. Wavelength (λ) is the distance between two 20. In sound waves,
consecutive points of the same phase on a wave. (i) loudness increases with amplitude.
8. The wave equation: v = fλ (ii) pitch increases with frequency.
9. Damping occurs when the oscillating system loses 21. Electromagnetic waves comprise gamma ray, x-ray,
energy to the surroundings in the form of heat. The ultraviolet, visible light, infrared, microwave and
amplitude of oscillation decreases. radio wave.
10. Resonance occurs when a system is force to oscillate 22. Electromagnetic waves travel at a speed of
with a large amplitude or its natural frequency. 3.0 × 108 m s–1 in vacuum.

1
Multiple-choice Questions

1.1 Waves If the time taken for the wave to 2 Which diagram shows the
travel from P to Q is 0.7 s and SPM production of longitudinal wave?
Clone
1 Diagram 1 shows the propagating travel with the speed of 10 m s–1, ’09 A
SPM wave.
Clone what is its wavelength?
’10 A 1m
P Q B 2m
C 6m
D 12 m
Rajah 1

Waves 362
B displacement (cm) 8 Diagram below shows two wave
4
SPM
pulses produced on a slinky spring
Clone
’11 moving towards each other.

0 0.5 time (s) a a


–4 O

C Diagram 2 Which of the following wave forms


Which of the following is true for will be observe if they meet at O.
the system? A
2a
Amplitude Frequency
(cm) (Hz) B
D A 4 4 2a

B 4 2
C
C 8 4
D a
D 8 2

6 Diagram 3 shows the 9 Which of the following statement


SPM displacement-distance graph of a
Clone
SPM is correct when sound waves are
Clone

’08 wave. The velocity of the wave is ’11 reflected by a reflector?


F
50 cm s–1. A The wavelength of sound O

1
waves becomes shorter after R
reflection

CHAPTER
M
3 Diagram below shows the circular B The amplitude of sound waves
wavefronts originated from the becomes smaller after 5
point O. reflection
C The velocity of sound waves
before and after reflection are
the same.
Diagram 3
What is the frequency of the 10 Diagram 7 shows an interference
S wave? pattern of two coherent water
R
Q A 3.3 Hz C 6.25 Hz waves originating from points P
P B 5 Hz D 10 Hz and Q.

1.2–1.5 Wave Phenomena B C D


A trough
7 Diagram below shows the
crest
Wavelength is the distance direction of a water travelling from
between the points region P to region Q. P Q
A P and R P Diagram 7
B Q and S Q
C P and S V1
Which point labelled A, B, C or D
D R and S V2
gives zero amplitude?

4 If a soprano sings with a high note,


11 What happens to the wavelength
a thin piece of glass may break.
Clone and the speed of water waves
SPM
This phenomenon is called ’08 when it is diffracted?
A diffraction Which of the following is true?
B interference Wavelength Speed
A v1 > v2 and P is deeper than Q
C resonance
B v1 > v2 and P is shallower than A Unchanged Unchanged
5 Diagram 2 shows the Q B Increases Increases
displacement-time graph for C v1 < v2 and P is deeper than Q
D v1 < v2 and P is shallower than C Decreases Increases
the oscillation of a weight on a
spring. Q D Increases Unchanged

363 Waves
12 Diagram 8 shows water waves The marksman fires a shot and
passing a house during a hears two echoes from the walls.
flood. If the speed of sound is
300 m s–1, what is the time
interval between the two echoes?
sonar A 0.71 s C 1.75 s
B 1.42 s D 2.83 s
seabed
18 Diagram 11 shows sound waves
What is the depth of the sea if produced by a tuning fork.
the reflected sound wave is L M N P Q R
detected 120 ms later?
[Speed of sound wave in water
= 1.5 × 103 ms–1]
A 90 m C 270 m
B 180 m D 360 m Diagram 11
Diagram 8 The wavelength of the sound
15 Which of the following frequencies
Which phenomenon explains the waves is the distance between
is an audio frequency?
propragation of waves at the SPM A L and M C L and P
A 1.2 × 10 Hz Clone

region X? ’08 B M and P D N and R


B 1.2 × 103 Hz
A Reflection C 1.2 × 106 Hz
F B Refraction D 1.2 × 109 Hz 1.7
O C Diffraction Electromagnetic Waves
1

R 16 When a guitar string is strummed,


D Interference
CHAPTER

M
Clone a loud and high pitch sound is 19 Which of the following is the
SPM

13 The arrangement of the apparatus ’06 produced. Which of the following


correct relationship between the
5 in Diagram 9 is used to produce graphs represents the situation? wavelength of an electromagnetic
the interference fringes of red A wave and the energy it carries?
displacement, s (cm)
light.
Wavelength Energy
red double-slit
filter plate O time, t (s) carried
A Short Low
light B displacement, s (cm) B Short High
source screen
C Long High
O time, t (s)
gaps
20
Electromagnetic wave Letter
Diagram 9
C displacement, s (cm) Ultraviolet ray U
The distance between two Visible light N
consecutive fringes can be
increased by Microwave M
O time, t (s)
A increasing the size of gaps. X-ray X
B increasing the distance Radio wave R
between the screen and the D displacement, s (cm)
double-slit plate. Based on the representation of
C increasing the intensity of the electromagnetic waves by
light source. O time, t (s)
letters of the alphabet shown
D replacing the red filter with the in the table, which of the
blue filter. following sequences of letters
17 A marksman stands between two
represents the arrangement of
high walls as shown in Diagram 10.
the electromagnetic waves in
100 m 525 m ascending order of
1.6 Sound Waves wavelength?
A R, M, N, U, X
14 Diagram below shows the use of B X, U, N, M, R
SPM
Clone
sonar to determine the depth of C X, N, U, R, M
’10 sea. D U, N, X, M, R
Diagram 10

Waves 364
Structured Questions
1 Diagram 1 shows a slinky spring being moved forth (a) Why must the depth of water in the ripple tank be
and back. uniform throughout?
forward and backward
(b) Plane waves are produced by the vibration of a
straight edged wooden bar on the surface of the
water.
fixed (i) When the ripple tank is lighted up from
above, a pattern of dark and bright stripes
are formed on the white paper. Draw a
Diagram 1 ray diagram and indicate the positions of
(a) Complete the sentence below by ticking () the the dark and bright stripes as shown in
correct box. Diagram 3.
The wave produced by the slinky spring is a
P Q
transverse wave
longitudinal wave
(b) In Diagram 1, mark ‘X’ on any one of the
Diagram 3
rarefaction of the wave.
(c) Complete the sentence below. (ii) If the vibrating wooden bar produces 12
waves per second and the distance F
The direction of the hand movement is _________
between P and Q is 5 cm, what is the O
the direction of the wave movement.

1
R
velocity of the water wave?

CHAPTER
(d) What is transferred by the wave? M
(iii) Calculate the wavelength of the water wave
P
2 Diagram 2 shows two loudspeakers connected to an if the frequency of vibration of the wooden 5
audio frequency generator. Students are standing at bar is doubled.
positions where loud sounds can be heard. (iv) What is the name of the instrument which is
used to make the moving waves appear
stationary?
(c) A plane reflector is placed at an inclined angle at
the centre of the tank. Draw the pattern of the
reflected wave in Diagram 4.

loud audio frequency


speakers generator

Diagram 2
Diagram 4
(a) State the phenomenon of the sound wave
produced in Diagram 2. 4 Diagram 5 shows the use of sonar equipment to
(b) What type of wave is the sound waves? measure the depth of the sea under a ship.
(c) The wavelength of the sound produced is 50 cm.
At 8.0 m from the loudspeakers, the distance
between two consecutive rows of students is
3.6 m. Calculate the distance between the two
loudspeakers.
(d) If the frequency of the audio generator is
decreased, what will happen to the distance
between two consecutive rows of students where
loud sounds are heard?
(e) Give a reason for your answer in (d). Diagram 5
3 A ripple tank is set up to investigate a water wave (a) State the phenomenon of sound wave applied in
phenomenon. A lamp is fixed above the ripple tank sonar equipment.
and a sheet of white paper is placed directly below the (b) Sound waves with a frequency of 6.6 × 105 Hz are
ripple tank. used to determine the depth of the sea.

365 Waves
(i) Why is high frequency sound used in sonar equipment?
(ii) What is the wavelength of the sound wave in sea water?
(iii) If the time interval between the instant the sound wave is sent to the instant the echo is received
is 1.4 s, what is the depth of the sea?
(c) State two other uses of sonar.
[Speed of sound in sea water is 1500 m s–1]

Essay Questions
5 cathode-ray (c) In daily life we find that sounds can be heard
audio oscilloscope more clearly at night than in the day. Give an
signal
generator explanation for this. State two factors that affect
balloon the speed of sound in a gas. [5 marks]

6 Diagram 7 shows a child in a pool noticing that his


legs appear shorter in the pool water.

loudspeaker microphone
(a)
shallow area
F
O
1

R
CHAPTER

5 Diagram 7
Diagram 8 shows waves in the sea approaching the
beach.
water waves
(b)
Diagram 6
First situation F
The oscilloscope display shows an increase in the O
1

amplitude of the wave when a balloon filled with R


CHAPTER

carbon dioxide is placed between the loudspeaker M


shallow area sea
and the microphone as shown in Diagram 6(a). waves
5
Second situation
The water waves travel towards a convex shaped
Diagram 8
region of shallow water. The pattern of the water
waves after passing through the region of shallow (a) (i) State one similarity and one difference
water is shown in Diagram 6(b). between light waves and water waves.
(a) (i) State one similarity and one difference [2 marks]
waterwave.
between a sound wave and a water waves (ii) Study Diagrams 7 and 8. State in full the
[2 marks] characteristics of both situations.
(ii) Study Diagrams 6(a) and 6(b). State in full Relate the characteristics to a physics concept
the characteristics of both situations and and name the concept. [6 marks]
relate the characteristics to a phenomenon. (b) If a closed hall is not properly constructed, sound
What is this phenomenon called? waves within the hall are subjected to a certain
[6 marks] phenomenon which affects the clarity of sounds
(b) Water waves get closer together as they approach that are heard. Using a suitable concept in physics,
the edge of a pond. explain ways of overcoming the problem.
(i) Explain this observation. [6 marks]
(ii) With the use of suitable apparatus, explain (c) Fishermen are keen to increase their catch. Using
how you would verify that the frequency of Diagram 9, propose a scientific way that can be
the water wave is constant throughout its used by fishermen to determine the location of
propagation on the surface of the pond. shoals of fish before they lower their nets.
[7 marks] [6 marks]

Waves 366
(d) A ship sends an ultrasonic sound wave to the
seabed to determine the depth of the sea.
The echo of the ultrasonic sound wave is received
after 1.2 ms.
(i) What is the name of the phenomenon
being applied in this case?
(ii) If the speed of ultrasonic sound waves in
sea water is 2 × 106 m s–1, what is the depth
of the sea?
[5 marks]

8 As a researcher, you are investigating the characteristics


of various electromagnetic waves to determine their
suitability for use in the investigation of structural
defects in the construction of an engine.
Diagram 9 (a) What is meant by the term electromagnetic
wave? [1 mark]
7 Table 1 shows that sound waves are longitudinal (b) Table 2 shows the characteristics of five types of
waves with lower frequencies, whereas X-rays are electromagnetic waves.
transverse waves with high frequencies.
Table 2
F
Table 1
O
Type of Wavelength Energy Penetrating

1
R
Wave Frequency Velocity in air Type of wave wave (m) power

CHAPTER
M
Sound 105 Hz 330 m s–1 Longitudinal P 10–2 low low
5
X-ray 18
10 Hz 3 × 10 m s
8 –1
Transverse Q 10–2 medium low
R 10–4 medium medium
(a) (i) Explain the difference between a
S 10–8 high high
longitudinal wave and a transverse wave.
–4
(ii) Compare the wavelengths of sound waves T 10 high very high
and X-rays.
[6 marks]
Based on the above table:
(b) Diagram 10 shows the use of ultrasonic sound
(i) Explain which characteristics of
waves to obtain the image of an unborn baby in
electromagnetic waves must possess in
the womb of the mother.
order to be suitable for use in the
investigation of structural defects in the
construction of an engine.
(ii) Determine which electromagnetic wave is
most suitable for use in the investigation
probe and give reasons for your selection.
[10 marks]
(c) Electromagnetic waves travel at the speed of light
in vacuum. State the relationship between
wavelength and frequency.
Draw a frequency-wavelength graph.
[3 marks]
(d) A water ripple of wavelength 1.5 cm is produced
by a dipper which oscillates at a frequency of
Diagram 10
20 Hz.
Why are sound waves used instead of X-rays? (i) What is the velocity of the water wave?
[6 marks] (ii) If the wavelength of the water ripple is
(c) State one other use of ultrasonic sound waves changed to 4 cm, what is the frequency of
and two uses of X-rays in our daily life. the dipper?
[3 marks] [6 marks]

367 Waves
Experiments
1 An experiment is carried out to determine the
gravitational acceleration at a certain place.
A simple pendulum is set up as shown in thread
Diagram 1. A stopwatch is used to time 20
 cm
complete oscillations for different values of .

bob

Diagram 1
Table 1 shows the readings obtained in the experiment.
Table 1

Length of pendulum,  (cm) Time of 20 oscillations, t (s) Period of oscillation, T (s) T 2 (s 2)


40 25.4
50 28.6
60 31.4
F
O 70 33.5
1

R 80 36.1
CHAPTER

5 (a) Based on the information obtained in this (d) From your graph,
experiment, state (i) state the relationship between T 2 and ,
(i) the manipulated variable, [1 mark] [1 mark]
(ii) the responding variable, [1 mark] (ii) calculate the gradient of the graph.
(iii) a constant variable. [1 mark] [3 marks]
(b) Complete Table 1 with values of period, T and (e) Calculate the gravitational acceleration, g from the
T 2. [5 marks] 4π2
(c) Based on Table 1, draw the graph of T 2 against . F formula g =
O 100m , where m is the gradient of
[5 marks]
1

R
the graph. [3 marks]
CHAPTER

2 Diagram 2 shows light seen through two single slits of Using your knowledge of diffraction;
different sizes. (a) State one suitable inference. [1 mark]
Diagram 2(a) shows the diffraction pattern for the (b) State one appropriate hypothesis. [1 mark]
wider slit and Diagram 2(b) is the diffraction pattern (c) With the use of apparatus such as a ripple tank,
for the smaller slit. mechanical stroboscope and other apparatus,
describe an experiment to investigate the
hypothesis stated in (b).
In your description, clearly state the following:
(i) The aim of the experiment
(ii) The variables in the experiment
(iii) The list of apparatus and materials
(a) (iv) The arrangement of the apparatus
(v) The procedure used in the experiment.
Describe how to control and measure the
manipulated variables and how to measure
the responding variables
(vi) The way to tabulate the data
(vii) The way to analyse the data
(b) [10 marks]
Diagram 2

Waves 368
FORM 5

2
CHAPTER

Electricity

SPM Topical Analysis


Year 2007 2008 2009 2010 2011
Paper 1 2 3 1 2 3 1 2 3 1 2 3 1 2 3
Section A B C A B A B C A B A B C A B A B C A B A B C A B
Number of questions 4 1 – – 1 1 4 1 – – – – 4 – – 1 – – 5 1/3 – 1 – – 5 1 – – – –

ONCEPT MAP

ELECTRICITY

Electric field Electric charge, Q Electromotive


force (e.m.f.)

Electric field Current, I Potential difference, V


Q Internal
pattern = W
t = resistance, r
Q
t = time taken W = work done
Effects of an
electric field
on a charge
Ohm’s law

Circuits
Resistance, R
Electrical energy & power
V
=
I
Series Parallel
circuits circuits
Electrical energy, E Power, P
• E = Vlt • P = Vl
Effective resistance, R Effective • E = I 2Rt • P = I2R
= R1 + R2 + … 1 V2 V2
resistance,
R • E= t • P=
R R
1 1
= + +…
R1 R2

Solve problems Efficiency


involving combination Power output
= × 100%
of circuits Power input

COMPANION WEBSITE
Learning Objectives 369
2.1 Analysing Electric Fields and Charge Flow

Activ To show the relationship between electric charge and current


ity 2.1
Apparatus/Materials 3 The motor is turned off and the crocodile clip
Van de Graaff generator, crocodile clip and held by wooden tongs is brought in contact with
galvanometer. the metal dome of the generator.
Arrangement of apparatus 4 When the crocodile clip touches the sphere, the
galvanometer pointer is deflected. This shows
metal dome
that a current is flowing through the
galvanometer.
crocodile clip
Perspex
roller
5 The electric current is produced by the flow of
electron water tap electrons from earth through the galvanometer
rotating electron to the metal dome to neutralise the positive
rubber
belt charges on its surface.
galvanometer
6 When the galvanometer pointer has returned to
motor its zero position, the metal dome of the
earth
F polythene roller generator can be safely touched with the hand as
O Figure 2.1 all the positive charges on it have been
2

R Procedure neutralised.
CHAPTER

M Conclusion
1 Figure 2.1 shows a Van de Graaff generator.
5 2 The motor of the Van de Graaff generator is The above activity shows that a flow of electric
turned on for a few minutes to produce positive charge (electrons) through a conductor produces an
charges on the metal dome of the generator. electric current.

positive terminal of an electric cell or battery,


Relationship between Electric Charge and
to the negative terminal.
Current
4 Actually, the electromotive force of the cell or
1 Figure 2.2 shows a simple circuit, with its the battery causes the negatively-charged
schematic drawing. electrons to flow from the negative terminal
to the positive terminal through the wires.
5 From Activity 2.1, we know that an electric
A
current current is caused by a flow of electric charge.
+ – 6 In general, we define current as the rate of
B flow of electric charge.
6V Charge Q
Current = or I =
+ – Time t

(a) (b) 7 The SI units of current is the ampere, A and


Figure 2.2 charge is the coulomb, C.
2 A circuit is a closed loop through which 1C
∴1 A = = 1 C s–1
charges flow continuously. 1s
(a) External circuit : The path taken by the 8 A current in a circuit is 1 ampere if the charge
charges outside the cell or the battery. is flowing at the rate of 1 coulomb per second.
(b) Internal circuit : The path taken by the 9 By rearranging the above formula, we obtain:
Activity 1.2

charges within the cell or the battery. Q = It


3 The direction of a current, by convention, is 1 C = 1 A × 1s=1 A s
the direction in which a positive charge 10 Thus, a coulomb is the charge which passes
would move. In other words, the direction of a any point in a circuit when a steady current of
current in the external circuit is from the 1 ampere is flowing in 1 second.

Electricity 370
How Many Electrons Make Up 1 Coulomb?
1
1 The charge in an electron, e = –1.6 × 10–19 C.
A charge of 300 C flows through a bulb every
2 If N electrons make up 1 C of charge, then:
2 minutes. What is the electric current in the bulb?
Number of electrons, N × Charge in an electron
= Total charge, Q Solution
N×e = 1 2 minutes = 2 × 60 s = 120 s
N = 1 The negative Current, I =
Q
e t
sign is omitted
1C since we need 300 C
= = = 2.5 A
1.6 × 10 C
–19
to know the
number only. 120 s
= 6.25 × 1018

2 SPM
’04/P1
SPM
’05/P1
SPM
’07/P1

≡1C An electric current of 200 mA flows through a


resistor for 3 seconds. What is the
charge
= –1.6 × 10–19 C
(a) electric charge?
6.25 × 1018 electrons (b) number of electrons which flow through the
F
resistor? O
Figure 2.3

2
Solution R
3 Therefore, 6.25 × 1018 electrons are required to

CHAPTER
M
make up 1 coulomb of charge. (a) Q = It
200 mA = 200 × 10–3 A
4 If a current of 1 A is flowing on a point in a = 0.2 × 3 = 0.2 A 5

circuit, it means 6.25 × 1018 electrons pass = 0.6 C


through that point in 1 second! (b) Number of electrons
= 0.6 × 6.25 × 1018 As 6.25 × 1018 electrons
make up 1 C of charge.
= 3.75 × 1018

Electric Field

is pushed is pushed

+ + – –

d d
lle lle
pu pu
is is
+

(a) A positively charged sphere (b) A negatively charged sphere

Figure 2.4 Pattern of electric field lines

1 A positive or negative charge produces an 4 A positive test charge moves away in a radial
electric field in the space surrounding the direction from a positively charged sphere,
charge. Any other charge in the electric field whereas it is pulled towards the centre of a
experiences an electric force acting on it. negatively charged sphere.
2 The electric field can be represented by arrow 5 The pattern of the electric lines of force
lines as shown in Figure 2.4. depends on the shape of the charged object
3 The lines are called electric field lines or and the number of charged objects in the
electric lines of force. These lines show the vicinity.
path that would be taken by a positive test 6 An electric field line is a vector quantity as it
charge placed in the electric field. has both force and direction.

371 Electricity
Activ To show the existence of an electric field and to trace the
ity 2.2
SPM
’08/P1
pattern of electric field lines

Apparatus/Materials 2 A glass dish is filled with olive oil until the pair
Glass dish, metal electrodes, high voltage supply, of electrodes is completely below the oil surface.
olive oil and semolina powder or grass seed. 3 Semolina powder is dispersed evenly on the
surface of the oil.
Arrangement of apparatus
4 The high voltage supply is switched on.
high voltage supply
5 The voltage is increased until a pattern of the
electric field becomes visible.
6 Other patterns of the electric field are observed
using electrodes of different shapes such as
spheres, planes or a combination of shapes.
metal glass
electrode dish Results
olive oil 1 When the high voltage supply is switched on,
the semolina powder on the oil surface becomes
semolina powder
aligned in the direction of the electric field and
Figure 2.5 Investigation of the pattern of an electric field thus maps out the electric field lines.
Procedure 2 Table 2.1 shows some patterns of the electric
F
O 1 The apparatus is set up as shown in Figure 2.5. field using different types of electrodes.
2

R Table 2.1
CHAPTER

M
(a) Single positively (b) Single negatively (c) A positively charged (d) Two positively
5 charged sphere charged sphere sphere and a negatively charged spheres
charged sphere

neutral point

Arrows point away Arrows point towards The two opposite charges The two similar charges
from the positive charge. the negative charge. attract each other. repel each other.
(e) Two negatively (f) A positively (g) A positively (h) A negatively
charged spheres charged plate and charged sphere charged sphere
a negatively and a negatively and a positively
charged plate charged plate charged plate

neutral point

The two similar charges A uniform electric A non-uniform electric A non-uniform electric
repel each other. field except at the sides. field. field.
Disscussion Conclusion
1 Olive oil is used because it is an insulator which An electric field exists in the space surrounding a
does not conduct electricity. This enables a very charged body. The pattern of the electric lines of
Activity 2.2

high voltage to be maintained across the two force depends on the shape and the number of
electrodes. charged objects.
2 The semolina powder aligns itself along the
electric lines of force due to the action of the
electric field.

Electricity 372
Effect of an Electric Field on a Charge

Properties of electric lines of force: Ping-Pong Ball Coated with a Conducting


1 The direction of the electric field lines is from the Material
positive charge towards the negative charge.
2 The electric field lines do not cross each other.
nylon thread
3 The closer the lines, the stronger the electric field is.
positive terminal negative terminal (earthed)
At P, field
lines are
insulator rod
closer
P (stronger
A B electric
field) ping-pong
ball
Q At Q, field metal plate
lines are far
apart (weaker
electric field) Figure 2.6 Effects of an electric field

A and B are spheres with positive charges. 1 A ping-pong ball which is coated with a
From the figure, please note that: metallic paint is hung between two metal
(a) Charge on B is greater (more electric field lines plates by a nylon thread (an insulator). The
are drawn). positive and negative terminals of a high F
(b) Electric field is stronger at P than at Q, i.e. the voltage supply are connected to each of the O

2
nearer the point to the source, the stronger the two plates respectively. R

CHAPTER
electric field. M
2 Table 2.2 shows the effects observed.
5
Table 2.2

Step Observation Explanation


High voltage Ping-pong ball remains stationary. • The forces of attraction by the positive plate
supply is nylon thread on the ping-pong ball is equal to that of the
switched on. negative plate, i.e. F1 = F2.
• The ping-pong ball is electrically neutral.

high voltage supply

The The ping-pong ball oscillates between • When the ping-pong ball touches the
ping-pong the two plates. negative plate, it receives electrons and
ball is becomes negatively charged. The
displaced and negatively charged ping-pong ball is
touches the repelled by the negative plate and
negative plate. attracted by the positive plate.
• When the ping-pong ball touches the
high voltage supply high v positive plate, its negative charge is
(i) (ii) neutralised and it is immediately charged
positive. The positively charged ping-pong
ball is repelled by the positive plate and
attracted by the negative plate.
• The ping-pong ball moves towards and
touches the negative plate and the cycle is
h high voltage supply
repeated. The ping-pong ball oscillates
(ii) until the voltage supply is turned off.

373 Electricity
Candle Flame Explanation
(a) The heat of the burning gases in the flame
1 A lighted candle is placed between two metal produces positive and negative ions.
plates which are connected to the terminals of (b) The positive ions are pulled towards the
a high voltage supply. negative plate while the negative ions are
2 The voltage supply is switched on and the pulled towards the positive plate.
movement of the candle flame is observed. (c) The heavier positive ions occupy a larger
3 The candle flame flattens and spreads out in proportion of the flame.
both directions. (d) The negative ions (electrons), which are
++
lighter, occupy a smaller proportion of the
--- ++
-- + flame.
candle flame (e) Hence, the flame is seen to spread out
P Q more to the negative plate than to the
++ – – positive plate.
++ – –
+ –
high voltage supply

Figure 2.7

F
O
1 SPM
2

Clone
R ’11
CHAPTER

M
Diagram shows the shape of a candle flame of a P Q
5 candle placed between two plates P and Q which are A Negative Negative
connected to an EHT supply. B Negative Positive
P candle flame Q C Positive Negative
D Positive Positive
Comment
The heavier positive ions in the flame occupy a larger
candle portion of the flame while the negative ion (electrons),
which are lighter, occupy a smaller portion of the
flame. Since the left portion of the flame is of greater
EHT portion and attracted to the left, plate P is negatively
charged, whereas plate Q is positively charged.
What are the charges on plate P and Q?
Answer B

2 SPM
Clone
’11

The charge flowing in a bulb in a circuit in one Comment


Q
minute is 6.0 C. Apply the formula I = to solve the problem
What is the current in the circuit? t
A 0.10 A Solution
B 0.17 A Q 6
C 5.0 A I= = = 0.10 A
t 60
D 12.0 A
Answer A

Electricity

374
2.1

1 Electric charges flow through a light bulb at the rate of


20 C every 50 seconds. What is the electric current
shown on the ammeter?
2 A milliammeter shows a reading of 200 mA. What is
the rate of flow of charge per minute? What is the
number of electrons flowing in 1 minute?
(The charge of 1 electron = –1.6 × 10–19 C)
3 5 × 1018 electrons flow through a light bulb in
20 seconds. What is the electric current flowing Q coulombs of charge flows through the light bulb X
through the bulb? in t seconds and 2Q coulombs of charge takes
(The charge of 1 electron = –1.6 × 10–19 C) 3t seconds to flow through the light bulb Y.
4 The current flowing through a resistor is 0.3 A. What is What is the ratio of the current flowing through X to
the time required for 60 C of charge to flow through the current flowing through Y ?
the resistor?
6 An electric current of 3.5 × 10–5 A is supplied by a
5 The diagram below shows two light bulbs X and Y.
SPM digital watch. What is the quantity of charge that flows
Clone
’05 in 6 hours?

F
O

2
R
2.2 Analysing the Relationship between

CHAPTER
M
Electric Current and Potential Difference 5

Ideas about Electric Potential Difference

(a) cylinder
(b) ball (c)
X

water
Y
valve

P Q

Figure 2.8 earth Figure 2.10


Pressure at point P > Figure 2.9 Electric potential at X
pressure at point Q Gravitational potential energy (positive terminal) >
⇒ Water will flow from P to Q of ball at X > gravitational electric potential at Y
when the valve is opened potential energy of ball at Y (negative terminal)
because of the difference in ⇒ The ball will fall from X to Y ⇒ Electric current flows from X
the pressure of water. because of the difference in to Y, passing through an
the gravitational potential electric bulb, because of the
energy. electric potential difference
across the battery.
Hence
(a) water (consists of water molecules) flows because of pressure difference,
(b) ball falls because of gravitational potential energy difference,
(c) current (consists of charges) flows because of electric potential difference, p.d.
Potential difference drives a current through a conductor.

375 Electricity
An analogy is often made between the flow of an electric electric current because of the potential difference across
current in a wire and the flow of water through a pipe. the battery.
Such an analogy is useful in explaining concepts such Hence:
as potential difference and the factors influencing 1 Pressure difference Electric potential
resistance. ≡
in water difference
In Figure 2.8, water flows in the pipe because of the 2 Flow of water ≡ Flow of electric charge
difference in the pressure of the water. In Figure 2.10, 3 Rate of flow of water ≡ Rate of flow of charge
electric charges flow in the circuit thereby producing an (i.e. current)

Meaning of Potential Difference Potential difference = Work done


Charge
1 Figure 2.11 shows an electric field surrounding
P with a positive charge. W
V =
F 2 An electric field is a region within which V = Q
O
electric force acts on a particle with an electric 9 The potential difference between two points is
2

R
charge. 1 volt if 1 joule of work is required to move a
CHAPTER

M
charge of 1 coulomb from one point to the
5 electric field line other.
1 joule
high electric low electric 1 volt = or 1 V = 1 J C–1
potential potential 1 coulomb
10 The symbol for potential difference is V (italic
letter). The unit of potential difference is the
potential difference
volt, V (Roman letter). The term ‘voltage’ is
often used to mean potential difference.
Figure 2.11 Concept of potential difference

3 Points within the region surrounding P possess Ammeter and Voltmeter in a Circuit SPM
’05/P2(B)

electric potential whose magnitude depends SPM


on their distance from P. 1 Figure 2.12 shows an ammeter and ’06/P2(A)

4 The electric potential at point A is higher voltmeter used in measuring current


than that at point B because point A is nearer and potential difference.
to P.
5 If a positive charge is placed at point A, it will
be repelled and move in the direction of the
electric force passing point B.
6 The positive charge moves because of the
electric potential difference between points A
and B.
7 Conversely, if the positive charge is moved
from point B to point A, work is done to
overcome the opposing force between the
positive charge and P.
(a) Ammeter (b) Voltmeter
8 The potential difference, V between points A
Figure 2.12
and B can be defined as the work done in
moving a unit charge from B to A in an 2 Table 2.3 shows the differences between an
electric field. ammeter and a voltmeter.

Electricity 376
Table 2.3

Ammeter Voltmeter
• Measures current in amperes. • Measures potential difference or voltage in volts.
• Connected in series with a resistor or a device. • Connected in parallel with or across a resistor,
a device or a cell/battery.
resistor
resistor
A

• Has a low resistance so that its existence has V


little effect on the magnitude of current flowing.
• Has a high resistance, current flowing through
it is negligible.

3
A battery of 3 V drives 3V Solution
12 C of electric charges W
From V = ,
through a resistor, R. electrons Q
flow
What is the work done by R
F
W = QV
the battery? O
I = 12 × 3

2
R
Figure 2.13 = 36 J

CHAPTER
M

Relationship between Electric Current and Potential Difference

Pressure of water,
P = hρg
water where h is the depth of water water
20 cm Pressure difference, ∆P 15 cm
6 cm = ∆h × ρg 13 cm

(a) Faster flow of water because of greater (b) Slower flow of water because of smaller
pressure difference pressure difference
Figure 2.14

1 The larger difference in the pressure of water causes a higher rate of water flow in Figure 2.14(a).
What is the relationship between the electric current flow in a conductor and the electric
potential difference across the conductor?
2 In 1826, Georg Simon Ohm, a German physics teacher conducted an experiment to investigate
the relationship between the electric current flowing through a conductor and the potential
difference across the conductor. From the results of the experiment, a relationship named
Ohm’s law was established in honour of him.

2 The relationship can also be represented by a


Ohm’s Law SPM SPM SPM SPM
’03/P1 ’05/P1 ’06/P1 ’09/P1 graph as shown in Figure 2.15.
1 Ohm’s law states that the electric current, I
flowing through a conductor is directly
proportional to the potential difference across
the ends of the conductor, if temperature and
other physical conditions remain constant. O
That is, V ∝ I. Figure 2.15

377 Electricity
3 Conductors with V-I graphs which are linear 3 The resistance, R of a conductor is defined as
and pass through the origin obey Ohm’s law the ratio of the potential difference across the
and are said to be ohmic conductors. Other conductor to the current, I flowing through
materials which do not obey Ohm’s law are the conductor. That is,
called non-ohmic conductors.
V
R=
I
Understanding Resistance
4 The unit of resistance is volt per ampere
1 By Ohm’s law: V ∝ I (V A–1) or ohm (Ω).
∴ V = constant × I 5 A resistor is represented by the symbol:
or
or V = constant
I
2 The constant is known as resistance, R of the Figure 2.16 Symbol for a resistor
conductor.

2.1
SPM
’06/P3(A)
The relationship between electric current and potential difference
F
O Hypothesis Procedure
2

R The higher the current flowing through an ohmic 1 The circuit is connected as shown in Figure 2.18.
CHAPTER

M
conductor, the higher the potential difference across 2 The switch is closed and the rheostat is adjusted
5 its ends. until the current reading on the ammeter is 0.1 A.
The reading of the potential difference, V on the
Aim voltmeter is recorded.
To investigate the relationship between current and 3 The rheostat is adjusted to obtain four more sets
potential difference for an ohmic conductor of readings of current, I and potential difference, V.
4 The values of V and I are tabulated.
Variables 5 A graph of potential difference, V against current,
(a) Manipulated : current, I I is plotted.
(b) Responding : potential difference, V
Tabulation of data
(c) Fixed : length of constantan wire and temperature
Table 2.4
Apparatus/Materials
Current, I (A) Potential difference, V (V)
Switch, voltmeter (0 – 3 V), ammeter (0 – 3 A),
0.1 V1
rheostat (0 – 15 Ω), 10 cm constantan wire
(s.w.g. 34), crocodile clips, connecting wire, dry cells 0.2 V2
(2 × 1.5 V) and battery holder. 0.3 V3
Arrangement of apparatus 0.4 V4
3.0 V switch
0.5 V5

Graph

rheostat
constantan wire
Experiment 2.1

crocodile
clip

Figure 2.18
Figure 2.19 Relationship between V and I

Electricity 378
Discussion 2 Precautions to be taken in the experiment:
(a) Avoid parallax errors in the ammeter and
1 The resistance of a conductor can be determined:
voltmeter readings by reading the position of
(a) by calculating the average value from the ratios the pointer on the scale perpendicularly.
V (b) The zero errors of ammeter and voltmeter
I for each of the 5 sets of values of V and I. must be checked and corrected if necessary
or before starting the experiment.
(b) by finding the gradient of the graph. (c) Physical conditions and temperature must be
kept constant.
3 (a) To ensure that the temperature remains
constant throughout the experiment,
(i) use low values of current,
(ii) turn off the switch when not taking any
readings to avoid heating of the wires.
(b) Since the same wire is used for all the sets of
readings of V and I, the same physical
conditions are maintained.
Figure 2.20
V Conclusion
From the formula R = :
I F
The V – I graph obtained is a straight line passing
⇒ V = RI through the origin. Hence, the potential difference, V
O

2
Compare with the linear equation, y = mx R
across the conductor is directly proportional to the

CHAPTER
M
(where m is the gradient of the straight line current flow, I provided the physical conditions and
graph). temperature do not vary. 5
∆V
R = m (= gradient of graph V – I) = The hypothesis is valid.
∆I

V (V) SPM
’03/P1
switch
SPM
3 ’09/P2/(C)
x (0.30, 2.50)

ammeter 2
rheostat
light bulb
1 x (0.20, 1.10)
x (0.12, 0.50)
voltmeter
I (A)
0.1 0.2 0.3

The table below shows a typical result obtained if the 0.3 A), and Experiment 2.1 is repeated. The brightness
constantan wire in Figure 2.18 is replaced by a bulb (2.5 V, of the bulb is recorded and the V – I graph is plotted.

Current flow in Potential difference across Resistance of filament,


Condition of light bulb light bulb, I (A) the light bulb, V (V) V
R = (Ω)
I
(a) Not lit (low temperature) 0.12 0.50 4.17
(b) Dimly lit (medium temperature) 0.20 1.10 5.50
Experiment 2.1

(c) Brightly lit (high temperature) 0.30 2.50 8.33

1 The graph of V against I is not a straight line but a non-ohmic conductor and it does not obey Ohm’s
curve passing through the origin. This shows that the law.
potential difference across the conductor is not 2 The value of resistance of a conductor depends on its
proportional to the current flowing. A light bulb is a temperature.

379 Electricity
Meaning of ‘Resistance of 1 ’ R=1Ω
I=1A

Referring to Figure 2.17, the resistance of a


conductor is 1 Ω if, it allows a current of 1 A to
V=1V
flow, when a potential difference of 1 V exists
across its ends. Figure 2.17

That is, R= 1 V =1 Ω
1A

5
Formulae for calculating V, I, and R: Figure 2.21 shows a torchlight that uses two 1.5 V
Any of the following formulae can be used. dry cells and its circuit diagram. The two dry cells are
• V = IR able to provide a current of 0.3 A when the bulb is at
V its normal brightness.
• R =
I

• I =
V
R
× + 1.5 V - + 1.5 V -

This diagram can be used to (a)


F determine which formula is
O applicable. Cover I if you wish to
2

R find I and so on.


CHAPTER

5
(b)
4
Figure 2.21
What is the current flowing through an immersion
What is the resistance of the filament?
heater of resistance 24 Ω when it is connected to a
240 V supply? Solution
Solution The resistance of the filament at normal brightness is
given by:
V 240
I = = = 10 A V 3.0 V
R 24
0.3 A = 10 
I R = =
I

V ohmic conductor. Its value, which is constant, is represented by the gradient of the
R=
I V – I graph.
is true for non-ohmic conductor. Its value varies according to the instantaneous values of V and I.

Factors that Affect the Resistance of a Conductor SPM


’07/P1
SPM
’09/P1
SPM
’09/P2/(C)

1 The resistance of a conductor is a measure of 3 When the value of the resistance, R is large, the
the ability of the conductor to resist the flow current, I flowing in the conductor is small,
of an electric current through it. and vice versa.
4 Factors influencing the resistance of a
2 From the formula I = V , it can be seen that
R conductor are:
when V is held constant, the current I is inversely (a) the length of the conductor,
proportional to the resistance, R. That is, (b) the cross-sectional area of the conductor,
(c) the type of material of the conductor,
I ∝ 1 (d) the temperature of the conductor.
R

Electricity 380
 Length of Conductor

2.2
SPM
’07/P3(A)
Factors affecting resistance
Hypothesis V
6 The resistance, R = is calculated for all the
I
The longer the length of the wire (conductor), the
greater its resistance. five lengths of wire used.
7 A graph of R against  is plotted with both axes
Aim starting at the origin.
To investigate the relationship between the length and
Tabulation of data
the resistance of a conductor (constantan wire)
Table 2.5
Variables
Length Potential Resistance,
(a) Manipulated : length of wire,  Current, V
of wire, difference, R = (Ω)
(b) Responding : resistance, R I (A) I
 (cm) V (V)
(c) Fixed : material of wire, cross-sectional
area and temperature 20.0 0.5 V1 R1
40.0 0.5 V2 R2 F
Apparatus/Materials O
60.0 0.5 V3 R3

2
R
Voltmeter, ammeter, rheostat, switch, constantan wire

CHAPTER
80.0 0.5 V4 R4 M
(s.w.g. 28) of length 110 cm, connecting wires,
crocodile clips, dry cells and battery holder. 100.0 0.5 V5 R5 5

Arrangement of apparatus
Graph
3.0 V switch
R (Ω)

rheostat crocodile
constantan clip
wire
O  (cm)
P Q
Figure 2.23 Relationship between R and 

Discussion
Figure 2.22
1 It is found that the resistance, R increases with the
length of the wire, .
Procedure
2 To ensure that the temperature remains constant
1 The circuit is connected as shown in Figure 2.22. throughout the experiment:
2 The length of the constantan wire between P and (a) use the same current for all the five lengths of
Q is adjusted so that its length,  = 20 cm. wire,
3 The switch is closed and the rheostat is adjusted (b) turn off the switch when not taking any
until the current, I, flowing in the circuit is 0.5 A. readings.
4 The potential difference across the wire is
recorded. Conclusion
Experiment 2.2

5 Steps 2 to 4 are repeated using wire lengths of The R –  graph is a straight line passing through the
40 cm, 60 cm, 80 cm and 100 cm. For each length origin. Hence, the resistance, R is directly
of wire used, the rheostat is adjusted so that the proportional to the length of the wire, .
current is at a constant value of 0.5 A. The reading
∴ R∝
of the corresponding value of the potential
difference, V, is recorded in a table. The hypothesis is valid.

381 Electricity
 Cross-sectional Area of Wire

2.3
SPM
’05/P3(A)
Factors affecting resistance
Hypothesis
The greater the cross-sectional area of a wire, the
(b)
smaller its resistance.
Aim
To investigate the relationship between the cross-
sectional area and the resistance of a conductor (c)
(constantan wire) Figure 2.24
Variables
(a) Manipulated : number of wires (represents the cross-
sectional area of the conductor) Procedure
(b) Responding : resistance, R 1 The circuit is connected as shown in Figure
(c) Fixed : material of wire, temperature, 2.24(a).
F length and cross-sectional area of 2 A 50 cm constantan wire (s.w.g. 28) is connected
O
each wire to the circuit using crocodile clips.
2

R
Apparatus/Materials 3 The switch is closed and the rheostat is adjusted
CHAPTER

M
until the current, I is 0.5 A. The voltmeter reading,
5
Voltmeter, ammeter, rheostat, switch, 5 × 50 cm
V is recorded.
constantan wires (s.w.g. 28), connecting wires,
4 Steps 2 and 3 are repeated using two, three, four and
crocodile clips, dry cells (2 × 1.5 V) and battery holder.
five 50 cm constantan wires connected in parallel
Arrangement of apparatus respectively as shown in Figures 2.24 (b) and (c).
3.0 V switch 5 The rheostat is adjusted for a current flow of
0.5 A in each case and the corresponding
voltmeter reading, V is recorded.
6 Graphs of resistance, R against cross-sectional
rheostat
1
1 constantan wire area, A and R against are plotted with both axes
P Q
A
crocodile
clip starting at the origin.

(a)

Tabulation of data

Table 2.6

Cross-sectional Inverse of Resistance,


area, A cross-sectional Potential difference, V
Current, I (A) R = (Ω)
(Number of wires) 1 V (V) I
area,
A
Experiment 2.3

1 0.5 1.00 V1 R1
2 0.5 0.50 V2 R2
3 0.5 0.33 V3 R3
4 0.5 0.25 V4 R4
5 0.5 0.20 V5 R5

Electricity 382
Graph The wires considered are of the same material and
the same length.
R (Ω) R (Ω)

fine wire
(s.w.g. 32)

medium wire
(s.w.g. 26)

1 thick wire
O O A (s.w.g. 22)

(a) (b)
Figure 2.25 O

Discussion Figure 2.26 Relationship between V and I for


1 The use of a thicker wire increases the rate of different cross-sectional areas
flow of electric charge resulting in a larger current
flow. Hence, the resistance of a wire decreases
when its cross-sectional area is increased. Conclusion
2 We can also use the highway as an analogy where
1
traffic flow is smoother on wider roadways. The graph of R against is a straight line passing F
A O
3 Figure 2.26 shows a comparison of the values of

2
R
R (represented by the gradient of the graphs) of through the origin. This shows that the resistance, R

CHAPTER
M
different thicknesses of wire (a higher s.w.g. is inversely proportional to the cross-sectional area of
number of a wire denotes a thinner wire). It is a conductor. 5
found that a thicker wire has a lower resistance. The hypothesis is valid.

The number of wires represents the total cross-sectional A constantan wire has a resistance of R Ω. What is the
area of the conductor. resistance, in terms of R, of an equal length of
Let p = the cross-sectional area of a wire constantan wire with twice the diameter?
With 2 wires arranged in parallel, the total area for
current flow, A = 2 × p.
With 3 wires in parallel, A = 3 × p. Solution
Therefore, n wires give an area, A = n × p (i.e. A ∝ n) Resistance is inversely proportional to the cross-
The number of wires in fact represents the cross- sectional area, A, not the diameter, d.
sectional area for current flow. 1
Cross-sectional area, A = πr 2 = πd 2
4
1 1 1
Since R ∝ , then R ∝ 2 . r= d
A d 2

For all wires made of same material (e.g. constantan): That is, resistance is inversely proportional to the square
of the diameter of wire.
Experiment 2.3

Short wire: R ↓ Thin wire: R ↑ Answer


When the diameter is twice the original diameter, the
1
resistance becomes R Ω.
Long wire: R ↑ Thick wire: R ↓ 4

383 Electricity
 Type of Material

2.4
Factors affecting resistance
Hypothesis Procedure
For a fixed length and thickness of a wire used, its 1 The circuit is connected as shown in Figure 2.27.
resistance is affected by the type of material. 2 The constantan wire (s.w.g. 28) of length 50 cm is
connected to the circuit. The voltmeter reading is
Aim
recorded when the current is 0.5 A by adjustment
To show that the resistance of a wire depends on the of the rheostat.
type of material of the wire. 3 Step 2 is repeated using the nichrome wire in
Variables place of the constantan wire.
4 The readings are tabulated.
(a) Manipulated : type of wire material
5 The resistances of both wires are calculated for
(b) Responding : resistance, R
comparison.
(c) Fixed : temperature, length and cross-
sectional area (thickness) of wire
Tabulation of data
F Apparatus/Materials
O
Voltmeter, ammeter, rheostat, switch, constantan wire
2

R
(s.w.g. 28) of length 50 cm, nichrome wire (s.w.g. 28) Table 2.7
CHAPTER

M
of length 50 cm, connecting wires, crocodile clips, Type of Current, Potential Resistance,
5 dry cells (2 × 1.5 V) and battery holder. V
wire I (A) difference, R = (Ω)
V (V) I
Arrangement of apparatus
switch
Constantan 0.5 V1 R1
Nichrome 0.5 V2 R2

F Note: R2 > R1
O
2

R
CHAPTER

rheostat M Conclusion
constantan or
5 wire
nichrome The results show that the resistance of nichrome,
R2 > R1. Thus, the resistance of a wire depends on the
type of its material as the resistances of different
materials vary.
The hypothesis is valid.
Figure 2.27

Experiment 2.4 shows that when wires of identical


Experiment 2.4

dimensions are used, the resistance, R of each wire is


determined by the type of material of the wire.
The following graph shows the comparison of the
copper silver nichrome constantan values of R (equal to the gradients of the graphs) for all
the wires.

* 4 wires of identical dimensions

Electricity 384
V It can be seen that the resistance of silver is the lowest
nichrome
and thus silver has the best conductivity followed by
constantan
copper. However, most wires are made of copper
copper because its price is much lower and it is in abundant
supply compared to silver. Constantan and nichrome are
two alloys with high resistances.
silver

I
O

 Temperature

2.5
Factors affecting resistance
Hypothesis Procedure
The higher the temperature, the greater the resistance 1 The length of fine iron wire is wound around a
of a metal conductor. U-shaped glass rod to form a coiled resistor. F
O
The coil is secured to the glass rod with glue.

2
R
2 The coiled resistor is connected to the circuit as

CHAPTER
M
Aim shown in Figure 2.28.
To investigate the relationship between the temperature 3 The coiled resistor is fully immersed in a beaker 5
and the resistance of a metal conductor of water.
4 The current is maintained at a value of 0.5 A
using a rheostat.
Variables
5 The water is heated to a temperature of 30 °C and
(a) Manipulated : temperature, θ the reading of the potential difference, V across
(b) Responding : resistance, R the coiled resistor is recorded.
(c) Fixed : type of material, length and cross- 6 Steps 4 and 5 are repeated with the temperature
sectional area of wire raised to 40 °C, 50 °C, 60 °C, 70 °C and 80 °C.
V
7 The corresponding value of the resistance, R =
I
Apparatus/Materials
is calculated for each of the above temperatures.
Iron wire (s.w.g. 34) of length 2 m, U-shaped glass 8 A graph of the resistance of the coiled resistor, R
rod, beaker, thermometer, stirrer, electrical power against temperature, θ is plotted.
supply, connecting wires, crocodile clips, Bunsen
Tabulation of data
burner, water, wire gauze and tripod stand.
Table 2.8
Arrangement of apparatus
Potential Resistance,
Temperature, Current,
difference, R = V (Ω)
θ (°C) I (A)
thermometer V (V) I
rheostat
Experiment 2.4 & 2.5

ammeter
30 0.5 V1 R1
coiled
resistor 12 V
40 0.5 V2 R2
water
wire stirrer 50 0.5 V3 R3
gauze
60 0.5 V4 R4
tripod
stand 70 0.5 V5 R5
Bunsen burner
80 0.5 V6 R6
Figure 2.28

385 Electricity
Graph electrons in motion are hindered by the vibrating
atoms and thus the electrons flow at a slower rate.
R (Ω) As a result, the resistance of the metal increases
with an increase in temperature.
2 Precautionary steps:
(a) The water is stirred continuously throughout
the experiment to ensure a uniform
temperature of the water.
(b) The rheostat is adjusted regularly to maintain
O θ (°C) the current, I, at the value of 0.5 A.
Figure 2.29
Conclusion

Discussion The graph of R against θ in Figure 2.29 is a straight


1 When the temperature is raised, the atoms in the line with a positive gradient. This shows that the
metallic lattice become more energetic and resistance increases when temperature increases.
vibrate with greater amplitude. The path of the The hypothesis is valid.

F
O
2

R
CHAPTER

(a) Substances whose resistance increases with 2 Their conductivities increase with temperature —
temperature i.e. resistance decreases with an increase in
temperature. The relationship between R and θ for a
Alloys such as semiconductor is shown below:
Pure metal constantan and R
F
O
nichrome
2

R (Ω) RR (Ω)
CHAPTER

M
Physics Blog semiconductor
5

O θ

θ (°C) θ (°C) 3 The resistance of carbon also decreases when its


F5/2/51
O O
temperature is increased. Carbon is thus used as
• Increases linearly • Increases only very resistors in radios.
with temperature slightly with
temperature (c) Thermistor
(gradient of graph 1 A thermistor is a type of substance which shows a
is very small) sharp change in resistance when its temperature
rises.
The resistances of alloys such as constantan and 2 The resistance of a thermistor increases or
nichrome increase only very slightly when decreases depending on its composition.
temperature increases. These alloys are thus
Experiment 2.5

suitable for use as standard resistors. Composition Increasing temperature


Metal oxides, nickel
(b) Substances whose resistance decreases with Resistance decreases
and cobalt
temperature increase Barium titanate Resistance increases
1 Semiconductors such as silicon and germanium are
not good conductors at low temperatures. 3 Thermistors are used in temperature control switches.

Electricity 386
Formulae from the Results of Experiments
7
1 From Experiments 2.2, 2.3 and 2.4, the Figure 2.31 shows a wire P of length,  with a cross-
following can be stated: sectional area, A and a resistance, R. Another wire, Q
Resistance of a conductor, R ∝  is a conductor of the same material with a length of
3 and twice the cross-sectional area of P.
Resistance of a conductor, R ∝ 1
A

∴ R ∝ and
A
 A 2A
R=ρ
A
where ρ is the resistivity of a substance.
Figure 2.31
The value of resistivity is different for each
substance. What is the resistance of Q, in terms of R?
2 Silver is a very good conductor with a very low Solution
resistivity, ρ.
Conductor P: R = ρ
A
F
= ρ 3
ʹ′
Conductor Q: Rʹ′ = ρ O
Aʹ′

2
2A R
6

CHAPTER
M
3 
= ρ
In Figure 2.30, PQ is a piece of uniform wire of 2 A 5
length 1 m with a resistance of 10 Ω. Q is connected 3
to an ammeter, a 2 Ω resistor and a 3 V battery. = R
2

Why is aluminium used as the material for power cables


jockey
although aluminium has a higher resistivity than copper?
power
cable
Figure 2.30

What is the reading on the ammeter when the sliding


contact (jockey) is at X?
Solution
1
Length of XQ,  = PQ
5
pylon
Since R ∝ ,
1
∴ Resistance of XQ = × 10 Ω Answer
5
• The ratio of the resistance of aluminium to copper is
=2Ω approximately 1.6.
V
Current, I = Total resistance • The ratio of the density of copper to aluminium = 3.3
R = resistance of XQ
• Aluminium has an advantage from the point of view
+ resistance of
3 of density. Its lower conductivity compared to copper
= resistor
4 =2Ω+2Ω is offset by its lower density.
=4Ω • If copper is used, more pylons need to be built to
= 0.75 A
support the heavy power cables.

387 Electricity
Use of a rheostat 2 Current flows through a coil in the rheostat from
slider
terminal A to B as shown in the diagram.
current current out
brass 3 When the position of the slider is changed, the length
rod of wire through which the current flows changes. This
is equivalent to a change in the resistance of the
current in
or rheostat and the total resistance of the whole circuit.
As a result, the current flowing in the circuit changes.
Symbol
4 The rheostat can also function by connecting
Physics Blog
coil of resistance wire terminals B and C in the circuit.

1 A rheostat is a common electrical device used in the Reminder:


F5/2/56 purposes:
laboratory for the following The connecting wires must be connected to either A and
(a) To control or regulate the current flowing in a B or B and C. Do not connect the wires to A and C
circuit by changing the total resistance in the because the current will flow through the whole coil in the
circuit. (See Experiment 2.1) rheostat and the resistance will not be varied although the
(b) To maintain a constant current. slider is adjusted.
(See Experiment 2.2)

F
O Superconductors
2

R
CHAPTER

M
1 The resistance of a metal conductor decreases absolute zero temperature. These metals include
5 when its temperature decreases. the best conductors like silver, copper and gold.
2 For certain materials like aluminium, the
Table 2.9
resistance decreases with temperature but the
resistance suddenly becomes zero when it is Element Critical temperature, Tc (K)
cooled below a certain temperature called the Zinc, Zn 0.88
critical temperature, Tc, as shown in Figure
Aluminium, Al 1.14
2.32. The critical temperature
F is normally a few
O
degrees above the absolute zero. Materials Tin, Sn 3.69
2

R
showing this characteristics are known as Mercury, Hg 4.15
CHAPTER

M
superconductors. Lead, Pb 7.26
5
Niobium, Nb 9.2

Applications of Superconductors

1 Construction of superconducting magnets


which can produce magnetic field strengths
O TC O more than ten times that of the best normal
electromagnets. These superconducting magnets
(a) Superconductor (b) Normal conductor
will be useful for magnetically levitated
Figure 2.32
vehicles to be developed in the future.
3 Table 2.9 shows some elements with their 2 Magnetic Resonance Imaging (MRI) also uses
critical temperatures. superconducting magnets. MRI is used as a
4 These materials offer no resistance to the flow diagnostic tool in medicine.
of current and act as perfect conductors with 3 Superconductors are used to produce computer
zero resistivity. Once the current is set up in chips which are faster and of smaller size.
these materials, it needs no applied voltage to 4 Cable or wires made of superconductors will
continue flowing without any loss. increase the efficiency of electrical power
5 Most metals do not show superconductivity. transmission as the loss of energy in the form
Their resistance does not go to zero even at of heat is greatly reduced.

Electricity 388
3 SPM
Clone
’10

An experiment is carried out and a voltage-current C D


V
graph is obtained as shown in the diagram.
voitage
A V
A

Comments
current Circuit B is used. When the resistance in the resistor
varies, the voltage across the ressistor and the
Which of the following circuit is used in the current in the circuit vary accordingly. When the
experiment? values of V are plotted against the corresponding
A B currents, a straight line through the origin is
obtained.
A A
Answer B

V
F
O

2
V R

CHAPTER
M

4 SPM
Clone
’07

Which of the following factors does not affect the Comments


resistance of a wire? The hardness of a wire does not influence the
A Material of the wire resistance of a wire. The resistance of a wire depends
B Hardness of the wire on the length, the cross-sectional area and the material
C Length of the wire of the wire.
D Cross-sectional area of the wire
Answer B

5 SPM
Clone
’09

The figure shows an electrical circuit. What would you change to the wire so as to produce the
greatest reading on the ammeter?
Length of wire Diameter of wire
A Longer Bigger
A
wire B Longer Smaller
C Shorter Smaller
D Shorter Bigger

Comments
To increase the current flow, we need to decrease the resistance of the wire.
By shortening and increasing the thickness of the wire, the resistance is decreased.
Answer D

389 Electricity
2.2

1 (a) In the diagram, P, Q and R represent three wires 4 A bread toaster with a resistance of 60 Ω is used on a
with the same length and thickness. P is a copper 240 V supply. What is the current flowing through the
wire, Q a steel wire and R a constantan wire. toaster?

5 The diagram shows a current circuit comprising two


resistors of R Ω and 1.0 Ω respectively. The reading
on the voltmeter V1 is 2 V and the ammeter reading
is 6 A.

Which switch must be closed so that the reading R


on the ammeter is the smallest?
(b) You are given a metal block with dimensions as V1 V2
shown in diagram (i).
Calculate the value of R and the reading of voltmeter
F V2. What is the voltage supplied by the cell?
O
2

R 6 When the potential difference across a resistor is 6 V,


CHAPTER

M the current flow is 1.5 A. What is the potential


difference across the resistor if the current flow is
5 2.0 A?

(i)
7 The graph below shows the relationship between the
potential difference and current flowing through a
Which pair of faces of the block must be copper voltameter with platinum electrodes.
connected to the circuit in diagram (ii) to
produce the largest ammeter reading?
V1

A
A B

O I
I1
(ii)

2 The diagram shows a circuit with three similar light Is a copper voltameter an ohmic conductor?
SPMbulbs. The resistance of the copper voltameter is given by
Clone
’06 the gradient of the V—I graph. Is this true or false?
A Explain your answer.

8 The diagram shows two metal rods P and Q made of


the same substance. The diameter of P is twice that
of Q.

What would happen to the reading on the ammeter


and the brightness of the bulbs if another bulb is
added to the circuit?

3 A light bulb is labelled 3.0 V, 0.35 A. What is the


resistance of the filament in the bulb when the bulb What is the length, ’, of rod Q in terms of , the
is in its normal ‘on’ state? length of rod P, if P and Q have the same resistance?

Electricity 390
2.3 Analysing Series and Parallel Circuits
SPM
Series and Parallel Circuits ’08/P2

1 An electrical circuit is a complete path along which electric current can flow.
2 A circuit is made of electrical components like cells, resistors, lamps, switches and so on.
3 Series and parallel electrical circuits are two basic ways of connecting the electrical components.

Series Circuit Parallel Circuit

1 A circuit is a series circuit if the circuit 1 A circuit is a parallel circuit if the components
components are connected end to end are placed side by side and their
consecutively so as to provide a single path corresponding ends are joined together.
for the current to flow through all the 2 The current from the source splits at one joint
components. into the components and rejoins at the other
2 All components in a series circuit have the joint. In other words, the components share
same current. the current from the source.

Activ To identify series and parallel circuits


ity 2.3
SPM SPM SPM F
’04/P1 ’08/P1 ’09/P1
O

2
R

CHAPTER
Apparatus/Materials M

Dry cells (3 × 1.5 V), battery holders, 3 × 5 V light bulbs and connecting wires. 5
Arrangement of apparatus

Components set-up Circuit diagram


dry cells

a x y b

The bulbs are connected one after the other. Currents at points a, x, y and b are equal.
(a)
Components set-up Circuit diagram
dry cell

a b
a b
Activity 2.3

The bulbs are connected side by side. Current splits at joint a and rejoins at joint b.
(b)
Figure 2.33

391 Electricity
Procedure For circuit (b)
For circuit (a) 1 The circuit in Figure 2.33(b) is connected.
1 The circuit in Figure 2.33(a) is connected. 2 One of the bulbs is unscrewed. The remaining
2 The brightness of the bulbs are observed. bulbs are observed.
3 One of the bulbs is unscrewed. The remaining
bulbs are observed. Observation and Discussion
Observation and Discussion 1 The remaining bulbs do not go off. Failure of
1 The bulbs are of equal brightness. The same one component does not affect the other
current flows through all the bulbs. components.
2 If one of the bulbs is unscrewed, the remaining
bulbs also go off. The whole circuit breaks off if Conclusion
one component fails to allow current to flow Circuit (a) is a series circuit while circuit (b) is a
through. parallel circuit.

Activ To study series and parallel electrical circuits


ity 2.4
SPM SPM
’04/P1 ’07/P1

Apparatus/Materials Procedure
Voltmeter, ammeter, two resistors, dry cells (2 × 1.5 V), 1 A 1 Ω resistor, P and a 2 Ω resistor, Q are
F
O and battery holder. connected to a 3 V battery in a series circuit or a
2

R Arrangement of apparatus parallel circuit as shown in Figures 2.34 (a) and


CHAPTER

M (b).
2 The ammeter and voltmeter readings for each
5
circuit are compared.

Results and Conclusion


Table 2.10 shows a comparison between the values
of current and potential difference in a series circuit
(a) Series circuit (b) Parallel circuit
F parallel circuits
Figure 2.34 Series and
and a parallel circuit.
O
2

R Table 2.10
CHAPTER

M
Series circuit Parallel circuit
5
(a) (c)
Readings of 3V Reading of ammeter
ammeters A1 and A2 A is the sum of
A
are the same. A1 readings on
∴ I1 = I2 A1 and A2.
A1 A2 Series resistors 1
∴ I = I1 + I2
have the same 2 3 The two resistors
A2
current in them. share the main
current.
(b) (d)
Reading of 3V Readings of
voltmeter V is the voltmeters V1 and V2
sum of readings on are the same.
Activity 2.3 & 2.4

V1
V1 and V2. 1Ω
∴ V1 = V2
V1 V2 ∴ V = V1 + V2 Parallel resistors
V The two resistors 2Ω have the same
share the applied voltage across
V2
voltage. them.

Electricity 392
Advantages of Parallel Circuits effective resistor and its resistance is the
effective resistance or combined resistance of
1 Most of the household electrical appliances are M and N.
connected in parallel.
2 There are two advantages of a parallel circuit
over a series circuit.
(a) If one of the electrical appliance does not
function, the other appliances can still be
used.
(b) Additional appliances can be connected in
parallel with the existing appliance
without the need for more voltage. (a)
3 If there are two lamps in a room, the failure of
one lamp will have no effect on the other
lamp.
4 Similarly, the addition of another lamp to the
room will not affect the brightness of the
existing individual lamps.

(b) F
Effective Resistance (Combined Resistance) O
Figure 2.35 Concept of effective resistance

2
R

CHAPTER
1 In Figure 2.35 (a), the resistors M and N allow 2 An effective resistor is a single resistor that M
a current of 1 A to flow in the circuit. If the replaces 2 or more resistors in series or parallel
5
resistors M and N are replaced by a single and yet has the same effect on the circuit by
resistor, L [Figure 2.35 (b)] which also allows a allowing the same amount of current to flow
current of 1 A to flow in the circuit, L is the in the circuit.

A circuit breaker is an automatic switch which trips when Solution


the current flowing exceeds a specific safe value. An First, we need to find the total current flowing when the
electric kettle of 28 Ω, a bread toaster of 30 Ω and a three appliances are used concurrently.
microwave oven of 20 Ω are connected in parallel to a For electric kettle:
240 V power supply and a 20 A circuit breaker.
V 240
I= =
R 28
circuit breaker
= 8.6 A
20 A
For bread toaster:
240 V
V 240
I= =
R 30
= 8.0 A
For microwave oven:
V 240
I= =
R 20
= 12.0 A
Total current = 8.6 + 8.0 + 12.0
= 28.6 A
Will the circuit breaker trip when all the three appliances Since the total current exceeds 20 A, the circuit breaker
are used concurrently? will trip.

393 Electricity
Derivation of a Formula for Effective Resistance

Resistors in series Resistors in parallel

1 1

1 2

2 2

1 2

(i) (ii) (i) (ii)


In the series circuits shown above, In the parallel circuits shown above,
Vac = Vab + Vbc I = I1 + I2
= V1 + V2 VPQ VPQ
= IR1 + IR2 = +
R1 R2
= I(R1 + R2) ……

F
= VPQ
( 1
+
1
R1 R2)……

O
2

R When the two resistors are replaced by a single When the two resistors are replaced by a single
equivalent resistor of resistance, R as shown in
CHAPTER

M equivalent resistor of resistance, R as shown in


diagram (ii), it allows the same current I, to flow in diagram (ii), it allows the same current I, to flow in the
5
the circuit under the same applied potential circuit under the same applied potential difference, VPQ.
difference, Vac.
VPQ
∴I = R
∴ Vac = I R …………..
From  Vac = I (R1 + R2)
F
I = VPQ
()
1
R …………..

( )
as V and I are the same for 
O and .
1 1
2

∴ R = R1 + RR2 From  I = VPQ +


R1 R2
CHAPTER

5 as V and I are the same for  and .


∴ 1 =
R
1
+
R1 R2
1
( )
Conclusion Conclusion
With three or more resistors in the circuit: With three or more resistors in the circuit:
Effective resistance, R is given by Effective resistance, R is given by

R = R1 + R2 + R3 + ….

The value of the effective resistance, R in a series


1
R = ( 1 1 1
R1 + R2 + R3 + … )
circuit is larger than each of the individual The effective resistance, R in a parallel circuit is
resistors. smaller than each of the individual resistors.
The combination of two resistors in series effectively The combination of two resistors in parallel effectively
forms a longer resistor with a higher resistance. forms a resistor with a larger cross-sectional area and
thus its combined resistance is lower.
1 2 A1
2

= 1 + 2
1 A2 A = A 1 + A2

Electricity 394
SPM
Combined Circuit ’08/P2
8
A circuit consisting of a combination of resistors
Calculate the effective resistance for (a) and (b).
connected in series and in parallel is called a
combined circuit.

9
(a) (b)
Figure 2.36
Calculate the effective resistance in each of the
following circuits.
Solution
(a) R = R1 + R2 + R3 Value of the effective
= 2Ω+3Ω+5Ω resistance, R is larger
= 10 Ω than the resistance of
each individual resistor. (a) (b)
Figure 2.37

Solution
(a)
1 1 1 1
(b) = + + F
R R1 R2 R3
O

2
1 1 1 R
= + +

CHAPTER
2 3 5 M
31 1 1 1
= Value of effective = + 5
30 R′ R1 R2
resistance, R is less than
30 Ω the resistance of each 1 1
R = individual resistor. = +
31 2 4
3
=
30 4
31
4 1
R′ = Ω=1 Ω
3 3
1
Effective resistance, R = 3 + 1
3
1
= 4 
3

(b)

For two resistors in series:


What is the effective resistance of P and Q in the R′ = 2 + 4 = 6 Ω
above diagram?
For two resistors in parallel:
1 1 1
1 =1+1 = +
R =1+1 Students R 6 6
2 3 forget to use R 2 3
2
=3+2 the reciprocal =3+2 =
6
6 of R. 6
=5Ω =5 =
1

6 6 3
R =6Ω Effective resistance, R = 3 
5

395 Electricity
Special formula for two resistors in parallel:
1 1 1 R + R2 R1R2 Product of 2 resistances
= + = 1 ∴ R = =
R R1 R2 R1R2 R1 + R2 Sum of 2 resistances

10 SPM
’08/P1

Calculate the current flowing in the circuit shown in Solution


Figure 2.38. 3 × 4 12
R = =
3+4 7
V
I =
R
6
=
12
7
7
F = 6 ×
O
12
2

R = 3.5 A
Figure 2.38
CHAPTER

5
A Short-cut for Finding Effective Resistance

When two or more resistors with the same resistance are connected in
parallel, a quick calculation of the effective resistance can be made.
1 1 1 2
F (a) = + =
O R 6 6 6
2

R 6
R = =3Ω
CHAPTER

M 2
5
⇒ Effective resistance is half of the resistance of one resistor.
1 1 1 1 3
(b) = + + =
R 6 6 6 6
6
R = =2Ω
3
⇒ Effective resistance is one third of the resistance of one resistor.
Generalised formula:
R
Reffective =
n

where n is the number of resistors with the same resistance.


For example,
12 Ω
12 Ω
12 Ω
12 Ω
12 Ω
R1 = R2 = R3 = … = Rn = R
12 12
R= =6Ω R= =4Ω
2 3

Electricity 396
Problem Solving Technique:
Many problems can be solved by following
this procedure: S

Step11
Step Find the effective resistance in the
circuit. I 2Ω
Step22
Step Use the formula, I = V to find the
R
main current in the circuit. (a) When the switch S is open, the current needs to
Step33
Step Use the respective formula flow through the 2 Ω resistor, therefore the effective
resistance is 2 Ω.
involving V, I and R for each
(b) When the switch S is closed, the current will flow
section of the circuit. through the switch (which is of negligible
resistance), therefore the effective resistance is
11 zero.
R1 R2
Three resistors of 1 Ω, 3 Ω and 6 Ω are connected to or use the formula R =
R1 + R2
a 6 V supply as shown in Figure 2.39.
or where R1 = 2 Ω, R2 = 0 Ω,
2×0 0
or then R = = =0Ω
2+0 2
F
O

2
12 R

CHAPTER
V1 M
V2 Two resistors of 2 Ω and 3 Ω are connected to a
5
3 V supply as shown in Figure 2.40.
Figure 2.39
(a) What is the potential difference across the 1 Ω
resistor?
(b) What is the current flowing through the 3 Ω
resistor?
Solution
(a) For resistors in parallel:
1 1 1 3
= + =
R′ 3 6 6 Figure 2.40
R′ = 2 Ω What is the ammeter reading
Total effective resistance, R = 1 + 2 = 3 Ω (a) if switch S is in the open position?
V 6 (b) if switch S is closed?
Main current, I = = = 2 A
R 3
Solution
Potential difference across the 1 Ω resistor, V1
(a) Switch S in open position:
= IR
Effective resistance, R = 2 Ω + 3 Ω = 5 Ω
=2×1
V 3
=2V Main current, I = = A
R 5
(b) Potential difference, V2 across the 3 Ω and 6 Ω
resistors is given by: (b) Switch S closed:
V2 = (6 – 2) V = 4 V V1 + V2 = 6 Effective resistance of the 3 Ω resistor and switch
Current flowing through V1=2V
S in parallel is zero.
the 3 Ω resistor: Effective resistance for the whole circuit,
V R=2Ω
4
I = 2 = V
R 3 Main current, I =
R
1 3
= 1 A = A
3 2

397 Electricity
13
Six resistors of 6 Ω each are connected as shown in Effective resistance for the whole circuit,
Figure 2.41. R = 6 + 3 + 2 = 11 Ω
V
For A1 : I1 =
R
6
A
A1
=
A2
A3 11
1 1 6
For A2 : I2 = I1 = ×
2 2 11
3
= A
Figure 2.41 11
1 1 6
(a) What are the readings on ammeters A1, A2 and A3? For A3 : I3 = I1 = ×
3 3 11
(b) What is the voltmeter reading?
2
= A
Solution 11
(a) For two resistors in parallel: (b) Voltmeter reading, V = I1 R
1 6
R2 = × 6 = 3 Ω = ×6
F 2 11
O
For three resistors in parallel: 3
2

R = 3 V
1 11
CHAPTER

M R3 = × 6 = 2 Ω
3
5

To form an eight-shaped resistor


The diagram shows an 8 Ω resistance wire. Two ends of the wire are joined as shown in diagram (b) after which it is
formed into a symmetrical figure of ‘8’ as shown in diagram (c).
connected here

8Ω P Q
A
8V

(a) (b) (c)


What is the current, I flowing when the ends P and Q are connected to an 8 V supply?

Solution
8
Each of the four branches of the resistor has a resistance of = 2 Ω.
4

2Ω 2Ω

= 2Ω 2Ω =
P Q 1Ω 1Ω

The resistor can be considered as two pairs of parallel resistors as shown.

Answer
∴ Effective resistance, R = 1 + 1 = 2 Ω
V 8
⇒I= = =4A
R 2

Electricity 398
Common current flowing through two resistors Current split between two resistors
in series in parallel

The diagram shows two resistors, R1 and R2 connected in The diagram shows two resistors, R1 and R2 connected
series. in parallel.

As the same current, I flows through both resistors, As the potential difference, V across each resistor is the
V1 V2 same,
I= =
R1 R2 V = I1R1 = I2R2
V1 R1 I1 R2
∴ = ∴ =
V2 R2 I2 R1
F
O
That is, V is directly proportional to resistance, R. That is, I is inversely proportional to resistance, R.

2
R
This means that for two resistors in series, a larger This means that for two resistors in parallel, a larger

CHAPTER
M
proportion of the voltage is across the larger resistance, proportion of the current flows through the smaller
and vice versa. resistance, and vice versa. 5
It can also be shown that: It can also be shown that:

R1 R2
V1 = V I1 = I
R1 + R2 R1 + R2

(This formula is useful in the study of electronics.)


If R1 = R2 If R1 = R2
R1 R2
V1 = V I1 = I
2R1 2R2
1 1
V1 = V I1 = I
2 2

14
Figure 2.42 shows a simple circuit. Solution
R1
VXY = V
R1 + R2
2
= ×3
2+4
= 1V

Alternative method
Figure 2.42 First, find the current, I and then use the formula
What is the potential difference across XY? V = IR.

399 Electricity
15
Two resistors R1 and R2 of 6 Ω and 18 Ω respectively What is the value of y?
are connected in parallel as shown in Figure 2.43. The
values of the current flowing through R1 and R2 are
Solution
150 mA and y mA respectively.
I1R1 = I2R2 The potential
differences across
150 × 6 = y × 18 resistors in parallel are
y = 50 mA the same.

I1 R1 = 6 Ω Alternative method
Find the potential difference, V across R1 using the
formula V = I1R1.
V
I2 R2 = 18 Ω Then using the formula I = R , find the value of I
flowing through R2. 2

Figure 2.43

F
O
2

R
CHAPTER

M When solving problems involving complex circuit diagrams, we must always simplify the circuit diagrams to facilitate the
calculations. Following are some examples.
5
(a) (c)

(b) (d)

V V
V
V

6 SPM
Clone
’06

Which of the bulbs in the following four circuits is the brightest? Comments
A C This type of problem is normally solved by
R R finding the circuit that allows the highest current
through the bulb.
R R R
R The three resistors in circuit D are in parallel.
This provides the smallest resistance to the
B D circuit. The current that flows through the bulb
R R in circuit D is the largest and the bulb is the
R
brightest.
R
Answer D
R R

Electricity 400
7 SPM
Clone
’10

The diagram shows a 6Ω Solution



simple circuit. For the two 6- resistors, the effective resistance is equal

What is the reading on 6Ω
the ammeter? A
to 3 Ω =  2
. 
A 0.3 A
B 0.5 A Total effective resistance, R = 3 + 3 = 6 Ω
C 0.6 A V 3
I= = = 0.5 A
D 2.0 A R 6

Answer B

2.3

1 Find the effective resistance for each of the following 4 A simple circuit is
arrangements of resistors. connected as
(a) (c) shown in the
diagram. F
Calculate the O

2
currents I, I1 , I2 R
and I3.

CHAPTER
M
(b) (d)
5
5 (a) A 6 V power supply is connected in series with
two 3 Ω resistors. A switch is connected in
parallel with the resistor P. When the switch S is
2 Three resistors and an ammeter are connected to a in the open position, the ammeter reading is I1
SPM
battery of 12 V whose internal resistance is negligible
Clone
and when the switched is closed, the ammeter
’08 as shown in the diagram.
reading is I2.
8V
6V

6Ω A
3
1Ω
P 3

6Ω S Q

(a) Determine the effective resistance of the circuit.


(b) Find the reading on the ammeter. Calculate the ratio, I1 : I2.
(c) What will happen to the ammeter reading when (b) Three resistors of 3 Ω each are connected to a
one of the 6 Ω resistor is removed from the 6 V power supply as shown in the diagram
circuit? below. When the switch is in the open position,
3 Two ammeters A1 and A2 are connected to a circuit as the voltmeter reading is V1 and when the switch
shown below. The three resistors are identical. is closed, the voltmeter reading is V2.
6V

R
3

A1 P 3

R R 3 S Q
A2

What is the reading on A2 if the reading on A1 is 0.9 A?


Calculate the ratio, V1 : V2.

401 Electricity
6 Figure (a) shows a metal plate connected to a 3 V 7 Figure (a) shows a 6 Ω constantan wire connected to
dry cell battery. The resistance of the metal plate is a 3 V dry cell battery. The length of the wire is 100 cm.
3 Ω. The metal plate is cut into three identical P and Q are at a distance of 25 cm from the ends X
portions. Two portions are connected in series to the and Y respectively.
battery as shown in Figure (b). Calculate the
ammeter reading.

(a)
What is the ammeter
reading when P and Q are
in contact as shown in
(a) (b) Figure (b)?
[Reminder : Resistance is inversely proportional to
(b)
the cross-sectional area.]

6 In simple terms, the resistance within the cell


2.4 Analysing Electromotive or battery itself is called internal resistance.
F Force and Internal
O
Resistance To Show the Existence of Internal Resistance
2

R
CHAPTER

M
Internal Resistance 1 A torchlight is turned on for 20 minutes. The
5 dry cells in the torchlight become hot.
1 Figure 2.44 shows a simple circuit. 2 When a current flows in a resistor, the resistor
measures terminal potential difference becomes hot.
V 3 The current flows through the cell and the
internal
circuit external circuit.
r 4 The resistors in the external circuit become hot
internal
A resistance due to the flow of current. It can thus be
R1 R concluded that the cells become hot because
of their internal resistance.
total external
resistance 5 The internal resistance, r is the resistance
V V within a cell (due to its electrolyte/electrodes)
measures potential difference
across a component or source of electricity.
Figure 2.44
Electromotive Force
2 A circuit is a closed loop through which
current can continuously flow. 1 A cell is an electrical source which uses
(a) External circuit: The path taken by the chemical reactions to produce a current, i.e.
current outside the cell or the battery. changes chemical energy into electrical energy.
(b) Internal circuit: The path taken by the 2 A battery is a combination of two or more cells
current within the cell or the battery. connected in series.
3 When the current flows in a circuit, it 3 Something has to be in a circuit to make a
encounters resistance. current flow through the circuit.
4 For the external circuit, the resistance is known 4 That something is the electromotive force of
as external resistance. the cell or the battery. Electromotive force is
5 A cell consists of electrodes in a chemical measured by a voltmeter in volts (V ).
electrolyte. When the cell is connected in a 5 However in section 2.2, potential difference is
circuit, the current flowing in the electrolyte also responsible for current flow.
through electrodes experience a resistance 6 In order to differentiate between potential
which is known as source resistance or difference and electromotive force, we need to
internal resistance. carry out Activity 2.5.

Electricity 402
Activ To show the relationship between electromotive force and
ity 2.5 potential difference
Apparatus/Materials Tabulation of data
Voltmeter, connecting wires, ammeter, dry cells, Table 2.11
switch and a 2 Ω resistor.
Condition Ammeter Voltmeter
Arrangement of apparatus reading, reading,
V V I (A) V (V)
switch switch
Open circuit zero 1.5
Closed circuit 0.6 1.2

Discussion
1 It is observed that when current flows in the
closed circuit, the potential difference across the
(
(a) Open circuit (b) Closed circuit cell drops from 1.5 V to 1.2 V. The potential
Figure 2.45 difference across the cell or source of electricity
Procedure in a closed circuit (current is flowing), is called
the terminal potential difference, Vt.
1 The circuit is set up as shown in Figure 2.45(a).
2 The potential difference across the cell or F
2 The voltmeter is connected to the terminals of a O
source of electricity in open circuit (no current

2
dry cell. R
flow), is called the electromotive force.
3 The voltmeter and ammeter readings in open circuit

CHAPTER
M
are recorded, i.e. before the switch is closed. Conclusion
5
4 The switch is closed and the voltmeter and Electromotive force is not equal to the potential
ammeter readings are recorded. difference.

The Relationship between Electromotive Force From this we can define:


and Terminal Potential Difference (a) Electromotive force, E, is the work done
by a source in driving 1 coulomb of charge
1 (a) A voltmeter connected to the terminals of around a complete circuit (external circuit
a cell on open circuit measures the and internal circuit).
electromotive force, e.m.f. or E of the cell. (b) Terminal potential difference, Vt, is the
(b) A voltmeter connected to the terminals work done by a source in driving
measures the terminal potential 1 coulomb of charge through the external
difference, Vt when the cell sends current resistor.
through the external resistor. It is noticed (c) The ‘lost volts’ (the voltage drop), Vd, is
that: the work done by a source in driving
Vt < E 1 coulomb of charge through the cell (the
internal resistance).
(c) The ‘lost volts’ or voltage drop, Vd is the (d) The potential difference across a component
difference between E and Vt. is the work done by a source in driving 1
Therefore, we have, from Activity 2.5: coulomb of charge through the component.
1.5 V = 1.2 V + 0.3 V 3 The work done will be converted into a
E = Vt + Vd different form of energy according to the
function of the electric component.
The ‘lost volts’, Vd (= E – Vt) is the potential 4 Since work done and energy are interchangeable,
difference required to drive the current we can also define the electromotive force of a
Activity 2.5

through the internal resistance. It cannot cell as the energy supplied by a cell per
be measured directly from the voltmeter. coulomb of charge to the whole circuit.
2 As 1 V = 1 J C–1, 5 A 1.5 V dry cell can supply 1.5 J of energy after
∴ 1.5 J C–1 = 1.2 J C–1 + 0.3 J C–1 1 C of charge passes through the whole circuit.

403 Electricity
Vt Vt = IR where R is the combined external resistance
Vd = Ir
V1 = IR1 (where R1 is the resistance of a component)
V2 = IR2
r
And:
E = Vt + Vd
Ι R1 R2 = V1 + V2 + Vd
The cell provides an electromotive force which sets up
potential differences across the various components
V1 V2 including the internal resistance and drives the current
through them.

SPM
’07/P1

V1 Formulae for calculations:

F
O R
2

R
Ι
CHAPTER

M E Vd
R

5 V2

Vt
Voltmeter connected Voltmeter connected
R
to the terminals of a to the terminals of a
cell resistor
E = Vt + Vd where Vt = IR and Vd = Ir
V1 = V2 E = IR + Ir
V = IR
E = I(R + r)
The two are the same because the connecting wires from
the cell to the resistor are of negligible resistance. E
I = ––––
R+r
Electromotive force
= –––––––––––––––––––––––
Total resistance in the circuit

16 SPM
’04/P1

Figure 2.46 shows a simple circuit consisting of a 2 V Calculate


dry cell with an internal resistance of 0.5 Ω. When the (a) the resistance, R,
switch is closed, the ammeter reading is 0.4 A. (b) the voltmeter reading in closed circuit.
Solution
(a) E = I (R + r)
2 = 0.4(R + 0.5)
5 = R + 0.5
R = 4. 5 Ω
(b) V = IR
= 0.4 × 4.5
Figure 2.46 = 1. 8 V

Electricity 404
17
A voltmeter registers a reading of 1.5 V when it is Solution
connected directly to a dry cell. When a resistor, R is (a) E = Vt + Ir (b) Vt = IR
connected to the cell, the voltmeter reading decreases 1.5 = 1.4 + 0.2r 1.4 = 0.2R
to 1.4 V. The current flowing is 0.2 A. Calculate 0.1 = 0.2r R = 7Ω
(a) the internal resistance of the cell, r = 0.5 Ω
(b) the value of R.

18
When a battery with an electromotive force, E and (a) Use the formula: E = I (R + r)
internal resistance, r is connected to a 2 Ω resistor, the For (i) : E = 0.6(2 + r)
current flow is 0.6 A. When the 2 Ω resistor is For (ii) : E = 0.2(7 + r)
replaced by a 7 Ω resistor, the current flow is As the electromotive force, E is the same,
0.2 A. Calculate ∴ 0.6(2 + r) = 0.2(7 + r)
(a) the internal resistance, r and 3(2 + r) = 7 + r
(b) the value of E. 6 + 3r = 7 + r
2r = 1
Solution F
r = 0. 5 Ω O
(b) The value of E can be calculated by substituting

2
R
the value of r either in (i) or (ii).

CHAPTER
M
E = I (R + r)
5
= 0.6(2 + 0.5)
= 1. 5 V
(i) (ii)

Activ To determine the electromotive force, E and the internal


ity 2.6
SPM
’05/P1
resistance, r of a battery
Apparatus/Materials Tabulation of data
Voltmeter, ammeter, rheostat, switch, connecting
wires, two dry cells and battery holder. Table 2.12
Procedure Arrangement of apparatus
1 The circuit Current, I (A) Potential difference, V (V)
is set up as switch S 0.2 V1
shown in 0.4 V2
Figure 2.47.
0.6 V3
2 The switch S
is closed. 0.8 V4
The rheostat 1.0 V5
is adjusted rheostat
to set the Graph
current flow Figure 2.47
to 0.2 A.
The ammeter and voltmeter readings are
recorded. =
Activity 2.6

3 Step 2 is repeated with current, I = 0.4 A, 0.6 A,


0.8 A and 1.0 A. The corresponding voltmeter
readings are recorded.
4 A graph of V against I is plotted with both axes O

of the graph commencing at the origin. Figure 2.48

405 Electricity
Discussion The electromotive force, E = intercept on the
1 The I-axis must start from the origin as the V-axis
intercept on the V-axis is required. = c
–r = m = gradient
2 From the formula: E = V + Ir a
∴ Internal resistance, r = –m = |m| =
b
⇒ V = –rI + E
Compare with the linear equation: Conclusion
The electromotive force and the internal resistance
y = mx + c
of the battery are determined from the intercept on the
V-axis and magnitude of the gradient respectively.

Comparison between total e.m.f. and total internal resistance in a series circuit and a parallel circuit.
(Cells of equal e.m.f. and internal resistance.)

Cells in series Cells in parallel


F E1 , r1 E1 , r1 E1 , r1
O
Circuit diagrams
2

R
CHAPTER

M E1 , r1

5
E = 2E1
E = E1
= 2 × 1.5 V
Total e.m.f. (effective e.m.f.), = 1.5 V
∴ The total e.m.f. of the
E ∴ The effective e.m.f. is equal to
cells is equal to the sum
that of one cell.
of the separate e.m.f.s.

1
r = r
r = 2r1 2 1
= 2 × 0.5 Ω 1
= × 0.5 Ω
∴ The total internal 2
resistance is equal to the Total internal resistance, r
∴ The effective internal resistance
sum of the separate
r
internal resistances of the is given by , where n is the
n
cells.
number of cells.

Two lamps labelled 12 V, 24 W are respectively connected to eight 12 V, 24 W


1.5 V dry cells and a car battery comprising six lead-acid
accumulators of 2.0 V each. Which lamp is more brightly lit?
battery

12 V, 24 W
Activity 2.6

y
Batter
dry
cells

Electricity 406
Solution
The internal resistance, r of the car battery is smaller than the total internal resistance of the eight dry cells.
E
From the formula I = , the smaller value of r results in a larger current in the car battery and thus the lamp is brighter.
R+r

Answer
The lamp connected to the car battery is more brightly lit than the lamp connected to the dry cells.

A brighter bulb Solution


Mr. Brown arranges circuit (b) instead of circuit (a) to The e.m.f. is the same but the internal resistance in circuit
make a light bulb brighter. (b) is halved.
E, r E
As I = (where r’ is the effective internal resistance),
E, r R + r’
the I in circuit (b) is larger.
E, r
Since power, P = EI, the bulb in circuit (b) is brighter.

Note: For cells in parallel, the cells share the work of F


(a) (b) driving the current. Therefore, they can last longer O

2
Explain why. than when one cell is used. R

CHAPTER
M

19
When a thick copper wire is connected to the Solution
terminals of a dry cell, the maximum current flow is E
I=
obtained. What is the maximum current that can be R+r
supplied by a dry cell with an e.m.f. of 1.5 V and
1.5 R = 0 for maximum
internal resistance of 0.5 Ω? I= = 3A current flow.
0.5

20
Three resistors are connected to a 3 V battery as Solution
shown in Figure 2.49. The internal resistance of the For 2 resistors in parallel:
battery is 1 Ω. 1 1 1 3 1
= + = =
E = 3 V, r = 1 Ω R′ 6 12 12 4
R′ = 4 Ω
Total external resistance, R = 4 + 1 = 5 Ω
E
I =
R+r
3
=
5+1
3
= = 0.5 A
Figure 2.49 6
V = IR = 0.5 × 1 = 0. 5 V
What is the voltmeter reading?

407 Electricity
8 SPM
Clone
’10

A resistor in series with an ammeter is connected to a Solution


cell which has an e.m.f 1.5 V. The potential difference Applying E = Vt + Ir
across the cell reads 1.2 V while the reading on the E – Vt
ammeter is 0.6 A. r =
I
What is the value of the internal resistance?
A 0.5 Ω 1.5 – 1.2
= 0.6
B 0.8 Ω
C 1.0 Ω = 0.5 Ω
D 2.0 Ω Answer A

9 SPM
Clone
’05

Which of the following circuits can be used to determine the electromotive force of a battery?
F
O A V C A
2

R
CHAPTER

5 V
A

B D

A V V A

Comments
A voltmeter and not an ammeter is to be used to measure the electromotive force.
The voltmeter should be connected across the battery with an open circuit.
Answer A

10 SPM
Clone
’07

A simple circuit is set A 5.0 Ω C 0.8 Ω


B 1.2 Ω D 0.6 Ω
e.m.f. = 3.0 V
up as shown beside.

A 0.6 A Solution
Applying E = I (R + r),
3 = 0.6(3.8 + r)
3.8 Ω
3 = 2.28 + 0.6r
r = 1.2 Ω
Calculate the internal resistance of the battery. Answer B

Electricity 408
2.4

1 The diagram shows a dry cell, with an electromotive difference across the battery when it is connected to
force of 1.5 V, connected to a resistor, R. The voltmeter a 2.5 Ω resistor?
and ammeter readings are 1.2 V and 0.3 A respectively. 3 Two 2 Ω resistors are connected in series to a battery
with an electromotive force, E and internal resistance,
V
r. The current flow is 0.4 A. When the same two
resistors are connected in parallel, the current flow is
E = 1.5 V
1.2 A. Calculate
(a) the electromotive force, E,
A
(b) the internal resistance of the battery.
4 A battery with an e.m.f. of P volts and internal
resistance, r Ω is connected to a resistor, R. What is
the potential difference (in terms of P) across the
R
battery if R = r?
Calculate 5 A thick wire is connected to the positive and negative
(a) the resistance of R, terminals of a lead-acid accumulator with an e.m.f. of
F5/2/126
(b) the internal resistance, r of the dry cell. 2 V and an internal resistance of 0.01 Ω. What is the
2 A battery has an electromotive force of 6 V and an maximum current that can be supplied by the
internal resistance of 0.5 Ω. What is the potential accumulator?
F
O

2
R

CHAPTER
M

2.5 Analysing Electrical Energy and Power 5

Electrical Energy

1 Figure 2.50 shows a battery connected to a light bulb.

electron flow

current

Energy conversion: Chemical energy → Electrical energy → Light energy + Heat energy

Figure 2.50 Energy is transferred from the battery to the bulb

2 Chemical energy in the battery is converted into electrical energy with the production of an
electric current which lights up the bulb.
3 Electrical energy is converted into light and heat energy in the bulb.
4 Table 2.13 shows the energy conversion in some common appliances which use electricity.
Table 2.13

Type of appliance Energy conversion


(a) Electric fan Electrical energy → Kinetic energy
(b) Electric kettle, hot plate, iron Electrical energy → Heat energy
(c) Hair dryer Electrical energy → Heat energy + kinetic energy
(d) Radio Electrical energy → Sound energy

409 Electricity
Relationship between Electrical Energy and Electrical Power SPM
’07/P2(A)
SPM
’07/P3(B)
SPM
’08/P1

1 The SI unit of electrical energy is the joule, J.


2 Power is the rate of transfer of electrical energy, that is, energy transferred per second.
The SI unit of electrical power is the watt, W, where 1 W = 1 J s–1.
3 Formulae for electrical energy and electrical power:
(a) From the definition of potential difference (b) From the definition of power, P:
or voltage, V across two points (page 376): Electrical energy
P = ––––––––––––––––
Electrical energy converted Time taken
V = ––––––––––––––––––––––––––
Charge VIt
P = ––––
V = –E– t
Q Substitute Q = It P = VI
and E = QV
= VIt

Summary:

V
F Substituting V = IR: Substituting I = R :
O P = IR × I
V
2

R
P=V× R
CHAPTER

2 1 3
V2
P=I R 2
P = VI P = ––
R

F
O
2

R Energy = Power × Time


CHAPTER

2 1 3
V 2t
E = I Rt 2
E = VIt E = ––
R

V
Substituting I = R :

Substituting V = IR: V
E=V× R ×t
E = IR × I × t

4 An electrical appliance is always labelled with its voltage rating (V)


which shows the suitable voltage for operating the appliance and its
power rating (W). The power rating is an indication of the electrical
power that will be consumed when operating at the rated voltage.
5 A light bulb labelled at 12 V, 24 W means that the light bulb will
release 24 J of light and heat energy in one second when it is connected
to a 12 V supply.

Electricity 410
21 SPM
’06/P1

Figure 2.51 shows three resistors L, M and N connected Solution


to a battery with an internal resistance of 1 Ω. (a) The effective resistance of M and N
1
= ×6=3Ω
E = 6 V, r = 1 Ω 2
E
Current, I =
R+r
6
=
2+3+1
= 1A
1
(b) Current flow in resistor M, Im = I
2
Figure 2.51 = 0.5 A
Time, t = 2 minutes
= 2 × 60
Calculate = 120 s
(a) the ammeter reading, Energy dissipated, E = I2Rt
(b) the energy dissipated by the resistor M in 2 minutes. = 0.52 × 6 × 120
F
O
= 180 J

2
R

CHAPTER
M

22
An immersion heater has a power rating of 240 V, Many students have the understanding that the
resistance of the filament causes it to be electrically
750 W.
heated and emits light. Hence, students conclude that a
(a) What is the meaning of its power rating?
light bulb with a higher power rating has a higher
(b) What is the resistance of the immersion heater? resistance. In actual fact, a light bulb with a high power
(c) What is the electrical energy consumed in rating has a low resistance. Please refer to Example 23.
15 minutes?
Solution
(a) The power rating of the heater shows that the
23
heater releases 750 J of energy per second Figure 2.52 shows a battery of negligible internal
when it is connected to a 240 V supply. resistance, connected to two lamps X and Y with
V2 power ratings of 12 V, 12 W and 12 V, 24 W
(b) Power, P =
R respectively. The resistance of the filaments of X and
240 2 Y are R1 and R2 respectively.
750 =
R
2402
R =
750
= 76.8 
(c ) Time, t = 15 minutes
= 15 × 60 s
= 900 s
Electrical energy consumed, E
= P×t
= 750 × 900 Figure 2.52
= 675 000 J
(a) Lamp Y is brighter. Explain why.

411 Electricity
(b) Calculate (b) (i) Power, P = VI
(i) the ratio I1 : I2, For lamp X: For lamp Y:
(ii) the ratio R1 : R2. 12 = 12 × I1 24 = 12 × I2
(Assume that resistance does not depend on I1 = 1 A I2 = 2 A
temperature) ∴ Ratio I1 : I2 = 1 : 2
(c)
(ii) Voltage, V = IR
For lamp X: For lamp Y:
12 = 1 × R1 12 = 2 × R2
R1 = 12 Ω R2 = 6 Ω
∴ Ratio R1 : R2 = 12 : 6 = 2 : 1

(c) The lamp Y which has a higher power rating has a


Figure 2.53 lower resistance. As a result, the voltage across Y
When lamps X and Y are connected in series as is less than the voltage across X. From P = VI,
shown above, then Y with a higher power is less where I is the same for both X and Y, X will
bright than X. Why? dissipate more energy and be brighter. However,
the brightness of both lamps are less than the
Solution
normal brightness.
F
(a) Lamp Y is brighter because it has a higher power The two lamps share the supply voltage of 12 V.
O rating than lamp X, and both are connected to the The voltage across each lamp is less than 12 V.
2

R voltage supply, 12 V.
CHAPTER

Definition of potential difference and electromotive force, e.m.f. as a ratio of power to current:

Power, P = VI Potential difference is defined as the


Potential
P Power supplied ratio of the power supplied to the current
difference, V V= =
I Current flowing.

Power, P = EI Electromotive force (e.m.f.) is defined


Electromotive
P Total power supplied as the ratio of the total power supplied
force, E (e.m.f.) E= =
I Current to the whole circuit to the current flowing.

Table 2.14
Power Rating and Energy Consumption of
Electrical Appliances SPM
’04/P2(A)
SPM
’09/P1 Appliance Power (watts)
1 We use electrical appliances in our homes. Air conditioner 1350
2 Different electrical appliances use electrical Water heater 2500
energy at different rates.
Computer 360
3 The power rating of an appliance denotes the
rate at which it consumes electrical energy. Television 200
For example, a 2000 W hair dryer uses 2000 Hair dryer 1250
joules of electrical energy per second. Iron 1200
4 The typical appliance power ratings are shown
Toaster 1100
in Table 2.14.
5 Another unit that is often being used is horse Washing machine 200
power (h.p.). Fluorescent lamp 40
1 h.p. = 746 watts Rice cooker 350 – 1350
6 Notice that the power ratings for electrical Refrigerator 250
appliances that involve heat energy are Electric kettle 2000
comparatively high.

Electricity 412
7 Since Energy = Power × Time:
25
E=P×t
Electrical appliances with high power ratings Figure 2.54 shows a
consume more electrical energy in a given time. lamp with two
filaments P and Q
SPM
Cost of Using Electrical Energy ’03/P1 rated at 50 W and
100 W respectively.
1 The use of electrical energy must be paid for The circuit of each
and the cost depends on the total quantity of filament can be closed
energy consumed in a given period. independently.
2 The total consumption of electrical energy in a
home is recorded by a kilowatt-hour meter Figure 2.54
which is located outside. A meter reader from
the power company, Tenaga National Berhad (a) Why are the two filaments connected in parallel?
(TNB) reads the meter on a monthly basis to (b) What is the cost of using the lamp at its brightest
issue the electricity bill according to the tariff for 1 hour? The cost of electricity is 21.8 sen for
rates determined by TNB as in Table 2.15. the first 200 units.

Table 2.15 Electricity tariff Solution


(a) The filaments are connected in parallel so that 3 F
Energy consumption (unit) Rate (sen/unit) different brightness can be obtained. O

2
1–200 21.8 (b) When the brightness is at its maximum: R

CHAPTER
201–400 34.5 P = 150 W = 0.15 kW M
Energy = Power × Time
5
3 For monthly consumption more than = 0.15 kW × 1 hr
400 units, a different tariff is imposed, starting = 0.15 unit
with 30 sen/unit for the first 500 units. Cost = 0.15 × 21.8 sen
4 The electricity tariffs for domestic use and = 3.27 sen
industrial/commercial use are different.
5 One unit of electrical energy is the electrical
energy used by an electrical appliance with a Efficiency of Electrical Appliances SPM
’09/P2/(C)
power rating of 1 kW for 1 hour. That is,
1 Conversion of energy takes place in an
1 unit = 1 kW × 1 hour = 1 kWh
electrical circuit. In the process of conversion,
Also: electrical energy input is converted into the
1 unit energy form of energy that is required as output.
= 1 kWh However, a portion of the energy output is lost
= 1 kW × 1 hour to the surroundings as heat.
= 1000 W × 3600 s 2 From the relationship between electrical
= 3 600 000 J 1 W = 1 J s–1 energy and electrical power (page 410), the
= 3.6 × 10 J
6
efficiency of an electrical appliance can be
defined as:
24
Eo
A lamp consumes electrical energy of 2 kWh in Efficiency = × 100%
Ei
40 hours. What is its power?
Po
Solution Efficiency = × 100%
Pi
Energy = Power × Time
2 = P × 40
2 where Eo = useful energy output
P= = 0.05 kW Ei = energy input
40
Po = useful power output
= 50 W
Pi = power input

413 Electricity
3 The efficiencies of all electrical appliances 4 For example, in the case of an electric motor
are less than 100% because not all the used in a lift, part of the energy input is
energy input is converted into useful energy converted into heat and to overcome frictional
output. forces.

26
An electric motor is used to lift a load of mass 600 g to Electrical energy supplied to motor, Ei
a height of 5 m in 8 s. If the supply voltage is 10 V and = VIt
the flow of current in the motor is 0.6 A, calculate the = 10 × 0.6 × 8
efficiency of the motor. = 48 J
Eo
Solution Efficiency of motor = × 100%
Ei
Energy output converted into potential energy, Eo.
Eo = mgh =
30
× 100%
= 0.6 × 10 × 5 48
F = 30 J = 62.5%
O
2

R
CHAPTER

Tungsten-filament Lamp and Fluorescent Lamp

bulb tungsten filament


argon gas

connecting wire

(a) Filament lamp

fluorescent powder

electron

mercury and argon gases

(b) Fluorescent lamp


Figure 2.55
1 The function of a tungsten-filament lamp and fluorescent lamp is to
produce light.
2 Both types of lamps produce heat in addition to light.
3 The fraction of the electrical energy that is lost as heat is wastage.
4 A fluorescent lamp produces less heat than a tungsten-filament lamp.
Therefore, a fluorescent lamp has a higher efficiency.

Light energy produced


Efficiency of lamp = × 100%
Electrical energy supplied
5 The efficiency of a fluorescent lamp is 4 to 6 times the efficiency of a
tungsten-filament lamp.

Electricity 414
11 SPM
Clone
’11

The diagram shows a A Solution


V2
simple circuit. Applying P =
What is the power 5V 6Ω R
dissipated in the 6-Ω 52
=
resistor when the switch 6
switch is turned on?
= 4.2 W
A 0.8 W C 4.2 W
B 1.5 W D 30.0 W Answer C

Ways of Increasing Energy Efficiency appliances, as a result they require more


energy/power to function.
1 The efficiency of an electrical appliance can be 5 Ways of operating an electrical appliance correctly:
increased by increasing the energy output with (a) Adopting good operating practice. For
respect to the energy input. example, turn off the appliance when not
2 The efficiencies of electrical appliances are low in use.
because a proportion of the energy output is 6 Research and development is undertaken by
F
lost in the form of heat. manufacturers to produce electrical appliances O
3 The loss of energy as waste heat can be reduced

2
which are more effective and energy efficient. R
by: For example:

CHAPTER
M
(a) Maintaining the appliance in good (a) Microprocessors are used in electrical
working condition. 5
appliances and equipment for precise
(b) Operating the appliance correctly. temperature control and regulation. These
(c) Producing electrical appliances with microprocessor controlled equipment,
higher efficiencies. for example, air conditioners and
4 Ways of maintaining electrical appliances are: refrigerators, operate with reduced energy
(a) Regularly cleaning and removing dust from wastage and thus their energy efficiencies
the air filters of air conditioners, blades of are high.
electric fans, cooling fins of refrigerators (b) The gradual replacement of computer
and filter bags of vacuum cleaners. screens of the cathode ray tube type with
(b) Dust and dirt sticking to electrical LCD screens reduces energy wastage.
appliances increase the load of the

12 SPM
Clone
’07

The figure shows a lighting circuit in a house. (b) The bulbs are labelled ‘240 V, 60 W’.
(i) Explain ‘240 V, 60 W’.
(ii) Calculate the current in the circuit when only
one bulb is lit.
(iii) Calculate the total resistance when the three
bulbs are lit.
L
(iv) How should the three bulbs be connected in
E order to increase the total resistance of the
N circuit?
(a) (i) Choose the correct answer in the following Answer
statement.
(a) (i) Parallel
‘The bulbs are connected in (series, parallel).’
(ii) The other bulbs will still light up.
(ii) What will happen to the other two bulbs if
one bulb blows?

415 Electricity
(b) (i) The bulb releases 60 J of light and heat Since the three bulbs are identical, they have
energy per second when a voltage of 240 V the same resistance = 960 Ω
is applied. Therefore, total (effective) resistance
V 1
(ii) Apply P = VI, (iii) Apply R = , = × 960 Ω
I 3
60 = 240I
240 = 320 
I=
60 R= (iv) Arrange the bulbs in series.
240 0.25
= 0.25 A = 960 Ω

2.5

1 A current of 2 A is allowed to flow through a 3 Ω 12 V


resistor. What is the heat energy produced in
5 minutes? What is the energy produced if the
current is doubled? I
2 The diagram shows an energy saving bulb labelled as
SPM 240 V 12 W. However, it produces only 8 joules of
Clone 1Ω 2Ω
F ’09 light energy in one second when it is connected to a
O 240 V power supply P Q
2

R
CHAPTER

M Calculate
(i) the current, I flowing,
5 (ii) the energy dissipated by each resistor.
(b) The diagram shows two resistors of 1 Ω and 2 Ω
connected in parallel to a 12 V battery.
240 V 12 V
11 W

P 1Ω
Calculate
(a) the current through the bulb,
(b) the heat energy lost per second and
(c) the efficiency of the bulb. Q 2Ω

3 The diagram shows three light bulbs A, B and C


What is the power dissipated by each resistor?
connected to a 12 V power supply.
6 An electric kettle has a power rating of 240 V, 2 kW.
If the efficiency of the kettle is 90%, what is the time
required to boil 3.6 litres of water from 25 °C?
[Specific heat capacity of water = 4200 J kg –1 °C–1,
Mass of 1 litre of water = 1 kg]
7 Sarimah uses an electric iron with a power rating of
1.5 kW for half an hour. If her sister Alimah spends
time surfing the internet which consumes 1.2 A, what
is the time, in hours, Alimah is allowed to surf so that
the same electrical energy is consumed by each of
the two sisters? [Voltage supply is 240 V]
Calculate the ammeter reading.
8 Azura intends to buy a microwave oven. She sees an
4 A lamp with a power rating of 12 V, 24 W is normally advertisement which says that a particular brand of
lit. What is the resistance of the lamp? If the efficiency microwave oven consumes only half a sen per
of the lamp is 45%, what is the heat energy minute. If the electricity tariff is 20 sen per unit, what
produced in 1 minute? is the energy consumed by the microwave oven, in
5 (a) The diagram shows two resistors of 1 Ω and 2 Ω units of joule (J), in 30 minutes? What is the power
connected in series to a 12 V battery. rating of the microwave oven?

Electricity 416
Q of semiconductors and carbon decreases when
1. Current is defined as rate of flow of charge, I = .
t temperature increases.
2. Electric field line is a path taken by a positive test 7. Effective resistance
charge in an electric field. (i) Resistors in series:
3. Ohm’s Law states that the current in a conductor is R = R1 + R2 + ……
proportional to the potential differences across the (ii) Resistors in parallel:
ends of the conductor if the physical conditions 1 1 1
remain constant. R = R1 + R2 + ……
4. From Ohm’s Law: 8. Internal resistance in a cell is due to the chemicals in
the cell.
V V 9. Electromotive force is the potential difference across
V = IR I= R=
R I the cell when there is no current flow.
10. From E = QV:
5. The resistance of a conductor depends on 2
(i) E = VIt (ii) E = I 2Rt (iii) E = V t
(i) length R
(ii) cross-sectional area, and energy
(iii) material From power, P = : F
time
6. When temperature increases, the resistance of O

2
2
metallic conductor increases. However, the resistance (i) P = VI, (ii) P = I 2R, (iii) P = V R
R

CHAPTER
M

2
Multiple-choice Questions

2.1 2 Which of the following shows the 3 Diagram 2 shows two identical
Electric Field and Flow of
correct electric field? spheres P and Q with the same
Electric Charge
A amount of charge.
1 Diagram 1 shows two resistors P + +

and Q.

P Q B

Diagram 1 Diagram 2
+ + + + +
At which point in the diagram would
18 coulombs of charge pass C an electron remain stationary?
through resistor P in 6 seconds.
How long would it take for
+ –

18 coulombs of charge to pass 2.2 Relationship between


through resistor Q? Electric Current and
D
A 3s +
+


Potential Difference
+ –
B 6s +
+
+



C 12 s +
+

– 4 P is an ohmic conductor. When the
D 54 s 1.1 Significant Figure voltage across P is 3 V, the current

417 Electricity
is 2 A. What voltage is needed to V (V)
increase the current to 6 A?
A 5V P
B 6V
C 8V 1Ω 1Ω
D 9V Q
3Ω
5 What is the main use of a I (A) 2Ω
rheostat? Diagram 7
A To control the magnitude of Diagram 4 If the current in resistor P is 3 A,
current in the circuit. what is the current in resistor Q?
What is the resistance of the
B To control the potential A 3.0 A C 1.5 A
resistor when the current is 3 A?
difference across the B 2.0 A D 1.0 A
A 0.7 Ω
battery.
B 1.3 Ω 13 Diagram 8 shows two resistors R1
C To increase the resistance of
C 2.0 Ω and R2 in parallel connected to a
the resistor used. SPM
Clone
D 3.0 Ω ’07 battery.
D To prevent heating up of the
connecting wires.
6 Which of the following electrical 2.3 A
Series and Parallel Circuits R1
SPM components is an ohmic
Clone
’09 conductor? 10 Diagram 5 shows two voltmeters
F A Constantan wire across two resistors of 5 Ω and R2
O B Vacuum diode 7 Ω respectively.
2

R
C Bulb filament 5Ω 7Ω Diagram 8
CHAPTER

M
D Thermistor
Which of the following physical
5 7 Diagram 3 shows two aluminium quantity is the same for both
rods P and Q of the same V V resistors?
length. 3V ?V A Power
Diagram 5 B Voltage
C Current
If the reading of the voltmeter
D Energy
P Q across the 5 Ω resistor is 3 V, what
is the reading of the voltmeter 14 Which of the bulbs in Diagram 9 is
a
a
across the 7 Ω resistor? the brightest, given that all the
d
A 2.1 V bulbs are identical.
Diagram 3 B 4.2 V
C 5.3 V
If both rods have the same D 11.7 V B
resistance, find the width of Q, a
A
in terms of the diameter of P, d. 11 Diagram 6 shows the
C
–
π 2 arrangement of six identical
A a= d C a = – d resistors in a circuit.
2 π
– D
B a = π d C a=d
Diagram 9
8 The readings on two voltmeters
15 Diagram 10 shows a simple circuit
across two identical light bulbs
with two resistors P and Q. When
P and Q are 1 V and 3 V
the switch is open, the reading on
respectively.
the ammeter is 1.5 A.
Which of the following statements Diagram 6
is correct?
A Bulb P and bulb Q are of the If the current in the resistor P is
2 A, what is the current in the A switch
same brightness.
B Bulb P is brighter than bulb Q. resistor Q? P
A 1A C 2A Q
C Bulb P is dimmer than bulb Q. 2Ω
1 1 2Ω
9 The graph in Diagram 4 shows the B 1 A D 2 A
3 2
relationship between the potential Diagram 10
difference applied to and the 12 Diagram 7 shows a simple circuit What is the reading on the ammeter
current flowing in a resistor. with four resistors. when the switch is closed?

Electricity 418
A 0.75 A 2.4 A 0.5 W C 2.0 W
B 1.5 A Electromotive Force and B 1.0 W D 2.5 W
C 3.0 A Internal Resistance
22 Which of the following correctly
D 6.0 A 19 Diagram 14 shows the graph of SPM
Clone shows the relationship between
the potential difference, V across a ’08 the power, P and the square of the
16 A wire PQ with resistance of 6 Ω is
battery against the current, I current, I2 in a wire with constant
cut into three equal portions
flowing through two types of resistance?
before been soldered as shown in
batteries P and Q. A P
Diagram 11. What is the
V (V)
resistance of the new wire?
P Q battery P
battery Q
Diagram 11
A 0.67 Ω I2
B 1.0 Ω I (A) B P
C 2.0 Ω Diagram 14
D 4.5 Ω
Which of the following
17 Three resistors of 2Ω are connected comparisons of electromotive
as shown in Diagram 12. force and internal resistance of I2
batteries P and Q are correct?
Q C P

e.m.f. Internal F
2Ω O
2Ω resistance

2
R
A e.m.f.P > e.m.f.Q rP > rQ

CHAPTER
M

P
B e.m.f.P > e.m.f.Q rP < rQ I2
2Ω D P 5
C e.m.f.P < e.m.f.Q rP > rQ
Diagram 12
D e.m.f.P < e.m.f.Q rP < rQ
What is the effective resistance of
the three resistors when points P
and Q are connected to a circuit? 20 Diagram 15 shows a battery with
A 1.3 Ω electromotive force of E V and I2
B 1.5 Ω internal resistance of 1 Ω 23 What is the resistance of a lamp
C 3.0 Ω connected to the midpoint, Q of a labelled 12 V, 24 W?
D 6.0 Ω wire, PR with resistance of 2 Ω. A 2Ω C 8Ω
The current flow is I A. B 6Ω D 18 Ω
18 Diagram 13 shows two circuits.
E V, 1 Ω 24 An electric iron is rated 240 V, 8 A.
What is its power rating?
A 30 W C 1920 W
B 240 W D 2000 W
25 Two resistors X and Y are connected
P Q R in series as shown below.
Diagram 15
What is the current flow if R is
connected instead of Q?
A 3I B 2I C 1I D 1I
4 3 2 3

Diagram 13 Diagram 16
The batteries and resistors are The resistance of X is twice the
2.5 Electrical Energy and Power
identical. What would be observed resistance of Y. Which of the
on voltmeters V1 and V2 when the 21 A battery with an e.m.f. of 3.0 V following comparison is correct
switches are closed? and internal resistance of 1 Ω is about the heat, Q dissipated per
A V1  V2 connected to a lamp with a second by the two resistors?
B V1 = V2 resistance of 2 Ω. What is the A QX  QY C QX  QY
C V1  V2 power supplied to the lamp? B QX = QY

419 Electricity
Structured Questions
1 (i) Find the effective resistance.
(ii) Find the new reading on the ammeter.
A 2Ω A 2Ω [3 marks]
1Ω 1Ω 1Ω 1Ω

3 P 0.4 Ω
2Ω 2Ω
(a) (b)
Diagram 1
Q 0.2 Ω
(a) Complete the circuits in Diagrams 1 (a) and (b) to
show how you could obtain effective resistances
of 3 Ω and 1.5 Ω respectively. [2 marks]
(b) For Diagram 1(b), calculate the reading on the
Diagram 4
ammeter if the electromotive force of the battery
is 3 V and the internal resistance is 0.5 Ω. (a) Two metal rods of the same length and material
[2 marks] are connected in parallel to a circuit as shown in
Diagram 4. The resistances of P and Q are 0.4 Ω
2 Diagram 2 shows a simple circuit. A voltmeter is
and 0.2 Ω respectively. What is the ratio of cross-
connected across the wire XY of length  to measure
sectional areas of P to Q? [2 marks]
F the potential difference while an ammeter is
(b) When the switch is closed, which rod will dissipate
O connected in series to measure the current flow.
heat at a higher rate? Give a reason for your
2

R 8V
answer. [3 marks]
CHAPTER

M
(c) (i) Name the force that acts between the two
5 rods placed close to each other when the
A switch is closed. [1 mark]
X  Y
(ii) What would happen if the direct current
source is replaced with an alternating
source? [2 marks]
V
(d) A copper wire is connected to the midpoints of
Diagram 2 P and Q as shown in Diagram 5. Find the effective
F
(a) The readings on the Oammeter and voltmeter are resistance of the system. [2 marks]
2

0.5 A and 1.0 V respectively.


R Find the resistance
copper wire
of the wire.
CHAPTER

M [1 mark] P
(b) What would happen to the reading on the
ammeter if 5
(i) the wire is submerged in water with ice
Q
cubes?
(ii) the wire is replaced with a thinner wire of Diagram 5
the same material and of the same length?
[2 marks]
4 Diagram 6 shows an immersion heater with a power
(c) The ends of the wire, X and Y are connected to
rating of 240 V, 500 W used to boil 500 g of water.
form a circle as shown in Diagram 3. The
common points X/Y and P are connected to the
1
circuit. P is at a distance of of the original length
4
of the wire from X/Y.
water
metal container

X/Y immersion heater

(a) (b)
1 Diagram 6

4
P The initial temperature of the water is 25 °C and the
Diagram 3 specific heat capacity of the water is 4200 J kg–1 °C–1.

Electricity 420
(a) Calculate the energy used by the heater in (ii) The actual time required is more than the
15 minutes in the following units: time calculated in (c)(i). Explain. [1 mark]
(i) Kilowatt-hour (d) A student proposes to replace the container with a
(ii) Joule [2 marks] similar container but with its external surface
(b) If the mass of the metal container is 100 g and its painted white. Will this reduce the time required
specific heat capacity is 1200 J kg–1 °C–1, calculate to boil the water? Explain. [1 mark]
the total heat required to boil the water (assume (e) Diagram 6 (b) shows a situation where the
the temperatures of the water and container are immersion heater is only half immersed in the
the same). [2 marks] water. Comment on the time taken to boil the
(c) (i) What is the minimum time required to boil water. [2 marks]
the water? [2 marks]

Essay Questions
5 Diagram 7(a) shows a smooth metal disc in a simple You are asked to help the hawker so that the lamp
circuit. The current recorded on the ammeter is gives the normal intensity.
I ampere. The disc is then cut into four equal parts and (i) Explain how you can achieve the result.
stacked up together before being connected to the [6 marks]
circuit as in Diagram 7(b). The current flow is (ii) Suggest further modifications that can be F
I′ ampere where I′ is smaller than I. made to further improve the efficiency of O

2
the lamp. Give reasons for your choice. R

CHAPTER
metal disc
[4 marks] M

5
I I
6 (a) A torch is used for 15 minutes. The dry cells in the
A A
torch become hot. Explain this observation.
[2 marks]
(b) Diagram 9 shows two circuits with a light bulb rated
I < I 1.5 V, 0.5 A.
(a) (b)
Diagram 7
(a) What is meant by resistance? [1 mark]
(b) Using Diagrams 7(a) and (b),
(i) compare the physical dimensions of
resistors (the disc and parts of discs),
(ii) compare the magnitudes of the two
currents, 1.5 V, 0.5 A 1.5 V, 0.5 A
(iii) relate the resistance to the physical (a) (b)
dimensions of a resistor, and
Diagram 9
(iv) state the relationship between the physical
dimensions of the resistors and the current
The light bulb in arrangement (b) is brighter
flows. [5 marks]
compared to arrangement (a). Explain.
(c) Find the ratio of I : I′. [4 marks]
[3 marks]
(d) A hawker uses 8 dry cells of 1.5 V each to light up
(c) Diagram 10 shows an electric kettle.
a lamp labelled 12 V, 24 W in his stall. He finds
that the light intensity is unsatisfactory.

Diagram 10

Table 1 shows the specifications of four electrical


kettles K, L, M and N of the same capacity. Each
Diagram 8 can be used to boil 1.2 kg of water.

421 Electricity
Table 1 Study the specifications of all the four electrical
kettles based on the following aspects:
Mass Specific heat The • The mass of the kettle
Kettle of capacity of surface of Power • The specific heat capacity of the kettle
kettle kettle the kettle • The surface of the kettle and
K 1.2 kg 500 J kg–1 °C–1 Shiny 1.5 kW • The power of the kettle
L 0.8 kg 500 J kg–1 °C–1 Dull 2.0 kW Explain the suitability of the aspects. [10 marks]
(d) A pump with power 2 kW can force m kg of water
M 0.8 kg –1
500 J kg °C –1
Shiny 2.0 kW
through a height of 16 m in 30 s. The pump works
N 1.5 kg 900 J kg–1 °C–1 Dull 1.2 kW with an efficiency of 70 %.
You are required to determine the most efficient (i) What is the energy input in 30 s?
electrical kettle. (ii) Determine the value of m. [4 marks]

Experiments
1 is the gradient of the graph. Calculate the
1 The graph of the reciprocal of current, against
I value of E. [4 marks]
resistance, R in Diagram 1 shows the results of an (ii) Calculate the internal resistance, r of the dry
experiment to determine the electromotive force, cell. [3 marks]
E and the internal resistance, r of a dry cell. (d) State one precaution to be taken in this
F experiment. [1 mark]
O 1 (A–1)
2 A heater coil is connected in series with a light bulb
2

R I
Graph 1 against R and a battery as shown in Diagram 2. The coil is fully
I
CHAPTER

M 2.0
immersed in a beaker of water. It is observed that
5 1.8 the brightness of the lamp progressively decreases
as the water is being heated.
1.6

1.4

1.2 heater
coil
beaker water
1.0

0.8

0.6
Diagram 2

0.4 Based on the above information.


(a) State one suitable inference. [1 mark]
0.2 (b) State one suitable hypothesis that could be
investigated. [1 mark]
0 R (Ω) (c) With the use of apparatus such as a voltmeter,
0.5 1.0 1.5 2.0 2.5
ammeter, fine iron wire and other apparatus,
Diagram 1 describe an experiment framework to investigate
the hypothesis stated in (b).
(a) (i) What is the effect on the current when the
In your description, clearly state the following:
value of R is increased? [1 mark]
(i) The aim of the experiment
(ii) Determine from the graph, the value of the
(ii) The variables in the experiment
current when R = 1.50 Ω. [2 marks]
(iii) The list of apparatus and materials
(b) The equation connected with this experiment is
(iv) The arrangement of the apparatus
E = I (R + r). From this equation, derive the
(v) The procedure used in the experiment.
1 R r Describe how to control and measure the
relationship: = + [1 mark]
I E E manipulated variables and how to measure
(c) (i) The electromotive force, E of the dry cell is the responding variables.
1 (vi) The way to tabulate the data
determined from the value of , where m
m (vii) The way to analyse the data [10 marks]

Electricity 422 COMPANION WEBSITE


Online Tests
FORM 5

3
CHAPTER

Electromagnetism

SPM Topical Analysis


Year 2007 2008 2009 2010 2011
Paper 1 2 3 1 2 3 1 2 3 1 2 3 1 2 3
Section A B C A B A B C A B A B C A B A B C A B A B C A B
Number of questions 5 1 – – – – 5 1 – – – 1 5 1 – – – 1 6 – – – – 1 3 1 1 – – –

ONCEPT MAP

ELECTROMAGNETISM

Magnetic effect of a The force on a Electromagnetic induction


current-carrying current-carrying
conductor conductor in a
magnetic field

Transformer Generation &


Electromagnet transmission
Factors affecting of electricity
the magnitude
of the force
Pattern and on a
direction of the current-carrying
magnetic field due conductor in a Direct Alternating
to a current in a Step-up
magnetic field transformer current current
• straight wire
• coil
• solenoid
Direct
current motor Step-down
Electricity
transformer
Factors affecting transmission
the strength of
the magnetic Factors affecting
field of an the speed of The importance
electromagnet rotation of an of renewable
electric motor Ideal transformer: energy
Vp Np
• =
Vs Ns
Applications of
electromagnets • V p Ip = V s Is

COMPANION WEBSITE
Learning Objectives 423
result of the presence of a magnet or a current-
3.1 Analysing the Magnetic carrying conductor.
Effect of a Current- 2 Figure 3.1 shows a method of tracing a
carrying Conductor magnetic field line with a magnetised needle.
The needle, which is supported on a float,
Ferromagnetic and Non-magnetic Materials moves along the path of a magnetic field line.
N pole
bar magnet
1 When a bar magnet is brought close to metals water
float

N
such as iron, nickel and cobalt, these materials

S
field line magnetised
are attracted to the magnet. needle
2 These, together with certain magnetic alloys
S pole
including steel, are called ferromagnetic
materials.
3 Substances like aluminium, copper, glass,
wood and brass are non-magnetic materials
because they are not attracted by a magnet. Figure 3.1 The ‘free’ N pole floats round from the N pole to
the S pole along the magnetic field line
Magnetic Fields 3 Magnetic field lines begin at the north pole
and end at the south pole.
F 1 A magnetic field is a region in which a 4 A magnetic field also exists around a current-
O magnetic material experiences a force as the carrying conductor.
3

R
CHAPTER

In 1820, Hans Christian Oersted, a Danish physicist, His discovery led to the inventions of the electric motor,
accidentally discovered the magnetic effect of a current- dynamo, telephone, loudspeakers and microphone
carrying conductor. He observed the deflection of a which are based on electromagnetism.
compass needle near a wire carrying an electric current.

Magnetic Effects of a Current-carrying Conductor

1 A length of wire, PQ which is straightened, is laid in a north-south direction as shown in Figure 3.2(a).
low-voltage
d.c. power
supply plotting Q thick copper wire Q Q
switch
– compass
+ above
the wire plotting compass X X
below the wire
P
Y Y
rheostat
P P
(a) (b) (c)
Figure 3.2
2 Before the passage of current in the circuit, the 4 When the direction of the current is reversed, it
north pole of the compass needle points is observed that the deflections of the two
north. compasses are also reversed as shown in Figure
3 When the power supply is switched on, current 3.2(c).
flows in the circuit. The north pole of the 5 The amount by which the needle deflects
compass above the wire is deflected to the east, increases when
whereas the compass below the wire is (a) the current in the wire is increased,
deflected to the west as shown in Figure 3.2(b). (b) the compass is brought nearer to the wire.

Electromagnetism 424
The Pattern and Direction of the Magnetic Field Due to a Current

The pattern of the magnetic field produced depends on the shape of the conductor used.

Activ To study the pattern and direction of the magnetic field due to a
ity 3.1 current in a (a) straight wire, (b) circular coil and (c) solenoid
Materials Iron filings SPM
’09/P1

Apparatus
A low-voltage d.c. supply, s.w.g. 18 PVC covered copper wire, thick copper coil, copper solenoid, 5 plotting
compasses, rheostat and a thick cardboard.
(A) Straight wire
Arrangement of apparatus
low-voltage –
d.c. supply +
H plotting
switch compass
thick cardboard
sprinkled with F
iron filings
O

3
R
Figure 3.3

CHAPTER
M
Procedure (B) Circular coil
1 A thick copper wire is passed through a hole, H Arrangement of apparatus 5

in the centre of a horizontal piece of thick thick copper coil


cardboard. The wire is straightened above and
below the hole, H. plotting
compass
2 The direction of the magnetic field can be
shown by placing 4 plotting compasses at equal
distances from the straight copper wire.
3 The power supply is switched on and the
rheostat is adjusted until a large current is

passed through the wire and the direction of the +
rheostat
magnetic field is noted. switch
4 A thin layer of iron filings is sprinkled onto the Figure 3.5 Current flowing in a coil
area on the cardboard surrounding the wire.
5 With the current on, the cardboard is gently tapped. Procedure
The pattern of the magnetic field is observed. 1 The straight wire in Activity (A) is replaced
6 The direction of the current is reversed by with a thick circular coil.
changing the power connection and the pattern 2 Four plotting composses are arranged on the
of the magnetic field and the needle of the cardboard as shown in Figure 3.5.
compasses are observed. 3 Steps 3–6 in Activity (A) are repeated.
Observation
The pattern of magnetic field: Observation
magnetic Pattern of magnetic field of a coil:
field
pattern
Activity 3.1

cardboard
sprinkled
with iron
filings

Figure 3.4 Figure 3.6

425 Electromagnetism
(C) Solenoid (long cylindrical coil of wire)
Arrangement of apparatus Observation
Pattern of the magnetic field of a solenoid:
cardboard sprinkled
with iron filings

switch
+ –

low-voltage rheostat
d.c. supply
Figure 3.8
Figure 3.7 Current-carrying solenoid Discussion
Procedure 1 Since high currents are used in the activities,
1 The coil in Activity (B) is replaced with a make sure that
solenoid which consists of several turns of (a) the switch is turned on only when everything
copper wire. is ready,
2 Five plotting compasses are arranged on the (b) the switch should be turned off immediately
F
cardboard as shown in Figure 3.7. after the observations to prevent overheating.
O
2 The north pole of the plotting compass indicates
3

R 3 Steps 3–6 in Activity (A) are repeated.


the direction of the magnetic field since the
CHAPTER

M
plotting compass aligns itself in the field.
5 3 The iron fillings become magnetised and align
themselves on the magnetic field lines, showing
the magnetic field pattern.

SPM
The Magnetic Field Due to a Current in a Straight Wire ’09/P1

magnetic magnetic
field line field pattern

(a) (b)
Figure 3.9

1 Figure 3.9 shows the pattern of the magnetic field produced by a


straight current-carrying conductor.
2 The iron filings align themselves and form concentric circles with the
wire as the centre as shown in Figure 3.9(b).
Activity 3.1

3 Figure 3.10(a) shows that the magnetic field is in an anticlockwise


direction. When the current is reversed, the needles of the compasses
also reverse and point in the opposite direction [Figure 3.10(b)].
The magnetic fields change direction when the direction of the current
is reversed.

Electromagnetism 426
current in upward current in downward Note
direction (out of page) direction (into page)
(a) The direction of the
current is represented
wire wire by a point  or a cross
⊗.
(b) A point  represents
(a) Current in upward direction (b) Current in downward direction
current flow in an
Figure 3.10
upward direction out
4 The characteristics of the magnetic field produced by a current flowing
of the paper. This can
in a straight wire:
be likened to looking
(a) The magnetic field lines form a pattern of concentric circles
at the tip of an arrow
around the wire.
head [Figure 3.10(a)].
(b) The magnetic field strength increases when the current is increased.
(c) A cross ⊗ represents
(c) The magnetic field strength is stronger near the current and weaker
current flow in a
further away (Figure 3.11).
downward direction
into the paper, like
I looking at the tail end
Field lines Stronger
are closer magnetic of an arrow [Figure
field strength
3.10(b)]. F
O
Weaker

3
Field lines R
magnetic
are further

CHAPTER
field strength M

5
Figure 3.11
(d) When the direction of the current is reversed, the direction of the
magnetic field reverses too.
(e) The direction of the magnetic field around a current-carrying
conductor depends on the direction of current flow but the
pattern of the magnetic field remains unchanged.
5 There are two methods to determine the direction of the magnetic
field around a wire, if the direction of the current is known.

To determine the direction of the magnetic field

Maxwell’s Screw Rule Right-hand Grip Rule


Imagine a screw is being turned so that it bores its Imagine your right hand gripping a wire carrying a
way along the wire in the same direction as the current so that the thumb points in the direction of
current. The direction of the rotation of the screw the current. Your fingers give the direction of the
gives the direction of the magnetic field. magnetic field.
screw driver

direction of
current flow

field direction

current direction
(conventional)
direction of
magnetic
field

Figure 3.12 Maxwell’s screw rule Figure 3.13 Right-hand grip rule

427 Electromagnetism
The Magnetic Field Due to a Current in a Coil

Figure 3.14(a) shows the pattern of the magnetic field produced by a current-carrying coil, viewed from
above while Figure 3.14(b) shows the magnetic field pattern when the direction of the current is reversed.

2 1
The field lines around the two wires The magnetic field line at the centre of
are in opposite directions. the coil is in the form of a straight line.

x x

(a) (b)
3
F The lines in the coil 4
O The right-hand grip rule can also be used to both sides of
(between the two wires)
3

R the coil to determine the direction of the magnetic field.


are more closely spaced
CHAPTER

M
than the lines outside.
5
Figure 3.14

The magnetic field strength increases when


(a) the current is increased,
(b) the number of turns of the coil is increased.

The Magnetic Field due to a Current in a Solenoid

1 Figure 3.15 shows the pattern of the magnetic field produced by a current-carrying solenoid, viewed
from the top.
4
The direction of field lines outside the
solenoid is opposite to the direction 1
A straight magnetic field line
of the field lines in the solenoid.
along the axis of the solenoid.

3 2
The field lines are uneven and more The magnetic field lines are close and evenly spaced
widely spaced outside the solenoid. in the solenoid showing a strong uniform field.

Figure 3.15

Electromagnetism 428
2 The strength of the magnetic field can be increased by
(a) increasing the current,
(b) increasing the number of turns of the coil.
3 The pattern of the field lines outside the solenoid is similar to the
magnetic field of a bar magnet. One end of the solenoid is a North Pole
while the other end is South Pole.
4 The polarity at the ends of the solenoid can be determined by:

(a) the right-hand grip rule (for a solenoid) (b) looking at the direction of the current from each end of
the solenoid
SPM
The thumb points ’07/P1
to the N pole.

anticlockwise

N S

Figure 3.17 Poles of a solenoid


Fingers curl in the
direction of current. (i) If the direction of the current is clockwise, the
viewed end of the solenoid is the south pole (S). F
Figure 3.16 Right-hand grip rule for a solenoid
(ii) If the direction of the current is anticlockwise, the O

3
Imagine gripping the current-carrying viewed end is the north pole (N). R

CHAPTER
M
solenoid with your right hand so that your
fingers curl round the solenoid in the direction 5
of the current. Your thumb will then point
towards the north pole (N pole) of the
solenoid.

The rules are also true for a single coil. One side of the coil
acts like a north pole while the other sides a south pole.

1
Name the poles P, Q, R and S in Figure 3.18. Please don’t get yourself confused over the two
right-hand grip rules.

Gripping the wire Gripping the solenoid


to determine the to determine the
direction of north pole (N)
magnetic field
(a) (b) – current
Figure 3.18
N S
Solution
Use the right-hand grip rule for a solenoid or find the
direction of current flow by viewing both ends of the
solenoid to determine the poles.
+
End P Q R S
Pole North South North South

429 Electromagnetism
Electromagnet

1 A simple electromagnet consists of a coil (solenoid) of insulated


copper wire wound on a core of soft iron. solenoid
paper
clips
attracted
∴ Coil + Soft iron core = Electromagnet to the
core
soft iron
2 An electromagnet is a temporary magnet; that is, it acts as a magnet core
magnetised
when the current is switched on and ceases to be a magnet when the
current is switched off. (a) When switch is closed
3 Soft iron is used because it has the following characteristics:
(a) Easily magnetised when the current in the solenoid is switched on
[Figure 3.19(a)].
(b) Loses all its magnetism when the current is switched off [Figure
3.19(b)].
4 If the soft iron core is replaced with a steel core, the steel core becomes the core
a permanent magnet when current flows in the solenoid. loses all it
magnetism
5 When the current is switched off, the steel core remains magnetised.
F Therefore, steel is not suitable as the core of electromagnets. paper clips fall
O 6 The strength of an electromagnet increases when into the beaker
3

R (a) the current is increased,


(b) When switch is open
CHAPTER

M (b) the number of turns of wire in the solenoid is increased.


Figure 3.19 Electromagnet
5

Factors Affecting the Strength of the Magnetic Field of an Electromagnet SPM


’05/P1
SPM
’08/P1

3.1
Factors that affect the strength of an electromagnet
Situation
A lifting machine for scrapt metal is basically an electromagnet which is made of solenoid and soft
iron core that functions when current is on. How would the strength of a lifting machine depend on?
Case (I) SPM Case (II)
’09/P2/(B)
Current Number of turns in solenoid
Hypothesis The strength of an electromagnet The strength of an electromagnet
increases when current increases. increases when the number of turns on
the solenoid increases.
Variables:
(a) Manipulated variable Current Number of turns of wire on the solenoid
(b) Responding variable The strength of the electromagnet The strength of the electromagnet
(represented by the number of paper (represented by the number of paper
clips attracted) clips attracted)
Experiment 3.1

(c) Fixed variables Number of turns of wire in the Value of current, material and shape of
solenoid, material and shape of the core the core
Aim: To investigate the relationship between To investigate the relationship between
current and the strength of magnetic the number of turns on solenoid and the
field of an electromagnet strength of magnetic field of an
electromagnet

Electromagnetism 430
Apparatus/Materials 2 It is found that the number of paper clips attached
to the end of the iron rod increases when the
Long iron rod (or nail), long cardboard tube (same
current is increased.
diameter as the iron rod), insulated (PVC) copper
wire, low-voltage high-current d.c. supply, ammeter
(0 – 5 A), rheostat (0 – 15 Ω), retort stand, wooden Conclusion
clamp, paper clips and beaker. The magnetic field strength of an electromagnet
increases when the current is increased.
Arrangement of apparatus The hypothesis is valid.

iron rod (or nail)


wooden clamp
switch (II) Relationship between the strength of an
electromagnet and the number of turns of
A ammeter
retort wire on the solenoid
stand
rheostat Procedure
insulated PVC
copper wire 1 The apparatus is set up as shown in Figure 3.20.
paper clips 2 The iron rod (or nail) is wound with 10 turns of
Figure 3.20 insulated copper wire to form an electromagnet.
3 The switch is closed and the rheostat is adjusted
so that the current, I = 2.0 A. F
(I) Relationship between the strength of an 4 The beaker containing paper clips is brought O

3
electromagnet and current close to the end of the iron rod. R

CHAPTER
M
Procedure 5 The beaker is withdrawn and the number of paper
1 An iron rod (or nail) is wound with 20 turns clips attached is counted. 5
of insulated copper wire to form an 6 The experiment is repeated with increasing
electromagnet. numbers of turns of wire, n in the solenoid. The
2 The iron rod is clamped onto the retort stand with number of paper clips attached to the iron rod,
a wooden clamp. The apparatus is set up as shown N is recorded.
in Figure 3.20.
3 The switch is closed and the rheostat is adjusted Results
so that the current, I = 0.5 A. 1 Table 3.2 shows the results obtained.
4 The beaker containing paper clips is brought
Table 3.2
close to the end of the iron rod.
5 The beaker is withdrawn and the number of paper Number of turns, Number of paper clips
clips attached is counted. n attached, N
6 The experiment is repeated with higher values of 10 N1
current, I in the solenoid. The number of paper
clips attached to the iron rod, N is recorded. 20 N2
30 N3
Tabulation of data 40 N4
1 Table 3.1 shows the results obtained. 50 N5
Table 3.1
Note: N1 < N2 < N3 < N4 < N5
Number of paper clips
Current, I (A)
attached, N 2 It is found that the number of paper clips attached
0.5 N1 to the end of the iron rod increases when the
number of turns of wire in the solenoid is
N2
Experiment 3.1

1.0 increased.
1.5 N3
2.0 N4 Conclusion
2.5 N5 The magnetic field strength of an electromagnet
increases as the number of turns of wire is increased.
Note: N1 < N2 < N3 < N4 < N5 The hypothesis is valid.

431 Electromagnetism
(III) The use of a soft iron core 1 Table 3.3 shows the results obtained.
Procedure 2 It is found that the number of paper clips attached
to the end of the solenoid increases when the soft
1 The apparatus is set up as shown in Figure 3.20.
iron is used.
2 The experiment is repeated with the iron rod
replaced with a cardboard tube (same diameter as
the iron rod).
3 The number of paper clips attached to the Explanation
solenoid is recorded. A magnetic field is produced when a current is passed
Results through the solenoid. This magnetises the soft iron
core which produces a magnetic field much stronger
Table 3.3 than that of the solenoid alone.
Number of
Situation paper clips
attached, N Conclusion
Solenoid with no soft iron core N1 An electromagnet needs a soft iron core.
Solenoid with a soft iron core N2 The use of a soft iron core greatly increases the
magnetic field strength of an electromagnet.
Note: N1 < N2
F
O
3

R
CHAPTER

5
U-shaped number of turns of wire. When the same current
iron core flows in their solenoids, it is observed that the
I
S U-shaped electromagnet attracts more paper clips.
straight
iron I The U-shaped core has two opposite poles adjacent
core to each other. As a result, the magnetic field lines are
concentrated in the space between the two poles
and hence the field strength is increased.

2 The magnetic field strength of an electromagnet


N N S increases when the number of turns of wire or the
current is increased, but this is true only up to a
1 The shape of the iron core also affects the magnetic certain value. The strength of an electromagnet
field strength of an electromagnet. This can be shown reaches a maximum value for a given number of
by comparing a straight iron core with a U-shaped turns of wire or value of current. At the maximum
iron core, both of which are wound with the same value, the iron core is said to be saturated.

N N 1 Figure (a) shows two wires carrying the same current


P Q M N in opposite directions. The magnetic field produced by
the wire P is cancelled by the magnetic field produced
by the wire Q. The net field produced is zero. The
needles of all the compasses point to the north.
Experiment 3.1

2 Figure (b) shows two wires M and N carrying the same


current in the same direction. The magnetic field
produced by the wire M overlaps the magnetic field
(a) (b) produced by the wire N. The strength of the resultant
magnetic field produced is doubled and the pattern is
the same as would be produced by one wire.

Electromagnetism 432
Factors that affect the magnetic field strength of an electromagnet

The strength of a magnetic field increases when…

(a) Current, I increases (b) Number of turns of wire increases


When the current is increased, the When a current flows in a solenoid, each turn of
strength of the magnetic field of every turn wire produces its own magnetic field. As the
of wire increases thus increasing the number of turns is increased, the magnetic fields
strength of the resultant magnetic field. produced by each turn overlap to produce a
resultant field which is much stronger.

2
A length of copper wire is wound round a Q 1 Advantages of using a soft
U-shaped iron core and connected to a battery iron core in electromagnets:
as shown in Figure 3.21. • A soft iron core is able F
Name the magnetic poles produced at positions to concentrate O

3
R
P, Q and R when the current is switched on. magnetic field lines

CHAPTER
P
through it, and thus a M
R stronger magnetic
5
Figure 3.21 field is produced.

Solution
soft iron core
Applying the right-hand grip rule to the left hand side of the solenoid, it is found
that P is a south pole (the direction of current flow is clockwise), whereas Q is a • A soft iron core can
also be magnetised
north pole. Applying the right-hand grip rule to the right hand side of the
by the magnetic field
solenoid, it is found that R is a south pole and Q is a north pole. Therefore, P and
of the solenoid. The
R are south poles and Q is a north pole. resultant magnetic
field (by the coil and
the core) produced is
much stronger.
2 For a U-shaped core, the
two solenoids are
oppositely wound on the
two sides of the core to
The two ends of the soft core in an electromagnet must be of opposite poles. produce opposite poles
on the two ends. As a
result, the magnetic field
The two ends of a soft core in an electromagnet need not be of opposite poles. lines are concentrated in
It depends on how the wire is wound around the core. the space between the
For example Y two poles and hence the
Z field strength is increased.
X

By applying the right-hand grip rule on the left hand side, X is a north pole
while Y is a south pole.
By applying the same rule on the right hand side, Y is a south pole while Z a
N S
north pole. Therefore, X and Z are north poles while Y is a south pole.

433 Electromagnetism
Uses of Electromagnets

Electromagnetic lifting machine

(a) • An electromagnetic lifting machine is constructed


thick cable from a circular soft iron core with a solenoid of
to carry large current thick insulated copper wire.
• Such a construction results in opposite poles at the
centre and sides which produce a magnetic field of
soft iron core
magnetic great strength. When the current is switched on, the
field lines soft iron is magnetised and a large lifting force is
S N S
produced.
(b)
• It lifts up iron and steel, and separate them from
other non-magnetic materials.
• When the current is switched off, the heavy objects
fall off.
Figure 3.22 Electromagnetic lifting machine

F Telephone earpiece Circuit breaker


O
3

R
• A circuit breaker is an automatic
CHAPTER

M 2
An earpiece consists of a A thin magnetic alloy switch designed to disconnect the
permanent bar magnet placed diaphragm is power supply when the current
5
between the two soft iron pole attracted to the pole
pieces with soleoids wound in pieces by the bar
flowing through the circuit is too
opposite directions. magnet. high.
• It acts as a safety device like a fuse.
However, unlike a fuse which has to be replaced
permanent
magnet after it operates once, the circuit breaker can be reset
N (turned on again, either automatically or manually)
to resume normal operation.
current to the
springy magnetic
earpiece live
alloy diaphragm live
wire spring B wire spring B
S reset button reset button
push
contacts contacts to reset
soft iron spring A are spring A
pole piece solenoid soft iron separated soft iron
armature armature
3 & 4 electromagnet
electromagnet
• When a caller at the other end of the line talks into the
microphone, the earpiece receives an alternating current from (a) Normal current flow (b) After excessive current
the telephone line. flow
• When the alternating current flows in the electromagnet, it Figure 3.24
produces its own magnetic field which will be added or
subtracted from the magnetic field of the bar magnet,
1 When the live wire carries the usual normal
according to the direction of the current. operating current, the force of the electromagnet is
not strong enough to separate the contacts.
position of 5 2 However, when the current gets too high, the
diaphragm The diaphragm thus
position of increased magnetic force becomes strong enough
when pulled vibrates backwards and
diaphragm
when pulled with smaller forwards according to to pull over the soft iron armature and release the
with stronger force the frequency of the catch.
force alternating current. 3 The contacts are separated and the current is
The movement of the
position of diaphragm backwards and
stopped. Meanwhile, the spring B keeps the
diaphragm forwards produces the contacts apart. Spring A pulls the armature back to
when current sound waves in the air. its original position.
is zero
4 The system can be reset when the reset button is
Figure 3.23 Construction of an earpiece pressed and the contacts close again.

Electromagnetism 434
Electromagnetic relay

• A magnetic relay acts as a switch using a small • It can guard safety of the user, since the input circuit
current to turn on a current in a much more can work on a low-voltage supply to control an
powerful circuit. electrical machinery drawing a high current or
using a high voltage.

insulating block spring metal


strips
power supply contacts C
(normally open) The iron core becomes magnetised and
electric one end of the soft iron armature is
motor soft iron pulled towards it. This action causes the
armature other end of the soft iron armature to
push and close the contacts C.
output
circuit electromagnet

switch When the switch in the input circuit is closed, a


input circuit small current flows through the electromagnet.
The output circuit is closed
and a larger current flows
When the switch is open, no current flows in the
through the motor. Thus, the F
input circuit and the iron core loses its
electric motor is turned on.
magnetism. The soft iron armature returns to its O

3
original position and the output circuit is open. R

CHAPTER
M

Figure 3.25 Electromagnetic relay 5

Electric bell

1 On/Off Switch
When the switch is
pressed, the
circuit is closed. battery 7
steel plate spring Steel plate
spring
battery
Pulls back the
iron armature to
3 Soft iron armature its original
Attracted to the iron position to close
core of the the circuit and
electromagnet. the process
electromagnet of producing the
ringing of the
bell continues.
electromagnet
contact adjusting screw

attraction contact adjusting screw


contacts
2 Iron core
Current flows 4 Hammer 6 Iron core
through the The iron core
Moves with the 5 Contacts
solenoid. loses its The movement of the
soft iron armature
The iron core magnetism armature separates
gong to strike the gong,
is magnetised. since no the contacts and
thus ringing
current flows. gong breaks the circuit.
the bell.

(a) (b)

Figure 3.26 Electric bell

435 Electromagnetism
3 1 SPM
Clone
’08
Figure 3.27 shows the end view of a solenoid. Two soft
The diagram shows a simple electromagnet.
iron rods are placed in the solenoid as shown in Figure
3.27 (a). When current flows in the solenoid, the two
rods move to the positions as shown in Figure 3.27 (b).
solenoid

soft iron rods


wire

(a) (b)
Figure 3.27 Which of the following changes will enable the
electromagnet to attract more paper clips?
(a) Explain the observation in Figure 3.27 (b). A Reduce the number of coils
(b) What is the effect of reversing the current flow in B Increase the magnitude of the current
the solenoid? C Use a thinner wire to form the coils
F (c) What happens when the current in the solenoid is
O D Use a non-insulated wire to form the coils
cut off?
3

R
(d) What will be observed when the iron rods are Comments
CHAPTER

M
replaced with steel rods and the current is cut off? The strength of an electromagnet increases when the
5 current is increased.
Solution
Answer B
(a) The two iron rods are magnetised to identical
magnets which repel each other.
(b) The effect remains the same. The positions of the
rods remain unchanged. 2 SPM
Clone
’11
(c) The two rods will return to their original positions.
(d) The two steel rods stay permanently in the Which of the following diagrams shows the pattern
positions as shown in Figure 3.27 (b) although of magnetic field around a straight current carrying
the current is cut off because the steel rods can conductor?
retain the magnetism which was induced in them A C
when the current flowed.

The principle of electromagnetism is also applied in the


operation of the Magnetic Levitated Train (MAGLEV B D
Train). The MAGLEV train is a floating train which does
not require wheels to move. This type of train is
commonly used in Germany, USA, Japan and recently
in Shanghai, China. Shanghai Magnetic Levitation is the
first commercial high-speed maglev line in the world.
Because the train floats, the frictional forces on the track
can be neglected. It travels at a high speed of 500 km h–1 Comments
(or 139 m s–1) The magnetic field around a current carrying
comparable to that of
conductor is concentric filed lines, not spiral lines.
commercial aeroplanes.
It is also energy efficient
Right-hand grip rule can be applied to determine
and environmentally the direction of the magnetic field.
friendly. Answer A

Electromagnetism 436
3.1

1 The diagram shows the top view of a compass placed Complete the table by filling in the poles formed at
between two solenoids, X and Y. The number of turns positions X, Y and Z of the U-shaped magnet.
of wire in the solenoid X is twice the number in the
solenoid Y. Before the current is switched on, the Position Pole
compass needle points north. X
N Y
Z

soft iron core 4 The diagram shows the circuit of a simple


electromagnet.

In which direction will the compass needle deflect to


when the current is switched on?
A East C North
B West D South
2 The diagram below shows three compasses P, Q and
R, placed beside a current-carrying wire. Before the
F
current is switched on, the needles of the three
O
compasses point north. State three ways of increasing the magnetic field

3
R
strength of the electromagnet.

CHAPTER
N M
5 The diagram below shows two lengths of wire P and
Q, placed parallel to each other 6 cm apart. 5
compass
A compass is placed between the two wires.

N
top view
wire
Draw a diagram to show the orientations of the
compass needles when the current is switched on.
3 The diagram shows a U-shaped electromagnet.

(a) (b)
State your observation of the compass needle when
currents of the same magnitude flow in P and Q
(a) in opposite directions, as shown in diagram (a),
(b) in the same direction, as shown in diagram (b).

3.2 Understanding the Force on a Current-carrying Conductor in a


Magnetic Field
Why does the pointer of an ammeter deflect, a
motor rotate and the diaphragm of a loudspeaker
vibrate when a current flows in them?
These motions are caused by the force produced
when a current flows through a conductor in a
magnetic field. How does this take place?

Figure 3.28

437 Electromagnetism
Activ To investigate the interaction between a current-carrying
ity 3.2 conductor in a permanent magnetic field

Apparatus/Materials Procedure
Low-voltage d.c. power supply, rheostat, switch, 1 The apparatus is set up as shown in Figure 3.29.
crocodile clips, two Magnadur magnets, U-shaped 2 The Magnadur magnets are arranged with
iron yoke, and thick, stiff copper wire. unlike poles facing each other. The stripped
Arrangement of apparatus thick copper wire, PQ is placed between the two
magnets.
3 The power supply is switched on. The
low- movement of the copper wire, PQ is observed.
voltage
d.c. 4 The poles of the Magnadur magnets are
power reversed and the effect on the copper wire is
supply
observed.
5 The above steps are repeated with the terminals
of the power supply reversed so that the current
flows in the opposite direction. The effect on the
copper wire, PQ is observed.
F
O
S 6 The above steps are repeated with both the
position of the two Magnadur magnets and the
direction of the current reversed (compared to
3

N
R
the original situation). The effect on the copper
CHAPTER

M
wire, PQ is observed.
5

Observations
Figure 3.29 Investigating the interaction between a Table 3.4 shows the effects observed.
current-carrying conductor and a magnetic
field
Table 3.4

(a) (b) (c) (d)

S Q N Q S Q N Q

N S N S
P P P P

When the switch is When the poles of the When only the When the terminals of the
closed, the copper wire, Magnadur magnets are terminals of the power supply and the
PQ swings outwards. reversed, the copper power supply are poles of the Magnadur
wire swings in the reversed, the copper magnets are reversed, the
opposite direction—i.e. wire, PQ swings copper wire, PQ swings
inwards. inwards. outwards as in situation (a).
Activity 3.2

Conclusion wire, PQ moves in the opposite direction. When the


A force is produced when a current-carrying directions of both the magnetic field and the current
conductor is in a magnetic field. When the magnetic are reversed, the wire, PQ moves in the original
field or the direction of the current is reversed, the direction.

Electromagnetism 438
upward
Catapult Field (Catapult Effect) SPM
’04/P1 force

1 When a current-carrying conductor is placed in the magnetic field of a


permanent magnet as shown in Figure 3.30, the interaction between N
the two magnetic fields produce a force on the conductor.
2 The two magnetic fields involved are: S
wire
(a) magnetic field of the permanent magnet, and
(b) magnetic field produced by the current-carrying conductor. –
3 The two magnetic fields interact to produce a resultant field known as a battery
+
catapult field as shown in Table 3.5.
Figure 3.30

Table 3.5

Magnetic field of Magnetic field of Resultant field of two


permanent magnet current-carrying conductor interacting fields
(a) (b) (c)

N S N S F
O

3
current direction R

CHAPTER
M
Uniform magnetic field lines Magnetic field lines in the form of A catapult field is produced.
from the north pole to the concentric circles, in an The distortion of the magnetic field 5
south pole. anticlockwise direction based on lines causes the wire to move from the
the right-hand grip rule. stronger magnetic field to the weaker
field.

Fleming’s Left-hand Rule (Motor Rule) SPM


’07/P1
SPM
’08/P1

1 The method of determining the direction of the force by drawing a


catapult field is complicated.
2 The relationship between the direction of the current, magnetic field To determine the direction of
and the force acting on the conductor can be easily determined with the force or motion correctly:
Fleming’s left-hand rule or motor rule.
1 Point the first finger of
3 The direction of the force on the conductor is perpendicular to the your left hand in the
direction of the current and the direction of the magnetic field. direction of the
upward magnetic field.
thuMb force
(force/Motion) Hold it with your right
hand.
N
2 Point the second finger
First finger S of your left hand in the
(Field) wire direction of the current
seCond finger –
magnetic while you still hold your
(Current) + field lines first finger firmly without
battery
changing its direction.
3 The thumb will point in
If the first finger, second finger, and the thumb of the left hand are held the direction of the
at right angles to each other, with the First finger in the direction of the force/motion.
Field (B), the seCond finger in the direction of the Current (I), then the
thuMb will point in the direction of the force (F) or Motion.

Figure 3.31 Fleming’s left-hand rule

439 Electromagnetism
4 5
Figure 3.32 shows a current-carrying conductor Figure 3.33 shows the movement of a current-
placed in a magnetic field in different orientations. carrying conductor to the left when placed between
two bar magnets.

N S Y

N
X

(a) (b)
Figure 3.32
Figure 3.33
Determine
(a) the direction of the force in Figure 3.32 (a),
Name the poles of the magnet.
(b) the direction of the current in Figure 3.32 (b).
F
Solution
O Solution
3

R Using Fleming’s left-hand rule, it is found that


Using Fleming’s left-hand rule, we find that
CHAPTER

M the direction of the magnetic field is upwards.


(a) the force acts upwards in Figure 3.32 (a).
Therefore, X is the north pole and Y the south pole.
5 (b) the current flows from P to Q in Figure 3.32 (b).

I = current, B = magnetic field Conclusion


(a) To reverse the direction of the
when the direction force or motion, either the
of the magnetic field direction of the current or the
is reversed direction of the magnetic field
needs to be reversed.
(b) If both the directions of the
the force (motion), F the force (motion), F current and the magnetic field
acts upwards acts downwards are reversed, the direction of
the force or motion remains in
when the directions of both its original direction.
when the
direction of the the current and magnetic
current is field are reversed
reversed

the force (motion), F the force (motion), F


acts downwards acts upwards

Electromagnetism 440
Angle θ between the directions of the current (I) and magnetic field (B)
The magnetic field need not necessarily be perpendicular to the direction of the current.
The following table shows three situations with different angles of θ.

(a) θ = 90° (b) θ < 90° (c) θ = 0°

F1 F2 < F1
θ θ

• F1 is maximum. • F2 < F1. However the direction of the force is still perpendicular • No force is produced.
to the current as well as the magnetic field.

Force between Two Current-carrying Conductors

1 When two current-carrying conductors are placed 5 Figure 3.35 (a) shows that when two current- F
O
close to each other, they attract or repel each other. carrying conductors, with currents flowing in

3
R
2 This is because each current-carrying conductor the same direction, are brought close to each

CHAPTER
M
is in the magnetic field of the other current- other, a weak magnetic field is produced in the
carrying conductor. region between the conductors. This is because 5
3 Whether the two conductors shown in Figure magnetic field lines from the two conductors
3.34 attract or repel each other can be are in opposite directions in this region. The
determined with Fleming’s left-hand rule. two conductors attract and approach each
other.
6 Figure 3.35 (b) shows two current-carrying
conductors, with currents flowing in opposite
directions, produce a stronger magnetic field
in the region between the conductors when
A B P Q they are brought close together. This is because
the magnetic field lines from the two
conductors are in the same direction. The two
A B F F
F F P Q
conductors repel each other.

Summary
View from above
Figure 3.34 • Two current-carrying conductors with currents
flowing in the same direction attract each other.
4 The resultant magnetic field produced when
• Two current-carrying conductors with currents
two current-carrying conductors are brought
flowing in opposite directions repel each other.
close to each other is shown in Figure 3.35.
(You may think of it as opposite to magnetic
poles where like poles repel while unlike poles
attract.)
X

Reminder
neutral point
Current-carrying conductors move from a region
(a) (b)
with a strong magnetic field to a region with a
Key: � Current up ⊗ Current down
weaker field.
Figure 3.35

441 Electromagnetism
Factors Affecting the Magnitude of the Force on a Current-carrying Conductor
in a Magnetic Field

3.2

Factors that affect the magnitude of the force on a current-carrying conductor in a


magnetic field

Aim
To investigate the factors that affect the magnitude of the force between a current-carrying conductor
and a magnetic field

Case (II)
Case (III)
Case (I) Magnitude of the
Length of the
Strength of the current
current-carrying
magnetic field SPM
’08/P3(B) conductor

F Hypothesis The greater the strength The greater the current, The longer the length
O of the magnetic field, the greater the force on of the conductor in
3

R the greater the force on the conductor. the magnetic field, the
CHAPTER

M the conductor. greater the force on


the conductor.
5
Variables:
(a) Manipulated Strength of magnetic Current Length of current-
variable field carrying conductor
(b) Responding Force on the conductor Force on the conductor Force on the conductor
variable (represented by its (represented by its (represented by its
displacement) displacement) displacement)
(c) Fixed variables Current and length of Strength of magnetic Strength of magnetic
current-carrying field and length of field and current
conductor current-carrying
conductor

Apparatus/Materials
Four Magnadur magnets, U-shaped iron yoke, three lengths of s.w.g. 18 stripped copper wire (or thin brass rod),
low-voltage d.c. supply, rheostat, block of wood of dimensions 6 cm × 6 cm × 5 cm and adhesive tape.

Arrangement of apparatus

Magnadur
magnet thick

B
S copper
wire
low-voltage
d.c. supply

Experiment 3.2

N +

rheostat
iron yoke wooden
block adhesive tape

Figure 3.36

Electromagnetism 442
(I) Relationship between the strength of a (II) Relationship between the current in the wire
magnetic field and the force acting on the wire and the force acting on the wire
Procedure Procedure
1 Two pieces of stiff bare copper wire are placed 1 Steps 1 – 5 of experiment (I) are repeated.
4 cm apart on a wooden block and secured in 2 The value of the current is increased by adjusting
position with adhesive tape. the rheostat. The motion of the 3 cm wire is
2 A 6 cm piece of stripped copper wire is crimped observed.
at both ends and placed on the two long pieces of Observation
copper wire. The 6 cm wire (the sliding The 6 cm wire is thrown out of the magnetic field at a
conductor) is free to move on the track formed by faster speed and a greater displacement.
the two long copper wires.
Conclusion
3 The apparatus is set up as shown in Figure 3.36.
The magnitude of the force acting on a current-
4 Two Magnadur magnets are placed on the carrying conductor in a magnetic field increases as
U-shaped iron yoke with unlike poles facing each the magnitude of the current increases.
other. The hypothesis is valid.
5 The current is switched on and the displacement
of the sliding conductor is measured. (III) Relationship between the length of the
6 Two additional Magnadur magnets are placed on current-carrying conductor and the
the iron yoke to increase the strength of the magnitude of the force acting on it F
O
magnetic field. Step 5 is repeated. Procedure

3
R

CHAPTER
1 Steps 1 – 5 of experiment (I) are repeated. M
2 The experiment is repeated with the distance
Observation 5
between the two long wires reduced to 2 cm. Note
When the number of Magnadur magnets is increased that length of wire carrying current is shorter, i.e.
(the strength of the magnetic field increases), the 2 cm.
sliding conductor is thrown out of the magnetic field
Observation
at a faster speed, and a greater displacement.
The 5 cm wire moves at a slower speed and a smaller
displacement.
Conclusion Conclusion
The magnitude of the force acting on a current- The force acting on a current-carrying conductor
carrying conductor in a magnetic field increases as increases as its length increases, provided the whole
the strength of the magnetic field increases. length of the conductor is within the magnetic field.
The hypothesis is valid. The hypothesis is valid.

If Experiment 3.2 is repeated with the pair of Magnadur magnets being rotated so that –
the angle θ between the current, I and the magnetic field, B is decreased from 90° +
S
(magnetic field perpendicular to conductor) to 45°, the short wire is thrown out with
Experiment 3.2

slower speed and shorter displacement.


When the angle is further reduced to 0° (magnetic field parallel to the conductor) as N
shown in Figure 3.37, the conductor does not move.
The force on a current carrying conductor decreases when the angle θ between the
magnetic field and the conductor decreases from 90° (perpendicular to each other) to 0°
(parallel to each other). Figure 3.37

443 Electromagnetism
6
Figure 3.38 shows a copper rod hung from a spring (a) the resistance in the rheostat is decreased?
balance. The ends of the rod are connected to a battery. (b) the copper rod is turned 180° about the vertical
axis through its midpoint?
Solution
spring
balance
(a) Using Fleming’s left-hand rule, it can be seen that
the direction of the force acting on the rod is
N S upwards. When the resistance in the rheostat is
reduced, the current in the copper rod increases.
The upward force acting on the rod increases.
The reading on the spring decreases.
(b) When the rod is rotated 180° about the
perpendicular axis at its midpoint, the direction of
Figure 3.38 the current is reversed. The direction of the force
What is the effect on the scale reading of the spring acting on it now is downwards. The reading on the
balance when spring balance increases.

F
O Turning Effect of a Current-carrying Coil in a Magnetic Field SPM
’03/P1
3

R
CHAPTER

M
1
1 The current in the coil produces a magnetic field around it.
5

I I
2

Two sides of the coil produce circular magnetic field lines in
opposite directions.
N S

3
(a) The interaction between the magnetic field of the current and the
magnetic field of the permanent magnet produces a resultant
2 magnetic field as shown in Figure 3.39(c).

4
The catapult field produced pushes the side AB in an upward
N S direction and the side CD downwards.

5
(b) The two forces which are in opposite directions constitute a
couple which produces a turning effect and the coil rotates in a
3 & 4 clockwise direction.

6
The direction of the forces acting on the sides AB and CD can also
be determined with Fleming’s left-hand rule.
N S

7
When the direction of the current is reversed, the direction of
(c) rotation also reverses.
Figure 3.39 A current-carrying coil in the

8
magnetic field of a permanent
Two types of electrical equipment whose operation is based on
magnet the rotational effects on a coil in a magnetic field are moving coil
meters and direct current (d.c.) motors.

Electromagnetism 444
A couple exists when two equal and opposite parallel, but not collinear forces act on a body as in the case of the
current–carrying coil in a magnetic field. The moment (turning effect) of a couple is the product of either force and the
perpendicular distance between the line of action of the two forces.

The turning effect and the position of the coil:

Coil in horizontal position or in line Coil in slanting position with the Coil in vertical position or at right
with the field field angles to the field

field direction
field
N S direction
N S
N S

F
O

3
The turning effect is the maximum The turning effect is less because There is no turning effect because R

CHAPTER
as the two forces are furthest apart. the distance between the lines of no couple exists in this position. M
The instantaneous speed of the action of the two forces is shorter.
sides of the coil at this position is However, the forces remain constant 5
also the fastest. since the field is uniform.

Direct Current Motor

1 A direct current motor is used to convert electrical energy into kinetic energy.

A rectangular coil of
coil
wire placed between two
permanent magnets.

N S
Both ends of the Q P
spring
Two carbon brushes, P and
coil are soldered Q are held against the
to commutators commutator Y X
carbon
brush
commutators, X and Y
X and Y made of with a slight pressure with
two semicircular Key: M = motion the aid of springs.
copper rings. variable battery switch
rheostat
Figure 3.40 A simple direct current motor

2 The use of a commutator is to enable a smooth change of direction of the current flow in the
coil so that the coil continuously rotates in one direction every half rotation. In other words, the
commutator is an automatic switch, which reverses the current in the coil twice in one
complete revolution.

445 Electromagnetism
Principle of Operation of a Simple d.c. Motor

Stage 1: First half rotation


S
se leming s left hand
rule, the force on the side Horizontal position
CD is upwards.
orce

The two forces produce a


couple which rotate the
coil in a clockwise
ield
direction.

Current
Cur
urrent
Current

force rotation
C B

D
A
The current flows
force
from brush P to the
FF coil and out of the Q P Current
OO coil via the brush Q. ield
33

Y X
RR
CHAPTER

M
CHAPTER

55 The carbon brushes P When the switch is orce


& Q are in contact with closed, current flows se leming s left hand rule,
the commutators X and Y in the direction the force on the side AB is
respectively. ABCD. downwards.

F
O Stage 2
St
3

R
Vertical position
CHAPTER

D
B

Q Y
A
P Q P
X
X
The contact between But, the coil continues
the carbon brushes P to rotate even though
and Q with the there is no turning
commutators X and Y effect because of its
is broken. inertia.

N current
No No turning
flows in the coil. effect.

446
Electromagnetism 446
Stage 3: Second half rotation
Stag
The force on the side
AB is upwards.
Horizontal position
orce
12
The coil is still
rotating in a
clockwise rotation.
ield

Current force
Current
B rotation
C
F
O

3
R
A

CHAPTER
M
D

force
orce 5
The commutators X Q P
and Y are in contact
with carbon brushes X Y
P and Q again.
FF
OO

33
Current RR

CHAPTER
MM

CHAPTER
iel d

55

Carbon brush P now The current now


w orce
touches Y and carbon flows in the
The force on the side
brush Q touches X. A.
direction DCBA.
CD is downwards.

The speed of the motor increases when Stage 4


(a) the strength of the magnetic field is
increased, Vertical position
(b) the number of turns of wire in the
coil is increased,
(c) the area of the coil is increased, B
(d) the coil is wound over an iron core.

Q X

P Q X D
P
Y
Y

14
14 15
13 The processes are
The contact between Current flow is cut
off. gain, the coil repeated and the
the carbon brushes motor continues to
and the commutator is continues to rotate
because of its rotate until the current
broken again. is switched off.
inertia.

447
447 Electromagnetism
shaft The simple motor in Figure 3.40 produces a weak turning
slot with coils effect and is jerky in action especially at low speeds.
stator magnets Electric motors used in industry have a much better
performance because they consist of several coils wound
in slots spaced at equal intervals in a soft iron cylinder
called the armature. The slots are connected to segments
brushes which correspond to a commutator which has many
stator coils
segments. With this arrangement, the motor rotates
brushes
smoothly at a higher speed.
commutator

Activ To study the factors that affect the speed of rotation of


ity 3.3
SPM
’09/P1
an electric motor

Apparatus/Materials 2 The current is turned on. The speed of rotation


Battery with 2 dry cells, simple d.c. electrical motor of the armature is observed and compared to
F kits. Activity (I).
O
3

R Arrangement of apparatus (III) Increasing the current in the coil


CHAPTER

M armature Magnadur magnet 1 The resistance in the rheostat is reduced.


axle
5
N support pin 2 The switch is closed. A larger current flows in
S the coil.
3 The speed of rotation is observed and compared
+ to Activity (I).
to battery iron yoke
with 2 dry – (IV) Increasing the number of turns in the coil
cells insulated
copper wire 1 The armature in Activity (I) is replaced with
fine rubber
tube bare copper wire another armature with the same dimension but
fine brush
with 20 turns in the coil.
metal 2 The current is turned on and the rheostat is
rod adjusted to allow a current of 1 A to flow.
Figure 3.41 3 The speed of rotation is observed and compared
Procedure to Activity (I).
(I) Original set (V) Increasing the area of the coil
1 A simple d.c. motor kit is used for this activity. 1 The activity is repeated with another armature
2 The motor is arranged with a battery (consists of of larger area but with other factors including
two dry cells) connected to the brushes of the current the same as in Activity (I).
motor. 2 The speed of rotation is observed and compared
3 The Magnadur magnets are arranged with to Activity (I).
unlike poles facing each other. Observations
4 A coil with ten turns is used. It is observed that all the factors considered in
5 The current is turned on. The rheostat is Activities (II) to (V) increase the speed of rotation
adjusted to allow a current of 1 A to flow. of the motor.
6 The speed of rotation of the armature is
Conclusion
observed.
Activity 3.3

The speed of a motor can be increased by


(II) Increasing the magnetic strength (a) increasing the strength of the magnetic field,
1 The strength of magnetic field is increased by (b) increasing the current,
adding another pair of Magnadur magnets to the (c) increasing the number of turns in a coil,
iron yoke. (d) increasing the area of the coil.

Electromagnetism 448
Radial Field

1 A radial field is a magnetic field with the field 3 Many electrical appliances such as loudspeakers
lines pointing towards or away from the centre and moving coil meters operate on the
of a circle like spokes of a wheel. principle of radial fields.
2 A radial field is advantageous because a 4 A radial field can be produced in two ways as
cylinder of soft iron can produce converging shown in Figures 3.42 (a) and (b).
magnetic field lines.

S N S N S N
concave
S magnet
S radial
magnetic soft iron core
top view field top view

(a) A cylindrical magnet with one pole at the centre and (b) A soft iron cylinder at the centre surrounded by
surrounded by the opposite pole circumferentially concave magnets
Figure 3.42

Applications of the Force on a Current-carrying Conductor in a Magnetic Field F


O

3
R
Moving Coil Loudspeaker

CHAPTER
M
1
A loudspeaker consists of a paper cone attached to a cylinder which is 5
(a) permanent wound with a coil.
magnet
coil 
2
N A permanent magnet, with its north pole at the centre surrounded
circumferentially by the south pole, is incorporated in the cylinder.
The radial magnetic field produced is perpendicular to the current
paper cone through the coil.


(b) radial magnetic 3


field
An alternating current from an amplifier flows in the coil and
S coil produces a magnetic field around it. The interaction of the magnetic
F field of the permanent magnet and the magnetic field of the coil
N produces a force on the paper cone.


F 4
S
In the first half of the cycle, assume that the current flows into the
paper at the upper end of the coil and flows out of the paper at the
(c) magnetic Key:
lower end of the coil. By applying Fleming’s left-hand rule, the force
field Current
into the acts on the coil to move the paper cone to the right (outwards).
paper
catapult Current


force, F out from 5


the paper In the second half of the cycle, the direction of current flow in the coil
catapult
force, F
is reversed. The direction of the force produced is also reversed and
the paper cone moves to the left (inwards).
(d)


catapult 6
force, F The paper cone vibrates at the frequency of the alternating current
catapult
(which has the same frequency as the original sound). The air in front
force, F of the cone undergoes a series of compressions and rarefactions to
Figure 3.43 reproduce the original sound.

449 Electromagnetism
Moving Coil Meter SPM
’09/P2(A)

4
The pointer is fixed to the axis of rotation to show
the magnitude of the current on a linear scale.

2
1 3

1
hair spring X A moving coil meter consists of a large
0

4
pointer current in
number of turns of fined insulated copper
wire round a light aluminium frame which
coil on can be rotated freely in the gap between the
F
F
aluminium permanent magnet and the soft iron cylinder.
frame

F
N S permanent
magnet 2
The soft iron cylinder and the concave-shaped
magnets produce a radial field in the space
fixed soft current out between the magnet and the cylinder. The
O
total flux passing through the rectangular coil
3

iron cylinder
R hair spring Y
is the same for all positions of the coil.
CHAPTER

M
bearing

5 3
Two spiral hairsprings X and Y are used to control
the angle of rotation of the coil.
The hairsprings are wound in opposite directions
to compensate for thermal expansion.

Figure 3.44 Pivoted moving coil meter

Operating principle 2 The sensitivity of a moving coil meter can be


1 When a current flows through the coil, two increased by
opposite forces are set up on the sides of the (a) increasing the strength of the radial
coil. magnetic field (using a more powerful
2 These two forces form a deflecting couple magnet),
which rotates the coil and moves the pointer (b) increasing the area of the coil,
across the scale. (c) increasing the number of turns of the coil,
3 The rotation of the coil is resisted by the two (d) using hairsprings of lower stiffness,
hairsprings. (e) using a coil and pointer of smaller mass.
4 The coil comes to rest when the turning couple 3 The above 5 factors enable a larger rotation of
is balanced by the opposing couple of the the coil with a smaller current.
hairsprings. The pointer shows the magnitude
of the current. Linear scale
5 When the current is cut off, the opposing The radial magnetic field and the spiral hairsprings
couple from the hairsprings restores the coil to which obey Hooke’s law, causes the angle of
its original position and the pointer returns to rotation of the coil to be directly proportional to
the zero mark. the current flowing—i.e. I ∝ θ. Therefore, the
moving coil meter has a linear (uniform) scale (i.e.
Sensitivity of a moving coil meter with numbers evenly spaced).
1 A moving coil meter can be used to measure
current in milliamperes and microamperes if
the sensitivity of the meter is increased.

Electromagnetism 450
3 SPM
Clone
3.2
’10
1 A current–carrying conductor is placed in a magnetic
The diagram shows the representation of Left-Hand field as shown in the diagram.
SPM
Clone
Rule using a left hand. ’08

X
Direction of
current out
of the paper

PX represents the
What is the direction of the force acting on the
A motion conductor?
B current A J B K C L D M
C magnetic field
2 The diagram shows two identical metal plates placed
D potential difference close together. Each plate is connected to a battery
Comments as shown.
The finger indicates the direction of the current.
F
Answer B O

3
R

CHAPTER
M

What is the force between the two metal plates? 5


Make an inference about the force acting between
the coils of wire in a solenoid.
4 SPM
Clone 3 Two aluminium rods are arranged close together as
’09
shown in the diagram.
The diagram shows a simple electric motor.
wire coil
thread
F magnet
N
S

F
aluminium rod

carbon Draw a diagram to show what will be observed


commutator brush
when both switches are closed.
4 Referring to the arrangement of apparatus in the
following diagram, it is found that the wire hung
The turning force on the coil will increase when from the hook moves around the north pole of a
A the number of turns of the coil is decreased. magnet in the container containing mercury.
B a weaker magnet is used.
C the current is reduced.
D the resistance of the wire coil is decreased.
wire in
Comments motion
When the resistance of the wire coil is reduced
(without reducing the number of turns or area of mercury N
coil), the current in the coil increases. Thereby S
increases the turning force.
rheostat
Answer D
Explain the observation.

451 Electromagnetism
3.3 Analysing Electromagnetic Induction
The force acting on a current-carrying conductor in a magnetic field was
discovered by scientists in 1821. Can the process be reversed, i.e. to produce
a current by moving a conductor in a magnetic field?
This question was proved valid by a physicist named Michael Faraday in
1831. He succeeded in producing a current by moving a conductor in a
magnetic field. This discovery led Faraday to invent the dynamo (or
generator).

Electromagnetic Induction

1 The production of an electric current by a changing magnetic field is


called electromagnetic induction.
2 The induced current is produced only when there is relative motion
between the conductor/coil and the magnetic field lines.
3 The induced current is produced when
(a) a conductor cuts across a magnetic flux,
F
(b) there is a change in magnetic flux linking a coil or a circuit.
O
3

R
Relative Motion
CHAPTER

5 1 There is a relative motion between two objects if the two objects are
getting closer or further apart.
2 When two objects are moving at the same speed in the same direction,
there is no relative motion because the distance between the two
objects does not change.

-
Activ To observe electromagnetic induction in
ity 3.4
SPM SPM SPM
’05/P3(B) ’06/P1 ’09/P1
(a) a straight wire and (b) a solenoid

Apparatus/Materials Procedure
Magnadur magnets, U-shaped iron yoke, PVC 1 A pair of Magnadur magnets with unlike poles
insulated copper wire, a bar magnet, cardboard tube facing each other is placed in the iron yoke.
and a centre-zero galvanometer.
2 A PVC insulated copper wire is connected to a
(I) A straight wire centre-zero galvanometer as shown in Figure
Arrangement of apparatus 3.45.

centre-zero 5
0
5
3 The actions taken in the investigation,
galvanometer
10 10

observations and inferences are recorded in


Table 3.7.
N Magnadur
magnet
S
Activity 3.4

PVC
insulated
thick wire

Figure 3.45 Investigation of the effect of relative motion


between a straight conductor and a magnet

Electromagnetism 452
Observations
Table 3.7

Action Observation: Inference


Momentary deflection of
galvanometer (Left / Right)
(a) The wire is moved downwards. Right (for example) Current flows in the wire.
(b) The wire is moved upwards. Left Current flow is reversed.
(c) The wire is moved horizontally No deflection No current is generated.
sideways.
(d) Magnet is moved upwards. Right Current flows in the wire in
the direction as in (a).
(e) Magnet and wire are moved at the No deflection No current is generated.
same speed in the same direction.
(f) The wire is held stationary No deflection No current is generated.
between the Magnadur magnets.
F
O

3
R
Discussion Conclusion

CHAPTER
M
1 Since there is no current source, the current 1 Current is induced when a conductor cuts the
5
(shown by the momentary deflection of magnetic flux.
galvanometer) must have been induced by the 2 The direction of the current induced depends on
magnetic field. the direction of motion of either the conductor
or the magnetic lines.
2 The current is induced when there is relative
motion between the wire and the magnetic field. (II) A solenoid
The current induced is the same whether the Arrangement of apparatus
wire moves downwords (to cut the magnetic
field lines) or the magnet moves upwards (so
centre-zero galvanometer
that the field lines cut the wire) because it is 1
0 1 2
3
G
their relative motion which causes an induced
4
2
3
4

current.
conductor
N S

N S solenoid
bar magnet

Figure 3.47 Effect of moving a solenoid


direction
relative to a magnet
of motion Procedure
1
4

1 A PVC insulated copper wire of length 2 m is


wound round a cardboard tube of diameter 5 cm
centre-zero galvanometer
to form a solenoid with 50 turns.
Figure 3.46 2 The ends of the copper wire are connected to a
centre-zero galvanometer as shown in Figure
Activity 3.4

3 When the wire is moved horizontally across the 3.47.


gap from N pole to S pole, it slides in between 3 The actions taken in the investigation,
the magnetic lines. There is no cutting of observations and inferences are recorded in
magnetic flux and no current is induced. Table 3.8.

453 Electromagnetism
Observations

Table 3.8

Observation:
Action
Momentary deflection of Inference
galvanometer (Left / Right)
(a) The north pole of the magnet Current flows in the
Right (for example)
is moved into the solenoid. solenoid.
(b) The bar magnet is moved out Current flow in the solenoid
Left
of the solenoid. is reversed.
(c) The south pole of the magnet is moved Current flow in the solenoid
Left
into the solenoid. is reversed.
(d) The solenoid is moved towards Current flows in the
Right
the bar magnet which is held stationary. solenoid as in (a).
(e) The magnet is held stationary
No deflection No current is generated.
in the solenoid.
(f) The bar magnet and solenoid are
F No deflection No current is generated.
O moved at same speed in the same direction.
3

R
CHAPTER

M
Discussion
5
1 In the above activities, an induced current is produced when there is a relative motion between the bar magnet
and the solenoid.
2 When there is linkage of magnetic flux in the solenoid, the magnetic field lines pass through the turns of wire
in the solenoid.
3 Figure 3.48 shows the change in magnetic flux linkage when a bar magnet approaches a solenoid.

S N S NS N S NS N S N

(a) No flux linkage in solenoid (b) Flux linkage in (c) Increased flux
solenoid exists linkage in solenoid

Figure 3.48 Bar magnet approaching a solenoid

4 The induced current produced in this case is due to the change in magnetic flux linkage in the solenoid.
(a) When the bar magnet is plunged into the solenoid, the magnetic flux linkage in the solenoid increases and
a current is induced.
(b) When the bar magnet is pulled out of the solenoid, the magnetic flux linkage in the solenoid is decreased
and a current is induced in the opposite direction.
(c) When there is no relative motion, there is no change in magnetic flux linkage in the solenoid and no
induced current is produced.
5 The change in flux linkage produces the induced current.
Activity 3.4

Conclusion
1 Induced current is produced when there is a relative motion between a solenoid and a magnet.
2 The direction of the induced current depends on the pole of the magnet used and the direction of motion of
either the magnet or the coil.

Electromagnetism 454
Induced Current and Induced Electromotive Force

1 You have learnt in Chapter 2 that the electromotive force is required to


drive the current in a closed circuit.
2 In Activity 3.4, induced electromotive force (induced e.m.f.) is
produced between the ends of the moving conductor or the solenoid.
3 Figure 3.49 (b) shows that when a galvanometer is connected to form a
closed circuit, the pointer of the galvanometer moves when the magnet
is moved back and forth inside the solenoid.

S N S S N
0

e.m.f.
(a) Induced e.m.f. in a (b) Induced current flows
solenoid from solenoid.
Note that the solenoid
is now a source
Figure 3.49
F
4 The induced e.m.f. is responsible for driving the current flow in the O

3
closed circuit through the galvanometer. The greater the induced e.m.f. R
is, the greater the induced current.

CHAPTER
M
5 The magnitude of the induced e.m.f. and direction of the induced
5
current can be determined by application of the laws of
electromagnetic induction.

Laws of Electromagnetic Induction

There are two principal laws of electromagnetic induction:


(a) Faraday’s law
(b) Lenz’s law

Faraday’s Laws of Electromagnetic Induction

1 Faraday’s law states that the magnitude of the induced electromotive The turns of wire act like cells
connected in series as shown
force (e.m.f.) is directly proportional to the rate of change of
in Figure 3.50.
magnetic flux linkage in the solenoid or the rate at which a conductor
cuts through the magnetic flux.
2 The induced e.m.f. increases when the rate of change of flux increases.
Hence, the induced current is increased when equivalent as
(a) a stronger magnet is used,
(b) the speed of relative motion is increased.
3 The induced e.m.f. increases when the number of turns increases Figure 3.50 Turns of wire as
because an e.m.f. is induced in each turn of wire. cells connected
in series
SPM Therefore, the induced e.m.f.
Lenz’s Law ’05/P1
in the solenoid is the same as
the total of the e.m.f. of the
1 Lenz’s law states that an induced electric current always flows in such
individual turns in the
a direction so as to oppose the change (or motion) causing it.
solenoid.
2 Table 3.9 gives an explanation of Lenz’s law.

455 Electromagnetism
Table 3.9
(a) (b) N
S
P Q P Q
S N S N
N

When a north pole is moved towards the end P, When a north pole is moved away from the solenoid,
the end P becomes a north pole to produce a force the end P becomes a south pole to produce a force of
of repulsion to oppose the change (in this situation, attraction to oppose the motion of the bar magnet
the motion of the bar magnet) which produces it. which produces the induced current. The direction of
The current flows in the direction as shown. the current is reversed and the galvanometer needle is
deflected in the opposite direction.
(c) (d)
N S
P Q P Q
N S N S

F
O
3

R
CHAPTER

M
When a south pole is moved into the solenoid, the When the south pole is moved away from the solenoid,
5 end P becomes the south pole by induction. the end P becomes a north pole by induction.

3 Lenz’s law is a form of the law of conservation of energy.


It can be explained by the principle of conservation of energy.
(a) When the magnet is moved towards the solenoid, current is induced in the solenoid. Why
does this occur? Electrical energy cannot be created without any form of work being done.
Hence, the end P must be induced to become the north pole so that a force of repulsion
exists between the solenoid and the magnet.
P Q P Q

S N S N
N S S N
force of force of
repulsion attraction
exists exists

Figure 3.51 Principle of conservation of energy


(b) When the magnet is moved away from the solenoid, the end P is induced to become the
south pole so that a force of attraction exists between the solenoid and the magnet.
(c) Therefore, when a magnet is moved towards or away from a solenoid, work must be done
to overcome the opposing force.
(d) The work done is converted into electrical energy which creates the induced current.

Situation: Near ends of both


So that Reason
Magnet moving magnet and solenoid
In Same poles force of repulsion exists work is done to produce the
Out Unlike poles force of attraction exists electrical energy

Electromagnetism 456
Fleming’s Right-hand Rule (Dynamo Rule) SPM
’06/P1

The direction of a current induced in a conductor moving at right angles to and cutting across a
magnetic flux can be determined with Fleming’s right-hand rule.
Motion, M Motion, M
magnetic
Field,B (thuMb)

(First finger)
magnetic
Field,B
induced
Current,I
(seCond finger)
induced
Current, I

(a) (b)
Figure 3.52 Fleming’s right-hand rule

If the first finger, second finger, and the thumb of the right hand are held at right angles to each other,
with the First finger indicating the direction of the Field (B), the thuMb indicating the direction of the
Motion of the conductor, then the seCond finger points in the direction of the induced Current (I).

F
O
Activ

3
To investigate the factors that affect the magnitude of the
ity 3.5 induced current in a conductor
R

CHAPTER
M

Apparatus/Materials 5
PVC insulated copper wire (20 cm × 1, 50 cm × 1), Magnadur magnets, U-shaped iron yoke and centre-
zero galvanometer.
Arrangement of apparatus Procedure
Magnadur magnet Magnadur magnet 1 The apparatus is set up as shown in Figure
3.53 (a). The Magnadur magnets are arranged
N S N with opposite poles facing each other.
S

2 The actions taken as well as the observations


iron iron and conclusions made are given in Table 3.10.
yoke yoke

(a) (b)
Figure 3.53
Observations
Table 3.10

Angle of momentary Observation


Case Action deflection of and
galvanometer conclusion
(I) (i) The wire is moved Small Observation
Speed downwards slowly. The galvanometer deflection
of conductor (ii) The wire is moved Large increases when the conductor is
(relative downwards quickly. moved at a faster rate.
motion) Conclusion
The size of the momentary deflection of
Activity 3.5

the galvanometer indicates the magnitude


of the induced current. Therefore, the
induced current increases when the
speed of relative motion is increased.

457 Electromagnetism
Angle of momentary Observation
Case Action deflection of and
galvanometer conclusion

(II) (i) The wire is moved Small Observation


Strength downwards at a fixed speed. The galvanometer deflection
of the (ii) Two additional Magnadur Large increases when more magnets are
magnetic magnets are placed on each used.
field side of the iron yoke to Conclusion
increase the strength of the The induced current increases
magnet. The wire is moved when the strength of the magnetic
at the same speed as in step (i). field is increased.

(III) (i) The wire is moved Large Observation


Cutting downwards at 90° to the The galvanometer deflection
angle magnetic field at a fixed decreased when the angle between
speed. the motion of the conductor and
(ii) The wire is moved at Small the magnetic field decreases.
the same speed as in step (i) Conclusion
F but at an angle of less The induced current is
O
than 90°. maximum when the cutting angle
3

R
is at 90°.
CHAPTER

5 (IV) (i) A 50 cm length of wire is used. Small Observation


Length The wire is moved The galvanometer deflection
of downwards at a fixed speed. increases when the length of the
conductor (ii) The wire is wound into Large conductor is increased.
a coil as shown in Figure Conclusion
3.53 (b). The coil is moved The induced current increases
at the same speed as in when the length of the conductor
step (i). This is equivalent is increased.
to cutting the magnetic flux
with a longer conductor.

Conclusion
1 The magnitude of the induced current increases when
(a) the speed of relative motion is increased,
(b) the strength of the magnetic field is increased,
(c) the length of the conductor is increased.
2 The magnitude of the induced current is maximum when the conductor cut across the magnetic field
perpendicularly.
Activity 3.5 & 3.6

Activ To investigate the factors that affect the magnitude


ity 3.6 of the induced current in a solenoid
SPM
’03/P1
SPM
’04/P1

SPM
’05/P3(B)
Apparatus/Materials
PVC insulated copper wires (3 m × 1, 5 m × 1), two bar magnets, cardboard tube and center-zero
galvanometer.

Electromagnetism 458
Arrangement of apparatus

N N N N
N
20 turns 20 turns 20 turns 40 turns

(a) Case I: Effect of speed (b) Case II: Effect of (i) (ii)
of relative motion strength of magnet (c) Case III: Effect of number of turns of solenoid (Same
length of wire so that the resistance of wire is the same)
Figure 3.54
Procedure
1 The apparatus is set up as shown in Figure 3.54 (a).
2 The actions taken as well as the observations and conclusions made are given in Table 3.11.
Observations
Table 3.11

Angle of momentary Observation


Case Action deflection of and
galvanometer conclusion
(I) (i) A solenoid with 20 turns is Small Observation F
Speed prepared using a 3 m length The galvanometer deflection O

3
of magnet of copper wire. The north increased when the bar magnet R

CHAPTER
(relative pole of the bar magnet is was moved into the solenoid at a M
motion) moved slowly into the higher speed.
5
solenoid.
(ii) The north pole of the bar Large Conclusion
magnet is moved into the The induced current increases
solenoid quickly. when the speed of relative motion
is increased.
(II) (i) The north pole of the bar Small Observation
Strength magnet is moved into the The galvanometer deflection
of the solenoid at a fixed speed. increases when more magnets are
magnetic (ii) Two bar magnets are tied Large used.
field together with the same
poles at each end to form a Conclusion
stronger magnet. The north The induced current increases
pole is moved into the when the strength of the magnet
solenoid at the same speed is increased.
as in step (i).
(III) (i) A solenoid with 20 turns is Small Observation
Number prepared using a 5 m length The galvanometer deflection
of of copper wire. The north pole increases when the number of
turns in of the magnet is moved into turns in the solenoid is increased.
solenoid the solenoid at a fixed speed.
(ii) A solenoid with 40 turns is Large Conclusion
then prepared with the same The induced current increases
length of copper wire. when the number of turns in the
The north pole is moved into solenoid is increased.
the solenoid at the same
speed as in step (i).
Activity 3.6

Conclusion
The induced current increases when (b) the strength of the magnetic field is increased,
(a) the speed of relative motion is increased, (c) the number of turns in the solenoid is increased.

459 Electromagnetism
7 8
Figure 3.55 shows a conductor PQ which moves to In Figure 3.56, a current is induced when the U-shape
the left on a plane of metal wire connected to a iron core is moved upwards.
galvanometer. conductor Y
at rest
P Q
X

Q
iron core galvanometer
galvanometer moves upward
P
Figure 3.56
In which direction will the current flow in the
Figure 3.55 conductor?
Solution
Determine the direction of the current induced in the
conductor PQ. The iron core is magnetised when current flows in the
solenoid. The end P is the south pole and the end Q is
F the north pole. The upward movement of the iron
O Solution core is equivalent to the downward movement of the
3

R Applying Fleming’s right-hand rule, it is found that wire XY. Applying Fleming’s right-hand rule, it is
CHAPTER

M the current flows in the conductor from P to Q. found that the induced current flows from X to Y.
5

Applications of Electromagnetic Induction

Direct Current Generator and Alternating Current Generator

1 The direct current generator (d.c. dynamo) and the alternating current generator (a.c. dynamo)
make use of electromagnetic induction to produce a current.
2 Table 3.12 compares the direct current generator and the alternating current generator.

Table 3.12

Direct current generator Alternating current generator


rotation rotation
b b c
c M
M I

I N M
S
N a
d M S carbon brush
a d
spring Q P coil
Key : M = motion
carbon
commutator brush slip rings

Key: M = motion spring

• The generator consists of a rectangular coil rotating


• The construction features are similar to an electric in a magnetic field.
motor. The difference is that the coil of the • The two ends of the coil are connected to two slip
generator is not connected to a battery but to a load rings which rotate with the coil.
(e.g. resistor or bulb). • Two spring-loaded carbon brushes are in contact
• The coil is rotated to produce a current. with the slip rings.

Electromagnetism 460
Operating Principle of a Direct Current Generator (d.c. Dynamo)

Coil in horizontal position Coil in vertical position


• Side ab moves upwards and side cd moves • The sides ab and cd moves in parallel with the
downwards, cutting across the magnetic field lines magnetic field lines.
and produces an induced current. • No induced current is produced because there is no
• Applying Fleming’s right-hand rule, the induced cutting of field lines.
current flows from a to b and from c to d. • The galvanometer returns to the zero mark.
• In the external circuit, current flows from brush P to • After this position, the two commutator halves
brush Q (the galvanometer pointer deflects to the change contact with brushes P and Q.
left). rotation
• After this position, the current increases.
• The current at
b c
this instant is M I b
maximum.
M
• After this
I
position, the a c
current
N a
d M S
spring Q P N M S
decreases d
because commutator
carbon
the magnetic brush
field lines are Key: F
cut obliquely. M = motion O

3
R
(a) Cutting the field lines at 90°, I = maximum (b) No cutting of field lines, I = 0

CHAPTER
M

direct current 5
3
1

2
4 angle of
0° 90° 180° 270° 360° rotation
4
Figure 3.57 (a) Output of a direct current generator
Coil in vertical position Coil in horizontal position
• Again, no current is induced as there is no cutting • Side ab moves downwards and side cd moves
of magnetic field lines. upwards, cutting across the magnetic field lines
• After this position, the current increases and the again.
process is repeated. • The current now flows from b to a and from d to c.
• However, in the external circuit, the current still
flows from brush P to brush Q.
c • The direction of the current is maintained and the
galvanometer shows a deflection in the same
M
direction.
d
b • After this position, M c b
the current
N M S decreases.
a
N d a M
S

(d) No cutting of field lines, I = 0 (c) Cutting the field lines at 90°, I = maximum

461 Electromagnetism
Operating Principle of an Alternating Current Generator (a.c. Dynamo)

Coil in horizontal position Coil in vertical position


• Side ab moves upwards and side cd moves • The sides ab and cd moves in parallel with the
downwards, cutting across the magnetic field lines magnetic field lines.
and produces an induced current. • No induced current is produced because there in no
• Applying Fleming’s right-hand rule, the induced cutting of field lines.
current flows from a to b and from c to d. • The galvanometer returns to the zero mark.
• In the external circuit, current flows from brush P to • After this position, the current increases.
brush Q (the galvanometer pointer deflects to the
left). b
• The current at this instant is maximum.
• After this position, M
the current M
b c a
decreases. c
N M S
N M
S
a d Q d
Q
P

F P
O
3

R
(a) Cutting the field lines at 90°, I = maximum (b) No cutting of field lines, I = 0
CHAPTER

Induced current, I
5
1

2 output current
from Q to P
Time, t
4 output current 4
from P to Q
3

Figure 3.57 (b) Output of an alternating current generator

Coil in horizontal position

Coil in vertical position • Side ab moves downwards and side cd moves


upwards, cutting across the magnetic field lines
• Again, no current is induced as there is no cutting of again.
magnetic field lines. • The current now flows from b to a and from d to c.
• After this position, the current increases and the • In the external circuit, the current flows from brush
process is repeated. Q to brush P.
• The direction of the current is reversed and the
c
galvanometer shows a deflection in the opposite
direction.
M
• After this position, the current decreases.
d
b M c b
N M S

Q a N M
S
d a
P Q

P
(c) Cutting the field lines at
(d) No cutting of field lines, I = 0 90°, I = maximum

Electromagnetism 462
Moving Coil Microphone

coil
pot
N magnet
A moving coil microphone consists of a diaphragm which is diaphragm
connected to a coil placed in a cylindrical pot magnet.
S
The coil is in a radial magnetic field.
to
N amplifier

(a)
When a person speaks through a microphone, the diaphragm
and the coil vibrate at a certain frequency. F

F
When the coil moves into the magnet, an induced current flows
in the coil. Figure 3.57 (b) shows the direction of the current in F
(b)
one winding. O

3
F R

CHAPTER
M

When the coil moves out from the magnet, the induced current
flows in the opposite direction [Figure 3.57 (c)]. As a result, an F
alternating current at the frequency of the sound is produced and
sent to an amplifier. (c)
Figure 3.57 Moving coil microphone

Using bicycle to generate electricity!

motion of wheel
moving bicycle
wheel

rotating
shaft
soft
S N
iron cylindrical
core magnet

terminal solenoid

A bicycle dynamo is a small generator fitted to a bicycle to provide electricity for the lights at night.
It consists of a cylindrical permanent magnet with poles on opposite sides placed within concave poles of a soft iron core
where a solenoid is wound.
A shaft connects the magnet to the driving wheel of the dynamo. This causes the magnet to rotate in the soft iron core
when the wheel is turning. An alternating current is produced.
The bicycle dynamo has the advantage that it needs no slip rings and commutators. It doesn’t need carbon brushes which
need to be replaced after wearing out.

463 Electromagnetism
Force acting on a current-carrying Induced current in a conductor
conductor in a magnetic field moving in a magnetic field
1 Direction of force is determined with Fleming’s 1 Direction of induced current is determined with
left-hand rule. Fleming’s right-hand rule.
F (M ) F (M )

S S
N B B N
(+) (+)

I I
(–) (–)

2 Magnitude of the force increases when 2 Magnitude of the induced current increases when
(a) the strength of the magnetic field is increased, (a) the strength of the magnetic field is increased,
(b) the length of the conductor is increased, (b) the length of the conductor is increased
(c) the current is increased, (c) the speed of relative motion is increased,
3 Force is maximum when the direction of the 3 Induced current is maximum when the direction
F magnetic field, B is perpendicular to the direction of the magnetic field, B is perpendicular to the
O of the current, I. direction of the force, F (or motion).
3

R
CHAPTER

M Differences between a direct current motor and a direct current generator:

5 Direct current motor Aspect Direct current generator

Current + Coil = Motor Rotation + Coil = Generator


(in magnetic field) Product (in magnetic field)

Uses a direct current to rotate the coil. Principle Rotates a coil to produce the current.

Direction of rotation is determined Direction of induced current is determined


using Fleming’s left-hand rule. Rule used using Fleming’s right-hand rule.

Speed of rotation increases when Induced current increases when


• strength of magnet increases • strength of magnet increases
• cross-sectional area of coil increases • cross-sectional area of coil increases
Factors
• number of turns increases • number of turns increases
• coil is wound on an iron core • coil is wound on an iron core
• current increases • speed of rotation increases

9
The graph in Figure 3.58 shows an induced current for Which graph represents the current-time graph when
an alternating current generator which is rotated at the coil armature is rotated at a higher speed?
two different speeds. Give an explanation.

current, I Solution
When the generator is rotated at twice the frequency,
A′o II the period of rotation is halved and hence the pattern of
Ao I the wave appears closer. Because the sides of the coil
O
time, t cuts the magnetic field lines at twice the rate, the rate of
change of flux is doubled. The induced maximum
T′ current also increases twofold. Hence, the current Ao′
with period T′ (i.e. graph II) represents the current-time
Figure 3.58 graph obtained when the coil is rotated at a faster speed.

Electromagnetism 464
A common mistake made by students is to draw a used to supply electrical energy to rotate the coil of a
diagram of a d.c. generator with a battery when asked to motor. Therefore, an electrical generator does not
explain the action of a d.c. generator. require the use of a battery.
Students must understand that an electrical generator is a
machine used to produce electric current. A battery is

Alternating Current

1 We have studied that the magnitude of the 2 Every time the coil passes the vertical/upright
output current from an alternating current position, the direction of the output current
generator varies with time as shown in Figure reverses.
3.59. 3 This type of current is called alternating
current current.
4 The maximum current produced when the coil
Imax
is in the horizontal position is called the peak
current, Io.
0.01 0.02 0.03 0.04 5 The current that we obtained from the main F
time O
supply is alternating current.

3
1 cycle R

CHAPTER
6 The frequency of the alternating current supplied M

is 50 hertz and the period is 1 s = 0.02 s. 5


50
1 cycle

Figure 3.59

Comparison of Direct Current (d.c.) and Alternating Current (a.c.)

Table 3.12

Aspect Direct Current (d.c.) Alternating Current (a.c.)


Current-time graph drawn from a lead-acid Current-time graph from a low voltage a.c.
accumulator: supply:
I (A) I (A)

2 1.5

O t (s)
0.01 0.02 0.03 0.04
Current-time graph O t (s)
–1.5

T = 0.02 s
1 1
f = –– = ––––– = 50 Hz
T 0.02
Peak current = 1.5 A
1 Magnitude of current is constant. 1 The current flows in one direction for
2 Current flows in one direction only. 0.01 s before changing to the opposite
Characteristics 3 It is called a steady direct current. direction. It repeats again after 0.02 s.
2 It is called a sinusoidal alternating
current.

465 Electromagnetism
Aspect Direct Current (d.c.) Alternating Current (a.c.)
A 3 V battery is connected to the Y-input A 2 V a.c. is connected to the Y-input of a
of a CRO with timebase on and Y-gain CRO with timebase on and Y-gain of
of 1 V cm–1. 1 V cm–1.
Display on the
CRO CRO
screen of a cathode-
ray oscilloscope
(CRO)

2V
3V
a.c.

2V

Effect on a bulb
bulb 2.5 V bulb 2.5 V
F
O
The bulb lights up. The bulb lights up.
3

R
CHAPTER

M
Conclusion
5
Both direct and alternating currents produce heating effect on the filament which cause the bulbs to light up.

3V 2V

Effect on a capacitor
The bulb does not light up. The bulb lights up.
(Actually, it lights up for a very short
while after the switch is closed.)

Conclusion
(i) Capacitor allows alternating current but not direct current to pass through.
(ii) This characteristic could be used to differentiate an alternating current from a direct current.

Effect on a moving
coil loudspeaker

The paper cone does not vibrate. The vibration of the cone could be felt
with a finger touching the paper cone.

Conclusion
The paper cone vibrates at the frequency of the alternating current, thereby reproducing the original sound.

Electromagnetism 466
Capacitor If you are given a few current-time graphs and asked to
A capacitor is an electrical component for storing charge. classify the type of current, it is very easy.
Basically it consists of two metal plates with an insulator If the graph shows positive values but no negative
in between. One plate is positive while the other is values (or the reverse), it is a direct current.
negative. The symbol for capacitor is as shown in figure
(b).

metal plate insulator

O O

Direct current from a A square-shaped d.c.


d.c. generator

I
(a) (b) Symbol
Note: Charge is stored in the insulator, not the metal
plates.
t
O
The capacitor can be used to separate an alternating F
current from a mixture of direct and alternating currents. O

3
A varying d.c. A square-shaped a.c. R

CHAPTER
M

5
Peak Current/Voltage and Root Mean Square Current/Voltage

I (A) 2 It can be shown that, a steady current of 0.71 A


produces the same heating effect as the
+ = More heating
1
effect
alternating current with peak current of 1 A
resistor
t (s) [Figure 3.60(c)]. The 0.71 A is the root mean
square (r.m.s.) current of the alternating current
with peak or maximum current Imax = 1 A.
(a) Direct current 3 The root mean square current (Ir.m.s.) of an
I (A) alternating current is defined as the value of a
steady current, which would produce the
1
same heating effect in a given resistor if it
t (s)
+ = Less heating
replaces the alternating current.
resistor effect 4 The root mean square voltage (Vr.m.s.) of an
–1 alternating current is defined as the value of a
steady voltage, which would produce the
(b) Alternating current same heating effect in a given resistor if it
I (A)
replaces the alternating current.
0.71 + = Same heating
5 The voltages labelled in ‘multi-tap’ transformer
resistor effect as (2 V, 4 V, ……. 12 V) and low voltage power
t (s) case (b) supply are Vr.m.s..
6 For sinusoidal alternating current or voltage:
(c) Root mean square current Imax
Figure 3.60 • Ir.m.s = or Ir.m.s = 0.71 Imax
2
1 Figure 3.60(b) shows that an alternating
current (sinusoidal) with a peak current of 1 A
produces less heating effect on a resistor Vmax
• Vr.m.s = or Vr.m.s = 0.71 Vmax
compared to a steady direct current of 1 A 2
[Figure 3.60(a)].

467 Electromagnetism
10 5
SPM
Clone
’11

Figure 3.61 shows a current-time graph from a power The diagram shows a solenoid connected to a
supply. galvanometer. A magnet is released from a height
and later falls through the coil.
S

Magnet

O N

Figure 3.61 Wire

coil
What is its Ir.m.s.? Galvanometer

Solution
I 3
Ir.m.s = max = = 2.12 A The deflection of the pointer of the galvanometer
2 2
increases when
F A the turns of the coil is further apart
O
B the polarity of the magnet is reversed
3

R 11 C the cross-sectional area of the coil is increased


CHAPTER

M
D the magnet is released from a higher position
The voltage supply to a house is 240 V. What is the
5
maximum voltage of the power supply? Comments
Solution When the magnet is released from a higher
Vmax position, it will enter the coil at a faster speed and
Vr.m.s = thus increases the rate of change of magnetic flux
2 lingkage. Hence the induced current increases,
Vmax following the Faraday’s laws of electromagnetic
240 =
2 induction.
Vmax = 240 × 2 = 339 V Answer D

3.3

1 The diagram beside Q Determine


shows the production (a) the poles at P and Q,
of a current when a N P R S (b) the pole at X.
copper rod moves in a
S
magnetic field. 3 Two magnets P and Q, of equal strength, are used to
In which direction, P, Q, R or S, must the rod be investigate induced currents. The conductor XY
moved to produce a current which flows into the moves downwards accurately at the midpoint
plane of the paper? between the two magnets P and Q as shown in the
2 The induced currents flow in the directions as shown following diagram.
in the diagrams below.
direction of movement direction
of movement

bar
magnet

(i) (ii)

Electromagnetism 468
(a) If an induced current flows from X to Y 5 The diagram below shows a copper ring falling freely
through the galvanometer, what are the poles at through a stationary magnetic field onto a table below.
P and Q?
(b) Give an explanation if no current is induced. copper ring
(plane perpendicular
to the magnetic field)
4 A bar magnet is placed between two coils of wire as
shown in the following diagram.

10 turns 20 turns N

(a) Give a full explanation of the flow of current in


the copper ring when viewed from the right-hand
When the bar magnet is withdrawn, what are the side.
poles at P and Q? (b) A second ring is released, at the same time as the
The induced current flows from A to B. first ring, to fall outside the magnetic field.
Give an explanation. Which ring will hit the surface of the table first?
Give an explanation.

F
O

3
SPM SPM SPM R
3.4 Analysing Transformers

CHAPTER
’04/P2(C) ’07/P2(A) ’08/P2(A) M

5
Function of a Transformer SPM
’06/P1
Constructing a Transformer

1 The potential difference from the electrical 1 A simple transformer consists of two coils
supply is 240 V. This voltage is not suitable for wound on a laminated iron core.
certain electrical equipment.
2 For example, the television tube requires a laminated
iron core
potential difference of 15 kilovolt, whereas a
radio requires a potential difference of only 12 V.
input output
a.c. a.c.

primary secondary
coil coil

(a)

Figure 3.62 A t.v. tube needs 15 kV while a


radio needs 12 V for operation F

3 Therefore, it is necessary to raise the electrical (b) Symbol


voltage from 240 V to 15 kV and to lower the
voltage from 240 V to 12 V so that the Figure 3.63 A simple transformer
television and radio can function.
4 The equipment used to raise or lower the 2 The coil which is connected to the power
potential difference of an alternating current supply whose voltage is to be raised or lowered
supply is called a transformer. is called the primary coil. The supply voltage is
5 Transformers also play an important role in called the primary (input) voltage, Vp.
the transmission and distribution of 3 The other coil which is connected to the
electrical energy to the whole country. electrical equipment or resistor is called the
6 The transformer transfers electrical energy secondary coil. The voltage across the
from one circuit to another through secondary coil is called the secondary
electromagnetic induction. (output) voltage, Vs.

469 Electromagnetism
Operating Principle of a Transformer

1 The operating principle of a transformer is based on electromagnetic The primary coil must be
induction. connected to an alternating
2 The current from the electrical supply that is connected to the primary current supply and not to a
coil is a sinusoidal alternating current; that is, a current whose steady direct current supply.
magnitude and direction varies continuously at a certain frequency. This is because an induced
3 The alternating current produces a flux or magnetic field lines which e.m.f. will not be produced
in the secondary coil if a
link the primary coil and the secondary coil. The magnetic flux
steady current is used.
produced varies in magnitude and direction. A steady direct current whose
magnitude and direction is
constant does not create a
changing magnetic flux in the
secondary coil.

(a) (b)
Figure 3.64 Direction of magnetic flux linking the coils changes when the current
direction in the primary coil changes
F 4 The changing magnetic flux produces an induced e.m.f. across the
O secondary coil. This induced e.m.f. also varies in magnitude and
3

R
direction. As a result, the secondary (output) voltage is also a
CHAPTER

M
sinusoidal alternating voltage.
5 5 If the secondary circuit is closed, a sinusoidal alternating current flows
in the circuit with the same frequency as the electrical supply.

Activ To gain an understanding of the structure and the


ity 3.7 operating principle of a simple step-up transformer
SPM
’03/P1

and a step-down transformer

Apparatus/Materials Procedure
Coil of 3600 turns, laminated U-core, PVC covered 1 A coil of 3600 turns is slipped over one arm of a
s.w.g. 26 copper wire, bulbs (2.5 V, 0.3 A), laminated U-core.
low voltage a.c. power supply, soft iron C-core and 2 30 turns of insulated copper wire are wound
C-clip. around the other arm of the U-core, and
connected to a bulb.
(I) Step-down transformer 3 A bar-core is placed over the U-core.
Arrangement of apparatus 4 The primary coil is connected to the mains.
5 The power is turned on. The brightness of the
bar-core
bulb is observed.
Observations
3600 turns
(primary coil) The bulb lights up.
Conclusion
30 turns mains 1 The transformer built is a step-down
Activity 3.7

(secondary
coil)
O transformer. It changes a high voltage (240 V) to
a low voltage (about 2 V).
2 The number of turns on the secondary coil is
bulb U-core less than the number of turns on the primary
Figure 3.65 coil.

Electromagnetism 470
(II) Step-up transformer 3 The two cores are attached together by a C-clip.
Arrangement of apparatus 4 The two bulbs are connected to the circuit as
shown in Figure 3.66.
5 The power is turned on. The brightness of the
bulbs are observed.
low voltage Observations
C-core a.c. power
supply The bulb connected to the low voltage power supply
clip bulb is dim, whereas the other bulb is of normal
brightness.
primary coil
secondary coil Conclusion
bulb 1 The primary voltage of 1 V (not high enough to
give normal brightness to the bulb) has been
Figure 3.66
changed to a higher voltage (2.5 V to give
Procedure normal brightness to the second bulb). The
1 10 turns of copper wire are wound on a soft iron transformer built is a step-up transformer.
C-core. The wire is connected to the 1 V 2 The number of turns on the secondary coil is
terminals of a low voltage power supply. more than the number of turns on the primary
2 25 turns of copper wire are wound on another coil. F
soft iron C-core. O

3
R

CHAPTER
M

Activ To investigate the relationship between the primary voltage (Vp),


ity 3.8 secondary voltage (Vs), number of turns in the primary coil (Np)
and number of turns in the secondary coil (Ns ) SPM
’07/P1

Apparatus/Materials at 2 V. The number of turns on the secondary


Low voltage a.c. power supply, two laminated coil, (Ns) is changed from 20 to 40 turns in steps
C-core, PVC covered s.w.g. 26 copper wire, two a.c. of 5 turns. The switch is turned on and the
voltmeter and C-clip. corresponding secondary voltages (Vs) are read.
Arrangement of apparatus Ns Vs
3 The ratios N and V are calculated and
p p
a.c. voltmeter
1 2 3
tabulated.
0
Tabulation of data

low voltage Table 3.13


a.c. power
C-core
clip
supply Np Ns Ns Vs
(turns) Vp (V) (turns) Vs (V) Np Vp
1 2 3 primary coil 30 2.00 20 1.35 0.67 0.68
0
secondary coil
30 2.00 25 1.65 0.83 0.83
30 2.00 30 2.00 1.00 1.00
a.c. voltmeter 30 2.00 35 2.35 1.17 1.18
Activity 3.7 & 3.8

Figure 3.67 30 2.00 40 2.65 1.33 1.33


Procedure
1 The arrangement is set up as shown in Figure Observations
N V
3.67. From Table 3.13, the ratios –––s and –––s are equal for
2 The number of turns on the primary coil (Np) is Np Vp
fixed at 30 and the primary voltage (Vp) is fixed every set of Np, Vp, Ns and Vs.

471 Electromagnetism
Graph
Vs is plotted against Ns, the graph obtained is as follows:
Vs

Ns

Figure 3.68

This shows that, Vs is proportional to Ns. The secondary voltage can always be increased by increasing the
number of turns on the secondary coil.
Conclusion
V Ns
From the activity, we can conclude: –––s = –––.
Vp Np

F
O
Relationship between Vp , Vs , Np and Ns
3

R
CHAPTER

M
1 For an ideal transformer (efficiency of 100%), 2 If the primary (input) voltage, Vp and the
5 the relationship between the voltage and the number of primary turns, Np is fixed, then:
number of turns is as follows. Vs ∝ Ns
Number of That is, the larger the number of secondary
Secondary voltage secondary turns turns is, the higher the secondary (output)
=
Primary voltage Number of voltage will be.
primary turns 3 Every turn on the secondary coil has an e.m.f.
induced in it. The sum of the e.m.f. across
Vs Ns every turn on the secondary coil is the
=
Vp Np secondary (output) voltage.
∴ Vs ∝ Ns
Or
4 This principle is used to produce ‘multi-tap’
transformers, auto transformers and variable
Voltage ratio, V = Turns ratio, N
transformers.

Two main types of transformers:

Step-up transformer Step-down transformer

symbol symbol symbol


Activity 3.8

Ns > Np Ns < Np
⇒ Vs (output) > Vp (input) ⇒ Vs (output) < Vp (input)

Electromagnetism 472
Type Design Explanation
‘Multi-tap’ transformer • Normally used in the laboratory to supply low voltage
power. For example, terminal 0 and 2 are used if a 2 V
supply is required.

Auto transformer • The voltage of the electricity supply in Malaysia is 240 V.


In other countries such as Japan, Taiwan and USA, the
electricity supply voltage is 120 V. For this reason, auto
transformers must be used with electrical equipment
exported from Malaysia to the countries mentioned.

Variable transformer brush • The output voltage can be varied from 0 V to 240 V by
changing the position of the brush. This type of
transformer is used to control the brightness of lamps on
stage or to control the speed of motors.
F
O

3
R

CHAPTER
M
12 14
5
A fluorescent lamp requires a voltage of 12 000 V to Figure 3.69 shows a closed soft iron core with 4 coils
ionise the mercury vapour in its tube. What is the ratio wound onto it. The number of turns in each coil is
of the number of turns on the secondary coil to the shown.
number of turns on the primary coil of the transformer
used if the electricity supply voltage is 240 V? 60 turns
S
Solution
Ns V
= s P
Np Vp
20 turns 80 turns
= 12 000 = 50
240 1 R
Ns : Np = 50 : 1
Q
40 turns

Figure 3.69
13 SPM
’05/P1

A transformer is used to lower the supply voltage What is required to be done if the transformer is to be
from 240 V to 3 V for use with a 3 V lamp. If the used as a transformer to step up the voltage of an
number of turns on the primary coil is 1600 turns, alternating current supply four times?
what is the number of turns on the secondary coil?
Solution Solution
Vs N
= s Connect the coil P to the power supply and coil R to
Vp Np
the appliance or the resistor.
3 Ns
= Ns
240 1600 = 80 = 4
Np 20
3 × 1600
Ns = = 20 turns
240

473 Electromagnetism
Efficiency of a Transformer 16
1 The efficiency of a transformer is normally less A transformer is used to lower the electrical supply
than 100%; that is, the power of the output is voltage of 240 V to supply power to a bulb with a rated
less than the power of the input. power of 12 V, 2 A. If the current in the primary coil is
2 A transformer transfers electrical energy from 0.15 A, what is the efficiency of the transformer?
one circuit to another circuit by electromagnetic
Solution
induction. In the process, a fraction of the
electrical energy is lost as heat energy. Po
Efficiency = × 100%
3 Formula for the efficiency of a transformer: Pi
Output: Po = Vs Is
Useful output power = 12 × 2
Efficiency = × 100%
Input power = 24 W
Po Input: Pi = Vp Ip
= × 100% = 240 × 0.15
Pi
= 36 W
24
Efficiency = × 100%
Ideal Transformer 36
F = 67%
O
1 If there is no loss of energy in a transformer, all
3

R the energy supplied to the primary coil will be


transferred to the secondary coil.
CHAPTER

M 17
2 Such a transformer is known as an ideal
5
transformer which has an efficiency of 100%. A transformer is used to supply power to two 12 V,
24 W bulbs which are connected in parallel.
∴ Output power = Input power The voltage of the electrical supply is 240 V and the
Po = Pi efficiency of the transformer is 96%.
Vs Is = Vp Ip Calculate
(a) the number of turns on the secondary coil if the
number of turns on the primary coil is 900,
(b) the current in the primary coil so that the bulb is
15 lit normally.
Figure 3.70 shows an ideal transformer used to Solution
supply power to two 12 V, 24 W bulbs at their rated Vs N
power. (a) = s
Vp Np
Ns Vs = 12 V because the
12
= bulbs are connected in
240 900 parallel.
12 × 900
Ns =
240
= 45 turns
Figure 3.70 Po
(b) Efficiency = × 100%
If the voltage of the electricity supply is 240 V and Pi Two 24 W
bulbs are
the current in the primary coil is 0.2 A, what is the Output power, Po = 48 W connected
current in the secondary coil? Input power, Pi = 240 × Ip in parallel.

Solution 48
96% = × 100%
240Ip
Vs Is = Vp Ip
because voltage across 48 × 100
Vs = 24 V each bulb is 12 V. Ip =
24 × Is = 240 × 0.2 96 × 240
Is = 2A = 0.21 A

Electromagnetism 474
SPM
Factors that Affect the Efficiency of a Transformer and ’09/P1

Ways to Improve the Efficiency of a Transformer

Factors that cause energy losses in a transformer Ways to minimise energy losses
1 Resistance of coil Thick copper wire is used to reduce the
A coil has electrical resistance. Energy is lost as heat energy in resistance of the coil.
the coil because of the heating effect of current flowing in a
conductor. The power loss is equal to I 2R, where R is the
resistance of the coil and I is the current flowing in the coil.
2 Magnetisation and reversal of magnetisation of iron core A soft iron core which can be
When current flows in the primary coil, the iron core is magnetised and demagnetised easily is
magnetised. When the current direction is reversed, the used to reduce the energy needed to bring
magnetisation of the iron core is reversed. The energy required about the magnetic reversal in the coil.
for the magnetisation and its reversal is converted into heat
energy in the iron core.
3 Eddy currents in iron core SPM
The core itself is a conductor, so the changing ’07/P1

magnetic flux in the iron core induces currents


in it. These circulating eddy currents generate
heat in the iron core. F
O

3
The iron core is laminated (layered). It is R

CHAPTER
made from thin, insulated sheet of iron M
(insulated with enamel paint) to increase
the resistance to the flow of eddy 5
currents. With less eddy currents, energy
loss is reduced.
4 Leakage of magnetic flux To overcome this problem, the iron core
Electrical energy is lost when a fraction of the magnetic flux should be a closed loop of iron with the
produced by the primary coil does not link with the secondary secondary coil wound over the primary
coil. As a result, there is a reduction in e.m.f. induced in the coil or placed very close to the primary
secondary coil. coil (along the same core).

18
Figure 3.71 shows an arrangement of a step-down (c) The brightness of the lamp is diminished after the
transformer which consists of two iron C-cores. two pieces of iron C-cores are separated. Give an
A 3 V lamp is used and the supply voltage is 240 V. explanation for this.
Solution
iron C-core (a) Ratio of turns = Ratio of voltage
Ns Vs
∴ =
Np Vp
Np NS
Vs 3 1
Vp = 240 = 80
Figure 3.71 Ns : Np = 1 : 80
(b) The iron core is magnetised when the transformer
(a) What is the ratio of the number of turns on the is in operation.
secondary coil to the number of turns on the (c) Leakage of flux occurs. A large portion of the
primary coil — i.e. Ns : Np? magnetic flux produced by the primary coil does
(b) A force is required to separate the two pieces of not link with the secondary coil. Both the voltage
iron C-cores. Explain why. across the lamp and the current flow are reduced.

475 Electromagnetism
6 SPM
Clone 7 SPM
Clone
’08 ’11

The bulb in the transformer shown in the diagram The table shows four wires with their corresponding
lights up at normal brightness. resistances. The four wires have the same thickness.
Wire Resistance (Ω)
A 0.06
B 1.72

What will happen if the a.c. input is replaced by a 3 C 2.00


V battery? D 3.80
A The brightness of the bulb remains unchanged.
B The bulb does not light up. Which wire, A, B, C or D is most suitable to be used
C The brightness of the bulb increases. as the coils in a transformer to increase the efficiency
D The bulb blows. of the transformer?
Comments Comments
The transformer does not function when the input One cause that lower the efficiency of a transformer
current is a steady direct current. is the heating of the coil in the transformer. The coils
F
A 3 V battery supplies steady direct current. The should be of low resistance so as to reduce the heat
O
transformer therefore does not work if the a.c. input generated in the coil and thus increases the efficiency
3

R
is replaced by a 3 V battery. of the transformer.
CHAPTER

5
Answer B Answer A

8 SPM
Clone
’07

The diagram below shows a simple transformer with (d) A television, which operates on direct current, is
two coils as shown. connected to the output of the transformer.
(i) The television does not function. Explain why.
soft iron core
(ii) An electrical component is to be connected to
the output of the transformer so that the
television works. Name the electrical
input
240 Va.c.
output component and state how the component is to
6 Va.c.
be connected.
secondary coil
primary coil
Solution
(a) (i) Name the type of the transformer. (a) (i) Step-down transformer.
(ii) Explain why soft iron is used as the (ii) It is easy to magnetise and to reverse the
transformer core. magnetism in the core.
(b) Calculate the number of turns on the secondary Ns V
(b) = s
coil if the number of turns on the primary coil is Np Vp
1200. Ns
6
(c) The secondary coil is connected to an electrical a =
240 1200
appliance. The current in the primary coil is
0.09 A and the efficiency is 80%. Ns = 30 turns
(i) Find the output power of the transformer. (c) (i) Input power
(ii) How would you modify the transformer if you = 240 × 0.09
wish to supply 20 W of power to the electrical = 21.6 W
appliance?

Electromagnetism 476
Useful output power (ii) Replace the core with laminated soft iron
Efficiency = × 100%
Input power core.
Useful output power (d) (i) Television uses direct current only.
80% = × 100% (ii) Diode. Connect the diode in series with the
21.6
television.
Useful output power
80
= × 21.6 = 17.3 W
100

The design of a transformer can also influence its efficiency. A well designed transformer has a higher efficiency.
In practice, transformers with efficiencies up to 99% can be obtained.
The diagram below shows two arrangements of the coils and iron core in a transformer with a very high efficiency.

secondary coil
secondary coil
F
O
Physics Blog

3
R
primary coil

CHAPTER
soft primary coil M
iron core
5

3.4

1 An electric door bell for the home uses a 9 V supply If the power dissipated by the resistor is 4.5 J s–1,
to function. The voltage of the electrical supply is calculate
240 V. What is the turns ratio of the transformer that (a) the voltage across the resistor,
must be used? (b) the current flowing in the primary coil.
2 An ideal transformer steps down a voltage of 240 V 5 A transformer with an efficiency of 90% is used to
to supply power to a lamp with a power rating of 3 V, supply power to two lamps P and Q with power ratings
0.3 A. If the brightness of the lamp is normal, what is of 12 V, 12 W and 12 V, 24 W respectively in normal
the current flowing in the primary coil? lighting. The voltage of the power supply is 240 V.
3 Two identical lamps of power rating 12 V, 24 W have (a) Should P and Q be connected in series or in
the same brightness when power is supplied by a parallel?
transformer as shown in the diagram below. (b) What is the current flowing on the secondary
coil?
(c) What is the power input supplied?
secondary
6 An ideal transformer is coil
used to lower the electrical
voltage from 240 V to supply power to 5 light bulbs
of power rating 12 V, 24 W in normal lighting.
primary iron core p
If the efficiency of the transformer is 80%, what is the The bulbs are connected in coil c
current flowing in the primary coil? (a) parallel, and
(b) series.
4 The transformer in the diagram below has an
(i) What is the ratio of the number of turns on
efficiency of 90%.
the secondary coil in the arrangement (a) to
its number of turns in arrangement (b)?
(ii) Find the current flowing in the primary coil in
the arrangement (b).

477 Electromagnetism
3.5 Understanding the Generation and Transmission of Electricity

Electrical Energy Sources of Energy


1 The consumption of electrical energy is very
large. It has become a necessity in our daily life. Non-renewable Renewable
2 Electrical energy must be generated as it does resources resources
not occur naturally (except during lightning). • Fossil fuels: • Hydropower
It is generated from other sources of energy. coal, petroleum • Biomass
3 Sources of energy can be divided into two and natural gas • Solar energy
classifications; non-renewable resources and • Nuclear fuel • Wind power
renewable resources.

Generation of Electricity from Various Sources


F
O Thermal Power Stations using Fossil Fuels
3

R
CHAPTER

5 stack

F
O steam
3

R turbine
generator
CHAPTER

steam water
5 boiler
pump
heater
cooling
pump
tower
cooling water

Figure 3.72 Thermal power station


Production process
(a) Fuels such as coal, petroleum and natural gas are burned in a furnace to produce heat used to boil water in a
boiler. Steam at high pressure is formed.
(b) The high-pressure steam drives a turbine which produces the rotary motion needed to run alternating current
(a.c.) generators. That is, heat energy is converted into kinetic energy which in turn is converted into electrical
energy.
(c) Not all of the energy produced in the turbine is converted into electrical energy. Only 30% of the heat energy
input is converted into electrical energy output.

Effects on the environment TC 3-134


(a) Smoke, dust and toxic gases such as sulphur dioxide and nitrogen oxides which pollute the air are produced.
(b) Excessive carbon dioxide in the atmosphere causes the greenhouse effect which gives rise to higher
temperatures in the environment (global warming).
(c) Acid rain due to the solution of sulphur dioxide and nitrogen oxides in water has an adverse effect on the
equilibrium of the earth’s ecosystem.

Electromagnetism 478
Hydroelectricity

dam
Production process
reservoir transformer
powerhouse
(a) A hydropower station uses the kinetic energy of
falling water to produce electrical energy.
power lines (b) Water is stored behind a massive wall of concrete
generator
which retains water in a valley to a high level—
called a dam.
(c) The water is then released and flows downhill
through turbines which drive electrical
generators. Gravitational potential energy is
changed to kinetic energy, and then to electrical
energy.

Effects on the environment


intake
control gate
turbine outflow The construction of a dam disturbs the equilibrium of
the ecosystem in the environment because a large
Figure 3.73 Hydropower plant area of forest land has to be destroyed to build the
F5/3/55
dam. Also, the flow of a river is affected.
F
O

3
R

CHAPTER
M

Solar Energy 5

Solar energy is energy from the Sun. Solar energy is Effects on the environment
converted into electrical energy by two methods: The facility for collecting the Sun’s rays to converge
in a solar furnace and in solar cells (photocells). to a solar furnace or onto solar panels requires a large
area.

Solar Furnace Solar Cells (Photocells)


solar cells

+ solar radiation module


solar cells
+ solar ray reflected

furnace
target area array
hill reflecting surface
tower

parabolic mirror
bank of heliostats

Figure 3.75 Solar cells as solar thermal collector


Figure 3.74 Solar furnace
(a) A solar cell consists of a semiconductor material
such as silicon which is able to convert light
A solar thermal power station
F5/3/56has a large parabolic
energy into electrical energy.
mirror to focus the Sun’s rays to a solar furnace.
b) Light energy releases electrons from the silicon
The heat energy is used to heat water to produce
atoms. The free electrons produced flow in the
steam whose kinetic energy is used to drive the
material and current is produced.
turbine of an electrical generator.
(c) Solar cells are largely used in wrist watches, lamps,
calculators and water heaters for domestic use.
More complex photocells are used in satellites.

479 Electromagnetism
Nuclear Energy

Effects on the environment


(a) The fuel used in a nuclear reactor cannot be easily
turbine and safely stored. Handling of radioactive waste
steam requires skillful management.
nuclear
reactor generator
(b) Any occurrences of leakage in a nuclear power
station will contaminate the atmosphere with
dangerous radioactive radiation, causing death to
humans and animals. Radioactive wastes released
to the atmosphere are spread by the wind and is a
pump cooling threat to health.
uranium tower
(c) The cost of constructing a nuclear reactor is very
Figure 3.76 Nuclear energy plant high because of the technology which needs to be
acquired and the necessary human resources to
Production process
F5/3/57 manage the facility.
(a) In a nuclear reactor, the fission of a uranium atom
is carried out at a controlled rate.
(b) The heat energy released in the fission process is
F
used to produce high-pressure steam. The high-
O pressure steam is used to drive a turbine that will
3

R then rotate a generator as in a thermal power


CHAPTER

M station.
5
Refer Chapter 5, Form 5

F
Wind Power O
3

R
CHAPTER

M Production process
5
(a) Wind turbines are used by farmers in windy
areas.
(b) A wind turbine consists of two or three blades
connected to the rotor of an electrical generator.
(c) The wind turns the blades, which spin a shaft,
which connects to a generator and create
electricity.
(d) A wind turbine with a blade of 50 m can produce
electrical power up to 1 MW.

Effects on the environment


(a) The construction of wind turbines requires a large
area near the peaks of mountains or along the
beaches. Forests have to be cleared for the wind
farms.
(b) The rotation of the turbines produces noise.

Figure 3.77 Windmill


F5/3/58

Electromagnetism 480
Energy from Biomass

Biomass is organic material from plants or waste matter of animals. Biomass can be directly
converted into a liquid fuel
electricity CO2 for use in motor vehicles.
A commonly used biofuel
tree plantation is ethanol and bio diesel.
In Brazil, sugar cane is used
to produce ethanol for use
power plant
as fuel in motor vehicles.
In Malaysia, PORIM has
succeeded in producing
animal waste
diesel from palm oil.

truck transport

Figure 3.78 Biomass plant

Production process
(a) Energy obtained from plant materials such as wood or waste matter from
animals is called biomass energy. F
(b) The decaying process by the action of bacteria on plant and animal waste O

3
produce biogas such as methane which can be used as fuel for the R

CHAPTER
M
production of electrical energy.
(c) Burning of biomass produces heat energy to generate electrical energy. 5

Effects on the environment


(a) Biomass must be kept in designated areas to avoid the spread of foul
smells from affecting the health of residents.
(b) Management of a biomass facility must be suitably qualified in science
and technology.

Electricity Transmission SPM SPM Model of an Electricity Transmission System


’05/P2(C) ’07/P1

wooden dowel rod 1.5 m constantan


1 Electricity generated in a power station is with 2 terminals wire (s.w.g. 28)
distributed to industries and residential areas
by electrical cables.
2 The electrical resistance of the cables causes power
supply
heat to be produced in the cables during
transmission.
3 The power loss is equal to I2R, where I is the lamp A lamp B
12 V, 24 W 12 V, 24 W
current flowing in the cable and R is the
resistance of the cable.
4 If a town is located at a great distance from Figure 3.79 Model of electricity transmission
the power station, large power losses are 1 Figure 3.79 shows a model of an electricity
incurred. transmission system.
5 To overcome this problem, electrical energy is 2 Two lengths of s.w.g. 28 constantan wire
transmitted at a high voltage, to reduce the (representing the electrical cable) are straightened
current and therefore reduce the I2R loss. to eliminate any kinks along the wire.

481 Electromagnetism
3 The lamp A represents a consumer of electrical power at the power station
and the lamp B represents a consumer at a village some distance away.
It is observed that the lamp A is lit brighter, whereas the lamp B is dim. Advantage of using
4 The brightness of the lamp B does not vary much when the lamp A is alternating current in the
removed. transmission of electricity:
5 Energy is lost during the transmission of electricity to the lamp B. Alternating current is used
6 Two transformers are added to the circuit as shown in Figure 3.80. in the transmission of
P is a transformer with a turns ratio of 1 : 20, and Q is a transformer electrical energy at a high
with a turns ratio of 20 : 1. voltage because its voltage
can be easily increased or
1.5 m constantan decreased with transformers.
wire (s.w.g. 28)
lamp A step-up step-down A transformer cannot
12 V, 24 W transformer transformer
(1:20) P (20:1) Q
lamp B function with a direct current.
12 V, 24 W
240 V

power
supply
coil coil coil coil
120 turns 2400 turns 2400 turns 120 turns

Figure 3.80 Model of electricity transmission with the use of transformers


F
7 The voltage is raised from 12 VF5/3/154to 240 V before transmission of
O
electricity through the model cable. At the other end of the
3

R
transmission cable, the voltage is reduced to 12 V again.
CHAPTER

M
8 It is observed that the brightness of both lamps A and B are practically the
5 same in this arrangement.
9 The two models described above clearly show the effects of electricity
transmission at a low voltage and high voltage.
10 At a high voltage, the current in the cable is reduced and therefore the
energy loss as heat in the cable is also reduced.
F
11 Transformers play an Oimportant role in the transmission of electricity
3

at a high voltage. R
CHAPTER

5
19
(a) Find the energy loss in a transmission cable when 20 kW is transmitted through a cable of 1.5 Ω
(i) at a voltage of 200 V,
(ii) at a voltage of 10 kV.
(b) What is the effect of the energy losses in case (a)(i) and (ii)?
Solution
(a) (i) Use the formula for power, P = IV. (ii) At voltage V = 10 kV:
At voltage V = 200 V: P
Current in cable, I =
P V
Current in cable, I =
V 20 000
=
20 000 10 000
=
200 = 2A
= 100 A Energy loss in cable = I 2R
Energy loss in cable = I 2R = 22 × 1.5
= 1002 × 1.5 =6W
= 15 000 W (b) It can be seen that by raising the voltage by a factor
= 15 kW of 50, the current is reduced by 50 times. As a result,
the energy loss is reduced by a factor of 2500!

Electromagnetism 482
9 SPM
Clone
’06

A transformer is used in an electrical energy


transmission to
V2 A reduce the resistance
Students use the formula, Ploss = , to find the
R
energy loss in a cable. B change the potential difference
C increase the output power
D reduce the time for energy transmission
Students must use the formula P = I 2R as the current,
I in the cable is known. The value of V, the potential Comments
difference across the ends of the cable is not known. The function of a transformer is to change the
The voltage for the transmission potential difference. A step-up transformer is used to
= Voltage across the cable (to and fro) step up the potential difference in the power station
= + Input voltage for the transformer at the other end before transmission via the national grid.
Answer B

National Grid Network SPM


’08/P1
SPM
’09/P1 F
O

3
1 The national grid is a network of electrical cables connecting electrical power stations to R

CHAPTER
consumers of electricity. M
2 Figure 3.81 shows the national grid network in Malaysia.
5
cable 132 kV pylon
cable 33 kV
(to relay station)
power station

25 kV kV 33 kV
2
13
step-down heavy industry
transformer sub-station
33 kV
25 kV
commuter train step-down
transformer
sub-station

33 kV
11 kV
light industry
step-down
transformer
sub-station
33 kV
240 V

office, house, school


step-down transformer
sub-station
Figure 3.81 National grid network in Malaysia

3 Electrical energy generated in power stations at at sub-stations, to suitable voltages of the


a voltage of 25 kV is increased to 132 kV with a respective consumers as shown in Table 3.14.
step-up transformer before transmission via Table 3.14
the national grid.
4 Electrical energy is transmitted at a high voltage Voltage Consumer
to reduce energy loss as heat in the cables. 33 kV Heavy industries
5 Before reaching the various consumers, the 11 kV Light industries
voltage is reduced, with step-down transformers
240 V Residential

483 Electromagnetism
Advantages of the National Grid Network 2 Greater research efforts must be made to find
alternative sources of fuel which are renewable
1 Savings and increased efficiency so that the efficiency of electrical energy
Energy loss, as heat in transmission cables, is generation can be increased besides reducing
reduced if the energy is transmitted at a high the cost of production.
voltage. The cost of production is reduced and 3 Examples of renewable energy sources are
the efficiency of transmission is increased. solar energy, wind power and biomass.
2 Efficient energy distribution 4 Renewable energy sources are beneficial to
The generation and distribution of electrical mankind. Most importantly, these energy
energy can be controlled according to sources are natural, clean, and they do not
requirements. Power stations in regions where pollute the environment.
the requirement of electrical power is low, can 5 Use of renewable energy sources gives an
be used to supply power to regions where the assurance of the continuation of mankind’s
demand for electrical power is high. existence because these energy sources can be
3 Energy management replenished.
If there is a break down or interruption of 6 Ocean wave energy, geothermal energy, tidal
supply at a particular power station, power can energy and heat energy from the sea are some
be supplied by other stations through the of the forms of energy which are worth
national grid network to the affected area. researching and developing.
F 7 Although there are many renewable energy
O Importance of Renewable Energy sources such as the wind, sun, hydropower
3

R and wave, their use have not been extensive


CHAPTER

M 1 Non-renewable sources of energy such as because the availability of these sources are
fossil fuels are not replaceable and the cost of influenced by geographical location and
5
production is increasing. It has been estimated weather conditions. Further research into these
that these sources will be depleted in the near sources must be encouraged.
future. Dependence on these sources alone
will lead to an energy crisis.
F
O
3

R
CHAPTER

510
SPM
Clone
’05

A model of an electrical transmission system is set up in laboratory as shown in the following diagram.

transmission wire
at 24 V a.c.
transformer Q
transformer P

X
factories
at 8 V a.c.
power station transformer R
at 12 V a.c.
Y

houses
at 6 V a.c.

The power station generates an alternating current at 12 V and the system transmits the electrical energy
to factories (represented by a lamp X) and houses (represented by a lamp Y) via the transmission wires
and transformers P, Q and R. F5/3/156

Electromagnetism 484
The table below shows components needed to make the model.

Number of turns Type of transformer Material of the


of the coil core transmission wire
120 Solid soft iron core Nichrome
160 Laminated soft iron core Copper
400 Laminated copper core Aluminium
500 Laminated aluminium core Constantan
1000 Laminated steel core
1200

You are asked to set up a model by using the Vs N


information in the diagram and table. For transformer Q: = s
Vp Np
Determine
8 Ns
(a) the number of turns in the primary coil and =
secondary coil for transformers P, Q and R. You 24 Np
need to show your working. Ns : Np = 1 : 3
(b) the type of core you will use in all the transformers Therefore, Ns = 400 while Np = 1200. F
and justify your choice. V N O
(c) the material you will use for the transmission wire For transformer R: s = s

3
R
and justify your answer. Vp Np

CHAPTER
M
Answer 6 Ns
= 5
(a) First, you need to find the turns ratio using the 8 Np
Vs Ns Ns : Np = 3 : 4
formula V = N .
p p Therefore, Ns = 120 turns while Np = 160 turns.
Vs N (b) The suitable core to be chosen is the laminated
For transformer P: = s soft iron core because it reduces the effect of eddy
Vp Np
currents.
24 Ns (c) Copper wire is chosen to be the transmission wire
=
12 Np because of its low resistance. Since the heat
Ns : Np = 2 : 1 produced in the transmission wire is proportional
to resistance, lower resistance will reduce the loss
Therefore, Ns = 1000 while Np = 500.
of energy in the transmission of energy.

3.5

1 A 50 kW electrical generator is connected to a factory 3 A power station supplies 1.6 × 106 W of electrical
by a cable with a total resistance of 5 ohm. If the power to a small town, situated 8 km away, through a
transmission voltage is 5000 V, transmission cable with a resistance of 5 × 10–1 Ω
(a) what is the current in the cable? per kilometer. If the power is transmitted at a voltage
(b) what is the maximum power received by the of 20 kV, what is the total energy loss in the cable?
factory?
2 A small power station supplies 1650 kW at a voltage 4 A power station supplies 2 × 106 W of electrical
of 66 kV to a factory. The transmission cable has a power to a town. If 0.5% of the power is lost as heat
resistance of 40 ohm. Calculate in the cable with a resistance of 8 Ω, what is the
(a) the current in the cable, transmission voltage?
(b) the energy loss in the cable,
(c) the transmission efficiency.

485 Electromagnetism
1. A magnetic field is a region in which a magnetic induced current is directly proportional to the rate of
material experiences a force. change of magnetic flux.
2. A current-carrying conductor produces a magnetic 9. Lenz’s Law states that an induced current always
field around it. The right-hand grip rule is flows in such a direction so as to oppose the change
used to determine the direction of the magnetic causing it.
field. 10. The direction of induced current can be determined
3. The right-hand grip rule is used to determine the by using Fleming’s right-hand rule.
magnetic pole of a current-carrying solenoid. 11. The induced current in a conductor increases when
4. The strength of an electromagnet increases when (a) the speed of conductor increases,
(a) the current increases, (b) the strength of magnetic field increases and
(b) number of turns on the solenoid increases. (c) the length of the conductor increases.
5. The direction of motion of a current-carrying 12. Voltage ratio = Turn ratio
conductor in a magnetic field can be determined by VS NS
using Fleming’s left-hand rule. VP = NP
6. The force on a current-carrying conductor increases
when 13. Efficiency of a transformer
(a) the current increases, Power output
F = × 100%
(b) the strength of magnetic field increases and Power input
O
3

R (c) the length of the conductor increases. 14. Alternative current is used in the transmission of
7. The production of an electric current by a changing electrical energy at a high voltage because its voltage
CHAPTER

M
magnetic field is called electromagnetic induction. can be easily increased or decreased by using
5 8. Faraday’s Law states that the magnitude of the transformer.

3
O
3

R
CHAPTER

Multiple-choice Questions

3.1 C copper A Fleming’s left-hand rule


Magnetic effect of a Curent- soft iron B Fleming’s right-hand rule
carrying Conductor C Right-hand screw rule
1 Which of the following D Right-hand grip rule
arrangements forms the D copper
electromagnet with the greatest alumminum 3 Diagram 2 shows an arrangement
magnetic field strength? Assume SPM
of apparatus to investigate the
Clone
’04
the lengths of all the wires and the pattern of magnetic fields due to a
core are the same, and all are 2 Diagram 1 shows a solenoid with current in a wire.
connected to the same power SPM
its poles as indicated.
Clone
source. ’07
A copper
N S
copper

cardboard

B soft iron Diagram 1


Diagram 2
soft iron Which of the following rules can
F5/3/22b
be used to find the direction of the Which of the following magnetic
current in the solenoid? field patterns is correct when both
1.1 Significant Figure
Electromagnetism 486
the switches in the diagram are supplying current at frequency of 9 A short copper rod is placed on
closed? 50 hertz. What would be observed? two horizontal stiff copper wires
A A P will not attract paper clips. connected to a low voltage direct
B P will attract less paper clips. current power supply as shown in
C P attracts and repels paper Diagram 7.
B clips 50 times per second.
D The paper clips will fly between
P and Q 50 times per second. C
C
6 Which of the following actions will B

not increase the speed of a direct A

current motor?
D

D
A Wind the coil onto a copper Diagram 7
core.
B Use a stronger magnet. In which direction should a
4 A straight wire passes through the C Use a higher current. magnetic field be directed in order
centre of a squared-shaped D Use a coil with more turns. to produce maximum force on the
cardboard. A compass is placed at short copper rod?
the position P as shown in 7 Diagram 5 shows an electron
Diagram 3. beam moving along the axis of a 10 Which of the following rules is
solenoid. used to determine the direction of
the force on a current-carrying
N conductor? F
P
A Fleming’s left-hand rule O

3
B Fleming’s right-hand rule R

CHAPTER
C Right-hand grip rule M
thick D Maxwell’s screw rule
cardboard 5

rheostat
Diagram 5
Diagram 3 3.3
What would happen to the beam Electromagnetic Induction
In what direction will the N pole of when the switch is closed?
11 Diagram 8 shows a conductor
the compass point to when a large A The beam of electron will SPM placed in between two Magnadur
current is allowed to flow in the deflect upwards. Clone
’09 magnets with unlike poles facing
circuit? B The beam is not affected and
each other. The conductor is
A South-east C North-west still moves along the axis.
connected to a galvanometer.
B North-east D South-west C The beam of electron will
deflect downwards. P
X

8 Diagram 6 shows a coil with the


3.2 Force on a Current-carrying commutator placed in a pair of
Conductor in a Magnetic magnet.
Field W V

5 Diagram 4 shows an iron core PQ


with a solenoid connected to a
d.c. souce. N S Q
Y

Diagram 8
brush
commutator Which motion will produce 1.1 S
Diagram 6 an induced e.m.f. in the
conductor?
The structure will work as a motor
F5/3/62
A Moving the conductor along WV.
if a student
B Moving the conductor along PQ.
A rotates the coil.
C Moving the conductor along XY.
B connects the brushes to a dry
Diagram 4 D Moving the conductor and
cell.
magnets together along PQ.
Some paper clips are attracted by C connects the brushes to a low
P as shown. The source is then voltage alternating current 12 A ring is moved towards and
1.1 Signifi
power cant Figure
unit. passes around a bar magnet
replaced with a 3V a.c. source

487 Electromagnetism
which is firmly fixed in position as into the solenoid from the same 17 Diagram 13 shows a step-down
shown in Diagram 9. distance, with the same speed transformer with coils P and Q as
into the coil as shown in Diagram shown.
S N 11(b), what will be observed on
observer the galvanometer?
Diagram 9 A More deflection to the right. resistor
P Q
B No deflection.
What is the direction of the
C More deflection to the left.
flow of current in the ring to
the observer? 15 Which of the following will Diagram 13
A Always flows anticlockwise. produce greatest induced current? What would happen to output
B Always flows clockwise. Given all the Magnadur magnets voltage and its frequency if another
C Flows anticlockwise then are identical and the rods are all similar coil of Q is connected in
clockwise. move with the same velocity. series with coil Q but the power
D Flows clockwise then A copper C soft iron supply to P is keep constant?
anticlockwise.
Output voltage Frequency
N S N S
13 Diagram 10(a) shows an a.c. A Doubled Doubled
dynamo with the coil rotating in
B Doubled Halved
clockwise direction.
B copper
copper
D aluminium C Halved Doubled
N S D Doubled Same
F
O NN SS N S
3

R 18 The input voltage and current for


(a) an ideal transformer as shown in
CHAPTER

M
I Diagram 14 are 240 V and 0.3 A.
5 soft iron
Q 0.3 A
R
P
t (s) N S
T 3.4 Transformer 240 V
P
S
(b) 16 Diagram 12 shows a step-up
F
Diagram 10 transformer
aluminium with a turns ratio of Diagram 14
O 1 : 2. A lamp with a power rating
What is the energy transferred to
3

Which of the point or points


R on of 12 V, 6 W and an alternating
the graph I – t, in DiagramM N S the lamp P in 10 s?
CHAPTER

current power supply of 6 V are


10(b), correspond to the times A 7.2 J C 720 J
connected to the transformer.
5
when the coil is in horizontal B 36 J D 1600 J
position? 19 What is the type of wire used as
A P only C Q only
Clone the coils and the function of a
SPM

B P, R and T D Q and S only ’09 laminated core used in a

14 When a bar magnet is plunged transformer?


into a solenoid as shown in Type of Function of
Diagram 11(a), a deflection to the Diagram 12
wire laminated core
right is observed on the Which of the following sets of
galvanometer. A Low To reduce eddy
information is true when the
resistance current
positions of the lamp and the
N S S N N
power S
supply are interchanged in B High To reduce eddy
the circuit? 1.1 resistance
Signifi current
cant Figure
? C Low To increase eddy
(a) (b) Potential resistance current
Condition
difference
N S S N N S of lamp D High To increase eddy
across the lamp
resistance current
A Brighter 3V
?
B Dimmer 3V 20 A step-down transformer with
(a) (b)
C Dimmer 6V an efficiency of 90% is used to
Diagram 11 lower the voltage of a supply
D Same 12 V from 240 V to 50 V. The
If two bar magnets, with the same
brightness current flow in the primary coil
magnetic strength, are plunged

Electromagnetism 488
is 2 A. What is the current in A Step-up transformer only 1 1
A C
the secondary coil? B Step-down transformer only 2 4
A 0.42 A C 2.6 A C Step-up and step-down 1 3
B 1.8 A D 8.64 A transformers B D
3 5
22 A power station supplies
electrical power of P kW to 23 12 kW of electrical energy is
3.5 Generation and a town at a potential difference transmitted through a cable with a
Transmission of Electricity of V kV. The loss in power resistance of 1 ohm at a voltage of
during transmission is Ph. 300 V. What is the total energy
21 Which type(s) of transformer(s) What would be the power loss, loss in the cable?
are used when power is in terms of Ph if the power is A 800 W C 3200 W
transmitted from the power transmitted at 2V kV ? B 1600 W D 4000 W
stations to the consumers?

Structured Questions
1 Diagram 1 shows a spring (i) To which points must the two bulbs
balance measuring the (connected in parallel) be connected so that
spring
weight of a bar magnet. A they are normally lit?
coil connected to a circuit is Show the relevant calculations. [2 marks] F
placed under the magnet. S
(ii) If the efficiency of the transformer is 90%, O

3
The reading on the spring bar magnet calculate the current in the primary coil. R

CHAPTER
balance is W N when the N [2 marks] M
switch is off. 1.1 Significant Figure 3 Diagram 3 shows a copper rod hung from a spring 5
(a) When the switch is
balance. The rod is connected to a circuit as shown.
turned on, the reading
on the spring is less
than W. Explain why. Diagram 1
10

20

[2 marks] 30

40
spring balance
(b) State two ways to reduce further the reading on
50

60

70

the spring balance. [2 marks] thread


(c) What will be observed on the spring balance
copper rod
when the current is reversed? [1 mark] S
X Y
2 Diagram 2 shows a schematic diagram of a multi-tap A
N
transformer.
T switch

40 turns Diagram 3
Table 1 shows four rods of different materials.
S The lengths and diameters of the rods are the same.
240 V
30 turns
Table 1
R
20 turns Rod Material
1000 Q
turns 10 turns P Copper
P
Q Aluminium
Diagram 2 R Lead
(a) Which of the points P, Q, R, S and T must be S Plastic
connected to produce
(i) the minimum output voltage? [1 mark] (a) When the switch is closed, the reading on the
(ii) the maximum output voltage? [1 mark] spring balance decreases. Explain.
(b) Give reasons for your choice. [1 mark] [2 marks]
(c) What is the ratio of the voltage across ST to the (b) Rods of other materials are used in turn in place of
voltage across RQ — i.e. VST : VRQ? [2 marks] the copper rod. The readings on the spring
(d) Two similar bulbs, each with a power rating of balance and the ammeter are recorded.
12 V, 24 W, are connected in parallel. Complete the following table.

489 Electromagnetism
Table 2 coil
bar magnet P Q
Reading on the spring balance Rod
S N S N
(i) Largest spring balance reading
before the switch is closed. Rod..............
smooth
(ii) Largest difference between runway

spring balance readings before


and after the switch is closed. Rod..............
centre-zero galvanometer
(iii) Same spring balance readings
before and after the switch is Diagram 4
closed. Rod.............. The galvanometer shows a deflection to the left
and then to the right.
[3 marks] (i) Draw in Diagram 4 the flow of current while
(c) The direct current source in Diagram 3 is replaced the magnet is moving towards it. [1 mark]
with an alternating current source of the same (ii) The velocity of the trolley after going through
voltage. What is the effect on the spring balance is less than that before entering. Explain.
reading when the switch is closed? Explain. [2 marks]
[2 marks] (b) (i) The runway is then placed beside the coil.
The trolley is given the same velocity.
F 4 (a) A bar magnet attached to a trolley is given a push
O Compare the magnitude of deflection in the
so that it moves with a constant velocity on a
3

R galvanometer with the deflection as in (a).


smooth runway towards and through a coil
[1 mark]
CHAPTER

M
connected to a centre-zero galvanometer as
(ii) Give an explanation for (b)(i). [2 marks]
5
shown in Diagram 4.

Essay Questions
5 wooden
wooden support
support
stripped
thick
stripped and stiff
ammeter copper ammeter copper
thick copper wire
rod rod
and stiff
copper
switch switch
wire Q
to d.c. power to d.c. power
supply Q
S Q supply S Q
P
P
iron P iron P
yoke N N
yoke
Magnadur Magnadur
magnet magnet
(a) (b)
Diagram 5
(a) (i) What is meant by magnetic field? [1 mark]
(ii) Using Diagrams 5(a) and (b), compare the
current and the maximum height reached cardboard rheostat
by the copper wire PQ.
P R
Relate the current with the maximum height aluminium
of the wire PQ to make a deduction foils
regarding the relationship between the Q

current and the force on the current carrying


conductor PQ in a permanent magnetic
field. [5 marks]
switch
(b) Diagram 6 shows an electrical circuit containing
aluminium foils PQ and RS. Diagram 6

Electromagnetism 490
Explain what happens to PQ and RS, when the switch is turned on. [4 marks]
(c) Diagram 7 shows a structure of a model of a moving coil meter.
magnadur magnet
armature
spiral spring
metal tube
for axle to
turn in
spiral spring

iron yoke
wooden frame
insulated copper wire

Diagram 6
You are required to give some suggestions to (a) the spiral spring,
design a moving coil meter with high sensitivity. (b) type of magnet,
Using the knowledge on electromagnetism, explain (c) number of turns on the coil and,
the suggestions based on the following aspects: (d) area of coil.
(i) the way to increase the sensitivity with (ii) the way to ensure a linear scale for the
F
respect to moving coil meter.
O

3
6 Diagram 8 shows a rectangular coil connected to a galvanometer moving from a uniform magnetic field to R

CHAPTER
another uniform but stronger magnetic field. M

P Q R Key : 5
Magnetic field
into the page

centre-zero galvanometer

Diagram 8

(a) (i) What is meant by an induced current? Calculate


[1 mark] (i) the number of turns on the secondary coil, if
(ii) When the coil is moving from position P the number of turns on the primary coil is
to Q, there is no deflection on the 900,
galvanometer. When the coil moves across (ii) the current in the primary coil so that the
from weaker field (position Q) to stronger bulbs are lit normally. [4 marks]
field (position R), the galvanometer shows a (c) Table 3 shows four transformers P, Q, R and S,
momentary deflection. with different specifications.
When the coil is moved from R to Q, the coil You are required to determine the most suitable
also shows a momentary deflection in the transformer to step down a voltage from 240 V to
opposite direction. 12 V.
Explain the above situations. Study the specifications of all the four transformer
[5 marks] from the following aspects.
(b) An ideal step-down transformer with 240 V power • the material of the coil
supply is used to lit up two 12 V, 6 W bulbs in • the design and type of core
series. • the turns ratio
The effeciency of the transformer is 96%. Explain the suitability of the aspects.
Justify your choice.

491 Electromagnetism
Table 3

Tranformer Number of turns on the coils Type of coil Design and type of iron core
iron core

Primary coil: 200


P Iron coil
Secondary coil: 20

laminated
iron core
Primary coil: 200 Copper
Q Secondary coil: 10

laminated
iron core
Primary coil: 200
R Copper
Secondary coil: 10

F
O
Primary coil: 300
S Aluminium iron core
Secondary coil: 25
3

R
CHAPTER

Experiment
1 A coil of 1200 turns is slipped over one arm of a laminated U-core. A length of PVC covered insulated copper
F
wire is connected to a bulb labelled 2.5 V, 0.3 A. The copper wire is then wound round the other arm of the
O
U-core as shown in Diagrams 1 (a) and (b).
3

R
CHAPTER

M bar-core bar-core

5 1200 1200
turns turns

mains mains

U-core U-core
dim bright
(a) (b)
Diagram 1

The bulb in (a) is dim while the bulb in In your description, clearly state the following:
(b) is bright. (i) The aim of the experiment
Based on the above informations and observations: (ii) The variables in the experiment
(a) Make one suitable inference. (iii) The list of apparatus and materials
[1 mark] (iv) The arrangement of the apparatus
(b) State one suitable hypothesis. (v) The procedure used in the experiment.
[1 mark] Describe how to control and measure the
(c) With the use of apparatus such as soft iron C–cores, manipulated variables and how to measure
low voltage a.c. power supply, a.c. voltmeters and the responding variables
other apparatus, describe an experiment to (vi) The way to tabulate the data
investigate the hypothesis stated in (b). (vii) The way to analyse the data [10 marks]

Electromagnetism 492
FORM 5

4
CHAPTER

Electronics

SPM Topical Analysis


Year 2007 2008 2009 2010 2011
Paper 1 2 3 1 2 3 1 2 3 1 2 3 1 2 3
Section A B C A B A B C A B A B C A B A B C A B A B C A B
Number of questions 4 – 1 – – – 4 1 – – – – 4 1 – – – – 1 1 – – – – 5 1 – – – –

ONCEPT MAP

ELECTRONICS

Thermionic emission Semiconductors Logic gates

Cathode rays Doping


Truth Boolean
table Symbol algebra

Cathode-ray p-type n-type


oscilloscope
(CRO) Functions
and
p – n junction working
principles
Structures Controls
and and
components uses
Diode Transistor Control system

pnp npn

Applications
• Amplifiers
• Automatic switches

COMPANION WEBSITE
Learning Objectives 493
4.1 Understanding the Uses of the Cathode-Ray Oscilloscope (CRO)

Thermionic Emission SPM


’08/P1

1 Thermionic emission is a process involving the


emission of electrons from a hot metal surface. Thermionic emission Evaporation
2 It is analogous to the evaporation of water. Enough energy must Enough energy must
3 A thermionic diode, as shown in Figure 4.1, is an be absorbed before be absorbed before
apparatus used to demonstrate thermionic electrons are emitted water molecules
emission. from a metal surface. escape from the water
surface.

A metal surface has many free electrons


which are bound to the surface because When the filament is heated, electrons are emitted from
they do not have enough kinetic energy to the surface of the filament. This is because some of the
break free. electrons have gained enough kinetic energy to break free
from the metal surface and escape to the surroundings.
electrons emitted from cathode

The negatively charged


tungsten filament anode electrons are repelled from the
(cathode) cathode and attracted to the
F
O anode.
4

R
+
CHAPTER

M
6V
5 d.c.

supply glass tube
The electrons that arrive at the
anode complete the 400 V d.c.
0 supply circuit. As a result, the
milliammeter
vacuum milliammeter registers a
reading.
thermionic diode

– +
Air is removed from the glass tube to create
400 V d.c. supply a vacuum so that the electrons can move
freely to prevent collision with air molecules.
A separate circuit of 6 V d.c. supply is used to heat
the filament by passing a small current through it.
Figure 4.1
4 If the connection to the 400 V d.c. supply is reversed, no reading
is shown on the milliammeter.
5 The electrons emitted from the filament are repelled by the
anode whenever the anode is negatively charged. No electrons A thermionic diode allows electrons to flow
can reach the anode. This is shown in Figure 4.2. in only one direction.

emitted electrons repelled


by anode

negatively A coating of metal oxide reduces the


charged anode temperature needed for emission of electrons
+
6V + from a metal surface. A nickel cylinder coated
d.c. – with a mixture of barium and strontium oxides
no current
supply 0 only needs to be heated to about 1100 K
before electrons are emitted.
coated cathode
+ – filament
400 V +
d.c. supply 6V
d.c. –
supply
Figure 4.2 Potential difference is connected in reverse

Electronics 494
Factors affecting the rate of thermionic emission

Surface area of metal Temperature of metal Types of metal Nature of metal surface
• Rate increases as the • Rate increases as the • Rate depends on • Rate increases when
size of the surface temperature of metal type of metal. the metal is coated
area increases. increases. • If the temperature with metal oxide.
• A larger surface area • At higher temperatures, and surface area • Metal oxides such as
allows more more electrons gain remain constant, barium oxide and
electrons to be sufficient energy to different types of strontium oxide emit
emitted. break free from the metals require electrons at a lower
metal surface. different amounts temperature.
of energy to eject
the electrons.

Cathode Rays
1 An electron gun is an assembly of cathode and anode in a vacuum tube. It is The minimum energy
used to produce beams of fast-moving electrons known as cathode rays. required to eject electrons F
2 Figure 4.3 shows the basic elements of an electron gun. from the surface is known O
3 The anode has a hole in it. It is connected to the positive terminal of the high

4
as the work function, ψ of R
potential difference (p.d.) supply. The cathode is heated by a tungsten

CHAPTER
the metal. The work M
filament and is connected to the negative terminal of the supply. function of a metal can be
reduced by the presence of 5
The heated cathode emits The focusing anode focuses impurities.
electrons that are accelerated the electrons into a fine beam
of electrons called cathode Example:
towards the anode.
rays. A small amount of thorium
electron gun
reduces the work function
vacuum of tungsten by 50%.

cathode
focusing
anode
fluorescent screen
+

6V
d.c. –
supply
electron
filament beam

– +
glass tube

high p.d. supply


The cathode rays reach a high
velocity before striking the
fluorescent screen. When this
happens, the screen glows
with a green light.

Figure 4.3

4 A cathode-ray tube uses an electron beam. This tube can be found in devices
such as cathode-ray oscilloscopes (CRO), computer displays and television
picture tubes.
5 A deflection tube or the Maltese cross tube is used to study the properties of
cathode rays.

495 Electronics
Activ To study the properties of cathode rays
ity 4.1
(A) Maltese cross tube
Apparatus/Materials
A Maltese cross tube, 6 V d.c. supply, 3 kV d.c. supply, and a bar magnet.
Arrangement of apparatus
cathode coated with
barium and
strontium oxides vacuum
fluorescent
anode screen
filament

+
6 V d.c. supply shadow

Maltese cross
F 3 kV d.c. supply
O
4

R Figure 4.4 Maltese cross tube


CHAPTER

M Procedure
1 A Maltese cross tube is connected to the power supplies as shown in Figure 4.4.
5
2 Only the 6 V d.c. supply is switched on. The shadow formed on the fluorescent screen is observed.
3 Both 6 V and 3 kV d.c. supplies are switched on. The shadow on the screen is observed again.
4 The north pole of a bar magnet is brought close to the fluorescent screen. The screen is observed again for any
changes in the shadow formed.
Results
Table 4.1
Step Observation Explanation Conclusion
Only 6 V d.c. supply is A ‘cross-shaped’ shadow Light rays travel in
switched on. appears where no straight lines.
fluorescence occurs. The
light rays from the bright
filament are blocked by
the Maltese cross.
Both 6 V and 3 kV d.c. The ‘cross-shaped’ Cathode rays travel in
supplies are switched on. shadow is observed at the straight lines and
shadow same position and is of the produce fluorescence.
same size as the shadow
formed by light rays.
Both 6 V and 3 kV d.c. The shadow is shifted Since cathode rays
supplies are switched on. downwards. Fleming’s are deflected by a
The north pole of a bar N
shadow Left-hand Rule is used to magnetic field, the
magnet is brought close determine the deflection rays must be
to the screen. current (in the opposite
of cathode rays. electrically charged.
Activity 4.1

direction to the electron)

magnetic field
(points to the
south)
force

Electronics 496
(B) The deflection tube Procedure
Apparatus/Materials 1 A deflection tube is connected to the power
A 6 V d.c. supply, 3 kV d.c. supply, a bar magnet, and a supplies as shown in Figure 4.5.
deflection tube containing a fluorescent screen between 2 The 6 V and 3 kV d.c. supplies are switched on.
two plates which are connected to a high p.d. supply. 3 The high p.d. supply connected to the metal plates
Arrangement of apparatus is switched off. The path of cathode rays is
cathode coated with barium observed on the screen.
and strontium oxides 4 The high p.d. supply is now switched on. The
focusing
filament anode
path of cathode rays is observed again.
+
+ 5 Step 4 is repeated but with the high p.d. supply
high potential
6V difference (p.d.) connected in reverse. The path of cathode rays is
d.c. – supply observed.

supply
6 The high p.d. supply is then switched off. The
vacuum
north pole of a bar magnet is brought close to the
metal fluorescent
3 kV d.c. plate screen. The screen is observed again for any
screen
supply changes in the path of the cathode rays.
Figure 4.5
Results
Table 4.2 F
O

4
Step Observation Explanation Conclusion R

CHAPTER
M
High p.d. supply to metal Cathode rays move Cathode rays travel in
plates is switched off. straight across the plates straight lines. 5
when there is no electric
field between the plates.

High p.d. supply to metal + Cathode rays are Cathode rays are
plates is switched on. deflected towards the negatively charged.
positively charged plate.
The path traced is a
parabola.

High p.d. supply to metal – Cathode rays are Cathode rays are
plates is connected in deflected towards the negatively charged.
reverse. positively charged plate.
The path traced is a
parabola.
+

High p.d. supply to metal Cathode rays are Since cathode rays
plates is switched off. deflected downwards. are deflected by a
The north pole of a bar The path traced is an arc magnetic field, the
magnet is brought close of a circle. Fleming’s rays must be
to the screen. Left-hand Rule is used electrically charged.
to determine the
Activity 4.1

N magnetic deflection of cathode rays.


field
current (into the
(electron to paper)
the right) force

497 Electronics
Cathode rays consist of fast-moving
electrons that travel in a straight line
in a vacuum.

Cathode rays possess momentum


and kinetic energy due to the moving Cathode rays are negatively
electrons. The rays produce a bright Properties of charged. Therefore, the rays are
spot (fluorescent effect) when they Cathode Rays deflected towards a positive plate
strike a fluorescent screen. in an electric field.

Cathode rays that strike a heavy Cathode rays are deflected by


metal target produce X-rays. The magnetic fields. The direction of
rest of the energy is released as heat. deflection is determined using
Fleming’s Left-hand Rule.

F
O
4

R • Electric current and electron flow are always in opposite directions.


• Only charged particles can be deflected by a magnetic field.
CHAPTER

5
Energy Conversion of Electrons in Cathode Rays

1 Figure 4.6 shows a cathode ray tube which describes the energy conversion of electrons qualitatively.
vacuum
cathode
filament cathode rays

fluorescent
+ 3 screen
6 V d.c. supply 1A

2

anode

3 kV d.c. supply

Figure 4.6

1
Electric current heats the filament, which in turn
emits electrons. 2
An electric field across the anode and cathode
Electrical energy → Heat energy accelerates the electrons towards the fluorescent
screen.
Electrical → Kinetic energy
3 potential energy of electrons
High speed electrons strike the screen producing (due to potential
fluorescence (light). difference across
anode and cathode)
Kinetic energy of electrons → Light energy

Electronics 498
2 Referring to Figure 5 From the principle of conservation of energy,
filament cathode (–)
4.7, let the potential anode (+) for each electron:
difference across the e
e
anode and cathode +
Electrical potential energy = Kinetic energy
be V. 1 v2
V eV = ––m
2 e
Figure 4.7 ∴ The final velocity of an electron,

3 The electrical potential energy of an electron v = 2eV
–––––
is given by eV, where the charge of an electron, me
e = 1.6 × 10–19 C.
4 The kinetic energy of an electron is given 6 From the formula derived, when the potential
1 v 2, where the mass of an electron, difference is increased, the final velocity of the
by ––m
2 e electron increases too. The electron, which has
me = 9.1 × 10–31 kg and the final velocity of the more kinetic energy now, strikes the screen and
electron is v. produces a brighter fluorescence.

1
Figure 4.8 shows an electron from a cathode which is Solution
F
accelerated towards an anode. The potential difference (a) From the principle of conservation of energy: O

4
across the metal plates is 500 V. Kinetic energy gained R

CHAPTER
anode M
cathode = Electrical potential energy lost
= eV 5
e
= 1.6 × 10–19 × 500
= 8 × 10–17 J
500 V
1 m v2,
Figure 4.8 (b) From eV = –– e
2
(a) How much kinetic energy does the electron gain 2eV
v = ––––
just before it strikes the anode? m
 e
(b) What is the speed of the electron just before the 2 × (8 × 10–17) = 1.33 × 107 m s–1
= –––––––––––––––
impact if it begins from rest?  9.1 × 10–31
(c) Calculate the final speed of the electron if the 2eV
(c) v = ––––
potential difference is increased to 1000 V. m
[Mass of electron, me = 9.1 × 10–31 kg,  e
2 × 1.6 × 10–19 × 1000 = 1.88 × 107 m s–1
charge of one electron, e = 1.6 × 10–19 C] = ––––––––––––––––––––––––
9.1 × 10–31

1 SPM
Clone
’05

Which diagram shows the correct movement of a cathode ray between two charged metal plates?
A B C
– – –

e e e

+ + +

Comments
Cathode rays are negatively charged. Therefore, they are deflected away from the negatively charged
plate and attracted towards the positively charged plate.
Answer C

499 Electronics
The Cathode-Ray Oscilloscope (CRO)

1 The cathode-ray oscilloscope (CRO) is a measuring and testing instrument used in the study of electricity and
electronics.
2 This is because the CRO can be used to
(a) display the wave forms of voltages, (c) test electronic equipments.
(b) measure a short duration of time, and
3 The main feature of the CRO is a cathode-ray tube with the air inside removed so that the electron beam does not
collide with air molecules.
4 Figure 4.9 shows the structure of a cathode-ray tube used in an oscilloscope.
5 There are three main parts in a CRO.

electron gun deflection system


fluorescent screen
4 focusing anode (coated with zinc sulphide)
Cathode-ray 3 control grid
5 accelerating
anode 6
tube 2 cathode
Y-plates
7 X-plates
F
O
4

R electron beam bright


CHAPTER

M spot

5 1 heating
_
filament
_ graphite
+ + coating vacuum

F
O Figure 4.9
4

R
(a) An electron gun M
which is used to produce a narrow beam of electrons. Table 4.3 describes the function of
CHAPTER

each component in the electron gun.


5
1 Heating 2 Cathode 3 Control grid 4 Focusing anode 5 Accelerating
Component filament anode

Function Heats the Emits electrons Regulates the Focuses the Accelerates the
cathode to a from its surface number of electrons electrons leaving the electron beam to a
high after being which reach the cathode to a narrow high velocity.
temperature. heated to a high anode and therefore beam so that they
temperature. control the arrive at the same
brightness of the spot on the screen.
spot on the screen.

Table 4.3
(b) A deflection system is used to deflect the electron beam. Table 4.4 describes the function of each
component in the deflection system.

Component 6 Y-plates 7 X-plates

Function To deflect the electron beam vertically To deflect the electron beam
(up and down). horizontally (from left to right).

Table 4.4

Electronics 500
(c) A fluorescent screen is made up of two components, as shown in Table 4.5.

The glass surrounding the


The inside surface coated fluorescent screen coated with
Component
with zinc sulphide graphite and connected to the Earth

Glows when the electron beam strikes Channels the electrons striking the
Function it. The kinetic energy of the electron is screen to the Earth.
converted to light energy.

Table 4.5
6 A sinusoidal wave form on the display of a CRO is due to the combined deflection caused by the Y-plates and
X-plates, as shown in Figure 4.10.

vertical movement of electron


beam due to the Y-plates

horizontal movement of
electron beam due to
the X-plates
F
Figure 4.10 O

4
R

CHAPTER
M

5
Using a CRO
Figure 4.11 shows a CRO with its control knobs.

a.c. / d.c. switch


• d.c. – Displays wave form of potential difference of d.c. and a.c.
• a.c. – Displays wave form of potential difference of a.c. only.
The d.c. component is blocked by a capacitor in the CRO circuit.

X-Shift & Y-Shift


• X-Shift – Displaces the Power switch (on/off)
spot horizontally. Controls the power supply to the
• Y-Shift – Displaces the CRO.
spot vertically.
d .c a .c

Power
Brightness
X-input & Y-input
switch

Controls the brightness


Brightness
Terminals connecting of the spot on the
the potential difference Focus screen.
across X-plates and 100 MS/TIM
2
E/CM
10us
VOLTS/CM
Y-plates. 2 1
5
10m 5us

Focus
10 20m 2us

20 2 30m
30 1us
100m 50m
40
50
1
Focuses the bright spot on
Earth connection
the screen so that it is
Connects the input
sharper.
terminal to the
Earth.
Timebase control
Applies a changing voltage on the
Y-gain
X-plate which makes the spot
Sets the magnitude of
move from left to right and back to
vertical shift of the spot
its starting point at a steady
on the screen.
frequency.
Figure 4.11

501 Electronics
Use of a Cathode-ray Tube in Television A colour television uses three electron guns. The screen is
The pictures on a television screen are built up by designed in such a way that one gun makes it emit green
electron beams. A television tube is essentially a cathode- light, another gun makes it emit red light and the
ray tube. The electron beam is deflected by currents remaining one makes it emit blue light. As these three
flowing in the different coils. The magnetic field of the colours are the primary colours of light, a whole range of
coils sweeps the electron beams across the screen. colours can be produced. A shadow mask is placed just
behind the screen. The mask has about 500 000 holes,
shadow mask one for each group of coloured dots. This ensures that
Y-deflection coil
red
Physics Blog the beams from each gun overlap so that the viewer can
perceive the correct colours on the screen.
electron green dot
guns blue blue dot
red dot
green

television
X-deflection coil screen

F
O
4

R
CHAPTER

5 Activ To use a cathode-ray oscilloscope (CRO)


ity 4.2
Apparatus/Materials Procedure
A cathode-ray oscilloscope, a dry cell, a.c. power 1 The CRO is switched on. The d.c. supply is
F
supply. switched on and the timebase circuit is switched
O
off.
4

R
Arrangement of apparatus 2 Both X-shift and Y-shift are adjusted until a bright
CHAPTER

M
spot is situated at the centre of the screen.
5 3 The brightness and focus knobs are adjusted until
ON/OFF
the spot is not too bright and sharp.
Brightness
This is to prevent it from damaging the
Focus
fluorescent screen.
X-Shift Y-Shift volt/cm
Y-input 4 The screen is observed again when the timebase
d.c.
circuit is switched on.
5 The timebase circuit is switched off. A dry cell is
time/cm
a.c.
dry cell
connected to the Y-input as shown in Figure 4.12.
Figure 4.12 The Y-gain is adjusted before the screen is
observed. The timebase circuit is switched on.
Any change in the spot is noted. Then, the
ON/OFF
terminals of the dry cell are connected in reverse.
Brightness
Again, the screen is observed for any changes
Focus
when the timebase circuit is switched off and on.
X-Shift Y-Shift volt/cm Y-input
6 An a.c. power supply is connected to the Y-input
d.c.
as shown in Figure 4.13. The timebase circuit is
Activity 4.2

a.c.
switched off, then switched on again. The screen
time/cm
a.c.

is observed for both occasions. The timebase


Figure 4.13 control is adjusted so that at least one complete
wave oscillation is seen.

Electronics 502
6 Figure 4.14 shows the observations on the
Results
screen.
Table 4.6
Y-input Timebase circuit
y y
Switched off Switched on
None

(a) Timebase circuit (b) Timebase circuit


F5/4/32
switched off switched on
Figure 4.14

Dry cell 7 Let the vertical displacement of the spot = y


(d.c. power If the Y-gain is Y volt per cm, then the potential
supply) difference, V = (Y × y) V.
8 Steps 1 to 5 are repeated, but with the dry cell
connected in reverse. Figure 4.15 shows the
observations on the screen.
When connection When connection F
is reversed: is reversed: O

4
R

CHAPTER
M
y y
5

(a) Timebase circuit (b) Timebase circuit


F5/4/33
switched off switched on
a.c. power Figure 4.15
supply
9 The potential difference, V = –(Y × y) V.

Measuring the Potential Difference of an a.c.


Supply

1 An a.c. power supply is connected to the


Y-input.
2 The timebase circuit is first switched off, then
switched on.
3 Figure 4.16 shows the observations on the
Applications of CRO
screen.
Measuring the Potential Difference of a d.c.
Supply

1 The timebase circuit is switched off. ypp ypp


2 A bright spot is formed at the centre of
the screen.
Activity 4.2

3 The focus knob is adjusted to obtain a spot


with an appropriate brightness.
4 A dry cell is connected to the Y-input. (a) Timebase circuit (b) Timebase circuit
5 The above steps are repeated when the time- switched off switched on
base circuit is switched on. Figure 4.16

503 Electronics
4 If the Y-gain is Y volts per cm, then the peak to peak potential
difference, Vpp = (Y × ypp) V.
1 (Y × y ) • When the Y-gain is set at
5 The peak potential difference, Vp = ––
2 pp y volt per cm, the spot
1 travels 1 cm up or down
= ––V for every y volt across the
2 pp
6 The root mean square (rms) potential difference, Y-input terminals.
• Vr.m.s. is used to represent
1
Vr.m.s. = –––V
–– p the potential difference of
2 an alternating current
supply.

Measuring a Short Duration of Time Measuring Wave Frequency

1 The timebase circuit is switched on. The time- 1 The timebase circuit is set at x ms cm–1.
base is set to x ms cm–1. 2 Figure 4.18 shows a wave on the screen.
2 The Y-gain is set to a value so that the wave
form displayed is easy to see. d
F 3 A microphone is connected to the Y-input.
O
4 Two quick hand claps are made near the
4

R
microphone.
CHAPTER

M
5 Figure 4.17 shows the wave form observed on
5 the screen.
Figure 4.18

d Let the distance between two successive wave


crests = d cm
F The period for one wave, T = d × x
O = dx ms
4

2 consecutive claps
R
dx – s
CHAPTER

M = ——
10
ms/dv
Y-input
1000
5 1
Timebase ∴ Frequency of the wave, f = —
T
1
= —–——
——–(
dx
1000 )
1000
Figure 4.17 = ––––– Hz
dx
F5/4/37
6 Two wave pulses, representing two hand claps,
can be seen on the screen. Displaying Different Wave Forms
7 Let the distance between the two pulses = d cm
1 The timebase circuit is switched on. An a.c.
Duration of time between the two claps, t
power supply is connected to the Y-input.
=d×x
2 A sinusoidal wave is displayed on the screen, as
= dx ms
shown in Figure 4.19.

Figure 4.19

Electronics 504
3 Table 4.7 shows the wave forms before and after the timebase or the
Y-gain is reset.
Table 4.7 A CRO can also display other
wave forms as shown in
Before After Figure 4.20.

a 2a
T T

Amplitude = a Amplitude = 2a
Frequency = f Frequency = f (a) Sound wave
1 1
Period, T = Period, T =
f f

a T a 2T

(b) Wave from audio


Amplitude = a Amplitude = a frequency generator
F
Frequency = f f O
Frequency = ––

4
1 2 R
Period, T =

CHAPTER
f Period, T′ = –––––
f ( )
1 = 2 ––
1
f
M

( )
––
2
5

= 2T
(c) Rectangular wave
1
T
3 Figure 4.20 The other
a
common
T
a wave forms
2

Amplitude = a a
Amplitude = ––
Frequency = f 2
1 Frequency = 3f
Period, T =
f Period, T′ = –1–– = ––
3f 3 f ( )
1 ––
1

1T
= ––
3

a 2T
a
T 2

Amplitude = a a
Amplitude = ––
Frequency = f 2
1 f
Period, T = Frequency = ––
f 2
1 = 2 ––
Period, T′ = ––––––
f
1
f( )
––
2( )
= 2T

505 Electronics
2
Figure 4.21 shows two pulses on a CRO. The pulses Calculate the time lapse between the signals if the
are caused by two consecutive signals detected by a timebase setting is 4 ms cm–1.
microphone connected to the CRO.
5.2 cm Solution
Time lapse between two pulses
= 5.2 cm × 4 ms cm–1
= 20.8 ms
= 20.8 × 10–3 s
Figure 4.21 = 0.0208 s

3
Figure 4.22 shows a (iii) Vertical distance from crest to trough
wave form obtained = 6 cm
from a signal Peak to peak potential difference, Vpp
F generator connected = 6 cm × 3 V cm–1
O to the Y-input of a = 18 V
4

R
CRO. 1
(iv) Peak potential difference, Vp = ––V
CHAPTER

M
2 pp
5 1 (18)
1 cm = ––
2
1 cm
=9V
Figure 4.22 (v) Root mean square potential difference,
1
Below are the settings on the CRO. Vr.m.s. = –––V
–– p
2
Y-gain setting = 3 V cm–1 1 (9)
Vr.m.s. = –––
––
Timebase setting = 5 ms cm–1 2
Vr.m.s. = 6.36 V
(a) Determine the
(b) Let the distance between two consecutive crests
(i) period of the signal,
=x
(ii) frequency of the signal,
(iii) peak to peak potential difference, Vpp, Period of signal, T = x cm ×Timebase setting
in ms cm–1
(iv) peak potential difference, Vp,
0.02 s = x × 10 ms cm–1
(v) root mean square potential difference, Vr.m.s..
0.02 s = x × 10 × 10–3 s cm–1
(b) Sketch the new wave form on the grid if the
0.02 s
timebase setting is now set at 10 ms cm–1. x=
10 × 10–3 s cm–1
Solution x = 2 cm
(a) (i) Horizontal distance between two consecutive The amplitude remains unchanged because the
crests = 4 cm Y-gain setting is unchanged.
Period of the signal, T = 4 cm × 5 ms cm–1
2 cm
= 20 ms
= 20 × 10–3 s
= 0.02 s
1
(ii) Frequency of the signal, f = ––
T
= –––1––
0.02
= 50 Hz

Electronics 506
2 SPM
Clone
’09

The figure shows an audio signal generator which is Which signal shows the softest and the lowest pitched
connected to a cathode-ray oscilloscope, CRO. audio signal?
A C

audio signal CRO B D


generator

Comments
The loudness and the pitch of an audio signal are determined by its amplitude and frequency respectively.
The softer the signal, the smaller is the amplitude. The lower is the pitch, the smaller is the frequency.
D has the smallest amplitude and the smallest frequency (i.e. largest period).
Answer D

F
O

4
R

CHAPTER
M
When calculating Vr.m.s., remember to use the value of Vp,
not Vpp. Vp Vp 5
Vr.m.s. =
2

4.1

1 (a) In a cathode-ray tube, the potential difference of 50 Hz is connected to the Y-plates. The timebase
across the anode and the cathode is 2500 V. setting is at 2 ms cm–1.
Calculate the energy of an electron due to the
electric field between the plates.
[Charge of an electron, e = 1.6 × 10–19 C]
(b) The number of electrons striking a fluorescent P Q
screen in a cathode-ray tube is 4.2 × 1014 in 15 cm
20 seconds. 5 kV potential difference is
connected to the tube. Determine Calculate
(i) the average rate of charge flow, (a) the period of the audio signal,
(ii) the power produced by the tube. (b) the time taken for the spot to move from P to Q.
[Charge of an electron, e = 1.6 × 10–19 C]
4 The figure shows a wave 1 cm
2 (a) An electron is accelerated between an anode
form produced when a 1 cm
and a cathode through a potential difference of
cathode-ray oscilloscope is
x V. If the initial speed and final speed of the
connected to an
electron are 2.1 × 106 m s–1 and 5.2 × 106 m s–1
alternating current power
respectively, find the value of x.
supply. The gain-control or
(b) Determine the change in momentum of the
Y-gain is set at 12 V cm–1.
electron mentioned in (a).
[Mass of one electron, me = 9.1 × 10–31 kg, Determine
charge of one electron, e = 1.6 × 10–19 C] (a) the peak to peak potential difference, Vpp,
3 The figure on the right shows a wave form produced (b) the peak potential difference, Vp,
when an audio frequency generator with a frequency (c) the root mean square potential difference, Vr.m.s..

507 Electronics
4.2 Understanding 9 When an electron is removed from a covalent
bond, it leaves a vacancy. An electron from a
Semiconductor Diodes neighbouring atom can fill this vacancy,
leaving the neighbouring atom now with a
Properties of Semiconductors vacancy. Therefore, the vacancy (called a hole)
can travel through the lattice and act as a
1 Conductors are materials which allow current charge carrier.
to flow through them easily. This is because 1 electrons leaving its
conductors have free electrons which can drift free electron outermost shell leaving
between their atoms. Hence, conductors have a vacancy (hole)

low resistance.
2 Most metals such as silver, copper and Si Si Si
aluminium are good conductors of electricity.
hole
3 Insulators are materials which do not conduct
electrical current.
4 Most non-metals such as rubber, polythene and
perspex are good insulators. However, carbon
is an exception. Si Si Si
5 A semiconductor is a material whose resistance
F is between those of good conductors and those 2 neighbouring electron filling in the
O vacancy, leaving a vacancy (hole)
of good insulators.
4

R direction of direction of
6 Silicon, germanium, boron, tellurium and
CHAPTER

M holes flow electrons flow


selenium are some examples of pure Figure 4.25 Flow of charge carriers in a semiconductor
5 semiconductors. Lead sulphide, gallium
arsenide and indium arsenide are examples of 10 Electrons are theF5/4/62
negative charge carriers while
compound semiconductors. holes are the positive charge carriers.
7 A silicon atom has four electrons in its 11 Pure semiconductors act as insulators at low
outermost shell. temperature because there are no free
F electrons available to act as charge carriers.
electron O 12 However, the number of free electrons
4

R
nucleus increases as the temperature of the semi-
CHAPTER

M
(14 protons, 14 neutrons) conductor rises. The heat energy supplied
Si 5 is converted to the kinetic energy of the
outermost shell
electrons. As a result, more electrons are
knocked free producing more charge carriers.
Therefore, semiconductors act as conductors
Figure 4.23 A silicon atom
F5/4/60 at high temperatures.
8 Each electron in the outermost shell can form 13 Figure 4.26 shows graphs for a typical metal
a covalent bond with one electron in the conductor and a particular type of thermistor
outermost shell of another atom. Hence, each which contains semiconductor material.
silicon atom forms four covalent bonds with resistance (Ω)
four neighbouring atoms.
r
onducto
metal c

Si
outermost
shell
thermistor
Si Si Si (containing semiconducting materials)
temperature (°C)
25 100
covalent
Si bond Figure 4.26
F5/4/63
14 The conductivity of semiconductors can also be
increased by shining light on them or adding
Figure 4.24 Silicon atoms in the solid state impurities to the semiconductors.
F5/4/61
Electronics 508
Doping SPM
’04/P1
SPM
’06/P1

1 Doping is a process of adding a small amount of impurities into the crystalline lattice of
semiconductors to increase their conductivity.
2 Atoms of the impurities added should have almost the same size as the atoms of the
semiconductors.
3 By adding different kinds of impurities, two types of semiconductors can be obtained; the
p-type semiconductor and the n-type semiconductor.

Made from pure silicon or germanium

Undergoes the ‘doping’ Has electrons and holes


Similarities
process as charge carriers

p-type semiconductor Types of semiconductor n-type semiconductor

Si Si Si Si Si Si
F
missing free O

4
electron (hole) electron
R

CHAPTER
Si Si Si Si
M

acceptor atom donor atom 5

F5/4/64 F5/4/65
Differences

Boron, indium, gallium, Antimony, phosphorus,


Doping substance
aluminium arsenic

Acceptor atom/trivalent Type/valency of atom Donor atom/pentavalent

Hole Majority charge carriers Electron

Electron Minority charge carriers Hole

1 Free electrons
2 Electrons are are attracted
attracted to to the positive
p-type the positive
semiconductor terminal.
terminal. Current
flow
n-type 2 Holes are shifted to the
1 Holes are shifted to the semiconductor negative terminal.
negative terminal.
Key: hole electron
Key: hole electron current flow
current flow

509 Electronics
Resistance Charge carrier Conductivity
Conductor Low Free electrons Decreases with temperature increase
Semiconductor Between conductor Free electrons and holes Increases when light shines on it, temperature
and insulator increases or with presence of impurities
Insulator High Very few or no free Not affected by temperature
electrons

hole electron

n-type semiconductors are neutral materials and not


negatively charged.
p-type n-type
They differ from pure semiconductors as they have material material
more free electrons.
Figure 4.28

7 As a result of these movements, the p-type


F Semiconductor Diodes SPM
’05/P1
SPM
’06/P1
SPM
’07/P1
SPM
’08/P1
O
material becomes more negative and the n-type
4

R 1 A semiconductor diode is also called a p-n material becomes more positive. This will
result in a potential difference acting from the
CHAPTER

M
junction diode.
2 It consists of a p-type semiconductor in contact n-type material to the p-type material across
5
with an n-type semiconductor. the junction. This potential difference is called
3 The regions of p-type and n-type materials are the junction voltage.
called anode and cathode respectively. 8 The effect of this junction voltage is to prevent
4 Figure 4.27 illustrates the structure and symbol charge carriers from drifting across the junction.
for a diode. F
9 Figure 4.29 shows the depletion layer and
O junction voltage of a diode.
4

R
junction voltage
CHAPTER

M
_ _ + +
5
(a) Diode p-type _ _ + + n-type
material _ _ + + material
_ _ + +
depletion layer
{

anode (+) cathode (–) hole depletion layer electron


p n
Figure 4.29
(b) Structure of a diode 10 In order for electric current to flow through the
anode (+) cathode (–) diode, the voltage applied across the diode
must exceed the junction voltage.
11 The junction voltages for germanium and silicon
(c) Symbol of a diode
are approximately 0.1 V and 0.7 V respectively.
Figure 4.27
I (mA)
5 A p-type material meets an n-type material germanium diode
across a bounding region called the depletion
silicon diode
layer ( p-n junction).
6 At this junction, electrons from the n-type
material drift across the junction to fill in the V (V)
holes in the p-type material. The holes from 0 0.1 0.7

the p-type material drift in the opposite


direction to unite with free electrons in the
n-type material. This is shown in the Figure Figure 4.30 Graph of current against voltage for
4.28. germanium and silicon diodes

Electronics 510
12 From the graph in Figure 4.30, electric current
only starts flowing through both diodes when
the voltage applied exceeds their respective The depletion layer of a diode is usually about 1 µm
junction voltages. thick.

Activ To observe the flow of current in a semiconductor diode


ity 4.3
Apparatus/Materials Results/Discussion
A semiconductor diode, connecting wires, light Forward-biased arrangement
bulb and dry cell. narrow depletion layer
p n electron
Arrangement of apparatus
+ −
hole low resistance
current flow bulb
lights
up
F
+ − O

4
R
current flow

CHAPTER
M
Figure 4.31 Forward-biased arrangement
Figure 4.33
5
+ − (a) When the potential difference supplied by the dry
cell exceeds the junction voltage, a large current
flows through the diode. Hence, the bulb lights up.
(b) In this arrangement, the depletion layer is narrow,
− + and the resistance of the diode decreases.
(c) The holes drift across the junction towards the
Figure 4.32 Reverse-biased arrangement n-type material.
Reverse-biased arrangement
wide depletion layer
Procedure p n
1 The anode end (with p-type material) and the
cathode end (with n-type material) of the diode
are connected to the positive and negative
terminals of a dry cell respectively, as shown in high resistance
Figure 4.31. bulb
does not
light up
2 This particular arrangement is said to be
forward-biased.
3 It is observed whether the bulb lights up Figure 4.34
(glows).
(a) Only a very small current flows through the
4 The anode end (with p-type material) and the diode. Hence, the bulb does not light up.
cathode end (with n-type material) of the diode (b) In this arrangement, the electrons and holes are
are now connected to the negative and positive pulled away from the depletion layer. This widens
terminals of a dry cell respectively, as shown in the depletion layer and increases the resistance of
Activity 4.3

Figure 4.32. the diode.


(c) The junction voltage will slowly build up until it
5 This arrangement is said to be reverse-biased.
reaches the same potential difference as the dry cell.
6 It is observed whether the bulb lights up. When this occurs, current stops flowing completely.

511 Electronics
SPM SPM SPM
Diode as a Rectifier ’07/P1 ’07/P2(B) ’08/P2(A)

1 When a p-n junction diode is in a forward-biased


arrangement, it only allows the current to flow
from the anode to the cathode. It is said to act as a
valve.
2 A diode can convert alternating current into
direct current. This is known as rectification.
Therefore a diode can act as a rectifier.
3 An alternating current is a current which changes both current and symbol of diode
in the same direction
its direction with a certain frequency. This is due to
the alternate change of the polarity of the power Figure 4.35 Diode as a valve
supply.
4 A complete cycle of alternating current consists of
two half cycles; a positive half-cycle and a negative
half-cycle.
5 Figure 4.36 shows a complete cycle of alternating current.

output potential 1
Positive half-cycle:
difference Current increases
F
from zero to a
O maximum value and
4

R back to zero. Current


1
CHAPTER

M is flowing in the
positive direction.
time
5
2
2 Negative half-cycle:
Current decreases
from zero to a
minimum value and
back to zero. Current
is flowing in the
negative direction.

Figure 4.36

6 There are two ways to convert an alternating current into a direct current:
(a) Half-wave rectification
(b) Full-wave rectification

Half-wave Rectification

1 Figure 4.37 shows a simple rectification circuit with CRO displays of the input and output wave
forms.
diode

input output
connected connected
to a.c. load to
CRO CRO
a.c.
varying d.c.

Figure 4.37

2 The current can only flow in the forward direction through the diode.
3 In the first half-cycle, the diode is forward-biased. The current can flow through the diode.
4 In the second half-cycle, the diode is reverse-biased. The diode blocks the current.
5 The process of rectification using a diode which allows current to flow in the half-cycle is known
as half-wave rectification.

Electronics 512
Full-wave Rectification Smoothing

1 Figure 4.38 shows a circuit used to produce 1 The output from a rectifier circuit can be
full-wave rectification. smoothed by connecting a capacitor across
2 The arrangement of diodes in Figure 4.38 is the load, as shown in Figure 4.39.
called a bridge rectifier, because it reverses
the negative half of each alternating current
cycle instead of blocking the flow of the current.
input output
input

capacitor
+ − Y X connected connected
to a.c. to
connected
a.c. CRO load CRO
to
CRO a.c. smoothed d.c.
− + X Y
a.c.

Figure 4.39 Half-wave rectification with smoothing


load
2 During the forward peaks (positive half-cycles),
the capacitor is charged up. Energy is stored in
connected to the capacitor.
output
CRO 3 In between the forward peaks (negative half- F
cycles), the capacitor releases its charge O

4
R
(discharges). It discharges partly through the

CHAPTER
M
load. The energy stored in the capacitor acts as
varying d.c. a reservoir and maintains the potential 5
difference across the load.
Figure 4.38 Full-wave rectification 4 Figure 4.40 shows the potential difference for
3 During the forward half of each cycle, diodes X half-wave rectification with smoothing.
are forward-biased but diodes Y are reverse-
biased. Therefore, diodes X conduct current
V V capacitor
(solid arrows) but diodes Y block current. charges up
4 During the reverse half, diodes Y are forward- capacitor
discharges
biased but diodes X are reverse-biased.
Therefore, diodes Y conduct current (broken
arrows) but diodes X block current. smoothing
5 As a result, the current always flows in the same 0 t 0 t
direction through the load regardless of which
way it leaves the power supply. Figure 4.40
6 The process of rectification using four diodes
to allow current to flow in a complete cycle 5 A capacitor with greater capacitance produces a
and in the same direction is called full-wave smoother current. This is because the capacitor
rectification. can store more charge.

3 SPM
Clone
’07

The diagram shows a circuit connected to a cathode-ray oscilloscope.

+ connected to CRO

F5/4/84 513 Electronics


Which of the following displays is shown on the Comments
CRO? During the first half-
A C cycle, both diodes are +
forward-biased. Hence, –

current flows through


the resistor.
First half-cycle

In the second half-


B D cycle, both diodes are
reverse-biased. Hence,

current is prevented +
from flowing through
the resistor.
Second half-cycle

Answer A

F
O
V V V
4

R
CHAPTER

5
half-wave smoothing
0 t rectification 0 t 0 t

V V V

full-wave smoothing
0 t rectification 0 t 0 t

4
Figure 4.41 shows a full-wave rectification circuit. (e) What is the name for the arrangement of diodes in
the circuit?
A B
a.c. Solution
X (a) Direct current (d)
D C
R
connected (b) V
to CRO
V

Figure 4.41

(a) What type of current passes point X? 0 t


(b) Sketch the display on the CRO. 0 t
(c) Name the component that must be connected
across R in parallel to smooth the current. (e) Bridge rectifier
(d) Sketch the display on the CRO for part (c). (c) Capacitor

Electronics 514
4.2

1 Which of the following bulb(s) will light up when the 3 The circuit in the following figure is a full-wave
switches are turned on? rectifier.
C

A D

B a.c.

connected
R to CRO
F G H

E
(a) Draw the symbol for diodes in the figure so that
the current flows in the direction indicated by the
arrow.
2 (a) Draw a circuit which produces half-wave (b) Draw the graph to show the effect of these
rectification. diodes on the alternating current.
F
(b) Draw a graph showing the effect of half-wave (c) If a capacitor is connected across R, what is the
O
effect on the alternating current?

4
rectification on an alternating current. R
(d) Is it possible to turn an alternating current into a

CHAPTER
(c) Draw on the graph in (b) the effect if a capacitor M
is connected across the resistor. constant direct current using a capacitor? Explain
(d) Draw another graph to show the effect of how this can be done. 5
connecting a capacitor with greater capacitance 4 State three ways in which the conductivity of a
across the resistor. semiconductor can be increased.

4.3 Understanding Transistors

Types of Transistors SPM


’08/P1

1 A transistor consists of a specially treated 3 It can link two separate circuits so that the
semiconductor crystal. It has three terminals current through one circuit controls the
instead of two. current through the other.
2 The three terminals are the emitter (E), base 4 These transistors make use of the properties of
(B) and collector (C). p-n junctions to achieve such control.
5 There are two types of transistors; the n-p-n
transistors and the p-n-p transistors.

The n-p-n Transistors

1 An n-p-n transistor consists of a layer of


collector
p-type semiconductor sandwiched between
emitter
two layers of n-type semiconductors.
base
2 The n-type layers are the emitter and collector,
Figure 4.42 whereas the thin p-type layer is the base.

515 Electronics
3 Figure 4.43 shows the structure and symbol for 2 A transistor has two p-n junctions. Each
an n-p-n transistor. junction acts as a diode. Figure 4.45 shows
collector (C) these junctions.
C
collector (C)
n C C

base (B) p B
n
n n
B
{
base (B) p
E

{
n B p p-n
emitter (E ) E junctions
n
(a) Structure emitter (E )
(b) Symbol
Figure 4.43 An n-p-n transistor
4 In the n-p-n transistor, current flows from C to E
E. Take note that the arrow is drawn from B to (a) n-p-n transistor
E in the symbol for the n-p-n transistor.

The p-n-p Transistors C

F 1 A p-n-p transistor consists of a layer of n-type


p
O semiconductor sandwiched between two layers
}
4

R
}
of p-type semiconductors. n p-n
CHAPTER

M B
2 The p-type layers are the emitter and collector, junctions

5 whereas the thin n-type layer is the base. p


3 Figure 4.44 shows the structure and symbol for
a p-n-p transistor.
E
collector (C)
C (b) p-n-p transistor
F Figure 4.45
p O C
4

base (B) n B R
CHAPTER

M
p
B
5 E The Flow of Current in a Transistor

emitter (E ) E 1 Figures 4.46 (a) and (b) show the n-p-n and
(a) Structure (b) Symbol p-n-p transistors in their respective circuits.
Figure 4.44 A p-n-p transistor Each circuit shows the flow of current. Take
4 In the p-n-p transistor, current flows from E to note that the emitter-base junction is forward-
C. Take note that the arrow is drawn from E to biased, while the base-collector junction is
B in the symbol for the p-n-p transistor. reverse-biased.

Functions of Terminals in a Transistor collector


(output)
1 Table 4.8 shows the functions of the terminals
IC
circuit
in a transistor. IB RB n C
B
p n reverse-
Table 4.8 biased
forward- E
Terminal Function base (input)
circuit
biased

Emitter (E) Supplies charge carriers to C. IE

Collector (C) Receives charge carriers from E.


Base (B) Controls the flow of charge carriers IE = I B + I C

from E to C or C to E. (a) n-p-n transistor in a circuit

Electronics 516
collector 8 Instead, the electrons are attracted across the
(output)
circuit
IC
base-collector junction due to a higher potential
difference at the collector terminal. The flow of
IB RB
B
p C electrons to the collector produces collector
n
p
reverse- current, IC, through the transistor.
E biased

base (input) forward-

circuit biased
IE Working Principle of a Transistor SPM
’05/P1

I E = IB + I C
1 Figure 4.48 shows two light bulbs, X and Y
connected to a circuit with an n-p-n transistor.
(b) p-n-p transistor in a circuit Y
Figure 4.46

collector
2 There are two parts to a transistor circuit; base RB C
circuit
circuit and collector circuit. IC
base
B

circuit
E
3 The current which flows in the base circuit is
called the base current, IB while the current IB IE
which flows in the collector circuit is called the
F
collector current, Ic. X O
4 Consider an n-p-n transistor in a circuit shown

4
Figure 4.48 R
in Figure 4.47.

CHAPTER
M

2 A variable resistor, RB is placed in the base 5


IC circuit to vary the magnitude of base current,
IB.
3 Initially, bulb Y lights up but bulb X does not.
collector
C

RB
n circuit This shows that IB is smaller than IC.
4 If there is no IC flowing in the collector circuit,
p
B IB still flows in the base circuit. Therefore, IB is
base independent of IC.
circuit n forward-biased
E
5 However, if there is no IB flowing in the base
circuit, no IC will flow in the collector circuit.
6 By reducing the resistance of RB, the magnitude
IB of IB in the circuit increases.
Key: electron hole 7 Both bulb X and bulb Y light up. This shows
Figure 4.47 Flow of charge carriers in a transistor that a small increase in IB causes a greater
increase in IC.
8 Typically, a small current in the base circuit can
5 When the circuit is switched on, the base- produce a current fifty to a hundred times
emitter junction is forward-biased. At the larger to flow in the collector circuit.
same time, the depletion layer narrows and its
resistance lowers. This causes the majority IB < IC < IE
charge carriers, electrons in the n-semiconductor, IE = IB + IC
to flow from E to B.
6 The number of holes in the depletion layer
also decreases. Therefore, very few electrons
combine with the holes. Some of the remaining
electrons flow back into the base circuit and
the rest into the collector circuit.
7 However, the base-collector junction is reverse- If IB = 0, then IC = 0.
biased. Therefore, electrons cannot flow from But when IC = 0, IB is not affected.
the collector terminal to the base terminal.

517 Electronics
Activ To study the relationship between IB and IC
ity 4.4
Apparatus/Materials Procedure
n-p-n transistor, switch, two d.c. power supplies of 1 A circuit is set up as shown in Figure 4.49.
3 V and 6 V, variable resistor, microammeter, and a 2 The switch is open (off). Readings are taken
milliammeter. from both microammeter and milliammeter.
Arrangement of apparatus 3 The switch is then closed (on). The variable
resistor is adjusted until the microammeter gives
IC a reading of 10 µA.
4 Reading is taken from the milliammeter.
variable resistor mA milliammeter
5 The variable resistor is adjusted again so that IB
values of 20 µA, 30 µA, 40 µA, 50 µA, 60 µA,
70 µA and 80 µA are obtained.
IB 6V
6 The corresponding readings of the milliammeter
3V
are taken. These values are recorded in a table.
7 A graph of IC against IB is then plotted.
µA
switch
microammeter

F Figure 4.49
O
Tabulation of data
4

R
The following table shows the values of IB and IC obtained.
CHAPTER

M
Table 4.9
5
Base current, IB (A) Collector current, IC (mA)
0 0
10 1
F
20 2
O 30 3
4

R
40 4
CHAPTER

M
50 5
5
60 6
70 7
80 8

Graph Discussion
Graph of IC against IB : From the graph in Figure 4.50, it can be inferred that:
IC (mA) (a) There is no collector current when no base
current is flowing.
8 (b) The collector current is directly proportional to
7 8 – 4 = 4 mA the base current.
= 4 × 10–3 A
6 (c) A small gain in base current is accompanied by
5 a large gain in collector current. The current gain
4
IC
3 80 – 40 = 40 µA is defined as –– .
= 4 × 10–5 A IB
2 I
From the graph plotted, the current gain = C
Activity 4.4

1
IB (µA)
IB
0 10 20 30 40 50 60 70 80
= Gradient of the graph IC against IB
–3
Figure 4.50 4— 10–—
×–— A = 100
=— —
4 × 10–5 A

Electronics 518
SPM
Applications of Transistors ’07/P1

The Transistor as an Amplifier

1 A transistor can be used to amplify (magnify) current. This is because a small change in base
current causes a large change in collector current.
2 3
Varying base current A small change in the base current causes a large
flows through capacitor. change in the collector current.
X

5
loudspeaker
40 kΩ
IC (output) Sound waves with higher
capacitor C amplitude but equivalent
Y B frequency are produced.
IB
potential E
divider 10 kΩ IE 4
microphone The varying collector
(input)
current flows into the
Z
loudspeaker. F
Figure 4.51 A transistor amplifier circuit
O
1

4
F5/4/111 R
Sound waves fed into the microphone cause its diaphragm to vibrate. The electrical output of

CHAPTER
M
the microphone changes according to the sound waves and produces varying base current
because of the small alternating voltage produced by the microphone. 5

2 Figure 4.52 shows the input and output wave Table 4.10
forms. The frequencies of both waves are Component Function
equivalent. However, the amplitude of the
sound waves from the loudspeaker is higher Microphone To change sound signal to
than the sound waves fed into the electrical signal.
microphone. Capacitor To block a steady current from
flowing into the transistor and
microphone.
Potential divider To apply a proportion of the total
amplify voltage across the emitter-base
junction so that the junction is
input wave form output wave form
forward-biased.
Figure 4.52
Transistor To amplify the input wave form.
3 Table 4.10 shows the function of each Loudspeaker To change electrical signal to
component in the circuit. sound wave.

• A potential divider uses two resistors in series to divide RYZ


the potential difference across them. VYZ = —————– × Total potential difference
RXY + RYZ
Based on Figure 4.51, 10
= ————— × 6
RXY 40 +10
VXY = —————— × Total potential difference
RXY + RYZ = 1.2 V
40 • A practical amplifier consists of a few transistors.
= ———–—— × 6 Output from one transistor supplies the input of the
40 +10
next and so on. Therefore, greater amplification and
= 4.8 V
Physics Blog power output are obtained.

519 Electronics
The Transistor as a Switch
1 In a transistor, no current can flow in the collector circuit unless a current flows in the base circuit. This property
allows a transistor to be used as a switch.
2 The transistor can be turned on or off by In darkness
changing the base current. 6
3 There are a few types of switching circuits Bulb does not light up.

operated by transistors. The resistance of


5
light-dependent resistor
IC becomes smaller.
(LDR) increases hugely
(A) Light-operated switch (about 1 MΩ).
• The circuit in Figure 4.53 is designed 4
IB is low and transistor
to light the bulb in a bright environ- 2 cannot be switched on.
ment and to turn it off in the dark. Voltage across LDR
increases.
3
Voltage across R
decreases.
light
IC

LDR bulb

1 kΩ C
potential B 6V
F divider
O IB E
R 10 kΩ
4

In bright environment
R
CHAPTER

M
The resistance of
5 light-dependent resistor
(LDR) falls greatly (about a
few hundred ohms) when
6 light falls on it. 2
Figure 4.53 A light-operated switch circuit Bulb lights up. Voltage across LDR
decreases.

• If the positions of the LDR and 5


resistor R are interchanged, the IC also increases. 3
bulb switches off in the bright Voltage across R
environment and turns on in the increases.
4
dark. IB is high and transistor
is switched on.

(B) Heat-operated switch


• In this circuit, a thermistor is connected to the potential divider.
• A thermistor is a type of resistor with its resistance dependent on the surrounding temperature.
Its resistance increases when the temperature is low, and vice versa.

1 IC
When heat is applied to the bulb
thermistor, its resistance drops. siren
thermistor C
2 1 kΩ B 6V
A greater share of supply 5
voltage is dropped across R. The bulb glows and the E
siren sounds. R 10 kΩ
IB
3
IB Increases. 4
Greater increase in IC.

Figure 4.54 A heat-operated switch circuit


• This particular circuit is suitable as a fire alarm system.

Electronics 520
Integrated Circuits (IC)

Integrated
Combined together in one wafer-thin
circuit (IC)
chip of silicon called a microchip.
The microchip is only a few millimetres
square (about 5 mm2) with a thickness
of 0.5 mm.

Transistors Resistors

Components

Microchip

Capacitors Diodes
F
O

4
R

CHAPTER
M

5 5

Figure 4.55 shows a transistor circuit. Resistor W Solution


has a resistance of 4 kΩ. Y is a fixed resistor and X is (a) P = base
a variable resistor. Q = collector
R = emitter
D
(b) To limit the amount of current flowing into
the base terminal.
W 4 kΩ
(c) (i) Let the resistance of X = x
Q x
——––– × 6 = 4
Y P 6 V
E 4+x
R
6x
X –––—–— = 4
4+x
6x = 4(4 + x)
F 6x = 16 + 4x
Figure 4.55 2x = 16
x = 8 kΩ
(a) Name the terminals P, Q and R.
(b) What is the function of Y? (ii) The potential difference across EF
(c) In order to light the bulb, the potential 1
= ——––– × 6 V
difference across EF must be at least 4 V. 1+4
(i) What is the resistance of X when the bulb
= 1.2 V
lights up?
(ii) What happens to the bulb if X has a Since the potential difference is 1.2 V, which is
resistance of 1 kΩ? less than 4 V, the bulb does not light up.
(d) What must be done to the circuit so that the bulb (d) Replace X with a light-dependent resistor
is switched off in the bright light and on in the (LDR).
dark?

521 Electronics
Types of IC

silicon chip

Digital IC Linear IC
notch
• Switching circuit • Switching circuit that
that handles high handles analogue
signal or low signal. signal.
small
• Used in computers dot • Used in amplification
plastic
and calculators. case
circuits.
Figure 4.56 Structure of an IC

Occupies a small space which reduces Allows smaller and more compact
the size of circuits (e.g. microchip). devices to be made.

Consumes a small amount of electrical


F
Advantages energy.
O of an IC
4

R
Very little heat is generated.
CHAPTER

M
(a) Mobile phone (b) Laptop
5
Can be built at a low cost. Applications of IC

4.3

1 The diagram below showsF a circuit with a transistor as (b) What happens to the collector current when the
O
an automatic switch. Bulb Q lights up when the base current increases?
4

R
potential difference dropped across resistor X is equal (c) What are the differences between the input and
CHAPTER

M
or more than 5.5 V. Initially, the resistance of the light- output sound waves in terms of
dependent resistor Y is 1.5 5 MΩ. When Y is exposed (i) amplitude? (ii) frequency?
to light, the resistance of Y decreases. 3 An n-p-n transistor is connected to a circuit as shown
light in the diagram below.

1.5 MΩ Y Q

15 V 9V
rheostat
3V
800 Ω X
µA mA
microammeter milliammeter
Find the value of Y when Q starts to light up.
(a) Label IB, IC and IE and their directions in the circuit.
2 The diagram below shows a transistor circuit. (b) What is the function of the transistor?
F5/4/93
IC
R2
earphone (c) IB and IC are related by –– = 200.
R1 IB
9V If IC is 250 mA, calculate
R3 (i) IB in µA, (ii) IE.
microphone (d) If the above rheostat is replaced with ,
determine the reading shown on the
(a) State the functions of the light microammeter.
(i) transistor, (iii) microphone, (v) capacitor. (e) Explain your answer in (d).
(ii) R1, (iv) earphone,
4 State two advantages of an integrated circuit (IC).
1.5 MΩ Y Q

Electronics 522 15 V

800 Ω X
4.4 Analysing Logic Gates
output = 0 output = 1
Logic Gates SPM
’08/P1
input = 0 input = 1
1 Logic gates are electronic switches with one
(a) (b)
or more inputs and one output only.
Figure 4.58
2 Figure 4.57 shows a schematic diagram of a
logic gate. 9 Logic gates are used in computers, calculators,
digital cameras, digital watches and many
inputs
input logic output
operation other devices.

(a)
input Truth Table

1 A truth table shows the result of every


possible output given every possible input.
output
2 The table uses two logic numbers; 0 and 1.
3 The possible number of combinations, N for n
operation inputs, can be determined using the formula: F
O

4
N = 2n R

CHAPTER
(b) M
Figure 4.57 4 The following chart shows the number of
combinations, N for 1, 2 and 3 inputs. 5
3 Input is data fed into a processor. The
processor will process the inputs by using the
operations of logic gates. The result of these Number of inputs
operations is called the output.
4 Logic gates operate on a binary input. This
n=1 n=2 n=3
means each input is either 0 or 1.
5 The outputs are given a value of 0 and 1 as well. N=2 1
N=2 2
N = 23
6 The output depends on =2 =4 =8
(a) the input,
(b) the type of logic gate. 5 A tree diagram can be used to list down all the
7 Table 4.11 and Table 4.12 show how each possible inputs.
input and output can be represented. 6 The diagram below shows the application of
tree diagrams and their corresponding truth
Table 4.11 tables.
Input
Number of inputs
0 1
• Switch is turned off • Switch is turned on
• Low voltage (0 V) • High voltage (+5 V)
1 input 2 inputs 3 inputs
0 (0, 0, 0)
Table 4.12 0 (0, 0) 0
0 1 (0, 0, 1)
0 0 (0, 1, 0)
Output 0
1
1 (0, 1)
0 1 0 (1, 0) 1 (0, 1, 1)
1 0 (1, 0, 0)
1
Bulb does not light up Bulb lights up 1
0
1 (1, 1) 1 (1, 0, 1)
0 (1, 1, 0)
8 Figure 4.58 shows circuits with different inputs 1
1 (1, 1, 1)
and outputs.

523 Electronics
Input Output Input Output 5 Figure 4.60 shows the symbol for a NOT gate.
Input Output
A X A B X A B C X
A X
0 0 0 0 0 0
1 0 1 0 0 1
Figure 4.60
1 0 0 1 0
6 The output is always opposite to the input.
1 1 0 1 1
Therefore, a NOT gate is also called an inverter.
1 0 0 7 The inversion by a NOT gate can be written in
1 0 1 the Boolean algebraic expression:
1 1 0
Bar sign represents
1 1 1 — inversion operation.
Output variable X = A
Input variable

Types of Logic Gates SPM


’04/P1
8 However, if the operation of inversion is
performed twice on an input variable, the
There are five types of logic gates:
output variable is the same as the input variable.
(a) NOT gate
F (b) AND gate —

O (c) OR gate X = A = A
4

R
(d) NAND gate
CHAPTER

M
(e) NOR gate
5
AND Gate

NOT Gate 1 Figure 4.61 shows a circuit that illustrates the


concept of an AND gate.
1 Figure 4.59 shows a circuit which illustrates
the concept of a NOT gate. 4.5 V
F
O bulb X R
4

These legends
R
are used
CHAPTER

throughout this M
chapter.
4.5 V switch A switch A switch B
Switch off: 0 5 bulb X
Switch on: 1
Bulb does not R
Figure 4.61
light up: 0
Bulb lights up: 1 2 When both switches A and B are turned off, the
bulb does not light up.
Figure 4.59
3 If either switch A or B is turned on, but not both,
2 When the switch is turned off (open), current the bulb still does not light up. This is because
flows through the bulb causing it to light up. the circuit is open and no current flows.
However, when the switch is turned on 4 Only when both switches are turned on, the
(closed), the bulb goes off. circuit is complete and the current flows.
3 These can be summed up in a truth table below. Hence, the bulb lights up.
5 These can be summed up in a truth table below.
Table 4.13 Truth table for a NOT gate
Table 4.14 Truth table for an AND gate
Input Output
Input Output
A X
A B X
0 1
0 0 0
1 0
0 1 0
1 0 0
4 A NOT gate has only one input and one
corresponding output. 1 1 1

Electronics 524
6 Therefore, the output is 1 if both inputs are 1. 3 However, if either one or both switches are
7 Figure 4.62 shows the symbol for an AND turned on, the bulb lights up. This is because
gate. the circuit is complete and the current flows.
4 These can be summed up in Table 4.15.
A
X Table 4.15 Truth table for an OR gate
B
Input Output
Figure 4.62
A B X
8 The Boolean algebraic expression for the 0 0 0
action of an AND gate is:
0 1 1
X=A•B 1 0 1
Dot sign represents 1 1 1
AND operation.

5 Therefore, the output is 1 if either one or both


OR Gate inputs are 1.
6 Figure 4.64 shows the symbol for an OR gate.
1 Figure 4.63 shows a circuit that illustrates the
concept of an OR gate.
A F
X O

4
4.5 V B
R

CHAPTER
bulb X R M
switch A Figure 4.64
5
7 The Boolean algebraic expression for the
action of an OR gate is:
switch B

Figure 4.63
X=A+B
2 When both switches A and B are turned off, no Plus sign represents
OR operation.
current flows in the circuit. The bulb does not
light up.

Activ To show the action of logic gates using diode, transistor and
ity 4.5 resistor
Apparatus/Materials (A) OR gate
n-p-n transistor, diodes, resistors, connecting wires, Arrangement of apparatus
and bulb.
+6 V
input A

Legends
R bulb X
Wire connected to 0 V = Input logic 0 input B
0V
Wire connected to 6 V = Input logic 1
Bulb does not light up = Output logic 0 Figure 4.65
Activity 4.5

Bulb lights up = Output logic 1 Procedure


1 A circuit is set up as shown in Figure 4.65.
2 By using different combinations of inputs for A
and B, the state of the bulb is observed to
determine the corresponding output.

525 Electronics
3 The results are shown in Table 4.16. Discussion
Tabulation of data 1 When both inputs A and B are 0 (0 V), the diodes
Table 4.16 are forward-biased. The resistances of the diodes
are low allowing current to bypass the bulb.
Input Output Therefore, the bulb does not light up (output 0).
A B X 2 When either input A or B is 1 (+6 V), one diode is
0 0 0 forward-biased and the other is reverse-biased.
0 1 1 Current flows through the forward-biased diode.
The bulb does not light up (output 0).
1 0 1 3 However, when both inputs A and B are 1 (+6 V),
1 1 1 both diodes are reverse-biased. The resistances of
Discussion the diodes are high. This causes current to flow
through the bulb and lights it up (output 1).
1 When both inputs A and B are 0 (0 V), the diodes
are reverse-biased. Therefore, current can only Conclusion
flow to Earth without going through the bulb and The circuit displays the characteristics of an AND
so the bulb does not light up (output 0). gate. The output is 1 if both inputs are 1.
2 When either input A or B is 1 (+6 V), the (C) NOT gate
corresponding diode is forward-biased. Current Arrangement of apparatus
F flows through it to the bulb and causes the bulb +6 V
O to light up (output 1).
4

R 3 When both inputs A and B are 1 (+6 V), the R2


CHAPTER

M
diodes are forward-biased. Current flows through
R1
5 the bulb and again, the bulb lights up (output 1). input A bulb X
Conclusion
The circuit displays the characteristics of an OR gate. 0V

The output is 1 if either one or both inputs are 1.


Figure 4.67
(B) AND gate Procedure
F
Arrangement of apparatus O 1 A circuit is set up as shown in Figure 4.67.
4

R +6 V 2 Input A is set at 0 (0 V) and then 1 (6 V). The


CHAPTER

input A M
R state of the bulb is observed to determine the
5
corresponding output.
bulb X 3 The results are shown in Table 4.18.
input B
0V Tabulation of data
Table 4.18
Figure 4.66
Procedure Input Output
1 A circuit is set up as shown in Figure 4.66. A X
2 By using different combinations of inputs for A 0 1
and B, the state of the bulb is observed to
1 0
determine the corresponding output.
3 The results are shown in Table 4.17. Discussion
Tabulation of data 1 When input A is 0 (0 V), current flows through
Table 4.17 the bulb causing it to light up (output 1).
2 When input A is 1 (+6 V), the transistor produces
Input Output a large collector current. This produces a large
A B X potential difference across R2. The potential
difference across the bulb is negligible. Hence,
Activity 4.5

0 0 0
the bulb does not light up (output 0).
0 1 0
Conclusion
1 0 0
The circuit displays the characteristics of a NOT
1 1 1 gate. The output is 1 if the input is 0, and vice versa.

Electronics 526
7 The Boolean algebraic expression for this
Combinations of Basic Logic Gates
gate:
SPM 1 The NOT gate, AND gate and OR gate are the ——–
’04/P1 X=A•B
three basic logic gates. These three gates are
SPM
’06/P1 the basis of all the other more sophisticated
SPM logic gates. By combining these gates in a
’07/P1
specific way, a desired output can be obtained. NOR Gate
2 NAND gate and NOR gate are examples
of logic gates obtained from different 1 A NOR gate is equivalent to an OR gate
combinations of the three basic logic gates. followed by a NOT gate.
2 The circuit in Figure 4.70 illustrates the
concept of a NOR gate.
NAND Gate

1 A NAND gate is equivalent to an AND gate


followed by a NOT gate. 4.5 V switch A switch B bulb X

2 The circuit in Figure 4.68 illustrates the


concept of a NAND gate.
R
F
Figure 4.70
O

4
switch A R
3 The output for this gate is obtained by

CHAPTER
4.5 V M
bulb X
performing the OR gate operation followed by
switch B 5
the NOT gate operation.
4 Below is a truth table for a NOR gate.
R

Figure 4.68 Table 4.20


3 The output for this gate is obtained by Input Output
performing the AND gate operation followed
by the NOT gate operation. A B C=A+B X=C=A+B
4 Below is a truth table for a NAND gate. 0 0 0 1
0 1 1 0
Table 4.19
1 0 1 0
Input Output 1 1 1 0
A B C=A•B X=C=A•B
0 0 0 1 5 It is clear that the output is 1 if both inputs are
0 1 0 1 0. Other combinations of inputs give an
1 0 0 1 output of 0. In other words, its output is that of
an OR gate inverted.
1 1 1 0 6 Figure 4.71 shows the symbol for a NOR gate.

5 It is clear that the output is 0 if both inputs are A


1. Other combinations of inputs give an X
B
output of 1. In other words, its output is that of
an AND gate inverted.
Figure 4.71
6 Figure 4.69 shows the symbol of a NAND
gate.
7 The Boolean algebraic expression for this
A gate:
X
B ——–
X=A+B
Figure 4.69

527 Electronics
NAND Gate as a Universal Gate

1 The NAND gate is considered a universal gate


due to its capacity to produce other gates.
N AND gate = AND gate followed by NOT gate
This makes the NAND gates one of the most
useful gates in electronics.
2 Table 4.21 shows a few combinations of
N OR gate = OR gate followed by NOT gate NAND gates.

Table 4.21

NOT gate OR gate A

A _ A+B
A
B

_
A A A
F
A+B
O B
4

R
CHAPTER

M
AND gate NOR gate A
5 A A B
A + B
B A B
B

A
A FB A
B O A + B
4

R B
CHAPTER

6
Figure 4.72 shows a logic gate. Solution
Q

S
P

Figure 4.72 Input Output


P Q R S
The above logic gate is equivalent to a 0 0 0 1
A AND gate
1 1 1 0
B OR gate
C NOT gate
The output S = 1 when the input P = 0 and vice-versa.
D NOR gate
Hence, it is equivalent to a NOT gate.
Answer C

Electronics 528
Truth table
Logic Boolean algebraic
Symbol Input Output Summary
gate expression
A B X

0 0 0

0 1 1
A Output is 1 if one or
OR X=A+B X 1 0 1 both inputs are 1.
B

1 1 1

0 0 0

0 1 0
A Output is 1 if both
AND X=A•B X inputs are 1.
B 1 0 0 F
O
1 1 1

4
R

CHAPTER
M
— 0 1
NOT X=A Output is 1 if input is
A X
0, and vice versa. 5
1 0

0 0 1

0 1 1
—–— A Output is 1 if one or
NAND X=A•B X both inputs are 0.
1 0 1
B

1 1 0

0 0 1

A
0 1 0 Output is 1 if both
—––—
NOR X=A+B X inputs are 0.
B 1 0 0

1 1 0

When considering a set of logic gates, write the inputs


and outputs on the symbols for a quick and easy
solution.

0011
0001 1110
0101

529 Electronics
7
Each of the following figures shows the inputs and (c) 1 1
their corresponding logic gate. Show the output in the A NOT gate
X
same form as the inputs. 0 0

(a) 1

0 0 0 Figure 4.73
A
AND gate
X
Solution
1 1 (a) 1 (c)
0 0
B 0 0 0 1 1
X

0 0
X
(b)
1 1 1

0
A (b)
OR gate
X
1 1 1 1 1
1 X
F B
O 0 0
4

R
CHAPTER

8
Construct the truth table Ffor each of the following
O Input Output
circuits.
4

R A B X Y
(a)
CHAPTER

A M
X 0 0 1 0
5 0 1 1 1
B 1 0 1 0

Y
1 1 0 0

(b) A (b) 0011 1100


A 1011
X
B 0101 X
B
Figure 4.74

Solution Input Output


In order to obtain the outputs easily, list all the inputs A B X
and their corresponding outputs for each gate. 0 0 1
(a) 0011 1100
A 1110
X
0 1 0
1 0 1
1010
0101 1 1 1
B

0101
0100
Y
1100

Electronics 530
9
Construct a truth table for the circuit shown in Figure 4.75.
A
B
X

Figure 4.75
Solution
0011
A
0101
B 1110
C 0110
X
0101 D
0111

0011

Input Output
A B C=A•B D=A+B X=C•D F
O

4
0 0 1 0 0 R

CHAPTER
0 1 1 1 1 M

1 0 1 1 1 5

1 1 0 1 0

10
Figure 4.76 shows a combination of logic gates.

Figure 4.76

Write the Boolean algebraic expression for the above combination.


Solution

A
A+B
A

B
_
C=A+B B

B _
B


∴ C = A_+ B • B
C=A+B B

531 Electronics
4 SPM
Clone
’05

The diagram shows a logic gate circuit with input Solution


signals, Q and R. OR NAND
1011
Q 1111 X 0101
0101 Z
Q
0101 Y
output R
0101 1010
R
NOT
input signals

Which of the following output signals is produced by Input Output


the logic gate circuit?
Q R X Y Z
A C
1 0 1 1 0
0 1 1 0 1
1 0 1 1 0
F
1 1 1 0 1
B D
O
4

R Answer D
CHAPTER

F
O
4

R5 SPM
Clone
CHAPTER

M ’07

The diagram shows a logic 5gate circuit.

J
L
K

Which of the following truth tables represents output L?


A B C D
J K L J K L J K L J K L
F5/4/171
0 0 0 0 0 1 0 0 1 0 0 0
0 1 1 0 1 1 0 1 0 0 1 0
1 0 0 1 0 1 1 0 1 1 0 0
1 1 1 1 1 0 1 1 0 1 1 1

Solution
0011
J
0001 1110
L
0101
K

Answer B

F5/4/172
Electronics 532
2 When smoke is detected by the smoke
Applications of Logic Gates in Control
detector, the input A is in the state of logic 1.
Systems
3 The heat detector will also cause input B to be
in the state of logic 1 when heat is detected.
1 Logic gates are designed primarily to SPM
’04/P2(A) The siren will sound when the output is 1.
make decisions based on the input(s) SPM 4 Since an OR gate is used, the siren will sound if
that are fed. In this sense, logic gates ’05/P2(A)
one or both inputs are 1.
function as automatic switches. SPM
’06/P2(A) 5 Therefore, if either one or both detectors are
2 In a control system, there are three
activated, the siren will sound.
main components. Figure 4.77 shows
6 Table 4.23 shows a truth table for the fire alarm
the components.
system.
Control system Table 4.23

Input Output
Siren
Input(s) A Smoke B Heat Y
0 0 0 Not activated
Consist of detectors that detect changes in the
environment (such as change in temperature and 0 1  1 Activated
light intensity) and convert the changes into 1  0 1 Activated
electrical signals. F
1  1  1 Activated
O
The electrical signals are fed into a logic gate or gates.

4
R

CHAPTER
M
Automatic Fan
5
Control circuit 1 Figure 4.79 shows a circuit for an automatic fan.
Consist of electronic devices (such as capacitors, light detector
diodes and transistors working together) to allow or A
Y fan
block the signal received (to make a decision). motor
B
heat detector

Figure 4.79
Output(s)
2 The light detector produces input A = 1 when it
Consist of devices (such as siren, bulbs, alarm, is bright and input A = 0 when it is dim.
heater, electric motor) that respond to a signal from 3 The heat detector produces input B = 1 when it
logic gates. is hot and input B = 0 when it is cold.
4 The fan motor is turned on when the output
Figure 4.77 Y = 1.
5 Since an AND gate is used, the fan motor is
3 Some examples of control systems are fire turned on when the room is bright and hot, i.e.
alarm system, automatic heater, automatic fan A = 1 and B = 1.
and car theft alarm system. 6 Table 4.24 shows a truth table for the
automatic fan.
Fire Alarm System
Table 4.24
1 Figure 4.78 shows a simple fire alarm system. Input Output
smoke A Environment B Temperature Y Automatic fan
detector A
Y
0 Dim 0 Cold 0 Off
siren
1 Bright 0 Cold 0 Off
heat B
detector 0 Dim 1 Hot 0 Off
1 Bright 1 Hot 1 On
Figure 4.78

533 Electronics
Developments in The Field of Electronics
1 In the early twentieth century, the vacuum 5 In the early sixties, electronic circuits
diode was invented and used in computers, (integrated circuits) consisting of thousands
radios and televisions. of transistors and electrical components on a
2 However, its large size resulted in rather large silicon chip were invented. The
electrical equipment. microprocessor is one example of the
3 The invention of the transistor in 1948 application of integrated circuits.
allowed the production of smaller electrical 6 The future of electronics is heading towards
equipment. further miniaturisation of electronic
4 The transistor is not only small and low in cost components. Nanotechnology is one such
but also consumes less energy than a vacuum example.
diode.

Contributions of Electronics in Our Daily Lives

Telecommunications
F • Allows communication with other people from anywhere in
O the world using mobile phones and telephones.
4

R • Surfing the Internet using WiFi on mobile phone and


CHAPTER

M computer are possible.


5
• Short Messaging System (SMS) and Multimedia Messaging
Service (MMS) enable instant communication in written and
audio-visual format.

Banking Services F Internet


• Banking can be done through
O the Internet • E-mail can be used to send
4

and telephone at anytimeRand from information and messages


CHAPTER

anywhere. M • very quickly at a low cost.


• Automated Teller Machine (ATM) • Searching information becomes
5
provides services without the hassle of easy and efficient using the
queuing up at the counter. Internet browser software.
• Smart-chip-enabled credit cards relieve • Enables people from different
users from carrying cash and the cards are Contributions parts of the world to play games
recognised and accepted in many places. together (multiplayer platform).

Household Appliances
• Appliances such as air conditioners, fans, washing
machines and others are electronically operated.
• Appliances in a house that can be controlled from
remote places using mobile phone or Internet.

Electronics 534
Some Benefits of Applied Electronics Saving Storage Space

1 Huge amounts of data can be stored in DVD


Increasing Work Efficiency
(Digital Versatile Disc), VCD (Video Compact
1 In the field of research, computers can process Disc), CD (Compact Disc), UMD (Universal
huge amounts of data in a relatively short period Mini Disc) and floppy discs. The quality of
of time. At the same time, computers can information stored is high. At the same time,
perform simulation of dangerous real situations these storage devices last longer.
and still gather data to be analysed. Examples of
this are car crash testing and pilot training.
2 Computers allow weather prediction to be
made using data collected from satellites and
weather stations.

Figure 4.82

2 These devices attempt to reduce the use of


paper files and save storage space. F
Figure 4.80 O

4
R
Saving Energy and Material

CHAPTER
3 Communication among users from different M
places (teleconferencing, videophone) using
1 The integrated circuit is small in size and uses 5
Internet and among users in the same place
less material.
using intranet increases interaction and
2 Computer networking enables users to do
information sharing.
their shopping, banking, hotel reservations
4 Traffic lights, LRT, commuter trains and
and credit card settlement, etc. without
monorails are controlled by computers to
physically being at the location.
ensure their smooth operations.
5 In the field of medicine, diagnostic machines
such as CT scan, MRI and PET scan allow Portability
doctors to see inside the patient’s body to
reveal tumours and organs non-invasively. As the physical size of electronic devices such as
notebooks, mobile phones, computers and others
becomes smaller, the users can enjoy the
convenience of portable devices.

Figure 4.81

6 Laser technology is currently being applied in


eye corrective procedures such as LASIK and
also to prevent bleeding during surgery. As a
result, wounds can heal faster.
7 EEG (electroencephalogram) and ECG
(electrocardiogram) are used to study and
analyse brain wave patterns and heart activity
respectively.
8 The use of computers and robots increase the
efficiency and speed of work in offices and
factories. Figure 4.83

535 Electronics
Compact Disc (CD) light detector
Compact discs (CD) are digital discs. There are billions of LED laser
‘pits’, with different lengths and spacings cut into the disc lens
surface. A shiny area called ‘flat’ is located between each light
pit. A laser beam is used to focus on one track and moves
slowly across the disc. The flats reflect the beam whereas
the pits do not. As a result, this produces a reflected beam
which is on or off (a sequence of I’s or 0’s). A light detector
will then produce a sequence of electrical pulses which
are sent to a decoder. A varying electric current is passed
pit CD surface
to an amplifier and loudspeaker to produce music. flat

4.4

1 Label each of the following symbols. (a)Complete the following truth table.

F Input Output
O A B C
4

R (a) (d) 0 0
CHAPTER

M
0 1
5 1 0
(b) (e) 1 1

(b) Draw a diagram which represents the above truth


table.
(c) Write a Boolean algebraic expression for output C.
(c)
4 Name the logic gates represented by the following
circuits.
2 Construct the truth table for each of the following (a)
gates.
(a) bulb X R
A A

X B

(b)
B
A bulb X
(b)
A R
B
X
5 Construct the truth table and name the equivalent
gate for each of the following circuits.
(a) A

3 The diagram shows the combination of a few X


gates. B

A (b) A
C
B B X

Electronics 536
6 The circuit shown is a two-switch system used in 7 The diagram shows a combination of logic gates.
a shop. If a hidden switch is turned on, the main
switch A will open the door. However, if the hidden
switch is turned off, switching on A will activate the A
alarm.
C
main switch A
A B
C lock
opener
hidden B
switch B The inputs A and B are shown below.
A
D
alarm
_ input A
B

(a) Complete the truth table below. 1

— 0
A B B C D
(i) 0 0 1 0 0 input B

(ii) 0 1
1
(iii) 1 0 F
1 1 0 O
(iv)

4
R

CHAPTER
M
(b) State what happens in (iii) and (iv). Complete the following timing diagram for output C.
(c) A mistake was made when the system was 5
installed. A NOT gate was accidentally fitted in
output C
the input line (as indicated by the arrow in the
figure).
(i) Redraw the circuit. 1
(ii) Construct a truth table for the new circuit.
0
(iii) What happens if the hidden switch is turned
on followed by the main switch?

1. Thermionic emission is a process involving the 6. There are two types of semiconductors: p-type
emission of electrons from a hot metal surface. semiconductor and n-type semiconductor.
2. Cathode rays are beams of fast-moving electrons. 7. A diode can convert alternating current into direct
3. The cathode-ray oscilloscope (CRO) is a measuring current in a process called rectification. A diode is a
and testing instrument used in the study of rectifier.
electricity and electronics. 8. There are two types of transistors: n-p-n transistor and
4. A semiconductor is a material whose resistance is p-n-p transistor.
between those of good conductors and those of good 9. The three terminals of a transistor are emitter (E),
insulators. Examples of semiconductors are silicon, base (B) and collector (C).
germanium, boron, tellurium, selenium, lead sulphide, 10. A transistor can be used as amplifiers and automatic
gallium arsenide and indium arsenide. switches.
5. Doping is the process of adding a small amount 11. There are five types of logic gates: NOT, AND, OR,
of impurities into the crystalline lattice of NAND, NOR.
semiconductors.

537 Electronics
4
Multiple-choice Questions

4.1 3 The diagram below shows four


The Uses of the Cathode- N
different surfaces in which
Ray Oscilloscope (CRO) V
thermionic emission occurs.
Y W
1 Diagram 1 shows a cathode-ray A 180˚C X fluorescent
screen
oscilloscope (CRO) display for a 5 m² S
sonar wave signal transmitted and
reflected back by the ocean floor Diagram 4
at sea. B 300˚C
Where can the spot be observed
10 m² now?
A V
B W
F C 300˚C C X
O 5 m² D Y
4

R
CHAPTER

M
6 A CRO is connected to a transformer
Diagram 1 D 180˚C
5 circuit as shown in Diagram 5.
10 m² The Y-gain is set at 5 V cm–1.
The timebase is set at
100 ms cm–1. If the depth of the
1.1 Signifiof
cant
sea is 748 m, find the speed of Which theFigure
surfaces A, B, C or D 50
the sonar wave signal. will emit the most number of a.c. 400 turns
turns
A 1350 m s–1 electrons thermionically?
1cm
F 100 turns
B 1415 m s–1 O
1cm
4

–1
C 1496 m s R 4 Diagram 3 shows a signal
D 1500 m s–1 Diagram 5
CHAPTER

M displayed on the screen of a CRO.


2 Diagram 2 shows a bright spot at Calculate the root mean square
5 Q potential difference of the
the centre of the screen of a CRO.
Y2 is adjusted so that its potential alternating current input.
–– ––
is higher than Y1 and X1 has a A 5√2 V C 20√2 V
P –– ––
lower potential than X2. B 10√4 V D 15√4 V
Y1
1 cm
7 An electron is accelerated in a
Diagram 3 cathode-ray tube from an initial
X1 X2 speed, u through a potential
If the frequency of the signal is difference of x kV. Which of the
12.5 Hz, how much time does it following expressions represent
Y2 take for a signal to move from the final speed of the electron, in
Diagram 2 point P to point Q? terms of m, e, x and u?
A 0.1 s [m = mass of electron,
The new position of the spot is
B 0.2 s e = charge of electron]
A C C 0.3 s
A 2000ex
D 0.4 s + u2
m
5 When cathode rays hit the screen B 2ex
+ u2
of a CRO, a bright spot is formed. m
B D A magnetic field is applied as 1
C mu2 + ex
shown in Diagram 4. 2
[The cathode rays are moving out 1 mu2
D
of the page.] 2 ex

Electronics 538
4.2 14 Which of the following diagrams is
Semiconductor Diodes
Clone a p-type semiconductor?
SPM

8 Diagram 6 shows a rectifier circuit and its input and output. ’06
A
Ge Ge Ge

input output
Ge Sb Ge
connected connected
to CRO a.c. X to CRO
Ge Ge Ge

Diagram 6 B
Si Si Si

Which of the following 11 Diagram 9 shows a p-n junction


components represents X ? SPM
Clone diode. Si In Si
A Resistor ’09 depletion layer
B Capacitor p type n type
Si Si Si
C Inductor
electrons
D Transformer C
p-n junction Ge
Ge Ge
9 Diagram 7 shows a circuit 1.1 Significant Figure
connected to a simple holes Ge P Ge
transformer. F
Diagram 9
O

4
When the diode is in reverse Ge Ge Ge R

CHAPTER
biased, the depletion layer M
A becomes thinner D
B disappears Si
5
a.c.
Si Si
C widens
D becomes charged Si As Si
12 Diagram 10 shows a few electrical
Diagram 7 components and ammeters in a Si Si Si

What is the effect on the bulb circuit.


when the switch is turned on? Key: • electron ° hole
A The bulb does not light up.
B The bulb is brightly lit. A1
C The bulb lights for a short 4.3 Transistors
time. A2 15 Diagram 11 shows a transistor
D The bulb is dimly lit.
circuit.
A3
10 Diagram 8 shows a full-wave A1
SPM
Clone rectifier circuit. A4
’09 1.2 Ω
resistor Diagram 10
Which ammeter will not give a V
reading when the switch is turned A2
to CRO
on?
A A1 Diagram 11
Diagram 8 B A2 The readings of ammeters A1 and
Key: CRO: Cathode-ray oscilloscope C A3 A2 are 0.76 mA and 1.80 mA
D A4 respectively. What is the reading of
What is the function of the
the voltmeter?
capacitor? 13 The conversion of alternating
A 1.248 mV C 1.248 V
A To convert alternating current SPM current into direct current using
Clone
B 12.48 mV D 12.48 V
into direct current. ’06 diodes is known as
B To store charge only. A amplification 16 Which of the following statements 1.1 S
C To smoothen the output wave B binary system about a transistor is not true?
form. C rectification A A transistor links two separate
D To act as a potential divider. D doping circuits.

539 Electronics
B It is used in some circuits as an B D
1 P Q R
automatic switch.
C The base terminal controls the 0 0 0 1
flow of charge carriers from C 1 0 1
the emitter terminal to the 1
0 1 1
collector terminal. 0
1 1 1
D A transistor can be used to D
store charge. 1
0 22 Diagram 16 shows a combination
17 Which of the following is not a SPM
of two logic gates.
Clone
terminal for a transistor? ’08
20 Diagram 14 shows a combination P
A Collector
of logic gates. R
B Modulator Q
C Base Diagram 16
D Emitter
Which truth table is correct?
18 Diagram 12 shows an automatic A
P Q R
SPM
Clone switch to activate a siren when Diagram 14
’07 the surrounding temperature is 0 0 0
high. The above combination is
0 1 1
equivalent to a
bulb A NOR gate 1 0 0
F thermistor
siren
B NOT gate 1 1 1
O 1 kΩ C NAND gate
4

R D AND gate B P Q R
CHAPTER

M
21 Diagram 15 shows a combination 0 0 0
R 10 kΩ
5
SPM
Clone of three logic gates. 0 1 0
’06
P 1 0 1
Diagram 12
Q 1 1 0
What changes should be done
to activate the siren when the C
F P Q R
surrounding temperature is R
low? O 0 0 1
4

R Diagram 15
A Replace the battery with an 0 1 0
CHAPTER

M
a.c. power supply. Which truth table is correct for the 1 0 1
B Replace the bulb with a5 combination of the three logic
capacitor. 1 1 0
gates?
C Interchange the positions of A
P Q R D
the thermistor and R. P Q R
D Place a diode next to the 0 0 0 0 0 1
siren. 1 0 0 0 1 1
0 1 1 1 0 0
1 1 1 1 1 0
4.4 Logic Gates B
P Q R
19 Diagram 13 shows a logic gate 23 Table 1 shows a truth table for a
circuit with input signals X and 0 0 1
SPM
Clone
particular combination of logic
’09 Y. 1 0 0 gates.
0 1 1
X output Input Output
1 1 0
P Q R
Y C
P Q R 0 0 0
Diagram 13
0 0 1 0 1 1
Which diagram shows the correct
shape of the output signal? 1 0 1 1 0 1
A 0 1 0 1 1 0
1
1 1 0
0 Table 1

Electronics 540

1.1 Significant Figure


Which of the following D The input signals of P and Q are
combinations gives the result P 0101 and 1100 respectively.
shown in the truth table? The output signal R is
A R
A 0100
P B 1101
R
Q C 1011
Q D 1110
B P R

25 The Boolean algebraic expression


24 Diagram 17 shows a combination ———–
A + B = X represents
Q of logic gates. A AND gate
P
B NAND gate
C R
C OR gate
P Q D NOR gate
R
Q
Diagram 17

Structured Questions F
O

4
1 Diagram 1 shows a cathode-ray oscilloscope. R

CHAPTER
M
fluorescent
control grid focusing
anode accelerating screen 5
cathode anode

heating
element X-plates
– – + +
Y-plates

F5/4/214 Diagram 1
(a) Explain the mechanism by which the electrons are 1 cm
emitted from the cathode. [1 mark]
1 cm
(b) Name the process stated in (a). [1 mark]
(c) State the function of
(i) control grid,
(ii) accelerating anode. [2 marks]
(d) A voltage supply of 240 V is connected across the
X-plates. The dot on the screen is deflected by
8 mm.
(i) Find the amount of deflection of the dot
when a voltage supply of 300 V is
connected across the X-plates.
[1 mark]
(ii) What is the use of Y-plates in the
oscilloscope when the oscilloscope is used
to display a waveform? Diagram 2
[1 mark] Calculate
(e) Diagram 2 shows a wave display on the screen of (i) the root mean F5/4/215
square, [1 mark]
an oscilloscope when a voltage is supplied to the (ii) the period, [1 mark]
input terminals. The Y-gain is set at 0.50 V cm–1 (iii) the frequency, [1 mark]
and the timebase is set at 3.0 ms cm–1. of the voltage.

541 Electronics
(f) State an advantage of using a cathode-ray (b) Using Diagrams 5(a) and 5(b), explain the
oscilloscope to measure time interval compared observations on the ammeter readings in terms of
to a stopwatch. [1 mark] the way the diodes are connected to the terminals
of the battery. Include one relevant physical
2 Diagram 3 shows a combination of logic gates to
concept in your explanation. [5 marks]
control an automatic fan.
(c) Diagram 6 shows a half-wave rectifier circuit.
P fan motor
A
A
light detector Y
B
connected to CRO a.c. load connected to CRO
heat detector
Diagram 3
(a) Name the logic gate P. [1 mark] B
(b) Using the electrical components shown below, Diagram 6
draw a circuit diagram which produces the same F5/4/220
(i) Draw the wave display of a half-wave
output as P. [2 marks] rectification. [1 mark]
(ii) A capacitor is placed across AB to smooth
the current.
Draw the wave display produced.
Diagram 4 Explain how a capacitor can smooth the
F current. [3 marks]
O (c) Table 1 below is a truth table for the automatic fan
(d) Diagram 7 shows a transistor circuit that acts as an
4

R system.
automatic switch.
Table 1
CHAPTER

5
Input light detector Input heat detector Output Y
thermistor siren
0 0
C
0 1 1 kΩ B
6V
1 0
E
1 1 R 10 kΩ
F
O
Complete the table. [2 marks]
4

R
(d) State one electrical Mcomponent which can be Diagram 7
CHAPTER

used as a light detector. [1 mark] F5/4/221when the surrounding


The siren is activated
3 Diagrams 5(a) and 5(b) 5show two electrical circuits temperature is high. Alex wants a bulb to light up
SPM
using semiconductor diodes.
Clone automatically when it is dark.
’07 Suggest modifications that can be made so that
the bulb lights up automatically when it is dark.
State and explain your answers in terms of
A
(i) the electrical components that are needed
to replace any components in the circuit,
0A
(ii) the positions of these components in the
circuit. [10 marks]

(a) 4 Diagram 8 shows a circuit to control the temperature


F5/4/218 of a room.
diode to
relay
switch heater
P 1 kΩ
A
0.5 A R S 6V
Q

(b)
Diagram 5
F5/4/219 Diagram 8
(a) Name the types of semiconductor in a diode. (a) Name the components labelled P, Q, R and S.
[1 mark] [4 marks]

Electronics 542
(b) How does the resistance of Q vary with temperature? [1 mark]
(c) State the functions of
(i) component S,
(ii) relay switch. [2 marks]
(d) Some heaters have a fan built into them. What is the advantage of such arrangement? [1 mark]
(e) Explain how the circuit operates. [3 marks]
(f) If the resistances of P and Q are in the ratio of 1 : 2, calculate the potential difference dropped across Q.
[2 marks]
(g) Why must a diode be connected in parallel with the relay switch? [1 mark]
(h) What will happen if the diode is not connected in parallel with the relay switch? [2 marks]

5 (a) A factory producing a chemical solvent employs a logic system to ensure the volume of chemical
SPM
Clone solvent in containers is within the predetermined range. Diagram 9 shows the set-up of the system.
’05 Two radioactive detectors, P and Q detect the levels of radiation penetrating the containers.

logic gate
detector P
radioactive input P
sources output
chemical solvent input Q
detector Q
container

conveyor belt
F
Diagram 9 O

4
R

CHAPTER
Detectors P and Q give an input 0 when they detect a low level of radiation and an input 1 when they M
detect a high level of radiation.
5
(i) State the level of the solvent in the container if input P is 0.
Give a reason for your answer. [2 marks]
(ii) State the level of the solvent in the container if input Q is 1.
Give a reason for your answer. [2 marks]
(b) Diagram 10 shows two combinations of logic gates, A and B, which are suggested for the logic
system.

input P
input P input P
input P
outputoutput outputoutput
K K
input Q
input Q input Q
input Q
A A B B

Diagram 10

(i) Name the logic gate K. [1 mark]


(ii) Complete the following truth tables for A and B.

Table 2 The truth table for A Table 3 The truth table for B

Input P Input Q Output Input P Input Q Output


0 0 0 0
1 0 1 0
1 1 1 1

[4 marks]
(c) When the output is 1, it shows that the volume of the solvent in the container is within the
predetermined range.
When the output is 0, it shows that the volume of the solvent in the container is outside the
predetermined range.
(i) Based on the outputs of the truth table in (b)(ii), choose the combination of logic gates which are
suitable for use in the factory. [1 mark]
(ii) State two reasons for your answer in (c)(i). [2 marks]

543 Electronics
Essay Questions
6 In order to construct a p-type semiconductor or n-type semiconductor, a doping process is performed on a
pure silicon crystal. When a p-type semiconductor is combined with an n-type semiconductor, a diode is
formed which acts as a rectifier.
(a) Explain
(i) p-type semiconductors.
[2 marks]
(ii) doping process.
[2 marks]
(b) Draw a circuit showing the arrangement of four diodes used as a full-wave rectifier.
Explain how the rectifier works and sketch the output wave.
[7 marks]
(c) Describe the working of an automatic light control switch. Include also a circuit in your description.
[7 marks]
(d) Give two advantages of an integrated circuit (IC).
[2 marks]

7 In a television set, a cathode-ray tube is used to produce pictures on the screen. Electron guns in the tube
produce cathode rays which hit the fluorescent screen.
F
(a) (i) What are cathode rays?
O
With the help of a diagram, explain how cathode rays can be generated.
4

R
[6 marks]
CHAPTER

M
(ii) State four properties of cathode rays.
5 [2 marks]
(b) (i) State the energy changes of electrons in cathode rays. Write down the corresponding equation.
[2 marks]
(ii) State three differences between an electromagnetic wave and a cathode ray.
[3 marks]
(c) Describe how a CRO can
F function as a voltmeter to measure
(i) d.c. voltage, O
4

(ii) root mean square


R value of an a.c. voltage.
CHAPTER

M [7 marks]

5
8 Diagram 11 shows a picture of a diode which is used frequently in many electronic devices.

Diagram 11

(a) (i) Draw the symbol for a diode.


[1 mark]
(ii) A diode consists of a p-n junction.
What is a p-n junction?
[1 mark]
(iii) Explain the mechanism of current flow in a diode when voltage from a dry cell is applied across
the p-n junction in forward-biased and reverse-biased arrangements. Plot a graph of current
against voltage for the p-n junction.
[8 marks]
(b) A semiconductor diode is also known as a rectifier. Explain the meaning of ‘rectifier’ with accompanying
circuit diagrams.
[4 marks]
(c) Draw a circuit showing how four semiconductor diodes can be arranged to produce full-wave
rectification of an a.c. power supply. Explain how you can ‘smooth’ the rectified wave output.
[6 marks]

Electronics 544
9 Diagram 12 shows a circuit for a combination lock. When a switch is pressed, it changes its input from
0 to 1.
P

J M
Q
K L

Diagram 12
(a) Construct a truth table showing the states of J, K, L and M for all combinations of P, Q and R.
[4 marks]
(b) When the output M is 1, it opens the lock.
What is the correct combination? [1 mark]
(c) Give one advantage and one disadvantage of the circuit. [2 marks]
(d) A comparator is a logic circuit that gives a ‘1’ as its output only when the logic states of its inputs are the
same. Diagram 13 is a block diagram for such a circuit.
A
comparator C
B

Diagram 13
Construct a truth table for a comparator. Use A and B to represent the two inputs and C to represent the F
output. [2 marks] O

4
R
(e) Bill has designed a comparator using some logic gates. Diagram 14 shows his design.

CHAPTER
M
However, there is a fault in his design.
A 5

Diagram 14
(i) Construct a truth table for the logic circuit that Bill has designed. [2 marks]
(ii) Redesign the system so that it works correctly. [2 marks]
(f) State a gate which is considered a universal gate. [1 mark]
(g) Write the Boolean algebraic expression for the answer in (f). [1 mark]

10 A student conducts a series of experiments to determine the properties of a few logic gates.
Table 4 shows the inputs and outputs for NOT gate and AND gate.
Table 4

Input Output
P Q R (NOT gate) R (AND gate)
0 0 1 0
0 1 – 0
1 0 – 0
1 1 0 1

(a) Define the meaning of


(i) logic gate,
(ii) input,
(iii) output. [3 marks]
(b) Draw the symbol and write the Boolean algebraic expression for each gate. [6 marks]
(c) Which of the two gates when combined together form a universal gate?
Give one reason for your answer. [2 marks]

545 Electronics
Experiments
1 An experiment is carried out to study the relationship between the base current, IB and the collector current,
IC in a circuit with a transistor.
Diagram 1 shows the arrangement of the circuit for the experiment.

mA

R
µA

Diagram 1
Switch S is closed and the rheostat is adjusted F5/4/200
so that the microammeter gives a reading of 10 µA.
The corresponding reading of the milliammeter is recorded.
The values of base current, IB and collector current, IC are given by the readings of the microammeter and
F milliammeter respectively.
O The experiment is repeated by adjusting the rheostat to give microammeter readings of 20 µA, 30 µA,
4

R
40 µA, 50 µA and 60 µA.
CHAPTER

M
Diagrams 2(a), (b), (c), (d) and (e) show the readings of the milliammeter when the base currents are
5 10 µA, 20 µA, 30 µA, 40 µA, 50 µA and 60 µA respectively.

2 3 2 3 2 3
1 4 1 4 1 4
0

0
5

5
mA mA mA
F
O
4

(a) IB = 10 µA R (b) IB = 20 µA (c) IB = 30 µA


CHAPTER

M
2 3 2 3 2 3
1 4 5 1 4 1 4
0

0
5

mA
mA mA

(d) IB = 40 µA (e) IB = 50 µA (f) IB = 60 µA


Diagram 2
(a) State the
(i) manipulated variable, (ii) responding variable, (iii) fixed variable,
in this experiment. [3 marks]
(b) Based on Diagrams 2(a) to (e), write down the values of collector current, IC in Table 1.

Table 1

Base current, IB (µA) Collector current, IC (mA)


10
20
30
40
50
60
[3 marks]

Electronics 546
(c) Using the information in (b), plot a graph of IC against IB on a graph paper. [3 marks]
(d) From your graph in (c), state the relationship between the collector current, IC and the base current, IB.
[1 mark]
IC
(e) The current gain in this circuit is defined as ––.
IB
Explain how the value of current gain can be obtained from your graph in (c). [1 mark]
(f) Hence, determine the value of current gain from your graph in (c). [2 marks]
(g) Using your graph in (c), find the value of collector current, IC produced if the base current, IB is 36 µA.
Show on the graph any lines drawn.
[2 marks]
(h) Predict, from calculation, the value of base current, IB which produces 4.2 mA of collector current, IC.
[2 marks]

2 Diagram 3 shows a circuit which consists of two switches, a resistor and a bulb.

A
6V
B
R F
O

4
Diagram 3 R

CHAPTER
M
The bulb lights up only when A and B are switched on.
Using your knowledge on logic gates: 5
(a) State one suitable inference [1 mark]
(b) State one suitable hypothesis [1 mark]
(c) With the use of apparatus such as resistors, transistors and other apparatus, describe one experiment
to investigate the hypothesis stated in (b).
In your description, clearly state the following:
(i) The aim of the experiment
(ii) The variables in the experiment
(iii) The list of apparatus and materials
(iv) The arrangement of the experiment
(v) The procedure used in the experiment. Describe how to control and measure the manipulated
variables and how to measure the responding variables
(vi) The way to tabulate the data
(vii) The way to analyse the data
[10 marks]

COMPANION WEBSITE 547 Electronics


Online Tests
FORM 5

5
CHAPTER

Radioactivity

SPM Topical Analysis


Year 2007 2008 2009 2010 2011
Paper 1 2 3 1 2 3 1 2 3 1 2 3 1 2 3
Section A B C A B A B C A B A B C A B A B C A B A B C A B
Number of questions 3 1 – – – – 3 – 1 – – – 3 – 1 – – – 3 1 – – – – 3 1 – 1 – –

ONCEPT MAP

RADIOACTIVITY

Atomic Radioactive Uses of Nuclear


structure decay radioisotopes energy

Nucleus Electrons • Medicine Safety


• Agriculture measures
• Industries

Management
• Nucleon number of radioactive
• Proton number wastes
• Number of neutrons

Radioisotopes

Types of decay Types of radiation Concept of Nuclear Nuclear


• α decay • α particles half-life fission fussion
• β decay • β particles
• γ decay • γ radiation
Radioactive
carbon
dating Nuclear Chain
Decay series Nuclear radiation
reactors reaction
detectors

COMPANION WEBSITE
548 Learning Objectives
5.1 Understanding the Nucleus of an Atom

The Structure of an Atom SPM


’07/P1

Fundamental particles electron, e


• actual mass = 9.11 × 10–31 kg
_
• relative mass = 1
_ 1840
neutron, n • relative charge = –1
• actual mass = 1.67 × 10–27 kg • value of charge = –1.6 × 10–19 C
• relative mass = 1 + _
• relative charge = neutral + +
orbit
• value of charge = 0 → path taken by electrons
dN
going around the nucleus
_
proton, p
• actual mass = 1.67 × 10–27 kg nucleus
• relative mass = 1 dA → consists of protons and
• relative charge = +1 neutrons
• value of charge = +1.6 × 10–19 C → together, protons and
diameter of atom, dA neutrons are called nucleons
≈ 104
diameter of nucleus, dN → nuclear density ≈ 1016 kg m–3 F
O

5
R
Figure 5.1 Rutherford - Bohr Model of the atom

CHAPTER
M

Proton Number (Z) and Nucleon Number (A) 5

1 An element with a chemical symbol X, nucleon number, A, and proton number, Z, is


represented by the symbol:

Nucleon number (mass number)


= Total number of protons and neutrons
A
Z X Chemical symbol

Proton number (atomic number)


= Number of protons

2 A neutral atom has equal numbers of protons and electrons. Hence, the proton number also
tells us the number of electrons in a neutral atom.
3 The number of neutrons, N in an atom is given by:

Number of neutrons, N = A – Z

4 Each fundamental particle is represented by 11p (proton), 10n (neutron) and –10e (electron).
5 Table 5.1 shows the number of fundamental particles and their corresponding symbols for
three elements.
Table 5.1

Number of fundamental particles


Element Nucleon number, A Proton number, Z Symbol
Proton Neutron Electron
1
Hydrogen 1 0 1 1 1 H
1
4
Helium 2 2 2 4 2 2
He
9
Beryllium 4 5 4 9 4 4
Be

549 Radioactivity
Nuclides and Radioisotopes 4 Therefore, isotopes of the same element have
the same number of protons but different
numbers of neutrons.
1 A sample of an element usually contains a
5 Since the chemical properties of an element
mixture of different types of atoms. They have
are determined by the number of electrons
the same number of protons but different
(which is equal to the number of protons),
numbers of neutrons.
isotopes of the same element have similar
2 Any particular type of an atom of an element is
chemical properties.
known as nuclide.
6 Figure 5.2 illustrates the meanings of the terms
3 Atoms of an element which have the same
nuclide and isotope.
proton number but different nucleon
numbers are called isotopes.
isotopes

Key:
n neutron p proton n
p p n p
electron nucleus n

+1 +1 +1
F
O
5

1 2 3
R 1H nuclide 1H nuclide 1H nuclide
CHAPTER

M Figure 5.2
5
7 Isotopes with unstable nuclei tend to decay. 5.1
These isotopes are called radioisotopes.
8 Table 5.2 shows a few examples of isotopes. 1 Find the number of protons and neutrons in a
nucleus of
(a) nickel, 5288Ni
Table 5.2
(b) fluorine, 199F
(c) lead, 20882Pb
Isotopes 2 Write the symbols for the following nuclides.
Element
Stable Unstable (radioisotopes) Nuclide Number of
protons electrons neutrons
1 2 3
Hydrogen 1
H 1
H 1
H (a) H 1 1 1
(b) 0 8 8 10
12 13 14
Carbon 6
C 6
C 6
C
12 13 14
3 State the differences between 6
C, C and
6 6
C in
Oxygen 16
O 17
O 18
O terms of their properties.
8 8 8

5.2 Analysing Radioactive


Decay
SPM SPM SPM SPM SPM
Radioactivity ’04/P1 ’05/P1 ’06/P1 ’07/P2(C) ’08/P2(B)

1 Radioactivity is the spontaneous disintegration


The statements below illustrate the meanings of the
of an unstable nucleus into a more stable
terms element, nuclide and isotope.
• Carbon is an element.
nucleus with the emission of energetic particles
• Carbon-12 is a nuclide; Carbon-13 is a nuclide; or photons. The decay occurs randomly.
Carbon-14 is a nuclide. 2 A nucleus is unstable if it is too big. It is found
• Carbon-12, Carbon-13 and Carbon-14 are isotopes. that all nuclei with Z > 83 or A > 209 are
unstable.

Radioactivity 550
3 There are three main types of nuclear radiation emitted:
(a) Alpha particles, α
(b) Beta particles, β
(c) Gamma rays, γ
4 More than one type of radiation can be emitted at any one time during
a radioactive decay.
5 Table 5.3 gives the nature and fundamental properties of the three types
of nuclear radiation.

Table 5.3

Property Alpha particle, α Beta particle, β Gamma ray, γ

Nature Helium nucleus Fast-moving electrons High frequency


electromagnetic radiation
(wavelength of the order
10–10 m or less)
4 0 F
Symbol He
2
e
–1 — O

5
Charge +2e –1e No charge R

CHAPTER
M
Mass Large Very small No mass
Speed 5
Approximately one-tenth of Up to nine-tenths of the
(c = speed of Speed of light, c
the speed of light (~ 0.1c) speed of light ( 0.9c)
light)

6 Below are some examples of radioactive 7 When electrons are added to neutral atoms,
elements, their corresponding symbols and the negative ions are formed. But when electrons
types of radiation emitted. are removed from neutral atoms, positive ions
are formed.
Table 5.4
8 Radiation from radioactive elements causes
Radioactive Radiation(s) ionisation of atoms in air by knocking
Symbol
element emitted electrons off atoms.
230
Thorium-230 90
Th α
3
243
Americium-243 95
Am α Positive ions formed.

14 α
Carbon-14 6
C β – – 1
+ +
90
Alpha particle passes
Strontium-90 38
Sr β near atoms.
60 2
Cobalt-60 27
Co γ Alpha particles pulling electrons
51
off atoms as they pass by
Chromium-51 24
Cr γ leaving positive ions.
24
Sodium-24 11
Na β, γ Figure 5.3 Ionisation of air molecules
210
Polonium-210 84
Po α, γ
Uranium-238 238
U α, β, γ 9 Table 5.5 shows the major properties of alpha,
92
beta and gamma radiations.
210
Lead-210 82
Pb α, β, γ
227
Actinium-227 89
Ac α, β, γ

551 Radioactivity
Table 5.5

Property Alpha particle (α) Beta particle (β) Gamma ray (γ)
Ionising power (ions per mm in air) Strong (= 105) Moderate (= 102) Weak (= 1)
Penetrating power Weak Moderate Strong
Range in air A few cm A few m A few hundred m
Can be stopped by Human skin or a thin A thin piece of A few cm of lead or
human skin aluminium concrete piece of paper. aluminium. a thick concrete.

Effect of electric field Deflected towards the Deflected towards Not deflected
negative plate due to the positive plate due because gamma ray
the positive charge of to the negative has no charge.
+ – the particle. charge of the
particle. However,
F the deflection is
O greater due to the
5

R small mass of
CHAPTER

M
electron.
radioactive source
5
Effect of magnetic field Small deflection Greater deflection No deflection
because alpha particle because beta particle because gamma ray
α has a large mass. has a very small has no charge.
S
mass.
N γ
F
β O
5

x x x x R
CHAPTER

α M
x x x x
γ
5magnetic
β field into
x x x x the paper

1 SPM
Clone
’03

The figure shows the path of radioactive rays, V and


Ray V Ray W
W.
A α β
B α γ
ray V C β γ
D γ β
ray W

paper aluminium lead


Comments
Alpha ray (α) can be stopped by a piece of paper
while beta ray (β) by an aluminium sheet and gamma
Which of the following shows the type of ray V and ray (γ) by a thick lead.
ray W? Answer C

Radioactivity 552
2 SPM
Clone
’05
It takes about 30 eV of energy to produce an ion pair for
Which of the following statements is not a air molecules. Radioactive particles have energies up to
characteristic of gamma radiation? several MeV. Therefore, the number of ions in air one
A It has a negative charge. radioactive particle can produce is in the order of 105.
B It is an electromagnetic wave.
C It has low ionising power.
D It can only be stopped by a thick lead or concrete Detection of Nuclear Radiation
wall.
Comments Most methods of detecting nuclear radiation are
based on these two properties:
Gamma ray has no charge. (a) The ionising effect of the radiation.
Answer A (b) The ability of the radiation to blacken a
photographic plate or film.

Geiger-Müller Tube (G-M Tube)

• A Geiger-Müller (G-M) tube consists of an • The end of the tube is sealed using a thin mica F
aluminium tube. It is filled with argon gas at low ‘window’. The window is thin enough to allow the O

5
pressure. Figure 5.4 shows a Geiger-Müller tube. alpha particles, beta particles and gamma rays to R

CHAPTER
• The central wire electrode is made of tungsten and pass into the tube. The gamma rays can also enter M
it acts as an anode—the positive electrode. via the tube wall because of its stronger penetrating
5
The aluminium wall of the tube acts as a cathode— power.
the negative electrode.

2 4
1
When a radioactive All these ions are discharged on reaching the electrodes.
A potential difference of about
particle or gamma
400 V is applied between the
ray enters the tube, aluminium tube (cathode)
central wire electrode and the
the argon gas atoms thin mica ‘window’ insulator wall of the tube.
are ionised.
+ – + –

3 + – + – 6
radioactive
The negative and positive source
The current pulse is
ions are attracted to the amplified and fed to an
scaler/ electronic counter such
wire and the wall low pressure central wire
ratemeter as a scaler or a
respectively. As the ions gas (mostly electrode (anode)
accelerate, they collide argon) ratemeter. A scaler
with other atoms, hence counts the pulses and
producing more ions. then display the total
The secondary ions number of counts or
produced are accelerated 5 pulses. A ratemeter
and they themselves A pulse of electrons flows round the circuit. gives the number of
collide with other atoms counts or pulses per
to create more ions. second.
Figure 5.4 Geiger-Müller tube

• When using a radiation detector such as the (b) The background reading is recorded after
G-M tube, allowance must be made to account for 3 minutes.
the background radiation. Background radiation Background reading = m
is low-level radiation whose sources are radioactive (c) The Geiger counter is reset. The tube is
materials naturally present in soil and rocks, and brought close to a radioactive source for
cosmic radiation. 3 minutes. The reading is again recorded.
• Below are the steps to be taken to account for the Counter reading = n
background radiation: (d) The actual reading is calculated as follows:
(a) The Geiger counter is switched on without the Actual reading = n – m
presence of radioactive material.

553 Radioactivity
Spark Counter
• A spark counter detects the presence of the alpha particles because they have a strong ionising effect on air
molecules.
• The counter consists of a wire gauze placed about 1 mm above a thin non-insulated wire.

4 5 3
The air molecules The heat and light energies generated A source of alpha particles is then
in the counter from these collisions produce the sparks. brought closer to the wire gauze.
become ionised.
The positive and
negative ions are wire gauze
radioactive source 1
attracted to the wire
gauze and thin wire A high voltage
xxxxxxxxxxxxx supply between
respectively. As the ionised –
ions move towards
+ – high the wire and the
air – +
their respective voltage wire gauze is
molecules
terminals, more supply increased slowly
collisions with air + until sparks are
molecules occur observed.
thin wire
causing secondary
ionisation.
2
F The voltage is then reduced slowly until no sparks are observed. This happens
O
when the thin wire is at a positive potential of between 3 and 4 kV.
5

R
CHAPTER

M Figure 5.5 Spark counter

5 • The intensity of the radiation is indicated by the number of sparks produced per unit time.
• The spark counter is unable to detect beta particles or gamma rays.

Gold Leaf Electroscope


• Alpha particles can be detected easily using a gold leaf electroscope. This is due to the strong ionising effect
of alpha particles.
radioactive + 4
3 source
+ + ionised air
A source of alpha particles – –

molecules
Due to the ionising effect of alpha
is then brought closer to + + + + + particles, both positive and negative
disc ions are produced in the air.
the disc.

metal
+
stalk 2
+
5 + + The gold leaf is deflected because the
The negative ions are attracted + +
+
charges on the gold leaf repel the charges
+ on the metal stalk.
towards the positive ions on the +
+ gold leaf
surface of the disc.

6
As more and more positive charges are
1 neutralised, the deflection of the gold leaf
The electroscope is decreases. This indicates the presence of
first positively charged. the alpha particles.

Figure 5.6 Gold leaf electroscope


• Nevertheless, this method is not suitable for detecting beta particles and gamma rays because they have a
weaker ionising effect on air molecules.

Radioactivity 554
Cloud Chamber

• A cloud chamber is a detector which displays the tracks of charged particles.

1
The felt strip round the top is soaked with alcohol.
3 This causes the air in the chamber to be saturated.
A source of
radioactive
particles is placed radioactive
inside. The particle source observer
or radiation ionises lid 5
the air in its path. felt strip soaked The trail of alcohol droplets
in alcohol is visible because it reflects
light light from the lamp.
source
tracks
black chamber floor
4 dry ice 2
The ions allow the The cooling unit beneath the
vapour to foam chamber floor is filled with dry
condense forming ice which cools the bottom
tiny alcohol of the chamber to around
droplets. –80 °C. As a result, the air in
the chamber becomes F
supersaturated. O

5
R
Figure 5.7 Cloud chamber

CHAPTER
M

• Table 5.6 shows the tracks produced by each type of radiation. 5

Table 5.6

Radiation Track Characteristics and explanation


Alpha (a) Thick, straight tracks of almost the same length.
particle, α (b) The tracks are thick due to strong ionising effect. A lot of alcohol
droplets are formed on the ions produced along the tracks.
(c) The tracks are straight because the alpha particles are not easily
deflected with its greater mass.
(d) The tracks are almost of the same length because each alpha
particle emitted has almost equal amount of energy.
Beta (a) The tracks are thin, twisted and different in length.
particle, β (b) The tracks are thin because the ionising effect of the particles is
weak.
(c) The particles are continually deflected by air molecules nearby
because they are light. Therefore, the tracks are twisted.
(d) The tracks have different lengths because the particles possess
different energies. The more energetic ones produce longer
tracks.
Gamma (a) The tracks are short, thin and scattered.
ray, γ (b) This is due to the very weak ionising effect of gamma rays.

555 Radioactivity
Photographic Plate or Film

1 A photographic plate works on the principle that


radioactive radiation can cause a chemical change
on the plate and produce a dark trace.
2 Basic working principles of a photographic plate
or film:
(a) A plate or film is coated with silver bromide.
(b) Radioactive radiations will strike the coating
and reduce the silver ion, Ag+ , to silver atoms,
Ag, in a chemical reaction. Figure 5.8

Ag + e Ag
5 This helps to monitor the amount of nuclear
(c) The silver produced is black in colour. radiation which the workers in the nuclear
(d) The degree of ’blackness’ on the plate or film industry are exposed to.
gives an indication of the intensity of the 6 The workers wear film badges which are then sent
radiation. regularly to a laboratory to be developed (they are
3 The photographic plate or film can detect all the developed just like our photographs). This gives us
three types of radioactive radiation. a way to measure the dose which each of the
4 With slight modifications, the photographic film workers have been exposed to.
can be used as a special badge or tag as shown in 7 The badges have windows made of different
F Figure 5.8, to record the dosage of radiation a materials which can help us to find out how much
O person is exposed to. of the radiation was due to particles, particles
R and rays.
CHAPTER

Detector
Radiation Photographic Gold leaf Spark Geiger-Müller Cloud
F plate electroscope counter tube chamber
O
5

Alpha particle R ✓ ✓ ✓ ✓ ✓
CHAPTER

M
Beta particle ✓ ✓ ✗ ✓ ✓
5 (Takes a long time)
Gamma ray ✓ ✗ ✗ ✓ ✓
Key: ✓ (can be detected) ✗ (cannot be detected)

Radioactive Decay
1 Radioactive decay is a process where an unstable nucleus becomes a more stable nucleus by
emitting radiations.
2 The process is spontaneous and random.
3 Table 5.7 explains the characteristics of radioactive decay.

Spontaneous Random
• The rate of decay cannot be increased or reduced, i.e., • It is impossible to predict which atom will
it cannot be controlled. decay at any moment of time.
• The decay happens on its own. • Each atom has the same probability of
• The decay is not affected by its chemical composition. decaying at any moment of time.
• The decay is not affected by physical factors such as
temperature and pressure.
Table 5.7

Radioactivity 556
Radioactive Decay

Alpha Decay Beta Decay Gamma Emission


1 An alpha particle is a nucleus 1 A beta particle is an electron 1 The emission of alpha and/or beta
of helium. It has 2 protons with a charge of –1e. particle from a nucleus sometimes
and 2 neutrons. Hence, it has 2 When beta decay occurs, a leaves the protons and neutrons in an
a charge of +2e. neutron disintegrates into a ‘excited’ arrangement.
2 When alpha decay occurs, proton and an electron 2 As the protons and neutrons
the radioactive parent nucleus according to the equation; rearrange to become more stable,
1 1 0
loses two protons and two 0 n 1 p + –1 e they lose energy. This is emitted as a
neutrons which carry away 3 The proton stays in the burst of gamma emission.
energy. nucleus but the electron is 3 Since gamma emission is not a particle
3 In other words, the proton emitted from the nucleus at but an electromagnetic radiation, there
number, Z, decreases by 2 high speed. is no change in the proton number
to become (Z – 2) and the 4 Hence, the proton number and nucleon number.
nucleon number, A, decreases becomes (Z + 1) and the 4 The only difference is that after the
by 4 to become (A – 4). nucleon number, A, remains gamma emission, the nucleus is less
4 The general equation for unchanged. energetic.
alpha decay: 5 The general equation for beta 5 The general equation for gamma
F
decay: emission: O

5
A A–4 4
X Z Z–2 Y 2He
parent daughter α A
X
A
Y
0
e
A
ZX X
A
Z + R
Z Z+1 –1

CHAPTER
nucleus nucleus particle parent daughter β particle (higher (lower M
energy) energy)
nucleus nucleus (electron)
5 Examples of alpha decay: 5
(a) 210 Po → 206 Pb + 24 He 6 Examples of gamma emission:
84 82 6 Examples of beta decay:
(a) 214 Bi → 214 Po + –10 e +
(a) 234
91
Pa → 234
92
U + –10 e 83 84

(b) 226
88
Ra → 222
86
Rn + 24 He
(b) 60
Co → 60
Co +
(b) 234
90
Th → 234
91
Pa + 0
–1 e 27 27

Radioactive Decay Series

1 When an unstable radioactive nucleus decays, the Nucleon number, A


resulting daughter nucleus may also be unstable. 238 238U

2 The daughter nucleus will continue to undergo a series α


of successive decays until a stable configuration is β β
234 234U
achieved. 234Th 234Pa
α
3 A radioactive decay series can be displayed on a graph
230
of nucleon number, A, against proton number, Z, or a 230Th

graph of number of neutrons, N, against proton 226Pa


α
Proton
number, Z. 226
number, Z
86 87 88 89 90 91 92
4 Figures 5.9 and 5.10 are two examples of radioactive
decay series. Figure 5.9
Figure 5.9 Part of 238 U decay series

• The nucleus which decays Always make sure that the total • Alpha decay can also be written as:
is called the parent nucleon number and the total proton
A α
α A–4Y
nucleus. number are equal on both sides of Z
X ⎯⎯→ Z–2
• The resulting nucleus is the equation. • Beta decay can also be written as:
called the daughter A β
β A
nucleus.
A
Z
X A–4
Z–2
Y + 42 He Z
X ⎯⎯→ Z+1
Y

• The daughter nucleus and Left side Right side • Gamma emission can also be
emitted particles are written as:
Nucleon number: A (A – 4) + 4 = A
known as the decay γγ A
products. Proton number: Z (Z – 2) + 2 = Z A
X ⎯⎯→ X
Z Z

557 Radioactivity
Number of neutrons, N (c)
238U
146
α
145
234Th
144 β
143 234Pa β234
U
142 Alpha particle Beta particle
α
141 The tracks are thick, The tracks are thin,
230Th
140 straight and almost twisted and different
α
139 of the same length. in length.
226Ra
138
137 α
222Rn Gamma ray
136
α The tracks are short, thin and
135
134
218Po scattered.
133 α
214Pb
132 β 214Bi
131
130
α
β
214Po 2
210Tl α
129 β (a) Identify P and Q in the equation below.
F 128
210Pb β 214Bi
226
88
Ra → 222
86
Rn + 42P + Q
O 127 β
5

R 126
210Po
(b) A nucleus 238 92
U decays by emitting an alpha
CHAPTER

M 125
α particle. The resulting nucleus decays further by
206Pb
124 emitting a beta particle producing a daughter
5 80 81 82 83 84 85 86 87 88 89 90 91 92 93
Proton number, Z nucleus Y. What is the symbol of Y?
(c) How many beta particles are emitted when 131 51
Sb
Figure 5.10 A complete decay series of 238U
decays into 131
55
Cs?
5 The decay series in Figure 5.10 can also be
Solution
represented as follows:
F (a) P is a helium nucleus or alpha particle.
α β O β Q is gamma radiation.
238
U → 234 Th → 234Pa → 234 U
5

92 90 R
91 92
(b) 238
92
U → 234
90
X + 42He
CHAPTER

M
→

α 234
X → 91Y + –10e
234
90
5 α The symbol of Y is 234 Y.
Ra ← 230
226
88 90
Th 91
(c) The difference in proton number is = 55 – 51= 4
The nucleon number (A = 131) remains unchanged.
131
51
Sb → 131
55
Cs + 4–10e
1 ∴ Four beta particles are emitted.
(a) When a radioactive source is brought close to the
disc of a positively charged electroscope, the
3 SPM
Clone
deflection of its gold leaf decreases. What ’05
happens to the gold leaf if a negatively charged
The following equation represents the fission of a
electroscope is used?
uranium nucleus.
(b) Name the radiations that can be deflected by both
magnetic and electric field.
235
92
U + 10n → 236
92
U → 141
56
Ba + x 92
36
Kr + y 10n + energy
(c) Sketch the track observed in a cloud chamber for What are the values of x and y?
each type of radiation.
x y
Describe the characteristics of each track.
A 0 1
Solution
B 1 1
(a) The deflection will decrease by the same
C 1 3
amount.
(b) Alpha particles and beta particles. D 2 2

Radioactivity 558
Solution 9 Different radioactive elements have different
Total proton number Total proton number half-lives, varying from millionths of a second
= to millions of years. Table 5.8 shows the half-
on the left on the right
92 = 56 + 36x + 0 (y) lives of some common radioisotopes.
36x = 36 Table 5.8
x= 1
Total nucleon number Total nucleon number Radioisotope Half-life
=
on the left on the right Uranium-238 5000 million years
236 = 141 + 92x + y (1) Uranium-235 700 million years
236 = 141 + 92 (1) + y
Plutonium-239 24 000 years
y= 236 – 141 – 92 = 3
Answer C Carbon-14 5700 years
Calcium-137 30 years
Cobalt-60 5 years
The Concept of Half-life
Iodine-131 8 days
Sodium-24 16 hours
The Typical Decay Curve
Strontium-93 8 minutes
1 The decay of a radioactive nucleus is a random Barium-143 12 seconds
process. F
O
2 It is not possible to predict the time when a

5
R
particular individual nucleus will suddenly

CHAPTER
M
undergo disintegration. Determining the Half-life SPM
’04/P1
SPM
’06/P1
SPM
’08/P1

3 However, for a large number of nuclei, we can 5


predict how many nuclei will disintegrate in a 1 Nuclei in a radioactive sample disintegrate at
certain period of time. random.
4 As time progresses, the number of atoms 2 Each nucleus has the same probability of
remaining decreases while the number of undergoing decay. At any one time, any nuclei
atoms disintegrated increases. can decay.
5 After a certain period of time, half of the initial 3 Due to a large number of atoms in a
number of atoms have disintegrated and the radioactive sample, radioactive decay may
other half remains. happen at each instant.
6 The time taken for half the atoms in a given 4 The average number of disintegrations per
sample to decay is called half-life. unit time in a radioactive sample is called the
7 After one half-life, the activity and the number activity.
of atoms remaining of any radioactive 5 Table 5.9 shows the changes which happen
substance are halved. during the decay of a radioactive sample.
8 Figure 5.11 shows a typical decay curve for a Table 5.9
radioisotope obtained by plotting the number
of atoms remaining against time. Number of
Total
atoms atoms which
Number of number
atoms remaining which have have not
of atoms
N
disintegrated disintegrated
Increases Decreases Constant

6 The rate of disintegration decreases as the


N number of atoms remaining which have not
2
yet disintegrated decreases. In other words, its
N activity decreases with time as the number of
4
N atoms that have not disintegrated decreases
8
O Time (s) with time.
T 2T 3T
7 Different radioactive elements decay at
Figure 5.11 different
Number of rates.
atoms remaining

N
559 Radioactivity
8 The half-life, T–1 of a radioactive isotope is the (d) After three half-lives from the start,

{ ( )} ( )
2
–1– × –1– –1– N
time taken for the activity of atoms of that 1 3 1
= –– N = –– N atoms
isotope to fall to half of its original value. 2 2 2 2 8
9 It can also be defined as the time taken for the 1 7
remain and N – –– N = –– N atoms have
number of radioactive atoms to decrease to 8 8
half of its original number. disintegrated.
10 Determining the half-life: (e) This decay process continues until a stable
(a) Let N be the number of original atoms in a atom is produced.
radioactive sample. (f) If x = number of half-lives
1 N = original number of atoms
(b) After one half-life, –– N atoms have
2 Nx = number of atoms remaining after
1 x half-lives
disintegrated and –– N atoms remain.
2
(c) After two half-lives from the start, ( )
1 x
∴ Nx = –– N
2
( ) ( )
–1– –1– N = –1– N = –1– N atoms remain and
2

2 2 2 4 11 Table 5.10 shows the changes in the number of


1 3 atoms undergoing decay.
N – –– N = –– N atoms have disintegrated.
4 4
F
O
Table 5.10
5

R
CHAPTER

M Half-life 0 1T1– 2T1– 3T1–


2 2 2
5 Number of atoms
remaining
N –1– N
2 ( )
–1– N = –1– N
2
2

4 ( )
–1– N = –1– N
2
3

8
Number of atoms that 1 1 1 3 1 7
N–N=0 N – –– N = –– N N – –– N = –– N N – –– N = –– N
have disintegrated 2 2 4 4 8 8
Key:
F 3 7
Number of atoms remaining 1 1 –– N
4
–– N
8
O N –– N –– N
Number of atoms that R 2 2
5

1 1
–– N –– N
have disintegrated 4 8
CHAPTER

Activ To simulate the radioactive decay


ity 5.1
Apparatus/Materials Procedure
1000 dice. 1 1000 dice are thrown onto a table.
Arrangement of apparatus 2 Any dice which shows a ‘6’ on its top face is
picked out and the number of dice remaining is
recorded.
3 The remaining dice are thrown onto the table
again.
4 Again, any dice with a '6' on its top face is
picked out and the number of dice remaining is
recorded.
Activity 5.1

5 Steps 3 and 4 are repeated 10 times.


6 All the results are recorded in a table.
Figure 5.12 7 A graph of the number of dice remaining against
the number of throws is plotted.

Radioactivity 560
Tabulation of data 2 Figure 5.13 shows the decrease in the number of
dice with the number of throws.
Table 5.11

Number Number of dice Number Number of dice remaining, x


Number of dice remaining
of which shows a ‘6’ of dice
throws on the top face remaining, x
1000

0 0 1000 750
1 167 1000 – 167 = 833
2 139 833 – 139 = 694 500

3 113 694 – 113 = 581


250
4 99 581 – 99 = 482
5 80 482 – 80 = 402 0 1 2 3 4 5 6 7 8 9 10
Number
of
6 67 402 – 67 = 335 T1 T1 throws
2 2
7 56 335 – 56 = 279
Figure 5.13
8 47 279 – 47 = 232
9 39 232 – 39 = 193
3 The curve is decreasing exponentially.
10 32 193 – 32 = 161 F
4 The half-life is the time for the number of dice O
remaining to be half of its initial number, i.e. the

5
R
Discussion time taken for x to decrease from 1000 to 500 or

CHAPTER
M
1 The assumptions are: 500 to 250, etc.
(a) Each dice represents a radioactive atom. 5 The half-life of this sample is about 3.8 throws. 5
(b) The dice which shows a '6' on its top face
represents an atom that has disintegrated.
Conclusion
(c) The remaining dice are considered as atoms
The theory of radioactive decay is based on the
that have not disintegrated.
assumption that the disintegrations happen
(d) The number of dice with '6' on the top face
randomly. As a result, the activity of radioactive
can be taken as the activity of a radioactive
sample decreases exponentially with time.
sample.
(e) Each throw of the dice represents one unit of
time.

Activ To study the half-life of a radioactive sample


ity 5.2
SPM
’05/P2(A)

Apparatus/Materials Procedure
A Geiger-Müller tube, a lead cylinder, a ratemeter, 1 An arrangement as shown in Figure 5.14 is set
and a radioactive sample. up.
Arrangement of apparatus 2 The count rate is taken every 10 seconds.
3 The activity of the sample is calculated.
ratemeter
4 The results are plotted as activity against time.
Activity 5.1 & 5.2

Tabulation of data
G-M tube
Table 5.12
lead
cylinder radioactive Time (s) 0 10 20 30 40
sample
Count 0 x1 x2 x3 x4
Figure 5.14

561 Radioactivity
Calculation Activity (s–1)
y
Table 5.13

Time (s) Activity (s–1)


x1 – 0
5 –––––– =y 1y
10 2

1y
x2 – x1 4
15 –––––– = y1 1y
10 8
Time (s)
x3 – x2 O 10 20 30 40 50
25 –––––– = y2
10 T1 2T1 3T1
2 2 2
x4 – x3
35 –––––– = y3 equal time
10
Figure 5.15

Discussion Conclusion
1 The activity decreases with time. The time taken for the activity of the sample to
2 The time taken for the activity to become half of decrease to half of its initial activity is known as its
F
its original value is constant. half-life, T1–.
O 2
5

R
CHAPTER

M
SPM SPM 3 By measuring the half-life of an unknown
Uses of Half-lives ’05/P1 ’06/P1
5 radioisotope, its identity can be determined.
1 A good knowledge of half-lives of different 4 The most significant use of the knowledge of
radioisotopes is essential in many different fields. half-lives is in the field of archaelogy to
2 Examples: determine the age of artefacts.
(a) Industries
(i) When being F used as tracers,
radioisotopesO used must have half-
5

lives that are Rlong enough for specific


CHAPTER

applications.MFor example, leaks in 3


underground5 pipes carrying oil can
be detected by injecting radioactive A radioactive source gives a reading of 640 counts
tracers into the flow. Then, a G-M per second when measured using a G-M tube.
tube can be used to detect leakage One day later, the reading drops to 40 counts per
from the surface above the pipe. second. What is the half-life of this source?
(ii) However, the half-lives must also be
short so that they can decay quickly,
Solution
leaving harmless daughter nuclei.
(iii) As an example, 1241Na is a suitable T—
1 T—
1

tracer with a half-life of 15 hours. 640 counts 2 320 counts 2 160 counts
(b) Medicine per second per second per second
(i) In the field of medicine, the radioisotope T—1
2
used must have a short half-life. This
is to prevent over exposure to radiation T—
1

for an unnecessarily long period of time. 40 counts 2 80 counts


(ii) Two examples of such radioisotopes are: per second per second
Activity 5.2

• Iron, 59Fe with a half-life of 45 days, 4 T—


1 = 24 hours
is used in testing for iron in blood 2
24
T—1 = — hours
plasma. 2 4
• Iodine, 131I with a half-life of 8 days, is = 6 hours
used in thyroid tests and treatments.

Radioactivity 562
4 5
A radioactive material has a half-life of 48 seconds. A radioactive source has a half-life of 50 minutes.
Its activity is 16 counts per second. What was its How much time does it take for 93.75% of its atoms
activity 6 minutes 24 seconds ago? to decay?
Solution Solution
The number of half-lives in 6 minutes 24 seconds Percentage of atoms that have not decayed
= 6 minutes 24 seconds = (100 – 93.75)%
48 seconds = 6.25%
384 seconds T— T— T— T—
= 1 1 1 1
48 seconds 100%
2
50%
2
25%
2
12.5%
2
6.25%
=8
Let the activity at 6 minutes 24 seconds ago = y The time it takes for 93.75% of its atoms to decay
()
∴ –1 y = 16
2
8 = 4 T—1
2
= 4 (50 minutes)
y = 16 × 28
= 200 minutes
= 4096 counts per second

F
5.2 O

5
R

CHAPTER
1 Complete the following. How much time is needed so that the ratio of M

(a) 234 Th → Pa + –10 e (b) 137


Cs → 137 Ba + —––––––– mass of Q
90 55 56 ——————— is 31? 5
2 A sample of radioactive P has 500 000 atoms. After mass of P
16 days, 375 000 atoms have decayed. What is the 5 The graph shows the decay of a radioactive element Q.
half-life of the sample? –1
Activity (s )
3 The figure shows lid
a cloud 120
radioactive
chamber. source
P
60

30
(a) Why is the surface P coloured black?
(b) Before using the cloud chamber, the lid is Time (minutes)
O 15 30
rubbed. State the reason for doing so.
(c) From the observed tracks, how can you deduce
that alpha particles have a larger mass compared How long will it take for the activity of Q to reach
to beta particles? Sketch diagrams to illustrate 3.75 s–1? F5/5/28
your answer. 6 Two radioactive materials, P and Q have half-lives of
4 A radioactive sample, P disintegrates into Q with a 2 minutes and 6 minutes respectively. Initially,
half-life of 4 hours. there are 256 g of P and 4 g of Q. How many
(a) The initial activity of sample P is 850 counts per minutes are required for both P and Q to have the
second. Find its activity 12 hours later. same mass?
1 of atoms in 7 A specimen of a radioactive isotope with a half-life of
(b) Calculate the time it takes for ––
16 4 hours has an initial activity of 720 counts per
sample P to remain undecayed. second. Draw up a table and plot a graph to show
(c) How much time does it take for 87.5% of atoms how the activity of this isotope in the specimen
in sample P to decay into Q atoms? changes during the first 16 hours.
(d) The table shows the initial masses of P and Q. 8 A radioactive sample has a half-life of y days.
Sample P Q A sample initially contains 3.0 × 1012 atoms.
Mass (g) 50 0 After 20 days, the number of remaining atoms is
1.875 × 1011 atoms. Find the value of y.

563 Radioactivity
SPM
5.3 Understanding the Uses Applications of Radioisotopes in ’07/P2(A)

of Radioisotopes Different Fields SPM


’07/P2(C)

Medicine
Radioisotopes
1 Radioisotopes are used in the diagnosis of
1 Isotopes are atoms of an element with the certain diseases.
same number of protons but different 2 Patients are given an intravenous injection of
numbers of neutrons. iodine-123 with a half-life of 13 hours or
2 In other words, isotopes of the same element iodine-131 with a half-life of 8 days. A detector
have the same proton number, Z but a is placed near the thyroid to read its activity or
different nucleon number, A. function.
3 Isotopes of a particular element have the same 3 A solution of sodium iodide labelled with
chemical properties because they have the radioactive iodine is also used to treat an
same number of electrons. However, their overactive thyroid gland and certain kinds of
physical properties are different because they thyroid cancer.
have different number of neutrons. 4 Sodium-24 is injected into the bloodstream to
4 Radioactive isotopes are called radioisotopes. detect the position of blood clots or
5 Some radioisotopes exist naturally. thrombosis in the blood vessels.
F For example: 31H, 178O, 40 19
K, 87
37
Rb, 238
92
U, 226
88
Ra 5 Brain tumour can be detected and treated
O 6 Others are produced artificially by using phosphorus-32.
5

R transmutations in a nuclear reactor. 6 Measurement of blood volume can be done


CHAPTER

M (a) A stable nucleus is bombarded by high by injecting a measured volume of human


5
speed alpha particles, neutrons or protons serum albumin (HSA) labelled with a sample
to produce artificial radioisotopes. of iodine-125 with half-life of 60 days of
(b) The bombarding particles are trapped in known activity. After a certain amount of time,
the nucleus creating a radioactive isotope. a sample of blood is withdrawn from the
(c) For example: patient and its activity is recorded. The volume
30
P, 52Mn, 47Be, 59
15 25 26
Fe, 24
11
51
Na, 24 Cr, 146C, 123
53
I of blood can then be estimated.
F
7 The following chart shows the properties of 7 Iron-59 is a suitable tracer to study the
O
radioisotopes which Rmake them suitable in
5

circulation of iron in the blood.


many applications. M
CHAPTER

8 The radioisotope cobalt-60 is used to


(a) destroy cancer cells in radiotherapy,
5
(b) sterilise medical equipments.
Properties
9 The radioisotope Caesium-137 is used to
• Emits radioactive radiation. destroy cancerous tissue.
10 Table 5.14 shows the uses of a few radioactive
• Radioactive radiations can kill cells. isotopes for diagnosis.
Table 5.14
• Radioactive radiations have different
penetrating ability with materials of Radioisotope Half-life Application
different thickness and densities.
Sodium-24 15 hours Plasma volume
• Radioactive radiations can cause cell Iodine-131 8 days Thyroid function
mutation. Iodine-125 60 days Vein flow
• Radioactive radiations can ionise Technetium-99m 60 hours Thyroid uptake
molecules.
• Its activity decreases with time. Industries
• Radioisotopes have the same chemical 1 In the manufacturing of paper, plastics, clothes
properties as non-radioactive isotopes and metal sheets, the thickness of each
of the same element. material must be controlled.

Radioactivity 564
(a) This is done by placing a radioactive (b) This happens when the tube detects a
source at one side of the material and a higher reading. Figure 5.18 shows how
detector on the other side. this is done.
(b) For sheets of metal, gamma rays are used. 6 A source of gamma rays can be used to take
For plastics, clothes and paper, beta photographs of metals to reveal cracks.
particles are used. 7 Radioisotopes are added into engine oil so
(c) The detector registers a higher count if the that its activity can be used to determine the
material is too thin and a lower count if it degree of wear and tear of engine parts.
is too thick. As a result, a computer will 8 Synthetic clothes tend to attract dust and dirt
make the proper adjustment to the from the surrounding air when the clothes are
thickness of the material. electrostatically charged. Alpha particles are
2 Figure 5.16 shows the mechanism used in used to neutralise the charges on the clothes.
industries to monitor the thickness of The alpha particles ionise the air particles into
materials produced. positive and negative ions which neutralise the
clothes.
rate monitoring
meter using 9 Gamma rays kill germs that cause food to
detector
computer spoil quickly. By exposing certain food to
material gamma rays, the food can be kept for a longer
period of time.
radioactive source
10 When latex is exposed to gamma rays, it F
(beta particle or gamma ray) becomes harder without the need for adding O

5
sulphur. R
Figure 5.16

CHAPTER
M
3 In order to ensure that containers such as cans
5
and food packages are filled to the specified
amount, a similar mechanism as shown in Two properties of sodium-24 which make it a suitable
Figure 5.16 is used. tracer to detect pipe leakage are:
4 When a container is inadequately filled, the (a) It decays by emitting gamma rays which can
detector will register a higher reading. This is penetrate the ground and the underground pipe.
shown in Figure 5.17. Alpha particles are not able to penetrate the ground.
(b) Its half-life of 15 hours enables it to decay to a level
detector which is harmless in a short time.

conveyor belt
container
Agriculture

radioactive source
1 Pests can be killed using radioactive rays
especially gamma rays.
Figure 5.17 2 The population of pests can also be controlled
5 Testing for leakage of underground pipes can by exposing them to a low dosage of gamma
be done by adding sodium-24 into the water rays. These rays induce mutation in the pests
in the storage tank. which stops them from reproducing.
(a) By moving a G-M tube above the under- 3 Radioisotopes such as nitrogen-15 and
ground pipe, a leakage can be detected. phosphorus-32 can be used as tracers in the
G-M tube is moved slowly
study of the effectiveness of fertilisers.
above the ground (a) These radioisotopes are added to soil
G-M tube
water.
(b) After the plant absorbs the soil water, the
beta particles
radioisotopes can track the uptake of
leakage fertiliser from roots to leaves in a plant.
4 Radioactive radiation is also used to induce
genetic mutation in a plant in order to
underground pipe
produce a better strain which has a higher
Figure 5.18 resistance against diseases.

565 Radioactivity
Archaeology

1 In the field of archaeology, it is important to be One difficulty with the carbon dating technique is that
able to determine the age of artefacts. the concentration of carbon-14 in the atmosphere
2 The method usually used for this purpose is changes over long intervals of time. This can be
called carbon dating. corrected on the basis of other data such as
(a) Cosmic radiations from outer space knock measurements on annual growth rings of trees.
out neutrons from nuclei in the Earth's
upper atmosphere.
(b) The neutrons then collide with nitrogen
6
nuclei to produce carbon-14.
1
0
n + 147 N → 146 C + 11 p It is found that 98.4375% of carbon-14 in an
(c) Living organisms, like plants and animals, archaeological artefact has disintegrated. Determine
absorb and give out carbon-14 when they the age of the artefact if the half-life of carbon-14 is
are alive. 5600 years.
(d) The half-life of carbon-14 is about 5730
years. So there is negligible disintegration Solution
over the lifetime of most organisms. Percentage of carbon-14 remaining = 100 – 98.4375
(e) However, when they die, no more carbon- = 1.5625%
F 14 is taken in. The carbon-14 they have Let the initial amount of carbon-14 = N
O already absorbed starts to decay to 1 N
The final amount of carbon-14 = –––
5

R nitrogen-14 by beta emission. 64


CHAPTER

M (f) The percentage of carbon-14 in a dead = 0.015625 N


1
() 1
n
5
plant or animal falls as the carbon-14 – N = ––– N
disintegrates. 2 64
()
1 1
n
(g) After 5730 years, the percentage of carbon- – = –––
14 falls to 50% of its initial value. 2 64
() ()
–1 = 1–
n 6
(h) The activity is proportional to the number
of atoms that have not disintegrated. 2 2
(i) By comparing the F activity of the dead ∴ n =6
O
sample with the activity of the same mass
5

R ∴ The age of the artefact = 6 × 5600 years


of living sample,M its age can then be
CHAPTER

= 33 600 years
estimated.
3 Measuring geological 5time:
(a) During the formation of rocks, some
radioisotopes such as uranium-238 are
trapped.
(b) As the decay continues, the proportion of The smoke detection system shown below has
uranium-238 decreases slowly resulting in Americium-241 which emits alpha particles. These in
the equally slow growth of its product, turn ionise the air, causing a small current to flow
lead-206. through the circuit. When a fire occurs, smoke particles
(c) An estimate of the age of the rock can be enter the chamber and attract ions. This results in the
reduction of the electric current in the detector. A circuit
inferred from the relative proportions of
senses the change in current and triggers the alarm.
lead and uranium in the rock.
Mass (g) hole on chamber allowing
smoke to enter
decayed uranium-238
– + source of americium-241
– +

ions NOT gate

alarm
remaining uranium-238 resistor system
Time (years)
O
Figure 5.19 The decay of uranium-238

Radioactivity 566
1.99 × 10–26
4 SPM
Clone ∴ 1 a.m.u. = —–——–——
’03 12
= 1.66 × 10–27 kg
The table shows the half-lives of the radioisotopes 4 The value is very close to the mass of one
G, H, I and J. proton or neutron.
5 The atomic mass unit (a.m.u.) is often used in
Radioisotope Half-life
nuclear physics because it is a more convenient
G 9 days unit to represent masses of particles which are
H 62 days very small.
I 82 days SPM SPM
Nuclear Energy ’04/P2(B) ’07/P1
J 5.5 years

Which of the radioisotopes is not suitable as a 1 In radioactive decay, one element changes into
tracer in the human body? another in a process called transmutation.
A G B H C I D J 2 The mass of the daughter particles and other
particles produced is less than that of the
Comments parent particle. This difference in mass is called
The radioisotope used must have a half-life long mass defect or mass loss.
enough for it to act as a tracer but not too long so as
Mass of Total mass of daughter
to cause side effects to the patient. Mass F
= parent – particles and other
Answer D defect O
particle particles produced

5
R

CHAPTER
M
3 The mass loss is converted into energy.
5.3 4 According to Einstein's Principle of Mass- 5
Energy Conservation, the change of energy is
1 Give two reasons for using sodium-24 as a tracer to
linked to the change of mass by the equation:
detect pipe leakage.
2 A factory produces plastic syringes for hospitals. The E = mc2
syringes are required to be sterile. However, the
factory cannot sterilise the syringes by heating where m = mass change, in kg
because the plastic would melt. Suggest a method c = speed of light, in m s–1
to overcome this problem. E = energy changed, in J
3 Radioactivity can be used to penetrate the body and
kill cancerous tissue. Why are gamma rays used,
rather than alpha particles for this purpose?
The value of c2 is so great that energy gained or lost by
4 In order to find out the condition of the thyroid
everyday objects is almost negligible. When a fast
gland, a patient is given food containing iodine-131,
moving car stops, its mass reduces by less than
which emits beta particles. Explain why beta particles
0.00000000000001%.
are more suitable for the purpose compared to
gamma rays.
7
5.4 Understanding Nuclear Below is an equation for the decay of radium-226.
Energy 226
Ra → 222 Rn + 24He + Energy
88 86

Atomic Mass Unit (a.m.u.) The masses of each atom:


226
Ra = 226.02536 a.m.u.
88
1 The atomic mass unit (a.m.u.) is used to 222
Rn = 222.01753 a.m.u.
86
measure the masses of atomic particles. 4
He = 4.00260 a.m.u.
2
2 It is defined as: (a) Find the mass defect in
Mass of carbon-12––atom (i) a.m.u. (ii) kg
1 a.m.u. or 1 u = ———–––————— ——–
12 (b) Calculate the amount of energy released in
3 Since the mass of one carbon-12 atom is (i) J (ii) eV
1.99 × 10–26 kg, [1 a.m.u = 1.66 × 10–27 kg; 1 eV = 1.66 × 10–19 J]

567 Radioactivity
Solution
(a) (i) Mass defect, m Amount of energy released, E
Mass of parent Total mass of = mc2
= – daughter particles
particle = (8.68 × 10–30) × (3.0 × 108)2
= 226.02536 – (222.01753 + 4.00260) = 7.81 × 10–13 J
= 0.00523 a.m.u. (ii) Since 1 eV = 1.60 × 10–19 J
(ii) Since 1 a.m.u. = 1.66 × 10–27 kg Amount of energy released, E
Mass defect, m = 0.00523 × 1.66 × 10–27 = 7.81 × 10–19
–13

= 8.68 × 10–30 kg 1.60 × 10


(b) (i) According to the Einstein’s Principle of = 4.88 × 106 eV
Mass-Energy Conservation: = 4.88 MeV

SPM
Nuclear Fission ’08/P1, P2(B)

1 Nuclear fission is a process involving the splitting of a heavy nucleus into two or more nuclei of
roughly equal mass with the release of several neutrons.
2 Nuclear fission seldom occurs spontaneously. Usually, it occurs when the heavy nucleus is bombarded
by a neutron.
F 3 Nuclei that undergo fission without initial neutron absorption are undergoing spontaneous fission.
O
3
5

R 1
Due to the greater average kinetic
CHAPTER

M As a result of energy, the surrounding atoms have a


neutron higher temperature and give out heat.
5 absorption,
the nucleus neutrons
undergoes 2
nuclear neutron
The reaction results in large
fission heavy mass defect which appear
(induced nucleus energy mostly as kinetic energy of the
fission). (mostly heat) fission fragments. They fly apart
at great speeds, colliding with the
F daughter particles surrounding atoms and raising
O their average kinetic energy.
5

R Figure 5.20 A nuclear fission (induced fission)


CHAPTER

M
4 Hence, nuclear fission can be a source of heat
energy. 5
5 Two typical examples of fission reactions: About 200 MeV of energy is released per atom during
(a) 235U + 10n → 144 Ba + 89 Kr + 3 01n + energy fission. This is about 50 million times greater than that
92 56 36
released per atom from a chemical reaction such as
(b) 235
92
U + 01n → 140
54
94
Xe + 38 Sr + 2 01n + energy burning!

8 neutrons

neutron
Below is an equation involving the fission of 235
92
U by a = 0.18 a.m.u. = 0.18 × 1.66 × 10–27 kg
fast-moving neutron. = 2.988 × 10heavy
–28
kg
nucleus
235
One atom of 92U releases an amount of energy given by: energy
235
92
U + 10n → 141
56
Ba + 92
36
Kr + 301n (mostly
E = mc2 heat)
Calculate the amount of energy released by 5 g of 235
92
U. = 2.988 × 10–28 × (3.0 × 10daughter
8 2
) particles
[235
92
U = 235.04 a.m.u., 92
36
Kr = 91.93 a.m.u., = 2.69 × 10 J –11

141
Ba = 140.91 a.m.u., 1
n = 1.01 a.m.u., In 235 g of 235
92
U, there are 6.02 × 1023 atoms of 23592
U.
56 0
1 a.m.u. = 1.66 × 10 kg]
–27 5
F5/5/34
In 5 g of 235
92
U, there are — — × 6.02 × 1023
235
Solution = 1.28 × 1022 atoms of 235 U
92
Mass defect, m The total amount of energy released by 5 g of 235 92
U
= (235.04 + 1.01) – (140.91 + 91.93 + 3 × 1.01) = 1.28 × 1022 × 2.69 × 10–11 = 3.44 × 1011 J

Radioactivity 568
Chain Reaction

1 If neutrons from the fission of uranium-235 continue to split other nuclei causing further
fission, a chain reaction has occurred.
2 The number of nuclei which undergo fission multiplies rapidly.
3 The energy released in this reaction is enormous compared to any chemical reaction.
4 Figure 5.21 shows a chain reaction.
90
38 Sr
235
92 U
n
143
235 54 Xe n
92
U
n
n
143
fission
n 54 Xe
neutron
144
n 55 Cs
n
94 90
Kr
36
n 38 Sr n

90 n
235
lost U 37 Rb F
n 92
neutron 94 O
1 36 Kr n

5
n
0 R

CHAPTER
235 M
92
U 90 n
139
Ba n 37Rb
56
n
5
n 235 139
Ba
92U 56

n fission
neutron
90
n 37 Rb n
235
92U 144
55Cs
n

144
n
235 Cs
92
U 55

1 st generation neutron 2nd generation neutron 3rd generation neutron 4 th generation neutron

Figure 5.21 Chain reaction


5 In nuclear reactions, a controlled chain reaction F5/5/35
takes place with a steady release of energy. 9
6 However, when employed in nuclear weapons,
an uncontrolled chain reaction takes place A nuclear explosion released 8.2 × 1013 J of energy.
with a huge amount of energy being released What is the mass of uranium-235 which is converted
in a short span of time. into this amount of energy?
7 In order for a chain reaction to take place, a [Speed of light = 3.0 × 108 m s–1]
minimum of one neutron from each fission Solution
must trigger further fission. E = 8.2 × 1013 J, c = 3.0 × 108 m s–1,
8 At the same time, the mass of fission material m = mass of uranium-235
must exceed a certain minimal mass known as E = mc2
the critical mass. If the material is less than E–
this value, too many neutrons escape without m= —
c2
hitting any nuclei, preventing a chain reaction
= 8.2 × 108 2
13
from happening. (3.0 × 10 )
9 The critical mass of a uranium sample is also = 9.11 × 10–4 kg
affected by its shape.

569 Radioactivity
Nuclear Fusion
1 In nuclear fusion, two or more small and light 4 Fusion is much more difficult to achieve than
nuclei come together to form a heavier nucleus. fission because the hydrogen nuclei repel each
2 This process is a accompanied by the release of a other. Therefore, the nuclei must be heated to 10 K
huge amount of energy. or more so that the nuclei will have enough of
kinetic energy to overcome the electrical repulsion
between the nuclei.
+ + 5 The Sun get its energy from the fusion of hydrogen
2
1H
1
0n
nuclei.
4
3
1H 2 He
energy
(a) Two hydrogen isotopes, deuterium ( H)
collides with tritium ( H) to form a helium
Figure 5.22 nucleus, ( He) at a high temperature.
(b) A neutron is released accompanied by a mass
3 Below are two examples of fusion reactions: defect.
(c) The mass defect produces a huge amount of
(a)
energy.
(b) 6 A hydrogen bomb uses the principle of nuclear
fusion for its design.

F 10
O
R
Determine the energy released in the reaction. Solution
CHAPTER

F
O
5

R
CHAPTER

Nuclear Reactor Reason for the Use of Nuclear Energy

1 A nuclear reactor produces tremendous amount of 1 Production of energy from nuclear fuels involves a
energy through nuclear fission. lower cost. This is because a small amount of
2 Figure 5.23 shows a schematic diagram and the nuclear fuel can provide a large amount of energy.
function of each main component in a nuclear 2 Nuclear reactors are relatively safe especially with
reactor. state of the art technology.
3 The energy liberated from the fission of nuclear 3 The dwindling supply of fossil fuels making it
fuel heats the surrounding water. neccessary to use alternative sources of energy.
4 As a result, steam is generated to drive turbines, 4 The use of nuclear energy does not emit
which in turn drive the electrical generators. greenhouse gases such as carbon dioxide.
5 Nuclear reactors are used in the production of
(a) high-intensity neutron beams for research, Reason against the Use of Nuclear Energy
(b) artificial radioactive isotopes for medical
research, 1 Radioactive residues from nuclear stations have
(c) fissionable transuranic elements such as rather long half-lives.
plutonium from uranium-238. 2 There is a possibility of leakage in the radioactive
waste containers placed underground or underwater.
3 High cost of constructing a nuclear power station.
4 No matter how advanced the technology is,
accidents due to human error can happen.

Radioactivity 570
To prevent the escape of
The rate of the fission harmful radiations. Water in the generator is heated and changed into
reaction is controlled steam. The steam then drives the turbines.
by inserting or
thick concrete wall
withdrawing these steam generator To turn the dynamo in the electrical
rods. The nuclei in generator to produce electricity.
the rods absorb
high pressure turbine
neutrons without steam generator
undergoing any
reactions.
Sometimes, the rod is
made of cadmium.
hot
water electric cable

high
control rod pressure pump low pressure
(boron rod) water steam steam condenser

uranium
fuel rod

cold coolant in
graphite low
pump water pressure
core
moderator water

coolant out

cooling tower
F
primary loop secondary loop
O

5
To take away the heat from the
To slow down the fast-moving R
nuclear reactor. Substances with high

CHAPTER
neutrons produced by the fission. specific heat capacity such as ‘heavy’ M
Fission reactions occur in the uranium rod to
In some nuclear power plant, the water and carbon dioxide are used.
produce nuclear energy. Neutrons smash into the
moderator is water. 5
nucleus of the uranium atoms in a controlled chain
reaction, releasing a large amount of energy.

Figure 5.23 A nuclear power plant

5.4

1 (a) State three safety issues with respect to the 5 In each reaction below, calculate
use of nuclear energy. (i) the mass defect, in kg,
(b) Give three advantages of a fusion reactor. (ii) the amount of energy released, in Joule and
(c) About one-eighth of the caesium-137 in MeV.
the Chernobyl reactor was released. (a) 2
+ 21 H → 3
+ 11 H
1H → 1H
Caesium-137 has a half-life of 30.17 years.
How many years would it take for the 3 2 4 1
(b) 1H + 1H →
→ 2 He + 0n
activity of caesium-137 to reduce to 0.0625
of its initial activity? 2 2 3 1
(c) 1H + 1H →
→ 2 He + 0n
2 (a) Why is fusion much more difficult to achieve
than fission? 3 2 4 1
(b) What is the difference between fusion in a (d) 2 He + 1H →
→ 2 He + 1H
nuclear reactor and in the Sun? 2 3
(c) Name two coolants that are usually used in [ 1H = 2.01410 a.m.u.; 1 H = 3.01605 a.m.u.;
reactors. Give a reason for using these 1 3
coolants. 1H = 1.00783 a.m.u.; 2 He = 3.01493 a.m.u.;

4 1
3 In nuclear fusion, uranium-235 is bombarded 2 He = 4.00260 a.m.u.; 0 n = 1.00867 a.m.u.;
with a high-speed neutron. Why is it more
8 –1 –19
difficult to bombard uranium-235 using a proton? c = 3.0 × 10 m s ; 1 eV = 1.60 × 10 J;
4 Calculate the amount of energy produced when
–27
1.2 g of uranium is converted completely into 1 a.m.u. = 1.66 × 10 kg]
energy.

571 Radioactivity
5.5 Realising the Importance of Proper Management of Radioactive
Substances
Negative Effects of Radioactive Substances

1 Radioactive substances give out radiations which are harmful to living


things. This is due to the ionisation and penetrating properties of
these radiations.
2 As the radiations pass through living cells, they ionise the
neighbouring atoms or molecules. The reactive ions produced will
(a) interfere with the chemical processes in the cell.
(b) induce mutations in the genetic structure of the cell.
3 At the same time, the radiations might kill the cells in the body tissues.
4 If too many cells are killed, the organism dies.
5 The amount of damage inflicted on a human being depends on four
factors as shown below.

Dosage and exposure time Exposure of different parts of the body


F 1 Exposure to high dosages of radiation Actively dividing cells are more
O in a short period of time results in vulnerable to radiations. Generally, skin
5

R immediate symptoms such as cells can withstand a higher dosage of


CHAPTER

M vomitting, increase in body radiation compared to the other internal


temperature, change in the blood organs.
5 composition, etc.
2 Continuous exposure to intermediate
dosages of radiation causes symptoms Factors
affecting the Types of radiation
that appear after a few years such as
severity of 1 Alpha particles outside the body are
cancer, infertility, cataract, etc.
radiation harmless because they can be stopped
F by the human skin.
O 2 Beta particles are more harmful
5

Methods of insertion into Rthe body because they have higher penetrating
CHAPTER

M power.
Alpha particles that enter the body through
food or breathing cause more damage 3 Gamma rays are the most harmful as
5
from the inside due to its high ionising they can penetrate deep into the skin
effect. and inflict damage onto the cells.

6 The harmful effects of radiation on humans can be divided into two


categories shown in the following chart.
Harmful effects of radiation on humans

Somatic effect Genetic effect


1 Damage to all parts of the body except the reproductive organs. 1 Damage to reproductive cells.
2 Symptoms include 2 Genetic defect can be passed
(a) fatigue, down to the next generations.
(b) vomitting, 3 Examples of genetic defects:
(c) hair loss, (a) Down Syndrome
(d) infertility in male, (b) Klinefelter Syndrome
(e) severe skin burn, (c) Turner Syndrome
(f) leukemia and cataracts (appear many years after exposure).

Radioactivity 572
5 Therefore, the management of radioactive
Precautionary Steps in Handling SPM
’08/P2(B) wastes is extremely important so that human
Radioactive Substances
beings are not directly exposed to the harmful
1 Experiments involving radioactive substances are radiations.
conducted in a room enclosed by concrete walls. 6 At the same time, the environment and sources
2 Strong radioactive substances are handled of water must be protected from contamination
using remote-controlled mechanical arms of radioactive wastes.
from a safe distance. 7 The management of radioactive wastes is
3 Weak radioactive substances can be handled based on the category which the wastes belong
using a pair of tongs or tweezers. to. The three different categories are:
4 Radioactive wastes must be disposed of using (a) Low level radioactive wastes
suitable and safe methods. (b) Intermediate level radioactive wastes
5 Rooms, buildings, containers and radioactive (c) High level radioactive wastes
storage places must be labelled with the sign 8 The characteristics used to categorise radioactive
wastes:
for radioactive substances.
(a) Half life (d) Type of radiation
6 Radioactive substances are kept in thick lead
(b) Heat emission (e) Radioactivity
containers.
(c) Concentration
7 Protective suits and gears such as gloves, and
eye glasses made of lead are to be used at all
times when handling radioactive substances. F
Low Level Radioactive Wastes
O
8 Shields made of thick lead protect workers in

5
R
their workplace from harmful radiations. Sources: Hospitals, nuclear power stations,

CHAPTER
M
9 Workers handling radioactive substances must industries, research laboratories.
wear special badges which detect the amount Examples: Contaminated equipments, shoes, 5
of radiation they are exposed to. biohazard suit, clothing, wrappers, air
10 Food and drinks are not allowed in places filters, gloves, etc.
where radioactive substances are handled. Half-life: Short
Radioactivity level: Low
Nuclear Energy as Alternative Source of Energy Management: Solid wastes are stored in special
drums and then buried underground.
1 Fossil fuels such as petroleum, coal and
natural gas are dwindling in their supply. Intermediate Level Radioactive Wastes
2 In the future, these sources of energy will be
depleted. Sources: Nuclear power stations, industries,
3 Burning of fossil fuels also contributes to the research laboratories.
pollution of the environment. Examples: Components in nuclear reactors,
4 Therefore, an alternative source of energy must chemical sediments.
be explored to replace fossil fuels. Half-life: Long
5 Nuclear energy is a promising alternative Radioactivity level: High
source of energy in the future. Management: Radioactive wastes are placed in concrete
blocks and then buried underground.
Management of Radioactive Wastes
High Level Radioactive Wastes
1 Radioactive wastes are the remaining isotopes
which results from the radioactive decays. Sources: Nuclear power stations.
2 Some industrial processes produce radioactive Examples: Fuel rods used in nuclear power stations.
wastes in the form of solids, liquids or gases as Half-life: Long
by-products. Radioactivity level: High
3 These wastes emit harmful radiations that Management: Fuel rods are submerged in a pool of
endanger human beings and the environment. water to cool them down. The rods
4 For example, a lifetime exposure to radon-222 are then stored in a steel container
reduces the average life expectancy by about 40 which are buried underground at a
years. depth of between 500 m and 600 m.

573 Radioactivity
5.5

1 What are the factors which affect the severity of radiation?


2 Why are food and drinks not allowed in places where radioactive substances are handled?
3 State two precautionary methods in handling radioactive substances.

Histories of Nuclear Accidents 1986 – Chernobyl, Ukraine


1952 – Chalk River, Ontario, Canada This accident resulted from a combination of unstable
Explosion of nuclear reactor core. The accident was due design and human error during a test of the emergency
to human error. No injuries reported. core cooling system. At the time, too many control rods
were withdrawn to compensate for a decrease in power
1961 – Idaho, United States of America due to the build-up of neutron absorbers such as Xe-135.
Three workers were exposed to high dosages of radiation In 4 seconds, the power level rose 100 times resulting in
F which resulted in deaths. a steam explosion. Pipes in the core cooling system
O
Physics Blog ruptured. The graphite moderator caught fire and burned
5

R 1979 – Three Mile Island, United States of America for several days. Radioactive particles escaped to the
CHAPTER

M The pump responsible for channelling water to the atmosphere and spread to other European countries. 31
reactor malfunctioned. As a result, the temperature of the deaths were reported. Other side effects included skin
5
core increased dramatically. Radioactive dusts escaped to burns and long-term effects such as thyroid cancer and
the atmosphere. leukemia.

F
O
5

R
CHAPTER

1. The number of protons in the nucleus of an atom is 10. Alpha decay: ZAX → ZA –– 24Y + 24He
called its proton number, Z. 11. Beta decay: ZAX → Z +A 1Y + –10e
2. The total number of protons and neutrons in an
12. Gamma: A
Z
X → ZA X + γ
atom is called its nucleon number, A.
3. Radioisotopes are isotopes with unstable nuclei. 13. Half-life is the time taken for half the atoms in a
4. Radioactivity is the spontaneous disintegration given sample to decay.
of unstable nucleus into a more stable nucleus 14. E = mc2 where E = energy in Joules (J), m = mass in
with the emission of energetic particles or kilogram (kg), c = 3.0 × 108 m s–1.
photons. 15. Mass defect
5. There are three types of radiation: alpha particle, = Mass of all parent nuclei – Mass of all daughter nuclei
beta particle and gamma ray. 16. Nuclear fission is a process involving the splitting of
6. Alpha particle is a positively-charged helium a heavy nucleus into two or more nuclei of roughly
nucleus. equal mass with the release of several neutrons at the
7. Beta particle is a negatively-charged fast moving same time.
electrons. 17. A chain reaction is a nuclear reaction in which the
8. Gamma ray is a high frequency electromagnetic number of nuclei which undergo fission multiplies
radiation. rapidly.
9. Most detectors of nuclear radiation operate using 18. Nuclear fusion is a process involving two or more
the ionising effect of the radiation. small and light nuclei joining together to form a
heavier nucleus.

Radioactivity 574
5
Multiple-choice Questions

5.1 A 5 The following equation


The Nucleus of an Atom SPM represents alpha decay.
Clone
Mass (g) Mass (g)
’11
X → Y + 42He
5.2 Radioactive Decay
10 10
Which of the following is correct
5 5
1 Which diagram shows the about element Y?
time time
SPM
Clone structure of the 126C atom and the 0 8 16 24 32 40 (hours) 0 8 16 24 Proton
32 40 Nucleon
(hours)
’09 composition of its nucleus?
number number
B Mass (g) Mass (g)
Key : A Increases Increases
neutrons 10Mass (g) 10
protons by 2 by 4
eletrons 5
10 5 Decreases Increases
B
A C 5 time by 2 time by 4
F
0 8 16 24 32 40 (hours) 0 8 16 24 32 40 (hours)
time C Increases Decreases O

5
0 8 16 24 32 40 (hours) by 2 by 4 R

CHAPTER
C M
D Decreases Decreases
Mass (g) by 2 by 4 5
B D 10
6 Which radioactive detector device
1.1 Signifi
5 cant Figure SPM allows the user to differentiate the
Clone

time ’11 type of radioactive ray?


0 8 16 24 32 40 (hours) A Spark counter
1.1
DSignificant Figure B Geiger Muller tube
C Photographic film
2 Diagram 1 shows three types of Mass (g)
D Cloud chamber
SPM
radioactive rays, P, Q and R, directed
Clone
10
’05
towards a hand, a sheet of aluminium 7 Diagram 3 shows a particle
5 moving in a magnetic field.
and a thick concrete slab.
time
0 8 16 24 32 40 (hours)
P N

Q 4 Diagram 2 shows a graph of


R SPM
nucleon number against proton
Clone
S
hand ’08 number.
aluminium concrete

Diagram 1 Diagram 3
Which of the following rays are
represented by P, Q and R? The particle is deflected upwards
after passing through the
P Q R magnetic field. The particle is
A Beta Alpha Gamma A alpha particle C proton
B beta particle D neutron
B Alpha Beta Gamma
C Gamma Alpha Beta Diagram 2 8 A radioactive sample is found to
Nucleus P decays by emitting an have a mass of x g at 7.00 a.m. on
D Beta Gamma Alpha
alpha particle and a beta particle. Monday. At 7.00 p.m. on Thursday,
3 The initial mass of sodium-24 is Which of the following nuclei does 15
of the initial mass has
SPM
10 g and its half-life is 16 hours. P decay to? 16
Clone
’09 Which graph shows the decay A Q C S decayed. The half-life of the
curve of sodium-24? B R D T radioactive sample is

575 Radioactivity
A 18 hours A 48 g Number Number Number
B 19 hours B 50 g of of of
C 20 hours C 56 g protons electrons neutrons
D 21 hours D 64 g A
m–n m m – 4n
9 P and Q are radioactive 13 When a count meter is first B
3m + 2n n 2m + 3n
substances. The half-lives of P and connected to a G-M tube, it C
gives a reading of 70 counts m–n m–n m + 4n
Q are 8 days and 4 days D
respectively. If the initial masses of per second. A radioactive 3m + 2n m 3m – 2n
P and Q are 10 g and 20 g source is then placed near the
respectively, how many days does tube. The reading is 1750 16 Diagram 5 shows a graph of
it take for the two substances to counts per second. After 40 activity against time for a
have equal mass? minutes, its reading drops to radioactive element.
A 4 days 490 counts per second. What Activity (counts per second)

B 6 days is the half-life of the radioactive


N
C 8 days source?
D 16 days A 10 minutes
B 15 minutes
10 A neutron is aimed at the nucleus C 20 minutes
of an atom. Which of the following E 30 minutes
shows the correct path of the
neutron 0 Time
14 Diagram 4 shows a graph of
nucleus Y
F neutron? T1 T1 T1 (hours)
nucleus activity against time for three 2 2 2
O A
radioactive elements.
neutron
5

R neutron Diagram 5
nucleus
CHAPTER

M nucleus Activity (counts per second) What is its activity at Y hours?


5
neutron neutron A —1– N C —1– N
SPM
Clone
2 8
’07 B 1 1 N
B —– N D —–
nucleus neutron 4 16
nucleus
M 17 Table 2 shows the activity of a
neutron L
K radioactive element.
neutron nucleus
C Fnucleus nucleus Time Table 2
nucleus
neutron (hours)
Onucleus
Time Activity
5

neutron
nucleus neutron R Diagram 4
(s) (counts per second)
CHAPTER

M
neutron F5/5/41a-d
Table 1 shows the half-lives for the 0 410
D 5
neutron elements.
50 201
neutronF5/5/41a-d Table 1 100 99
nucleus Element P Q R 150 x
nucleus

Half-life –1– hour 15 1200


nucleus Find the most suitable value for x.
neutron nucleus
2 minutes seconds A 35 C 65
neutron
B 56 D 70
11 P and Q have half-lives of 30 Which of the following is
minutes and 40 minutes correct?
5.3
F5/5/41a-d Initially, they have
respectively.
K L M
The Uses of Radioisotopes
equal mass. Find the ratio of the
F5/5/41a-d
remaining mass P to Q 2 hours A Q P R 18 Which of the following radioisotopes
later. is used in smoke detectors?
B R Q P
A 1:3 C 3:1 A Sodium-24
C Q R P B Americium-241
B 2:1 D 1:2
D P R Q C Cobalt-60
12 The half-life of element P is D Phosphorus-32
SPM
Clone 12 days. A sample contains x g of 15 It is given that the nuclear symbol 19 In a factory, a radioactive source
’08 element P. After 60 days, only 2 g of an element X is 2mm+–3nn X. Which SPM
and a detector are used to ensure
Clone
of element P still remains. of the following shows the correct ’07 that the containers are filled to a
What is the initial mass of number of subparticles in X ? specified amount. The set-up is
element P? shown in Diagram 6.

Radioactivity 576

1.1
detector What is the name of this process? 23 When a sample of radioactive
A Chain reaction SPM
Clone substance decays, the energy
container B Nuclear fusion ’10 released is 7.05 x 10 –13 J. What is

C Nuclear fission the mass defect?


conveyor
belt radioactive D Nuclear reaction A 7.83 × 10–30 kg
source
B 5.16 × 10–21 kg
Diagram 6 21 In order for a chain reaction to C 3.25 × 1020 kg
SPM take place, D 1.18 × 1029 kg
Which of the following radiations is Clone
’10 A pressure must be applied to
the best for this device? 24 The following equation represents
the parent particles.
SPM a nuclear fusion.
Radiation Half-life B neutron must move at a very Clone

high speed. ’11 2 H + 1H → 3He +  + energy


A α 5 days 1 1 2
C the daughter particles The man defect from the
B β 50 years produced are radioactive. reaction is 0.007 u. What is the
C γ 12 years D the mass of fission material energy released in the reaction?
D α 3 months must exceed critical mass. (1 a.m.u. = 1.66 × 10–27 kg;
22 In which of the following can c = 3.0 × 108 m s–1)
nuclear fusion be found? A 1.389 × 102 J
5.4 Nuclear Energy A Sun B 3.142 × 10–10 J
B Medical laboratories C 2.935 × 10–11 J
20 A large amount of energy is C Nuclear reactors D 1.046 × 10–12 J
SPM
Clone released when one heavy nucleus D Waste factories F
’09, is split into two light nuclei. O

5
’10
R

CHAPTER
M

Structured Questions
1 The diagram shows the activity-time graph of 2 (a) Diagram 2 shows an incomplete path of a beam
SPM radioisotope sodium-24. This radioisotope emits beta
Clone
of radiation consisting of α-particle, β-particle and
’10 particles and is used as a tracer to detect leakage in γ-ray.
underground pipes.
magnetic field
Activity (counts per second)

1600
1.1 Significant Figure α, β and γ

1200
Diagram 2
800
Complete the diagram to show the path of each
400 radiation. Label each path with the type of
radiation. [3 marks]
Time (hour) (b) Fill in the blanks. [2 marks]
0
5 10 15 20 25 30
(i) 228
90
Th → 42 + 224
88
Ra
(a) State one characteristic of beta particles. [1 mark]
(b) Based on the diagram, determine the half-life of (ii) 40
19
K→ e+ 0
–1
40
Ca [2 marks]
sodium-24. Show on the graph, how you (c) State the number of protons and neutrons in each
determine the half-life of sodium-24. of the following.
Half-life of sodium-24: hours (i) 234
91
Pa
[2 marks] (ii) 238
92
U
(c) Based in the half-life, state why sodium-24 is (iii) 228
90
Th [3 marks]
suitable to be used as the tracer. (d) Explain the change that happens in the nucleus of
(d) Calculate the time taken for radioisotope sodium- a radioactive atom that undergoes beta decay.
1 [2 marks]
24 to reduce to of its initial activity.
8

577 Radioactivity
(e) (iii) State one characteristic, in terms of half-life,
Statement I: for the radioactive source. [1 mark]
Cobalt-60, which emits γ-rays, is used to (iv) Explain your answer in (a)(iii). [1 mark]
destroy harmful microorganisms on medical
instruments. Table 2 below shows a few isotopes with their
Statement II: respective half-lives and radiation emitted.
A patient is given food containing iodine-131 Table 2
which emits β-particles to determine the
functioning of thyroid gland. Isotope Half-life Radiation
Uranium-239 24 minutes β
(i) For statement I, explain why γ-rays are more Polonium-213 4 × 10–6 seconds α
suitable than β-particles. [3 marks] Bromine-82 36 hours β, γ
(ii) For statement II, explain why β-particles are Strontium-90 59 years β
more suitable than γ-rays. [3 marks]
(v) Based on your answer in (iv), choose the
3 (a) Diagram 3 shows the monitoring of the thickness most suitable isotope from the table to
of metal foil produced in a factory. detect the thickness of the metal foil.
A radioactive source, a detector and a ratemeter [1 mark]
are used to detect the thickness of metal sheets. (vi) Sketch a graph of activity per minute against
time for the isotope in (v). [3 marks]
rollers radioactive source (b) At high temperature, hydrogen-2 (deuterium)
F metal
O radiation reacts according to the equation below.
5

R 2
H + 21H → 32He + 10 n
CHAPTER

1
M detector
ratemeter [21H = 2.01410 a.m.u., 32He = 3.01493 a.m.u.,
5 1
n = 1.00867 a.m.u., 1 a.m.u. = 1.66 × 10–27 kg]
Diagram 3 0

(i) State the type of reaction above. [1 mark]


(i) Name the type of radiation which is suitable
(ii) Calculate the mass defect, in a.m.u., and the
to detect the thickness of metal foil. [1 mark]
energy released. [4 marks]
(ii) Give the reason for using the radiation in
(iii) What is the advantage of using the above
(a)(i), but not the other radiations. [3 marks]
F reaction in a nuclear power station?
O [1 mark]
5

R
CHAPTER

M
Essay Questions
5
4 Diagrams 4 and 5 show the rate of decay of radioisotopes strontium-93 and barium-143 respectively.
SPM
Clone Activity (counts per minute) Activity (counts per minute)
’09
2000 2000
1500 1500
1000 1000
500 500
0 Time 0 Time
480 960 1440 1720 (seconds) 12 24 36 48 (seconds)
Diagram 4 Diagram 5

(a) (i) What is the meaning of radioisotope? (b) Alpha and beta particles are called ionising
[1 mark] radiation. Explain the meaning of the phrase
(ii) Based on Diagram 4 and Diagram 5, ‘ionising radiation’.
compare the shape of the graphs, the State two effects of ionising radiations on cells.
change of the activity with time, and the [4 marks]
time taken for the activity to reduce to half (c) The use of radioisotopes in the treatment of
its original activity. State the name for the cancer can cause side effects to the patient due to
time taken for the activity to reduce to half the radioactive ray.
its original activity. Using the knowledge about radioactivity, explain
[5 marks] the steps taken to reduce the side effects.

Radioactivity 578
In your explanation, emphasize the aspect of: Using Diagram 8, suggest and explain:
(i) the type of radiation used, (i) The equipment to be used in handling
(ii) total dose of radiation, radioactive waste.
(iii) dose rate, (ii) The equipment to detect the radiation
(iv) the half-life of the radioisotope. emitted.
[10 marks] (iii) Other precautions that need to be taken
when handling radioactive waste.
5 Diagrams 6(a) and (b) show the ionisation due to two
[10 marks]
radioactive emissions X and Y. Electrons are knocked
SPM
Clone off atoms in the air during ionisation. 6 Radioactive isotopes are used to diagnose medical
’08 SPM problems and in the detection of leaks in
Clone
’06 underground pipes.
(a) A patient is given a solution containing sodium
iodide labelled with radioactive iodide. The thyroid
gland absorbs the iodine as time passes. The rate
(a) of uptake, i.e. the activity of the patient‘s thyroid,
and the activity of an identical ‘phantom’ solution
prepared at the same time is measured 24 hours
later.
Diagram 9 shows the arrangement of apparatus
for such a test.
F
(b) COUNTER O

5
Diagram 6 R
photomultiplier tube

CHAPTER
M
(a) State the meaning of radioactivity. [1 mark] lead
(b) Using Diagrams 6(a) and (b), scintillation crystal 5
(i) compare the number of atoms ionised,
[1 mark]
(ii) compare the ionisation power of the lead plate
radioactive emissions. [2 marks]
(c) State the particles which represent
(i) radioactive emission X, [1 mark] thyroid gland
(ii) radioactive emission Y. [1 mark]
(d) Diagram 7 shows a type of nuclear reaction.
(i) Name the type of nuclear reaction in Diagram 9
Diagram 7. [1 mark] A lead plate is placed between the photomultiplier
tube and the patient before the test begins. A
reading is then measured.
A normal thyroid has a percentage uptake of
between 20% and 50% after 24 hours.
Table 3 shows the readings obtained after 24 hours.
Table 3

Diagram 7 Count rate of the thyroid 845 s–1


(ii) Briefly explain the process of nuclear chain Count rate of the ‘phantom‘ solution 1410 s–1
reaction. [3 marks] Background count rate 120 s–1
(e) Diagram 8 shows a worker in a radioactive waste
spill site. (i) Give the definition of half-life. [1 mark]
(ii) State the reason for placing the lead plate
between the photomultiplier tube and the
patient before the test begins. [1 mark]
(iii) The percentage uptake is calculated using
the formula:
The corrected count rate
of the thyroid
× 100%
The corrected count rate
Diagram 8 of the ‘phantom’ solution

579 Radioactivity
Calculate the percentage uptake by the (d) Actinium-227 isotope emits three rays. The
patient using the information in Table 3. isotope is placed in front of a magnetic field and
Then, state if the patient’s thyroid is the radioactive radiation paths are shown in
functioning properly. [3 marks] Diagram 10.
(b) Explain why medical equipment should be
sterilised using radiation from radioactive sources P
rather than immersing them in boiling water.
[2 marks] Q
(c) It is given that iodine-131 has a proton number
53 and a nucleon number 131. It decays by
emitting gamma rays. Its half-life is 8 days.
(i) State the meaning of isotope, proton
number and nucleon number. [3 marks] R
(ii) What is gamma ray? [1 mark] magnetic field into the paper
The initial activity of a sample of iodine-131 is Diagram 10
3.4 × 103 s–1.
(iii) Estimate the time required for its activity to (i) Name the radiations P, Q and R emitted by
fall to 106.25 s–1. [2 marks]
the isotope. [3 marks]
(iv) State one use of iodine-131. [1 mark] (ii) Name the F5/5/47
rule used to determine the
(v) Give a reason for your answer in (iv). radiations P, Q and R. [1 mark]
[1 mark] (iii) Explain why radiation Q is not deflected by
F the magnetic field. [1 mark]
O
5

R 7 (a) The diagram shows two types of tracks formed in a cloud chamber. The tracks are formed due to the radiation
from radioisotope.
CHAPTER

M SPM
Clone
’11
5

M N
F
O
(i) What is the meaning of radioisotope? [1 mark]
5

R that causes the tracks in cloud chamber M and cloud chamber N?


(ii) Name the radiation [2 mark]
CHAPTER

M in 7(a)(ii))
(iii) Explain the answer [2 mark]
(b) A researcher is asked to determine the most suitable radioisotope to be used as a tracer to measure blood
5
volume in patients.
Radioisotope State of matter Type of emission Penetrating power Ionising power Half-life
V Solid Alpha particle Low High 12 days
W Solid Gamma ray High Low 20 days
X Liquid Beta particle Moderate Low 60 days
Y Liquid Beta particle Moderate Low 2 hours
Z Liquid Alpha particle Low High 5 years

Explain the suitability of each characteristic of the 4


2
He = 4.000260 u
radioisotope to be used as a tracer. Determine 1 u = 1.66 × 10–27 kg
the most suitable radioisotope that can be used Based on the equation, calculate
as a tracer to measure blood volume in patients. (i) the mass defect [2 mark]
Give reasons for your choice. (ii) the energy released [3 marks]
(c) A radioactive decay is represented by the 131
following equation: 8 (a) 53
I is used to treat overactive thyroid glands. 131
53
I
226
Ra → 226 Rn + 42 He + Energy decays by emitting a beta particle and gamma
88 86
Mass of: rays. The daughter particle produced is xenon, Xe.
226
Ra = 226.02536 u (i) Write an equation for the nuclear
88
226
Rn = 222.01753 u transformation described. [1 mark]
86
(ii) What is a beta particle? [1 mark]

Radioactivity 580
(iii) State the number of neutrons in xenon. Table 5
[1 mark]
(b) A patient is injected with 131I. A few days later, a Activity
53
Time Activity due to sample
detector is placed close to the thyroid gland, (counts per
(min) alone (counts per minute)
outside the patient’s body. minute)
(i) Can the detector detect the beta particles? 0 100
[1 mark]
(ii) Explain your answer in (b)(i). [1 mark]
10 74
(c) It is given that the half-life of 131
53
I is 8 days. 20 53
The initial dose of 131
53
I emits 4.2 × 108 gamma rays 30 39.5
per second.
40 29.5
(i) Calculate the number of gamma rays
emitted per second after 32 days. [2 marks] 50 22.5
(ii) After 32 days, the number of gamma rays 60 17.75
detected is less than the answer in (c)(i).
Give one explanation for this observation. (i) Complete Table 5. [2 marks]
[1 mark] (ii) Explain the meaning of background
(d) In a radioactivity experiment, a Geiger counter was radiation. [1 mark]
used to measure the activity of material W over a (iii) On a graph paper, plot the values for activity
period of one hour. Its activity was measured every due to sample alone against time. [4 marks]
ten minutes. During this period, the background (iv) Based on your graph, determine the half-life
radiation was regularly measured at 6 counts per of material W. F
minute. The results are shown in Table 5. Show how the answer is obtained from the O

5
R
graph. [1 mark]

CHAPTER
M

Experiment
1 A scientist monitored the activity of a radioactive sample over a period of few days. The scientist used a G-M
tube connected to a ratemeter to detect and measure the emitted radiation. Diagrams 1(a) to (g) show the
readings of the ratemeter.

   s-1    s-1    s-1    s-1

Day 0 Day 1 Day 2 Day 3


(a) (b) (c) (d)

   s-1    s-1    s-1

Day 4 Day 5 Day 6


(e) (f) (g)

Diagram 1

Based on the above observations: (ii)The variables in the experiment


(a) State one suitable inference. [1 mark] (iii)The list of apparatus and materials
(b) State one suitable hypothesis that could be (iv) The arrangement of the apparatus
investigated. [1 mark] (v) The procedure used in the experiment.
(c) With the use of apparatus such as G-M tube, Describe how to control the manipulated
ratemeter and other apparatus, describe one variable and how to measure the
experiment to investigate the hypothesis stated responding variable.
in (b). (vi) The way to tabulate the data
In your description, clearly state the following: (vii) The way to analyse the data
(i) The aim of the experiment [10 marks]

COMPANION WEBSITE 581 Radioactivity


Online Tests
PAPER 1 (50 marks) Time: 1 hour 15 minutes

Instructions: This paper consists of 50 questions. Each question is given four choices, A, B, C and D.
Choose the most suitable answer. Answer all questions.

1 Diagram 1 shows the position of the scales of a vernier callipers ? 4 Diagram 3 shows Newton’s cradle
SPM
Clone with identical steel balls
’11 x
6 7 8
cm Main
scale

Vernier
scale X Y Z
0 5 10

Diagram 1
Diagram 3
What is the value of x ? 2 Which quantity is a derived
If ball X collides with Y at the
A 0.43 cm quantity ?
speed of v m s-1, what will
B 0.45cm A height
happen to balls X, Y and Z after
C 0.47cm B temperature
collision?
D 0.49 cm C electric current
D speed X Y Z

3 Diagram 2 shows an electric circuit used to determine the internal resistance A Stationary Moves with Moves with
SPM
of the cell. v m s-1 v m s-1
Clone
’11 B Stationary Stationary Moves with
V
v m s-1
C Moves with Moves with Moves with
v m s-1 v m s-1 v m s-1
D Stationary Stationary Stationary
A

5 Diagram 4 shows a wet


rheostate umbrella is rotated vigorously
Diagram 2 and is stopped abruptly.

Which of the following is correct?

Manipulated Responding Constant


variable variable variable
A potential electromotive
current
difference force
B electromotive potential
current Diagram 4
force difference
C potential electromotive This way of drying the umbrella is
current due to
difference force
A impulse
D potential electromotive
current B equilibrium
difference force
C momentum
D inertia

582
6 Diagram 5 shows a coconut falling 8 Which of the following has zero 11 Diagram 9 shows the velocity-
from a tree. net force? SPM
Clone time graph of a motion object.
A ’11
12 N v (m s-1)

friction 30°
s 6N
12

B
12 N

friction 60° t(s)


6N 0 2 4 6 8 10

C
Diagram 5 12 N 6

Which graph shows the relationship 45° Diagram 9


between the momentum of the friction 30°
6N What is the displacement of the
coconut, M and its distance fall, s?
A M D object ?
12 N A –56 m
50° B –30 m
friction C 30 m
6N D 56 m
s
B M 12 Diagram 10 shows two spherical
9 Diagram 7 shows a boy sliding
balls, X and Y, in a container of
SPM down a smooth slide with
Clone
water.
’11 vertical height 1.25m

s
X
C M Y Water

SPM MODEL TEST


Diagram 10
1.25 m
P Density of Y is larger than density
s
of X but both have the same
D M
size.
Which is the correct comparison
of the upthrust acted on X and
Diagram 7
Y?
s What is his velocity at P? A Upthrust on X = upthrust on Y
A 5 ms-1 C 10 m s-1 B Upthrust on X > upthrust on Y
7 Diagram 6 shows a long jumper B 7.5 m s -1
D 12.5 m s-1 C Upthrust on X < upthrust on Y
landing on the sand pitch.
10 Diagram 8 shows a wooden 13 Diagrams 11 shows a simple
block of mass 4 kg is being hydraulic system in equilibrium.
pulled up a slope.
F1
Q
F A2
A3
d A1
8 mcline
i n 6m
th
oo F2
smlane F3
P p

Diagram 6 Diagram 8 Diagram 11


She bends her knees while What is the work done by the A1, A2 and A3 are the cross-
landing on the pitch to reduce wooden block from P to Q? sectional area of the pistons
A the momentum on the pitch. A 160 J where A1  A2  A3. F1, F2 and F3
B the collision time on the pitch. B 240 J are the forces acted on the
C the impulse on her feet. C 320 J pistons. Which of the following
D the impulsive force on her feet. D 400 J comparisons is correct?

583 SPM Model Test


A F1 = F2 = F3 20 900 kg of water at a temperature
B F1  F2  F3 oil of 80°C is poured into 1.8 kg of
C F1  F2  F3 water at a temperature of 50°C.
D F1  F2 = F3 What is the final temperature of
the mixture, in °C ?
14 Which of the following is the
water A 55
application of the Bernoulli’s
B 60
principle?
C 65
A Diagram 13 D 70
Which of the following statements
21
is true?
Temperature/°C
A The weight of oil displaced
= The weight of water 80
displaced 70
B The weight of oil displaced
60
B  The weight of water
displaced 50
C The weight of oil displaced 40
 The weight of water
30
displaced
20
C 17 Diagram 14 shows the same
spring exerted by masses 600 g 10
and 800 g. 0 Time/
1 2 3 4 5 min

600 g Diagram 15

800 g Diagram 15 shows the graph of


D temperature against time for
10 cm heating of a substance from solid
6 cm
SPM MODEL TEST

state. The power rating of the


heater is 0.5 kW.
Which of the following statements
Diagram 14
is true ?
What is the original length of the A The whole substance is liquid
15 Diagram 12 shows a simple spring ? at the first minute.
mercury barometer. A 12 cm C 22 cm B The heat absorbed by the
B 16 cm D 24 cm substance in 5 minutes is
vacuum 2.5 kJ
18 The boiling point of water at the
C Specific heat capacity of the
summit of a mountain is less than
substance in liquid state is
h cm 100 °C. This is due to
higher than in solid state
A the lower temperature at the
D Heat is not absorbed by the
summit.
substance from the first minute
mercury
B the windy condition at the
to the third minute
summit.
C the higher humidity at the 22 In an experiment to verify Charles’
Diagram 12 summit. law, two constant variables are
The value of h will increase if D the lower atmospheric A temperature and volume
A some mercury is added to the pressure at the summit. B mass and temperature
container. C mass and pressure
19 11.5 kJ of heat is needed to
B some water is poured into the D pressure and volume
increase the temperature of
container. 500 g of iron from 170°C to 23 Diagram 16 shows trapped air in
C the tube is slanted. 220°C. What is the specific heat an airtight cylinder.
D the barometer is brought to the capacity of iron?
peak of a mountain. A 230 J kg-1 °C-1
16 Diagrams 13 shows the same B 460 J kg-1 °C-1 piston is
pushed
spherical solid floating on oil and C 500 J kg-1 °C-1
water. D 690 J kg-1 °C-1 Diagram 16

SPM Model Test 584


When the piston of the cylinder is 27 Diagram 19 shows an object 28
pushed, the quantity which will placed in front of a convex mirror
remain constant is with focal length of 7.5 cm.
A the number of air molecules
object convex lens
per unit volume.
35°
B the pressure of the trapped
air. O F C
15 cm
C the average distance between
the air molecules. Diagram 19
D the average speed of air
molecules in the cylinder. The characteristics of the image
formed are
24 A real, inverted, same size as the Diagram 20
object.
Coin appears Diagram 20 shows an incident ray
Water to be here B real, inverted, smaller than the
object. of light travelling into a prism. The
C virtual, upright, bigger than the critical angle of the prism is
object. A 35°
15 cm
D virtual, upright, smaller than the B 45°
h cm True position
of the coin object. C 55°
D 65°

29
Diagram 17 Q trough
Diagram 17 shows a coin S
immersed in water. If the refractive P R
crest
index of the liquid is 1.3, find the
value of h.
A 3.46 cm
B 6.92 cm

SPM MODEL TEST


C 10.5 cm
D 11.5 cm
25 The characteristics of the image
formed by a convex mirror are Diagram 21
A virtual, upright and diminished Diagram 21 shows a interference pattern of two coherent water waves. The
B virtual, upright and magnified constructive interference occurs at
C real, inverted and diminished A P and Q C Q, R and S
D real, inverted and magnified B P, Q and R D R and S
26 30 31 H

P 1

Q
2

Diagram 23
Diagram 22 Diagram 23 shows the side view
Diagram 18 Diagram 22 shows the wavefronts of a ripple tank with the uniform
Diagram 18 shows a ray of of a plane wave moving towards a depth at the middle and
incidence light travelling from triangular shallow region. Which of decreases towards both sides. H is
liquid P to liquid Q. If θ1 > θ2, then the following characteristics of a iron bar that vibrates at a
A refractive index of liquid P = waves remain unchanged when constant frequency. Which pattern
refractive index of liquid Q travelling in the region? of the warefronts will be observed
B refractive index of liquid P < A Wavelength in the ripple tank?
refractive index of liquid Q B Speed A
C refractive index of liquid P > C Frequency
refractive index of liquid Q D Direction of propagation

585 SPM Model Test


B The distance between two A
consecutive loud sounds can be
increased by increasing the
A frequency of the audio source. B
C B distance between the
loudspeakers.
C wavelength of the audio source.
C
35 V
D

A
32 cork
R

S
R 38 An electric current of 400 mA
Diagram 25 flows through a lighting bulb for
2 minutes. What is the electric
What will be the effects on the charge which flows through the
Diagram 24 reading of ammeter and voltmeter bulb?
when the switch S is closed ? A 0.8 C
A set of plane waves is produced B 48 C
in a ripple tank. The period of the Ammeter Voltmeter
C 800 C
waves is T seconds. A cork is A increase increase D 48 000 C
placed in the tank as shown in B increase decrease
Diagram 24. What is the time
C decrease increase 39
taken for the cork to move from
the trough to the crest ? D decrease decrease
SPM MODEL TEST

T T
A 8 C 4 36 In Diagram 27, the voltmeter S M N
registered a reading of 3 V. When I
T T the switch is closed, the readings
B 6 D 2
of the ammeter and voltmeter are K L
33 Which of the following waves is a 1 A and 2.5 V respectively. N
longitudinal waves ?
A radio wave C microwave V
B water wave D sound wave battery Diagram 28

Diagram 28 shows a current-


34 Diagram 25 shows the setup
carrying wire in a magnetic field.
experiment for interference of
A The wire experiences a force in
sound waves.
the direction of
Audio Signal
Generator R A K
B L
Diagram 27 C M
Find the internal resistance of the D N
battery.
Speaker A 0.2 Ω 40 Diagram 29 shows an insulated
B 0.3 Ω wire wound on a soft iron core
a and connected to a battery.
C 0.4 Ω
D 0.5 Ω
D

X 37 In which of the following


LS LS S LS LS L S LS LS L
arrangements does the identical
Microphone resistors with resistance 3 Ω each X Y
L = Loud sound
S = Soft sound gives the effective resistance of
Diagram 25 2 Ω? Diagram 29

SPM Model Test 586


Which of the following is correct Which of the following actions will Table 1 above shows the truth
about the poles at X and Y? cause the reduction in the table for a logic gate. The symbol
ammeter reading ? for the logic gate is
Pole at X Pole at Y A The silicon is immersed in A X
Z
A North North boiling water.
B The silicon is immersed in cold Y
B South North
water. B X
C North South
C The silicon is doped with
D South South gallium Y
D The silicon is doped with C X
antimony Z
41 Diagram 30 shows a straight
conductor moved vertically Y
45
downwards between two D X
K Z
opposite poles of magnets.
Y
I

N S 48 Isotopes are nuclides with the


Diagram 32 same
direction of motion A number of neutrons
Diagram 32 shows the symbol of
B atomic number
Diagram 30 a transistor. Name the electrode K
C nucleon number
and state the type of transistor.
The direction of induced current in
the straight conductor can be Electrode K Type of 49 A radioactive sample has an initial
determined by transistor activity of 480 disintegrations
A Right-hand grip rule A Collector pnp per minute. After 2 hours, its
B Right-hand screw rule activity decreases to 7.5
B Collector npn
C Fleming’s left-hand rule disintegrations per minute. What
D Fleming’s right-hand rule C Emitter pnp is the half-life, in minutes, of the
D Emitter npn radioactive sample?

SPM MODEL TEST


42 An ideal transformer has the same A 20
46
value of quantity in both primary B 25
and secondary coils. The quantity C 30
is the D 35
A number of turns of coils.
B power. M N 50 The equation below represents
C voltage. Diagram 34 alpha decay
D current. P → Q + 42He
Diagram 34 shows circuits M and
N consisting of capacitor, bulb, dry Which of the following is true
43 Direct current and alternating cell and alternating current supply. about element Q?
current voltages can be measured Which of the following about the Proton Mass
by circuits M and N is true ? number number
A moving coil voltmeter and hot
wire voltmeter. Circuit M Circuit N A Increase by 2 Increase by 4
B moving coil voltmeter and A Lighted Not lighted B Decrease by 2 Increase by 4
moving iron voltmeter. C Increase by 2 Decrease by 4
B Not lighted Lighted
C moving iron voltmeter and hot
wire voltmeter. C Lighted Lighted D Decrease by 2 Decrease by 4
D Not lighted Not lighted
44 Figure 31 shows an electrical
circuit used to study the 47 Input Output
conductivity of silicon. X Y Z
R 0 0 1
A 0 1 1
1 0 1
Silicon 1 1 0
Figure 31 50
Table 1

587 SPM Model Test


PAPER 2 Time: 2 hour 30 minutes

Section A (60 marks)

Answer all questions in this section. The suggested time to complete this section is 90 minutes.
1 You are required to do an experiment on electricity in which the expected current is between 0 to 1.0 A.
You are provided with ammeters and miliammeters as shown in Diagram 1.

0.4 0.6
0.8 200 300 40
0.2 100 0

50
1.
0

0
0

0
A
mA
Pointer Pointer
Adjustment Adjustment
screw screw

P Q

2 3
4 100 150 20
1 50 0
25
0
0

0
5

A
mA
Pointer Pointer
Adjustment Adjustment
screw screw
R S
Diagram 1.1
(a) Which one of the instrument is the most sensitive? [1 mark]
(b) Why do you choose instrument P rather than Q and S? [1 mark]
(c) Give two reasons for choosing P instead of R. [2 marks]
SPM MODEL TEST

2 (a) Diagram 2.1 shows a concave mirror and a convex lens with the same focal length.

object

C F 2F F O F 2F
P image
image

Diagram 2.1

Based on the diagram, write a similarity and a


difference of the images formed. [2 marks]
1
(b) The concave mirror forms an image of equal size
side by side when the object is placed at 20 cm Point object F
from the mirror. 2 F
(i) What will be the distance between the 3
object and its image when the convex lens
forms an image of same size for the object?
[2 marks] Diagram 2.2
(ii) The concave mirror and the convex lens are
A point source is placed at the focal point of the
then placed so that they are coaxial as
concave mirror.
shown in Diagram 2.2.
Complete the light ray path for ray 1, 2 and 3.
[2 marks]

SPM Model Test 588


3 Diagram 3.1 shows the decay curve for a radioactive Diagrams 4.1, 4.2 and 4.3 show three voltage-time
sample X. graphs for three power sources, P, Q and R.
(a) Determine the frequency of the power supply Q.
Activity (count per min)
[2 marks]
(b) Which power source will heat up a resistor more
2400 effectively? [1 mark]
(c) Power sources P, Q and R are connected in turn to
2000 a cathode-ray oscilloscope (CRO) as shown in
Diagram 4.4.
1600

1200

800 power +
CRO
supply -
400
X

0 T (min) Diagram 4.4


10 20 30 40 50 60 70
(i) What is the effect of the diode on the input
Diagram 3.1 current from the power supply? [1 mark]
(ii) In Diagram 4.5, draw the traces on the
(a) Sample X decays after emitting an alpha particle
screen for power sources P, Q and R.
to become Y. Write an equation for the radioactive
[3 marks]
decay. [1 mark]
(b) Draw on the curve to obtain two values of half-life,
T1 of the sample. Calculate the average value of

2
T1. [2 marks]

2
(c) Draw on the curve the decay curve for another
radioactive sample, P with activity of 1600 counts

SPM MODEL TEST


For P For Q For R
per minute at t = 0 minute, given that its T1 is

2
Diagram 4.5
twice the half-life of sample X. [1 mark]
5
(d) Compare the activity or count rates of samples P
and X. [2 marks]
5 cm 10 cm
4 V (V) V (V)
51 cm
51 cm
3 3
Diagram 5.1 Diagram 5.2
thermometer
t (s) t (s) t
0.01 0.02
0 0.03 0.01 0.02 0.03 0.04

51 cm
P Q
Diagram 4.1 Diagram 4.2

V (V)
Diagram 5.3
Diagram 5.1 shows a glass tube with an open end on
top. The trapped air column and the mercury column
2 are of lengths 51 cm and 5 cm respectively. Given the
atmospheric pressure is 75 cm Hg and the density of
t t (s)
air is 13 600 kg m–3.
(a) What is the air pressure of the trapped air? [1 mark]
(b) Under what situation will the trapped air has a
R minimum air pressure? What is the air pressure at
Diagram 4.3 this situation? [2 marks]

589 SPM Model Test


(c) (i) If the length of the mercury column is V(V)
Graph of V against I
doubled, i.e. 10 cm, determine the length of
the trapped air column as shown in Diagram
5.2. [2 marks] 3.0
(ii) Glass tube in Diagram 5.2 is then heated as
shown in Diagram 5.3. Determine the
temperature of the water when the air
column becomes 51 cm again.
[Room temperature = 27°C] [3 marks]
(1.5,1.5)

6 Diagram 6.1 shows four arrangements of circuiits


consisting of four resistors of 1 Ω. The potential
difference across the battery which consists of two dry O I(A)
cells and the main current are measured by the
voltmeter and the ammeter respectively. Diagram 6.2
From the graph,
(i) State the value of electromotive force of the
V V
battery. [1 mark]
(ii) Calculate the internal resistance of one dry
cell. [3 marks]

1Ω 1Ω A 1Ω A 7
1Ω 1Ω Audio Signal Generator
Loud Speaker
1Ω 1Ω 1Ω

Circuit P Circuit Q
D D

V
V
SPM MODEL TEST

Diagram 7.1 Diagram 7.2


1Ω A Antinodal line
A
1Ω 1Ω Nodal line
1Ω 1Ω
1Ω
1Ω 1Ω Diagram 7.1 and Diagram 7.2 show the formation of
nodal lines and antinodal lines for sound interference
Circuit R Circuit S using two speakers connected to a audio signal
generator.
Diagram 6.1 (a) Based on Diagram 7.1 and Digram 7.2
(a) Arrange the effective resistance of the external (i) Compare the distance between the
resistors in ascending order for the four circuits. speakers of both arrangement. [1 mark]
[2 marks] (ii) Compare the distance between the
(b) Draw a diagram to show how you would arrange antinodal lines at a perpendicular distance D
the resistors for the maximum current flow. Hence from the speakers. [1 mark]
calculate the effective resistance for that (iii) State the relationship between the distance
arrangement. [2 marks] between antinodal lines with the distance
(c) A student performs the experiment and plots a between the speakers. [1 mark]
graph of potential diffetence, V across the battery (b) Explain another method by which you can also
against the current flow, I as shown in Diagram change the interference pattern on Diagram 7.1 to
6.2. Diagram 7.2 without changing the distance
between the speakers. [2 marks]
(c) What would you observe if you walk along.
(i) an antinodal line? [1 mark]
(ii) a nodal line? [1 mark]
(d) A boy walks across the nodal and antinodal lines
at a perpendicular distance of 3.0 m from the
speakers which are 0.50 m apart. If the frequency

SPM Model Test 590


of the signal is 660 Hz and the speed of sound in Displacement (m)
air is 330 m s–1, what is the distance berween two
antinodal lines? [3 marks]

8 Mr. Gerald of weight 600 N is in a hot-air balloon of


mass 300 kg which is rising vertically with a constant H
velocity of 10 ms–1. At a height of H m, Mr. Gerald
stretches his hand out of the basket and releases a
small rubber ball as shown in Diagram 8.1. An
observer on the ground observes the displacement, H
of the ball as measured from the ground. The
displacement-time graph is shown in Diagram 8.2.

Time (s)
1 4

Diagram 8.2
(a) Tabulate and classify all the physical quantities as
in the passage above into two categories.
[2 marks]
(b) (i) What is the resultant force acting on the
rising hot-air balloon? [1 mark]
(ii) Hence, calculate the buoyant force on the
hot-air balloon. [2 marks]
(c) (i) State the velocity of the ball at the instant it
is released by Mr. Gerald. [1 mark]
(ii) Describe briefly the motion of the ball after
its release, as seen by the observer. [2 marks]
(iii) Tick  at the box which corresponds to the
observation on the motion of the rubber ball
by Mr. Gerald on the hot-air balloon.

SPM MODEL TEST


constant
Mr Gerald velocity : Appears to be stationary
releasing 10 m/s
the rubber
ball : As observed by the man on the
ground
: Appears to move downwards.
H
H [1 mark]
(d) The maximum height reached by the ball is 45 m
above the ground.
observer
(i) Find the distance travelled by the ball in the
first second. [2 marks]
ground
(ii) Hence, deduce the value of H. [1 mark]
Diagram 8.1

591 SPM Model Test


Section B (20 marks)
Answer any one question from this section. The time suggested to answer this section is 30 minutes.

9 (a) (i) What is meant by buoyant force? (ii) Compare the buoyant force on the
(ii) Diagram 9.1 shows two helium balloons submarine when it is stationary under the
each with a weight attached floating in the sea to when it is floating on the surface of
air. the sea. [5 marks]
(c) Diagram 9.2 shows a simple hydraulic system

F1 F2
Input Output
piston piston
helium

hydraulic fluid

Diagram 9.2
Given
weight Output piston area
Multiplying factor =
Input piston area
Diagram 9.1
You are required to give some suggestions to design
Using Diagram 9.1, relate the weight an industrial hydraulic system which will produce
attached, the size of the balloon and the a great force efficiently, and can last longer. The
buoyant force. Make a deduction regarding system will operate with temperature reaching
the relationship between the air displaced hundreds of degrees Celsius. State and explain
by the balloon and the buoyant force. the suggestions, based on the following aspects:
SPM MODEL TEST

[4 marks] – The multiplying factor


(b) The ballast tank in a submarine is three quarters – The boiling point and the specific heat capacity
full of water when it is stationary at a depth of of the hydraulic fluid
30 m below the surface of water. – The lubrication quality of the hydraulic fluid
(i) Explain how the submarine can dive further – The compressibility of the hydraulic fluid
to the bottom of the sea which is 100 m – The material used for the fluid transmission
below the surface of the sea, and then rise pipe.
and float on the surface of water. [10 marks]

10 (a) A soft iron bar is suspended from a spring balance. A fixed electromagnet is placed below the magnet
bar as shown in Diagram 10.1 and 10.2. The electromagnet is connected to a d.c. power supply. Wo is
the reading on the spring balance before the power is turned on.

Wo Spring balance

Iron bar

d.c. power supply

1 2
0

A
Electromagnet A

Ammeter
Diagram 10.1

SPM Model Test 592


WI
Spring balance

Iron bar
d.c. power supply

1 2

3
A
Electromagnet A

Ammeter

Diagram 10.2

(i) What is an electromagnet? [1 mark] (c)


(ii) Using Diagrams 10.1 and 10.2, compare Insulated
Iron wire
the reading of both the ammeter, the
reading on the spring balances, and the
force on the iron bar. Primary Secondary
Relate the reading of the ammeter with the coil coil
(40 turn) (20 turns)
force on the iron bar to make a deduction
regarding the relationship between the
electric current and the strength of
electromagnet. [5 marks]
Iron core

SPM MODEL TEST


(b) Diagram 10.3 shows a simple magnetic relay.
Diagram 10.4
pivot
spring
Diagram 10.4 shows a step down transformer
made by one member of the Science Society in
soft iron your school. The transformer is meant to step
armature
motor down a voltage from 12 V to 3 V. As the head of
contact Physics Section in the Science Society, you are
required to give some suggestions to design a
step-down transformer which will work and
function with the highest possible efficiency.
Using your knowledge of electromagnetism and
properties of material, state and explain the
suggestions based on the followng aspects:
(i) the number of turns on the primary and
Diagram 10.3 secondary coils,
(ii) the type of wires used for the coils,
(iii) the way the two coils are wound,
(i) What is the main use of a magnetic relay?
(iv) modification on the core,
(ii) Explain how a magnetic relay works as in the
(v) ways to reduce flux leakage.
situation as shown in Diagram 10.3.
[10 marks]
[5 marks]

593 SPM Model Test


Section C (20 marks)
Answer any one question from this section. The suggested time to answer this section is 30 minutes.
11 (a) Diagram 11.1 shows the heating graph for ice till it turns into steam.
Temperature (°C)

D
100
E

B C
O Time (min)

–50 A

Diagram 11.1
(i) Explain why there is an increase in temperature from point A to B and from point C to D.
[1 mark]
(ii) Explain why the temperature does not increase from point B to C and from point D to E.
[2 marks]
(iii) What happened to the heat energy supplied in b(i)?
[2 marks]
(b) Table 11.1 shows 6 arrangements, P, Q, R, S, T and U to be used to determine the specific latent heat of
vaporisation of water in the school laboratory. The reading on the scale when the water in the container
boils steadily is noted. The time taken to boil away 20 g of water is recorded. The specific latent heat is
determined from L = Pt , where P is the power of the immersion heater used.
20  10–3
SPM MODEL TEST

Table 11.1

Power of Condition of
Arrangement immersion the external Weighing Room
heater surface of scale temperature
the beaker
P 100 W Shiny Electronic (0 – 500 g) ± 0.01 g 10°C
Q 100 W Dull Electronic (0 – 500 g) ± 0.01 g 26°C
R 100 W Shiny Electronic (0 – 500 g) ± 0.01 g 26°C
S 100 W Dull Spring (compressive) 10°C
(0 – 500 g) ± 5 g
T 40 W Shiny Electronic (0 – 500 g) ± 0.01 g 10°C
U 40 W Dull Spring (compressive) 26°C
(0 – 500 g) ± 5 g

You are required to determine the most suitable arrangement that can be used to determine as accurately
as possible the specific latent heat of vaporisation of water.
Study the specifications of all the arrangements based on the following aspects:
(i) The power of the immersion heater. [2 marks]
(ii) The container used. [2 marks]
(iii) The room temperature. [2 marks]
(iv) The weighing instrument used. [2 marks]

Explain the suitability of each aspect and then determine the most suitable arrangement.
Give a reason for your choice.
[2 marks]

SPM Model Test 594


(c) An immersion heat of 100 W is used to boil away 20 g of boiling water at 100°C in
9 minutes 30 seconds.
(i) Calculate the heat needed to boil away 20 g of water at 100°C. [2 marks]
(ii) Determine the heat lost to the surroundings during the process and hence calculate the efficiency of
the immersion heater in this arrangement. [3 marks]

12 Diagram 12.1 shows a water wave before and after passing through an opening.
displacement (mm)

cork Q time (s)

wavefront displacement (mm)

cork P
barrier time (s)

Diagram 12.1
The two displacement-time graphs show how the displacement of two small corks varies with time at points P
and Q respectively.
(a) (i) State the physical quantity that affects the intensity of a wave.
(ii) Compare the period of oscillation of the two corks at points P and Q.
(iii) Explain why the amplitude at point Q is less than that at point P, with reference to the amount of
wave energy that passes through the opening.
[Assume that no energy is lost due to absorption in the process of propagation.]
[5 marks]
(b) Table 12.1 shows the specifications of the reflector and the conditions of the pot for four solar hotpots.
You are required to determine the most suitable solar hotpot. Study the specifications of the four solar

SPM MODEL TEST


hotpots based on the following aspects:
— The diameter of the parabolic disc
— The orientation of the disc with respect to the incoming sunlight
— The outer surface and conditions of the hotpot
Explain the suitability of the aspects.
[10 marks]
Table 12.1
System Reflector Pot
Diameter of the parabolic Outer surface of the pot: Shiny
disc = 20 cm

Fixed disc orientation Condition of the pot:


P Exposed to the air

Diameter of the parabolic Outer surface of the pot:


disc = 50 cm Dull and black

Adjustable disc orientation Condition of the pot:


Placed in a glass vessel
Q

595 SPM Model Test


System Reflector Pot

Diameter of the parabolic Outer surface of the pot:


disc = 50 cm Dull and black

Fixed disc orientation Condition of the pot:


Placed in a glass vessel
R

Diameter of the parabolic Outer surface of the pot:


disc = 30 cm Dull and black

Adjustable disc orientation Condition of the pot:


Exposed to the air
S

(c) A student carries out Young’s double-slit experiment using a red light. He finds that the distance between
5 bright fringes, including the central one is 1.2 cm. The distance between the slits is 0.5 mm and the
distance between the double-slit plate and the screen is 2.2 m.
(i) Determine the wavelength of the red light.
SPM MODEL TEST

(ii) The student then moves the screen. In what direction does the student need to move the screen,
and how far does he move the screen so that 7 fringes, including the central one occupies the same
space as the 5 fringes before?

PAPER 3 Time: 1 hour 30 minutes

Section A (28 marks)


Instructions: Answer all questions in this section.

1 A student carries out an experiment to study the relationship between the distance between two consecutive bright
fringes, x and the distance between the two slits, d, in Young’s double-slit experiment.
The apparatus set-up for this experiment is shown in Diagram 1.

sodium-vapour lamp
magnifying lens

Diagram 1.1
The student starts the experiment with a = 0.3 mm. The screen is adjusted to obtain a clear interference pattern.
The distance across 11 consecutive bright fringes (that is 10x) is measured.

SPM Model Test 596


The experiment is repeated with a = 0.5 mm, 0.8 mm, 1.0 mm and 1.2 mm with results as shown in Diagrams 1.2, 1.3,
1.4, 1.5 and 1.6.

Diagram 1.2 a = 0.3 mm Diagram 1.4 a = 0.8 mm

Diagram 1.3 a = 0.5 mm Diagram 1.5 a = 1.0 mm

Diagram 1.6 a = 1.2 mm


(a) Based on the experiment described, identify
(i) the manipulated variable, [1 mark]
(ii) the responding variable, [1 mark]
(iii) the constant variable. [1 mark]
(b) Use a metre rule to measure the distance between 11 bright fringes.
Determine x, the distance between two consecutive bright fringes.

SPM MODEL TEST


1
Tabulate your results for all values of a, and x. [7 marks]
a
1
(c) Plot a graph of x against . [5 marks]
a
1
(d) Based on your graph, state the relationship between x and . [1 mark]
a

2 A student carries out an experiment to determine the relationship between the current flow, I and the number of dry
1 1
cells (in parallel) connected in series with a resistor of resistance, R. The student then plots a graph of against and
I t
the graph obtained is as shown in Diagram 2.1.
(a) Based on the graph in Diagram 2.1,
(i) What will happen to I when n increases? [1 mark]
1 1
(ii) Determine the value of I when = 0.
o n
1
Show on the graph how you determine the value of I . [2 marks]
o

1 R
(iii) Given = where E is the electromotive force of one dry cell and R the external resistance.
Io E
Calculate E if R = 2.0 Ω [1 mark]
Show on the graph how you determine m. [2 marks]
1 1
(ii) Determine the value of when = 0
Io n
1 1
(b) (i) Calculate the gradient, m, of the graph against .
I n
Show on the graph how you calculate the value of m. [3 marks]

597 SPM Model Test


1 -1 1 1
I (A ) Graph of
I
againt n

1.70

1.65

1.60

1.55

1.50

1.45

1.40

1.35

1.30

1
1.25 n
0.1 0.2 0.3 0.4 0.5 0.6 0.7 0.8 0.9 1.0
SPM MODEL TEST

Diagram 2.1

r
(ii) Given that the gradient of the graph, m = where r is the internal resistance of one dry cell.
E
Calculate the value of r, using the values of E and m in 2(a)(iii) and 2(b)(i) [1 mark]
(c) Determine the value of I when two dry cells are used in this experiment.
Show how you use the graph in the process of determining the value of I. [3 marks]
(d) State one precaution that can be taken to improve the accuracy of the readings in this experiment. [1 mark]

Section B (12 marks)

Instructions: Answer any one question from this section


3 Diagram 3.1 shows two identical coins being pressed by the same force onto a slab of plasticine. The area of contact
with the plasticine in the direction of the force is less for coin Q. However, coin Q forms a deeper cavity.

cavity
P
plasticine
coin slab
Q
Diagram 3.1
Based on the information and observation:
(a) State one suitable inference. [1 mark]
(b) State one suitable hypothesis. [1 mark]
(c) With the use of apparatus such as electronic balance (compressive type), thin wooden disks of different area A and
other appratus, describe one experiment to investigate the hypothesis stated in (b)

SPM Model Test 598


In your description, clearly state the following:
(i) The aim of the experiment,
(ii) The variables in the experiment.
(iii) The list of apparatus and materials,
(iv) The arrangement of the apparatus,
(v) The procedure of the experiment which should include one method of controlling the manipulated variable
and the method of measuring the responding variable,
(vi) The way to tabulate the data,
(vii) The way to analyse the data. [10 marks]

4 Diagram 4.1 shows electrical kettle P (2 kW, 2 litres) and kettle Q (2 kW, 1.2 litres). Kettle Q boils faster than kettle P
when both are in normal use.

kettle Q
(2 kW, 1.2  )
kettle P
(2 kW, 2  )

Diagram 4.1
Based on the information and observation:
(a) State one suitable inference. [1 mark]
(b) State one suitable hypothesis that could be investigated. [1 mark]
(c) With the use of apparatus such as immersion heater, beaker and other apparatus, describe one experiment to
investigate the hypothesis stated in 4(b).

SPM MODEL TEST


In your description, clearly state the following:
(i) The aim of the experiment,
(ii) The variables in the experiment,
(iii) The list of apparatus and materials,
(iv) The arrangement of the apparatus,
(v) The procedure used in the experiment. Describe how to control the manipulated variable and how to measure
the responding variable.
(vi) The way to tabulate the data,
(vii) The way to analyse the data. [10 marks]

599 SPM Model Test


Answers
Form 4
(b) 10–3 6 Actual diameter
1 Introduction to Physics (c) 3.86 × 106 = (7.50 + 0.35) – (–0.03)
Self Assess 1.1 (d) 103 = 7.88 mm
1 C (e) 103
2 Physics is an experimental science. (f) 6.43 × 10–4 Self Assess 1.5
It tries to describe the fundamental 8 0.4 mm = 0.4 × 10–3 m 1 (a) Inference: The period of
nature of the universe and how it = 4 × 10–4 m oscillation, T depends on the
works and works towards the 240 mg = 240 × 10–3 g mass, m, of the load.
simplest explanations for complex = 0.240 × 10–3 kg (b) Hypothesis: The larger the
situations. = 2.4 × 10–4 kg mass, m, the longer the period
3 We study physics because it is a 0.06 m = 0.06 × 103 mm of oscillation, T.
basic science. Scientists from all = 60 mm (c) Aim: To investigate the
other fields of study make use of 40 cm2 = 40 × 10–4 m2 relationship between the
the concepts developed in physics = 4 × 10–3 m2 period of oscillation of a loaded
to conduct experiments to expand 0.02 m2 = 0.02 × 106 mm2 spring and the mass of the
into new fields of study. Physics also = 2 × 104 mm2 load.
forms the basis of engineering and (d) Manipulated variable: Mass of
F technology. Mankind is able to the load, m.
invent useful apparatus and tools Self Assess 1.3
O Responding variable: The
with a good understanding of basic 1 A scalar quantity has only magnitude.
R A vector quantity has both magnitude
period of oscillation, T.
physics. Fixed variable: The stiffness of
M 4 Engineering, computer science, and direction. the spring (two identical
telecommunications, medicine, Volume is a scalar quantity. springs are used).
4 science of outer space. Momentum is a vector quantity. (e) Tabulation of data:
Self Assess 1.2 2 Time, temperature, power, speed and
1 1 m = 109 nm mass.
Time for 20 Period, T
0.000000475 m 3 Force, acceleration, displacement, m (g) oscillations, t20 (s) t
= (s)
= 4.75 × 10–7 m velocity and momentum. 20
= 4.75 × 10–7 × 109 nm t1 t2 Mean, t
= 4.75 × 102 nm m1
2 1 cm = 10–2 m Self Assess 1.4
m2
1 cm3 = 10–6 m3 1 A compression weighing scale is used
Volume of the wooden block to weigh objects with a large mass. m3
CHAPTER 1

= (4.0 × 1.5 × 1.4) cm3 Rigid springs are used in this type of m4
= 8.4 cm3 weighing scale. As a result, objects with
m5
= 8.4 × 10–6 m3 a small mass have no effect on the
72 km spring. Measurements involving large
3 (a) 72 km h–1 = 1 h values of mass are exact because the
(f) Graph:
error is small.
72 × 1000 m
= 3600 s 2 Sensitivity of an instrument is its T (s)
ability to detect or respond to a
= 20 m s–1
small change in the quantity to be
15 m
(b) 15 m s–1 = 1 s measured. Accuracy of a
measurement is how close the
15
_____ km value of the measurement is to the
1000
_________
= real or actual value.
1
_____ Consistency of an instrument is its
h m (g)
3600 ability to register the same reading
= 54 km h–1 when a measurement is repeated.
4 C The SI base unit of force is the (g) From the graph, the period of
3 2.4 cm
newton, N or kg m s–2. oscillation increases as the
5 B The SI base unit of area is m2 and 4 0.0245 cm. The advantage is the load increases.
the SI base unit of frequency is Hz. measurement is more accurate. The hypothesis is valid.
6 (a) 5.21 × 10–10 m 5 Actual width
(b) 2.413 × 10–2 J = 3.23 – (+ 0.02) 2 (a) θ0 = 27.3 °C
(c) 3.036 × 1012 km = 3.23 – 0.02 cm 1
(b) θ increases linearly with .
7 (a) 109 = 3.21 cm m

Answers 600
θ (°C) 1
(ii) Relative deviation is a
Graph of θ against m
34 measure of consistency
(0.6, 33.5) (precision). Measurements
33
are consistent if the relative
32 deviation obtained is small. Consistent Not accurate
31 4 (a) Volume but not and not
(b) Circle eye P accurate consistent
30
(c) 0.1 cm3
29 (d) Although any liquid can be used, (c) (i) The diameter of the wire can
28 the tube (column) used in a be determined by winding
(0.1, 28.3)
mercury barometer is shorter 20 turns of the wire round a
27 glass rod as shown below.
1 (kg-1)
because the density of mercury is
26
0 0.1 0.2 0.3 0.4 0.5 0.6 0.7 m very much higher. 20 turns of the wire
5 (a) 1 °C glass
(33.5 – 28.3) °C rod
(b) Mercury expands when the
(c) (i) k =
(0.6 – 0.1) kg–1 temperatures rises.
(c) A thin wall allows a quick heat metre
= 10.4 kg °C rule
transfer between the
4.5 × 104 4.5 × 104
(ii) c = = thermometer and its It must be ensured that
k 10.4
surroundings. Hence, this there is no space between
= 4.3 × 103 J kg–1 °C–1
improves its sensitivity. the coils.
1 (d) smaller
(d) When θ = 31 °C, m = 0.36 The total thickness of the wire,
6 (a) (i) Student S  cm, is measured using a
∴ m = 2.78 kg (ii) Student Q metre rule.
(e) The water should always be (b) (i) Micrometer screw gauge. F
The average diameter of the
stirred. (ii) Vernier callipers. 
O
wire is 20 cm. R
SPM Exam Practice 1 The main advantage of M
Essay Questions
Multiple-choice Questions 7 (a) (i) The sensitivity of a measuring using this method compare
1 D 2 C 3 D 4 D 5 A instrument is its ability to to measuring the diameter 4
6 B 7 D 8 A 9 C 10 B respond quickly to a small of only one turn of the wire
11 D 12 D 13 B 14 C 15 D change in the quantity to be using a ruler is that the
16 C 17 C 18 B 19 A 20 C measured. sensitivity is improved to
21 C 22 A 23 A 24 C 25 B (ii) Ammeter J shows a reading of 0.01 cm.
26 B 27 D 28 C 29 A 30 B 0.9 A. (ii) Micrometer screw gauge
Ammeter K shows a reading (iii) Micrometer screw gauge will
Structured Questions
0.75 A. give a more accurate
1 (a) (i) Voltmeter: 1.8 V
(iii) The scale on ammeter K is measurement because it can
(ii) Milliammeter: 2.35 mA
more sensitive than that of measure up to the sensitivity
(b) Zero error due to an incorrect

CHAPTER 1
ammeter J. of 0.001 cm
position of the zero point. It could
A measuring instrument with a (iv) Precautions:
occur when the pointers of the
scale of smaller divisions is • Make sure that the object is
meters do not start from exactly
more sensitive. gripped gently by the anvil
zero.
The scale on ammeter K gives and spindle but not too
(c) The correct reading can be
the smallest division of 0.05 A firmly.
obtained by making sure that the
while the scale on ammeter J • The ratchet knob is used to
eye is exactly in front of the
gives the smallest division of avoid over exertion of
pointer where the reflection of
0.1 A. pressure.
the pointer in the mirror is right
(b) Accuracy is how close the • Make a correction for zero
behind it.
measurement is to the actual error if it exists.
2 (a) 2.04 cm
value. • Avoid parallax error when
(b) Zero error
Consistency is the ability to taking readings from both
(c) Zero error = +0.04 cm
register the same reading when a the main and thimble
Diameter (with correction)
measurement is repeated. scales. (Any two)
= 2.04 – (0.04)
= 2.00 cm Distributions of shooting marks
Experiment
3 (a) (i) Ratchet on a target:
1 (a) (i) Vertical height of the centre of
(ii) The ratchet is used to avoid the pendulum bob from the
excessive pressure being bench, h.
exerted on the wire. (ii) Period of oscillation, T or time
(b) (i) Random errors can be for 20 oscillations, t
reduced by repeating the Accurate Accurate (Any one)
measurements a number of and but not (iii) The weight/mass of the
times and find the mean. consistent consistent pendulum bob.

601 Answers
(b) From the diagram: 7 u = 20 m s–1, v = 10 m s–1, s = 30 m
– Displacement, s • Applying v 2 = u2 + 2as,
h t1 t2 t T T2 = 62 + 82 102 = 202 + 2a(30)
(cm) (s) (s) (s) (s) (s2) –300
= 100 a = = –5 m s–2
30.0 33 34 33.5 1.675 2.81 = 10 km 60
40.0 32 31 31.5 1.575 2.48 Time = 30 minutes = 30 × 60 s Deceleration = 5 m s–2
= 1800 s • u = 10 m s–1, v = 0, a = –5 m s–2
50.0 27 28 27.5 1.375 1.89
Displacement Applying v 2 = u2+ 2as,
60.0 25 25 25.0 1.250 1.56 Velocity, v = 0 = 102 + 2(–5)s
Time taken
70.0 23 22 22.5 1.125 1.27 10 000 m s = 10 m
= 8 u = 0, v = 33 m s–1, a = 3 m s–2
1800 s
(c) Graph of T 2 against h: = 5.6 m s–1 Applying v 2 = u2 + 2as,
8. 332 = 02 + 2(3)s
T 2(s 2) tan θ = s = 181.5 m
6
θ = 53.1° 9 u = 0, v = 39 m s–1, s = 3 m
3.0 ∴ v = 5.6 m s–1 in the direction 53.1° Applying v 2 = u2 + 2as,
from the north 392 = 02+ 2a(3)
2.5 3 Time for 10 ticks = 0.2 s a = 253.5 m s–2
2 cm
(a) u = Self Assess 2.2
0.2 s
2.0
1 (a) Velocity = Gradient
= 10 cm s–1
15 m
10 cm =
v = 3s
1.5
0.2 s
= 50 cm s–1 = 5 m s–1
F t = 5 × 0.2 s (b) Time = (10 – 3) s = 7 s
O 1.0 =1s (c) t = 15 s
R v–u (d) (i) Average speed
a = Distance
M t =
0.5
(50 – 10) cm s–1 Time
= (15 + 15) m
4 1s =
h (cm) = 40 cm s–2 15 s
0
10 20 30 40 50 60 70 80
12 cm = 2 m s–1
(b) u =
0.1 s (ii) Average velocity
(d) T2 decreases linearly with h. Displacement
= 120 cm s–1 =
6 cm Time
v =
0.1 s 0
=
= 60 cm s–1 15
2 Forces and Motion
t = 3 × 0.1 s = 0
Self Assess 2.1 = 0.3 s 2 (a) Distance travelled
v–u
CHAPTERS 1 & 2

1 Total distance a = = Area under the graph


= 20 + 12 + 10 t 1
= × 12 × (3 + 10)
= 42 m (60 – 120) cm s–1 2
=
Final displacement 0.3 s = 78 m
= 20 – 12 + 10 = –200 cm s–2 (b) Time = (6 – 3) s
= 18 m to the east = –2 m s–2
=3s
Total distance Deceleration = 2 m s–2
Speed = (c) Acceleration a = Gradient
Time taken 4 u = 0, s = 90 m, t = 6 s
0 – 12
1 =
42 m Applying s = ut + at 2, 10 – 6
= 2
30 s = –3 m s–2
= 1.4 m s–1 1
90 = 0(6) + a(62) Deceleration = 3 m s–2
Final displacement 2
Velocity = ∴ Ratio of acceleration to
Time taken a = 5 m s–2 deceleration = 4 : 3
18 m 5 u = 0, v = 20 m s–1, t = 8 s (d) Distance travelled
=
30 s Applying v = u + at, = Area under the graph
= 0.6 m s–1 to the east 20 = 0 + a(8) 1
2 = × 12 × (3 + 10)
8 km a = 2.5 m s–2 2
6 u = 0, v = 15 m s–1, s = 1.2 m = 78 m
6 km 1 Average velocity, v
Applying s = (u + v)t,
2 Distance travelled
12 km =
1 Time taken
1.2 = (0 + 15)t
2 78 m
s =
2.4 10 s
θ t= = 0.16 s
15 = 7.8 m s–1

Answers 602
0 – 12 3 Give each block a push. The one that 3 Friction = 30 N as trolley moves
3 (a) a = Gradient =
13 – 10 offers more resistance to motion is the with a uniform velocity of 1 m s–1
= –4 m s–2 heavier one. (a = 0).
Deceleration = 4 m s–2 4 The larger the mass, the greater its Fnet = ma
(b) Displacement resistance to change in its state of 60 – 30 = m × 2
= Area under the graph motion. The massive brick with large m = 15 kg
1 inertia is sufficient to resist the force. 4 First, find the acceleration.
= × 12 × (4 + 13) However, the force breaks the brick. u = 16 m s–1, v = 0, t = 6 s
2
5 At Q, there is a change in the Using v = u + at,
= 102 m northwards
direction of motion. The water 0 = 16 + 6a
(c) Time = (20 – 15) s = 5 s –16
resists the change in direction and is a= m s–2
1
(d) Area of triangle = × 10 × 5 more likely to be spilt. Similarly, at P 6
2
and R, the water’s resistance to F = ma
= 25 m
change in its state of motion will –16
∴ Final displacement F = 72 ×
also spill some water. 6
= 102 – 25
= 77 m towards the north F = –192 N
Self Assess 2.4
(e) Average velocity ∴ The opposing force is 192 N.
1 m = 0.12 kg, v = 2 m s–1
Final displacement 5 (a) u = 0
= p = mv
Time v = 72 km h–1
= 0.12 × 2 72 × 1000 m
77 m = 0.24 kg m s–1 = = 20 m s–1
= 3600 s
20 s 2 p = mv
v–u
= 3.85 m s–1 750 = 250 v a=
t
4 Velocity (m s –1) v = 3 m s–1
20 – 0
3 m1v1 + m2v2 = 0 = = 2.5 m s–2 F
8
30
20
1.5 × v1 + 0.03 × 300 = 0 O
–0.03 × 300 (b) Fnet = ma R
I II III v1 =
1.5 = 1200 × 2.5
IV
M
20 50 60 70 Time (s) = 3000 N
= –6 m s–1
v–u ∴ The recoil velocity = 6 m s–1 6 (a) The friction acts towards the left.
(a) a = 4
t 4 m1v1 + m2v2 = (m1 + m2)v Fnet = ma
30 – 0 60 × 6 + 20 × 2 = (60 + 20)v 12 – Friction = 2 × 3
= 360 + 40 = 80v Friction = 6 N
20
v = 5 m s–1 (b) Now the friction acts towards the
= 1.5 m s–2
5 m1v1 + m2v2 = 0 right (to oppose motion).
(b) Distance travelled
m1 × 0.05 + 0.008 × (–0.03) = 0 Fnet = ma
= Area under the graph
0.05m1 = 0.008 × 0.03 20 – (12 + 6) = 2a
= I + II + III + IV
m1 = 0.0048 kg a = 1 m s–2
=
1
2 ( ) ( )
× 20 × 30 + 30 × 30 + = 4.8 g
6 Taking the direction to the right as
Self Assess 2.6

CHAPTER 2
1 Ft = mv – mu
[ 21 × 10 × (20 + 30)] + positive.
m1u1 + m2u2 = m1v1 + m2v2
F×6 = 6×2–0
∴ F = 2N
150 × 2 + 250 × (–1) = 150 × (–0.5) + 250 v2
( 21 × 10 × 20) 300 – 250 = –75 + 250 v2 2 Impulse = Change in momentum
= mv – mu
= 300 + 900 + 250 + 100 125 = 250 v2
v2 = 0.5 m s–1 =2×9–2×1
= 1550 m = 16 N s
1550 m 7 For Sau Fei:
(c) Average velocity = m1v1 + m2v2 = 0 3 Impulse = Change in momentum
70 s
0.5 × 8 + 60v2 = 0 Ft = mv – mu
= 22.1 m s–1 F × 2 = 0 – (60 × 12)
–4
v2 =
Self Assess 2.3 60 F = –360 N
1 The steel sphere moves with the same = –0.067 m s–1 The magnitude of the force on the box
velocity as the bus. At the time as it falls For Siew Ling: is 360 N.
downwards, it also moves forward (due m1v1 + m2v2 = (m1 + m2)v 4 Impulse = Ft
to its inertia) with the same velocity as 0.5 × 8 + 0 = (0.5 + 49.5)v = 1500 × 0.008
the boy. Therefore, to the boy’s eyes, it 4 = 50v = 12 N s
is falling downwards. v = 0.08 m s–1 Ft = mv – mu
2 When the lorry starts to move with 12 = 0.5v – 0
forward acceleration, the inertia of the Self Assess 2.5 v = 24 m s–1
durian tries to maintain its state of rest. 1 F = ma = 3 × 2 = 6 N 5 Take the direction to the right as
Thus, it seems to roll to the back. Similarly, 2 F = ma = 2 × 3 = 6 N positive.
when the lorry stops abruptly, the inertia of Applying F = ma again, Impulse = mv – mu
the durian causes it to continue in motion F 6 = 0.045 × 20 – 0.045(–30)
a= = = 1 m s–2
and thus it moves forward. m 6 = 2.25 N s

603 Answers
mv – mu 1 2 8 u = 0, s = 9 m
F= (a) Applying s = ut + at ,
t 2 1 1 5
g′ = g = × 10 =
2.25 1 6 6 3
500 = 9 = 0 + × 10t 2
t 2
1 2
t = 4.5 × 10–3 s t2 = 1.8 Applying s = ut + at ,
2
6 Take the direction to the right as t = 1.34 s
positive.
F =
mv – mu
(b) Applying v
v
=
=
u + at,
0 + 10 × 1.34
9 = 0+
1 5 2
2 3
t ()
t = 13.4 m s–1 ––––
t = 10.8
0.14 × 60 – 0.14 × (–40) 5 u = 20 m s–1 =3.3 s
=
5 × 10–2 v = 10 m s–1
8.4 + 5.6 a = –10 m s–2
=
5 × 10–2 Applying v 2 = u2 + 2as, Self Assess 2.9
= 280 N 102 = 202 + 2(–10)s 1 (a) The greatest force:
7 Take the direction to the right as positive. 20s = 300
80 N 140 N
mv – mu s = 15 m =
F =
t The height is 15 m. 60 N
1500 × 2 – 1500 × (–15) 6 True weight = mg
= The greatest force
0.18 = 50 × 10
= 500 N = 60 + 80
3000 + 22 500
= = 140 N in the same direction
0.18 Since the apparent weight > true
weight, the lift is accelerating The smallest force:
= 1.42 × 105 N
upwards. 60 N 80 N 20 N
8 Take the downward direction as
F positive.
R = mg + ma =
O mv – mu 550 = 500 + 50a
The smallest force
R F= 50 = 50a
t = 80 – 60
M a = 1 m s–2
50 × 0 – 50 × u = 20 N in the direction of the 80 N
–250 = 7 (a)
2 force
4 50u = 250 × 2 (b)
u = 10 m s–1 T T
FR
The negative sign indicates an 60 N
opposing force acting upwards.
θ
80 N 20 N
80 N
Self Assess 2.8 For the 8-kg load:
1 First, find the mass of the object. Resultant force, FR = 602 + 802
80 – T = 8a …… (i)
W = mg = 100 N
For the 2-kg load:
60 = m × 10 60
T – 20 = 2a …… (ii) tan θ =
CHAPTER 2

m = 6 kg 80
(i) + (ii): 80 – 20 = 10a
Applying W = mg again. = 0.75
10a = 60
W ′ = 6 × 18 θ = 36.9°
a = 6 m s–2
= 108 N From (ii), T = 20 + 2a The resultant force, FR is 100 N which
2 u= 0 = 20 + 2(6) acts in a direction 36.9° from the
a = 10 m s–2 = 32 N original 80 N force.
s = 3m (b)
v 2 = u2 + 2as (c) Scale 1 cm : 10 N
T
= 0 + 2 × 10 × 3 6 kg B
C
v = 7.75 m s–1
3 (a) The time taken for the coin to T FR
1 60 N
move up = × 1.2 s = 0.6 s 2 kg
2
120°
v= 0 46°
20 N O A
a = –10 m s–2 80 N
Applying v = u + at, For the 2-kg load:
0 = u + (–10) × 0.6 (diagram not to scale)
20 – T = 2a …… (i)
u = 6 m s–1 For the trolley: By measurement, length of OB
(b) Applying v 2 = u2 + 2as, 20 – T = 6a …… (ii) = 7.2 cm
0 = 62 + 2(–10) h (i) + (ii): 20 = 8a θ = 46°
h = 1.8 m a = 2.5 m s–2 The resultant force, FR
4 s = 9m From (ii), T = 6a = 7.2 × 10 N
u= 0 = 6(2.5) = 72 N in a direction 46° from the
a = 10 m s–2 = 15 N 80 N force

Answers 604
2 Scale 1 cm : 10 N (c) Applying Fnet = ma, Considering the horizontal forces:
C B F – (10 + 3) = 2 × 1 T1 sin 50° = T2 sin 40°
F = 13 + 2 sin 40°
T1 = T2 ×
= 15 N sin 50°
FR 7 =0.8391T2
45 N T1
T1 sin 30° Considering the vertical forces:
°
60 T2 T1 cos 30° T2 T1 cos 50° + T2 cos 40° = 150
34° 30°
O = 0.8391T2 cos 50° + T2 cos 40° = 150
35 N A
35 N 35 N 0.5394 T2 + 0.766 T2 = 150
(diagram not to scale) 150
T2 = = 115 N
1.305
By measurement, length of OB = 6.9 cm Considering the vertical forces: T1 = 0.8391 × 115 = 96.5 N
θ = 34° T1 sin 30° = 35
The resultant force, FR Alternative method
35
= 6.9 × 10 N T1 = = 70 N (by drawing the triangle of forces)
sin 30°
= 69 N in a direction 34° from the
Considering the horizontal forces:
35 N force
T2 = T1 cos 30° T2
Applying FR = ma,
=70 × 0.866 150 N
69 = 800a
=60.6 N
69
a= 8 R
800 motion 50°
T1
a = 0.086 m s–2
3 The effective force, Fx = 6 cos 35° mg sin θ T1
= 4.9 N W = cos 50°
4 20 = F sin 50°
30° 150 F
20
T1 = 150 cos 50° = 96.4 N O
F= = 26.1 N (a) The block moves with an T2 R
sin 50° = sin 50°
acceleration. 150
5 M
Applying Fnet = ma, T2 = 150 sin 50° = 115 N
mg sin θ = ma
a = g sin θ Self Assess 2.10
4
T T sin 35°
125 N
= 10 × sin 30° 1 Work done, W = mgh
35° T cos 35°
= = = 5 m s–2 = 80 × 10 × 1.7
35° T cos 35° u = 0, s = 2 m, a = 5 m s–2 = 1360 J
T T sin 35° 1 2 Work done, W
s = ut + at 2
2 = 0 (since there is no upward
1 displacement)
2 = 0 + (5)t 2
The tension in the rope, T = Weight 2 3 Vertical height = 20 × 0.15 m
= mg 2×2 =3m
t2 =
For the horizontal forces: 5

CHAPTER 2
Work done, W = mgh
2T cos 35° = 125 t = 0.9 s = 30 × 10 × 3
2 mg cos 35° = 125 (b) v 2 = u2 + 2as = 0 + 2(5)2 = 20 = 900 J
125 v = 4.47 m s–1
m= 4 Work done, W = Fs
20 cos 35° 9 = 30 × 25
= 7.6 kg T cos 75° T cos 75°
= 750 J
75° 75°
Mass required = 7.6 kg 5 Work done, W
T T =
6 T sin 75° T sin 75° = 2 × Fs cos θ
R = 2 × 5000 × 1600 × cos 30°
6N 6N = 1.39 × 107 J
F
6 First, find the vertical height, h.
h
W = mg = 0.6 × 10 = 6 N cos 30° =
30° 10
sin N Considering the vertical forces:
mg =3
ion h =10 cos 30°
30° frict mg cos 30° 2T cos 75° = 6 = 8.66 m
3 Work done, W = F × s
T= = 11.6 N
cos 75° = mg × h
(a) Normal force, R
10 = 70 × 10 × 8.66
= mg cos θ
T2 cos 40° = 6062 J
= 20 cos 30° T1 cos 50°
= 17.3 N T1 50° 40° T 1
7 Kinetic energy = mv2
(b) Component of the weight down
2 = 2
T1 sin 50° T2 sin 40°
the plane = mg sin θ 150 N 1
150 N
= × 7 × 32
= 20 sin 30° 2
= 10 N = 31.5 J

605 Answers
8 According to the principle of 1.8 kW For 3 springs in parallel:
Efficiency = × 100%
conservation of energy: 2.0 kW 12
Extension = = 4 cm
Gain in kinetic energy = 90% 3
= Loss of potential energy Length of system
1 = Original length of system +
mv 2 = mg∆h Self Assess 2.12
2 Total extension
F1 x1
1 1 F = x = (18 + 18 + 18) + (12 + 6 + 4)
× v 2 = 10 × (1.5 – 0.3)
2 2 2 = 76 cm
10 6 7 For each spring, a compression of
v 2 = 24 =
20 x 5 cm requires a compressive force
v = 4.9 m s–1
x = 12 cm of 250 g.
9 Original potential energy, EP
∴ Length of spring = 20 – 12 = 8 cm m1 x1
= mgh =
= 60 × 10 × 3 m 3 m2 x2
2 =
= 1800 J m + 500 5 250 5
Loss of potential energy 5m = 3(m + 500) m2 = 8
= Gain in kinetic energy + 2m = 1500
m = 750 g m2 = 400 g
Work done against friction The compressive force on each spring
∴ Ep = Ek + (F × s) m1 1 – 0
3 in Figure (b) = 400 g
1 m2 = 2 – 0
= mv 2 + (F × s) ∴ m = 3 × 400 g = 1200 g
2 300 20 – 0 F
1 = 8 (a) k =
1800 = × 60 × 52 + (F × 3) 500 25 – 0 x
2
75 – 30 = 100 – 50 0.6 × 10
3F = 1050 20 = 25 =
0.12
F F = 350 N 0 = 12.5 cm
O 10 (a) Total mechanical energy = 50 N m–1
300 3 m1 x1
R = EP + Ek 4 For M:
600 x
=
(b) =
M 1 m2 x2
= mv 2 + mgh x = 6 cm
2 600 12
500 6 =
4 1 For N: = 800 a
= × 70 × 22 + 70 × 10 × 5 600 x
2 800 × 12
x = 7.2 cm a= = 16 cm
= 3640 J 600
y = Original length of system +
(b) Work done + Kinetic energy 1 2
Total extension (c) Ep = kx
= Original mechanical energy 2
1 = (12 + 12) + (6 + 7.2)
Work done + mv 2 = 3640 = 37.2 cm 1
2 = × 50 × 0.162
5 In Figure (a) load for each spring 2
1 = 0.64 J
Work done = 3640 – × 70 × 82 = 300 g
2
In Figure (b), the load for each spring 1 1
= 1400 J 9 (a) Ep = kx 2 = × 12.5 × 0.062
= 450 ÷ 3 2 2
11 Power = Force × Velocity
CHAPTER 2

= 150 g = 0.0225 J
= Fv
m1 x1 (b) Kinetic energy of sphere
= 35 × 8 =
m2 x2 = Potential energy of spring
= 280 W
300 3 1
Work done = × m × v 2 = 0.0225
12 (a) Power = 150 x 2
Time taken
150 ×3 1
mgh x = –––––– × 0.02v 2 = 0.0225
= 300 2
t
= 1.5 cm v = 1.5 m s–1
500 × 10 × 2
= p = Original length of system +
60
Total extension SPM Exam Practice 2
= 167 W = 9 + 1.5 Multiple-choice Questions
(b) Efficiency = 10.5 cm 1 B 2 D 3 C 4 B 5 A
Useful power output 6 First, find the extension for a single 6 C 7 C 8 C 9 C 10 D
= × 100%
Power input spring. 11 B 12 B 13 B 14 A 15 D
167 m1 x1 16 A 17 A 18 A 19 A 20 C
80% = × 100% m2
=
x2
Pi 21 C 22 C 23 A 24 B 25 D
Pi = 209 W 200 4
=
mgh 600 x Structured Questions
13 Power = 1
t 600 × 4 1 (a) s = ut + at2
x= = 12 cm 2
300 × 10 × 12 200
= For 2 springs in parallel: 1
20 24 = 0 + × a × 52
12 2
= 1800 W Extension = = 6 cm
= 1.8 kW 2 a = 1.92 m s–2

Answers 606
(b) F = ma 1 2 The skateboard deck should be
(b) Ep = kx
= 60 ×1.92 2 made of material with less density
= 115.2 N 1 so that the skater can accelerate
(c) v= u + at1 = × 5000 × 0.082 faster.
2
= 0 + 1.92 × 5 Grip tape should be applied to the
= 16 J
= 9.6 m s–1 upper surface of the skateboard
(100 –24) m (c) (i) Principle of the conservation
deck to provide better grip to the
t2 = of momentum
9.6 skater’s shoe.
Momentum Momentum
= 7.92 s (ii) = The truck should be made of
to the left to the right
Total time = 5 + 7.92 = 12.92 s durable alloy of less density to make
2m × v = 3m × 6 the skateboard lighter and stronger.
2 (a) Kinetic energy to potential energy v = 9 m s–1
1 The wheel should be made of
(b) mv2 = mgh (d) Total kinetic energy material with high resiliency so that
2
= Elastic potential energy they will give real good grip to the
1 1 1
× 6 × 6 = 9.8h × 3m × 62 + × 2m × 92 = 16 ground (urethane wheel will flatten
2 2 2 a bit on contact with the ground
h = 1.84 m 135m = 16 but resume to its round shape
(c) (i) Vertically m = 0.12 kg immediately).
(ii) Using v 2 = u2 + 2as, The wheel should be
v 2 = 0 + 2(9.8)(1.84) Essay Questions comparatively large, as large
v = 6 m s–1 5 (a) (i) Momentum is defined as the skateboard wheels roll a lot faster
mv – mu product of a mass and its (at the expense of stability).
(iii) F =
t velocity. The bearing should be fully
0 – 45(6) (ii) Since both eggs are released lubricated to reduce friction and
= F
1.2 from the same height, they enable the skateboard to move
= –225 N acquire the same velocity faster. O
Impulsive force on his legs whether it is hitting the 6 (a) The force constant of a spring is R
= 225 N concrete floor or the thick- the ratio of the stretching force to M
folded towel. Since both eggs the extension of the spring that is
3 (a) Work done
are of same mass, they being stretched. 4
= Force × Displacement
possess the same (b)
= 100 × 1.2
momentum. Force, F (N)
= 120 J
As both eggs are finally
(b) (i) 120 J
stopped by the impact, the
(ii)
changes in momentum are
89° 89° also equal. However, in the
T T case of the second egg, it
takes a longer time for the
100 N
change in momentum to
occur. This means the 3 Distance, x (cm)

CHAPTER 2
T cos 89° T cos 89°
impulsive force acting on it is The gradient of the slope in the
smaller. Thus, the egg remains force-distance graph is equal to the
= unbroken. force constant.
T sin 89° T sin 89°
The physical concept: For the first 3 cm, the force is
100 N To reduce the impulsive force, pushing one spring only. The force
one needs to prolong the constant is smaller. From 3 cm
2T cos 89° = 100 time of impact during a onwards, the force is pushing
T = 2865 N collision. against two springs. The force
(iii) It is impossible to pull the (b) Before the egg is released, it constant and, thereby, the gradient
ropes horizontally. possesses gravitational potential of the slope increase.
Let T = tension in the rope energy. (c) (i) 190 N is needed to stretch a
2T cos θ = 100 When the egg is falling, the spring from 30 cm to 40 cm.
Supposing θ = 90° potential energy changes to kinetic F = kx
energy, which increases with F 190
The ropes are horizontal. k= = = 1900 N m–1
increased velocity. x 0.1
100 When the egg hits the folded (ii) To stretch a spring by 22 cm
T=
2 cos θ The value towel, all the potential energy was (= 52 cm – 30 cm), using
50 cannot be changed to kinetic energy. The F1 F2
T= determined. kinetic energy is used to do work =
cos 90° x1 x2
50 against the resistance force of the
T= 190 F2
0 folded towel. =
10 22
(c) The skater’s attire should be tight
4 (a) 5000 N is needed to compress or and light so that there is less air 190
F2 = × 22 = 418 N
stretch the spring by 1 metre. resistance against motion. 10

607 Answers
∴ Stretching force needed (d) Velocity of the trolley, v, is Minimum pressure, P
= 3 × 418 proportional to the compression, F
=
= 1254 N e. A
(c) The force constant should be 0.05 N
=
higher so that more work will be 10–3 m2
done on the spring. This work is = 50 N m–2
3 Forces and Pressure
stored as elastic potential energy in = 50 Pa
the spring. Self Assess 3.1
There should be more 3 (a) Contact area
1 (a) The wooden block exerts
compression to increase the elastic = 405 cm2
maximum pressure when its
energy stored in the spring which = 405 × 10–4 m2
smallest surface is placed in
1 = 0.0405 m2
is equal to × force constant × contact with the table.
2 Weight of Wen Hui
= 47 × 10 = 470 N
compression2. Pressure exerted on the floor, P
The density of the dart should be
Force, F
small so that the mass of the dart 0.8 =
Area, A
is small. When the mass is small, it
470 N
can accelerate more to achieve =
0.0405 m2
faster velocity.
0.2 0.3 = 1.16 × 104 Pa
The body of the dart should be
smooth as this will reduce = 11.6 kPa
Maximum pressure (b) Contact area
resistance against the barrel of the
Weight = 1.5 cm2
toy gun, and will encounter less air =
Minimum area = 1.5 × 10–4 m2
resistance when it moves in the air.
F Toy gun Q is chosen as the spring 12 N 470 N
= Pressure exerted =
O used has the greatest force (0.2 × 0.3) m2 1.5 × 10–4 m2
R constant and can be more = 200 N m–2 = 3.13 × 106 Pa
M compressed than the others. = 200 Pa = 3130 kPa
The dart is smooth and of the (b) The wooden block exerts
4 Weight of car, F = 1000 × 10
4 smallest density. minimum pressure when its
= 104 N
largest surface is placed in contact
Experiment Area of tyre in contact with the road, A
with the table.
1 (a) (i) Compression, e = 25 cm2
(ii) Velocity, v = 2.5 × 10–3 m2
(iii) Mass of trolley, m Total pressure, P
(b) (i), (ii) F
0.8 =
0.3 A
e (cm) x (cm) v (cm s–1) 104 N
Minimum pressure =
1.0 2.2 11.0 2.5 × 10–3 m2
Weight
= 4 × 106 N m–2
CHAPTER 2 & 3

2.0 4.5 22.5 =


Maximum area Pressure on each tyre
3.0 6.5 32.5 12 N
= 4 × 10 N m
6 –2
=
4.0 8.9 44.5 (0.3 × 0.8) m2 4
5.0 11.2 56.0 = 50 N m–2 = 1 × 106 N m–2
= 50 Pa
(c) 5 (a) (i) Pressure exerted by
2 Dimensions of copper block:
fingertip, Pf
Graph of v against e 1.8 N
v (cm s )
–1 =
1.2 × 10–4 m2
1 cm 5 cm = 1.5 × 104 N m–2
60
x 2 cm (ii) Pressure exerted by needle, Pn
50 1.8 N
ρ = 500 kg m–3 =
x 2.4 × 10–7 m2
= 0.5 g cm–3
40 V = 1 cm × 2 cm × 5 cm = 7.5 × 106 N m–2
= 10 cm3 Pn 7.5 × 106
x (b) =
30 Mass of copper block Pf 1.5 × 104
x = Density × Volume of block = 500
20 = 0.5 g cm–3 × 10 cm3 The pressure exerted by the
= 5 g = 0.005 kg needle is 500 times the
10 x Minimum pressure is exerted when the pressure due to the fingertip.
block is resting on its maximum surface The pressure exerted by a
area, i.e., given force increases as the
O 1 2 3 4 5 e (cm) 5 cm × 2 cm = 10 cm2 = 10–3 m2 surface area decreases.

Answers 608
Self Assess 3.2 4 (a) Atmospheric pressure (b) Weight of water displaced = 35 N
1 h=3m = 0.75 m × 13 600 kg m–3 × Volume of water displaced = V
Pressure, P 10 N kg–1 FB = ρVg
= hρg = 102 000 N m–2 35 = 1000 × V × 10
= 3 m × 1000 kg m–3 × 10 N kg–1 = 102 000 Pa 35 N
∴V =
= 30 000 N m–2 = 102 kPa (10 N kg–1 × 1000 kg m–3)
= 30 kPa (b) Pressure at a depth of 20 m = 3.5 × 10–3 m3
= 102 000 Pa + (20 m)
2 (a) Water pressure at the base of 2 smallest
(1025 kg m–3)(10 N kg–1) measurement
the dam
= 102 000 Pa + 205 000 Pa largest
= hρg
= 307 000 Pa measurement
= 25 m × 1000 kg m–3 × 10 N kg–1
= 307 kPa
= 250 000 N m–2
5 PA = PB (same level)
= 2.5 × 105 N m–2
Patm + h1 ρ1g = Patm + h2 ρ2 g
(b) The width of the wall of a dam
h1 ρ1 = h2 ρ2 lead
is much larger at the base than shots
12 × ρ1 = 8 × 1
at the surface of the water to
8 liquid of liquid of
ensure that the wall is able to ρ1 = higher density lesser density
12
withstand the pressure of the
water which increases with = 0.67 g cm–3 In a liquid of lesser density, a greater
depth. volume of liquid must be displaced
Self Assess 3.4
for the buoyant force to equal the
3 (a) Pressure, P 1 (a) The liquid pressure on the
weight and so more of the
= hρg piston of the small cylinder is
hydrometer is submerged. The
= 0.6 m × 1000 kg m–3 × 10 N kg–1 equal to the liquid pressure on
hydrometer floats higher in a liquid of F
= 6000 Pa the piston of the large cylinder.
higher density. For this reason, the O
(b) To obtain a pressure of 15 000 Pa This shows that pressure
hydrometer is marked with larger
at the shower head: applied to a stationary enclosed R
values of density at the lower part and
15 000 = hρg fluid is transmitted equally in all
lower values towards the top end.
M
15 000 directions. This relationship is
h= 3 Mass of balloon filled with helium gas, m
1000 × 10 known as Pascal’s principle. 4
= Mass of empty balloon + Mass of
=1.5 m (b) Pressure on the small piston
20 N helium gas
∴ The tank must be raised by = = 500 kg + (0.18 kg m–3 × 2000 m3)
1.5 m – 0.6 m = 0.9 m 5 × 10–4 m2
= 500 kg + 360 kg
= 40 000 N m–2 = 860 kg
Self Assess 3.3 Pressure on the large piston Weight of balloon filled with helium gas
1 Total pressure at the base of the pool = Pressure on the small piston = mg
= Atmospheric pressure + hρg = 40 000 N m–2 = 860 kg × 10 N kg–1
= 105 N m–2 + (6 m) (1000 kg m–3) = 40 kPa = 8600 N
(10 N kg–1) ∴ Force on the large piston, F Buoyant force

CHAPTER 3
= 105 N m–2 + 60 000 N m–2 = PA = Weight of air displaced
= 160 000 N m–2 = 40 000 N m–2 × 8 × 10–3 m2 = ρVg
Total force, F = 320 N = 2000 m3 × 1.3 kg m–3 × 10 N kg–1
= Total pressure × Area F = 26 000 N
2 (a) Pressure at A =
= 160 000 N m–2 × (50 m × 100 m) A ∴ Resultant force upwards
= 8 × 108 N 20 N = Buoyant force – Weight of balloon
=
2 Pressure at A = Pressure at B 0.1 m2 = 26 000 N – 8600 N
hA ρw a t e r g = hB ρX g = 200 Pa = 17 400 N
hA ρwater (b) Pressure at B = Pressure at A = 17.4 kN
ρX = hB = 200 Pa Self Assess 3.6
F2 A2 1 (a)
9 × 1000 (c) = L
= F1 A1
12
F2 0.5
= 750 kg m–3 =
Density of liquid X is 750 kg m–3. 20 0.1 H
0.5
3 Pressure at A = Pressure at B F2 = 20 × (b) Region L has a lower pressure
0.1
Patm + h1 ρ1g = Patm + h2 ρ2 g because the velocity of the airflow
h1 ρ1 = h2 ρ2 = 100 N is high.
(d) (i) Upward force increases. (c) Bernoulli’s principle.
h1 ρ1
ρ2 = (ii) Upward force increases. (d) The difference between the
h2
pressures at regions L and H
11 × 1000 Self Assess 3.5 produces a net upward force on
= –––––––––
9 1 (a) Buoyant force = (65 – 30) N the wing, which helps the airplane
= 1222 kg m–3 = 35 N to take-off.

609 Answers
(e) The trailing edge of the wing is the weight of the container Some lead are put in the glass
curved downwards during take-off and the object. bulb. This will ensure that the
so that the air velocity on the (iv) Archimedes’ principle. hydrometer is upright when it
underside of the wing is reduced. floats in a liquid (i.e. stability).
Essay Questions The upper part of the glass is made
SPM Exam Practice 3 3 (a) (i) Density is mass per unit of a small but long stem so that a
Multiple-choice Questions volume of the material. small change in density produces
1 D 2 B 3 D 4 A 5 A (ii) The wooden cube sinks more a large change in the level at which
6 A 7 B 8 D 9 A 10 B in the oil than in the water. it floats. This gives a greater
11 B 12 C 13 C 14 A 15 A When the cube floats in a sensitivity.
16 D 17 B 18 D 19 C 20 D liquid, it displaces its own The hydrometer consists of a big
21 A 22 B mass of liquid. Therefore, the bulb and a long stem. This will
Structured Questions mass of oil displaced is equal ensure that it does not sink fully
1 (a) (i) The piece of paper is pushed to the mass of water when it floats in a liquid with low
upwards. displaced. density and still floats with the
(ii) When air is blown across the Mass = Density × Volume lower part of the stem submerged
upper surface, it creates a As the mass is constant, in a liquid of high density. Thus the
region of low pressure. volume is more when density hydrometer can determine a wide
The higher pressure below is less. Hence, more oil is range of liquid densities.
the paper exerts a force displaced as it is less dense. A rough calibration on the stem
causing the paper to move Thus, the cube sinks more in can be done by floating the
upwards. the oil. hydrometer in turn in a liquid of
(b) Bernoulli’s principle A floating object sinks more in low density like alcohol, water and
(c) (i) Air flows faster at P. a less dense liquid. a liquid of high density like carbon
F (ii) The pressure of air decreases (iii) The principle of physics tetrachloride. Then the stem is
O when the air flows faster. involved is Archimedes’ scaled using the three levels of
R (iii) Principle (or law of flotation). known densities.
M (b) 4 (a) The irregular impact of a number
air flow balloon of marbles on the scale pan
helium produced a constant average force
4 gas
air on the scale pan. The collision of
the marbles on the scale pan
represents the elastic collisions
between the gas molecules and
the walls of the container. When
the molecules of the gas collide
2 (a) A stone has a large density and a with the walls of the container, a
small volume. As a result, the change in momentum occurs.
volume of water displaced by the The balloon displaces some air The rate of change in momentum
stone is not enough to produce a which is much denser than helium produces a force acting on the
CHAPTER 3

buoyant force to support the gas. walls of the container. The force
weight of the stone. On the other This results in a buoyant force acting on the walls produces gas
hand, a boat that is full of stones is which is equal to the weight of air pressure.
able to float because the shape of displaced. (b) The excess pressure is measured
the boat enables it to displace a The weight of the helium gas in the by the difference in height of the
large quantity of water; the balloon and the fabric material of liquid columns in the manometer.
buoyant force produced is large the balloon is less than the The liquid used must be mercury.
enough to support the weight of upward buoyant force. If water is used, the difference in
the boat and its contents. The net force causes the balloon to height would be about 10 m.
(b) (i) Additional volume rise to the sky when released. The height of the mercury level
= 0.02 m × 0.02 m2 (c) should be 40 cm to ensure that
low
= 4 × 10–4 m3 density the difference in height of 76 cm
stem
(ii) Mass of object can be achieved. If the height of
= 1020 kg m–3 × 4 × 10–4 m3 the mercury level is 20 cm, the
= 0.408 kg whole column of liquid would be
(iii) The density of distilled water is high pushed over to the open end.
density
lower (1000 kg m–3). If the The diameter of the U-tube should
container with the object is bulb be small. This reduces the volume
placed in distilled water, the of liquid to be used.
immersion depth will increase The distance between the two
so that a larger quantity of lead shots limbs should be quite close for
distilled water will be displaced easy comparison of liquid levels
to produce a buoyant force The design is as shown in the and for reducing the volume of
that is large enough to support diagram above. liquid to be used.

Answers 610
The manometer selected is Q to B and vice versa are the same; 3 (a) The specific heat capacity of a
because the liquid used is mercury, that is, the net rate of heat flow is substance is the quantity of
the height of mercury level is zero. Two bodies which are in heat required to raise the
40 cm, the separation between the thermal equilibrium have the same temperature of 1 kg of the
two limbs and the diameter of the temperature. substance by 1 °C.
U-tube are comparatively small. (c) A mercury thermometer is a type The heat capacity of a body is
(c) (i) 76 cm Hg (atmospheric of liquid-in-glass thermometer. Its the quantity of heat required to
pressure) function is based on the expansion raise the temperature of the
(ii) Pressure of gas supply and contraction of mercury with body by 1 °C.
= (76 + 30) cm Hg temperature change. The (b) C = mc
= 106 cm Hg thermometer consists of a thin- where C = heat capacity of the
= (1.06 m) × walled glass bulb filled with object,
(1.36 × 10 4 kg m–3) × mercury. The glass bulb is attached m = mass of object,
(10 N kg–1) to a glass capillary tube with a c = specific heat capacity
= 1.44 × 105 Pa small diameter. The space above of the object.
(iii) At the 30 cm mark the mercury column in the capillary (c) Heat energy = mcθ
tube is a vacuum so that there is = 2 × 380 × 15
Experiment
no air pressure to affect the = 11 400 J
1 (a) (i) The immersed portion of the
reading of the thermometer. In the (d) m1c(80 – θ) = m2c (θ – 30)
cylindrical container (h)
measurement of the temperature 1000 2500
increases linearly with the (80 – θ) = (θ – 30)
of an object, heat is transferred 1000 1000
increase in mass (m) of the
between the object and the
lead shots added. 3.5θ = 155
mercury in the bulb. The mercury
(ii) When m = 0, h = 2.0 cm. θ = 44.3 °C
expands until thermal equilibrium
(iii) Gradient of graph (e) The plastic handle of the pot can F
is reached between the mercury
absorb large amount of heat with
=
(13 – 4) cm and the object. The expanding O
only a small increase in
(80 – 15) g mercury rises up the capillary tube
temperature, so that the handle is
R
= 0.14 cm g–1 to a temperature reading marked
not too hot to be held by bare M
(b) Cross-sectional area of the on the capillary. The thermometric
hands.
cylindrical container, A property is the volume of mercury. 4
1 Self Assess 4.3
= = 7.14 cm2 Self Assess 4.2
0.14 1 Pt = m
1 ∴ 5 g → 90 000 J
(c) Archimedes’ principle. ∴ 1 g → 18 000 J P × 5 × 60 = 0.0893 × 3.36 × 105
(d) Ensure that the cylindrical Heat required to raise the temperature P = 100 W
container is stationary while taking of water from 27 °C to 60 °C 2 Heat released by water
the reading h. = mcθ = mcθ
= 50 × 4200 × (60 – 27) = 1 × 4200 × (25 – 0)
= 6 930 000 J = 105 000 J
Minimum mass of methylated spirit Heat released by water
4 Heat

CHAPTER 3 & 4
6 930 000 = Heat absorbed by ice
= = 385 g ∴ 105 000 = mice
Self Assess 4.1 18 000
1 A thermometric property of a substance Quantity of ice melted, mice
2 (a) One kilogram of water requires
is a physical property which is sensitive 105 000
4200 J of heat energy to raise its =
to temperature and varies linearly with 3.36 × 105
temperature by 1 °C.
a change in temperature. (b) The temperature in the beaker X = 0.3125 kg
2 (a) (i) 0 °C (ii) 100 °C is lower than the temperature in ∴ Remaining ice = 1 – 0.3125
(b) (i) 75 °C (iii) –50 °C the beaker Y when thermal = 0.6875 kg
(ii) 25 °C equilibrium is reached in both Percentage of ice remaining
(c) It is not possible to have a mark at beakers. This is because = 68.75%
point C to represent a temperature aluminium has a higher specific 3 (a) (i) Total heat = P × t
of –50 °C because the freezing heat capacity and it releases a = 60 W × 10 × 60 s
point of mercury is –39 °C. greater quantity of heat. = 36 000 J
27 – 2 (c) Let the temperature at thermal (ii) Specific latent heat of ice
3 × 100 = 75.8 °C
35 – 2 equilibrium be T. Heat
=
4 (a) Thermal equilibrium is a state in Heat released by Heat absorbed Mass
=
which the net rate of heat transfer the aluminium by water 36 000
=
between two objects is zero and sphere 0.2
the temperature of the two objects 0.5 × 900(100 – T) = 1.0 × 4200(T – 25) = 1.8 × 105 J kg–1
is equal. 45 000 – 450T = 4200T – 105 000 (b) Heat from the surroundings.
(b) Thermal equilibrium has been 150 000 = 4650T (c) The experimental value is lower
reached. Two bodies A and B are T = 32.3 °C than the standard value. The
said to be in thermal equilibrium (d) The assumption is that no heat is reason for this is that the total
when the rate of heat flow from A lost to the surroundings. mass of water collected is larger

611 Answers
than the correct amount. The water (c) According to Boyle’s law: (ii) No heat is lost to the
collected is not solely due to the P1V1 = P2V2 surroundings.
heater but includes water melted P2V2
due to the heat from the P1 =
V1 2 (a) (i) Solid
surroundings. 11 (ii) Solid and liquid
(d) To obtain an accurate result, the = 76 × (iii) Liquid
10
mass of ice melted by heat from (b) At its melting point, heat is
= 83.6 cm Hg
the surroundings must be taken absorbed to overcome the bonds
(d) • Mass of gas is constant.
into account in the calculation. between the particles of the
• Temperature of gas remains
The method is to set up a substance.
constant.
duplicate of the original The particles are freed from their
3 (a) When the temperature in the tank
apparatus but with the heater fixed positions and are able to
increases, the trapped air in the
switched off. The mass of water move about.
tube expands and the mercury
collected in 10 minutes is The heat absorbed does not
thread is pushed to the right. When
measured. In this way, the mass increase the kinetic energy of the
the temperature of the tank
of water melted solely by the particles of the substance and
reaches the set temperature, the
heater can be determined. therefore the temperature remains
mercury thread reaches C to
(e) The heater is not fully covered by constant even though there is a
complete the circuit and activate
the ice. This causes a loss of heat transfer of heat.
the audible alarm.
to the surroundings. (c) 40 °C
(b) When the atmospheric pressure
(f) Ice at → Water at → Water at (d) (i) Assuming no heat lost:
increases, the mercury thread will
0 °C H 0 °C H 100 °C Energy supplied by the heater
be positioned at a further distance
1 2

H1 = m = 0.5 × 3.4 × 105 from C. = Heat required to increase


= 1.7 × 105 J (c) The effect of this is that the alarm the temperature of the
F H2 = mcθ = 0.5 × 4.2 × 103 × 100 substance
will be activated at a higher
O = 2.1 × 105 J temperature. Q = mcθ
R Total heat = H1 + H2 (d) According to Charles’ law: = 0.8 × 1000 × (60 – 40)
M = 3.8 × 105 J V1 V2 = 16 000 J
Energy = Energy
Time = T1 T2 Power =
4 Power Time
V 16 000
T2 = T1 × 2
= 3.8 × 10
5
V1 =
500 (1200 – 800)
(273 + 27)(7) = 40 W
= 760 s =
5 (ii) Assuming no heat lost:
Self Assess 4.4 = 420 K Energy supplied by the heater
1 (a) Boyle’s law states that the = 147 °C = Heat absorbed by the
pressure, P of a fixed mass of gas substance when melting
varies inversely with its volume, V SPM Exam Practice 4 Pt = m
at constant temperature. Multiple-choice Questions Pt
1 B 2 B 3 C 4 C 5 A =
1 m
CHAPTER 4

∴ P ∝ (at constant temperature) 6 B 7 C 8 C 9 C 10 C


V 40(800 – 200)
11 C 12 D 13 B 14 C 15 A =
Charles’ law states that the 0.8
16 B 17 C 18 D 19 D 20 B
volume, V of a fixed mass of = 30 000 J kg–1
21 D 22 C 23 B 24 A 25 B
gas is directly proportional to = 30 kJ kg–1
26 B 27 C 28 C 29 A 30 A
its absolute temperature if the
pressure of the gas is held Structured Questions 3 (a) (i) 75 + 25
constant. 1 (a) (i) 100 °C = 100 cm Hg
∴ V ∝ T (at constant pressure) (ii) M has a lower specific heat (ii) Boyle’s law:
(b) According to Boyle’s law: capacity. P2V2 = P1V1
P1V1 = P2V2 When sphere M is heated to P1
V2 = 12 ×
350 100 °C, a smaller amount of ( )
Ah2 = P × Ah1
2
76 heat energy is stored in it
=(
100 )
75
= 55.3 cm3 compared to sphere N. This × 20
2 (a) Pressure of air in the capillary tube heat energy is released when
= 15 cm
= P, cm Hg it is placed in beaker S and
(b) Charles’ law:
Pressure due to the weight of only manages to raise the
V2 V1
water to a lower temperature.
the mercury T2 = T1
(b) In the horizontal position, the (b) (i) Q = mcθ

( )
Q T2
volume of the trapped air increases θfinal – θinitial = Ah2 = T × Ah1
mc
because its pressure is reduced. 1

The pressure of the trapped air in 7200


θfinal – 30 = 348
the horizontal position is the 0.5 × 4200 = × 15
290
atmospheric pressure. θfinal = 33.4 °C = 18 cm

Answers 612
(c) Boyle’s law: Heat is released during (ii) Heat energy, Q
P3V3 = P2V2 freezing in Diagram 5(b). 60
P2 = 8.1 × 107 ×
( )
Both processes involve a 100
Ah3 = P × Ah2 change of phase without a
3 = 4.86 × 107 J
change in temperature. The (iii) Heat removed by the liquid
=(
75 )
100
× 18 temperature remains the = Heat generated
same because the heat = 4.86 × 107 J
= 24 cm
transferred is used to break or mcθ = 4.86 × 107
(d) The mass of the trapped air
restore the forces of attraction 4.86 × 107
remains the same.
between the naphthalene c =
mDθ
Essay Questions molecules (and not their
4 (a) Melting point is the temperature at kinetic energy). 4.86 × 107
=
which a solid changes into a liquid. (ii) Latent heat (of fusion). 200 × (91 – 36)
(b) (i) Diagram 5(a) involves the (c) Q = mcθ = 4418 J kg–1 °C–1
phase change of naphthalene = 0.3 × 1700 × (100 – 80) (c) The cooling liquid is circulated
from solid to liquid (melting) = 10 200 J around using a pump.
while Diagram 5(b) involves Since the average rate of heat loss Cooling liquid flows along the
the reverse phase change is 120 J min–1, the time required spaces in the engine walls
from liquid to solid (freezing). 10 200 absorbing heat generated from the
is = 85 minutes.
Heat is absorbed during 120 combustion of the fuel-air mixture.
melting in Diagram 5(a). (d) The cooling liquid becomes hot
and it flows to the radiator where
Way or modification Explanation heat is lost to the cooler air which
flows through the cooling fins. The
Container is made using material with a The container will be lighter. Hence, more
cooler air is sucked in from outside F
low density. portable.
by the radiator fan. O
Container is made using material with high Container is less easy to heat up or cool. R
specific heat capacity. This prevents the temperature inside the Experiments M
container from decreasing too quickly. 1 (a) θ0 = 26 °C
Ice is placed in the container. Ice keeps the seafood at a low θ (°C) 4
temperature. The high latent heat of
30
fusion of ice makes it a suitable coolant.
(0.6, 29.6)
A large amount of heat is absorbed by it as
it melts. 29

Saw dust is placed around and on top of It forms an air barrier which acts as an
the ice. effective insulation. It also absorbs the 28

melted ice water.


Container is white in colour. To avoid the absorption of heat from the 27 (0.2, 27.2)

surroundings.

CHAPTER 4
θ0 26
5 (a) (i) Specific heat capacity of a substance is the amount of heat that must be
supplied to increase the temperature of 1 kg of the substance by 1 °C or 1 K. 0
1
(kg–1)
0.1 0.2 0.3 0.4 0.5 0.6 0.7 0.8 m

(ii) Characteristic Explanation (b) θ is inversely proportional to m.


High specific Able to absorb more heat from the engine with only a (c) (i) Gradient of the graph, k
heat capacity. slight increase in temperature. 29.6 – 27.2
=
Relatively low Does not freeze in the winter and stop the cooling 0.6 – 0.2
freezing point. system from functioning in cold weather. =6
1.8 × 104
Relatively high Does not evaporate/boil easily when it gets very hot (ii) c =
k
boiling point. and causing an increase in the pressure of the engine.
1.8 × 104
Low rusting Does not cause the engine parts to rust and become c=
6
rate on metal. corroded. = 3000 J kg–1 °C–1
1
Liquid K should be used. (d) is 0.5 kg–1 when θ is 29 °C.
m
It is most suitable because it has a high specific heat capacity, relatively low 1
freezing point, relatively high boiling point and a low rusting rate on metal. Therefore, m = = 2 kg
0.5
(b) (i) Output power (e) The container is wrapped with
= Mechanical energy generated per second tissue paper or felt cloth and
8.1 × 107 40 placed on a polystyrene sheet to
= ×
1 × 60 × 60 100 prevent heat loss to the
= 9000 W surroundings and the table OR the

613 Answers
liquid is stirred with the stirrer as it (vi) Tabulation of data: 7 Distance travelled
is being heated up to maintain a = 0.8 × 5 × 2
uniform temperature. Temperature, Length of air column, =8m
2 (a) Inference: θ (°C)  (cm) 8 Place a concave mirror P on the
The volume of a fixed mass of gas 20 right side of B such that B is at the
is influenced by the temperature, if 30 principal focus of the concave
the pressure remains constant. mirror P. The reflected rays from the
40
(b) Hypothesis: light source A are parallel to the
The higher the temperature of a 50 principal axis and travel towards the
fixed mass of gas, the larger is its 60 concave mirror P. The parallel rays
volume, if the pressure remains are then reflected to converge at
70
constant. the principal focus at B.
(c) (i) Aim: (vii) Analysis of data: P
To investigate the relationship
 (cm)
between the temperature of a
fixed mass of gas and its
volume when the pressure is A B
constant.
(ii) Variables:
concave mirror
• Manipulated: Temperature θ (°C)
of air, θ 9 r = 2f
• Responding: Volume of The volume of a fixed mass of 20 cm = 2f
air/length of trapped air,  gas is directly proportional to f = 10 cm
• Fixed: Pressure and the its absolute temperature at (a) u = f (image at infinity)
F mass of the air in the tube constant pressure. = 10 cm
O (iii) Apparatus/Materials: (b) u = 2f (image is of the same size)
R Capillary tube, thermometer, = 2 × 10 cm
M beaker, half-metre rule, stirrer, 5 Light = 20 cm
rubber bands, beaker, retort 10 The image of the ladybird is inverted
Self Assess 5.1
4 stand with clamp, tripod stand, and diminished. It is flying towards
1 The image of the girl is 4 m behind the
Bunsen burner, ice and north.
mirror.
concentrated sulphuric acid. 11 concave
∴ Distance between the flower pot and
(iv) Arrangement of the 12 (a) The upper part of the image
the image of the girl = 3 m + 4 m
apparatus: disappears.
= 7m
(b) The image is unchanged except
ruler thermometer 2
that its brightness is reduced.
capillary tube
stirrer Self Assess 5.2
concentrated 1 After drawing the normal, it is clear that
ice sulphuric
the ray of light is refracted away from
acid index
CHAPTER 4 & 5

the normal, which means that medium


wire trapped air 1 is denser.
gauze
Bunsen 2 Does not change, does not change.
burner 3 From the formula:
3
(v) Procedure: T EW Refractive index, n
1 The capillary tube together Real depth, D
=
with half-metre rule is 4 For Aziz to be at a distance of 3 m from Apparent depth, d
immersed in a mixture of his image: The apparent depth, d is small if n is
ice and water with the 1 large. As n1 > n2, the apparent depth,
Object distance = × 3
whole length of the air 2 d, for liquid I is smaller. Hence, the
column beneath the water. Object distance = 1.5 m coin P appears nearer to the liquid
2 The water is then heated. ∴ Distance Aziz has to walk surface.
The water is stirred = (6 – 1.5) m 4 Refractive index, n
continuously so that its = 4.5 m Real depth, D
=
temperature is uniform. 5 ∠ABN = 60° Apparent depth, d
When the temperature, θ, ∠ABO = 90° – 60° 1.2
is 20 °C, the length of the 1.33 =
= 30° d
air column, , is read on ∠BAO = 180° – 100° – 30° 1.2
the ruler scale and = 50° d = = 0.9 m
1.33
recorded. ∠BAM = 90° – 50°
sin i
3 The experiment is repeated = 40° 5 n=
sin r
by heating up the water to ∴ θ = 40°
30 °C, 40 °C, 50 °C, 60 °C 6 ∠BOB′ = 60° – (2 × 15°) sin 60°
= = 1.35
and 70 °C. = 30° sin 40°

Answers 614
6 normal 4
(ii) m =
7
60°
i 90° (b) Virtual, diminished and upright.
6 (a) Concave lens (because image is
90° upright and diminished).
r v 1
(b) =
(d) u 2
u 20
sin i v= = = 10 cm
Use the formula: n = 1 2 2
sin r 5 sin c =
2.4 1 1 1
Draw a normal line at the point of + =
sin c = 0.4167 20 –10 f
incidence to determine the values
sin c = 24.6° f = –20 cm
of i and r.
6 c = 39° Focal length = 20 cm
i = 151° – 90°
= 61°
1 7 (a) Lens +2.5 D
n=
sin 39° 1
r = 126° – 90° (b) fo = = 0.4 m = 40 cm
= 36° n = 1.59 2.5
sin i 1
n= Self Assess 5.4 fe = = 0.0714 m = 7.1 cm
sin r 14
1 1 1
sin 61° 1 (a) (i) + = fo
n= u v f 40
m= = = 5.6
sin 36° 1 1 1 fe 7.1
+ =
0.8746 40 v 10 Distance between the lenses
n=
0.5878 1 = fo + fe
v = 13 cm F
n = 1.49 3 = 47.1 cm
O
Height of image v
(ii) = SPM Exam Practice 5
R
Height of object u
Self Assess 5.3 Multiple-choice Questions M
1 plane mirror Height of image
1 C 2 D 3 B 4 B 5 B
2 (a) an optically denser, less dense 1 4
13 6 A 7 C 8 D 9 A 10 B
(b) exceeds 3
= ×6 11 C 12 D 13 D 14 C 15 B
3 40 16 A 17 A 18 C 19 C 20 C
= 2 cm 21 B 22 B 23 D 24 A 25 B
(b) As u > 2f, the image is
real, inverted and diminished. Structured Questions
2 (a) v = 30 cm, u = 20 cm 1 (a)
49° v 30
m= = = 1.5
u 20
1 1 1 66mm
(b) + =

CHAPTER 5
u v f N
1 1 1 5m
4 + =
20 30 f
f = 12 cm
30° P Q
3 Focal length (f ) of lens R
= 5 cm
= 0.05 m (b) (i) P is seen walking straight
(a) 1 towards Q.
Power = = 20 D
0.05 (ii) Displacement
v = 52 + 62
4 m= =3
42° u = 7.8 m
v= 3u s
Velocity =
1 1 1 t
+ =
u v f 7.8 m
(b) Time =
1 1 1 5s
+ =
u 3u 15 Time = 1.56 m s–1
u = 20 cm (c) Q cannot be seen after both have
45° 1 1 1 walked 5 m to the right.
5 (a) (i) + =
u v f Distance
Time =
1 1 1 Velocity
+ =
9 v –12 5m
Time =
1 1 m s–1
(c) v =–5 cm
7 Time = 5 s

615 Answers
2 (a) (i) Because the light ray enters (b) If the size of the image and the The image formed is upright if
the glass block along the object are the same, then v = u. the object distance is less than
normal line. 1 1 1 the focal length.
+ =
(ii) The speed of light decreases u v f The image formed is inverted
because the nature of the 1 1 1 if the object distance is more
+ = than the focal length.
medium determines the u u f
speed of light. 2 1 (c)
(b) (i) 90° = objective eyepiece
u f construction
(ii) lens
u = 2f line
object Fo Fe I1 Fe
O = 2 × 11.25 observer
=22.5 cm 2Fo Fo
final
∴ The object must be moved virtual
(22.5 – 15) = 7.5 cm further image
(c) (i) & (ii) away from the lens. I2
normal v = 2f (d)
P = 22.5 cm objective lens
(c) Distance moved by image
=3×6 total
= 18 cm internal
O 18 cm reflection
v =
2s
Q normal
= 9 cm s–1
(d) (i)
F Note: A line drawn from any point
O on the circumference to the
R centre of a circle is the eyepiece
normal. O I
M (i) The prisms are glass prisms
3 (a) A prism periscope
(b) (i) that allow high transmission of
4 (ii) • Move the screen away from light so that all the light energy
the lenses. can be totally reflected.
• Move the object away from (ii) The prisms are right-angled
the lenses. triangles with the angles
measuring 45°–90°–45° so
Essay Questions that the light rays can deviate
5 (a) A real image is the image that can by 90° at each of the four
be seen and caught on a screen
points where total internal
whereas a virtual image can be
reflections take place.
(ii) So that total internal reflection seen but cannot be caught on a
(iii) The two prisms are arranged
occurs twice at the screen.
CHAPTER 5

as shown above. The two


hypotenuses of the prisms. (b) (i) Object distance in Diagram
hypotenuses are parallel but
The observer is able to see an 8 is shorter than the focal
diagonally perpendicular to
upright image of the object length while the object
each other. This will ensure
above the sea water. distance in Diagram 9 is
the image observed is upright.
(c) (i) As in (b)(i) longer than the focal
The advantages are:
(ii) Virtual (or upright) length.
• the prism binoculars produces
1 (ii) The image is on the same side
(d) n= upright images whereas the
sin c as the object in Diagram 8
astronomical telescope
1 1 while the image is on the
produces inverted images.
sin c = = opposite side of the object in
n 1.52 • The binoculars are shorter.
Diagram 9.
c = 41.1° 6 (a) The refractive index of a
(iii) For a virtual image to be
4 (a) (i) v = 3u transparent material is the ratio
formed by a convex lens, the
(ii) u + v = 60 cm of the speed of light in vacuum
object must be placed
4u = 60 cm to the speed of light in the
between the lens and the
u = 15 cm medium.
focal point. The object
v = 45 cm Speed of light in vacuum, c
distance must be less than the n=
1 1 1 Speed of light in medium, v
+ = focal length.
u v f Since the speed of light in a
For a real image to be formed
1 1 1 by a convex lens, the object medium is always less than the
+ =
15 45 f must be placed beyond the speed of light in vacuum,
45 focal point. (i.e. v < c), therefore the value for
f=
4 The object distance is longer c
n = is always greater than 1.0.
= 11.25 cm than the focal length. v

Answers 616
(b) The line of sight to the observer’s the coin after refraction at the Experiments
eyes is as drawn below. water surface does not enter the 1 (a) (i) Manipulated variable:
observer’s eye. The apparent Object distance, u
refracted ray does refracted ray
not enter the eye enters the eye
depth is not high enough. (ii) Responding variable:
For S, the apparent depth of the Magnification, m
R S image is raised higher (due to (iii) Fixed variable:
lin lin
sig e of sig e of greater depth) and the image is in Focal length of lens
ht ht
the line of sight. The light ray from (b) Image is real and inverted.
the coin is refracted and travels (c) Table:
along the line of sight to enter the
For R, the image of the coin is not y
observer’s eyes. u (cm) x (cm) y (cm) m=
in the line of sight. Light ray from x

12.0 1.0 5.00 5.00


(c) 15.0 1.0 2.00 2.00
The two prisms should be isosceles Periscopes work with light being
right-angled prisms as shown. deviated by 90° at the hypotenuse 18.0 1.0 1.25 1.25
45°
of the prism as shown. 21.0 1.0 0.90 0.90
24.0 1.0 0.70 0.70

(d) Graph of m against u:

F
The two prisms should be arranged This will ensure the light is deflected O
with hypotenuses parallel to each by 90° twice before entering the R
other and inclined at 45° with the observer’s eyes as shown. M
horizontal.
4

(e) The magnification of the image


decreases as the object distance
observer’s eye increases.

CHAPTER 5
2 (a) (i)
The refractive index, n chosen is 1.5. When n = 1, c, the critical angle is
42°. Total internal reflection will
occur at the hypotenuses of the
prisms. (If n = 1.3, the critical
angle > 45°, so periscope does
not work.) (15, 30)

The material for the prisms should not The image observed will be bright
scatter and absorb light as light rays and sharp.
pass through the prisms.

The periscope chosen is Q as the (d) 50°


prisms are isosceles right-angled
prisms arranged with the
hypotenuses parallel to each other From the graph,
20

and inclined at 45° with the v = 14.0 cm, when


°

horizontal, the refractive index for u = 35.0 cm.


the medium of the prisms is 1.5 sin i sin 50° y2 – y1
(i) n = = = 1.53 (ii) Gradient =
and has low scattering and sin r sin 30° x2 – x1
absorption of light. 1 1 30.0 – 10.0
(ii) sin c = = =
n 1.53 10.0 – 30.0
sin c = 40.8°
= –1

617 Answers
(b) u’ = 15.0 cm, v ’ = 30.0 cm Self Assess 1.2 4
1 1 1 1 (a) 5λ = 6 cm sea wave
+ =
15 30 f λ = 1.2 cm
2+1 1 (b) Frequency of the water wave port
=
30 f = nf′
f = 10.0 cm =6×5
= 30 Hz wall barrier
(c) The lens and object are of the
same height or the lens and screen Speed of the water wave, v
are parallel to each other. = fλ The wall barrier can protect the port
= 30 × 1.2 from large waves because of the
= 36 cm s–1 effects of diffraction. A wave passing
Form 5 2 (a) constant through the gap between the walls
(b) change (direction changes) carries only a small portion of the
1 Waves (c) constant energy with it.
(d) constant
Self Assess 1.1 3 25° Self Assess 1.5
Number of oscillations 1 Node points: Q, R, V
1 Time taken = Antinode points: P, S, T, U
Frequency of oscillation Self Assess 1.3
120 1 2 (a), (b) and (c)
=
4
Water wave travels antinodal line
= 30 s Characteristic from a shallow region nodal line
2 v = fλ to a deeper region antinodal line
= 240 × 6 Speed Increase nodal line
= 1440 m s–1 antinodal line
F Wavelength Increase
3 (a) f = 3 Hz
O Frequency Unchanged
3 oscillations in 1 second. 3 a = 1.0 cm,
R ∴ Number of oscillations in D = 20 cm,
M 2
10 seconds = 10 × 3 = 30 x = 2 × 3.0 cm
1 = 6.0 cm
4 (b) Period, T =
f ax
λ=
& 1 D
5 =
3 1×6
λ=
= 0.33 s 3 In the shallow region, 20
4 Frequency, f1 = x v = fλ λ = 0.3 cm
Wavelength, λ1 = λ cm 2 = f(1) 4 a = 0.2 mm
Given that f2 = 2x, find λ2. ∴ f = 2 Hz = 2 × 10–4 m
From v = fλ, v is a constant as the depth Frequency is constant. D = 2.5 m
of water doesn’t change (the same ∴ Frequency in region ABCD = 2 Hz 4x = 2.4 cm
ripple tank). Speed in region ABCD, v = fλ x = 0.6 cm
CHAPTER 5 & 1

f1 λ1 = f2 λ2 =2×5 = 6 × 10–3 m
f1 λ1 = 10 cm s–1 ax
xλ 1 4 (a) The depth of water in the middle λ=
λ2 = f = = λ D
2 2x 2 of the sea is almost uniform.
= 2 × 10 × 6 × 10
–4 –3
5 (a) 4 cm (b) On approaching the beach,
the depth of water decreases. 2.5
3
(b) T=3 The speed of the waves = 4.8 × 10–7 m
2
decreases and the wavelength is 5 λ = 450 nm
2
∴ Period, T = 3 × =2s decreased. = 4.5 × 102 nm
3
(c) The depth of water varies = 4.5 × 102 × 10–9 m
1 1 = 4.5 × 10–7 m
Frequency of wave, f = = Hz across the area of the bay and
T 2 D = 4.0 m
the energy of the water wave
(c) v = fλ spreads to a wider area x = 1.6 mm
v 8 compared to the region near = 1.6 × 10–3 m
λ= = = 16 cm
f 1 the cape. The amplitude of the ax
λ=
2 water wave near the bay is low D
11 and hence the water at the bay λD
6 (a) Amplitude, A = is comparatively calmer. ∴a = x
2
= 5.5 cm 4.5 × 10–7 × 4
36 Self Assess 1.4 =
(b) Wavelength, λ = 1 smaller than the wavelength of the 1.6 × 10–3
2
waves = 11.25 × 10–4
= 18 cm
2 noticeably less = 1.125 × 10–3 m
7 f3 > f1 > f2 3 circular = 1.125 mm

Answers 618
6 Self Assess 1.6 3 (a) A = X-ray
x 1 (a) 0.006 s B = ultraviolet ray
(b) 1.5 m C = infrared
1.5 m D = radio wave
(c) Speed of sound =
0.006 s (b) (i) Radio wave
= 250 m s–1 (ii) γ-ray
2 (a) Wave form C because this wave (c) Electromagnetic waves are
O a transverse waves.
form has a very high amplitude.
Sounds with higher amplitudes Electromagnetic waves travel at the
are louder. speed of light in vacuum.
7 x1 = 0.6 mm
(b) Wave form A because this wave (d) Visible light
= 6 × 10–4 m
form has a very low frequency. (e) (i) Kill cancer cells
λ1 = 6.5 × 10–7 m
The pitch of a sound depends on (ii) For vision
Light P: x2 = 0.4 mm
its frequency. (iii) For cooking in microwave
= 4 × 10–4 m
3 a = 2.0 m ovens
λ2 = ?
ax v 330 SPM Exam Practice 1
From λ = , λ= = = 0.825 m
D f 400 Multiple-choice Questions
λ a x = 2 × 60 cm = 120 cm = 1.2 m 1 C 2 C 3 D 4 C 5 A
x = D (constant) ax 6 B 7 B 8 B 9 D 10 D
λ=
λ1 λ2 D 11 A 12 C 13 B 14 D 15 B
ax 16 D 17 D 18 B 19 B 20 B
x1 = x2 D=
λ
Structured Questions
6.5 × 10–7 m λ2 2 × 1.2
= 4 × 10–4 = 1 (a) longitudinal wave
6 × 10–4 m 0.825 F
(b) O
6.5 × 10
–7 = 2.91 m
X
∴ λ2 = × 4 × 10–4 4 t = 0.3 s, d = 45 m R
6 × 10–4
vt M
= 4.33 × 10–7 m From d = :
2
(c) parallel to
2d 90 5
8 (a) v = = = 300 m s–1 (d) Energy
t 0.3
2 (a) Interference
2d (b) Longitudinal wave
5 Time interval, t =
v (c) λ = 0.5 m, D = 8.0 m, x = 3.6 m
2(400 – 150) ax
yellow
= λ=
300 D
500 a(3.6)
= 0.5 =
(b) 300 8.0
2 0.5 × 8
=1 s ∴a=
3 3.6

CHAPTER 1
6 v = fλ = 1.11 m
blue v is constant. (d) The distance between two
f1 λ1 = f2 λ2 consecutive rows of students
(c) fλ = (3f )λ2 increases.
1 (e) Wavelength increases when the
∴ λ2 = λ
3 frequency decreases.
3 (a) So that the velocity and
Self Assess 1.7 wavelength of the water waves are
green 1 Radio waves, infrared, visible light, uniform throughout the ripple tank.
ultraviolet rays, X-rays (b) (i)
The greater λ is, the greater the 2
value of x.
Electromagnetic Sound
9 a = 0.05 cm wave wave
= 5 × 10–4 m • Transverse • Longitudinal
D = 2.5 m wave wave
7x = 21 mm
• Can propagate • Cannot
x = 3 mm dark bright dark bright dark
through propagate
= 3 × 10–3 m (ii) f =12 Hz, λ = 5 cm
vacuum through
ax v = fλ = 60 cm s–1
λ= vacuum
D (iii) f = 24 Hz, v = 60 cm s–1
• Travels at the • Travels at a
= (5 × 10 ) × (3 × 10 )
–4 –3
v 60
speed of light, low speed, ∴λ= = = 2.5 cm
2.5 f 24
3 × 108 m s–1 330 m s–1 in air
= 6.0 × 10–7 m (iv) Stroboscope

619 Answers
(c) wavelength of the water wave source of disturbance which gives
decrease as the water waves rise to 'dead spots', where sound
incident
waves approach the shallower edge cannot be heard at certain areas of
of the pond. the hall. Echoes in a hall can be
(ii) The water waves at the deep reduced by the use of soft
region (middle of the pond) materials, laminations and
normal
and shallower edge of the perforated panels for the walls,
pond appear stationary when cushioned chairs, curtains and
reflected waves observed through a carpeting; that is, materials which
stroboscope. This confirms do not reflect sound.
4 (a) Reflection that the frequencies of both (c) Ultrasonic sound waves have
(b) (i) A high frequency sound has waves are the same. frequencies larger than 20 kHz.
high energy and it can be (c) • This occurrence is due to the Ultrasonic waves are used by
propagated over a long distance refraction of the sound waves. fishermen to increase their catch.
with little loss of energy. • At night, the layers of air near the Ultrasonic waves are emitted to
(ii) v = fλ ground are cooler compared to the shoal of fish and the time taken
1500 = 6.6 ×105λ the upper layers of air. to receive the echo, t is used to
1500 • Sound travels at a slower speed determine the distance of a shoal
λ=
6.6 × 105 in cold air and it is refracted away of fish from the ship by using
= 2.27 × 10–3 m from the normal on entering vt
the formula, d = , where d is the
(iii) Let d = depth of the sea warmer air. 2
t = 1.4 s, v = 1500 m s–1 • The sound waves are refracted distance and v is the speed of
vt 1500 × 1.4 in a downward curve towards the sound in water.
d = =
2 2 ground instead of disappearing 7 (a) (i) A transverse wave is a wave in
F into the upper layers of air. which the vibration of particles
= 1050 m
O (c) • Fishermen use sonar equipment • The two factors are air of medium is at right angles to
R to find the location of a shoal of temperature and density. the direction in which the
M fish before lowering their nets. 6 (a) (i) Similarity: wave is travelling.
• Geologists use sonar equipment Both waves are transverse A longitudinal wave is a wave
5 to find the location of minerals in waves. in which the vibration of the
the ground. Difference: particles of medium is along
Light waves travel at a speed the line in which the wave is
Essay Questions of 3 × 108 m s–1 in vacuum, travelling.
5 (a) (i) Similarity: whereas the speed of water (ii) The wavelength of sound
Both waves demonstrate waves is much less. waves is much more than that
refraction. (ii) Characteristics: of X-rays.
Difference: • Light travels from water to (b) Ultrasound is safer than X-rays
A water wave is a transverse air (leg is the object) and whose high penetrating power can
wave, whereas a sound wave water waves travel from a kill the foetus in the mother's
is a longitudinal wave. region of deep water to a
CHAPTER 1

womb. In addition, ultrasonic sound


(ii) • A wave is refracted towards region of shallower water. waves can differentiate between
the normal when it enters a • The speed of light increases layers of different soft tissues.
denser medium (balloon of and the speed of water (c) Uses:
carbon dioxide). waves decreases. • Ultrasound: To detect defects
• A wave refracts away from • The wavelength of light such as cracks in metal structures.
the normal when it exits a waves increases and the • X-rays:
denser medium. wavelength of water waves — To detect bone deformation
• The sound waves converge decreases but their or fracture by taking an X-ray
at the microphone which is frequencies are constant. photograph.
connected to the amplifier. • Light is refracted away from — Examination of passenger
The water waves converge the normal and the water luggage in airports.
at a point. waves are refracted towards (d) (i) Reflection of waves.
• At the point of convergence, the normal on emerging (ii) v = 2 × 106 m s–1,
a high amplitude wave is from water and entering the t = 1.2 ms = 1.2 × 10–3 s
produced. region of shallower water vt
• The balloon and the region respectively. The physics d =
2
of shallow water act as concept involved is refraction
= 2 × 10 × 1.2 × 10
6 –3
convex lenses. of waves.
• The wave phenomenon is (b) In a closed hall, the effects of echo 2
called refraction. prevent speeches from being = 1200 m
(b) (i) The depth of water decreases heard clearly. Listeners will hear 8 (a) Electromagnetic waves are waves
towards the edge of the pond. the original sounds followed by which are comprised of electric
Refraction of water waves their echoes. Apart from this, the and magnetic fields oscillating at
occurs. The velocity and effects of interference are another right angles to each other and both

Answers 620
are at right angles to the direction cracks) and the image formed (iv) Arrangement of apparatus:
of propagation of wave. γ-rays, on photographic plate will
X-rays, ultraviolet rays, visible light, show the defects more clearly. lamp
infrared rays and radio waves are (c) Frequency is inversely proportional motor
wooden bar
electromagnetic waves. to wavelength. rubber band
(b) (i) Characteristics of frequency (f) metal bar
water
electromagnetic waves: ripple tank
• High penetrating power aperture
shadow
enables waves to be used
white paper
for detecting structural
defects in an engine block. (v) Procedure:
• High energy carried by 1 The ripple tank is set
waves. up and filled with water
• Very short wavelengths and wavelength (λ) to a depth of about
hence their frequencies are 8 mm.
(d) λ = 1.5 cm, f = 20 Hz
1 2 The motor is switched on
high since λ ∝ . (i) v = fλ = 20 × 1.5
• f to produce plane water
= 30 cm s–1
(ii) S and T are suitable for use in waves. The rheostat is
(ii) Velocity is constant.
the investigation. T has high adjusted so that water
∴ f1 λ1 = f2λ2
penetrating power to the 20 × 1.5 = f2 (4) waves with a wavelength of
extent that it will fully penetrate f2 = 7.5 Hz ~3.0 cm are produced.
the material of the engine 3 The metal bars are
block. The image produced on Experiments arranged to form an
the photographic plate cannot 1 (a) (i) Length of pendulum aperture (opening)
(ii) Time for 20 2.0 cm wide.
F
be effectively interpreted.
Hence, S is more suitable. oscillations/Period 4 The pattern of the waves O
Its waves will penetrate only (iii) Mass of bob passing through the slit is R
the defective parts (e.g. (b) Table: observed with the M
mechanical stroboscope
and the pattern is drawn. 5
5 Steps 3 and 4 are repeated
Length of Time of 20 Period of oscillation,
T2 (s2) with the width of the slit
pendulum,  (cm) oscillations, t (s) T (s) increased to 4.0 cm.
40 25.4 1.27 1.613 (vi) Results:
50 28.6 1.43 2.045 (a) a = 4 cm
60 31.4 1.57 2.465
70 33.5 1.68 2.822 a
80 36.1 1.81 3.276

CHAPTER 1
(c) Graph of T 2 against : 2 (a) The effects of diffraction of waves
(b) a = 2 cm
are influenced by the size of the
T 2 (s2)
slit.
4 (b) If the size of the slit is a
X increased, the effects of the
3 X diffraction are reduced.
X

2 X
(c) (i) Aim: To determine the
X relationship between the (vii) Analysis of data:
1 effects of diffraction and If the width of the slit is less
the size of the slit. than or equal to the
0 (ii) Manipulated variable: wavelength of the water
10 20 30 40 50 60 70 80  (cm) Size of slit waves, the plane waves
Responding variable: become circular waves which
(d) (i) T 2 is directly proportional spread from a source behind
Effects of diffraction
to . the metal bars. In the case
Fixed variables:
(ii) Gradient, m where the width of the slit is
Wavelength and depth of
3.3 2 cm (i.e. less than 3.0 cm),
= water
80 the pattern of the diffracted
(iii) Apparatus/Materials:
= 0.04125 s2 cm–1 Ripple tank and its waves is clear. When the width
4π2 4 × 3.1422 accessories, mechanical of the slit is increased to
(e) g = =
100m 100 × 0.04125 stroboscope and metal 4.0 cm, the effect of
bars. diffraction is not obvious.
= 9.573 m s–2
The hypothesis is valid.

621 Answers
instantaneous current flowing through 4 For 3 resistors in parallel:
2 Electricity
it. If the instantaneous current is I1 and 1
Effective resistance = × 3 Ω
Self Assess 2.1 the corresponding voltage is V1 , then 3
Q 20 V1 Effective resistance = 1 Ω
1 I= = = 0.4 A the resistance at that instant is R = .
t 50 I1 Resistance of the whole external
2 Q = It = 0.2 × 60 = 12 C ρ ρ 4ρ circuit = 2 + 1 = 3 Ω
8 R= A = 1 = πd 2
Number of electrons πd 2 V 6
4 I= = =2A
= 12 × 6.25 × 1018 = 7.5 × 1019 R 3
3 Q = ne The two rods have the same resistance. The current of 2 A is divided equally
= 5 × 1018 × 1.6 × 10–19 4ρ 4ρ′ between the 3 identical resistors
πd 2 =
I=
= 0.8 C
Q
=
0.8
= 0.04 A = 40 mA
π d
2 ( )
1 2 connected in parallel.
2
∴ I1 = I2 = I3 = A
t 20 3
1
Q ′ = 
4 I= 4 5 (a) When the switch S is open,
t R=3+3=6Ω
Alternative solution:
Q 60 When the diameter is halved, the V 6
t= = = 200 s I1 = = = 1 A
I 0.3 resistance is increased by a multiple of R 6
Q 4. To have the same resistance, the When the switch is closed, the
5 IX =
t 1 effective resistance = 3 Ω
length of the rod Q must be .
2Q 4 V 6
IY = I2 = = = 2 A
3t R 3
IX 3 Self Assess 2.3 ∴ I1 : I2 = 1 : 2
Dividing: I =
F Y 2 1 (a) R = 2 + 3 + 6 = 11 Ω (b) When the switch S is open,
1 1 1 1 R = 6 Ω.
O fi IX : IY = 3 : 2 (b) = + +
6 Q = It R 2 3 6 V 6
R I1 = = = 1 A
= 3.5 × 10–5 × 6 × 3600 3+2+1 R 6
M = V1 = IR = 1 × 3 = 3 V
= 0.76 C 6
=1Ω When the switch is closed, the
5 effective resistance of the two
Self Assess 2.2 (c) For resistors in parallel:
1 (a) Switch C. Resistor R (constantan 1 1 1 3 1 resistors in parallel
= + = =
wire) has the highest resistance. R 3 6 6 2 1
= × 3 = 1.5 Ω
Therefore, when the switch is R =2Ω 2
closed, the current in the circuit is Effective resistance for the whole Effective resistance of the whole
the minimum. circuit = 2 + 2 = 4 Ω circuit = 3 + 1.5 = 4.5 Ω
(b) V and W. This is because this (d) Effective resistance for two V 6 4
section has the largest area and resistors in series = 2 + 3 I2 = = = A
R 4.5 3
the shortest length. The resistor =5Ω 4
with the smallest resistance 1 1 1 6 + 5 11 V2 = IR = × 3 = 4 V
3
CHAPTER 2

produces the largest current. = + = =


R 5 6 30 30
2 Reading on ammeter: decrease V1 : V2 = 3 : 4
Effective resistance for the whole 6 Resistance of one section
Brightness of bulbs: decrease
30 =3×3
V 3.0 circuit, R = = 2.73 Ω
3 R= = = 8.57 Ω 11 =9Ω
I 0.35
2 (a) R = 1 Ω + 3Ω = 4 Ω (resistance is inversely proportional to
V 240 V 8
4 I= = =4A (b) I = = =2A area).
R 60 R 4 Effective resistance of two sections in
V 2 1 (c) The reading will decrease 1
5 R= 1 = = Ω parallel = × 9 Ω
I1 6 3 (because the total resistance has 2
V2 = I2R = 6 × 1 = 6 V increased to 7 Ω). parallel = 4.5 Ω
Voltage supplied by the cell 3 The current in A1 is the main current V 3 2
I= = = A
= V1 + V2 = 8 V which branches out to the resistors R 4.5 3
6 Find the value of R first. arranged in parallel. The resistance of 7 Resistance of wire (100 cm)
V 6 the lower branch is twice the resistance =6Ω
R= = =4Ω
I 1.5 of the upper branch. Thus, the main Resistance of XP and QY
V = IR = 2 × 4 = 8 V current is split into a ratio of 1 : 2 as it = 1.5 Ω each.
7 The copper voltameter is not an ohmic enters the resistors in parallel. In the circuit (b), the effective
1 resistance = 1.5 Ω + 1.5 Ω
conductor because the graph is not a And, of the main current flows
3 =3Ω
straight line which passes through the
origin. Therefore, its resistance is not through A2. (current does not flow in the circular
given by the gradient of the graph. 1 section, PWQ).
∴ A2 = × 0.9 A
The resistance of the voltameter is 3 V 3
∴I= = =1A
not constant and it depends on the ∴ A2 = 0.3 A R 3

Answers 622
Self Assess 2.4 3 P = 48 W Energy
1 (a) Vt = IR P = VI Power =
Time
1.2 = 0.3R 48 = 12 I
2 700 000
R=4Ω I=4A =
30 × 60
(b) E = Vt + Ir V 2

1.5 = 1.2 + 0.3r 4 P= = 1500 W = 1.5 kW


R
0.3 = 0.3r 12 × 12 SPM Exam Practice 2
r=1Ω 24 =
R Multiple-choice Questions
2 E = I (R + r) 1 B 2 A 3 C 4 D 5 A
R=6Ω
6 = I (2.5 + 0.5) 6 A 7 A 8 C 9 C 10 B
Energy, E = Power × Time
I=2A 11 B 12 A 13 B 14 D 15 C
= 24 × 60 = 1440 J
Vt = IR = 2 × 2.5 = 5 V 16 A 17 A 18 C 19 A 20 B
55% of the energy is in the form of heat.
3 (a), (b) 21 C 22 D 23 B 24 C 25 A
55
E = I (R + r) ∴ Heat energy = ×E
100 Structured Questions
When two resistors are connected in
series: = 0.55 × 1440 1 (a)
External resistance = 4 Ω = 792 J
E = 0.4(4 + r) . . . (i) V 12
5 (a) (i) I = = =4A
When two resistors are connected in R 3
A
parallel: (ii) For P: 2Ω
1
External resistance = × 2 Ω = 1 Ω P = I2R = 4 × 4 × 1 = 16 W 1Ω
2
For Q:
E = 1.2 (1 + r) . . . (ii) P = I2R = 4 × 4 × 2 = 32 W 1Ω
(i) = (ii) (b) For P:
∴ 1.2 (1 + r) = 0.4 (4 + r) 2
12 × 12
2Ω F
1.2 + 1.2r = 1.6 + 0.4r P= V = = 144 W 1 O
0.8r = 0.4
For Q:
R 1 R=1+
2 (
×2 +1=3Ω ) R
r = 0.5 Ω
12 × 12
M
From (i): E = 0.4 (4 + 0.5) P= = 72 W
2
= 1.8 V 5
4 From E = I (R + r): 6 Energy output used A
P = I (R + R) = Heat energy used to boil water
2Ω (= 3 Ω)
P Energy = mcθ
I=
2R = 3.6 × 4200 × (100 – 25) 1Ω
P = 1 134 000 J
V = IR = ×R Energy input = Power × Time 1Ω (= 3 Ω)
2R
1 =P×t 2Ω
V = IR = P = 2000t
2
Energy used 1
5 E = I (R + r) Efficiency = × 100% R= × 3 Ω = 1.5 Ω
Energy input 2

CHAPTER 2
2 = Imax (0 + 0.01)
1 134 000 E 3
2 90% = × 100% (b) I = I=
Imax = = 200 A 2000 × t R+r 1.5 + 0.5
0.01
t = 630 s = 10 minutes 30 seconds 3
= = 1.5 A
Self Assess 2.5 7 The quantities of energy used by the I 2I
1 E = I2Rt = 2 × 2 × 3 × 300 = 3600 J two sisters are the same. V 1.0
2 (a) R = = = 2.0 Ω
E varies directly with the square of the For the elder sister: I 0.5
current (E ∝ I2). When the current E = Pt (b) (i) Reading increases
increases to 2I, the square of the = 1500 × 30 × 60 (ii) Reading decreases
current increases 4 times. = 2 700 000 J (c) (i)
∴ E = 4 × 3600 For the younger sister:
1.5 Ω
= 14 400 J 2 700 000 = VIt
2 (a) P = VI 2 700 000 = 240 × 1.2 × t X /Y = X /Y P
12 = 240 I t = 9375 s
I = 0.05 A = 2.6 hours P 0.5 Ω
(b) Heat lost per second 8 1 unit of energy = 1 kWh
= 12 – 8 1 1 1
= 1000 × 3600 = +
=4J R 1.5 0.5
= 3 600 000 J
(c) Efficiency 20 sen → 3 600 000 J 1+3
=
Puseful 1 3 600 000 1 1.5
= P × 100% sen → × = 90 000 J
2 20 2 1.5
input R = = 0.375 Ω
8 In 30 minutes: 4
= × 100% V 1
12 E = 90 000 × 30 (ii) I = = = 2.7 A
= 67% = 2 700 000 J R 0.375

623 Answers
1 (d) Yes, because by this method, heat For the accumulator, the
3 (a) Since R ∝ , where A is the cross-
A loss to the surroundings can be internal resistance, r is much
sectional area, and RP = 2RQ, reduced. White surface is not a smaller, resulting in a greater
1 good radiator. flow of current and
therefore, AP = AQ. (e) The time taken to boil the water subsequently larger light
2
will increase because part of the intensity.
The ratio is 1 : 2. heat from the heater is lost to the (ii) • Use two lead acid
(b) Since P and Q are in parallel, the surroundings by radiation. accumulators connected in
potential differences across P and parallel. The effective
Q are the same. internal resistance becomes
2 Essay Questions
From E = V , Q with a smaller 5 (a) Resistance is the ratio of
one half. This will further
R increase the current flow.
potential difference across the
resistance will dissipate more • Install lamp reflector to the
resistor to the current flow
heat. lamp so as to reflect light to
through the resistor.
(c) (i) Force of attraction (since P improve the light intensity.
(b) (i) The resistor in Diagram (b) is
and Q carry current in the
longer and of smaller cross- 12 V, r
same direction). r
sectional area. r effective =
(ii) Force of attraction still 2
(ii) The current in Diagram (b) is
exists. When the current
less than the current in install lamp two
supplied to the rods 12 V accumulator
Diagram (a). reflector
reverses direction, the two
(iii) Since I′  I, the resistance of light will not be
currents in the rod reverse wasted, but reflected
the resistor in Diagram (b) is
together, that is, they are in to improve the light
greater than the resistance
the same direction. Thus, intensity
of the resistor in Diagram
F the two rods still attract
(a).
O each other.
(iv) The resistance increases when
6 (a) When a current flows in a resistor,
R (d)
the length of the resistor
heat is produced and the resistor
M 0.2 Ω 0.2 Ω increases and the cross-
becomes hot. The dry cell
Rʹ Rʹ becomes hot because of its
sectional area decreases.
internal resistance. The internal
5 (c) Let the resistance of the disc be R Ω.
resistance is caused by the
0.1 Ω 0.1 Ω ∴ Resistance of 1 part
electrolyte in the cell.
1
=
1
+
1

(
= 4R Ω because R ∝ )
1
A
(b) The electromotive force in both
R′ 0.2 0.1 circuits are the same (1.5 V).
Resistance for stacked up parts However, the total resistance in
1+2 = 4 × 4R
= circuit (b) is less because the
0.2 = 16R Ω effective internal resistance of
0.2 V
R′ = For (a): I = two dry cells connected in
3 R
parallel is half the internal
∴ R = 2R′ V resistance of one dry cell.
For (b): I′ =
CHAPTER 2

0.2 16R Therefore, a larger current flows


=2×
3 V V in the circuit (b) and the light
∴ I : I′ = : = 16 : 1
= 0.133 Ω R 16R bulb is more brightly lit.
4 (a) (i) Energy (d) (i) The battery of 8 dry cells (c) For electric kettle, the useful
= Power × Time should be replaced with a energy output is the heat absorbed
1 12 V lead acid accumulator or to boil the water.
= 0.5 kW × h
4 a 12 V car battery. To increase the efficiency of an
= 0.125 kW h The formula for current flow is electric kettle, the heat absorbed
(ii) Energy = 500 × 15 × 60 given by: by the kettle and the heat lost to
= 450 000 J E the surroundings during the
I=
(b) Total energy (R + r) process of heating the water
= Energy to heat metal container where E and r are the should be reduced.
+ Heat to boil water electromotive force and total The mass and the specific heat
= mc cc Dθ + mw cw Dθ internal resistance respectively. capacity of the kettle should be as
= (0.1 × 1200 × 75) + If r is small, I is large. small as possible since the heat
(0.5 × 4200 × 75) Both arrangements have the absorbed by the kettle is equal to
= 9000 J + 157 500 J same electromotive force, E mcθ. The smaller the mass and the
= 166 500 J but the internal resistance, r smaller the specific heat capacity
(c) (i) Pt = Total energy for the accumulator is much of the kettle, the less heat is
500t = 166 500 smaller. absorbed by the kettle.
t = 333 s For the original arrangement, The surface of the electric kettle
= 5 minutes 33 seconds the total internal resistance, r should be shiny as shiny
(ii) This is due to heat loss to the is large. This makes the surfaces reduce the loss of heat
surroundings. current flow small. by radiation.

Answers 624
The power should be as high as (ii) Compare the formula with (iv) Arrangement of the apparatus:
possible because with higher power, linear equation, y = mx + c.
the time needed to boil the water is r thermometer
=c rheostat
less. As a result, the heat lost to the E
surroundings will also be reduced. coiled ammeter
1 water
= intercept on the -axis resistor 12 V
The electric kettle with the highest I
wire stirrer
efficiency is kettle M as it has a = 0.32 gauze
smaller mass and specific heat r = 0.32 × E tripod
capacity, its surface is shiny and it = 0.32 × 1.55 stand
has higher power. Bunsen burner
= 0.496 Ω
(d) (i) Energy = Power × Time (d) While taking the reading, the line of (v) Procedure:
= 2000 × 30 sight should be perpendicular to 1 The fine iron wire is wound
= 60 000 J the scales on the meters, to avoid around a U-shaped glass
(ii) Efficiency parallax errors. rod to form a coiled resistor.
Useful energy 2 (a) Inference: The coil is secured to the
output The current in the circuit decreases glass rod with glue.
= × 100%
Energy input when the resistance of wire 2 The coiled resistor is
m × 9.8 × 16 increases due to an increase in connected to the circuit as
70% = × 100% shown in the above figure.
60 000 temperature.
m = 268 kg (b) The higher the temperature 3 The coiled resistor is fully
of the wire, the greater is its immersed in a beaker of
Experiments water.
resistance.
1 (a) (i) When the value of R increases, 4 The current is maintained
(c) (i) Aim: To investigate the
1 relationship between the at a value of 0.5 A using a
the value of also increases.
I resistance of a wire and its rheostat. F
Thus, the value of I decreases. temperature. 5 The water is heated to a O
(ii) From the graph, when (ii) Variables: temperature of 30 °C and R
1
R = 1.50 Ω, = 1.29 A–1 (a) Manipulated variable: the reading of the potential M
I Temperature, θ difference, V across the
∴ I = 0.775 A (b) Responding variable: coiled resistor is recorded. 5
(b) E = I (R + r) Resistance, R 6 Step 5 is repeated with the
E (c) Fixed variables: Type of temperature raised to 40 °C,
= I
R+r material, length and cross- 50 °C, 60 °C, 70 °C and 80 °C.
sectional area of wire 7 The corresponding value of
1 R+r
= (iii) Apparatus/Materials: V
I E the resistance, R = is
Insulated iron wire (s.w.g. I
1 R r 34) of length 2 m, U- calculated for each of the
= +
I E E shaped glass rod, beaker, above temperatures.
(c) thermometer, stirrer, 8 A graph of the resistance of
1 (A–1)

CHAPTER 2 & 3
I electrical power supply, the coiled resistor, R against
2.0 Graph 1 against R connecting wires, crocodile temperature, θ is plotted.
I
1.8 clips, and Bunsen burner. (vi) Tabulation of data:

1.6 (2.0, 1.61)


Temperature, θ (°C) Current, I (A) Potential difference, V (V) Resistance, R (Ω)
1.4 30 0.5 V1 R1
1.29
1.2 40 0.5 V2 R2
1.0 50 0.5 V3 R3
0.8 60 0.5 V4 R4
0.6 70 0.5 V5 R5
0.4 80 0.5 V6 R6
(0, 0.32)
0.2
(vii) Relationship between resistance, line with a positive gradient. This
0 R (Ω) R and temperature, θ: shows that the resistance increases
0.5 1.0 1.5 2.0 2.5 3.0
R (Ω) when temperature increases.
(i) First, find the gradient, m. The hypothesis is valid.
(1.61 – 0.32) A–1
m=
(2.0 – 0) Ω
3 Electromagnetism
= 0.645 A–1 Ω–1
1 Self Assess 3.1
∴E= θ (°C)
0.645 A–1 Ω–1 1 A
= 1.55 A Ω Conclusion: The ends of both solenoids nearer to
= 1.55 V The graph of R against θ is a straight the compass are north poles. The north

625 Answers
pole of the compass is repelled. But the 4 By using the right-hand grip rule, coil VP Output voltage is directly
magnetic field of the solenoid X is generates a current that flows from V to proportional to the number of
stronger than the solenoid Y. Therefore, P and then to B and A. At the same turns in the secondary coil.
the north pole is deflected eastwards. time, coil QW produces a current that Therefore, turns ratio = 12 : 60
2 N flows from W to Q and then to A and B. =1:5
However, the current by coil QW is (ii) For an ideal transformer:
S

Q
more as it has more turns than coil VP. Power input = Power output
P R The net current flows from A to B. In a series circuit:
N

N
S

5 (a) When viewed from the right, the Power output = 5 × 24 W


S

current in the coil flows in the = 120 W


clockwise direction as the ring Power input = Vp Ip
approaches the magnet. The 120 = 240 Ip
3 X : South current flows in the anticlockwise Ip = 0.5 A
Y : Neutral direction when the ring is moving
Z : North away from the magnet.
Use the right-hand grip rule to (b) The ring outside the influence Self Assess 3.5
determine the poles. of the magnetic field reaches 1 (a) P = VI
4 To increase the magnetic field strength the tabletop first. The ring 50 000 = 5000 × I
of the electromagnet: passing through the magnetic I = 10 A
• Replace the copper core with a soft field experiences a force of (b) Ploss = I 2R = 10 × 10 × 5
iron core. repulsion followed by a force of = 500 W
• Increase the number of turns. attraction. The acceleration of Power received by the factory
• Decrease the resistance of the the ring is reduced. = 50 000 – 500
rheostat. = 49 500 W
F 5 (a) The north pole of the compass is Self Assess 3.4 2 (a) P = VI
O deflected eastwards. 1 Ns : Np = 9 : 240 1 650 000 = 66 000I
R (b) The north pole of the compass is = 3 : 80 I = 25 A
M pointing northwards. 2 Vp Ip = Vs Is (b) Ploss = I 2R
240 × Ip = 3 × 0.3 = 25 × 25 × 40
Self Assess 3.2 Ip = 0.00375 A = 25 000 W
5 1 B. Using Fleming’s left-hand rule, we find 3 Power output = 48 W (c) Power received by the factory
the force acting in the direction of K. Power output = 1 650 000 – 25 000
2 Force of attraction because the currents Efficiency = × 100%
Power input = 1 625 000 W
flow in the same direction in the metal
80% =
48
× 100% Efficiency = 1 625 000 × 100%
plate. 240 × Ip 1 650 000
Inference: The coil windings attract
Ip = 0.25 A = 98.5%
each other.
2 3 P = VI
3 When the switch is closed, the currents 4 (a) P=V 1 600 000 = 20 000I
in the two rods are in opposite R
2
I = 80 A
directions. A force of repulsion is
4.5 = V Total resistance of cable
CHAPTER 3

produced. 2 = 8 × 5 × 10–1
V2 = 9 =4Ω
thread
V=3V Ploss = I 2R
Power output = 80 × 80 × 4
(b) Efficiency = × 100%
Power input = 25 600 W
F F 90% =
4.5
× 100% 4 Ploss = 0.5 × 2 000 000 W
240 × Ip 100
aluminium rod Ip = 0.021 A = 10 000 W
5 (a) Parallel Ploss = I 2R
4 Current flows in the wire in the (b) P = VI 10 000 = I 2 × 8
magnetic field. A force perpendicular to 36 = 12 I I = 35.4 A
the wire is produced to push the wire I =3A P = VI
around the bar magnet. 2 000 000 = V × 35.4
Power output
(c) Efficiency = ×100% V = 56 500 V
Self Assess 3.3 Power input
= 56.5 kV
1 Q 36
90% = × 100%
Use Fleming’s right-hand rule. Power input
2 (a) P is a south pole and Q is a north Power input, Pinput = 40 W SPM Exam Practice 3
pole. Multiple-choice Questions
6 (i) When the bulbs are connected in
(b) X is a south pole. 1 C 2 D 3 B 4 A 5 B
parallel, Voutput = 12 V
3 (a) P is a south pole and Q is a north 6 A 7 B 8 B 9 A 10 A
pole. When the bulbs are connected in 11 B 12 C 13 D 14 B 15 B
(b) P and Q are similar poles of the series, Voutput required = 5 × 12 16 B 17 D 18 C 19 A 20 D
same magnetic strength. = 60 V 21 C 22 C 23 B

Answers 626
Structured Questions The alternating current causes A force is produced to move
1 (a) When the current flows, the a continuous reversal in the the foil from a region with
upper end of the coil becomes direction of the force acting on stronger magnetic field to a
a north pole. Since like poles the copper rod. As a result, the region with weaker magnetic
repel, the magnet is pushed resultant force is zero. field.
upwards. Thereby reduces the 4 (a) (i) Therefore, the foils PQ and RS
reading on the spring balance. P Q are pushed inwards, i.e. attract
(b) • Increase the magnitude of each other.
current by reducing the S N 2
3
resistance in the rheostat. 1

• Increase the number of turns


hair spring

0
of the coil.

4
pointer current
(c) The reading increases. (ii) Part of the kinetic energy is in
2 (a) (i) Q and P F coil on
used to produce induced F aluminium
(ii) T and P current for electrical
frame
(b) Output voltage is directly
proportional to the number of
turns on the secondary coil.
energy.
Or
N S permanent
magnet

current
While the magnet is fixed soft out
When Q and P are connected, moving towards the coil,
iron cylinder
hair spring Y
the number of turns is the the end P is induced to be
bearing
minimum and when T and P a north pole. The magnet
are connected, the number of (c)(i) 1 Spiral hair spring used
experiences a force of
turns on the secondary coil is should be of low
repulsion. While moving
the maximum. stiffness so that the coil
away from the coil, the end F
(c) Since voltage is directly will rotate more before
Q is induced to be a north
proportional to the number being balanced by the O
pole. The magnet
of turns: experiences a force of
opposing couple from R
Ratio VST : VRQ = Ratio NST : NRQ attraction.
the springs. M
∴ VST : VRQ = 40 : 20 2 The magnet used
(b) (i) The magnitude of
= 2:1 should be strong. 5
deflection is less.
(d) (i) Q and S 3 The number of turns on
(ii) Only magnetic field lines
Vs N the coil should be
= s from one side of the bar
more.
Vp Np magnet are cut.
4 The area of the coil
12 Ns Essay Questions should be large.
=
240 1000 5 (a) (i) Magnetic field is the region The factors stated in 2 and
Ns = 50 turns where magnetic force acts. 4 will produce greater
Poutput (ii) The current in diagram 5(b) is turning force (couple) that
(ii) Efficiency = × 100% more than the current in will increase the angle of
Pinput
diagram 5(a). rotation for a current flow.

CHAPTER 3
Use the formula: P = VI
The maximum height reached The greater angle of
Power output of lamp,
by wire PQ in diagram 5(b) is rotation increases the
Poutput = 48 W
higher than the height in sensitivity of the meter.
90% = 48 × 100% diagram 5(a). A radial magnetic field is
240 × Ip Therefore, the maximum set up by winding the coil
Ip = 0.22 A height reached increases as on a spherical cylinder
3 (a) When the switch is closed, the current increases. and placing it between a
current flows from X to Y. Using The force is due to the pair of concave-shaped
Fleming’s left-hand rule, it is interaction of the magnetic magnets.
found that the direction of the fields by the permanent (ii) The radial magnetic field
force acting on the copper rod magnet and the current- and the spiral springs
is upwards. Therefore, the carrying conduct PQ. which obey Hooke’s law
reading on the weighing scale The magnitude of the force on cause the angle of rotation
is decreased. the wire increases when the of the coil to be directly
(b) (i) Rod R (heaviest rod). current increases. proportional to the current
(ii) Rod P (force acting (b) PQ and RS attract each other. flowing. A linear scale is
upwards is very strong as The currents flowing in PQ and thus prepared.
the current is large). RS produce magnetic fields 6 (a) (i) Induced current is current
(iii) Rod S (plastic is not a around each foil and between produced by a changing
conductor; no effect the foils. magnetic field.
observed). The interaction of the fields (ii) From position P to Q, the
(c) The reading on the spring produce a weak magnetic field flux linkage in the coil does
balance, before and after the in between the foil but stronger not change (i.e. does not
switch is closed, is unchanged. fields outside PQ and RS. increase or decrease).

627 Answers
There is no induced current as (b) Hypothesis: Vs against number of turns, n
there is no change in flux The greater the number of turns in is plotted.
linkage. the secondary coil, the greater the Vs (V)
From position Q to R, the flux secondary (output) voltage.
linkage in the coil increases. (c) (i) Aim:
Therefore, an induced current To investigate the relationship
is produced and deflects the between secondary voltage
galvanometer. and the number of turns in
When the coil is moved from the secondary coil.
position R to Q, the flux (ii) Variables:
n (turns)
linkage is decreased. Thus a Manipulated variable:
current is induced again in the Number of turns in Conclusion:
opposite direction. secondary coil, n From the table and the graph,
(b) (i) Responding variable: it is found that the secondary
Secondary voltage, Vs voltage, Vs increases with the
240 V
12 V, 6 W Fixed variables: number of turns, n on the
12 V, 6 W Number of turns in secondary coil.
primary coil, primary The hypothesis is valid.
voltage
The voltage output is 24 V
(iii) Apparatus and materials:
when the two bulbs are lit
Low voltage power
normally. 4 Electronics
supply, two soft iron
Vs Ns
= C-cores, PVC-covered copper
Vp Np Self Assess 4.1
wire, voltmeter (a.c.), and
F Ns 1 (a) Energy of electron
24 C-clip.
= = eV
O 240 900 (iv) Arrangement of apparatus:
= 1.6 × 10–19 × 2500
R Ns = 90 = 4 × 10–16 J
M Po PVC-covered C-clip
(b) (i) Average rate of charge flow
(ii) Efficiency = × 100% copper wire secondary
Pi
= 4.2 × 10 × 1.6 × 10
14 –19

5 20
12 2V V
96% = × 100% = 3.36 × 10–6 A
240 × Ip voltmeter
(a.c.) (ii) Power produced by the tube
Ip = 0.052 A primary laminated soft iron = 3.36 × 10–6 × 5 × 103
(c) Copper coil is used so as to reduce (20 turns) C-core = 0.0168 W
the resistance in the coil and to
2 (a) Let u = Initial speed
improve the efficiency of the (v) Procedure: = 2.1 × 106 m s–1
transformer. 1 The apparatus is set up v = Final speed
The core should be laminated to as shown in the above = 5.2 × 106 m s–1
reduce the eddy current in the core figure. V = Potential difference
CHAPTER 3 & 4

so as to reduce the heating of the 2 The number of turns on the From the principle of
core. primary coil is fixed at 20. conservation of energy:
The iron core should form a close 3 The experiment is started with Electrical potential energy
loop of iron with the secondary coil 10 turns on the secondary = Gain in kinetic energy
wound over the primary coil. This is coil. The secondary voltage, = Final kinetic energy
to ensure no leakage of magnetic Vs is measured by an a.c. – Initial kinetic energy
flux. voltmeter. 1 1
The turns ratio = The voltages ratio ∴ eV = mev 2 – meu2
4 The experiment is 2 2
= 12 : 240 repeated with n = 15, 20, 1
( )
2 2
= 1 : 20 25 and 30. x = me v – u
Therefore, the number of turns on 2 e
(vi) Tabulation of data:
the secondary coil and primary coil 1
= × 9.1 × 10–31 ×
are 200 and 10 respectively. Number of turns Secondary 2

[ (5.2 × 101.6) ×–10(2.1 × 10 ) ]


The transformer selected is on secondary voltage, 6 2 6 2

transformer R as it consists of a coil, n (turns) Vs (V) –19

closed loop with laminated core,


10 Vs1 = 64.35 V
copper coils and with turn ratio of
15 Vs2 (b) Change in momentum
1 : 20 as required to step down a
= Final momentum
voltage from 240 V to 12 V. 20 Vs3
– Initial momentum
Experiment 25 Vs4 = mev – meu
1 (a) Inference: 30 Vs5 = me(v – u)
The output voltage is affected by = 9.1 × 10–31 ×
the number of turns in the (vii) Analysis of data: (5.2 × 10 6 – 2.1 × 10 6)
secondary coil. Graph of secondary voltage, = 2.821 × 10–24 kg m s–1

Answers 628
1 (b) (d) No reading
3 (a) Period =
Frequency (e) The diode is reverse-biased.
No current can flow through the
1
= diode.
50
4 (i) Consumes a small amount of
= 0.02 s
electrical energy.
(b) Time taken (c) The alternating current will be (ii) Very little heat is generated
= 15 cm × 2 ms cm–1 ‘smoothed’. (or any appropriate answer).
= 30 ms (d) Yes. By using a capacitor with a
= 0.03 s higher capacitance.
4 (a) 6 cm × 12 V cm–1 4 (i) Increase its temperature. Self Assess 4.4
= 72 V (ii) Shining light on the 1 (a) NOR gate
72 V semiconductor. (b) NOT gate
(b) = 36 V
2 (iii) Adding impurities to the (c) NAND gate
36 semiconductor. (d) AND gate
(c) Vr.m.s. = __
2 (e) OR gate
Self Assess 4.3
Vr.m.s. = 25.46 V 2 (a) (b)
1 Let the resistance of Y = Y
Self Assess 4.2 800
× 15 V = 5.5 V A B X A B X
1 800 + Y
Bulb Does not Lights up 800 5.5 0 0 1 0 0 1
light up =
800 + Y 15 0 1 1 0 1 1
A  4400 + 5.5Y = 12 000 1 0 1 1 0 1
B  5.5Y = 7600
1 1 0 1 1 0
Y = 1381.8 Ω F
C  2 (a) (i) To amplify the current. 3 (a) O
D  (ii) To control the amount of R
E  current flowing in base Input Output M
circuit. A B C
F 
(iii) To convert sound energy into
G  electrical energy. 0 0 0 5
H  (iv) To convert electrical energy 0 1 0
into sound energy. 1 0 0
2 (a) (v) To block the steady current
1 1 1
from flowing into the transistor
diode
and microphone.
(b)
(b) The collector current increases with
connected a.c. load connected greater magnitude. A
to CRO to CRO (c) (i) The amplitude increases. A•B
(ii) The frequency remains the B

CHAPTER 4
same.
(b) 3 (a) (c) C=A•B
4 (a) OR gate
IC
(b) NOT gate
IB 5 (a) NOR gate

(c) 3V Input Output


IE
with capacitor A B X
µA mA
0 0 1
0 1 0
(b) To amplify the current in collector
circuit. 1 0 0
(d)
IC 1 1 0
(c) (i) IB =
200
(b) AND gate
250 × 10–3
3 (a) =
200 Input Output
= 1.25 × 10–3 A
A B X
= 1250 × 10–3 × 10–3
= 1250 × 10–6 A 0 0 0
= 1250 µA 0 1 0
(ii) IE = IB + IC 1 0 0
= 1.25 × 10–3 + 250 × 10–3
= 0.25125 A 1 1 1

629 Answers
6 (a) Root mean square voltage During the positive half-cycle,

A B B C D 1 the capacitor is charged up. In
= __ (2) = 1.41 V
(i) 0 0 1 0 0 2 between the positive and negative
(ii) Period = 4 × 3.0 half-cycles, the capacitor releases
(ii) 0 1 0 0 0
= 12 ms its charge. It discharges partly
(iii) 1 0 1 0 1 (iii) Frequency through the load. The energy
(iv) 1 1 0 1 0 1 stored in the capacitor acts as a
=
Period reservoir and maintains the
(b) (iii) Alarm sounds potential difference across the load.
(iv) Door unlock but the alarm 1
= (d) (i) (ii)
does not sound. 12 × 10–3
(c) (i) = 83.3 Hz
R 10 kΩ bulb
(f) A cathode-ray oscilloscope can
A B C 6V
A measure a shorter time interval
C compared to a stopwatch. 1 kΩ E
B
B
2 (a) AND gate LDR
A (b)
D The components needed are a
B light dependent resistor (LDR) and
a bulb. The thermistor and the
(ii) – siren are replaced with the LDR
A B A C D
and the bulb respectively.
0 0 1 0 0 (c)
Output Y However, the positions of the
0 1 1 1 0 10 kΩ resistor and LDR are
F 0
interchanged. The LDR is
1 0 0 0 0
O 0 connected across the base-emitter
R 1 1 0 0 1 0 junction and the 10 kΩ resistor is
M 1 connected across the base-
(iii) The door is still locked but the collector junction.
alarm sounds. When it is dark, the resistance of
5 (d) Light dependent resistor (LDR).
7 output C the LDR increases. More of the
3 (a) p-type semiconductor and n-type supply voltage is dropped across
1 semiconductor. the LDR, thus raising the base
(b) In Diagram 5(a), the terminals of current. The transistor is now
0 the battery are connected in switched on. Collector current
reverse to the terminals of the flows and the bulb lights up.
SPM Exam Practice 4 semiconductor diode. In this
Multiple-choice Questions arrangement, the junction voltage 4 (a) P – variable resistor
1 C 2 B 3 B 4 A 5 D slowly builds up until it reaches Q – thermistor
6 C 7 A 8 B 9 A 10 C the same potential difference as R – resistor
CHAPTER 4

11 C 12 C 13 C 14 B 15 A the battery. At the same time, the S – transistor


16 D 17 B 18 C 19 A 20 A resistance of the diode increases. (b) As the temperature increases, the
21 D 22 B 23 D 24 C 25 D Hence, no current flows through resistance decreases and vice-
the ammeter. versa.
Structured Questions In Diagram 5(b), the positive and (c) (i) As a control switch.
1 (a) When the cathode is heated to a negative terminals of the battery (ii) To switch on the heater which
high temperature, electrons gain are connected to the anode and requires a large voltage.
enough energy to break loose and cathode of the semiconductor (d) To prevent the heater from
escape from the surface. diode respectively. In this overheating.
(b) Thermionic emission arrangement, the resistance of the (e) When the temperature in the room
(c) (i) Regulates the number of semiconductor diode decreases. is low, the resistance of the
electrons which reach the Hence, current flows through the thermistor increases. More
anode and therefore control ammeter. potential difference is dropped
the brightness of the spot on (c) (i) across the thermistor. The
the screen. transistor is activated and the
(ii) Accelerates the electron heater is turned on.
beam to a high velocity. 2
(f) ×6=4V
(d) (i) 300 × 8 = 10 mm 3
240
(g) To protect the transistor when it is
(ii) To deflect the electron beam switched off.
vertically. (ii)
(h) The relay switch will induce a large
1 voltage across the transistor when
(e) (i) Peak voltage = (0.50 × 8)
2 switched off. The voltage will
=2V damage the transistor.

Answers 630
5 (a) (i) The level of the solvent (b) 7 (a) (i) A cathode ray is a stream of
exceeds the maximum C fast-moving electrons
A
volume permitted within the P X produced by thermionic
range. The excess volume emission from a hot cathode.
Y
partially absorbs the radiation
to CRO R D vacuum
from the radioactive source B
which results in an input P cathode anode
Q screen
of 0.
(ii) The level of the solvent is
When the X terminal is positive for
less than the predetermined
the first half-cycle, current flows cathode
range. The radiation - + ray
through diode A, resistor R, diode
penetrates easily with little
D and then returns to Y. When the
absorption as the level of When the cathode is heated,
Y terminal is positive for the
the solvent is low. it produces electrons by the
second half-cycle, current flows
(b) (i) AND gate process of thermionic
through diode B, resistor R, diode
(ii) Truth table for A: emission. The voltage across
C and then returns to X. For both
the anode and cathode
Input P Input Q Output cycles, current flows through R
accelerates the electrons
in the same direction i.e. from
0 0 0 toward the screen.
P to Q.
1 0 1 The electrons move in a
The output wave is:
narrow beam with high speed
1 1 0 producing a cathode ray.
(ii) • Move in a straight line.
Truth table for B: • Negatively charged.
• Can be deflected by F
Input P Input Q Output
magnetic field. O
0 0 1 • Possess momentum and R
1 0 1 energy. M
1 1 1 (c) A circuit showing an automatic light (b) (i) Electric potential energy
control switch: → Kinetic energy 5
(c) (i) Logic gate A
(ii) • When the level of the solvent eV = 1 mv2
light 2
is within the predetermined dependent bulb e = charge of an electron
range, i.e. input P = 1 and resistor (LDR)
B C V = voltage across the anode
input Q = 0, gate A gives an 6V
R2 and the cathode
output of 1. E
m = mass of an electron
• When the level of the R1
v = velocity of electron
solvent is outside the
(ii)
predetermined range, i.e.
input P = 0 and input Q = 0 The light dependent resistor Electromagnetic Cathode ray

CHAPTER 4
or input P = 1 and input (LDR) is a resistor which has a wave
Q = 1, gate A produces an high resistance under dim 1 Consists of electric Consists of fast-
output of 0. environment. When it is dimly lit, and magnetic fields moving electrons
a small potential difference is
produced across R1. As a result, 2 Possess no charge Negatively charged
Essay Questions
6 (a) (i) A p-type semiconductor is the base current is too small to 3 Move at the Move at a speed
produced by adding activate the transistor. The bulb speed of light lower than the
trivalent atoms such as does not light up. However, in speed of light
indium, boron and gallium bright environment, the
into a pure silicon or resistance of LDR drops (c) (i) A dry cell is connected to the
germanium. As a result, dramatically. The potential inputs of a CRO. The Y-gain is
the semiconductor difference across R1 is now set at y V cm–1. The timebase
produced has holes as its greater. A greater amount of base control is switched on.
majority charge carriers current flows through the A horizontal line will be
and electrons as its transistor. This, in turn produces a observed on the screen.
minority charge carriers. greater collector current which The vertical distance from the
(ii) Doping process is the process lights up the bulb. For the bulb to base line is recorded.
of adding impurities into the light up in the dark, the position of
crystal lattice of a R1 and LDR is reversed.
semiconductor material to (d) Advantages: a cm
increase the conductivity. • Requires a small amount of
The atoms added usually have electrical energy to operate.
the same size as the atom of • Produces very little heat.
the semiconductor material. [or any appropriate answer]

631 Answers
The d.c. voltage is calculated electrons (c)
as follows:
Vertical distance from the base n p
a.c.
line = a cm
d.c. voltage
to CRO
= Vertical distance from the
base line × Y-gain
= a cm × y V cm–1
=zV • When the p-terminal is
(ii) An a.c. power supply is connected to the The rectified wave output can be
connected to a CRO. negative terminal and the smoothed by connecting a
The Y-gain is set at n-terminal is connected capacitor to the circuit.
y V cm–1. The timebase to the positive terminal of 9 (a)
control is switched on. A a dry cell, the electrons
sinusoidal wave form will and holes cannot flow Input Output
across the junction. J K L
be observed. The distance P Q R M
from the peak to the base • As a result, no current
flows through the 0 0 0 1 1 0 0
line is recorded.
diode. 0 0 1 1 1 0 0
• In this state, the diode is 0 1 0 0 0 1 0
a cm reverse-biased.
0 1 1 0 1 0 0
current, I
1 0 0 1 1 0 0
1 0 1 1 1 0 0
F 1 1 0 0 0 1 1
O The root mean square 1 1 1 0 1 0 0
R voltage, Vr.m.s. is calculated as O
voltage, V
M follows:
(b) P = 1, Q = 1 and R = 0
Distance from the peak to the
(c) Advantage:
5 base line = a cm
The current combination can be
Peak voltage, Vp
(b) A diode is a rectifier as it only reset easily by changing the
= Distance from the peak to
allows current to flow in one arrangement / or component of
the base line × Y-gain
direction. logic gates.
= a cm × y V cm–1
Disadvantage:
=zV + –
Vp It depends on the availability of
Vr.m.s. = __ power supply in order to work.
÷2
(d)
z Input Output
= __
÷2
CHAPTER 4

+ – A B C
=xV current 0 0 1
8 (a) (i) The bulb lights up because the 0 1 0
diode allows the current to flow 1 0 0
in the direction as shown. 1 1 1
(ii) A p-n junction is a site on the The positive and negative terminals
diode where the p-type of the diode are connected to the
semiconductor meets the positive and negative terminals of (e) (i)
n-type semiconductor. the dry cell respectively. The diode
(iii) • When the p-terminal and n- is forward-biased. Input Output
terminal are connected to + – A B C
the positive and negative 0 0 0
terminals of a dry cell
respectively, electrons will 0 1 1
be forced to move across 1 0 1
the p-n junction. – +
1 1 0
• The holes will move in the
opposite direction across The bulb does not light up because
(ii)
the junction. the current cannot flow through
• As a result, current is the diode. The positive terminal of A
flowing through the the diode is connected to the
diode. negative terminal of the dry cell
• In this state, the diode is and vice versa. The diode is
forward-biased. reverse-biased. B

Answers 632
(f) NAND gate (f) Current gain (vi) Tabulation of data:
(g) C = A • B = Gradient of the graph IC
10 (a) (i) A logic gate is an electronic against IB Input Output
device with one or more 2.50 mA A B X
inputs but only one =
50 µA 0 0 0
output. 2.50 × 10–3
(ii) An input is the factor that = 0 1 0
50 × 10–6
needs to be considered in 1 0 0
= 50
the process of decision 1 1 1
(g) 1.8 mA
making.
(iii) An output is the decision IC
(h) = 50 Key:
obtained from the process of IB Input 0: The jockey is
decision making. 4.2 mA connected to 0 V
= 50
(b) NOT gate IB wire.
– 4.2 Input 1: The jockey is
R R = P or IB =
P or Q – 50 connected to 6 V
R=Q
IB = 0.084 mA wire.
AND gate
= 84 µA Output 0: The bulb does not
P R light up.
Q Output 1: The bulb lights up.
2 (a) When both A and B are
switched on, current passes (vii) When both inputs A and B are
(c) NOT gate and AND gate through the bulb. 0, the diodes are forward-
(or NAND gate). When either A or B or both are biased. The resistances of the
NAND gate can be used to diodes are low allowing
switched off, current does not F
produce all the other logic current to bypass the bulb.
pass through the bulb.
Therefore, the bulb does not
O
gates. (b) AND gate gives an output of 1 only
light up. When either A or B is R
when both inputs are 1. M
(c) (i) To investigate the working of 1, one diode is forward-biased
Experiments and the other is reverse-
1 (a) (i) Base current AND gate.
(ii) Manipulated variable:
biased. Current flows through 5
(ii) Collector current the forward-biased diode. The
(iii) The type of transistor used. The state of input
bulb does not light up.
(b) (high voltage = 1 and
When both inputs A and B
low voltage = 0)
are 1, both diodes are reverse-
Responding variable:
Base current, Collector current, biased. The resistances of the
The state of the bulb
IB (µA) IC (mA) diodes are high. This causes
(lighted = 1 and
10 0.5 current to flow through the
not lighted = 0)
bulb and lights it up.
20 1.0 Fixed variables:
The type of transistor, the

CHAPTER 4 & 5
30 1.5
value of resistors, the type of
40 2.0 5 Radioactivity
bulb.
50 2.5 (iii) Apparatus/Materials: Self Assess 5.1
60 3.0 n-p-n transistor, diodes, 1 (a) Number of protons = 28
resistor, wires and bulb. Number of neutrons = 58 – 28
(iv) Arrangement of apparatus: = 30
(c) Graph of IC against IB :
(b) Number of protons = 9
6V
IC (mA) Number of neutrons = 19 – 9
input A
3.5 R = 10
3.0 (c) Number of protons = 82
2.5 Number of neutrons = 208 – 82
input B X
2.0 = 126
0V 2 (a) 21H (b) 188O
1.5 2.75 – 0.25
= 2.50 mA
1.0 3 They have different densities, melting
0.5 points and boiling points.
O
55 – 5 = 50 µA
IB (v) Procedure:
10 20 30 36 40 50 60 (µA)
1 Both jockeys are connected Self Assess 5.2
to the 0 V wire. 1 (a) 23941 Pa (b) –10e
(d) The collector current, IC is 2 The condition of the bulb is 2 The number of remaining atoms
directly proportional to the base observed. = 500 000 – 375 000 = 125 000
current, IB. 3 The following table Let the half-life = x
x 250 000 → x 125 000
(e) The current gain is equal to shows the action taken 500 000 →
the gradient of the graph IC and the corresponding 2x = 16 days
against IB. observations. ∴ x = 8 days

633 Answers
3 (a) So that the tracks can be observed (d) Let the initial mass of P n n

( 21 ) = 641

2 6
easily and light is not reflected =m
from surface P. The mass of P after n half-lives n

( 21 ) = ( 21 )
6
(b) To discharge any ions in the cloud 1 n
()
3
= m
chamber. 2
(c) The mass of Q after n half-lives n
=6
3
Alpha particle Beta particle =m–
1 n
2
m () n =6×3
Mass of Q = 18 minutes
= 31 7
Mass of P
Activity
The tracks The tracks
m–
2 ( ) = 31
1 n
m Time (hours) (counts per second)
0 720
(2 ) m
n
observed are observed are 1
straight which twisted which 4 360
shows that it is not shows that it is
/ [1 – ( ) ]
n
1 8 180
deflected by air easily deflected by m
2 12 90
molecules. air molecules. = 31

(2 )
n
This means that This means that it 1 16 45
m/
its mass is much has a small mass.
activity (counts per second)
greater than a
1 – ( ) = 31( )
n n
1 1
beta particle. 2 2 720 x

1 = 32( )
n
1
F 4 (a) T1 = 4 hrs
2
850 →
2
425 360 x

O
32 ( 2 )
n
counts per counts per 1 = 1
180
R second second 90
x

x
M 45
0
x
time
( 21 ) = ( 21 )
n
→

5
T1 = 4 hrs
2
4 8 12 16 (hours)
T1 = 4 hrs
5 106.25 ← 2

212.5 ∴n =5 8
counts per counts per The total time needed in order for T1 T1
3.0 × 1012⎯→1.5 × 1012⎯→ 7.5 × 1011
2 2

second second mass of Q


is 31 atoms atoms atoms
∴ Its activity 12 hours later mass of P

⎯→
T1
=5×4
2

= 106.25 counts per second T1


(b) Let the initial number of atoms = N = 20 hours 1.875 × 10 11
⎯→ 2
3.75 × 1011
The number of atoms that remain 5 atoms atoms
1 15 mins 15 mins
undecayed = N 120 s–1 → 60 s–1 → 30 s–1 4T1 = 20 days
16 2
15
→
CHAPTER 5

4y = 20 days
()
1 1
n
mins
N=
2 16 N 20
15 mins 15 mins y =
3.75 s–1 ← 7.5 s–1 ← 15 s–1
() ()
1 n 1 4 4
=
2 2 Total time taken = 5 days
n =4 = 15 mins × 5
Self Assess 5.3
1 = 75 mins
The time it takes for of atoms 1 • It decays by emitting beta particles
16 6 Mass of P after n minutes which can penetrate the ground.
in sample P to remain undecayed • It decays to a harmless element in
n n number of
= 4×4
= 16 hours
= ( 21 ) 2
256 =
2 half-lives of a relatively short time.
2 Radioactive isotopes which produce
P in n minutes
(c) The percentage of atoms P that gamma rays can be used. The gamma
have decayed = 87.5% Mass of Q after n minutes rays will sterilise the syringes without
The percentage of atoms P that melting them.
n
n number of
have not decayed
= (100 – 87.5)%
= ( 21 ) 6
4 =
6 half-lives of 3 Gamma rays have higher penetrating
power. They are also safer because
= 12.5% Q in n minutes
gamma rays do not cause much
T1 T1 T1
 50% →
100% → 2
 25% →
 12.5%
2 2
n n
ionisation, unlike alpha particles, which

The time it takes for 87.5% of ( 21 ) 2


256 = ( 21 ) 6
4 can damage healthy cells.
4 Inside the body, beta particles have less
atoms in sample P to decay into n

Q atoms = 3T1
2
( 21 ) 2

= 4
damaging effect on healthy cells
compared to gamma rays. At the same
n 256 time, beta particles can penetrate to the
= 3(4)
= 12 hours
( 21 ) 6
outside of the body and can be
0detected easily.

Answers 634
Self Assess 5.4 (b) (i) Mass defect, m 11 D 12 D 13 C 14 C 15 C
1 (a) • Nuclear radiation damages = (3.01605 + 2.01410) – 16 C 17 B 18 B 19 B 20 C
or kills living cells. (4.00260 + 1.00867) 21 D 22 A 23 A 24 D
• Fuel from nuclear reactors = 0.01888 a.m.u.
contain highly active decay = 0.01888 × 1.66 × 10–27 kg Structured Questions
products with long half-lives. = 3.13408 × 10–29 kg 1 (a) It is negatively charged
• Radioactive gas or dusts (ii) E = mc2 (b) 15
which escape into the = (3.13408 × 10–29) ×
Activity (counts per second)
atmosphere may enter the (3.0 × 108)2
bodies of living things = 2.820672 × 10–12 J 1600
through food or water. 2.820672 × 10–12
= 1200
(b) • Fuels such as deuterium are 1.60 × 10–19
readily obtainable. 800
= 17 629 200 eV
Deuterium can be extracted 400
= 17.6292 MeV
from sea water.
(c) (i) Mass defect, m Time
• Helium, the main waste 0 5 10 15 20 25 30 (hour)
= (2.01410 + 2.01410) –
product, is not radioactive.
(3.01493 + 1.00867)
• Fusion reactors have built-in (c) It has a short half-life which allows
= 4.6 × 10–3 a.m.u.
(inherent) safety. If the it to decay quickly so that it will not
= 4.6 × 10–3 × 1.66 × 10–27 kg
system fails, fusion reactors stay too long in the water.
= 7.636 × 10–30 kg
stop automatically.
(ii) E = mc2 T–1 T–1 T–1
1 = (7.636 × 10–30) × 2 2
(c) 0.0625 = (d) 1 1 2
1 1
16 (3.0 × 108)2 2 4 8

( 21 )
4
= 6.8724 ×10–13 J 3T–1 = 3 × 15 F
0.0625 =
6.8724 × 10–13 2
O
= = 45 hours
Therefore, 4 half-lives are required. 1.60 × 10–19 R
2 (a)
Number of years required α
= 4 295 250 eV M
= 4 × 30.17 = 4.29525 MeV γ
= 120.68 years (d) (i) Mass defect, m 5
2 (a) The hydrogen nuclei repel each = (3.01493 + 2.01410) –
other. (4.00260 + 1.00783) β
(b) The nuclei in nuclear reactor have = 0.0186 a.m.u.
to be heated up to 108 K by = 0.0186 × 1.66 × 10–27 kg (b) (i) He
electrical discharge. = 3.0876 × 10–29 kg (ii) 20
In the Sun, the nuclei are (ii) E = mc2
compressed by a very strong = (3.0876 × 10–29) × (c) Number of Number of
force of gravity which binds them (3.0 × 108)2 neutrons protons
together. = 2.77884 × 10–12 J (i) 143 91
(c) Water and carbon dioxide. 2.77884 × 10–12

CHAPTER 5
= (ii) 146 92
Both have high specific heat 1.60 × 10–19
capacity. (iii) 138 90
= 17 367 750 eV
3 A proton, which is positively charged, = 17.36775 MeV
will experience electrical repulsion from (d) A neutron disintegrates into a
the positively charged nucleus of Self Assess 5.5 proton and an electron. The
uranium-235. 1 Type of radiation, dosage and exposure electron is ejected from the
time, methods of insertion into the body, nucleus as beta particles.
4 Energy, E = mc2 exposure to different parts of the body. (e) (i) γ-rays can penetrate the
= 1.2 × 10–3 × (3.0 × 108)2 2 The food and drinks may be medical instruments and
= 1.08 × 1014 J contaminated by radioactive substances. destroy harmful
5 (a) (i) Mass defect, m If consumed, the radiations would do microorganisms easily.
= (2.01410 + 2.01410) – more damage from inside the body. β-particles may not be able
(3.01605 + 1.00783) 3 (i) Strong radioactive substances are to penetrate certain medical
= 0.00432 a.m.u. handled using remote-controlled instruments.
= 0.00432 × 1.66 × 10–27 kg mechanical arms from a safe (ii) β-particles cause less damage
= 7.1712 × 10–30 kg distance. to the surrounding healthy
(ii) E = mc2 (ii) Radioactive substances are kept cells compared to γ-rays.
= (7.1712 × 10–30) × in thick lead containers. 3 (a) (i) Beta particles.
(3.0 × 108)2 (or any appropriate answers) (ii) Beta particles can penetrate
= 6.45408 × 10–13 J the metal foil. Alpha particles
6.45408 × 10–13 SPM Exam Practice 5 cannot penetrate the metal
= Multiple-choice Questions
1.60 × 10–19 foil whereas gamma rays have
1 A 2 D 3 B 4 C 5 D high penetration power.
= 4 033 800 eV
6 C 7 B 8 D 9 C 10 A Hence, gamma rays always
= 4.0338 MeV

635 Answers
penetrate the metal foil of a (iv) The half-life should be short 6 (a) (i) Half-life is the time taken for
wide range of thickness. to prevent overexposure to half the atoms in a given
(iii) Long half-life. radiation for unnecessarily sample to decay.
(iv) The radioactive source need long period of time. (ii) To obtain the background
not be replaced so 5 (a) Radioactivity is the spontaneous count.
frequently. disintegration of an unstable 845 – 120
(v) Strontium-90 nucleus into a more stable (iii) × 100%
1410 – 120
(vi) nucleus with the emission of 725 100% = 56.2%
energetic particles or photons. = ×
activity per minute (min–1) 1290
N
(b) (i) More atoms are ionised in Therefore, the patient’s
N Diagram 6(b) compared to thyroid is not functioning
2 that of Diagram 6(a). properly.
N time (ii) Radioactive emission Y has
4 59 118 (years) (b) Radiation kills most
higher ionisation power microorganisms effectively. The
compared to radioactive method of boiling is not suitable
(b) (i) Nuclear fusion
emission X. for certain medical equipment
(ii) 2(2.01410) – 3.01493 –
(c) (i) Beta particle made from plastic as they can
1.00867 = 0.0046 a.m.u.
(ii) Alpha particle melt easily.
Energy released, E
(d) (i) Nuclear fission (c) (i) Isotope — elements with the
= mc2
= (0.0046 × 1.66 × 10–27) × (ii) The fission of uranium-235 same number of protons
(3.0 × 108)2 produces neutrons which but different number of
= 6.8724 × 10–13 J continue to split other nuclei neutrons.
(iii) The products of nuclear fusion causing further fission. The Proton number — the number
are not radioactive. number of nuclei which of protons in the nucleus.
F undergo fission multiplies Nucleon number — the total
O Essay Questions rapidly. A huge amount of number of protons and
R 4 (a) (i) A radioisotope is an isotope energy is released in a short neutrons in the nucleus.
M which gives out radioactive span of time. (ii) High frequency
rays. (e) (i) Strong radioactive electromagnetic radiation.
5 (ii) Both graphs have the shape substances are handled
(21 )
N
of exponential curves. using remote-controlled (iii) (3.4 × 103) = 106.25
Strontium-93 takes a longer mechanical arms. This is to
time for its activity to reduce minimise exposure to where N
to half its original activity radiations. = number of half-lives

( 21 ) = 321
N
compared to barium-143. (ii) A G-M tube is used to detect
The times taken for radiation.
strontium-93 and barium-143 A G-M tube can detect
( 21 ) = ( 21 )
N 5

to reduce its activity to half its the different types of


original activity are 480 radiation.
∴ N =5
seconds and 12 seconds (iii) – Protective suits and gears
CHAPTER 5

Time required = 5 × 8
respectively. such as gloves, and eye
= 40 days
The time taken for the activity glasses made of lead are
(iv) Diagnosis of thyroid gland
to reduce to half its original worn at all times.
disorder.
activity is called half-life. The workers do not come
(v) It has a short half-life which
(b) Ionising radiation is radiation which into contact with the
makes it harmless in a short
has the ability to remove electrons radioactive waste. At the
period of time.
from atoms, thus forming ions. same time, the workers
(d) (i) P - alpha particles
Ionising radiation can damage are shielded from the
Q - gamma rays
cells. Ionising radiation can induce harmful radiations.
R - beta particles
mutations in cells. – Radioactive wastes must
(ii) Fleming’s left-hand rule
(c) (i) Gamma radiation directed be kept in thick lead
(iii) Gamma rays have no
onto target cancer cells containers. The chance of
electrical charge.
from outside the patient’s spillage is minimised.
7 (a) (i) An unstable isotope which
body. This radiation can Radiation cannot
decays spontaneously by
penetrate the body and kill penetrate the thick lead
emitting radioactive
cancer cells. containers.
radiations.
(ii) Total dose should be sufficent – Radioactive wastes are
(ii) M – Alpha particles,
to kill only the cancer cells. placed in concrete blocks
N – Beta particles
(iii) The dose is administered over and then buried
(iii) M – Tracks are thick and
a longer duration to enable underground. The wastes
straight due to high
the non-cancerous cells to are far from any human
ionising power and the
repair themselves and activities which may
low mass of alpha
therefore reduce the cause leakage of
particles.
damaging effect. radioactive wastes.

Answers 636
N – Tracks are thin and (ii) Background radiation is a (v) Procedure:
twisted because weak radiation which originates – The arrangement shown
ionising power and the from radioactive materials on above is set up.
low mass of beta particles. Earth or cosmic rays from – The activity is recorded
(b) outer space. every 30 seconds.
(iii) – The activity of the radioactive
Characteristic Reason
sample is then calculated.
Liquid So that it can be Activity (counts per minute)
(vi) Tabulation of data:
administered into 100
X
the blood easily 90 Time (s) 0 30 60 90 120
Moderate It can be detected 80 Count (s–1) 0 p1 p2 p3 p4
penetrating outside the patient’s
70 Calculation:
power body
Low ionising It does little damage 60
Time (s) Activity (s–1)
power to the cell 50
47 X p1 – 0
Short half-life It does not stay too 40 15 =q
30
long inside the body.
30
X p2 – p1
Radioisotope X is the most 45 = q1
20 30
suitable because it is a liquid, X
10 p3 – p2
emits beta particles and has a
Time 75 = q2
short half-life. 0 30
10 20 30 40 50 60 (minutes)
(c) (i) mass defect
p4 – p3
= 226.02536 u – 105 = q3
(iv) 20 minutes 30 F
(222.01753 + 4.00260) u
= 0.00523 u
O
(vii) Analysis of data:
= (0.00523 × 1.66 × 10–27) kg R
Experiment A graph of activity against time
= 8.6818 × 10–30 kg is plotted.
M
1 (a) The activity of the radioactive
(ii) Energy released,
sample decreases with time as the Activity (s )
–1

E = mc2
number of radioisotopes which
5
= 8.6818 × 10–30 × q
have not disintegrated decreases
(3.0 × 108)2
with time.
= 7.81 × 10–13 J
(b) The time taken for the activity 1
2
q
8 (a) (i) 131 I → 131 Xe + –10e + γ
53 54 of the radioactive sample to 1
q
(ii) A fast-moving electron 4
decrease to half its original
(iii) 131 – 54 = 77 neutrons
value is a constant. O T12 2T12 Time (s)
(b) (i) No
(c) (i) Aim:
(ii) Beta particles cannot
To investigate the time taken Conclusion:
penetrate the human tissues.
for the activity of a radioactive Using the graph, the time

CHAPTER 5
(c) (i) Number of half-lives = 32 sample to decrease to half its taken for the activity of the
8 original activity. radioactive sample to
Number of half-lives = 4 (ii) Variables decrease to half its original
Number of gamma rays Manipulated: activity is a constant. This
emitted after 32 days The time interval constant is called the half-life,
( )
4
= 1 (4.2 × 108) Responding: T1 of the radioactive sample.
2 The activity recorded 2

= 2.625 × 107 Fixed: SPM Model Test


(ii) Only a certain percentage of The type of radioactive Paper 1
the injected 13153
I is absorbed. sample, the mass of 1 B Vernier scale has accuracy of
(d) (i) radioactive sample 0.01 cm
(iii) Apparatus/Materials: 2 D Height, temperature and electric
Activity due to
Activity A lead cylinder, a ratemeter, a current are base quantities.
Time sample alone
(counts Geiger-Muller tube, a 3 C
(min) (counts per
per minute) radioactive sample. 4 B Momentum of X is totally
minute)
(iv) Arrangement of apparatus: transferred to Z through Y. Hence X
0 100 94 The apparatus is arranged as stops and Z moves with velocity
10 74 68 shown below. v m s–1 while Y remains stationary.
20 53 47 5 D Owing to inertia, water droplets on
ratemeter the umbrella continue to move when
30 39.5 33.5
G-M tube the umbrella is stopped abruptly.
40 29.5 23.5 radioactive
lead 6 C The coconut falls from rest
sample
50 22.5 16.5 cylinder (u = 0 m s–1). The variation of its
60 17.75 11.75 velocity with distance is given by

637 Answers
v 2 = u2 + 2gs, where g is the ∴ The weight of oil displaced 29 D
gravitational acceleration. = The weight of water displaced 30 C When the waves pass through
∴ v 2 = 2gs 17 C (800 – 600) = 200 g gives a different depth, refraction of waves
v =  2gs compression of 4 cm. occurs. The speed, wavelength and
600 g direction of propagation of the
v 2 varies directly with s. = 3,
200 g refracted waves changes, but the
2
v v 3 × 4 cm = 12 cm of compression, frequancy remains unchanged.
Original length of spring 31 A
= 10 cm + 12 cm = 22 cm 32 D The time taken for the cork to
18 D The boiling point of water is move from the trough and back to
affected by atmospheric pressure. the trough for the first time is T
O s O s At the summit of mountain, the seconds. Hence, the time taken for
atmospheric pressure is lower. the cork to move from the trough
7 D She bends her knees to lengthen
Water will boil at a temperature
the time of impact on the pitch to the crest is T s.
below 100 °C. 2
reducing the impulsive force acting
19 B mcθ = 11 500 J
on her feet. 33 D Radio wave, water wave and
11 500
8 B Net force = 0 when the forces c = (0.5)(220 – 170) microwave are transverse waves.
acting on an object are in equilibrium. 34 C Using interference formula,
1 = 460 J kg–1 °C–1
9 A mgh = mv2
2 20 B Let T be the final temperature of λ = ax
D
10(1.25)(2) = v 2 the mixture.
λD
v = 5 ms–1 Heat gained by 1.8 kg of water ∴x = a
10 B Work done through smooth = Heat lost by 900 g of the water
F inclined plane m1cθ1 = m2cθ2 The distance between two
1.8 × (T – 50) = 0.9 × (80 – T ) consecutive loud sounds, x ,
O = work done to bring object
1.8T – 90 = 72 – 0.9T increases when the
R through vertical height
wavelength of the sound wave
= mgh 2.7T = 162
M is increased or the audio
= 4(10)(6) = 240 J T = 162
11 C Displacement 2.7 frequency is decreased and the
4 = 60 °C distance between the two
= Area below – Area below
& graph t0 – t6 graph t6 – t10 21 C Gradient of graph increases, loudspeakers is reduced.
5 = 48 –18 = 30 m specific heat capacity decreases. 35 B
12 A Gradient of Gradient of 36 D E = 3 V, V = 2.5 V and I = 1 A
F1 F2 F3 graph in the > graph from Using E = V + Ir
13 C = = 1st minute 3rd to 5th 3 = 2.5 + (1) r
A1 A2 A3
(solid state) minute r = 3 – 2.5
(Pascal’s principle) = 0.5 Ω
(liquid state)
Since A1  A2  A3, 1 1
22 C In Charles’ law, two constant 37 C 1 = +
therefore F1  F2  F3. R1 + R2 R3
variables are mass and pressure.. R’
14 B
CHAPTER 5 & SPM MODEL TEST

Bunsen burner is the application of


23 D 1 1
Bernoulli’s principle. = +
24 A The real depth = 15 cm (3 +3) 3
15 B The value of h will increase if
Let the apparent depth = 15 – h
extra pressure acts on the surface 1 1 3
Using the formula, refractive index, = + =
of mercury in the container. 6 3 6
n= Real depth
When some water is poured into the
Apparent depth 1
container, the value of h will increase. =
16 A For a floating spherical solid: 15 2
1.3 =
According to Achimedes’ principle, 15 – h R =2Ω
15 – h = 15 38 B I = 400 mA
1.3 = 0.4 A
oil h = 15 – 11.54 = 3.46 cm t = 2 minutes
25 A = 120 s
Buoyant force 26 C Q = It
= The weight of the solid 27 A The image formed is real, inverted = 0.4 × 120
= The weight of oil displaced and same size at the object. = 48 C
28 C 39 A Using Fleming’s left-hand rule,
35˚ the force acts in the direction of K.
water
55˚ 40 B The current flows from the positive
terminal to the negative terminal.
Looking at X from the left, the
Buoyant force critical current flow is in the clockwise
= The weight of the solid angle direction.
= The weight of water displaced ∴ X is a south pole.
Critical angle = 55º

Answers 638
Looking at Y from the right, the (d) The activity for P is the same as X Since there are two dry cells in
current flow is in the anticlockwise at t = 22 minutes. The activity of P the battery, internal resistance
direction. is less before and more after for one dry cell
∴ Y is a north pole. t = 22 minutes.
= 1.0 = 0.50 Ω
41 D Fleming’s right-hand rule (dynamo 4 (a) T = 0.02s 2
rule) is used to determine the
f= 1 7 (a) (i) The distance between the
direction of the induced current.
T spekers in Diagram 7.2 is
42 B For an ideal transformer,
1 smaller than that for Diagram
output power = input power. =
0.02 7.1.
43 C
(ii) The distance between the
44 B = 50 Hz
antinodal lines in Diagram 7.2
45 C (b) Source P because the voltage is
is greater than that in
46 B always 3 V whereas for Q, the
Diagram 7.1.
47 B voltage varies from zero to 3 V and
(iii) The separation between the
48 B for R, the voltage is 2 V.
antinodal lines increases
49 A (c) (i) To allow current to flow in one
when the distance between
50 D direction only.
the speakers decreases.
(ii)
(b) Decrease the frequency of the
Paper 2 signal so as to increase the
Section A wavelength of the signal.
1 (a) S is most sensitive. (c) (i) He hears loud sound all along
(b) Q and S are not suitable because (ii) He hears no sound or soft
they cannot measure current more sound
than 0.50 A and 0.25A
(d) λ =
v
=
330
= 0.50 m F
respectively. The current to be f 660
measured may be more than
O
0.25 A or 0.5 A as the expected λD 0.50 × 3.0 R
x= = = 0.25 m
range is from 0 to 1 A a 0.60 M
(c) P is more sensitive than R, and P is 8 (a)
equipped with anti-parallax mirror. 4
5 (a) Pu = 75 + 5 = 80 cm Hg Scalar Quantity Vector Quantity
2 (i) Similarity: Both images are
(b) Glass tube in vertical position
&
diminished, inverted and real.
with the open end at the
mass, time, height weight, velocity, 5
Difference: For concave mirror, displacement
bottom.
image is in front of the mirror. For
(c) (i) P1 V1 = P2 V2 (b) (i) Nil
convex lens, image is formed
80 × 51 = 85  (ii) 0 = Buoyant force – (Weight
behind the lens.
 = 48 cm of hot-air balloon +
(ii) 40 cm
V1 V Weight of Mr Gerald)
(Distance between object and lens (ii) = 2
T1 T2 Buoyant force = 3000 + 600
= 2f + 2f = 4f = 40 cm)
= 3600 N

SPM MODEL TEST


(iii) 48 51
= (c) (i) 10 ms–1
273 + 27 T2
(ii) The rubber ball is seen to
1 T2 = 319 K rise higher for one second
2 Point object F
 = 319 – 273 before coming down from
3 = 46°C t = 1 to t = 4 s
6 (a) S, R, P, Q (iii) ✓ : Appears to move
(b) down straight after
release.
3 (a) ZA X → A–4
Z–2
Y + 24He
(d) (i) u = 10 m s–1, a = –10 m s–2,
(b) & (c) 1Ω
t = 1.0 s
(18 + 18)
T1 =
2
= 18 minutes s = ut + 1 at2
2 2
Activity (counts per minute)
= (10 × 1) + 1 (–10)(1)2
2400 R = 1 × 1 Ω = 0.25 Ω 2
4 =5m
2000
(ii) H = 45 – 5 = 40 m
1600 (c) (i) 3.0 V
1200 (ii) Total internal resistance
Section B
of the battery
800 9 (a) (i) The buoyant force is an
= | Gradient of graph V – I|
400 P upward force resulting from
= 3.0 – 1.5
0
X
0 10 20 30 40 50 60 70 90 t (min)
T1 T1
 0 – 1.5  an object being wholly or
partially immersed in a fluid
2 2
= 1.0 Ω (liquid or gas).

639 Answers
(ii) The bigger balloon needs a pipe. This will prevent corrosion secondary coil at the same time,
heavier weight to support or or rusting on the pipe and thus and hence increases the efficiency.
to prevent it from rising up. extends the service life of the The iron coil should be laminated.
The buoyant force hydraulic system. This will reduce the eddy current
experienced by the bigger 10 (a) (i) An electromagnet is a and thus reduce the heating effect
balloon is greater than the temporary magnet consists on the core.
smaller one. of copper coil and a core of The iron core should be of closed
However, the bigger balloon soft iron. loop and as one piece without any
displaces more air. (ii) The reading of the ammeter gap which leads to flux leakage.
Thus, the more the air in Diagram 10.2 is greater With no flux leakage, the efficiency
displaced, the greater the than the reading on the will increase.
buoyant force is. ammeter in Diagram 10.1. The modified step down
(b) (i) The submarine dives by letting The reading of the spring transformer is as shown in the
more water into the ballast balance in Diagram 10.2 is diagram below.
tank. This can be done by greater than the reading on
reducing the air pressure the spring balance in
above the water surface in the Diagram 10.2. secondary
ballast tank. The weight of the Thus the force on the iron coil
submarine is greater than the rod due to the eletromagnet laminated
buoyant force and thus it dives for Diagram 10.2 is greater. primary coil core
into the bottom of the sea. The deduction is: the greater
When water is forced out of the current, the stronger is Section C
the ballast tank with the strength of the 11 (a) (i) The heat energy absorbed is
compressed air, the weight electromagnet. used to increase the kinetic
F of the submarine becomes (b) (i) The main use of a magnetic energy of the molecules.
O less than the buoyant force. relay is as a switch using a The temperature thus rises as
R Thus, the submarine rises. small current to turn on a large the temperature of a
M (ii) The buoyant force on the current in a much more substance depends on its
submarine when it is floating powerful circuit. kinetic energy.
4 on the surface of water is (ii) When the switch is closed, a (ii) The heat energy absorbed is
less than the buoyant force small current flows through
& not used to increase the
on the submarine when it is the electromagnet.
5 stationary under the surface The iron core in the
kinetic energy of the
molecules. The energy
of the sea (because less electromagnet is magnetised absorbed is used for other
water is displaced while and attracts the soft iron purposes.
floating resulting in a smaller armature. (iii) For BC, the heat energy
buoyant force). The contacts are thus closed absorbed is used to break the
(c) The output force is directly and a large current flows in bonds between the ice
proportional to the multiplying the output circuit to turn on molecules.
factor. The multiplying factor must the motor.
SPM MODEL TEST

For DE, the heat energy


be high so that a great force can When the switch is open, no absorbed is used to
be produced on the output piston. current flows in the — do work to overcome the
The boiling point and the specific electromagnet and the soft forces of attraction
heat capacity should be high so iron core losses all its between the molecules so
that the fluid does not vaporises magnetism and ceases to that the molecules are
or change to gaseous state easily. attract the soft iron armature. further apart from each
Once the gas is formed, the The spring mechanism other, and
efficiency of the system will be then pulls the soft iron — do work against the
greatly affected. armature back to its original atmospheric pressure as
The lubrication quality of the fluid position, and thereby opens water expands and
should be excellent as this will the contacts. becomes steam.
reduce the friction on the pistons (c) The turns ratio should be 4 : 1 (b) The power should be high so that
and also extend the service life of (the number of turns on primary the time taken to boil away 20 g of
the hydraulic system. coil to secondary coil), therefore water is short. This will reduce the
The compressibility of the fluid the number of secondary coil total heat lost to the surrounding.
should be low so that no work should be reduced to 10 turns. This helps to improve the accuracy
will be spent on compressing the Thick copper wires should be used of the experiment.
hydraulic fluid. If work is done to instead of iron wire. This will The surface of the beaker used
compress the fluid, the efficiency reduce power loss (= I2R) and should be shiny as shiny surface
of the hydraulic system is reduced. increases the ourput power. reduces the rate of heat lost through
Steel or other material which The secondary coil should be radiation to the surroundings.
resists corrosive effect from the wound over the primary coil. This Electronic weighing scale should
fluid should be used as the will ensure all the flux produced by be used as it is more sensitive and
material for the fluid transmission the primary coil will link the accurate.

Answers 640
The temperature in the room Aspect Reason ∴ The distance moved
should be 26°C because the rate = 2.2 – 1.47 = 0.73 m
of heat lost to the surroundings The outer surface This improves the
depends on the difference in of the pot should rate of absorption Paper 3
temperature of the boiling water be dull and black. of sunlight and Section A
and the room temperature. The increases the 1 (a) (i) Distance between the double-
greater the difference, the greater efficiency of the slit, a
the rate of loss of heat to the solar hotpot. (ii) Distance between two
surroundings. The pot should be Isolating the air consecutive bright fringes, x
Arrangement R is chosen since within a glass inside the pot (iii) Wavelength of sodium light
more accurate result will be vessel. from the air (b)
obtained as the power of the outside improves
immersion heater is higher, the the efficiency of 1
a (mm) (mm–1) x (mm)
external surface of the breaker is the solar hotpot. a
shiny, the room temperature is The heat will be 0.3 3.33 6.4
higher and electronic weighing trapped inside by
scale is used. greenhouse 0.5 2.00 3.8
(c) (i) Q = mL effect. 0.8 1.25 2.4
= 0.020 × 2.26 × 106 1.0 1.00 1.9
= 45 200 J Solar hotpot Q will be chosen 1.2 0.83 1.6
(c) (ii) E = Pt because its disc has a larger
= 100 × (9 × 60 + 30) diameter, the orientation of the disc 1
= 57 000 J is adjustable to collect more sunlight (c) Graph of x against :
a
Heat lost to the surroundings for any position of the sun. It has a
= 57 000 – 45 200 hotpot with dull and black surface x (mm)
F
1
= 11 800 J and it is placed in a glass vessel. Graph of x against a O
7
R
Efficiency = 45200 × 100% (c) (i)
57 000 6 M
= 79.3%
5
12 (a) (i) The amplitude of the wave 4
affects the intensity or energy 4x 4 &
of the wave.
4x = 1.2 cm
5
(ii) The periods at points P and Q 3
are the same. x = 0.3 cm
ax 2
(iii) The wavefront at the opening Applying λ = ,
is spread out due to D 1
0.5 × 10–3 × 0.3 × 10–2 1 (mm-1)
diffraction. Since the a
λ=
wavefront at point Q is longer 2.2 O 0.5 1.0 1.5 2.0 2.5 3.0 3.5
than that at point P, the λ = 6.82 × 10 –7 m

SPM MODEL TEST


amplitude of the wave has to (d) The distance between two
(ii)
be reduced so that the consecutive bright fringes, x, is
amount of energy carried by inversely proportional to the
the wavefront at point P distance between the double-slit,
remains the same as the a.
1.2 cm
wavefront that reaches point 2 (a) (i) When n increases, I increases
Q. 6x′ = 1.2 cm 1 1
( increases linearly with ,
(b) x′ = 0.2 cm n I
Since the distance between therefore, when n increases, I
Aspect Reason two fringes, x decreases, from increases).
The diameter of This enables the the formula 1(A¯¹) Graphh of 1 against n1
the parabolic disc disc to collect λD I I
x= , D must also be
should be large. more sunlight and a 1.70
to reflect the decreased. Therefore, the 1.65
sunlight to the pot screen is moved towards the 1.60
to cook the food double-slit plate. 1.55
faster.
λD′ 1.50
The orientation of More energy will Applying x ′ = , 1.45
a
the disc should be be collected by 1.40
adjustable to the pot to speed ax′
D′ = 1.35
focus the incident up the heating. λ
1.30
sunlight onto the 0.5 × 10–3 × 0.2 × 10–2 1
D1 = 1.25
pot. 6.82 × 10–7 0.1 0.2 0.3 0.4 0.5 0.6 0.7 0.8 0.9 1.0 n
X1 = 1.47 m

641 Answers
(i) From the y- intercept, (v) Procedure: (ii) Variables:
1 1 The experiment is started Manipulated variable:
= 1.335 A–1
I0 with the smallest disk. The mass of water, m
R The area, A of the disk is Responding variable: The time
(ii) E = recorded. to reach boiling point, t
1
2 The apparatus is arranged Fixed variable: The power, P of
I0
as shown. the immersion heater (or the
2.0 Ω 3 By using your thumb or specific heat capacity, c of
(ii) E =
1.335 A–1 with the aid of a rod, the water)
disk is pressed (iii) Apparatus and materials:
= 1.50 V
downwards until the Immersion heater 50 W,
y2 – y1
(b) (i) m = electronic balance shows 1000 m glass beaker,
x2 – x1 a reading of 100.00 g. stopwatch, water,
1.670 – 1.400 4 The force is removed and thermometer, stirrer, weighing
(b) (i) m =
1.00 – 0.20 the depth of cavity scale and power supply.
formed is measured and (iv) Arrangement of apparatus:
= 0.338 A–1
(ii) r = m × E recorded.
(ii) r = 0.338 × 1.50 5 The procedure is
repeated for the other beaker
= 0.507 Ω
disks on other part of the
1 1
(c) When n = 2, = = 0.5 plasticine slab. The areas, water
n 2 immersion
A and the corresponding heater
1 depths, d, of the cavities
From the graph, = 1.50 A–1 (v) Procedure:
I are measured and
F recorded. 1 The apparatus is arranged
I = 0.667 A
O (d) When taking the readings on the Table as shown in the diagram.
R ammeter, the line of sight must be m (g) t (s)
2 200 g of water (measured
M perpendicular to the plane of the
with weighing scale) is
200 t1 poured into the glass beaker.
scale so as to avoid parallax error.
400 t2 3 The power is turned on.
4
The time, t for the water to
& 600 t3
start boiling is noted.
Section B
5 3 (a) Inference: The depth of the cavity, 800 t4 4 The procedure is repeated
d depends on the area in contact, 900 t5 for m = 400 g, 600 g,
A. 800 g and 900 g.
(b) Hypothesis: The smaller the area in Graph: 5 The corresponding times to
contact, A, the greater the pressure, d (cm) reach boiling point are
P. recorded.
(c) (i) Aim: To study the relationship (vi) Tabulation of data:
between the pressure
SPM MODEL TEST

A (cm2) d (cm)
(represented by the depth of
the cavity, d, formed) and the A1 d1
area in contact, A A2 d2
A (cm2)
(ii) Variables: A3 d3
Manipulated variable: The Conclusion: Form the table
area in contact, A and the graph, it can be seen A4 d4
Responding variable: The that the smaller the area in A5 d5
pressure P (represented by contact A, the greater the
the depth of the cavity, d) pressure P (represented by (vii) Graph:
Constant variable: The force, F the depth of the cavity). The t (s)
(iii) Apparatus/materials: hypothesis is valid.
plasticine slab, electrical 4 (a) Inference:
balance, thin wooden disks of The time to boil the water, t
different known areas and depends on the mass of water, m.
metre rule. (b) Hypothesis: m (g)
(iv) Arrangement of apparatus: The more the water, m, the longer
Conclusion:
the time, t needed to boil the
From the table and the graph,
Force water.
it can be seen that the more
disk (c) (i) Aim:
the mass, m of water, the
To study the relationship
plasticine longer the time, t to reach the
between the time taken to
slab boiling point.
electronic
reach the boiling point of
The hypothesis is valid.
balance water, t and the mass, m of
water used.

Answers 642

Você também pode gostar